You are on page 1of 602

Hàm số FB: http://www.facebook.

com/VanLuc168

I. TÍNH ĐƠN ĐIỆU CỦA HÀM SỐ


Chuyên đề: Hàm số

A. Tóm tắt lí thuyết


I. KIẾN THỨC CƠ BẢN

1) Định lý 1: Cho hàm số y  f (x) có đạo hàm trên K.


a) Nếu hàm số f (x) đồng biến trên K thì f '(x)  0 với mọi x  K
b) Nếu hàm số f (x) nghịch biến trên K thì f '(x)  0 với mọi x  K
 [ f(x) đồng biến trên K]  [ f '(x)  0 với mọi x  K ]
 [ f(x) nghịch biến trên K]  [ f '(x)  0 với mọi x  K ]
[ f '(x)  0 với mọi x  K ]  [ f(x) không đổi trên K]

2) Định lý 2: Cho hàm số y  f (x) có đạo hàm trên K.


a) Nếu f '  x   0 với mọi x  K thì hàm số f (x) đồng biến trên K
b) Nếu f '  x   0 với mọi x  K thì hàm số f (x) nghịch biến trên K
c) Nếu f '  x   0 với mọi x  K thì hàm số f (x) không đổi trên K
 [ f '(x)  0 với mọi x  K ]  [ f(x) đồng biến trên K]
 [ f '(x)  0 với mọi x  K ]  [ f(x) nghịch biến trên K]

3) Định lý 3: (Định lý mở rộng) Cho hàm số y  f (x) có đạo hàm trên K.


a) Nếu f '  x   0 với mọi x  K và f '  x   0 chỉ tại một số điểm hữu hạn thuộc K
thì hàm số f (x) đồng biến trên K.
b) Nếu f '  x   0 với mọi x  K và f '  x   0 chỉ tại một số điểm hữu hạn thuộc K
thì hàm số f (x) nghịch biến trên K.

4) Định lý 4: Cho hàm số bậc ba y  f  x   ax 3  bx 2  cx  d  a  0  , ta có


f '  x   3ax 2  2bx  c .
a) Hàm số y  f  x   ax 3  bx 2  cx  d  a  0 
đồng biến trên R  f '  x   3ax 2  2bx  c  0 x  R
b) Hàm số y  f  x   ax 3  bx 2  cx  d  a  0 

nghịch biến trên R  f '  x   3ax 2  2bx  c  0 x  R

NGUYỄN VĂN LỰC  0933.168.309 SP Toán K35 - ĐH Cần Thơ


Hàm số FB: http://www.facebook.com/VanLuc168
NHẮC LẠI
Định lý: Cho tam thức bậc hai f ( x) ax 2 bx c (a 0) ta có:
0
 f ( x) 0 x
a 0
0
 f ( x) 0 x
a 0

B. Phương pháp giải toán


Dạng : Định giá trị tham số để hàm số đơn điệu trên tập hợp X cho trước.
1. PHƯƠNG PHÁP
B1. Tập xác định: D ?
B2. Tính y ' ?
B3. Lập luận:
y đồng biến trên X y' 0, x X

y nghịch biến trên X y' 0, x X

Chú ý quan trọng: Trong điều kiện trên dấu bằng xảy ra khi phương trình
y' 0 có hữu hạn nghiệm, nếu phương trình y ' 0 có vô hạn nghiệm thì trong
điều kiện sẽ không có dấu bằng.

2. CÁC VÍ DỤ
1
Ví dụ 1. Cho hàm số y  (m 2  m) x 3  2mx 2  3x  1 . Tìm m để hàm số luôn đồng biến
3

trên R .
Bài giải:
♦ Tập xác định: D  R
♦ Đạo hàm: y '  (m 2  m) x 2  4mx  3
♣ Hàm số luôn đồng biến trên R  y '  0 x  R
m  0
♥ Trường hợp 1: Xét m2  m  0  
m  1

+ Với m  0 , ta có y '  3  0, x  R , suy ra m  0 thỏa.


3
+ Với m  1 , ta có y '  4 x  3  0  x   , suy ra m  1 không thỏa.
4

NGUYỄN VĂN LỰC  0933.168.309 SP Toán K35 - ĐH Cần Thơ


Hàm số FB: http://www.facebook.com/VanLuc168
m  0
♥ Trường hợp 2: Xét m2  m  0   , khi đó:
m  1

 '  m2  3m  0 3  m  0
♣ y '  0 x  R      3  m  0
m  m  0
 m  0  m  1
2

♦ Từ hai trường hợp trên, ta có giá trị m cần tìm là 3  m  0 .

Ví dụ 2. Cho hàm số y  x 3  3mx 2  3(m2  1)x  2m  3 . Tìm m để hàm số nghịch biến trên
khoảng 1; 2  .

Bài giải
♦ Tập xác định: D  R
♦ Đạo hàm: y '  3x 2  6mx  3(m 2  1)
♣ Hàm số nghịch biến trên khoảng 1; 2   y '  0 x  1; 2 

Ta có  '  9m2  9(m2  1)  9  0, m


Suy ra y ' luôn có hai nghiệm phân biệt x1  m  1; x2  m  1 ( x1  x2 )
x  1 m  1  1
Do đó: y '  0 x  1; 2   x1  1  2  x2   1    1 m  2
 2
x  2  m  1  2

♦ Vậy giá trị m cần tìm là 1  m  2 .


Bài tập tương tự
Cho hàm số y  2 x 3  3  2m  1 x 2  6m  m  1 x  1 . Tìm m để hàm số đồng biến trên

khoảng  2;   .

Đáp số: m 1 .

Ví dụ 3. Cho hàm số y  x 3  3x 2  mx  2 . Tìm m để hàm số đồng biến trên khoảng


 0;   .
Bài giải
♦ Tập xác định: D  R
♦ Đạo hàm: y '  3x 2  6 x  m
♣ Hàm số đồng biến trên khoảng  0;  

 y '  0 , x   0;   (có dấu bằng)

NGUYỄN VĂN LỰC  0933.168.309 SP Toán K35 - ĐH Cần Thơ


Hàm số FB: http://www.facebook.com/VanLuc168
 3x  6 x  m  0 , x   0;  
2

 3 x 2  6 x  m , x   0;   (*)

♣ Xét hàm số f ( x) 3x 2 6 x , x   0;   , ta có:


f '( x) 6x 6 ; f '( x) 0 x 1

Bảng biến thiên:


x 0 1

f '( x ) 0
f ( x) 0
3

♣ Từ BBT ta suy ra: (*) m 3

♦ Vậy giá trị m cần tìm là m 3 .

Bài tập tương tự


Cho hàm số y   x 3  3x 2  3mx  1 . Tìm m để hàm số nghịch biến trên khoảng  0;   .

Đáp số: m 1.

mx  7m  8
Ví dụ 4. Cho hàm số y  . Tìm m để hàm số đồng biến trên từng khoảng xác
xm

định của nó.


Bài giải
♦ Tập xác định: D  R \ m
m 2  7m  8
♦ Đạo hàm: y '  . Dấu của y ' là dấu của biểu thức m 2  7m  8 .
 x  m
2

♣ Hàm số đồng biến trên từng khoảng xác định


y '  0 , x  D (không có dấu bằng)
m2  7m  8  0 8 m 1

♦ Vậy giá trị m cần tìm là 8 m 1 .

NGUYỄN VĂN LỰC  0933.168.309 SP Toán K35 - ĐH Cần Thơ


Hàm số FB: http://www.facebook.com/VanLuc168

BÀI TẬP TỰ LUYỆN

Câu 1. Xác định m để hàm số sau đồng biến trong khoảng (0; +∞):
xm
y
x2  1

+ TXĐ: D = R
mx  1
+ y’ =
( x  1) x 2  1
2

Hàm số ĐB trong (0; +∞) y’ ≥ 0 mọi x  (0; +∞).


-mx + 1 ≥ 0 mọi x  (0; +∞). (1)
. m = 0 (1) đúng
. m > 0: -mx + 1 ≥ 0 x ≤ 1/m. Vậy (1) không thỏa mãn.
. m < 0: -mx + 1 ≥ 0 x ≥ 1/m. Khi đó (1) 1/m ≤ 0 t/m.
Giá trị cần tìm là: m ≤ 0.

Câu 2. Tìm m để hàm số luôn nghịch biến: y   x3  (3  m) x 2  2mx  12 .

+ Tập xác định: D  R .


+ Đạo hàm: y '  3x 2  2(3  m) x  2m
+ Để hàm số luôn nghịch biến thì y '  0 x

a  0 3  0
 
 '  0 9  m  6m  (3)(2m)  0
2

 m 2  12m  9  0
 6  3 3  m  6  3 3.

Câu 3. Tìm m để hàm số luôn nghịch biến x : y  mx 3  3x 2  3x  1 .

+ Tập xác định: D  R


+ Đạo hàm: y '  3mx 2  6 x  3
+ Để hàm số luôn nghịch biến x thì y '  0 x
 3mx 2  6 x  3  0 x 1
+ TH 1 : m0

NGUYỄN VĂN LỰC  0933.168.309 SP Toán K35 - ĐH Cần Thơ


Hàm số FB: http://www.facebook.com/VanLuc168
(1)  6 x  3  0
 6 x  3
1
 x ( không thỏa x )
2
+ TH 2 : m0
a  0 3m  0 m  0 m  0
(1)       m  1 .
  0 9  9m  0 9m  9 m  1

+ Vậy m  1 thì hàm số thỏa đề bài.

NGUYỄN VĂN LỰC  0933.168.309 SP Toán K35 - ĐH Cần Thơ


Hàm số FB: http://www.facebook.com/VanLuc168

II. CỰC TRỊ CỦA HÀM SỐ


Chuyên đề: Hàm số

A. Tóm tắt lí thuyết


I. KIẾN THỨC CƠ BẢN

1) Định lý 1: (điều kiện cần để hàm số có cực trị)


Giả sử hàm số f đạt cực trị tại điểm x0 . Khi đó nếu f có đạo hàm tại x0 thì f '( x0 ) 0

2) Định lý 2: (điều kiện đủ thứ I để hàm số có cực trị). Quy tắc 1


Giả sử hàm số y f ( x) liên tục trên khoảng a; b chứa điểm x0 và có đạo hàm trên các
khoảng a; x 0 và x0 ; b . Khi đó

a) Nếu f '( x) 0 với mọi x a; x0 và f '( x) 0 với mọi x x0 ; b

thì hàm số f ( x ) đạt cực tiểu tại điểm x0 .


b) Nếu f '( x) 0 với mọi x a; x0 và f '( x) 0 với mọi x x0 ; b

thì hàm số f ( x ) đạt cực đại tại điểm x0 .

3) Định lý 3: (điều kiện đủ thứ II để hàm số có cực trị). Quy tắc 2


Giả sử hàm số f có đạo hàm trên khoảng a; b chứa điểm x0 , f '( x0 ) 0 và f có đạo hàm
cấp hai khác không tại điểm x0 . Khi đó
a) Nếu f ''( x0 ) 0 thì hàm số f ( x ) đạt cực đại tại điểm x0
b) Nếu f ''( x0 ) 0 thì hàm số f ( x ) đạt cực tiểu tại điểm x0

4) Định lý 4:
a) Hàm số y  f  x   ax 3  bx 2  cx  d  a  0  có hai điểm cực trị

 f '  x   3ax 2  2bx  c  0 có hai nghiệm phân biệt.

b) Hàm số y  f  x   ax 4  bx 2  c  a  0  có ba điểm cực trị

 f '  x   4ax 3  2bx  0 có ba nghiệm phân biệt.

NGUYỄN VĂN LỰC  0933.168.309 SP Toán K35 - ĐH Cần Thơ


Hàm số FB: http://www.facebook.com/VanLuc168
B. Phương pháp giải toán
Dạng 1: Định giá trị tham số để hàm số bậc ba (trùng phương) có 2 cực trị (có 3
cực trị).
1. PHƯƠNG PHÁP
B1. Tập xác định: D ?
B2. Tính y ' ?
B3. Lập luận:
Lưu ý:
a) Hàm số y  f  x   ax 3  bx 2  cx  d  a  0  có hai điểm cực trị

 f '  x   3ax 2  2bx  c  0 có hai nghiệm phân biệt.

b) Hàm số y  f  x   ax 4  bx 2  c  a  0  có ba điểm cực trị

 f '  x   4ax 3  2bx  0 có ba nghiệm phân biệt.

2. CÁC VÍ DỤ
1
Ví dụ 1. Cho hàm số y  (m 2  1) x 3  (m  1) x 2  3 x  5 . Tìm m để hàm số có hai điểm cực
3

trị.
Bài giải
♦ Tập xác định: D 
♦ Đạo hàm: y '  (m2  1) x 2  2(m  1) x  3
y' 0 (m 2  1) x 2  2(m  1) x  3  0

♣ Hàm số có hai điểm cực trị


y' 0 có hai nghiệm phân biệt
m2 1 0 m 1
' (m 1)2 3(m2 1) 0 2m 2 2m 4 0

m 1 m 1
1 m 2 1 m 2

m 1
♦ Vậy giá trị m cần tìm là .
1 m 2

NGUYỄN VĂN LỰC  0933.168.309 SP Toán K35 - ĐH Cần Thơ


Hàm số FB: http://www.facebook.com/VanLuc168
Ví dụ 2. Cho hàm số y  mx  (m  9)x  10 . Tìm m để hàm số có 3 điểm cực trị.
4 2 2

Bài giải
♦ Tập xác định: D 
♦ Đạo hàm: y '  4mx3  2(m2  9) x  2 x.(2mx 2  m2  9)
x 0
y' 0
2mx 2 m2 9 0 (1)

♣ Hàm số có ba điểm cực trị


y' 0 có ba nghiệm phân biệt (1) có hai nghiệm phân biệt khác 0
m 0
m 0
m 3 m 3
' 2 m( m 2 9) 0
0 m 3 0 m 3
m2 9 0
m 3

m 3
♦ Vậy giá trị m cần tìm là .
0 m 3

Bài tập tương tự


Cho hàm số y  x 4  (m  1)x 2  2m  1 . Tìm m để hàm số có 3 điểm cực trị.
Đáp số: m 1.

Dạng 2: Định giá trị tham số để hàm số đạt cực trị tại điểm x0.
1. PHƯƠNG PHÁP
B1. Tập xác định: D ?
B2. Tính y ' ?
B3. Lập luận:
a) Điều kiện cần: Hàm số có cực trị tại x 0 y '( x0 ) 0 Giá trị của
tham số m.
b) Điều kiện đủ: Thay giá trị tham số vào y ' thử lại. Khi thử lại có thể
dùng quy tắc 1 hoặc quy tắc 2.

2. VÍ DỤ
1
Ví dụ . Cho hàm số y  x 3   m 2  m  2  x 2  (3m 2  1) x  m  5 .
3

Tìm m để hàm số đạt cực tiểu tại x 2 .

NGUYỄN VĂN LỰC  0933.168.309 SP Toán K35 - ĐH Cần Thơ


Hàm số FB: http://www.facebook.com/VanLuc168
Bài giải
♦ Tập xác định: D 
♦ Đạo hàm: y '  x 2  2  m2  m  2  x  3m2  1

a) Điều kiện cần:


Hàm số đạt cực tiểu tại x 2 y '( 2) 0

m 1
m2 4m 3 0
m 3

b) Điều kiện đủ:


♣ Với m 1 , ta có: y ' x 2 4 x 4 , y ' 0 x 2

Bảng biến thiên


x 2

y' 0

Từ BBT ta suy ra m 1 không thỏa.


x 14
♣ Với m 3 , ta có: y ' x 2 16 x 28 , y ' 0
x 2

Bảng biến thiên


x 14 2
y' 0 0

y CĐ
CT
Từ BBT ta thấy hàm số đạt cực tiểu tại x 2.

♦ Vậy giá trị m cần tìm là m 3 .


Bài tập tương tự
Cho hàm số y  x 3  mx 2  3x  2 . Tìm m để hàm số đạt cực tiểu tại x 2 .
15
Đáp số: m
4

NGUYỄN VĂN LỰC  0933.168.309 SP Toán K35 - ĐH Cần Thơ


Hàm số FB: http://www.facebook.com/VanLuc168
Dạng 3: Định giá trị tham số để hàm số đạt cực trị thỏa điều kiện cho trước.
1. PHƯƠNG PHÁP
B1. Tập xác định: D ?
B2. Tính y ' ?
B3. Lập luận

2. CÁC VÍ DỤ
Ví dụ 1. Cho hàm số y  x 3  (2m  1)x 2  (2  m)x  2 .
Tìm m để hàm số có cực đại, cực tiểu và các điểm cực trị của đồ thị hàm số có hoành
độ dương.
Bài giải
♦ Tập xác định: D 
♦ Đạo hàm: y '  3x 2  2(2m  1) x  2  m
y' 0 3x 2  2(2m  1) x  2  m  0

♦ Hàm số có cực đại, cực tiểu và các điểm cực trị của đồ thị hàm số có hoành độ
dương

' (2m 1) 2 3(2 m) 0


2 m
y' 0 có hai nghiệm dương phân biệt P 0
3
2(2m 1)
S 0
3

5
m 1 m
4m 2 m 5 0 4
5
2 m 0 m 2 m 2
4
2m 1 0 1
m
2
5
♦ Vậy giá trị m cần tìm là m 2 .
4

2 2
Ví dụ 2. Cho hàm số y  x 3  mx 2  2(3m 2  1) x  .
3 3

Tìm m để hàm số có hai điểm cực trị x1 và x2 sao cho x1 x2 2( x1 x2 ) 1 .

Bài giải
♦ Tập xác định: D 

NGUYỄN VĂN LỰC  0933.168.309 SP Toán K35 - ĐH Cần Thơ


Hàm số FB: http://www.facebook.com/VanLuc168
♦ Đạo hàm: y '  2 x  2mx  2(3m  1)
2 2

y' 0 2 x 2  2mx  2(3m 2  1)  0 (1)


♦ Hàm số có hai điểm cực trị x1 và x2
y' 0 có hai nghiệm phân biệt
' m2 4(3m 2 1) 0

2 13 2 13
13m2 4 0 m m (*)
13 13

x1 x2 m
Vì x1 và x2 là nghiệm của (1) nên theo định lý Viet ta có:
x1 x2 1 3m2

m 0
Do đó: x1 x2 2( x1 x2 ) 1 1 3m 2
2m 1 3m 2
2m 2 (**)
m
3
2
♦ Từ (*) và (**) ta suy ra giá trị m cần tìm là m .
3

1 1
Ví dụ 3. Cho hàm số y  mx 3  (m  1) x 2  3(m  2) x  . Tìm m để hàm số có hai điểm
3 3

cực trị x1 và x2 sao cho x1 2 x2 1 .

Bài giải
♦ Tập xác định: D 
♦ Đạo hàm: y '  mx 2  2(m  1) x  3(m  2)
y' 0 mx 2  2(m  1) x  3(m  2)  0 (1)
♦ Hàm số có hai điểm cực trị x1 và x2 y' 0 có hai nghiệm phân biệt
m 0
m 0
2 6 2 6 (*)
' 2m 2 4m 1 0 m
2 2

Vì x1 và x2 là nghiệm của (1) nên theo định lý Viet ta có:


2(m 1)
x1 x2 (2)
m
3(m 2)
x1 x2 (3)
m

Theo đề bài : x1 2 x2 1 (4)

NGUYỄN VĂN LỰC  0933.168.309 SP Toán K35 - ĐH Cần Thơ


Hàm số FB: http://www.facebook.com/VanLuc168
3m 4
x1
Từ (2) và (4) suy ra m (5). Thay (5) và (3) ta được:
m 2
x2
m

2
3m 4 2 m 3(m 2) 2 m
6m 16m 8 0 3 (**)
m m m
m 2

2
♦ Từ (*) và (**) ta suy ra giá trị m cần tìm là m và m 2 .
3

Ví dụ 4. Cho hàm số y  x 3  3mx  1 (1), với m là tham số thực. Cho điểm A(2;3) . Tìm
m để đồ thị hàm số (1) có hai cực trị B và C sao cho tam giác ABC cân tại A .
Bài giải
♦ Tập xác định: D 
♦ Đạo hàm: y '  3x 2  3m
y' 0 3x 2  3m  0 (1)
♦ Đồ thị hàm số (1) có hai cực trị B và C y' 0 có hai nghiệm phân biệt
m 0 (*)
Khi đó y ' 0 có hai nghiệm phân biệt là x m

♣ Với x m y 2 m3 1

♣ Với x m y 2 m3 1

Tọa độ các điểm cực trị B và C là B m ; 2 m3 1 ,C m ; 2 m3 1

♦ Tam giác ABC cân tại A AB AC AB 2 AC 2

2 2
m 0
2 2
3 3 3
2 m 2 2 m 2 m 2 2 m 4 m 8 m 0 1 (**)
m
2
1
♦ Từ (*) và (**) ta suy ra giá trị m cần tìm là m .
2

Ví dụ 5. Cho hàm số y  x 4  2mx 2  2m  m 4 (1), với m là tham số thực. Tìm m để đồ


thị hàm số (1) có ba điểm cực trị A, B, C đồng thời các điểm A, B, C tạo thành một tam
giác vuông.
Bài giải

NGUYỄN VĂN LỰC  0933.168.309 SP Toán K35 - ĐH Cần Thơ


Hàm số FB: http://www.facebook.com/VanLuc168
♦ Tập xác định: D 
♦ Đạo hàm: y '  4 x3  4mx  4 x( x 2  m)
x  0
y' 0  2 (1)
x  m

♦ Đồ thị hàm số (1) có ba điểm cực trị A, B, C y' 0 có ba nghiệm phân biệt
m 0 (*)
Khi đó y ' 0 có ba nghiệm phân biệt là x 0 , x m

♣ Với x 0 y 2m m4

♣ Với x m y m4 m2 2m

Tọa độ các điểm cực trị A, B, C là


A 0; 2m m4 ; B m ; m4 m2 2m ; C m ; m4 m2 2m

Suy ra: AB m ; m 2 ; AC m ; m2

♦ Tam giác ABC vuông Tam giác ABC vuông tại A


m 0
AB. AC 0 m m4 0 (**)
m 1

♦ Từ (*) và (**) ta suy ra giá trị m cần tìm là m 1 .

NGUYỄN VĂN LỰC  0933.168.309 SP Toán K35 - ĐH Cần Thơ


Hàm số FB: http://www.facebook.com/VanLuc168

BÀI TẬP TỰ LUYỆN

1 1
Câu 1. Tìm cực trị của của hàm số y  x3  x 2  2 x  2 .
3 2

Cách 1.
* Tập xác định:R.
 x  1
Ta có: y '  x 2  x  2; y '  0   .
x  2
* Bảng biến thiên:
x  –1 2

y’ + 0 – 0 +
y

Vậy hàm số đạt cực đại tại x = -1 và giá trị cực đại yCĐ  y  1  19
6
Hàm số đạt cực tiểu tại x = 2 và giá trị cực tiểu yCT  y  2   4 .
3
Cách 2.
* Tập xác định:.
 x  1
Ta có: y '  x 2  x  2; y '  0   .
 x  2
* y ''  2 x  1, y ''  1  3  0 nên hàm số đạt cực đại tại điểm x = -1 và giá trị cực đại

yCĐ  y  1  19
6
* y ''  2   3  0 nên hàm số đạt cực tiểu tại x = 2 và giá trị cực tiểu .

Câu 2. Tìm các điểm cực trị của đồ thị hàm số y  x3  3x 2  6

Tìm các điểm cực trị của đồ thị hàm số y  x3  3x 2  6


* Tập xác định:

NGUYỄN VĂN LỰC  0933.168.309 SP Toán K35 - ĐH Cần Thơ


Hàm số FB: http://www.facebook.com/VanLuc168
x  0
y '  3x 2  6 x, y '  0  
x  2
Bảng xét dấu đạo hàm
x  0 2 
y + 0 - 0 +
Từ bảng xét đấu đạo hàm ta có
Hàm số đa ̣t cực đa ̣i ta ̣i x  0 và giá tri ̣ cực đa ̣i y  6 ; đa ̣t cực tiể u ta ̣i x  2 và giá tri ̣ cực
tiể u y  2 .
Vậy điểm cực đại của đồ thị hàm số là M  0; 6  , điểm cực tiểu của đồ thị hàm số là
N  2; 2 

Câu 3. Tìm các điểm cực trị của hàm số y  2 x 4  4 x 2  1 .

TXĐ: D 
y '  8 x 3 -8x  8 x ( x 2 -1) x  D
x  0
y'  0  
 x  1
Bảng xét dấu của y’:
x - -1 0 1
+
y’ - 0 + 0 - 0
+

Kết luận: Hàm số đạt cực đại tại x = 0 và ycd  y (0)  1.
Hàm số đạt cực tiểu tại x = ± 1 và yct  y ( 1)  3.

Câu 4. Cho hàm số y  x3  3mx 2   m2  1 x  2, m là tham số.


Tìm tất cả các giá trị của m để hàm số đã cho đạt cực tiểu tại x 2.

Ta có: y '  3x 2  6mx  m 2  1; y ''  6 x  6m


 y '(2)  0
Hàm số đã cho đạt cực tiểu tại x  2  
 y ''(2)  0
m2  12m  11  0
  m 1
12  6m  0
Vậy với m = 1 thì thỏa mãn yêu cầu bài toán.

NGUYỄN VĂN LỰC  0933.168.309 SP Toán K35 - ĐH Cần Thơ


Hàm số FB: http://www.facebook.com/VanLuc168
Câu 5. Cho hàm số y  x  3mx  3(m  1) x  m  m
3 2 2 3
(1). Tìm m để hàm số (1) có
cực trị đồng thời khoảng cách từ điểm cực đại của đồ thị hàm số đến gốc tọa độ O bằng
2 lần khoảng cách từ điểm cực tiểu của đồ thị hàm số đến gốc tọa độ O.

Ta có y  3x 2  6mx  3(m2  1)
Hàm số (1) có cực trị thì PT y  0 có 2 nghiệm phân biệt
 x2  2mx  m2  1  0 có 2 nhiệm phân biệt    1  0, m
Khi đó, điểm cực đại A(m  1;2  2m) và điểm cực tiểu B(m  1; 2  2m)
 m  3  2 2
Ta có OA  2OB  m2  6m  1  0   .
 m  3  2 2

m
Câu 6. Tìm m để hàm số y 2 x đạt cực tiểu tại điểm x = 1.
4
x 1 m
4

y '  m  x  1  m  2
3

Điều kiện cần y ' 1  0  m  2


Thử lại m = 2 : y '  2  x  1 đổi dấu từ âm sang dương khi đi qua x = 1
3

Vậy nhận m = 2

Câu 7. Cho hàm số y  x 3  3(m  1) x 2  9 x  m , với m là tham số thực.


Xác định m để hàm số đã cho đạt cực trị tại x1 , x2 sao cho x1  x2  2 .

Ta có: y'  3x  6(m  1) x  9.


2

Hàm số đạt cực đại, cực tiểu tại x1 , x2  Phương trình y '  0 có hai nghiệm pb là x1 , x2
 Pt x  2(m  1) x  3  0 có hai nghiệm phân biệt là x1 , x 2
2

  '  (m  1) 2  3  0
 m  1  3
 (1)
 m  1  3

Với ĐK (1), theo định lý Viet ta có: x1  x2  2(m  1); x1 x2  3.


x1  x2  2   x1  x2   4 x1 x2  4
2

 4  m  1  12  4
2

NGUYỄN VĂN LỰC  0933.168.309 SP Toán K35 - ĐH Cần Thơ


Hàm số FB: http://www.facebook.com/VanLuc168
 (m  1)  4
2

 m  3
 (2)
m  1

 m  3

Từ (1) và (2) ta được:  m  1 TMYCBT.

Câu 8. Cho hàm số: y  x3  3(m  1) x 2  9 x  m , với m là tham số thực.Xác định m để


hàm số đã cho đạt cực trị tại x1 , x2 sao cho x1  x2  2 .

 Ta có y '  3x2  6(m  1) x  9.


 Hàm số có cực đại, cực tiểu x1, x2.  PT y’ = 0 có hai nghiệm phân biệt là x1, x2.
 x 2  2(m  1) x  3  0 có hai nghiệm phân biệt là x1 , x2 .
  '  (m  1) 2  3  0  m  1  3  m  1  3 (1)
Theo đề ta có: x1  x2  2   x1  x2   4 x1x2  4
2
(*)
Theo định lý Viet ta có: x1  x2  2(m  1); x1 x2  3.
(*)  4  m  1  12  4  (m  1)2  4  3  m  1
2
(2)
Từ (1) và (2) suy ra giá trị m cần tìm là: 3  m  1  3 hoặc 1  3  m  1.

Câu 9. Cho hàm số: y  x4  2(m2  1) x2  1 (1)


Tìm các giá trị của tham số m để hàm số (1) có 3 điểm cực trị thỏa mãn giá trị cực
tiểu đạt giá trị lớn nhất.

y’ = 4x3 – 4(m2+1)x
x  0
y’ = 0    hàm số (1) luôn có 3 điểm cực trị với mọi m
 x   m  1
2

xCT   m2  1  giá trị cực tiểu yCT  (m2  1)2  1


Vì (m 2  1) 2  1  yCT  0 max( yCT )  0  m2  1  1  m  0

Câu 10. Cho hàm số y   x3  3mx  1 (1). Tìm m để đồ thị của hàm số (1) có 2 điểm
cực trị A, B sao cho tam giác OAB vuông tại O ( với O là gốc tọa độ ).

y '  3x 2  3m  3  x 2  m 
y '  0  x 2  m  0  *
Đồ thị hàm số (1) có 2 điểm cực trị  PT (*) có 2 nghiệm phân biệt  m  0 **
NGUYỄN VĂN LỰC  0933.168.309 SP Toán K35 - ĐH Cần Thơ
Hàm số FB: http://www.facebook.com/VanLuc168
Khi đó 2 điểm cực trị A   m ;1  2m m  , B  m ;1  2m m 
1
Tam giác OAB vuông tại O  OA.OB  0  4m3  m  1  0  m  ( TM (**) )
2
1
Vậy m 
2

Câu 11. Cho hàm số f ( x)  x4  2(m  2) x2  m2  5m  5 (Cm)


Tìm m để (Cm) có các điểm cực đại, cực tiểu tạo thành 1 tam giác vuông cân.

Hàm số có CĐ, CT khi m < 2 . Toạ độ các điểm cực trị là:
A(0; m 2  5m  5), B( 2  m ;1  m), C ( 2  m ;1  m)

Tam giác ABC luôn cân tại A  ABC vuông tại A khi m = 1.

Câu 12. Cho hàm số y 2x 3 3x 2 1 1


Tìm tọa độ giao điểm của đường thẳng d : y 2x 1 với đồ thị (C). Tìm tọa độ điểm

M thuộc d và cùng với hai điểm cực trị của đồ thị (C) tạo thành một tam giác vuông tại
M.

Xét phương trình hoành độ giao điểm của d : y  2 x  1 và đồ thị (C) là:
2 x 3  3 x 2  1  2 x  1  2 x 3  3 x 2  2 x  0 (*)
1
Giải phương trình (*) ta được ba nghiệm phân biệt x1  0, x 2  2, x 3  
2
 1 
Vậy d cắt (C) tại ba điểm phân biệt A(0;1), B(2;5),C   ; 0 
2  
M  d : y  2x  1  M (t;2t  1) , tọa độ các điểm cực trị của (C) là D(0;1),T (1; 0)
M cùng với hai điểm cực trị của đồ thị (C) tạo thành tam giác vuông tại M
 DM .TM  0(**) , mặt khác ta có DM  (t;2t ),TM  (t  1;2t  1)
1
 (**)  5t 2  t  0  t  0 hoặc t  
5
1  1 3
t  0  M (0;1)  D (loại); t    M  ; 
5  5 5

Câu 13. Cho hàm số y  x4  2m2 x2  1 Cm  (1). Tìm m dể hàm số (1) có ba điểm cực
trị là ba đỉnh của một tam giác vuông cân.

x  0
Ta có: y '  4 x3  4m 2 x  4 x  x 2  m 2   0    m  0 (*)
x  m
2 2

Với điều kiện (*) thì hàm số (1) có ba điểm cực trị. Gọi ba điểm cực trị là:

NGUYỄN VĂN LỰC  0933.168.309 SP Toán K35 - ĐH Cần Thơ


Hàm số FB: http://www.facebook.com/VanLuc168
A  0;1 ; B  m;1  m  ; C  m;1  m  . Do đó nếu ba điểm cực trị tạo thành một tam giác
4 4

vuông cân, thì đỉnh sẽ là A.


Do tính chất của hàm số trùng phương, tam giác ABC đã là tam giác cân rồi, cho nên
để thỏa mãn điều kiện tam giác là vuông, thì AB vuông góc với AC.
 AB   m; m4  ; AC   m; m 4  ; BC   2m;0 

Tam giác ABC vuông khi: BC 2  AB 2  AC 2  4m2  m2  m8   m2  m8 


 2m2  m4  1  0;  m4  1  m  1
Vậy với m = -1 và m = 1 thì thỏa mãn yêu cầu bài toán.

Câu 14. Cho hàm số y  x4  2m2 x2  1 (1).Tìm tất cả các giá trị m để đồ thị hàm số (1)
có ba điểm cực trị A, B, C và diện tích tam giác ABC bằng 32 (đơn vị diện tích).

x  0
+) Ta có y’ = 4x3 – 4m2x ; y’ = 0   ; ĐK có 3 điểm cực trị: m  0
x  m
2 2

+) Tọa độ ba điểm cực trị: A(0 ; 1), B(- m ; 1 – m4), C(m ; 1 – m4) ;
+) CM tam giác ABC cân đỉnh A. Tọa độ trung điểm I của BC là I(0 ; 1 – m4).
1
 AI .BC  m4 m  m  32  m  2 (tm)
5
+) S ABC
2

Câu 15. Cho hàm số y  x4  2mx 2  m  1 (1), với m là tham số thực. Xác định m để
hàm số (1) có ba điểm cực trị, đồng thời các điểm cực trị của đồ thị tạo thành một tam
giác có bán kính đường tròn ngoại tiếp bằng 1 .

x  0
y '  4 x3  4mx  4 x  x 2  m   0   2
x  m
Hàm số đã cho có ba điểm cực trị  pt y '  0 có ba nghiệm phân biệt và y ' đổi dấu khi
x đi qua các nghiệm đó  m  0
 Khi đó ba điểm cực trị của đồ thị hàm số là:
  
A  0; m  1 , B  m ; m 2  m  1 , C m ; m 2  m  1
1
 S 
yB  y A . xC  xB  m2 m ; AB  AC  m4  m , BC  2 m
ABC
2
m  1
R
AB. AC.BC
1

m4  m 2 m   1  m  2m  1  0  
3

4S ABC 4m m2 m  5  1
 2

Câu 16. Cho hàm số y = x3 – 3x2+2 (1)

NGUYỄN VĂN LỰC  0933.168.309 SP Toán K35 - ĐH Cần Thơ


Hàm số FB: http://www.facebook.com/VanLuc168
Gọi d là đường thẳng đi qua điểm A(1;1) và có hệ số góc bằng 3. Tìm điểm M thuộc
đường thẳng d sao tổng khoảng cách từ M tới hai điểm cực trị nhỏ nhất.

+ d: y=3x-2
+ Xét biểu thức P=3x-y-2. Thay tọa độ điểm (0;2)=>P=-4<0, thay tọa độ điểm (2;-
2)=>P=6>0. Vậy 2 điểm cực đại và cực tiểu nằm về hai phía của đường thẳng d. Từ
đây, để MA+MB nhỏ nhất => 3 điểm A, M, B thẳng hàng
+ Phương trình đường thẳng AB: y=-2x+2
 4
 x
 y  3x  2  5
+ Tọa độ điểm M là nghiệm của hệ:  
 y  2 x  2 y  2
 5

Câu 17. Cho hàm số y  x 3  6 x 2  9 x  2 (1).


Viết phương trình đường thẳng đi qua điểm A 1;1  và vuông góc với đường thẳng đi
qua hai điểm cực trị của (C).

Viết phương trình đường thẳng đi qua điểm A 1;1  và vuông góc với đường thẳng đi
qua hai điểm cực trị của (C).

Đuờng thẳng đi qua 2 c ực trị A(1;2) và B(3;-2) là y=-2x+4


Ta có pt đt vuông góc với (AB) nên có hệ số góc k= ½
1 3
Vậy PT đ ư ờng thẳng cần tìm là y  x 
2 2

Câu 18. Cho hàm số y  x3  3mx 2  4m2  2 (1), m là tham số.


Tìm m để đồ thị hàm số (1) có hai điểm cực trị A và B sao cho điểm I (1; 0) là trung
điểm của đoạn AB.

Ta có y '  3x 2  6mx.
x  0
y '  0  3x 2  6mx  0  
 x  2m.
Đồ thị hàm số (1) có hai cực trị khi và chỉ khi y '  0 có hai nghiệm phân biệt  m0.
Tọa độ các điểm cực trị là A(0; 4m2  2), B(2m; 4m3  4m 2  2) .
m  1
Điểm I (1; 0) là trung điểm của đoạn AB khi và chỉ khi 
2m  4m  2  0
3 2

Giải hệ, ta được m  1. Vậy m 1 là giá trị cần tìm.

NGUYỄN VĂN LỰC  0933.168.309 SP Toán K35 - ĐH Cần Thơ


Hàm số FB: http://www.facebook.com/VanLuc168
Câu 19. Cho hàm số y   x  (2m  1) x  (m  3m  2) x  4 (m là tham số) có đồ thị là
3 2 2

(Cm). Xác định m để (Cm) có các điểm cực đại và cực tiểu nằm về hai phía của trục
tung.

y  3x 2  2(2m  1) x  (m2  3m  2) .


(Cm) có các điểm CĐ và CT nằm về hai phía của trục tung  PT y  0 có 2 nghiệm
trái dấu  3(m2  3m  2)  0  1  m  2

Câu 20. Cho hàm số y  x3  3mx2  4m3 (m là tham số) có đồ thị là (Cm). Xác định m
để (Cm) có các điểm cực đại và cực tiểu đối xứng nhau qua đường thẳng y = x.

x  0
Ta có: y’ = 3x2  6mx = 0  
 x  2m
Để hàm số có cực đại và cực tiểu thì m  0.
Giả sử hàm số có hai điểm cực trị là: A(0; 4m3), B(2m; 0)  AB  (2m; 4m3 )
Trung điểm của đoạn AB là I(m; 2m3)
Điều kiện để AB đối xứng nhau qua đường thẳng y = x là AB vuông góc với đường
thẳng y = x và I thuộc đường thẳng y = x

 2m  4m  0
3

 3
 2m  m

2
Giải hệ phương trình ta được m   ;m=0
2
2
Kết hợp với điều kiện ta có: m  
2

NGUYỄN VĂN LỰC  0933.168.309 SP Toán K35 - ĐH Cần Thơ


Hàm số FB: http://www.facebook.com/VanLuc168

III. GIÁ TRỊ LỚN NHẤT – GIÁ TRỊ NHỎ NHẤT


Chuyên đề: Hàm số

A. Tóm tắt lí thuyết & phương pháp giải toán


I. KIẾN THỨC CƠ BẢN

1) ĐỊNH NGHĨA: Giả sử hàm số y  f  x  xác định trên tập hợp D.


 Số M được gọi là GTLN của hàm số y  f  x  trên tập D nếu các điều sau được
thỏa mãn
i) f  x   M x  D
ii) x  D : f x  M
 0  0
Ký hiệu: M  Max f  x 
xD

 Số m được gọi là GTNN của hàm số y  f  x  trên tập D nếu các điều sau được
thỏa mãn
i) f  x   m x  D
ii) x  D : f x  m
 0  0
Ký hiệu: m  min f x
xD

 Quy ước: Ta quy ước rằng khi nói GTLN hay GTNN của hàm số f mà không
nói "trên tập D" thì ta hiểu đó là GTLN hay GTNN trên TẬP XÁC ĐỊNH của
nó.
 Đối với GTLN và GTNN đối với hàm nhiều biến cũng có định nghĩa tương tự.

2) CÁC PHƯƠNG PHÁP THƯỜNG DÙNG ĐỂ TÌM GTLN & GTNN CỦA
HÀM SỐ MỘT BIẾN
a) Phương pháp 1: Sử dụng bất đẳng thức (hay phương pháp dùng định
nghĩa).
Một số kiến thức thường dùng:
b 2 
a) f ( x )  ax 2  bx  c  a( x  ) 
2a 4a
b) Bất đẳng thức Cô-si:
ab
 Với hai số a, b không âm  a, b  0  ta luôn có:  ab  a  b  2 ab
2
Dấu "=" xảy ra khi a  b
abc 3
 Với ba số a, b, c không âm  a, b, c  0  ta luôn có:  abc  a  b  c  3 3 abc
3
Dấu "=" xảy ra khi a  b  c
c) Một số bất đẳng thức cơ bản thường dùng
a 2  b2
1) a 2  b 2  2ab  ab 
2

NGUYỄN VĂN LỰC  0933.168.309 SP Toán K35 - ĐH Cần Thơ


Hàm số FB: http://www.facebook.com/VanLuc168
( a  b) 2
2) (a  b) 2  4ab  ab 
4
( a  b) 2
3) (a  b)2  2(a 2  b 2 )  a 2  b 2 
2

CÁC VÍ DỤ
Ví dụ 1: Tìm GTLN của hàm số f  x   2x 2  8x  1 .
Bài giải
♥ Tập xác định: D
♥ Ta có
f  x   2x 2  8x  1  9  2  x  2   9, x  D
2

Dấu “=” xảy ra khi x 2 D


♥ Vậy max
x D
f ( x ) 9 .

Ví dụ 2: Tìm GTNN của hàm số f  x   2x 2  4x  12 .


Bài giải
♥ Tập xác định: D
♥ Ta có
f  x   2x 2  4x  12 = 2  x  1  10  10 ,x  D
2

Dấu “=” xảy ra khi x 1 D


♥ Vậy min
x D
f ( x) 10 .

2
Ví dụ 3: Tìm GTNN của các hàm số f  x   x  với x  1;   .
x 1
Bài giải
♥ D 1;
♥ Theo bất đẳng thức Cô-si ta có:
2 2 2
f x  x   x 1   1  2  x  1 .  1  2 2  1, x  1;  
x 1 x 1 x 1
2 2
Dấu “=” xảy ra khi x 1 x 1 2 x 1 2 D
x 1
♥ Vậy min
x D
f ( x) 2 2 1 .
Bài tập tương tự
7
Tìm GTNN của hàm số f (x)  x  3 
x 3
b) Phương pháp 2 : Sử dụng điều kiện có nghiệm của phương trình (hay phương
pháp miền giá trị).
Cơ sở lý thuyết của phương pháp: Cho hàm số xác định bởi biểu thức dạng
y  f x
 Tập xác định của hàm số được định nghĩa là :
D  { x  | f(x) có nghĩa}
 Tập giá trị của hàm số được định nghĩa là :
NGUYỄN VĂN LỰC  0933.168.309 SP Toán K35 - ĐH Cần Thơ
Hàm số FB: http://www.facebook.com/VanLuc168
T = { y  | Phương trình f(x) = y có nghiệm x  D }
Do đó nếu ta tìm được tập giá trị T của hàm số thì ta có thể tìm đựơc GTLN và
GTNN của hàm số đó.
Một số kiến thức thường dùng:
a) Phương trình ax 2  bx  c  0  a  0  có nghiệm    0
b) Phương trình a cos x  b sin x  c  a, b  0  có nghiệm  a 2  b 2  c2

CÁC VÍ DỤ
x2  x  2
Ví dụ 1 : Tìm GTLN và GTNN của hàm số y  . (1)
x2  x  2
Bài giải
♥ Tập xác định: D
♥ Xem (1) là phương trình theo ẩn x ta có:
x2  x  2
y  yx 2  yx  2y  x 2  x  2
x x2
2

y 1 x 2 y 1 x 2y 2 0 (2) (Dạng ax 2  bx  c  0 )
+ Trường hợp 1: Với y 1 thì (2) có nghiệm x 0
+ Trường hợp 2: Với y 1 thì (2) có nghiệm 0
7 y 2 18y 7 0
9 4 2 9 4 2
y
7 7
9 4 2 9 4 2
Suy ra tập giá trị của hàm số là T ; .
7 7

9 4 2 9 4 2
♥ Vậy min y ; max y .
x D
7 x D
7

1  sin x
Ví dụ 2: Tìm GTLN và GTNN của hàm số y  . (1)
2  cos x
Bài giải
♥ Tập xác định: D
♥ Xem (1) là phương trình theo ẩn x ta có:
1  2y  y cos x  1  sin x y cos x sin x 1 2y (2) (dạng a cos x  b sin x  c )
2 2 3
(2) có nghiệm a2 b2 c2 y2 1 1 2y 3y 2 4y 0 0 y
4
3
Suy ra tập giá trị của hàm số là T 0; .
4
3
♥ Vậy min y 0; max y .
x D x D
4

c) Phương pháp 3 : Sử dụng đạo hàm (hay phương pháp giải tích).
 Điều kiện tồn tại GTLN và GTNN:
Định lý: Hàm số liên tục trên một đoạn  a; b  thì đạt được GTLN và GTNN
trên đoạn đó.
NGUYỄN VĂN LỰC  0933.168.309 SP Toán K35 - ĐH Cần Thơ
Hàm số FB: http://www.facebook.com/VanLuc168
 Phương pháp chung: Muốn tìm GTLN và GTNN của hàm số y  f  x  trên
miền D, ta lập BẢNG BIẾN THIÊN của hàm số trên D rồi dựa vào BBT suy
ra kết quả.
 Phương pháp riêng:
Trong nhiều trường hợp, có thể tìm GTLN và GTNN của hàm số trên một
đoạn mà không cần lập bảng biến thiên của nó. Giả sử hàm số f liên tục
trên đoạn  a; b  và có đạo hàm trên khoảng  a; b  , có thể trừ một số hữu hạn
điểm . Nếu f '( x)  0 chỉ tại một số hữu hạn điểm thuộc  a; b  thì ta có quy tắc
tìm GTLN và GTNN của hàm f trên đoạn  a; b  như sau:
Quy tắc
1) Tìm các điểm x1 , x2 ,..., xm thuộc  a; b  mà tại đó hàm số f có đạo hàm bằng
0 hoặc không có đạo hàm.
2) Tính f ( x1 ), f ( x2 ),..., f ( xm ), f (a), f (b) .
3) So sánh các giá trị tìm được.
 Số lớn nhất trong các giá trị đó là GTLN của f trên đoạn  a; b  .
 Số nhỏ nhất trong các giá trị đó là GTNN của f trên đoạn  a; b  .

CÁC VÍ DỤ
i. XÉT HÀM TRỰC TIẾP
Ví dụ 1: Tìm giá trị lớn nhất và giá trị nhỏ nhất của hàm số y 2 x 3 3 x 2 12 x 2 trên
đoạn 1;2 .
Bài giải
♥ D 1;2
♥ Ta có: y ' 6 x 2 6 x 12
x 2 D
y' 0
x 1 D
Do y 1 15; y 2 6; y 1 5 min y 5; max y 15
x D x D

♥ Vậy min
x D
y 5; max y
x D
15 .

Ví dụ 2: Tìm giá trị lớn nhất và giá trị nhỏ nhất của hàm số y e x x 2 x 1 trên đoạn
0;2 .
Bài giải
♥ D 0;2
♥ Ta có: y ' e x x 2 x 2
x 2 D
y' 0
x 1 D
Do y 0 1; y 2 e2 ; y 1 e min y
x D
e; max y
x D
e2
♥ Vậy min
x D
y e; max y
x D
e2 .

NGUYỄN VĂN LỰC  0933.168.309 SP Toán K35 - ĐH Cần Thơ


Hàm số FB: http://www.facebook.com/VanLuc168

Ví dụ 3: Tìm giá trị lớn nhất và giá trị nhỏ nhất của hàm số y x 4 x2 .
Bài giải
♥ D 2;2
4 x2 x
♥ Ta có: y '
4 x2
y' 0 x 2 D
Do y 2 2; y 2 2; y 2 2 2 min y 2 2; max y 2
x D x D

♥ Vậy min
x D
y 2 2; max y
x D
2 .

ii. ĐỔI BIẾN (ĐẶT ẨN PHỤ)

Ví dụ 4: Tìm giá trị lớn nhất và giá trị nhỏ nhất của hàm số y 2 sin 2 x cos x 1 .
Bài giải
♥ Tập xác định: D
♥ Đặt t cos x với t 1;1 , hàm số trở thành: y 2t 2 t 3
1
Ta có: y ' 4t 1 ; y' 0 t 1;1
4
1 25 25
Do y 1 2; y 1 0; y min y 0; max y
4 8 x D x D
8
♥ Vậy min
x D
y 2 2; max y
x D
2 .

BÀI TẬP TỰ LUYỆN

Câu 1. Tìm giá trị lớn nhất và nhỏ nhất của hàm số : y  x 4  2 x 2  3 trên đoạn 0;4 .

y’= 0  x=0, x=1  0;4 x= -1 loại


Ta có: f(0) =3 , f(1)=2 , f(4)=227
Vậy GTLN y = 227 , trên 0;4 khi x=4
GTNN y= 2 trên trên 0;4 khi x=1

Câu 2. Tìm giá trị lớn nhất và giá trị nhỏ nhất của hàm số f  x   x  4  x 2 . trên đoạn
 1
 2; 2  .

x
+ Ta có f '(x)  1 
4  x2

NGUYỄN VĂN LỰC  0933.168.309 SP Toán K35 - ĐH Cần Thơ


Hàm số FB: http://www.facebook.com/VanLuc168
1
+ f '(x)  0  x  2  [  2; ]
2
1 1  15
+ Có f (2)  2;f ( ) 
2 2
1  15
maxf(x)  2
; minf(x)  2
1 1
[-2; ] [-2; ]
2 2

Câu 3. Tìm giá trị lớn nhất và giá trị nhỏ nhất của hàm số f  x    x  2   x  2  trên
2 2

đoạn   ; 2  .
1
 2 

Ta có f  x   x 4  4 x 2  4 ; f  x  xác định và liên tục trên đoạn   ;0 ;


1
 2 
f '
 x  4x 3
 8 x.

Với x   ; 2 , f '  x   0  x  0; x  2


1
 2 
Ta có f     3 , f  0   4, f  2   0, f  2   4 .
1 1
 2 16

Giá trị lớn nhất và giá trị nhỏ nhất của hàm số f  x  trên đoạn   ;0 lần lượt là 4 và 0 .
1
 2 

Câu 4. Tìm giá trị lớn nhất và giá trị nhỏ nhất của hàm số
y  f  x   x 2  ln 1  2 x  trên đoạn  1;0.

Tìm giá trị lớn nhất và giá trị nhỏ nhất của hàm số y  f  x   x 2  ln 1  2 x  trên đoạn
 1;0.
x  1
2
Ta có f '  x   2 x  ; f ' x  0  
1 2x x   1
 2
Tính f  1  1  ln 3; f      ln 2; f  0   0
1 1
 2 4
1
Vậy min f  x    ln 2; max f  x   0
 1;0 4  1;0

Câu 5. Tìm giá trị nhỏ nhất của hàm số y  x.log x trên khoảng (0;10).

1
Hàm số đã cho liên tục trên (0;10]. Ta có f '( x)  log x  x.  log x  log e .
x ln10
1
f '( x)  0  log x   log e  x  .
e
BBT:

NGUYỄN VĂN LỰC  0933.168.309 SP Toán K35 - ĐH Cần Thơ


Hàm số FB: http://www.facebook.com/VanLuc168
x 0 1/e 10
f’(x) - 0 +

f(x)
log e

e

log e 1
Từ BBT ta suy ra min f '( x)   x .
(0;10] e e

4
Câu 6. Tìm giá trị lớn nhất và nhỏ nhất của hàm số f  x   x  3  trên đoạn  2;5 .
x 1

- Ta có f  x  liên tục và xác định trên đoạn  2;5 ; f '  x   1 


4
 x  1
2

- Với x   2;5 thì f '  x   0  x  3

- Ta có: f  2   3, f  3  2, f  5   3

- Do đó: Max f  x   3  x  2  x  5 , min f  x   2  x  3


 2;5  2;5

x 1
Câu 7. Tìm giá trị lớn nhất và giá trị nhỏ nhất của hàm số y  trên đoạn 2; 4  .
2x  1

Hàm số liên tục trên đoạn 2; 4 


1
Ta có y '   0, x  2; 4 
 
2
2x  1
1 3
Có y 2   ; y  4  
3 7
3 1
Vậy max y= khi x  4 và min y = khi x  2
  2;4  7 2;4  3

Câu 8. Tìm giá trị lớn nhất và giá trị nhỏ nhất của hàm số f ( x)  2 x 4  4 x 2  10 trên
đoạn  0; 2

f ( x) xác định và liên tục trên đoạn  0; 2 , ta có: f '( x)  8 x3  8 x

x  0
Với x   0; 2 thì: f '( x)  0   . Ta có: f(0) = 10; f(1) = 12; f(2) = -6
x  1

Vậy: M0;2ax f ( x)  f (1)  12; min f ( x)  f (2)  6


  0;2  

NGUYỄN VĂN LỰC  0933.168.309 SP Toán K35 - ĐH Cần Thơ


Hàm số FB: http://www.facebook.com/VanLuc168

IV. SỰ TƯƠNG GIAO CỦA HAI ĐỒ THỊ


Chuyên đề: Hàm số

A. Tóm tắt lí thuyết


I. KIẾN THỨC CƠ BẢN
Bài toán tổng quát
(C1 ) : y  f(x)
Trong mp(Oxy) . Hãy xét sự tương giao của đồ thị hai hàm số : 
(C2 ) : y  g(x)
y (C1 ) y (C1 ) y (C1 )
M 1 y2 M2
y1 (C 2 ) M0
x x
x
O x1 O x2 O
(C 2 )
(C 2 )

(C1) và (C2) không có điểm chung (C 1) và (C2) cắt nhau (C1) và


(C2) tiếp xúc nhau

Phương pháp chung:


* Thiết lập phương trình hoành độ giao điểm của đồ thị hai hàm số đã cho:
f(x) = g(x) (1)
* Tùy theo số nghiệm của phương trình (1) mà ta kết luận về số điểm chung
của hai đồ thị (C1) và (C2) .
Lưu ý:
Số nghiệm của phương trình (1) chính là số giao điểm của hai đồ thị (C1) và (C2).
Ghi nhớ: Số nghiệm của pt (1) = số giao điểm của hai đồ thị (C 1) và (C2).

Chú ý 1 :
* (1) vô nghiệm  (C1) và (C2) không có điểm điểm chung
* (1) có n nghiệm  (C1) và (C2) có n điểm chung
Chú ý 2 :
* Nghiệm x0 của phương trình (1) chính là hoành độ điểm chung của
(C1) và (C2).
Khi đó tung độ điểm chungylà y0 = f(x0) hoặc y0 = g(x0).

y0

x
x0 O

NGUYỄN VĂN LỰC  0933.168.309 SP Toán K35 - ĐH Cần Thơ


Hàm số FB: http://www.facebook.com/VanLuc168
B. Phương pháp giải toán
(C1 ) : y f ( x)
Dạng 1: Tìm tọa độ giao điểm của hai đồ thị .
(C2 ) : y g ( x)

1. PHƯƠNG PHÁP
B1. Lập phương trình hoành độ giao điểm: f ( x) g ( x) (1)
B2. Giải phương trình (1) tìm x y
B3. Kết luận

2. VÍ DỤ
2x  1
Ví dụ . Tìm tọa độ giao điểm của đường cong (C): y  và đường thẳng y  x  2 .
2x  1

Bài giải
2x 1
♦ Phương trình hoành độ giao điểm: x 2 (1)
2x 1
1
Điều kiện: x
2

♦ Khi đó: (1) 2x 1 (2 x 1)( x 2)

x 1
2
2x x 3 0 3
x
2
3 1
♣ Với x y
2 2

♣ Với x 1 y 3

3 1
♦ Vậy tọa độ giao điểm cần tìm là ; và 1;3 .
2 2

(C1 ) : y f ( x)
Dạng 2: Tìm tham số để hai đồ thị cắt nhau tại 2( 3, 4) điểm phân
(C2 ) : y g ( x)
biệt.
1. PHƯƠNG PHÁP
B1. Lập phương trình hoành độ giao điểm: f ( x) g ( x) (1)
B2. Lập luận
Lưu ý: Số nghiệm của phương trình (1) chính là số giao điểm của hai đồ thị.

NGUYỄN VĂN LỰC  0933.168.309 SP Toán K35 - ĐH Cần Thơ


Hàm số FB: http://www.facebook.com/VanLuc168
2. CÁC VÍ DỤ
2x  1
Ví dụ 1. Cho hàm số y  có đồ thị là (C). Tìm m để đường thẳng (d): y  x  m
x 1

cắt đồ thị (C) tại hai điểm phân biệt.


Bài giải
2x 1
♦ Phương trình hoành độ giao điểm: x m (1)
x 1

Điều kiện: x 1
♦ Khi đó: (1) 2x 1 ( x m)( x 1) x2 (m 1) x m 1 0 (2)
♦ (d) cắt (C) tại hai điểm phân biệt (1) có hai nghiệm phân biệt
(2) có hai nghiệm phân biệt khác 1
2
m 1 4 m 1 0
1 m 1 .1 m 1 0

m2 6m 5 0 m 1 m 5

♦ Vậy giá trị m cần tìm là m 1 m 5 .


Ví dụ 2. Cho hàm số y mx3 x 2 2 x 8m có đồ thị là Cm . Tìm m đồ thị Cm cắt trục
hoành tại 3 điểm phân biệt.
Bài giải
♦ Phương trình hoành độ giao điểm: mx3 x 2 2 x 8m 0 (1)
x 2 mx 2 (2m 1) x 4m 0

x 2
2
mx (2m 1) x 4m 0 (2)

♦ Cm cắt trục hoành tại 3 điểm phân biệt


(1) có ba nghiệm phân biệt (2) có hai nghiệm phân biệt khác 2

m 0 m 0 m 0
2
12m 4m 1 0 1 1 1 1
m m
12m 2 0 6 2 6 2
1
m
6

m 0
♦ Vậy giá trị m cần tìm là 1 1
.
m
6 2

NGUYỄN VĂN LỰC  0933.168.309 SP Toán K35 - ĐH Cần Thơ


Hàm số FB: http://www.facebook.com/VanLuc168
Ví dụ 3. Cho hàm số y x 4
(3m 4) x 2
m 2
có đồ thị là Cm . Tìm m đồ thị Cm cắt trục
hoành tại bốn điểm phân biệt.
Bài giải
♦ Phương trình hoành độ giao điểm: x 4 (3m 4) x 2 m 2 0 (1)
Đặt t x2 t 0 , phương trình (1) trở thành:
t2 (3m 4)t m2 0 (2)
♦ Cm cắt trục hoành tại bốn điểm phân biệt (1) có bốn nghiệm phân biệt
(2) có hai nghiệm dương phân biệt
5m 2 24m 16 0
2
P m 0
S 3m 4 0

4
m 4 m
5 4
m
m 0 5
4 m 0
m
3

4
m
♦ Vậy giá trị m cần tìm là 5 .
m 0

(C1 ) : y f ( x)
Dạng 3: Tìm tham số để hai đồ thị cắt nhau tại 2( 3, 4) điểm phân
(C2 ) : y g ( x)
biệt thỏa điều kiện cho trước.
1. PHƯƠNG PHÁP
B1. Lập phương trình hoành độ giao điểm: f ( x) g ( x) (1)
B2. Lập luận
Lưu ý:
Số nghiệm của phương trình (1) chính là số giao điểm của hai đồ thị.
Nghiệm x0 của phương trình (1) chính là hoành độ điểm chung của (C1) và (C2).
Khi đó tung độ điểm chung là y0 = f(x0) hoặc y0 = g(x0).

NGUYỄN VĂN LỰC  0933.168.309 SP Toán K35 - ĐH Cần Thơ


Hàm số FB: http://www.facebook.com/VanLuc168
2. CÁC VÍ DỤ
mx 1
Ví dụ 1. Cho hàm số y có đồ thị là Cm . Tìm m để đường thẳng (d): y 2 x 1
x 2

cắt đồ thị Cm tại hai điểm phân biệt A, B sao cho AB 10 .

Bài giải
mx 1
♦ Phương trình hoành độ giao điểm: 2x 1 (1)
x 2

Điều kiện: x 2

♦ Khi đó: (1) mx 1 (2 x 1)( x 2)

2 x2 (m 3) x 1 0 (2)
♦ (d) cắt Cm tại hai điểm phân biệt A, B (1) có hai nghiệm phân biệt
(2) có hai nghiệm phân biệt khác 2
2
m 3 8 0
8 2m 6 1 0

1
m (*)
2

Đặt A x1; 2 x1 1 ; B x2 ; 2 x2 1 với x1 , x2 là hai nghiệm của phương trình (2).


m 3
x1 x2
Theo định lý Viet ta có: 2
1
x1 x2
2

Khi đó: AB
2 2 2
x1 x2 4 x1 x2 10 5 x1 x2 4 x1 x2 10

2
m 3
2 2
2

m 3 [thỏa mãn (*)]


♦ Vậy giá trị m cần tìm là m 3 .

Ví dụ 2. Cho hàm số y x3 3x 2 (m 1) x 1 có đồ thị là Cm . Tìm m để đồ thị Cm cắt

đường thẳng (d ) : y x 1 tại ba điểm A 0;1 , B, C sao cho BC 10 .

Bài giải
♦ Phương trình hoành độ giao điểm: x3 3x 2 (m 1) x 1 x 1 (1)
x x2 3x m 2 0
NGUYỄN VĂN LỰC  0933.168.309 SP Toán K35 - ĐH Cần Thơ
Hàm số FB: http://www.facebook.com/VanLuc168
x 0
2
x 3x m 2 0 (2)

♦ Cm cắt trục hoành tại 3 điểm phân biệt (1) có ba nghiệm phân biệt
(2) có hai nghiệm phân biệt khác 0
17
9 4(m 2) 0 m
4 (*)
m 2 0
m 2

Đặt B x1; x1 1 ;C x2 ; x2 1 với x1 , x2 là hai nghiệm của phương trình (2).


x1 x2 3
Theo định lý Viet ta có:
x1 x2 m 2

Khi đó: BC
2 2 2
x1 x2 x1 x2 10 2 x1 x2 4 x1 x2 10

9 4 m 2 5

m 3 [thỏa mãn (*)]


♦ Vậy giá trị m cần tìm là m 3 .
Ví dụ 3. Cho hàm số y x4 (3m 4) x 2 m2 có đồ thị là Cm . Tìm m để đồ thị Cm cắt
trục hoành tại bốn điểm phân biệt có hoành độ lập thành một cấp số cộng.
Bài giải
♦ Phương trình hoành độ giao điểm: x 4 (3m 4) x 2 m 2 0 (1)
Đặt t x2 t 0 , phương trình (1) trở thành:
t2 (3m 4)t m2 0 (2)
♦ (C) cắt trục hoành tại bốn điểm phân biệt (1) có bốn nghiệm phân biệt
(2) có hai nghiệm dương phân biệt
5m 2 24m 16 0
P m2 0
S 3m 4 0

4
m 4 m
5 4
m
m 0 5 (*)
4 m 0
m
3

NGUYỄN VĂN LỰC  0933.168.309 SP Toán K35 - ĐH Cần Thơ


Hàm số FB: http://www.facebook.com/VanLuc168
Khi đó phương trình (2) có hai nghiệm 0 t 1 t2 . Suy ra phương trình (1) có
bốn nghiệm phân biệt là x1 t2 x2 t1 x3 t1 x4 t2

♦ Bốn nghiệm x1 , x2 , x3 , x4 lập thành cấp số cộng x2 x1 x3 x2 x4 x3

t1 t2 2 t1

t2 3 t1 t2 9t1 (3)
t1 t2 3m 4 (4)
Theo định lý Viet ta có: 2
t1t2 m (5)

3m 4
t1
10
Từ (3) và (4) ta suy ra được (6).
9(3m 4)
t2
10

Thay (6) vào (5) ta được:

3 3m 4 10m m 12
9
[thỏa mãn (*)]
2 2
3m 4 m 12
100 3 3m 4 10m m
19

m 12
♦ Vậy giá trị m cần tìm là 12 .
m
19

NGUYỄN VĂN LỰC  0933.168.309 SP Toán K35 - ĐH Cần Thơ


Hàm số FB: http://www.facebook.com/VanLuc168

BÀI TẬP TỰ LUYỆN

x2
Câu 1. Cho hàm số y  có đồ thị kí hiệu là (C ) .
x 1
a) Khảo sát và vẽ đồ thị (C ) của hàm số đã cho.
b) Tìm m để đường thẳng y  x  m cắt đồ thị (C ) tại hai điểm phân biệt A, B sao cho
AB  2 2.

Tìm m để đường thẳng y  x  m cắt đồ thị (C ) tại hai điểm phân biệt A, B sao cho
AB  2 2.
Phương trình hoành độ giao điểm của (C) và d: y=-x+m là:
x2 x  1 x  1
 x  m    2
x 1  x  2   x  mx  x  m  x  mx  m  2  0 (1)
2

d cắt (C) tại hai điểm phân biệt khi và chỉ khi (1) có hai nghiệm phân biệt khác 1
1  m  m  2  0
 2  m2  4m  8  0(*)
 m  4( m  2)  0
Khi đó d cắt (C) tại A( x1 ;  x1  m), B( x2 ;  x2  m) , với x1 , x2 là nghiệm phương trình (1).
Theo Viet, ta có AB   x2  x1 2   x1  x2 2  2 ( x1  x2 )2  4x1.x2   2  m2  4m  8
Yêu cầu bài toán tương đương với :
 m  2
2  m2  4m  8  2 2  m2  4m  12  0   (thỏa mãn (*)).
m  6
Vậy m  2 hoặc m  6.

x 1
Câu 2. Tìm m để đường thẳng  d  : y  x  m cắt đồ thị  C  của hàm số y  tại hai
x 1
điểm A, B sao cho AB  3 2

x 1
Pt hoành độ giao điểm  x  m  x  1   x  m  x  1 (vì x  1 không là nghiệm của pt)
x 1
 x2   m  2 x  m  1  0
(1)
Pt (1) có 2 nghiệm phân biệt x1 , x2    m2  8  0  m  .
 x1  x2  m  2
Khi đó A  x1; x1  m  , B  x2 ; x2  m  .Theo hệ thức Viet ta có 
 x1 x2  m  1
AB  3 2  AB 2  18  2  x1  x2   18   x1  x2   9 
2 2

  x1  x2   4 x1 x2  9   m  2   4  m  1  9  m  1
2 2

NGUYỄN VĂN LỰC  0933.168.309 SP Toán K35 - ĐH Cần Thơ


Hàm số FB: http://www.facebook.com/VanLuc168
x
Câu 3. Cho hàm số y  (1).
x 1
a) Khảo sát sự biến thiên và vẽ đồ thị (C) của hàm số (1).
b) Tìm m để đường thẳng y  x  m cắt đồ thị (C) tại hai điểm phân biệt A, B sao cho
tam giác IAB có diện tích bằng 3 , với I là giao điểm của hai tiệm cận.

Gọi d : y  x  m .
x
Phương trình hoành độ giao điểm của đường thẳng d và đồ thị (C) là:  xm
x 1
 x   x  1 x  m  (Vì x 1 không phải là nghiệm của phương trình)
 x2   m  2 x  m  0 (1)
Ta có   m 2  4  0, m nên đường thẳng d luôn cắt đồ thị ( C) tại hai điểm phân biệt A,
B với mọi m .
Khi đó, A  x1; x1  m  , B  x2 ; x2  m  , với x1 , x2 là hai nghiệm của phương trình (1).
m
Ta có: I 1;1  d  I , AB   .
2

và AB   x2  x1 2   x2  x1 2  2  x1  x2 2  8x1x2  2  m2  4 .

1 m m2  4
Ta có: S IAB  AB.d  I , AB   . Theo giả thiết, ta có:
2 2
m m2  4
S IAB  3   3m 2.
2

Câu 4. Cho hàm số y=x4-2x2-3.


a). Khảo sát sự biến thiên và vẽ đồ thị (C ) của hàm số.
b).Tìm tham số m đề đồ thị hàm số y=mx2-3 cắt đồ thị ( C) tại 3 điểm phân biệt và tạo
128
thành hình phẳng có diện tích bằng .
15

Ta có f1(x)=f2(x) <=>x4-(2+m)x2=0


Điều kiện để phương trình trên có 3 nghiệm phân biệt là 2+m>0 =>m>-2
Lúc đó ta có các nghiệm x=0; x=  2  m
0 2 m 2 m

diê ̣n tích S=  x  (2  m) x dx   x  (2  m) x dx  2  ( x 4  (2  m) x 2 )dx


4 2 4 2

 2 m 0 0
5
x 5 (2  m) x 3 2  m 2m ( 2  m )5
=2 (  )  2 2 4
5 3 0 15 15

( 2  m )5 128
Suy ra 4   ( 2  m )5  32  2  m  4  m  2(tm)
15 15

NGUYỄN VĂN LỰC  0933.168.309 SP Toán K35 - ĐH Cần Thơ


Hàm số FB: http://www.facebook.com/VanLuc168
2x  1
Câu 5. Cho hàm số: y  C 
x 1
a) Khảo sát và vẽ đồ thị hàm số (C)
b) Định m để đường thẳng (d): y = mx + 3 cắt đồ thị (C) tại 2 điểm A, B sao cho tam
giác OMN vuông tại O

Định m để đường thẳng d: y = mx + 3 cắt đồ thị (C) tại 2 điểm M, N sao cho  OMN
vuông tại O.
2x  1
Phương trình hoành độ giao điểm của (C) và d:  mx  3
x 1
 2x  1  (mx  3)(x  1)  mx 2  (1  m)x  4  0 (*)
(C) cắt d tại hai điểm phân biệt
 m  0  m  0
 2  m  7  4 3  
 m  14m  1  0  m  7  4 3
 m 1
 x1  x 2  m
Gọi x1, x2 là 2 nghiệm của phương trình (*)  
 x x  4
 1 2 m
Khi đó OM  (x1;mx1  3) , ON  (x 2 ;mx 2  3)
OMN vuông tại O nên OM.ON  0  (1  m2 )x1x 2  3m(x1  x 2 )  9  0
4(1  m2 ) 3m(m  1)  m  3  5 (n)
  9  0  m 2  6m  4  0 
m m  m  3  5 (n)

Câu 6. Cho hàm số y  x3  3mx 2  3(m2  1) x  m3  m (1). Tìm m để hàm số (1) có


cực trị đồng thời khoảng cách từ điểm cực đại của đồ thị hàm số đến gốc tọa độ O bằng
2 lần khoảng cách từ điểm cực tiểu của đồ thị hàm số đến gốc tọa độ O.

Ta có y  3x 2  6mx  3(m2  1)
Hàm số (1) có cực trị thì PT y  0 có 2 nghiệm phân biệt
 x2  2mx  m2  1  0 có 2 nhiệm phân biệt    1  0, m
Khi đó, điểm cực đại A(m  1;2  2m) và điểm cực tiểu B(m  1; 2  2m)
 m  3  2 2
Ta có OA  2OB  m2  6m  1  0   .
 m  3  2 2

Câu 7. Cho hàm số y  x3  3x2  4  C  .Gọi d là đường thẳng đi qua điểm A(- 1; 0)

với hệ số góc là k ( k thuộc R). Tìm k để đường thẳng d cắt (C) tại ba điểm phân biệt
và hai giao điểm B, C (B, C khác A ) cùng với gốc tọa độ O tạo thành một tam giác có
diện tích bằng 1.
NGUYỄN VĂN LỰC  0933.168.309 SP Toán K35 - ĐH Cần Thơ
Hàm số FB: http://www.facebook.com/VanLuc168

Đường thẳng d đi qua A(-1; 0) với hệ số góc là k, có phương trình là:


y = k(x+1) = kx+ k.
Nếu d cắt (C) tại ba điểm phân biệt thì phương trình: x 3 – 3x2 + 4 = kx + k
 x3 – 3x2 – kx + 4 – k = 0  (x + 1)( x2 – 4x + 4 – k ) = 0
 x  1
 có ba nghiệm phân biệt  g(x) = x2 – 4x + 4 – k = 0 có hai
 g ( x)  x  4 x  4  k  0
2

 '  0 k  0
nghiệm phân biệt khác - 1     0  k  9 (*)
 g (1)  0 9  k  0
Với điều kiện: (*) thì d cắt (C) tại ba điểm phân biệt A, B, C.Với A(-1;0), do đó B,C có
hoành độ là hai nghiệm của phương trình g(x) = 0.
Gọi B  x1; y1  ; C  x2 ; y2  với x1; x2 là hai nghiệm của phương trình: x2  4 x  4  k  0 . Còn
y1  kx1  k ; y2  kx2  k .

Ta có: BC   x2  x1; k  x2  x1    BC   x2  x1  1  k 2   x2  x1 1  k 
2 2

k
Khoảng cách từ O đến đường thẳng d: h 
1 k2
Vậy theo giả thiết:
1 1 k 1 1 1
S h.BC  . 2 k 1 k 2  2 k3  1  k3   k3   k  3
2 2 1 k2 2 4 4

2x  1
Câu 8. Cho hàm số y   C  Tìm tham số m để đường thẳng d: y = - 2x + m cắt đồ
x 1

thị tại hai điểm phân biệt A, B sao cho diện tích tam giác OAB bằng 3.

Xét phương trình hoành độ giao điể m của d và (C):


2x  1
 2 x  m ( x  1)  g ( x)  2 x 2  (m  4) x  1  m  0 (1)
x 1
D cắt (C) tại 2 điể m phân biệt  (1) có hai nghiệm phân biệt khác -
  (m  4) 2  8(1  m)  0 m 2  8  0
1.     m2  8  0  m  R .
 g (1)  0  g (1)  1  0
Chứng tỏ với mọi m d luôn cắt (C) tại hai điểm phân biệt A, B
Gọi A  x1; 2 x1  m  ; B  x2 ; 2 x2  m  . Với: x1 , x2 là hai nghiệm của phương trình (1)

 1

Ta có AB  x2  x1 ;2  x  x2   AB   x2  x1   4  x2  x1   x2  x1 5 .
2 2

Gọi H là hình chiếu vuông góc của O trên d, thì khoảng cách từ O đến d là h:
m m
h 
22  1 5

NGUYỄN VĂN LỰC  0933.168.309 SP Toán K35 - ĐH Cần Thơ


Hàm số FB: http://www.facebook.com/VanLuc168
1 1 x2  x1 1  1
Theo giả thiết: S  AB.h  5 .  . m2  8  3
2 2 5 2 2 4
Vậy: m2  8  42.3  m2  8  42.3  m2  40  m  2 10 (*)
Với m thỏa mãn điều kiện (*) thì d cắt (C) tại A, B thỏa mãn yêu cầu bài toán.

Câu 9. Cho hàm số y  x3  2mx 2   m  3 x  4 (1). Tìm m để đường thẳng d: y = x + 4


cắt đồ thị hàm số (1) tại ba điểm phân biệt A, B, C sao cho tam giác MBC có diện tích
bằng 4. (Điểm B, C có hoành độ khác không ; M(1;3) ).

Đồ thị (1) cắt d tại ba điểm A, B, C có hoành độ là nghiệm của phương trình:
x  0
 x3  2mx 2   m  3 x  4  x  4;  x  x 2  2mx  m  2   0   2
 x  2mx  m  2  0
  '  m 2  m  2  0  m  1  m  2 (*)
Với m thỏa mãn (*) thì d cắt (1) tại ba điểm A(0; 4), còn hai điểm B,C có hoành độ là
hai nghiệm của phương trình:
 '  m2  m  2  0
 x  2mx  m  2  0  
2
 m  1  m  2; m  2
m  2  0
- Ta có B  x1; x1  4  ; C  x2 ; x2  4   BC   x2  x1; x2  x1 
 BC   x2  x1    x2  x1   x2  x1 2
2 2

-Gọi H là hình chiếu vuông góc của M trên d. h là khoảng cách từ M đến d thì:
1 3  4 1 1
h  2S  BC.h  x2  x1 2. 2  x2  x1
2 2 2
- Theo giả thiết: S = 4  x2  x1  4;  2  '  4;  m2  m  2  4  m2  m  6  0
Kết luận: với m thỏa mãn: m  2  m  3  m  3 (chọn).

NGUYỄN VĂN LỰC  0933.168.309 SP Toán K35 - ĐH Cần Thơ


Hàm số FB: http://www.facebook.com/VanLuc168

V. TIẾP TUYẾN CỦA ĐỒ THỊ HÀM SỐ


Chuyên đề: Hàm số

A. Tóm tắt lí thuyết & phương pháp giải toán


I. KIẾN THỨC CƠ BẢN

1. Dạng 1: Viết phương trình tiếp tuyến với đồ thị (C):y = f(x) tại điểm
M 0 (x 0 ; y 0 )  (C)
y (C): y=f(x)

y0 M 0 

x
x0
Phương pháp:

Phương trình tiếp tuyến với (C) tại M(x0;y0) có dạng:

y - y0 = k ( x - x0 ) hay y  f '(x 0 )(x  x 0 )  f(x 0 )


Trong đó: x0: hoành độ tiếp điểm
y0: tung độ tiếp điểm và y0 = f(x0)
k: hệ số góc của tiếp tuyến và được tính bởi công thức : k = f'(x0)

2x 3
Ví dụ: Cho hàm số y có đồ thị là C . Viết phương trình tiếp tuyến của C
x 1
tại các giao điểm của C và đường thẳng y x 3.
Bài giải
2x 3
♦ Phương trình hoành độ giao điểm: x 3 (1)
x 1

Điều kiện: x 1
x 0
Khi đó: (1) 2x 3 ( x 3)( x 1) x2 2x 0
x 2

Suy ra tọa độ các giao điểm là A 0; 3 , B 2; 1


1
♦ Ta có: y ' 2
x 1

♣ Phương trình tiếp tuyến tại A là y 3 y '(0)( x 0) y x 3


♣ Phương trình tiếp tuyến tại B là y 1 y '(2)( x 2) y x 1
♦ Vậy có hai tiếp tuyến thỏa đề bài là y x 3 và y x 1 .

NGUYỄN VĂN LỰC  0933.168.309 SP Toán K35 - ĐH Cần Thơ


Hàm số FB: http://www.facebook.com/VanLuc168

BÀI TẬP TỰ LUYỆN

Câu 1. Viết phương trình tiếp tuyến của đồ thị hàm số y  x3  3x 2  2 tại điểm
M(–1;–2)

(C) : y  x 3  3x 2  2  y  3x 2  6 x tại điểm M(–1; –2) ta có: y (1)  9


 PTTT: y  9 x  7

x 1
Câu 2: Cho hàm số y  có đồ thị (H). Viết phương trình tiếp tuyến của (H) tại
x 1
A(2; 3).

x 1 2
y  y 
x 1 ( x  1)2
Tại A(2; 3)  k  y (2)  2  PTTT : y  2 x  1

Câu 3: Cho hàm số f ( x)  x3  3x  4 . Lập phương trình tiếp tuyến của đồ thị hàm số
tại điểm M(1; 2).

f ( x )  x 3  3x  4  f ( x )  3x 2  3  f (1)  0  PTTT: y  2 .

3x  1
Câu 4: Cho hàm số y  có đồ thị (C). Viết phương trình tiếp tuyến của (C) tại
1 x
điểm A(2; –7).

3x  1 4
y  y  , k  y (2)  4
1 x ( x  1)2
 PTTT: y  4 x  15

2  x  x2
Câu 5: Cho hàm số y  có đồ thị (C). Viết phương trình tiếp tuyến của (C)
x 1
tại điểm M(2; 4).

2  x  x2 x2  2x 1
y  y'   k  f (2)  1
x 1 ( x  1)2
x0  2, y0  4, k  1  PTTT : y   x  2

NGUYỄN VĂN LỰC  0933.168.309 SP Toán K35 - ĐH Cần Thơ


Hàm số FB: http://www.facebook.com/VanLuc168
3 2
Câu 6: Cho hàm số y  x  3x có đồ thị (C). Viết phương trình tiếp tuyến của (C) tại
điểm I(1; –2).

y  x 3  3x 2  y '  3x 2  6 x  k  f (1)  3
x0  1, y0  2, k  3  PTTT : y  3x  1

Câu 7. Cho hàm số y  x 4  x 2  3 có đồ thị (C). Viết phương trình tiếp tuyến của (C)
tại điểm có hoành độ bằng 1.

x0  1  y0  3
y  4 x 3  2 x  k  y (1)  2
Phương trình tiếp tuyến là y = 2x + 1

Câu 8. Cho hàm số: y  2 x3  7x  1 (C). Viết phương trình tiếp tuyến của đồ thị (C)
tại điểm có hoành độ x = 2.

y  2 x3  7x  1  y '  6 x 2  7
Với x  2  y  3, y (2)  17  PTTT : y  17x  31
0 0

Câu 9. Cho (C): y  x3  3x 2  2 . Viết phương trình tiếp tuyến của (C) tại các giao
điểm của (C) với trục hoành.

Cho (C): y  x3  3x 2  2 .
y  3x 2  6 x . Giao của ( C) với trục Ox là A(1; 0), B 1  3; 0  , C 1  3; 0 
Tiếp tuyến tại A(1; 0) có hệ số góc là k = –3 nên PTTT: y  3 x  3
Tiếp tuyến tại B 1  3; 0  có hệ số góc là k = 6 nên PTTT : y  6 x  6  6 3
Tiếp tuyến tại C 1  3; 0  có hệ số góc là k = 6 nên PTTT : y  6 x  6  6 3

1
Câu 10: Viết phương trình tiếp tuyến của đồ thị hàm số yx tại giao điểm của
x
nó với trục hoành .

1 1
y x  y  1 
x x2
 Các giao điểm của đồ thị hàm số với trục hoành là A  1; 0  , B 1; 0 
 Tại A(–1; 0) tiếp tuyến có hệ số góc k1  2 nên PTTT: y = 2x +2
 Tại B(1; 0) tiếp tuyến cũng có hệ số góc k2  2 nên PTTT: y = 2x – 2

NGUYỄN VĂN LỰC  0933.168.309 SP Toán K35 - ĐH Cần Thơ


Hàm số FB: http://www.facebook.com/VanLuc168
2x 1
Câu 11. Cho hàm số: y . Viết phương trình tiếp tuyến của (C ) tại điểm trên
x 1
(C ) có tung độ bằng 5.

2x 0 1
Ta có: y0 5 5 2x 0 1 5x 0 5 x0 2
x0 1
3
 f (x 0 ) 3
(2 1)2
 Phương trình tiếp tuyến cần tìm: y 5 3(x 2) y 3x 11

2x 1
Câu 12. Cho hàm số y . Viết phương trình tiếp tuyến của đồ thị (C) biết
x 1
tiếp điểm có tung độ bằng 3.

2x 0  1
Go ̣i tiế p điể m là M ( x0 ; y0 ) , ta có y0  3   3  x 0  2  M (2; 3)
x0  1
Suy ra, hê ̣ số góc k của tiế p tuyế n là: k  y '(2)  1
Do đó phương trình tiế p tuyến cầ n lâ ̣p là: y  1(x  2)  3 hay y  x  5

Câu 13. Cho hàm số y   x 3 +3x 2 1 . Lập phương trình tiếp tuyến của (C) tại các giao
điểm của đồ thị với trục hoành.

Đồ thị cắt trục hoành tại các điểm A(0;0) và B(3;0).


Phương trình tiếp tuyến của đồ thị tại A(0;0) là: y  0
Phương trình tiếp tuyến của đồ thị tại B(3;0) là: y  y , 3x  3  9 x  27
Vậy tiếp tuyến cần tìm là y  0 và y  9 x  27 .

Câu 14. Cho hàm số y  x3  3x 2  2 (C ) . Gọi giao điểm của đồ thị (C ) và đường thẳng
y   x  3 là M , viết phương trình tiếp tuyến với đồ thị (C ) tại điểm M.

 y  x3  3x 2  2
Tọa độ của M là nghiệm của hệ 
 y  x  3
 y  x  3  y  x  3
 3   M (1; 2)
 x  3x  x  5  0  x  1
2

Phương trình tiếp tuyến với (C) tại M là y  f '(1)( x  1)  2  y  9( x  1)  2  y  9 x  7.

NGUYỄN VĂN LỰC  0933.168.309 SP Toán K35 - ĐH Cần Thơ


Hàm số FB: http://www.facebook.com/VanLuc168

2. Dạng 2:
Viết phương trình tiếp tuyến với đồ thị (C): y = f(x) biết tiếp tuyến có hệ
số góc k cho trước

y (C): y = f(x)

y0 M 0 

x
x0

Phương pháp: Ta có thể tiến hành theo các bước sau

Bước 1: Gọi M ( x0 ; y0 )  (C ) là tiếp điểm của tiếp tuyến với (C)


Bước 2: Tìm x0 bằng cách giải phương trình : f ' ( x0 )  k , từ đó suy ra y0  f ( x0 ) =?
Bước 3: Thay các yếu tố tìm được vào pt: y - y0 = k ( x - x0 ) ta sẽ được pttt cần tìm.

2x 1
Ví dụ: Cho hàm số y có đồ thị là C . Viết phương trình tiếp tuyến của C ,
x 2
biết hệ số góc của tiếp tuyến bằng 5 .
Bài giải
♦ Gọi M ( x0 ; y0 )  (C ) là tiếp điểm của tiếp tuyến với (C)
5
♦ Ta có: y ' 2
x 2
Hệ số góc của tiếp tuyến bằng 5 y '( x0 ) 5
5
2
5
x0 2
x0 1
x0 3
♣ Với x0 1 y0 3 : M 1 (1; 3) pttt: y 5x 2

♣ Với x0 3 y0 7 : M 2 (3; 7) pttt: y 5 x 22

♦ Vậy có hai tiếp tuyến thỏa đề bài là y 5x 2 và y 5 x 22 .

Bài tập tương tự


2x 1
Cho hàm số y có đồ thị là C . Viết phương trình tiếp tuyến của C , biết hệ
x 1
số góc của tiếp tuyến bằng 4 .
Đáp số: y 4 x 2; y 4 x 10

NGUYỄN VĂN LỰC  0933.168.309 SP Toán K35 - ĐH Cần Thơ


Hàm số FB: http://www.facebook.com/VanLuc168
Chú ý : Đối với dạng 2 người ta có thể cho hệ số góc k dưới dạng gián tiếp như :
tiếp tuyến song songtiếp tuyến vuông góc với một đường thẳng cho trước .
y (C): y=f(x)
y
(C): y=f(x) ka 
y  ax  b
x x
1 O
2 k  1 / a

 2 : y  ax  b

Khi đó ta cần phải sử dụng các kiến thức sau:

Định lý 1: Nếu đường thẳng (  ) có phương trình dạng : y = ax + b thì hệ số


góc của (  ) là:
k  a

Định lý 2: Trong mp(Oxy) cho hai đường thẳng (1 ) vaø ( 2 ) . Khi đó:

1 // 2  k 1  k 2
1  2  k 1 .k 2  1

Ví dụ 1: Cho hàm số y x3 3x 2 2 có đồ thị là C . Viết phương trình tiếp tuyến


của C , biết tiếp tuyến song song với đường thẳng ( ) : y 9 x 2 .

Bài giải
♦ Ta có: y ' 3x 2 6 x
♦ Do tiếp tuyến song song với đường thẳng ( ) nên hệ số góc của tiếp tuyến là
k 9

♦ Gọi M ( x0 ; y0 )  (C ) là tiếp điểm của tiếp tuyến với (C)


Hệ số góc của tiếp tuyến k 9 y '( x0 ) 9

x0 1
3x02 6 x0 9 0
x0 3

♣ Với x0 1 y0 2 : M 1 ( 1; 2) pttt: y 9 x 7
♣ Với x0 3 y0 2 : M 2 (3; 2) pttt: y 9 x 25
♦ Vậy có hai tiếp tuyến thỏa đề bài là y 9 x 7 và y 9 x 25 .

NGUYỄN VĂN LỰC  0933.168.309 SP Toán K35 - ĐH Cần Thơ


Hàm số FB: http://www.facebook.com/VanLuc168

Bài tập tương tự


Cho hàm số y x3 3x 2 3x có đồ thị là C . Viết phương trình tiếp tuyến của C ,
biết tiếp tuyến song song với đường thẳng ( ) : y 3 x .
Đáp số: y 3x 4

x 2
Ví dụ 2: Cho hàm số y có đồ thị là C . Viết phương trình tiếp tuyến của C ,
x 2
biết tiếp tuyến vuông góc với đường thẳng ( ) : y x 2.
Bài giải
4
♦ Ta có: y ' 2
x 2

♦ Do tiếp tuyến vuông góc với đường thẳng ( ) nên hệ số góc của tiếp tuyến
là k 1
♦ Gọi M ( x0 ; y0 )  (C ) là tiếp điểm của tiếp tuyến với (C)
Hệ số góc của tiếp tuyến k 1 y '( x0 ) 1

4
2
1
x0 2
2
x0 2 4

x0 2 2 x0 0
x0 2 2 x0 4

♣ Với x0 0 y0 1 : M 1 (0; 1) pttt: y x 1

♣ Với x0 4 y0 3 : M 2 ( 4;3) pttt: y x 7

♦ Vậy có hai tiếp tuyến thỏa đề bài là y x 1 và y x 7 .

Bài tập tương tự


3 2x
Cho hàm số y có đồ thị là C . Viết phương trình tiếp tuyến của C , biết
x 1
tiếp tuyến vuông góc với đường thẳng ( ) : x y 1 0 .
Đáp số: y x 1; y x 3.

NGUYỄN VĂN LỰC  0933.168.309 SP Toán K35 - ĐH Cần Thơ


Hàm số FB: http://www.facebook.com/VanLuc168

BÀI TẬP TỰ LUYỆN

Câu 1. Cho hàm số y  x3  3x2 (C). Viết phương trình tiếp tuyến của đồ thị (C) biết
hệ số góc của tiếp tuyến k =3.

Ta có: y '  3x2  6 x


Gọi M ( x0 ; y0 ) là tiếp điể m  Tiếp tuyến tại M có hệ số góc k  f ' ( x0 )  3x02  6 x0
Theo giả thiết, hệ số góc của tiếp tuyến k = - 3 nên:
3x02  6 x0  3  x02  2 x0  1  0  x0  1
Vì x0  1  y0  2  M (1; 2) .
Phương trình tiếp tuyến cần tìm là y  3( x  1)  2  y  3x  1

Câu 2. Cho hàm số: y  2 x3  7x  1 (C). Viết phương trình tiếp tuyến của đồ thị (C)
có hệ số góc k = –1.

y  2 x3  7x  1  y '  6 x 2  7
 x  1
Gọi ( x0 ; y0 ) là toạ độ của tiếp điểm. Ta có: y ( x0 )  1  6 x02  7  1   0
 x0  1
 Với x0  1  y0  6  PTTT : y   x  7
 Với x0  1  y0  4  PTTT : y   x  5

Tiếp tuyến song song với đường thẳng d

x 1
Câu 3. Cho hàm số y  .Viết phương trình tiếp tuyến của đồ thị hàm số biết tiếp
x 1
x 2
tuyến song song với d: y .
2

x 1 2
y  y  ( x  1)
x 1 ( x  1)2

x 2 1 1
d: y  có hệ số góc k   TT có hệ số góc k  .
2 2 2
1 2 1 x  1
Gọi ( x0 ; y0 ) là toạ độ của tiếp điểm. Ta có y ( x 0 )      0
2 ( x0  1)2 2  x0  3
1 1
+ Với x0  1  y0  0  PTTT: y  x  .
2 2

NGUYỄN VĂN LỰC  0933.168.309 SP Toán K35 - ĐH Cần Thơ


Hàm số FB: http://www.facebook.com/VanLuc168
1 7
+ Với x0  3  y0  2  PTTT: y  x  .
2 2

x 2  3x  2
Câu 4. Cho hàm số f ( x )  (1). Viết phương trình tiếp tuyến của đồ thị hàm
x 1
số (1), biết tiếp tuyến đó song song với đường thẳng d: y  5 x  2 .

x 2  3x  2 x2  2x  5
f (x)   f  (x) 
x 1 ( x  1)2
Tiếp tuyến song song với d: y  5 x  2 nên tiếp tuyến có hệ số góc k  5 .
x02  2 x0  5
Gọi ( x0 ; y0 ) là toạ độ của tiếp điểm. Ta có: f ( x0 )  5   5
( x0  1)2
x  0
  0
 x0  2
 Với x0  0  y0  2  PTTT: y  5 x  2
 Với x0  2  y0  12  PTTT: y  5 x  22

Câu 5. Cho hàm số f(x) = -x3 + 3x + 1 (có đồ thị (C)). Lập phương trình tiếp tuyến
của đồ thị (C) biết tiếp tuyến song song với đường thẳng d: y = -9x -15.

Tiếp tuyến // d: y = -9x -15 nên phương trình tiếp tuyến có dạng
y = -9x + m, m  -15.
 x 3  3x  1  9 x  m (1)
Điều kiện tiếp xúc: hệ  có nghiệm.
 3x 2  3  9 (2)
 x  2  m  15
(2)    m  17
 x  2  m  17
Vậy phương trình tiếp tuyến là: y = -9x +17.

Câu 6. Cho hàm số y  x 2 ( x  1) có đồ thị (C). Viết phương trình tiếp tuyến của đồ thị
(C), biết tiếp tuyến song song với đường thẳng d: y  5 x .

Vì tiếp tuyến song song với d: y  5 x nên tiếp tuyến có hệ số góc là k = 5


Gọi ( x0 ; y0 ) là toạ độ của tiếp điểm.
 x0  1
y '( x0 )  5  3x  2 x0  5  3 x  2 x0  5  0  
2 2
0 0 x   5
 0 3
Với x0  1  y0  2  PTTT: y  5x  3
5 50 175
Với x0    y0    PTTT: y  5 x 
3 27 27

NGUYỄN VĂN LỰC  0933.168.309 SP Toán K35 - ĐH Cần Thơ


Hàm số FB: http://www.facebook.com/VanLuc168
x 1
Câu 7. Cho hàm số y  có đồ thị (H). Viết phương trình tiếp tuyến của (H) biết
x 1
1
tiếp tuyến song song với đường thẳng y   x  5 .
8

x 1 2
y  y 
x 1 ( x  1)2
1
Vì tiếp tuyến song song với đường thằng y   x  5 nên hệ số góc của tiếp tuyến là
8
1
k
8
Gọi ( x 0 ; y0 ) là toạ độ của tiếp điểm 
2 1  x  3
y ( x0 )  k      ( x0  1)2  16   0
( x0  1) 2 8  x0  5
1 1 1
 Với x0  3  y0   PTTT : y  
2 8
 x  3 
2
3 1 3
 Với x0  5  y0   PTTT : y    x  5 
2 8 2

1
Câu 8. Viết phương trình tiếp tuyến của đồ thị hàm số y  biết tiếp tuyến song song
x
với đường thẳng y   4 x  3 .

1 1
y  y   2 ( x  0)
x x
Vì tiếp tuyến song song với đường thẳng y  4 x  3 nên tiếp tuyến có hệ số góc k=–4
 1
 x0  2
1
Gọi ( x0 ; y0 ) là toạ độ của tiếp  y ( x0 )  4   2  4  
x0 x   1
 0 2
1
 Với x0   y0  2  PTTT : y  4 x  4
2
1
 Với x0    y0  2  PTTT : y  4 x  4
2

Câu 9. Cho hàm số: y  x3  3x 2  2 x  2 . Viết phương trình tiếp tuyến của đồ thị hàm
số, biết tiếp tuyến đó song song với đường thẳng d: x  y  50  0 .

y  x 3  3 x 2  2 x  2  y  3 x 2  6 x  2
Vì tiếp tuyến song song với đường thẳng d: x  y  50  0 nên tiếp tuyến có hệ số
góc k = –1.
Gọi ( x 0 ; y0 ) là toạ độ của tiếp điểm. Ta có:
3 x02  6 x0  2  1  x02  2 x0  1  0  x0  1

NGUYỄN VĂN LỰC  0933.168.309 SP Toán K35 - ĐH Cần Thơ


Hàm số FB: http://www.facebook.com/VanLuc168
Khi đó y0  2  phương trình tiếp tuyến là y  ( x  1)  2  y   x  3 .

Câu 10. Cho hàm số y x3 3x 2 2 có đồ thị là C . Viết phương trình tiếp tuyến của
C , biết tiếp tuyến song song với đường thẳng ( ) : y 9 x 2 .

Ta có: y ' 3x 2 6 x
Do tiếp tuyến song song với đường thẳng ( ) nên hệ số góc của tiếp tuyến là k 9

Gọi M ( x0 ; y0 )  (C ) là tiếp điểm của tiếp tuyến với (C)


Hệ số góc của tiếp tuyến k 9 y '( x0 ) 9

3x02 6 x0 9 0

x0 1
x0 3

Với x0 1 y0 2 : M 1 ( 1; 2) pttt: y 9 x 7
Với x0 3 y0 2 : M 2 (3; 2) pttt: y 9 x 25
Vậy có hai tiếp tuyến thỏa đề Câu là y 9 x 7 và y 9 x 25 .

x 2
Câu 11. Cho hàm số y có đồ thị là C . Viết phương trình tiếp tuyến của C ,
x 2
biết tiếp tuyến vuông góc với đường thẳng ( ) : y x 2.

4
Ta có: y ' 2
x 2

Do tiếp tuyến vuông góc với đường thẳng ( ) nên hệ số góc của tiếp tuyến là k 1

Gọi M ( x0 ; y0 )  (C ) là tiếp điểm của tiếp tuyến với (C)


Hệ số góc của tiếp tuyến k 1 y '( x0 ) 1

4 2
2
1 x0 2 4
x0 2

x0 2 2 x0 0
x0 2 2 x0 4

Với x0 0 y0 1 : M 1 (0; 1) pttt: y x 1

Với x0 4 y0 3 : M 2 ( 4;3) pttt: y x 7

Vậy có hai tiếp tuyến thỏa đề Câu là y x 1 và y x 7.

NGUYỄN VĂN LỰC  0933.168.309 SP Toán K35 - ĐH Cần Thơ


Hàm số FB: http://www.facebook.com/VanLuc168
Câu 12. Viết phương trình tiếp tuyến của đồ thị hàm số y  x3  3x2  1 (C). Biết tiếp
tuyến đó song song với đường thẳng y = 9x + 6.

Ta có: y '  3x2  6 x


Gọi M ( x0 ; y0 ) là tiếp điểm  Tiếp tuyến tại M có hệ số góc k  f ' ( x0 )  3x02  6 x0
Theo giả thiết, tiếp tuyến đó song song với đường thẳng y = 9x + +6  tiếp tuyến
 x0  1  M (1; 3)
có hệ số góc k = 9  3x02  6 x0  9  x02  2 x0  3  0  
 x0  3  M (3;1)
Phương trình tiếp tuyến của (C) tại M(-1;-3) là: y  9( x  1)  3  y  9 x  6 (loại)
Phương trình tiếp tuyến của (C) tại M(3;1) là: y  9( x  3)  1  y  9 x  26

Câu 13. Cho hàm số y  x3  3x  2 (C). Viết phương trình tiếp tuyến của (C) biết tiếp
1
tuyến đó vuông góc với đường thẳng y  x.
9

Ta có y '  3x2  3 . Do tiếp tuyến của (C) biết tiếp tuyến đó vuông góc với đường
1
thẳng y  x nên hệ số góc của tiếp tuyến k = 9.
9
Do đó y '  k  3x 2  3  9  x 2  4  x  2.
+) Với x = 2  y  4 . Pttt tại điểm có hoành độ x = 2 là:
y  9( x  2)  4  y  9 x  14.
+) Với x  2  y  0 . Pttt tại điểm có hoành độ x = - 2 là:
y  9( x  2)  0  y  9 x  18 .
1
Vậy có hai tiếp tuyến củả (C) vuông góc với đường thẳng y  x là:
9
y =9x - 14 và y = 9x + 18.

Tiếp tuyến vuông góc với đường thẳng d

Câu 14. Cho hàm số y  x 4  x 2  3 (C). Viết phương trình tiếp tuyến của (C) vuông
góc với d: x  2 y  3  0 .

1
d: x  2 y  3  0 có hệ số góc kd    Tiếp tuyến có hệ số góc k 2.
2
Gọi ( x0 ; y0 ) là toạ độ của tiếp điểm. Ta có: y ( x0 )  2  4 x03  2 x0  2  x0  1
( y0  3 )
 PTTT: y  2( x  1)  3  y  2 x  1 .

NGUYỄN VĂN LỰC  0933.168.309 SP Toán K35 - ĐH Cần Thơ


Hàm số FB: http://www.facebook.com/VanLuc168

Câu 15. Viết phương trình tiếp tuyến của đồ thị hàm số y  x3  3x 2  2 vuông góc với
1
đường thẳng d: y   x  2 .
9

(C) : y  x 3  3x 2  2  y  3x 2  6 x
1
Tiếp tuyến vuông góc với d: y   x  2  Tiếp tuyến có hệ số góc k 9.
9
Gọi ( x0 ; y0 ) là toạ độ của tiếp điểm.
 x0  1
Ta có: y ( x0 )  9  3x02  6 x0  9  x02  2 x0  3  0  
 x0  3
 Với x0  1  y0  2  PTTT: y  9 x  7
 Với x0  3  y0  2  PTTT: y  9 x  25

Câu 16. Cho đường cong (C): y  x3  3x 2  2 . Viết phương trình tiếp tuyến của (C),
1
biết tiếp tuyến vuông góc đường thẳng y   x  1 .
3

y  x 3  3x 2  2  y '  3x 2  6 x
1
Vì tiếp tuyến vuông góc với đường thẳng y   x  1 nên tiếp tuyến có hệ số góc là k
3
= 3.
x  1 2
Gọi ( x0 ; y0 ) là toạ độ của tiếp điểm  3 x02  6 x0  3  x02  2 x0  1  0   0
 x0  1  2
 Với x0  1  2  y0  2  PTTT: y  3  x  1  2   2  y  3x  4 2  3
 Với x0  1  2  y0   2  PTTT: y  3  x  1  2   2  y  3x  4 2  3

x2  x  1
Câu 17. Cho hàm số y . Viết phương trình tiếp tuyến của đồ thị hàm số
x 1
4 1
biết tiếp tuyến vuông góc với đường thẳng y   x 
3 3

TXĐ: R \ 1
x 2  2x
Có f '( x) 
( x  1) 2
Phương trình tiếp tuyến của đồ thị hàm số tại M ( x0 ; f ( x0 )) là:
y  f ' ( x0 )( x  x0 )  f ( x0 )
4 1 4
Đường thẳng y   x  có hệ số góc k= 
3 3 3
4 1  4
Vì tiếp tuyến vuông góc với đường thẳng y  x  nên f '( x0 ).     1
3 3  3

NGUYỄN VĂN LỰC  0933.168.309 SP Toán K35 - ĐH Cần Thơ


Hàm số FB: http://www.facebook.com/VanLuc168
x  2x 0 3
2
 x0  1
 0   4 x02  8 x0  3x02  6 x0  3  x02  2 x0  3  0  
( x0  1)  x0  3
2
4
3 3 3
Với x0  1 ta có f (1)   tiếp tuyến y  x 
2 4 4
7 3 5
Với x0  3 ta có f (3)   tiếp tuyến y  x 
2 4 4
3 3 3 5
Vậy có hai tiếp tuyến thỏa mãn đề Câu: y  x  và y  x 
4 4 4 4

1
Câu 18. Cho hàm số y  x3  2 x 2  3x . Lập phương trình đường thẳng đi qua điểm
3
cực đại của đồ thị (C) và vuông góc với tiếp tuyến của đồ thị (C) tại gốc tọa độ.

+ Điểm Cực đại của ( C ) là M(1;4/3)


+T.T của ( C ) tại gốc toạ độ có hệ số góc k= y’(0)=3
+Đường thẳng cần tìm đi qua điểm M và có hệ số góc k’= -1/3 nên có pt:
y= - 1/3(x-1)+4/3=-1/3x+5/3

Phương trình tiếp tuyến dạng đặc biệt

Câu 19. Cho hàm số : y   x3  6 x 2  9 x . Tìm trên trục hoành những điểm mà từ đó kẽ
được các tiếp tuyến với (C), sao cho trong đó có hai tiếp tuyến vuông góc nhau .

M  a;0  là điểm cần tìm.Tiếp tuyến của (C) kẽ từ M là đường thẳng


 t  : y  k  x  a  …. k thỏa:
 x 3  6 x 2  9 x  k  x  a   x 3  6 x 2  9 x   3 x 2  12 x  9   x  a  1
 
3 x  12 x  9  k
2
3 x  12 x  9  k
2
 2
x  3  0
1   x  3  2 x 2  3ax  3a   0  
 2 x  3ax  3a  0 *
2

Lập luận đi đến (*) có hai nghiệm phân biệt x1 , x2 : k x1  k x2   1


9a 2  24a  0
Vậy M   ;0 
82 82
...   a
27a  81  1 27  27 

Câu 20. Cho hàm số mx  1 ,


y  Cm  . Gọi I là giao điểm hai đường tiệm cận của
xm
đồ thị Cm  . Tiếp tuyến tại điểm bất kì của Cm  cắt tiệm cận đứng và tiệm cận
ngang lần lượt tại A và B . Tìm m để diện tích tam giác IAB bằng 12 .

Với mọi m , đồ thị hàm số có tiệm cận đứng x  m , tiệm cận ngang y  m ,
I  m; m  .
 m2  1 
Giả sử M  x0 ; m     Cm  , phương trình tiếp tuyến tại M của Cm  :
 xm 
NGUYỄN VĂN LỰC  0933.168.309 SP Toán K35 - ĐH Cần Thơ
Hàm số FB: http://www.facebook.com/VanLuc168
m 1
2
m 1
2

2 
y x  x0   m  ,  x0   m  .
 x0  m  x0  m

Tìm được

A  m; m 
2m 2  2 
, B  2 x0  m; m  , từ đó suy ra IA  2
m2  1
,
 x0  m  x0  m
IB  2 x0  m .
1
S IAB  IA. IB  2 m 2  1  12  m   5 .
2

2x  1
Câu 21. Cho hàm số y  . Viết phương trình tiếp tuyến của (C), biết khoảng
x 1
cách từ điểm I(1;2) đến tiếp tuyến bằng 2 .

*Tiếp tuyến của (C) tại điểm M (x 0 ; f (x 0 ))  (C ) có phương trình


y  f '(x 0 )(x  x 0 )  f (x 0 )
Hay x  (x 0  1)2 y  2x 02  2x 0  1  0 (*)
*Khoảng cách từ điểm I(1;2) đến tiếp tuyến (*) bằng 2
2  2x 0
  2
1  (x 0  1) 4
giải được nghiệm x 0  0 và x 0  2

*Các tiếp tuyến cần tìm : x  y  1  0 và x  y  5  0

x2
Câu 22. Cho hàm số y  (C) Cho điểm A(0;a). Tìm a để từ A kẻ được hai tiếp
x 1
tuyến tới (C) sao cho hai tiếp điểm tương ứng nằm về hai phía trục hoành.

Đk: ,
PT đường thẳng d qua A và có hsg k có dạng:

d tiếp xúc với (C) hệ pt sau có nghiệm

Thay (2) vào (1) ta được:

Đặt
Để qua A kẻ được 2 tiếp tuyến có 2 nghiệm phân biệt 1

Theo viet ta có: và

Để 2 tiếp điểm nằm về 2 phía của trục hoành

NGUYỄN VĂN LỰC  0933.168.309 SP Toán K35 - ĐH Cần Thơ


Hàm số FB: http://www.facebook.com/VanLuc168

Từ (*)
Kết hợp với điều kiện (1) ta được: Với thỏa mãn bài toán

Câu 23. Cho hàm số y  x3  6 x 2  9 x  2 (1) có đồ thị (C). Chứng minh rằng trên (C)
không thể tồn tại hai điểm có hoành lớn hơn 3 sao cho hai tiếp tuyến với (C) tại hai
điểm đó vuông góc với nhau

Giả sử trên (C) có hai điểm A( x1 ; y1 ), B( x2 ; y2 ) với x1, x2 > 3 sao cho tiếp tuyến với (C)
tại hai điểm này vuông góc với nhau
Khi đó, ta có: y '( x1 ). y '( x2 )  1  (3x12 12 x1  9)(3x22 12 x2  9)  1
 9  x1  1 x1  3 x2  1 x2  3  1 (*)
Do x1 > 3 và x2 > 3 nên VT(*) > 0. Do đó (*) vô lí
Vậy: Trên (C) không thể có hai điểm sao cho tiếp tuyến với (C) tại hai điểm này
vuông góc với nhau

x2
Câu 24. Cho hàm số y  (C). Viết phương trình tiếp tuyến với (C) biết rằng
2x  3
tiếp tuyến cắt trục hoành tại A, trục tung tại B sao cho tam giác OAB vuông cân tại
O, ở đây O là góc tọa độ.

1
Ta có: y ' 
(2 x  3) 2
Vì tiếp tuyến tạo với hai trục tọa độ một tam giác vuông cân nên hệ số góc của tiếp
tuyến là: k  1
Khi đó gọi M  x0 ; y0  là tiếp điểm của tiếp tuyến với đồ thị (C) ta có y ' ( x0 )  1
1  x0  2
  1  
(2 x0  3)  x0  1
2

Với x0  1 thì y0  1 lúc đó tiếp tuyến có dạng y   x (trường hợp này loại vì tiếp
tuyến đi qua góc tọa độ, nên không tạo thành tam giác OAB)
Với x0  2 thì y0  4 lúc đó tiếp tuyến có dạng y   x  2
Vậy tiếp tuyến cần tìm là y   x  2

2x 1
Câu 25. Cho hàm số y = có đồ thị (C). Lập phương trình tiếp tuyến của đồ
x 1
thị (C) sao cho tiếp tuyến này cắt các trục Ox, Oy lần lượt tại các điểm A và B thỏa
mãn OA = 4OB.

NGUYỄN VĂN LỰC  0933.168.309 SP Toán K35 - ĐH Cần Thơ


Hàm số FB: http://www.facebook.com/VanLuc168
Giả sử tiếp tuyến d của (C) tại M ( x0 ; y0 )  (C ) cắt Ox tại A, Oy tại B sao cho
OA  4OB .
OB 1 1 1
Do OAB vuông tại O nên tan A    Hệ số góc của d bằng hoặc  .
OA 4 4 4
 3
 x0  1 ( y 0  )
1 1 1
Hệ số góc của d là y ( x0 )   0   
2
( x0  1) 2 ( x0  1) 2 4 x  3 ( y  5)
 0 0
2
 1 3  1 5
 y   ( x  1)   y   x 
4 2 4 4
Khi đó có 2 tiếp tuyến thỏa mãn là:   .
 y   1 ( x  3)  5  y   1 x  13
 4 2  4 4

NGUYỄN VĂN LỰC  0933.168.309 SP Toán K35 - ĐH Cần Thơ


Hàm số FB: http://www.facebook.com/VanLuc168

3. Dạng 3:
Viết phương trình tiếp tuyến với (C): y = f(x) biết tiếp tuyến đi qua
điểm A(xA;yA)

y
(C ) : y  f ( x)

A( x A ; y A )
x
O

 : y  y A  k(x  xA )  y  k(x  xA )  y A

Phương pháp : Ta có thể tiến hành theo các bước sau

Bước 1: Viết phương trình tiếp tuyến (d) với (C) tại điểm M0(x0;y0)  (C )
(d ) : y  f '( x0 )( x  x0 )  f ( x0 ) (*)

Bước 2: Định x0 để (d) đi qua điểm A(xA;yA). Ta có:


(d) đi qua điểm A(xA;yA)  y A  f '( x0 )( x A  x0 )  f ( x0 ) (1)
Bước 3: Giải pt (1) tìm x0. Thay x0 tìm được vào (*) ta sẽ được pttt cần tìm.

Ví dụ 1: Cho hàm số y x3 3x 2 2 có đồ thị là C . Viết phương trình tiếp tuyến


của C , biết tiếp tuyến đi qua điểm A 2; 2 .
Bài giải
♦ Ta có: y ' 3x 2 6 x
♦ Gọi M x0 ; y0 C với y0 x03 3x02 2 là tiếp điểm và là tiếp tuyến với C tại
M0

♦ Phương trình : y y0 y '( x0 )( x x0 ) y ( x03 3x02 2) (3x02 6 x0 )( x x0 )

♦ đi qua điểm A 2; 2 2 ( x03 3x02 2) (3x02 6 x0 )(2 x0 )

2 x03 9 x02 12 x0 4 0

x0 2
2
x0 2 2x 0 5 x0 2 0 1
x0
2

♣ Với x0 2 :y 2

NGUYỄN VĂN LỰC  0933.168.309 SP Toán K35 - ĐH Cần Thơ


Hàm số FB: http://www.facebook.com/VanLuc168
1 9 5
♣ Với x0 :y x
2 4 2
9 5
♦ Vậy có hai tiếp tuyến thỏa đề bài là y 2 và y x .
4 2

x 2
Ví dụ 2: Cho hàm số y có đồ thị là C . Viết phương trình tiếp tuyến của C ,
x 2
biết tiếp tuyến đi qua điểm A 6;5 .
Bài giải
4
♦ Ta có: y ' 2
x 2

x0 2
♦ Gọi M x0 ; y0 C với y0 là tiếp điểm và là tiếp tuyến với C tại M 0
x0 2

♦ Phương trình : y y0 y '( x0 )( x x0 )

x0 2 4
y 2
(x x0 )
x0 2 x0 2

x0 2 4
♦ đi qua điểm A 6;5 5 2
( 6 x0 )
x0 2 x0 2

x02 6 x0 0

x0 0
x0 6

♣ Với x0 0 :y x 1

1 7
♣ Với x0 6 :y x
4 2
1 7
♦ Vậy có hai tiếp tuyến thỏa đề bài là y x 1 và y x .
4 2

NGUYỄN VĂN LỰC  0933.168.309 SP Toán K35 - ĐH Cần Thơ


Hàm số FB: http://www.facebook.com/VanLuc168

BÀI TẬP TỰ LUYỆN

 x 1
Câu 1. Cho hàm số : y  (C). Viết phương trình tiếp tuyến với (C), biết
2x  1
tiếp tuyến đó đi qua giao điểm của đường tiệm cận và trục Ox.

Giao điểm của tiệm cận đứng với trục Ox là  1 


A  ,0 
 2 

Phương trình tiếp tuyến () qua A có dạng  1


y  k x  
 2
 x  1  1
 2x  1  k  x  2 
() tiếp xúc với (C)  /
 
  x  1 
  k coù nghieäm
 2x  1 
 x  1  1
 2x  1  k x  2  (1)
  

 3  k (2 )
 2x  12

Thế (2) vào (1) ta có pt hoành độ tiếp điểm là


 1
3 x  
x  1 2
 
 2x  1
2
2x  1

1 1 3
 (x  1)(2x  1)  3(x  ) và x    x  1 
2 2 2
5 1
x . Do đó k  
2 12
1 1
Vậy phương trình tiếp tuyến cần tìm là: y    x  
12  2

2x  4
Câu 2. Cho hàm số y  (C ) . Cho hai điểm A(1; 0) và B (7; 4) . Viết phương
x 1

trình tiếp tuyến của (C ) , biết tiếp tuyến đi qua điểm trung diểm I của AB .

Gọi  qua I  3; 2 có hệ số góc k   : y  k ( x  3)  2


 2x  4
 k ( x  3)  2
 x  1
.Điều kiện  tiếp xúc (C)  2
 k
 ( x  1) 2
Giải hệ  x  2  k  2
Vậy phương trình tiếp tuyến :  : y  2 x  4
NGUYỄN VĂN LỰC  0933.168.309 SP Toán K35 - ĐH Cần Thơ
Hàm số FB: http://www.facebook.com/VanLuc168
Câu 3. Cho đồ thị (C): y  x  3x  1, viết phương trình tiếp tuyến với (C) biết tiếp
3

tuyến đi qua điểm A(-2; -1).

Ta có: y '  3x2  3


Gọi M  x0 ; x03  3x0  1 là tiếp điểm. Hệ số góc của tiếp tuyến là y '( x0 )  3x02  3 .
Phương trình tiếp tuyến với (C) tại M là  : y   x03  3x0  1  (3x02  3)( x  x0 )
 qua A(-2;-1) nên ta có: 1   x03  3x0  1  (3x02  3)(2  x0 )  x03  3x02  4  0
 x0  1  y0  1
 ( x0  1)( x02  4 x0  4)  0  
 x0  2  y0  1
Vậy có hai tiếp tuyến cần tìm có phương trình là:  : y  1;  : y  9 x  17

NGUYỄN VĂN LỰC  0933.168.309 SP Toán K35 - ĐH Cần Thơ


Hàm số FB: http://www.facebook.com/VanLuc168

VI. BIỆN LUẬN SỐ NGHIỆM CỦA PHƯƠNG TRÌNH


Chuyên đề: Hàm số

A. Tóm tắt lí thuyết & phương pháp giải toán


I. KIẾN THỨC CƠ BẢN
Cơ sở của phương pháp

Xét phương trình f(x) = g(x) (1)


Nghiệm x0 của phương trình (1) chính là hoành độ giao điểm của (C 1):y = f(x) và
(C2):y = g(x) y
(C ) 1

(C 2 )
x
x0

Bài toán: Bằng đồ thị hãy biện luận theo m số nghiệm của phương trình dạng :
f(x) = m (*)
Phương pháp:

Bước 1: Xem (*) là phương trình hoành độ giao điểm của hai đồ thị:

 (C ) : y  f ( x ) : (C) laø ñoà thò coá ñònh


 () : y  m : ( ) laø ñöôøng thaúng di ñoäng cuøng phöông Ox
vaø caét Oy taïi M(0;m)
Bước 2: Vẽ (C) và (  ) lên cùng một hệ trục tọa độ
Bước 3: Biện luận theo m số giao điểm của (  ) và (C)
Từ đó suy ra số nghiệm của phương trình (*)
(C ) : y  f ( x) y
Minh họa:
m2
x
O
m1
 ym
(0; m)

NGUYỄN VĂN LỰC  0933.168.309 SP Toán K35 - ĐH Cần Thơ


Hàm số FB: http://www.facebook.com/VanLuc168
Dạng: f x g m giải tương tự

1 3
Ví dụ: Cho hàm số y  x3  x 2  5
4 2

1) Khảo sát sự biến thiên và vẽ đồ thị (C) của hàm số.


2) Tìm m để phương trình x3  6 x 2  m  0 có ba nghiệm phân biệt.
Bài giải
1) Học sinh tự giải

2) Tìm m để phương trình x3  6 x 2  m  0 có ba nghiệm phân biệt.


♦ Xét phương trình x3  6 x 2  m  0 (1), ta có:
1 3 3 2 m
(1) x  x 5  5 (2)
4 2 4

1 3 3 2
C :y x x 5
♦ Xem (2) là phương trình hoành độ giao điểm của hai đồ thị 4 2
m
:y 5
4

Khi đó số nghiệm của phương trình (1) chính là số giao điểm của C và

♦ Phương trình (1) có ba nghiệm phân biệt cắt C tại ba điểm phân biệt
m
3 5 5
4

0 m 32

♦ Vậy giá trị m cần tìm là 0 m 32 .

NGUYỄN VĂN LỰC  0933.168.309 SP Toán K35 - ĐH Cần Thơ


Hàm số FB: http://www.facebook.com/VanLuc168

BÀI TẬP TỰ LUYỆN

Câu 1. Cho hàm số y  x3  6 x2  9 x  1


a) Khảo sát sự biến thiên và vẽ đồ thị (C) của hàm số đã cho.
1 3 9
b) Tìm các giá trị thực của tham số m để phương trình x  3 x 2  x  m  0 có một
2 2

nghiệm duy nhất:

x  3
TXĐ: D  , y /  3x 2  12 x  9 .
y'  0  
x  1
Hàm số nghịch biến trên các khoảng(-  ;1) và (3;+  ), đồng biến trên khoảng (1;3)
lim y  , lim y  
x  x 

BBT x  1 3 
y' + 0 – 0 +
y 3 
 -1

Đồ thị : đi qua các điểm (3;-1), (1;3), (2;1), (0;-1)


1 3 9
Pt : x  3 x 2  x  m  0  x 3  6 x 2  9 x  1  2m  1 (*)
2 2
Pt (*) là pt hoành độ giao điểm của (C) và đường thẳng d y  2m  1 (d cùng phương trục
Ox) . Số nghiệm của phương trình là số giao điểm của (C) và d. Dựa vào đồ thị (C), để
 2m  1  1 m  0
pt có một nghiệm duy nhất thì :   
 2m  1  3 m  2

Câu 2. Cho hàm số y  x3  6x2  9x  1 (1).


a) Khảo sát sự biến thiên và vẽ đồ thị (C) của hàm số (1).
b) Tìm m để phương trình x(x  3)2  m có 3 nghiệm phân biệt.

x
O 1 2 3 4

-1

NGUYỄN VĂN LỰC  0933.168.309 SP Toán K35 - ĐH Cần Thơ


Hàm số FB: http://www.facebook.com/VanLuc168
b) Ta có: x(x  3)  m  x  6x  9x  1  m  1 .
2 3 2

Phương trình có ba nghiệm phân biệt khi và chỉ khi đường thẳng y = m – 1 cắt (C) tại 3
điểm phân biệt  1  m  1  3  0  m  4

Câu 3. Cho hàm số y  x 4  x 2 .


a) Khảo sát sự biến thiên và vẽ đồ thị  C  của hàm số đã cho.

b) Dựa vào đồ thị  C  hãy tìm tất cả các giá trị của tham số k để phương trình sau có

bốn nghiệm thực phân biệt 4 x 2 1  x 2   1  k .

k 1
+ Đưa về được PT hoành độ giao điểm: x 4  x 2 
4
+ Lập luận được: Số nghiệm PT đã cho chính là số giao điểm của (C) và đường thẳng
k 1
(d): y  .
4
1 k 1
+ Lập luận được: YCBT    0
4 4
+ Giải ra đúng 0  k 1

2x 1
Câu 4. Cho hàm số y 
x 1
Tìm k để đường thẳng (d) : y=kx+2k+1 cắt (C) tại 2 điểm phân biệt.

2x 1
Xét pt =kx+k+1
x 1
kx2+(3k-1)x+2k=0(x  -1)
kx2+(3k-1)x+2k=0 ( vì x=-1 không phải là nghiệm của pt với mọi k)
 k 0
Do đó d cắt ( C ) tại 2 điểm phân biệt  
   k  6k  1  0
2

 k 0

   k  3  2 2 (*)

  k  3  2 2
Vậy với k thõa (*) thì thõa yêu cầu bài toán

Câu 5. Cho hàm số y x 3 3x 2 1 (C)


a) Khảo sát sự biến thiên và vẽ đồ thị (C) của hàm số.
b) Tìm m để phương trình x 3 3x 2 m 0 có 3 nghiệm phân biệt.

Đồ thị :
Cho x = -1  y = 3 , ( -1 ; 3 )
Tâm đối xứng I (1;1)

NGUYỄN VĂN LỰC  0933.168.309 SP Toán K35 - ĐH Cần Thơ


Hàm số y FB: http://www.facebook.com/VanLuc168

3 y=m-1

1
O
-1 1 2 3 x
-1

Tìm m để phương trình x 3 3x 2 m 0 có 3 nghiệm phân biệt.


Ta có x3 3x 2 m 0 x3 3x 2 m (*)
3 2 3 2
x 3x m x 3x 1 m 1
Số nghiệm của phương trình (*) bằng số giao điểm của (C) và d: y = m – 1
Dựa vào đồ thị (*) có 3 nghiệm phân biệt 1 m 1 3 0 m 4

2x  2
Câu 6. Cho hàm số y  (C)
x 1
1*. Khảo sát sự biến thiên và vẽ đthị (C).
2*. Tìm m để đường thẳng d: y = 2x + m cắt đồ thị (C) tại 2 điểm phân biệt A, B

Phương trình hoành độ giao điểm: 2x2 + mx + m + 2 = 0 , (x≠ - 1)


d cắt (C) tại 2 điểm phân biệt  PT(1) có 2 nghiệm phân biệt khác -1
 m2 - 8m - 16 > 0
m  4  4 2

 m  4  4 2

Câu 7. Cho hàm số y  x 4  2 x 2  3


a*) Khảo sát sự biến thiên và vẽ đồ thị của hàm số.
b*) Tìm m để phương trình x 4  2 x 2  m  3 có 4 nghiệm phân biệt.

Đồ thị (C) của hàm số nhận Oy làm trục đối xứng, giao với Ox tại 2 điểm (  3 ; 0).
y

 3 1 O 1 3 x

3
4
b) Ta có x 4  2 x 2  m  3  x 4  2 x 2  3  m (1).
Số nghiệm của phương trình (1) bằng số giao điểm của (C) và đường thẳng y  m
NGUYỄN VĂN LỰC  0933.168.309 SP Toán K35 - ĐH Cần Thơ
Hàm số FB: http://www.facebook.com/VanLuc168
Theo đồ thị ta thấy đường thẳng y  m cắt (C) tại 4 điểm phân biệt khi và chỉ khi
 4  m  3 .
Vậy phương trình đã cho có 4 nghiệm phân biệt khi m  (4;3) .

Câu 8. Cho hàm số y   x 4  3x 2  1 có đồ thị (C)


a) Khảo sát sự biến thiên và vẽ đồ thị hàm số (C).
b) Dựa vào đồ thị (C) tìm m để phương trình x 4  3x 2  m  0 có 4 nghiệm phân biệt.

 Đồ thị: Điểm đặc biệt: (0; 1), (-1; 3), (1; 3)

x 4  3x 2  m  0   x4  3x2  1  m  1
Số nghiệm của phương trình là số giao điểm của đồ thị (C) với đường thẳng y=m+1.
13 9
Dựa vào đồ thị, phương trình có 4 nghiệm phân biệt  1  m  1  0m
4 4

NGUYỄN VĂN LỰC  0933.168.309 SP Toán K35 - ĐH Cần Thơ


Hàm số FB: http://www.facebook.com/VanLuc168

VII. TÌM ĐIỂM TRÊN ĐỒ THỊ HÀM SỐ


THỎA MÃN ĐIỀU KIỆN CHO TRƯỚC
Chuyên đề: Hàm số

A. Tóm tắt lí thuyết & phương pháp giải toán


I. KIẾN THỨC CƠ BẢN
Định nghĩa: Cho hàm số y  f ( x ) xác định trên tập D.
Trong mặt phẳng toạ độ (Oxy ) , tập hợp (C) tất cả các điểm có toạ độ  x; f ( x)  với x  D

được gọi là đồ thị của hàm số y  f ( x ) .

Từ định nghĩa ta có: (C )  M / M ( x; y) vôùi x  D vaø y  f(x)

M ( x0 ; y 0 )  (C )  x0  D và y 0  f ( x0 )

Phương pháp chung


♦ Đặt M x0 , y0 C với y0 f x0 là điểm cần tìm;

♦ Từ điều kiện cho trước ta tìm một phương trình chứa x0 ;


♦ Giải phương trình tìm x0 , suy ra y0 f x0 M x 0 ; y0 .

3x 1
Ví dụ 1: Cho hàm số y có đồ thị là C . Tìm điểm M thuộc đồ thị C sao cho
x 3

khoảng cách từ M đến tiệm cận đứng bằng hai lần khoảng cách từ M đến tiệm cận
ngang.
Bài giải
♦ Đồ thị C có tiệm cận đứng 1 :x 3 0 và tiệm cận ngang 2 :y 3 0

NGUYỄN VĂN LỰC  0933.168.309 SP Toán K35 - ĐH Cần Thơ


Hàm số FB: http://www.facebook.com/VanLuc168
3x0 1
♦ Gọi M x0 , y0 C với y0 x0 3 , ta có:
x0 3

d M, 1 2.d M , 2 x0 3 2. y0 3

3x0 1
x0 3 2. 3
x0 3

16
x0 3
x0 3

2 x0 1
x0 3 16
x0 7

♦ Vậy có hai điểm thỏa đề bài là M1 1;1 và M 2 7;5 .


x 1
Ví dụ 2: Cho hàm số y có đồ thị là C . Tìm điểm M C sao cho khoảng cách
x 1
3
từ điểm M đến đường thẳng  : y  2 x  1 bằng .
5
Bài giải
 x 1 
♦ Gọi M  x0 ; 0   (C ). Khi đó ta có:
 x0  1 

 x0  1
2 x0  1
3 x0  1 3
d ( M , )   
5 12  22 5

x0  1
 2 x0  1   3  2 x02  2 x0  2  3 x0  1
x0  1

 2 x02  2 x0  2  3( x0  1)  2 x02  5 x0  5  0  x0  1
 2  2 
 x0  1 .
 2 x0  2 x0  2  3( x0  1)  2 x0  x0  1  0  2

♣ Với x0  1 y0 0, ta có M 1 ( 1; 0)

ta có M 2  ; 3 
1 1
♣ Với x0  y0 3,
2 2 

và M 2  ; 3  .
1
♦ Vậy có hai điểm thỏa đề bài là M 1 ( 1; 0)
2 

NGUYỄN VĂN LỰC  0933.168.309 SP Toán K35 - ĐH Cần Thơ


Hàm số FB: http://www.facebook.com/VanLuc168

BÀI TẬP TỰ LUYỆN

x 1
Câu 1. Cho hàm số y = (C)
x 3
a) Khảo sát sự biến thiên và vẽ đồ thị (C) của hàm số đã cho.
b) Tìm điểm M thuộc đồ thị (C) sao cho khoảng cách từ M đến tiệm cận ngang của
đồ thị (C) bằng 4.

 x 1 
Gọi M  x0 ; 0  , (x0 ≠3) là điểm cần tìm, ta có:
 x 3
0 
4
Khoảng cách từ M đến tiệm cận ngang: y =1 là d  .
x0  3
4 x0  2
 4  x0  3  1  
x0  3 x 0  4
Với x0  2 ; ta có M  2; 3 . Với x0  4 ; ta có M  4;5
Vậy điểm M cần tìm là M  2; 3 và M  4;5 .

2x  1
Câu 2. Cho hàm số y 
x 1
Tìm điểm M trên (C) để khoảng cách từ M đến tiệm cận đứng của đồ thị (C) bằng
khoảng cách từ M đến trục Ox.

2x  1
Gọi M  x 0 ; y0  ,  x 0  1 , y0  0 , Ta có
y

x
-8 -6 -4 -2 2 4 6 8

x0 1
-5

2x 0  1
 x0 1    x 0  1  2x 0  1
2

x0 1
1 x  0
Với x 0  , ta có : x 02  2x 0  1  2x 0  1   0 Suy ra M  0; 1 , M  4;3
2 x0  4
1
Với x 0  , ta có pt x 02  2x 0  1  2x 0  1  x 02  2  0 (vô nghiệm) .
2
Vậy M  0; 1 , M  4;3

NGUYỄN VĂN LỰC  0933.168.309 SP Toán K35 - ĐH Cần Thơ


Hàm số FB: http://www.facebook.com/VanLuc168

x 1
Câu 3. Cho hàm số y  (1)
x 1
1) Khảo sát sự biến thiên và vẽ đồ thị của hàm số (1)
2) Tìm trên đồ thị hàm số (1) các điểm M có hoành độ âm sao cho M cùng với hai
5
điểm A 1;0  , B  3;1 tạo thành một tam giác có diện tích bằng
2

AB   2;1 , AB  5 , phương trình đường thẳng AB:


y

x
-8 -6 -4 -2 2 4 6 8

-5

 x 1 1
M  x;  là điểm cần tìm, ta có S MAB  AB. d  M ;( AB) 
 x 1  2
x 1
x2 1
1 x 1 x2  4 x 1  x2  9 x  4  0
 S MAB  5 5  2  x  3 (vì x  0)
2 5 x 1  x  x  6  0

ĐS: M  3; 
1
 2

2x 1
Câu 4. Cho hàm số y  . Tìm tọa độ điểm M sao cho khoảng cách từ điểm
x 1
I (1; 2) tới tiếp tuyến của (C) tại M là lớn nhất.

 3 
Nếu M  x0 ; 2    (C ) thì tiếp tuyến tại M có phương trình
 x0  1 
3 3
y2  ( x  x0 ) hay 3( x  x0 )  ( x0  1) 2 ( y  2)  3( x0  1)  0
x0  1 ( x0  1) 2
Khoảng cách từ I (1;2) tới tiếp tuyến là
3(1  x0 )  3( x0  1) 6 x0  1 6
d   .
9   x0  1 9  ( x0  1) 4
4
9
 ( x0  1) 2
( x0  1) 2

9
Theo bất đẳng thức Côsi  ( x0  1) 2  2 9  6 , vây d  6 .
( x0  1) 2

Khoảng cách d lớn nhất bằng 6 khi


9
 ( x0  1) 2   x0  1  3  x0  1  3 .
2

( x0  1) 2


Vậy có hai điểm M: M 1  3;2  3 hoặc M 1  3;2  3   

NGUYỄN VĂN LỰC  0933.168.309 SP Toán K35 - ĐH Cần Thơ


Hàm số FB: http://www.facebook.com/VanLuc168
Câu 5. Cho hàm số y  x  3x  m (1).
3 2

Tìm m để tiếp tuyến của đồ thị (1) tại điểm có hoành độ bằng 1 cắt các trục Ox, Oy lần
3
lượt tại các điểm A và B sao cho diện tích tam giác OAB bằng .
2

Với x0  1  y0  m  2  M(1 ; m – 2)
- Tiếp tuyến tại M là d: y  (3x02  6 x0 )( x  x0 )  m  2
 d: y = -3x + m + 2.
m2 m2 
- d cắt trục Ox tại A: 0  3x A  m  2  x A   A ; 0
3  3 
- d cắt trục Oy tại B: yB  m  2  B(0 ; m  2)
3 1 3 m2
- SOAB   | OA || OB | | OA || OB | 3  m  2  3  (m  2) 2  9
2 2 2 3
m  2  3 m  1
 
 m  2  3  m  5
Vậy m = 1 và m = - 5

x2
Câu 6. Cho hàm số: y  (C)
x 1
a) Khảo sát sự biến thiên và vẽ đồ thị (C) của hàm số.
b) Chứng minh rằng mọi tiếp tuyến của đồ thị (C) đều lập với hai đường tiệm cận một
tam giác có diện tích không đổi.

a) Tự làm
a2
b) Giả sử M  a;   (C).
 a 1 
a2 3 a 2  4a  2
PTTT (d) của (C) tại M: y  y (a).( x  a)   y x 
a 1 (a  1) 2 (a  1) 2
a5
Các giao điểm của (d) với các tiệm cận là: A 1;  , B(2a  1;1) .
 a 1 

 6  6 
IA   0;   IA  ; IB  (2a  2;0)  IB  2 a  1
 a 1  a 1
1
Diện tích IAB : S IAB = IA.IB = 6 (đvdt)  ĐPCM.
2

NGUYỄN VĂN LỰC  0933.168.309 SP Toán K35 - ĐH Cần Thơ


Hàm số FB: http://www.facebook.com/VanLuc168
2x  3
Câu 7. Cho hàm số y  .
x2
1) Khảo sát sự biến thiên và vẽ đồ thị (C) của hàm số.
2) Cho M là điểm bất kì trên (C). Tiếp tuyến của (C) tại M cắt các đường tiệm cận của
(C) tại A và B. Gọi I là giao điểm của các đường tiệm cận. Tìm tọa độ điểm M sao cho
đường tròn ngoại tiếp tam giác IAB có diện tích nhỏ nhất.

 2 x0  3  1
Giả sử M  x0 ; , x0  2 , y '( x0 ) 
x0  2   x0  2
2

1 2x 0 3
Phương trình tiếp tuyến () với (C) tại M: y 2
(x x0)
x0 2 x0 2

 2 x0  2 
Tọa độ giao điểm A, B của () với hai tiệm cận là: A  2;  ; B  2 x0  2;2 
 x0  2 
x x 2  2 x0  2 y  yB 2 x0  3
Ta thấy A B   x0  xM , A   yM suy ra M là trung điểm của
2 2 2 x0  2
AB.
Mặt khác I(2; 2) và IAB vuông tại I nên đường tròn ngoại tiếp tam giác IAB có diện
tích
  2 x0  3  
2
 1 
S =  IM   ( x0  2)  
2 2
 2     ( x0  2) 2  2
 2
  x0  2    ( x0  2) 

1  x0  1
Dấu “=” xảy ra khi ( x0  2)2   
( x0  2)2  x0  3
Do đó điểm M cần tìm là M(1; 1) hoặc M(3; 3)

2x
Câu 8. Cho hàm số y  (C ) tìm điểm M  (C ) sao cho tiếp tuyến của đồ thị hàm
x 1
số tại M cắt hai trục tọa độ tại A
1
B sao cho tam giác OAB có diện tích bằng
4

2 x0 2
Gọi M ( x0 , y0 )  (C )  y0  , y' 
x0  1 ( x  1)2
Tiếp tuyến tại M có dạng:
2 2 x0 2 2 x02
y  y '( x0 )( x  x0 )  y0  y  ( x  x0 )   y x (d )
( x0  1)2 x0  1 ( x0  1)2 ( x0  1) 2
Gọi A  (d )  ox  tọa độ điểm A là nghiệm của hệ:

NGUYỄN VĂN LỰC  0933.168.309 SP Toán K35 - ĐH Cần Thơ


Hàm số FB: http://www.facebook.com/VanLuc168
 2 2x 2

y  x  x   x02
0

 ( x0  1) 2
( x0  1) 2   A( x02 ,0)
y  0 y  0

Gọi B  (d )  oy  tọa độ điểm B là nghiệm của hệ:
 2 2 x02
y  x  x  0 2 x02 2 x02
 ( x0  1) 2 ( x0  1) 2   B(0, )
 y  ( x0  1) ( x0  1) 2
2
x  0

2 x02 2 x02
Tam giác OAB vuông tại O ; OA =  x02  x02 ; OB = 
( x0  1) 2 ( x0  1) 2
Diện tích tam giác OAB:
1 1 2 x04 1
S= OA.OB = . 
2 2 ( x0  1)2 4

 2 x02  x0  1  2 x02  x0  1  0  1
x    y0  2
 4 x  ( x0  1)   2
4 2
 2  0
2
0
 2 x0   x0  1  2 x0  1x0  1 (vn) 
 x0  1  y0  1
1
Vậy tìm được hai điểm M thỏa mãn yêu cầu bài toán: M1 ( ; 2) ; M 2 (1,1)
2

NGUYỄN VĂN LỰC  0933.168.309 SP Toán K35 - ĐH Cần Thơ


Hàm số mũ – hàm số logarit FB: http://www.facebook.com/VanLuc168

I. KIẾN THỨC CƠ BẢN VỀ HÀM SỐ MŨ


Chuyên đề: Hàm số mũ – hàm số logarit

1. Các định nghĩa:

 an  a.a...a (n  Z , n  1, a  R)
n thöøa soá
1
 a  a a
 a 0  1 a  0
1
 a n  n
(n  Z , n  1, a  R / 0)
a
m
n
 an  am ( a  0; m, n  N )
m
 1 1
 a n  m

n m
a
an

2. Các tính chất :

am
m
 a .a  a n m n
 n
 am n
a
m n n m m.n
 (a )  (a )  a  (a.b)n  an .b n
a an
 ( )n 
b bn

3. Hàm số mũ: Dạng : y  ax ( a > 0 , a  1 )


 Tập xác định : D  R
 Tập giá trị : T  R  ( a x  0 x  R )
 Tính đơn điệu:
*a>1 : y  ax đồng biến trên R
* 0 < a < 1 : y  ax nghịch biến trên R
 Đồ thị hàm
y
số mũ : y
y=ax y=ax

1
1 x
x

a>1 0<a<1
NGUYỄN VĂN LỰC  0933.168.309 SP Toán K35 - ĐH Cần Thơ
Hàm số mũ – hàm số logarit FB: http://www.facebook.com/VanLuc168
 Đạo hàm của hàm số mũ:
e  '  e
x x

 a  '  a .ln a
x x

 e  '  e .u ' (với u là một hàm số)


u u

 a  '  a . ln a . u ' (với u là một hàm số)


u u

DẠNG 1: RÚT GỌN

A. LŨY THỪA VỚI SỐ MŨ NGUYÊN

 Đơn giản các biểu thức sau ( với giả thiết chúng có nghĩa )

3 y  x2  y 2   3

 x 4  x3 y  xy 3  y 4
 x  y   1  :  x  y
1
Ví dụ 1. D   2
 x  2 xy  y
2
x x  y 

3 y  x2  y 2  

 x 4  x3 y  xy 3  y 4 3

 x  y  1 :  x  y
1
D 
 x  2 xy  y
2 2
x  x  y 

1
  x3  y 3   x  y 2

  3 xy
 x  y  x  y   3
1

  x  y
2
 x  y    x  y
1

  x  y   :  x  y  1
3 3 1
 

2
 4a  9a 1 a  4  3a 1 
Ví dụ 2. B 1  1 
 2 
1 1
 
 2a  3a 2
a a
2 2

2
 
 4a  9a 1 a  4  3a 1 
2    
2

 4a  9
2
a  4a  3   
2
2 a  3  
 a  3    9a
B 1  1     
 2 
1

1

a
 2a  3 a  a  1   1

 2a  3a 2 a2  a 2   a2 
 
1 1
 a 2
a 2

a  n  b n a  n  b n
Ví dụ 3. A    ab  0; a  b 
a n  bn a n  bn

 a n  bn    bn  a n 
2 2
a  n  b n a  n  b n a n  bn bn  a n 4a n b n
A  n     
a  b n a  n  b n  bn  a n   a n  bn 
a nb n  n n  a nb n  n n   a n  bn bn  a n  b2n  a 2n
 ab   ab 

NGUYỄN VĂN LỰC  0933.168.309 SP Toán K35 - ĐH Cần Thơ


Hàm số mũ – hàm số logarit FB: http://www.facebook.com/VanLuc168
a x a x  1 1 1 1
Ví dụ 4. B 
1
 xa 1  ax -1   1 1  1 1 
4 a x a x 

-1  a  x 1 a 1  x 1  1  x 2  a 2   x  a x  a 
1
B
1
 xa 1
 ax   a 1  x 1  a 1  x 1   4  ax   x  a  x  a 
4    
1 2  x  a  1 x2  a2
2 2

 
4 ax 2 ax

B. LŨY THỪA VỚI SỐ MŨ HỮU TỶ

 Cho a,b là các số dương. Rút gọn biểu thức sau:

2
 a b   12 1

Ví dụ 1.  1  2   :  a  b 2 
 b a  

 .
2
b a
2 2
 a b   12   a
 
1
2 1 1
1  2   :  a  b 2   1   : a b  
b 
 
2
 b a    b a b b

1 9 1 3

a4  a4 b 2
 b2
Ví dụ 2. 1 5
 1 1

a a
4 4
b b2 2

1 1
a 4 1  a 2  1  b   1  a  1  a  2
1 9

1 3 
2
a a b
2
4 4
b 2 2 b
1 5
 1 1
 1
 1

b  1
 
a a4 4
b b 2 2
a 1  a 
4
b 2 2

 2 
 
2
Ví dụ 3. 3
a  3 b  a 3  b 3  3 ab 
 

 2 
     a  b     a    b 
2
a3b 
2 2 3 3
3
a  3 b  a 3  b 3  3 ab   3 3
3 a3b 3 3 3
 a b
  

 1 1
  a 3 b
Ví dụ 4.  a 3  b 3  :  2  3  
   b a 

 13 1
 13 13  13 1
 13 13
a b a b a b a b
3 3 1 1
 13 1
  a b      a 3 3
b
 a  b 3
 
: 2  3  3
  1 1 2 2
 2
 1 1
   b a
2a 3 b 3  a 3  b 3  13 1
 
a b 
3 3
3 a b
 
 Đơn giản các biểu thức sau ( với giả thiết chúng có nghĩa )

NGUYỄN VĂN LỰC  0933.168.309 SP Toán K35 - ĐH Cần Thơ


Hàm số mũ – hàm số logarit FB: http://www.facebook.com/VanLuc168
 3 3
2
 a b 2  a
    14 1

Ví dụ 5. A   3      
: a  b 4


b a   a b3    

 3 3
2  32 12  
     1 1

 : a 4  b 4   a b   a  :  a4  b4 
1 1
A   3   
2
a b a
 b a   a b3      3 1  a 2b 3   
       b 2 a 2    
    
a 1   1 1
 a 2b 2  1
   3  :  a4  b4  
 b ab     1 1

ab3  a 4  b 4 
 

a2  4
Ví dụ 6. B  2
 a2  4 
a   4
 2 a 

a2  4 a2  4 2a 2 :  a  0
B   
 a2  4 
2
a 2
 4
2 a 2 :  a  0
a   4 a
 2a  4a 2

1
 1  x  x2 1  x  x2 
Ví dụ 7. A   2  5  2 x  với
2
x  3,92
 2x  x 2x  x2 
2

1
 1  x  x2 1  x  x2 
1
 
 5  2 x2    x44x x10
3
A  2    5  2 x 
2

 2x  x 2x  x2     
2 2


 4  x  2 5  2x   8  2x
2 2
2

5  2x  2

Với x= 3,92  x 2  3,92  4  x 2  0, 08  2  4  x 2   0,16

5
 32 3
 
  2  27 y 5
 2 
Ví dụ 8. B    3 32 y  2 .3  . Với y = 1,2
10 2

 2  35 y 
  

5
  1 3  1 3  
 32 3
 
5
  2 2    3. y 5   
 2  27 y     1 1
 
  310 32 y  2 .3 
 
5

B 2 2
   3.2 2 y 5  2  32  
 2  35 y  1 1
 
   22  3y 5  
  
  
5
  2   52 
1 1 2 1 1 5

  2  2 .3 y  3 y  3.2 y  2  3    y   y 2 .
2 5 2 5 2 5
Với y=1,2 suy ra y 2  1, 44
    

NGUYỄN VĂN LỰC  0933.168.309 SP Toán K35 - ĐH Cần Thơ


Hàm số mũ – hàm số logarit FB: http://www.facebook.com/VanLuc168
 Rút gọn các biểu thức sau

4 1
1

a 3  8a 3 b b 2
Ví dụ 9. A 2 2 
.  1  2 3
  a 3
a
a 3  2 3 ab  4b 3 

4 1 1 1
1
a  8a b 3  3
b 2
a 3  a  8b  a3 2
A 2 . 1  2
2 
3
  a  2
3
1 1 2
. 1 1
 a3
a
a 3  2 3 ab  4b 3   a 3  2a 3 b 3  4b 3 a 3  2b 3
2 2
a 3  a  8b  2
a 3  a  8b  2
 a  3
 a3  0
2 1 1 2 2 1 1 2
a  8b
a  2a b  4a b  2a b  4a b  8b
3 3 3 3 3 3 3 3

 1 1 1 1

8b  a  a 3 b 3 a 3  2b 3 
Ví dụ 10. B 
6   13 
1

2 1

1

2 

 2a  b 4a  2a b  b 
3 3 3 3 3

  13 1
 32 32 
    a  2b  a b 
1 1 1 1 2 2 3

8b  a  a b 3
a  2b
3 3 3
  8b  a  a b  
3 3
 
B 
6   3 1 1 2 1 1 2   1 1
 2 1 1 2 


 




 6
 2b 3  a 3  4b 3  2a 3 b 3  a 3  
 
3 3 3 3 3
2 a b 4 a 2 a b b
 
  
 2 1 1 2
 13 1 2 

 4b  2a b  a   a  2b 3   2 2
3 3 3 3

8b  a     a 3 b 3  8b  a  6ab   ab
    
6 3
 13   13 
3
6  8b  a 
 
  2b    a  
     

1

  2 3   4    3 4 2   
3 5 7 1 1 1

2
Ví dụ 11. A=   3 .5  : 2  : 16 :  5 .2 .3   
 
       

1
 32 53 74 13 14 14 2
1 1
  3
   
5 7
   1 1 1 2
   3 5  2  15
2 2

 3 5 2 .5 2 3
A=   3 .5  : 2  : 16 :  5 .2 .3   
  22 
2 3 4 3 4 2
 24
         



2

1
1
 1
Ví dụ 12. B   0,5  19.  3
4 2 3
 6250,25   2 
 4

1
1
 1
B   0,5   625  19. 3
4 2 3
0,25
2 
 4
3
4 2.
1 3
1
     54  4   
 2 1 8 19
 19  16  5    10
2 2  3
3
27 27

NGUYỄN VĂN LỰC  0933.168.309 SP Toán K35 - ĐH Cần Thơ


Hàm số mũ – hàm số logarit FB: http://www.facebook.com/VanLuc168
 1
1
1 1
 a b a  b   14
2 2 
Ví dụ 13. A 3
 1 1
 1 1 
:  a  b4 
 a 4  a 2 b 4 a 4  b 4   

 
 1 1
  1 1

 ab a 2  b 2   14 1
  a b a 2  b 2   14 1

A 3  :  a  b 4
   :  a  b 4

 1 1 1 1    1 1 1
 1 1 
   
 a  a b a  b  a a  b  a  b 
4 2 4 4 4 2 4 4 4 4

   
1 1 1
 1 1

a  b  a  a 2b 2 1 b2  a2  b2  b
 . 1  1 1 
1
 1 1
  1
  1 
a 2  a 4  b 4   a 4  b 4  a 2  a 2  b 2 
a
   

  34 3
  34 3
 
  a  b  a  b4 
4

Ví dụ 14. B       ab 
 1 1 
 a2  b2 
 
 

  34 3
 34 3
   32 3 1 1
 12 1
   12 1

               a  b
4 4 2 2 2 2 2
a b a b a b a b a b a b
B     
 ab          a  b
 1 1   1 1   21 1

 a b
2 2
  a b
2 2
 a b 
2
   
     

 32 3
  12 1

 x a 2 1
  x  a 2

Ví dụ 15. C 1   ax  2
 1  xa 
 x 2  a 2   

2
 32 3
  12 1

 x  a 2 1
  x  a 2

C 1   ax  2
 1  x a 
 x  a   
2 2

2
  12 1
 1 1
  
  x  a  x  x 2 a 2  a 
2
  1 1

   
1 1
2 2
x2  a2 
 x a
 1 1   1 1
 1 1 

 x2  a2    x 2  a 2  x 2  a 2  
      
2
 12 1

 x  a 2

   1
1 2
 12 
x a 
2

 

NGUYỄN VĂN LỰC  0933.168.309 SP Toán K35 - ĐH Cần Thơ


Hàm số mũ – hàm số logarit FB: http://www.facebook.com/VanLuc168
 Chứng minh:

 
3
Ví dụ 16. a 2  3 a 4b2  b2  3 b4 a 2  3
a 2  3 b2

   
 a 2  3 a 4 b 2  b 2  3 a 2 b 4  2 2a 2 b 2  a 2 3 a 2 b 4  b 2 3 a 4b 2  a 2  3 3 a 4b 2  3 3 a 2 b 4  b 2

 2 a 2 b 2  a 2 3 a 2 b 4  b 2 3 a 4 b 2  3 a 2b 4  3 a 4b 2
 2a 2b 2  3 a 8b 4  3 a 4b8  3 a 8b 4  2 3 a 6b 6  3 a 4b8

847 3 847
Ví dụ 17. 3 6   6 3
27 27

847 3 847   847  847  


Đặt y  6  3  6 3 
 y  12  3 y  6   6  
27  27  
3
27 27   
 
847 125
12  3 y 3 36   12  3 y 3  12  5 y
27 27
 y 3  5 y  12  0   y  3  y 2  3 y  4   0
 y3

Ví dụ 18. 8
1
38 2
  8
38 2  4
3 4 2  3 2 

1  8
3 8 2  8
38 2  4
3 4 2  3  2 ;VP   4
3 4 2  4
3 4 2  3 2 
  3 2  
3  2  3  2  1  VT

 Viết dưới dạng lũ thừa với số mũ hữu tỉ các biểu thức sau:

Ví dụ 19. A  5 2 3 2 2

 1
 1

  1  3
1
 5 
  3  3
1
 5 1
 1
 31 3
A  2 3 2 2    2 2.2  .2     2 2  .2    2 2.2   2 2 5  210
5
5
   
         
     

11
Ví dụ 20. B  a a a a : a 16  a  0 

NGUYỄN VĂN LỰC  0933.168.309 SP Toán K35 - ĐH Cần Thơ


Hàm số mũ – hàm số logarit FB: http://www.facebook.com/VanLuc168
1
 1
 1 2

  3  2    3 2  2 11
1 2 1 1 15
11
 
11
  1 
  7
1  2
11
a 16 1
B  a a a a : a    a  a .a  : a   a  .a : a   a  : a  11  a 4
16 2 16 4 6 8 16
    
       
      a 16

C. LŨY THỪA VỚI SỐ MŨ VÔ TỶ

 Đơn giản các biểu thức:

2 1
1
Ví dụ 1. a 2 . 
a

2 1
1
a  1 2 1
a 2 .  a 2
 a 2 a1 2
a
a

Ví dụ 2. a  . 4 a 2 : a 4

1
1
a2
a . 4 a 2 : a 4  a   a 2  a
a

a 
3
Ví dụ 3. 3

a 
3
3
a 3. 3
 a3

Ví dụ 4. a 2. .a1,3 : 3 a3 2

a 2. .a1,3
2.
a .a : a 1,3 3 3 2
 2
 a1,3
a

a2 2
 b2 3
Ví dụ 5. 1
 
2
a 2
b 3

a2 2
 b2 3
1 
a 2
b 3
 a 2
b 3
 1  a 2
b 3 a 2
b 3

2a 2

a 2
b 3

2
a 2
b 3

2
a 2
b 3
 a 2
b 3

Ví dụ 6.
a 2 3

 1 a 2 3  a 3  a3 3

a 4 3
a 3

NGUYỄN VĂN LỰC  0933.168.309 SP Toán K35 - ĐH Cần Thơ


Hàm số mũ – hàm số logarit FB: http://www.facebook.com/VanLuc168

a 2 3
1 a 2 3
a 3
a 3 3
  a 3
1 a  3
1 a a 1 a   a
3 3 2 3

 3

1
a 4 3
a 3
a 3
a 3
 1 a  1  a 
3 2 3

a 5
b 7
Ví dụ 7. 2 5 3 7 2 7
a 3
a 3 b 3 b 3

 5 7  2 5 3 7 2 7 
 a 3
 b 3  a 3  a 3 b 3  b 3 
5
b 7  5 7
    a 3 b 3
a
2 5 3 7 2 7 2 5 3 7 2 7
a 3
a 3 b 3 b 3
a 3
a 3 b 3 b 3


 1 
a 
2
Ví dụ 8. 
b 
  4 ab 
 


 1 
a  a  b   a   b
  2 2
b   4  ab   a 2  b 2  2a b  4a b  

 

DẠNG 2: SO SÁNH CÁC CẶP SỐ

 Nếu hai số là hai căn không cùng chỉ số , thì ta phải đưa chúng về dạng có cùng
chỉ số , sau dó so sánh hai biểu thức dưới dấu căn với nha .
 Nếu hai số là hai lũy thừa , thì ta phải chú ý đến cơ số , sau đó sử dụng tính chất
của lũy thừa dạng bất đẳng thức .

 Hãy so sánh các cặp số sau:

Ví dụ 1. 3 30  5
20

 3 30  15 305  15 243.105
Ta có   3 30  5 20
 20  20  8.10
5 15 3 15 3

Ví dụ 2. 4 5  3
7

 4 5  12 53  12 125
Ta có :  3745
3
 7  7  2401 12 4 12

Ví dụ 3. 17  3
28

NGUYỄN VĂN LỰC  0933.168.309 SP Toán K35 - ĐH Cần Thơ


Hàm số mũ – hàm số logarit FB: http://www.facebook.com/VanLuc168
 17  6 173  6 4913
Ta có :   17  3 28
 28  28  784
3 6 2 6

Ví dụ 4. 4 13  5
23 .

 4 13  20 135  20 371.293
Ta có :   4 13  5 23
 5 23  20 234  20 279.841

3 2

Ví dụ 5.   1
1
   .
 3  3

3 2
1 1
Vì 3  2    
3  3

Ví dụ 6. 4 5
 4 7;

7  54 5
4 7

Ví dụ 7. 21,7  20,8 ;

Vì 1,7  0,8  21,7  20,8

1,7 0,8

Ví dụ 8.      ;
1 1
2 2

1,7  0,8 1,7 0,8


 1 1
do :  1    
0  2  1  2  2

1,2 2
 3  3
Ví dụ 9.    
 2 
;
 2   

1, 2  2 1,2 2
  3  3
do :  3      
0  1  2   2 
 2

5

5 2
Ví dụ 10.    1;
7

NGUYỄN VĂN LỰC  0933.168.309 SP Toán K35 - ĐH Cần Thơ


Hàm số mũ – hàm số logarit FB: http://www.facebook.com/VanLuc168
 5
 0 5
 0
2  5  2 5
do :       1
0  5  1  7  7
 7

2,5
1
Ví dụ 11. 2  12
   ;
2


 12   6,25  2,52
  2   2
  6,25
do :   2 12

2  1

5 1
Ví dụ 12. 0,7 6  0,7 3 ;

  5   5   4    1 2
2

  6  36  36   3 
5 1
do :     0,7 6  0,7 3
 0  0,7  1

Ví dụ 13. 20
2  30 3  2

 20 2  20 1  1 20
Ta có : 30 30  2  30 3  2
 3  1  1

 Tìm GTLN của các hàm số sau:

Ví dụ 14. y  3 x x

Đặt t  x  0  y   x  x  t 2  t  t  0   y '  2t  1  0  t   maxy=y   


1 1 1
2 2 4
1
Do vậy : y  3 x x  3  4 3  GTLNy  4 3
4

Ví dụ 15. y   0,5
2
sin x

1 1
Vì : 0  sin 2 x  1  0  0,5sin x  0,51  y  0,5sin x   GTLNy 
2 2

2 2

 Tìm GTNN của các hàm số sau:

Ví dụ 16. y  2 x  2 x

GTNNy  2
y  2 x  2 x  2   x
 x  x  x  0
 2  2
x

NGUYỄN VĂN LỰC  0933.168.309 SP Toán K35 - ĐH Cần Thơ


Hàm số mũ – hàm số logarit FB: http://www.facebook.com/VanLuc168

Ví dụ 17. y  2 x 1  23 x

x 1 3 x x 13 x 2 x 1  23 x
y2 2 2 2  2 2  4  min y  4  
2
x2
 x  1  3  x

Ví dụ 18. y  5sin x  5cos x


2 2

5sin x  5cos x  
2 2

sin 2 x  cos 2 x
y5 sin 2 x
5 cos 2 x
2 5  2  min y  2    cos2x=0  x=  k
sin x  cos x 4 2
2 2

Ví dụ 19. y  e1 x 2

x x 1
ye 1 x 2
e 2x
 e  e  x  1
2

DẠNG 3: TÍNH ĐƠN ĐIỆU

 Xét sự đồng biến, nghịch biến của các hàm số sau:

2 x  2 x
Ví dụ 1. y
2

2 x1  2 x2 1
 2 x1  2 x2 1 2 x1  2 x2 1
Giả sử : x1  x2   1  x1  1  x2    x1 
 2    2    2     2   2 
 x2  x1  x2
      
 2  2
2 x1  2 x1 2 x2  2 x2  x1  x2

   . Vậy hàm số luôn đồng biến trên R.
2 2  y  x1   y  x2 

 
x

Ví dụ 2. y  
3

  
x

Do 1 y    . Là một hàm số đồng biến


3 3

x
2
Ví dụ 3. y  .
e

x
2 2
Do 0  1 y    Là một hàm số nghịch biến
e e

NGUYỄN VĂN LỰC  0933.168.309 SP Toán K35 - ĐH Cần Thơ


Hàm số mũ – hàm số logarit FB: http://www.facebook.com/VanLuc168
x

Ví dụ 4. y   3  .
 3 2

Do 3
3 2
3   
3  2 1 y  
3 
 là một hàm số nghịch biến
 3 2

Ví dụ 5. y  3  
x 1

 3 2

x
 x   
x
x  1   1   3  2
y3     là một hàm số đồng biến ( 3  2  3 )
 3 2  3
  
3  2  

3 

NGUYỄN VĂN LỰC  0933.168.309 SP Toán K35 - ĐH Cần Thơ


Hàm số mũ – hàm số logarit FB: http://www.facebook.com/VanLuc168

II. KIẾN THỨC CƠ BẢN VỀ HÀM SỐ LOGARIT


Chuyên đề: Hàm số mũ – hàm số logarit

1. Định nghĩa: Với a > 0, a  1 và N > 0

dn
log a N  M  aM  N

a  0

Điều kiện có nghĩa: log a N có nghĩa khi a  1
N  0

2. Các tính chất :

 log a 1  0
 log a a  1
 loga aM  M
 aloga N  N
 log a (N1 .N 2 )  log a N1  log a N 2
N1
 log a ( )  log a N1  log a N 2
N2
 loga N  . loga N Đặc biệt : loga N2  2. loga N

3. Công thức đổi cơ số :

 log a N  log a b. log b N


log a N
 log b N 
log a b
* Hệ quả:
1 1
 loga b  và log N log a N
log b a ak k

4. Hàm số logarít: Dạng y  log a x ( a > 0, a  1 )


 Tập xác định : D  R
 Tập giá trị TR
 Tính đơn điệu:
*a>1 : y  log a x đồng biến trên R
* 0 < a < 1 : y  log a x nghịch biến trên R

NGUYỄN VĂN LỰC  0933.168.309 SP Toán K35 - ĐH Cần Thơ


Hàm số mũ – hàm số logarit FB: http://www.facebook.com/VanLuc168
 Đồ thị của hàm số lôgarít: y
y
y=logax y=logax

1 x
x O
O 1

a>1 0<a<1

 Đạo hàm của hàm số lôgarit:


 ln x  ' 
1
x
và  ln x  '  1x
 ln u  ' 
u'
u
và  ln u  '  uu' (với u là một
hàm số)
 log a x  ' 
1
x ln a
và  log a x ' 
1
x ln a

 log a u  ' 
u'
u.ln a
và  log a u ' 
u'
u.ln a
(với u là một hàm số)

I. SỬ DỤNG ĐỊNH NGHĨA VỀ LOGARIT

 Bài 1. Tìm tập xác định của các hàm số sau :

x 1
a/ y  log 1 .
2 x5

 x 1  x 1
0  x 1  2
 x 1  1
 log 1
 2 x 1   1  0   0  x  1
Điều kiện :     x 1   x 1
 x 1  0  x 1  0  x  1  x  1  x  1  x  1

 x 1 
 x  1
Vậy D= 1;  

 x2  1 
b/ y  log 1  log5 .
5  x3 

NGUYỄN VĂN LỰC  0933.168.309 SP Toán K35 - ĐH Cần Thơ


Hàm số mũ – hàm số logarit FB: http://www.facebook.com/VanLuc168
  x 1 
2
 x2  x  2
log 1  log 5 0  x3  0
 3 x3   x2  1  2
 x2  1  x  3  1  x  5 x  14
Điều kiện : 0  log 5 1   0
 x3 0  x  1  5
2
 x3
 x2  1  x3  x  3
 0  5 
 x3 
3  x  1  x  2
  x   3; 2    2;7 
  x  3  2  x  7

 Bài 2. Tính giá trị của các biểu thức sau :

 14  12 log9 4 log125 8 
a.  81  25  .49
log7 2

 

 14  12 log9 4 log125 8   4 14  12 log9 4  2log53 23  2log7 2


 25  .49   3   5
log7 2
 81 7
   
 2 .3log5 2 
1
3 
  31 log3 4  5 3  7 7    4  4  19
log 4

  4 

1
log 2 3 3log5 5
1 log 4 5
b. 16 4 2

1
log 2 3 3log5 5
4 
2 1 log 4 5
16 1 log 4 5
4 2
 2log2 36log5 5  16.25  3.26  592

 12 log7 9log7 6  log 3 4 


c. 72  49 5 
 

 12 log7 9  log7 6  log 4   9 1


72  49  
 5 5   72 7log7 9  2 log7 6  52 log5 4  72     18  4,5  22,5
 36 16 
 

1 lg 2
d. 36 6  10  3 9
log 5 log 36

36log6 5  101lg2  3log9 36  6log6 25  10log5  25  5  30

II. SỬ DỤNG CÁC CÔNG THỨC VỀ LO-GA-RÍT

 Bài 1. Tính giá trị của các biểu thức sau :

a. A  log 9 15  log 9 18  log 9 10


NGUYỄN VĂN LỰC  0933.168.309 SP Toán K35 - ĐH Cần Thơ
Hàm số mũ – hàm số logarit FB: http://www.facebook.com/VanLuc168

15.18 1 3
A  log 9 15  log 9 18  log 9 10  log 9  log 9 33  log 3 33 
10 2 2

1
b. B  2log 1 6  log 1 400  3log 1 3 45
3 2 3 3

1  36.45 
B  2 log 1 6  log 1 400  3log 1 3 45  log 1    log 1 9   log 3 3  4
2 4

3 2 3 3 3  20  3

1
c. C  log36 2  log 1 3
2 6

1 1 1 1 1
C  log36 2  log 1 3  log 6 2  log 6 3  log 6 2.3 
2 6 2 2 2 2

d. D  log 1  log3 4.log 2 3


4

1 1
D  log 1  log 3 4.log 2 3   log 4  log 2 3.log 3 4    log 4  log 2 4    log 2 2  
4 2 2

 Bài 2. Hãy tính

  
a. A  log 2  2sin   log 2 cos
 12  12

          1
A  log 2  2sin   log 2 cos  log 2  2sin .cos   log 2 sin   log 2  1
 12  12  12 12   6 2

b. B  log 4  3 7  3 3   log 4  3 49  3 21  3 9 

B  log 4  3

7  3 3  log 4  3

49  3 21  3 9  log 4 
  3
733  3

49  3 21  3 9   log 4  7  3   1

c. C  log10 tan 4  log10 cot 4

C  log10 tan 4  log10 cot 4  log  tan 4.cot 4   log1  0

1
d. D  log 4 x  log 4 216  2 log 4 10  4 log 4 3
3

1
D  log 4 x  log 4 216  2log 4 10  4log 4 3
3
1 6.34 35
 log 4 63  log 4 102  log 4 34  log 4 2  x 
3 10 50

NGUYỄN VĂN LỰC  0933.168.309 SP Toán K35 - ĐH Cần Thơ


Hàm số mũ – hàm số logarit FB: http://www.facebook.com/VanLuc168

 Bài 3. Hãy tính :

a. A  log a a3 a 5 a

 3 1  1  1 1 37
A  log a a3 a 5 a  log a  a 2 5   3   
  2 5 10

b. B  log a a 3 a 2 5 a a

 1 1  1  2  3 
1
1

 2 5  
   27  3 3
B  log a a 3 a 2 5 a a  log a a  1    1 3
   10  10
 

a 5 a3 3 a 2
c. log 1
a a4 a

 1 53  23 
a 5 a3 3 a 2
  a     34  3    91
log 1 log a  11   
a a4 a  a2 4   15 4  60
 

d. log tan10  log tan 20  log tan 30  ....  log tan 89 0

log tan10  log tan 20  log tan 30  ....  log tan 890  log  tan10 tan 890.tan 20.tan 87 0...tan 450   0
( vì : tan 890  cot10  tan10 tan 890  tan10 cot10  1 ; Tương tự suy ra kết quả)

e. A  log3 2.log 4 3.log 5 4......log15 14.log16 15

1
A  log 3 2.log 4 3.log 5 4......log15 14.log16 15  log16 15.log15 14....log 5 4.log 4 3.log 3 2  log16 2  
4

1 1 1 1
f. A     ..........   x  2011!
log 2 x log 3 x log 4 x log 2011 x

1 1 1 1
A    ..........   log x 2  log x 3  ...  log x 2011  log x 1.2.3...2011
log 2 x log3 x log 4 x log 2011 x
 log x 2011!. Nếu x=2011! Thì A= log 2011!  2011!  1

 Bài 4. Chứng minh rằng :

NGUYỄN VĂN LỰC  0933.168.309 SP Toán K35 - ĐH Cần Thơ


Hàm số mũ – hàm số logarit FB: http://www.facebook.com/VanLuc168
a. Nếu : a  b  c ; a  0, b  0, c  0, c  b  1 , thì :
2 2 2

log c b a  log c b a  2 log c b a.log c b a

Từ giả thiết : a 2  c 2  b2   c  b  c  b   2  log a  c  b   log a  c  b 


1 1
2   2log c b a.log c b a  log c b a  log c b a
log c b a log c b a

b. Nếu 0<N  1 thì điều kiện ắt có và đủ để ba số dương a,b,c tạo thành một cấp số
nhân ( theo thứ tự đó ) là :
log a N log a N  logb N
  a, b, c  1
log c N logb N  log c N

Nếu 3 số a,b,c theo thứ tự đó lập thành cấp số nhân thì ta có : b 2  ac


Lấy lo ga rít cơ số N 2 vế :
1 1 1 1
 2log N b  log N a  log N c    
logb N log a N log c N logb N
log a N  log b N log b N  log c N log a N log a N  log b N
    . ( đpcm )
log a N .log b N log c N .log b N log c N log b N  log c N

c. Nếu : log x a,log y b,log z c tạo thành cấp số cộng ( theo thứ tự đó )thì :
2log a x.log c z
logb y   0  x, y, z, a, b, c  1
log a x  log c z

Nếu : log x a,log y b,log z c tạo thành cấp số cộng thì log x a  log z c  2log y b
1 1 2 2log a x.log c z
    logb y 
log a x log c z logb y log a x  logc z

a  b ln a  ln b
d. Giả sử a,b là hai số dương thỏa mãn : a 2  b 2  7ab . Chứng minh : ln 
3 2

ab
2

Nếu : a 2  b2  7ab   a  b   9ab     ab . Lấy ln hai vế ta có :


2

 3 
 ab   a  b  ln a  ln b
2ln    ln a  ln b  ln  
 3   3  2

e. Chứng minh :
log a b  log a x
log ax  bx  
1  log a x

log a bx log a b  log a x


Vế trái : log ax bx    VP   dpcm 
log a ax 1  log a x

f. Chứng minh :

NGUYỄN VĂN LỰC  0933.168.309 SP Toán K35 - ĐH Cần Thơ


Hàm số mũ – hàm số logarit FB: http://www.facebook.com/VanLuc168
1 1 1 k  k  1
  .........  
log a x log a2 x log ak x 2log a x

k 1  k 
VT= log x a  log x a 2  ...log x a k  1  2  3  ...  k  log x a   VP
2log a x

III. SỬ DỤNG CÔNG THỨC ĐỔI CƠ SỐ

 Bài 1. Tính

a. A  log 6 16 . Biết : log12 27  x

log3 27 3 3 3 x 3 x
A  log 6 16 . Từ : log12 27  x    x  log3 4   1   log 3 2  (*)
log3 12 1  log3 4 x x 2x
log 24 4log3 2 2  3  x  .2 x 12  4 x
Do đó : A  log6 16  3  . Thay từ (*) vào ta có : A= 
log3 6 1  log3 2 x  x  3 x3

b. C  log 3 135 . Biết: log 2 5  a;log 2 3  b

log 2 5 a a  3b
Từ : C  log3 135  log3 5.33  log3 5  3  3  3 
log 2 3 b b

c. D  log 6 35 . Biết : log 27 5  a;log8 7  b;log 2 3  c

1 1
Ta có : a  log 27 5  log 3 5  log 3 5  3a; b  log 8 7  log 2 7  log 2 7  3b (*)
3 3
log 2 5.7 log 2 5  log 2 7 log 2 3.log3 5  log 2 7 b.3a  3b 3b  a  1
Suy ra : D  log 6 35     
log 2 2.3 1  log 2 3 1  log 2 3 1 b b 1

d. Tính : log 49 32 . Biết : log 2 14  a

Ta có : log 2 14  a  1  log 2 7  a  log 2 7  a  1


log 2 25 5 5
Vậy : log 49 32   
log 2 7 2
2 log 2 7 2  a  1

 Bài 2. Rút gọn các biểu thức

a. A   log a b  logb a  2  log a b  log ab b  logb a  1

2
 log a b  1 
A   log a b  logb a  2  log a b  log ab b  logb a  1    1  log ab a   1 
 log a b 

NGUYỄN VĂN LỰC  0933.168.309 SP Toán K35 - ĐH Cần Thơ


Hàm số mũ – hàm số logarit FB: http://www.facebook.com/VanLuc168
2 2 2
 log a b  1   log a a   log a b  1   1   log a b  1   log a b 
  1   1    1   1      1
 log a b   log a ab   log a b   1  log a b   log a b   1  log a b 
log a b  1 1
 1   logb a
log a b log a b

1
b. B  log 2 2 x 2   log 2 x  x log  logx 2 x 1
 log 22 x 4
2

1 1
B  log 2 2 x 2   log 2 x  x  log 22 x 4  1  2 log 2 x   log 2 x  log 2 x  1   4 log 2 x  
log x  log 2 x 1 2

2 2
1  3log 2 x   log 2 x   8  log 2 x   9  log 2 x   3log 2 x  1
2 2 2

c. C  log a p  log p a  2  log a p  log ap p  log a p

 log a p  1
2
 log a p 
C  log a p  log p a  2  log a p  log ap p  log a p   log a p   log a p 
2
log p
a  1  log a p 
 log a p  1  log 2a p 
 
3
   log a p  log a p
log a p  1  log a p 

 Bài 3. Trong mỗi trường hợp sau , hãy tính log a x , biết log a b  3;log a c  2 :

a. x  a 3b 2 c

Ta có : log a x  log a  a 3b 2 c   3  2 log a b  log a c  3  2.3  1  8  23


1
2

a4 3 b
b. x 
c3

 a4 3 b  1 1 2 28
Ta có : log a x  log a  3   4  log a c  3log a c  4   2   6  10  
 c  3 3 3 3

a 2 4 bc 2
c. x  3
ab 4 c

Ta có :
 a 2 4 bc 2  1 1 1
log a x  log a  3 4   2  log a b  2log a c   4log a b  log a c
 ab c  4 3 2
3 1 161
 2   4   12  1 
4 3 12

 Bài 4. Chứng minh

NGUYỄN VĂN LỰC  0933.168.309 SP Toán K35 - ĐH Cần Thơ


Hàm số mũ – hàm số logarit FB: http://www.facebook.com/VanLuc168

1
a. log  a  3b   log 2   log a  log b  với : a  3b  0; a 2  9b 2  10ab
2

Từ giả thiết : a  3b  0; a 2  9b 2  10ab  a 2  6ab  9b 2  4ab   a  3b   4ab


2

1
Ta lấy log 2 vế : 2 log  a  3b   2 log 2  log a  log b  log  a  3b   log 2   log a  log b 
2

b. Cho a,b,c đôi một khác nhau và khác 1, ta có :


b c
 log 2a  log 2a ; log a b.log b c.log c a  1
c b
c a b
 Trong ba số : log 2a ;log 2b ;log 2c luôn có ít nhất một số lớn hơn 1
b b c c a a

b c
Chứng minh : log 2a  log 2a .
c b
1 2

* Thật vậy : log a  log a     log a  log 2a    log a   log 2a


b c c b c c
c b b c  b b
* log a b.logb c.log c a  1  log a b.log b a  log a a  1
* Từ 2 kết quả trên ta có :
2
c a b  b c a
log log 2b log 2c   log a .log b log c   1 Chứng tỏ
2
a trong 3 số luôn có ít nhất một số
b
b c c a a  bc c a a b

lớn hơn 1

IV. BÀI TẬP VỀ SO SÁNH

 Nếu so sánh hai loga rít có cùng cơ số thì ta chú ý đến cơ số trong hai trường
hợp (0;1) và lớn hơn một để so sánh hai biểu thức bị lo ga rít hóa với nhau
 Trong trường hợp hai lo ga rít khác cơ sô , khác biểu thức bị lo ga rít hóa thì ta
chọn một số b nào đó . Sau đó ta so sánh hai lo ga rít với số b . Từ đó suy ra kết
quả
1
 Ví dụ 1: so sánh hai số : log 3 4  log 4 . Ta có :
3
1 1
log 3 4  log 3 3  1;log 4  log 4 4  1  log 3 4  log 4
3 3
 Ví dụ 2. So sánh : 3log 6 1,1
7 log 6 0,99
. Ta có :
3log6 1,1  3log6 1  1; 7 log6 0,99  7 log6 1  1  3log6 1,1  7 log6 0,99

 Bài 1. Không dùng bảng số và máy tính .Hãy so sánh :

a/ log0,4 2  log0,2 0,34 .

NGUYỄN VĂN LỰC  0933.168.309 SP Toán K35 - ĐH Cần Thơ


Hàm số mũ – hàm số logarit FB: http://www.facebook.com/VanLuc168
 2  1  log 0,4 2  log 0,4 1  0
Ta có :   log 0,2 0,3  log 0,4 2
0,3  1  log 0,2 0,3  log 0,2 1  0

3 2
b/ log 5  log 3 .
3 4 4 5

5 3 3
 3  1  0  4  1  log 5 4  log 5 1  0
 2 3
 log 3  log 5
3 3
Ta có : 
0  3  1, 0  2  1  log 2  log 1  0 4 5 3 4
 4 5
3
4 5
3
4

1
log5
c/ 2log 3  3
5 2
.

log 5 3  log 5 1  2log5 3  2log5 1  20  1


 1
Ta có :  1 log5
1  log 5 3  log 5
log 5  log 5 1  3 2  3 5  3  1 2
log 1 0

 2

d/ log3 2  log 2 3 .
log3 1  log3 2  log3 3  0  log 3 2  1
Ta có :   log 2 3  log3 2
log 2 2  log 2 3  log 2 4  1  log 2 3  2

e/ log 2 3  log 3 11 .

1  log 3  2
 log3 11  log 2 3
2
Ta có : 
log3 11  log 3 9  2

2log 2 5 log 1 9
f/ 2 2
 8.

2log 2 5 log 1 9 25
25 log 2 25
Ta có : 2log 2 5  log 1 9  log 2 25  log 2 9  log 2 2 2
2 9 
2 9 9
2log 2 5 log 1 9
2
25 25 625 648
Nhưng :     82 2
 8
9 92 81 81

5
log 2 3 log 4
g/ 4 11
 18 .

5 1 5 5 9 11
log 2 3 log 4 2log2 3 log2 log 2 9 log 2 log 2 9 11 81.11
Ta có : 4 11
2 2 11
2 11
2 5
 
5 5
5
81.11 891 90 log 2 3 log 4
Nhưng :    18  4 11
 18
5 5 5

NGUYỄN VĂN LỰC  0933.168.309 SP Toán K35 - ĐH Cần Thơ


Hàm số mũ – hàm số logarit FB: http://www.facebook.com/VanLuc168
8
log3 2  log 1
 5.
9
h/ 9 9

log3 2  log 1
8
8 8  2.3 
2log3 2  log9 log3 2  log3 log3   6 36 40
3 3 3     5
9  8
Ta có : 9 9 9 9

8 8 8

1
log 6 2  log 5
1 2 6

k/    3 18 .
6

1
log6 2  log 5
1
1
2 6
 log6 2  log6 5  log6 10
log6 1 3 1
Ta có :   6 6 6 10
   3 18
6 10 1000

 Bài 2. Hãy so sánh :

a/ log 2 10  log 5 30 .

log 10  log 8  3
Ta có :  2  log 2 10  log5 30
2

log5 30  log5 36  3

b/ log3 5  log 7 4 .

log 5  log 3  1
Ta có :  3  log3 5  log 7 4
3

log 4  log 7  1
 7 7

1
c/ 2 ln e3  8  ln .
e

2 ln e3  2.3  6
 1
Ta có :  1  8  ln  2 ln e3
8  ln  8  1  9 e
 e

 Bài 3. Hãy chứng minh :

1
a/ log 1 3  log3  2 .
2 2

1 1 1
Ta có : log 1 3   2  *
 log 3 
1 1 2
2 log 3
log 3
2 2
1 1 1 1 1
Nhưng : log 3  0   log 3   2  log 3   2
2 2 log 1 2 log 1
3 3
2 2

NGUYỄN VĂN LỰC  0933.168.309 SP Toán K35 - ĐH Cần Thơ


Hàm số mũ – hàm số logarit FB: http://www.facebook.com/VanLuc168
b/ 4log5 7  7log5 4 .

Ta có : 4log 7   7log 4 
log5 7
5 7
 7log5 7.log7 4  7log5 4 . Vậy 2 số này bằng nhau

c/ log 3 7  log 7 3  2 .

1
Ta có : log3 7  0  log3 7  log 7 3  log3 7  2
log3 7

d/ 3log 5  5log 3 .
2 2

Ta có : 3log 5   5log 3 
log2 5
2 5
 5log2 5.log5 3  5log2 3

1
e/  log 3  log19  log 2 .
2

1
  log 3  log 10  log 3  log 3 10  log 900
2
Ta có : 
log19  log 2  log 19  log 361

 2 4
361 1
 log 900  log   log 3  log19  log 2
4 2

5 7 log 5  log 7
f/ log  .
2 2

5 7 5 7 log 5  log 7
Ta có :  5. 7  log  log 5. 7 
2 2 2

 Bài 4. Hãy so sánh :

6 5
a. log 3  log 3
5 6

 6 5
log 3 5  log 3 5  0 6 5
6 5
  6 5
Ta có :   log 3  log 3 . Hoặc :  5 6  log 3  log 3
log 5  log 6  0 5 6 3  1 5 6
 3
6
3
6

b. log 1 9  log 1 17
3 3

NGUYỄN VĂN LỰC  0933.168.309 SP Toán K35 - ĐH Cần Thơ


Hàm số mũ – hàm số logarit FB: http://www.facebook.com/VanLuc168
 1
0   1
Ta có :  3  log 1 9  log 1 17
9  17 3 3

c. log 1 e  log 1 
2 2

 1
0   1
Ta có :  2  log 1 e  log 1 
e   2 2

HÀM SỐ LO-GA-RÍT

I. ĐẠO HÀM :

 Bài 1. Tính đạo hàm các hàm số sau :

a. y   x 2  2 x  2  e x

y   x2  2x  2 ex  y '   2x  2 e x   x2  2x  2 e x   x2  e x

b. y   s inx-cosx  e2 x

y   s inx-cosx  e 2 x  y '   cosx+sinx  e 2 x  2  s inx-cosx  e 2 x   3sin x  cosx  e 2 x

e x  e x
c. y 
e x  e x

e x  e x
y  x x  y ' 
 e x  e x  e x  e  x    e x  e x  e x  e  x 

4
e e  e x  e x   e x  e x 
2 2

d. y  ln  x 2  1

y  ln  x 2  1  y ' 
2x
x 1
2

ln x
e. y 
x

ln x 1 1  1  ln x
y  y '  2  .x  ln x  
x x x  x2

NGUYỄN VĂN LỰC  0933.168.309 SP Toán K35 - ĐH Cần Thơ


Hàm số mũ – hàm số logarit FB: http://www.facebook.com/VanLuc168
f. y  1  ln x  ln x

ln x 1  ln x 1  2 ln x
y  1  ln x  ln x  y '   
x x x

 Bài 2. Tính đạo hàm các hàm số sau:

a. y  x 2 ln  x2  1 
y  x 2 ln  
x 2  1  y '  2 x.ln  
x2  1 
x2 x
2  x 2  1
 2 x.ln  
x2  1 
x3
2  x 2  1

b. log 2  x 2  x  1

2x 1
y  log 2  x 2  x  1  y ' 
 x  x  1 ln 2
2

c. y  3 ln 2 x

 2
 2 1
 1 2
y  3 ln 2 x  y '   ln x  3  '   ln x  3  3
  3 x 3x ln x

x4
d. y  log 2  
 x4

 x4 1  16 x4 16
y  log 2   y'   :  2
 x4 ln 2   x  4  x  4   x  4  ln 2
 2

 x2  9 
e. y  log3  
 x5 

 x2  9  1  2 x  x  5  x  9 x 2  9  x 2  10 x  9
2

y  log 3   y'   : 
 x5  ln 3   x  5
2
x  5   x  5   x 2  9  ln 3

 1 x 
f. y  log  
 2 x 

 1 x 
y  log   

1  x 1 1 x 

 x 1  
 y ' :
 2 x  ln10  16 x x 2 x  8 x ln10 1  x
   

NGUYỄN VĂN LỰC  0933.168.309 SP Toán K35 - ĐH Cần Thơ


Hàm số mũ – hàm số logarit FB: http://www.facebook.com/VanLuc168
II. GIỚI HẠN

 Bài 1. Tìm các giới hạn sau :

ln  3 x  1  ln  2 x  1
a. lim
x 0 x

ln  3 x  1  ln  2 x  1 ln  3 x  1 ln  2 x  1
lim  lim  lim  3 2 1
x 0 x x 0 3 x 0 2
x x
3 2

ln  3 x  1
b. lim
x 0 sin 2 x

ln  3x  1
ln  3x  1 3x
3x 3
lim  lim 
x 0 sin 2 x x 0 sin 2 x 2
2x
2x

ln  4 x  1
c. lim
x 0 x

ln  4 x  1 ln  4 x  1
lim  lim 4 4
x 0 x x 0 4x

e5 x  3  e3
d. lim
x 0 2x

3 
e5 x  3  e 3 e5 x  1 5e3
lim  lim e 5 
x 0 2x x 0 2.  5 x  2

ex 1
e. lim
x 0 x 1 1

ex 1 ex 1
lim
x 0
 lim
x  1  1 x 0 x
 
x  1  1  1.2  2

 Bài 2. Tìm các giới hạn sau

ln  2 x  1
a. lim
x 0 tan x

NGUYỄN VĂN LỰC  0933.168.309 SP Toán K35 - ĐH Cần Thơ


Hàm số mũ – hàm số logarit FB: http://www.facebook.com/VanLuc168
ln  2 x  1
ln  2 x  1 2x
2x
lim  lim 2
x 0 tan x x  0 tan x
x
x

e 2 x  e3 x
b. lim
x 0 5x

e 2 x  e3 x e2 x  1 e3 x  1 2 3 1
lim  lim  lim 3   
x 0 5x x  0 5
.2 x
x  0 5  3x  5 5 5
2

e3 x  1
c. lim
x 0 x

e3 x  1 e3 x  1
lim  lim 3 3
x 0 x x 0 3x

 1

d. lim  xe x  x 
x 
 

 1 
 1x   1x   e x 1 
lim  xe  x   lim x  e  1  lim   1
x 
  x   x  1 
 x 

sin 3 x
e. lim
x 0 x

sin 3 x sin 3 x
lim  lim 3 3
x 0 x x  0 3x

1  cos5x
f. lim
x 0 x2

5x
2sin 2
1  cos5x 2  25
lim 2
 lim 2
x 0 x x 0
4  5x  2
 
25  2 

 Bài 3. Tìm các giới hạn sau :

cosx  cos3x
a. lim
x 0 sin 2 x

NGUYỄN VĂN LỰC  0933.168.309 SP Toán K35 - ĐH Cần Thơ


Hàm số mũ – hàm số logarit FB: http://www.facebook.com/VanLuc168
cosx  cos3x 2sin 2 x sin   x  2
4 cos x.sin x
lim 2
 lim 2
 lim 4
x 0 sin x x 0 sin x x 0 sin 2 x

b. lim  
1
 t anx 
x   cosx 
2

 1 
lim   t anx  .
 cosx
x  
2

  1 1   1 1  cost
Đặt : t   x  x  t   t anx=  tan   t    cot t 
2 2 cosx    2  sin t sint
cos   t 
2 
t t
2sin 2 tan
2 t   1  2 2 
  tan . Khi x  ; t  0  lim   t anx   lim

t t 2 2 x   cosx  t  0 t t
2sin cos 2
2 2 2

3
c. xlim

 x  2  sin
x

 x  ; t  0
3 1  3
lim  x  2  sin . Đặt : t  3  1  lim  x  2  sin  lim  6t  3  3
 x  2  x   2  t  3t  6t  3
x  x x x  x t 0
  

 
 2  2 cos x 
d. lim  
x   
4
 sin  x  4  
  

  
 x  t  ; x  ;t  0
  4 4
 2  2 cos x  
   
lim   . Đặt : x   t   2  2 cos   t 
x 
   4  2  2 cos x   4   2 1  cost+sint 
4
 sin  x  4     
   sin t sint
 sin  x  4 
  
t t t t t
2 1  cost+sint  2sin 2  2sin cos sin  cos
Do đó :  2 2 2 2 2 2 2  2 tan t  2
sint t t t 2
2sin cos cos
2 2 2
 
 2  2 cos x 
  lim  2 tan  2   2
t
Vậy : lim 
x      t o  2 
4
 sin  x  4  
  

NGUYỄN VĂN LỰC  0933.168.309 SP Toán K35 - ĐH Cần Thơ


Hàm số mũ – hàm số logarit FB: http://www.facebook.com/VanLuc168

III. PHƯƠNG TRÌNH MŨ & LOGARIT


Chuyên đề: Hàm số mũ – hàm số logarit

1. CÁC ĐỊNH LÝ CƠ BẢN:

1. Định lý 1: Với 0 < a  1 thì : aM = aN  M=N

2. Định lý 2: Với 0 < a <1 thì : aM < aN  M > N (nghịch biến)

3. Định lý 3: Với a > 1 thì : aM < aN  M < N (đồng biến )

4. Định lý 4: Với 0 < a  1 và M > 0;N > 0 thì : loga M = loga N  M = N

5. Định lý 5: Với 0 < a <1 thì : loga M < loga N  M >N (nghịch biến)

6. Định lý 6: Với a > 1 thì : loga M < loga N  M < N (đồng biến)

2. CÁC PHƯƠNG PHÁP GIẢI PHƯƠNG TRÌNH MŨ & LOGARIT:


Dạng cơ bản: ax  m (1)
 m  0 : phương trình (1) vô nghiệm
 m  0 : ax  m  x  loga m
Dạng cơ bản: loga x  m
 m  : loga x  m  x  am

NGUYỄN VĂN LỰC  0933.168.309 SP Toán K35 - ĐH Cần Thơ


Hàm số mũ – hàm số logarit FB: http://www.facebook.com/VanLuc168

a. Phương pháp 1: Biến đổi phương trình về dạng : aM = aN ; log a M  log a N

(Phương pháp đưa về cùng cơ số)

x
 2
Ví dụ 1: Giải phương trình 0,125.4 2x 3
  (1)
 8 
 

Bài giải
♥ Đưa hai vế về cơ số 2, ta được:
5 x

1 2 3.24 x 6
2 2

5
x 5 3
24 x 9
22 4x 9 x x 9 x 6
2 2

♥ Vậy nghiệm của phương trình là x 6 

Tự luyện: Giải các phương trình sau


x 1 x
5x 7 2 1
1) 1,5 2) 4.2 x
3) 3x.23 x 576
3 4
2
1 x
4) 3x 3x 2
3

Ví dụ 2: Giải phương trình log 2 x 1 2 log 4 3x 2 2 0 (1)

Bài giải
x 1
x 1 0
♥ Điều kiện: 2 x 1 (*)
3x 2 0 x
3

♥ Khi đó: 1 log 2 x 1 log 2 3 x 2 2

x 1
log 2 2
3x 2
x 1 1
3x 2 4

4x 4 3x 2 x 2 [thỏa (*)]
♥ Vậy nghiệm của phương trình là x 2 

Ví dụ 3: Giải phương trình log 2 x  log3 x  log 6 x  log36 x (1)

NGUYỄN VĂN LỰC  0933.168.309 SP Toán K35 - ĐH Cần Thơ


Hàm số mũ – hàm số logarit FB: http://www.facebook.com/VanLuc168
Bài giải
♥ Điều kiện: x 0
♥ Áp du ̣ng công thức log a c  log a b  logb c ,  0  a, b, c; a  1; b  1 , ta có
1  log 2 x  log3 2  log 2 x  log 6 2  log 2 x  log36 2  log 2 x
 log 2 x  log 3 2  log 6 2  1  log 36 2   0 *
Do log3 2  log 6 2  1  log36 2  0 nên
 *  log 2 x  0  x  1

♥ Vậy nghiệm của phương trình là x 1 


Tự luyện: Giải các phương trình sau
1) log3 x log 3 x 2 1 2) log3 x 1 log3 x 2 log3 6
3) log x 2 7 x 6 log x 1 1 4) 2 log2 2x  2  log 1 9x  1  1
2
1 1 1
5) log 1  log 3 3 (2  3x 1 ) 6) log 2 log 1 x 2 x 3
3
32 x 1 3 x 2

7) log 4 x 12 .log x 2 1 8)
log 1 x 1 log 1 x 1 log 1 7 x 1
2 2 2

9) log 4  x  3  log 2  x  7   2  0 10) log 7  x 2  2   log 1 8  x   0


7

11) log3  2 x  7   log 1  x  5  0


3

Ví dụ 4: Giải phương trình: log3(x  1)2  log 3 (2x  1)  2 (1)

Bài giải
x 1
x 1 0
♥ Điều kiện: 1 (*)
2x 1 0 x
2

♥ Khi đó: 1 2 log 3 x 1 2 log 3 2 x 1 2

log 3 x 1 log 3 2 x 1 1

log 3 x 1 2 x 1 1

x 1 2x 1 3 (2)
1
Với x 1 thì 2 1 x 2x 1 3 2 x2 3x 4 0 : phương trình vô
2

nghiệm
1
x loaïi
Với x 1 thì 2 x 1 2x 1 3 2x 2
3x 2 0 2 [thỏa (*)]
x 2

♥ Vậy nghiệm của phương trình là x 2 


NGUYỄN VĂN LỰC  0933.168.309 SP Toán K35 - ĐH Cần Thơ
Hàm số mũ – hàm số logarit FB: http://www.facebook.com/VanLuc168

Tự luyện: Giải các phương trình sau


1) log 2 x 2 2 log 2 3 x 4
2
2) log2 x 2 log 4 x 5 log 1 8 0
2
2
3) 2 log3 x 2 log 3 x 4 0
4) log2 x 2 log2 x 5 log 1 8 0
2

5) log 2 1 2 x x 2 2 log 2 3 x

b. Phương pháp 2: Đặt ẩn phụ chuyển về phương trình đại số

Ví dụ 5: Giải phương trình 9x 4.3x 45 0 (1)


Bài giải
♥ Đặt t 3x với t 0 , phương trình (1) trở thành t 2 4t 45 0 (2)
t 5 loaïi
2
t 9

Với t 9 thì 3x 9 x 2

♥ Vậy nghiệm của phương trình là x 2 


Tự luyện: Giải các phương trình sau
1) 16x 17.4x 16 0
2) 25x 6.5x 5 0
3) 32x+8  4.3x+5 + 27 = 0
2 2

4) 9 x x 1
10.3x x 2
1 0

Ví dụ 6: Giải phương trình 3x 1


18.3 x
29 (1)
Bài giải
♥ Biến đổi phương trình (1) ta được
18
1 3.3x 29 (2)
3x

♥ Đặt t 3x với t 0 , phương trình (1) trở thành 3t 2 29t 18 0 (3)

NGUYỄN VĂN LỰC  0933.168.309 SP Toán K35 - ĐH Cần Thơ


Hàm số mũ – hàm số logarit FB: http://www.facebook.com/VanLuc168
2
t
3 3
t 9

Với t 9 thì 3x 9 x 2
2 2 2
Với t thì 3x x log 3
3 3 3
2
♥ Vậy nghiệm của phương trình là x 2; x log 3 
3

Tự luyện: Giải các phương trình sau


1) 5x 1 53 x
26 0
x2 x2
2) 101 101 99

Ví dụ 7: Giải phương trình 6.9x  13.6x + 6.4x = 0 (1)


Bài giải

♥ Chia hai vế phương trình (1) cho 4 x ta được


x 2 x
3 3
1 6. 13. 6 0 (2)
2 2
x
3
♥ Đặt t với t 0 , phương trình (1) trở thành 6t 2 13t 6 0 (3)
2
2
t
3
3
3
t
2
x
3 3 3
Với t thì x 1
2 2 2
x
2 3 2
Với t thì x 1
3 2 3

♥ Vậy nghiệm của phương trình là x 1; x 1 


Tự luyện: Giải các phương trình sau
1) 4.9x 12x 3.16x

2) 3.16 x  2.81x  5.36 x


3) 32 x  4  45.6 x  9.22 x  2  0
4) 5.2x  7. 10x  2.5x
5) 27 x 12x 2.8x

NGUYỄN VĂN LỰC  0933.168.309 SP Toán K35 - ĐH Cần Thơ


Hàm số mũ – hàm số logarit FB: http://www.facebook.com/VanLuc168

Ví dụ 8: Giải phương trình log 22 x 3log 2 2 x 1 0 (1)

Bài giải
♥ Điều kiện: x 0
♥ Khi đó: 1 log 22 x 3log 2 x 2 0

Đặt t log 2 x , phương trình (1) trở thành t 2 3t 2 0 (3)


t 1
3
t 2

1
Với t 1 thì log 2 x 1 x [thỏa (*)]
2
1
Với t 2 thì log 2 x 2 x [thỏa (*)]
4
1 1
♥ Vậy nghiệm của phương trình là x ;x 
4 2

1 2
Ví dụ 9: Giải phương trình 1 (1)
5 log x 1 log x

Bài giải
x 0
♥ Điều kiện: log x 5 (*)
log x 1
1 2
♥ Đặt t log x t 5, t 1 , phương trình (1) trở thành 1 (3)
5 t 1 t
t 2
3 1 t 2 5 t 5 t 1 t t2 5t 6 0
t 3
Với t thì log x 2
2 x 100 [thỏa (*)]
Với t 3 thì log x 3 x 1000 [thỏa (*)]
♥ Vậy nghiệm của phương trình là x 100; x 1000 
Tự luyện: Giải các phương trình sau
6 4
1) log 22 x 2 4 log 2 x3 8 0 2)  3
log2 2x log2 x 2

3) log3 3x 1 .log3 3x 1
3 6

Ví dụ 10: Giải phương trình 2log x 3 1


2log 3 x 2
x (1)
Bài giải
♥ Điều kiện: x 0
NGUYỄN VĂN LỰC  0933.168.309 SP Toán K35 - ĐH Cần Thơ
Hàm số mũ – hàm số logarit FB: http://www.facebook.com/VanLuc168
♥ Đặt t log 3 x x 3t thì phương trình (1) trở thành
t
t 1 t t 9 t t 2 4
2.2 .2 3 .2 3 t 2
4 4 3 9

Với t 2 thì x 9 (thỏa điều kiện)


♥ Vậy nghiệm của phương trình là x 9 

5.2 x 8
Ví dụ 11: Giải phương trình log 2 3 x (1)
2x 2

Bài giải
♥ Điều kiện 5.2x 8 0 (*)
5.2 x 8
♥ Ta có: 1 23 x

2x 2

2 x 5.2 x 8 8 2x 2

5.22 x 16.2x 16 0 (2)


♥ Đặt t 2x với t 0 , phương trình (2) trở thành 5t 2 16t 16 0 (3)
t 4
3 4
t
5

Với t 4 thì 2x 4 x 2 [thỏa (*)]


♥ Vậy nghiệm của phương trình là x 2 
Tự luyện: Giải phương trình sau log 2 3.2 x 1 2x 1

c. Phương pháp 3: Biến đổi phương trình về dạng tích số A.B=0,..

Ví dụ 12: Giải phương trình 4.5x 25.2x 100 10x (1)


Bài giải
♥ Ta có: 1 4.5x 2 x.5 x 25.2 x 100 0

5x 4 2 x 25 2 x 4 0

4 2 x 5x 25 0

5x 25
x
x 2
2 4

♥ Vậy nghiệm của phương trình là x 2 

NGUYỄN VĂN LỰC  0933.168.309 SP Toán K35 - ĐH Cần Thơ


Hàm số mũ – hàm số logarit FB: http://www.facebook.com/VanLuc168

Tự luyện: Giải các phương trình sau


1) 3.7 x 49.3x 147 21x
2) 32 x x  3  9x  3 x 1
3) log 2 x 2 log 7 x 2 log 2 x.log 7 x

d. Phương pháp 4: Lấy lôgarít hai vế theo cùng một cơ số thích hợp nào đó
(Phương pháp lôgarít hóa)

Ví dụ 13: Giải phương trình 3x.2 x


2

1 (1)
Bài giải
♥ Lấy lôgarit hai vế với cơ số 3, ta có
2

1 log 3 3x.2 x log 3 1


2

log3 3x log3 2 x 0

x x 2 log 3 x 0

x1 x log 3 2 0

x 0
1
x log 2 3
log 3 2

♥ Vậy nghiệm của phương trình là x 0, x log 2 3 

e. Phương pháp 5: Nhẩm nghiệm và sử dụng tính đơn điệu để chứng minh
nghiệm duy nhất (thường là sử dụng công cụ đạo hàm)

♥ Ta thường sử dụng các tính chất sau:


 Tính chất 1: Nếu hàm số f tăng ( hoặc giảm ) trong khoảng (a;b) thì
phương trình f(x) = C có không quá một nghiệm trong khoảng (a;b). ( do
đó nếu tồn tại x0  (a;b) sao cho f(x0) = C thì đó là nghiệm duy nhất của
phương trình f(x) = C)

NGUYỄN VĂN LỰC  0933.168.309 SP Toán K35 - ĐH Cần Thơ


Hàm số mũ – hàm số logarit FB: http://www.facebook.com/VanLuc168
 Tính chất 2 : Nếu hàm f tăng trong khoảng (a;b) và hàm g là hàm một
hàm giảm trong khoảng (a;b) thì phương trình f(x) = g(x) có nhiều nhất
một nghiệm trong khoảng (a;b). (do đó nếu tồn tại x0  (a;b) sao cho
f(x0) = g(x0) thì đó là nghiệm duy nhất của phương trình f(x) = g(x))

Ví dụ 14: Giải phương trình 3x 4x 5x (1)


Bài giải
♥ Chia hai vế phương trình (1) cho 5x 5 x 0, x , ta có
x x
3 4
1 1 (2) ( Dạng f x C )
5 5
x x
3 4
♥ Xét hàm số f x trên , ta có
5 5
x x
3 3 4 4
f' x ln ln 0, x f x nghịch biến trên (*)
5 5 5 5

♥ Mặt khác f 2 1 (2) có nghiệm x 2

(**)
Từ (*) và (**) ta suy ra phương trình (2) có nghiệm duy nhất x 2
♥ Vậy nghiệm của phương trình (1) là x 2 

x
1
Ví dụ 15: Giải phương trình 2x 1 (1) (Dạng f x g x )
3

Bài giải
x
1
♥ Xét các hàm số f x và g x 2x 1 trên , ta có
3

f x nghịch biến trên và g x đồng biến trên


(*)
♥ Mặt khác f 0 g 0 (1) có nghiệm x 0

(**)
Từ (*) và (**) ta suy ra phương trình (1) có nghiệm duy nhất x 0
♥ Vậy nghiệm của phương trình là x 0 

NGUYỄN VĂN LỰC  0933.168.309 SP Toán K35 - ĐH Cần Thơ


Hàm số mũ – hàm số logarit FB: http://www.facebook.com/VanLuc168
Bài tập:
Giải các phương trình sau
x
1) 2x = 1+ 3 2 2) 2 x  3  x 3) 2 3 x
 x 2  8x  14

4) 2.2 x  3.3 x  6 x  1 5) 3.25x 2   3x  10  .5x 2  3  x  0

6) 9 x x 12 3x 11 x 0 7) log 22 x x 1 log 2 x 6 2x

Ví dụ 16: Giải phương trình 2log 5 x 3


x (1)
Bài giải
♥ Điều kiện: x 3

Khi đó: 1 log 5 x 3 log 2 x (2)


♥ Đặt t log 2 x x 2t thì phương trình (2) trở thành
t t
2 1
log5 2 t
3 t t
2 3 t
5 3 1 (3)
5 5
t t
2 1
♥ Xét hàm số f t 3 trên , ta có
5 5
t t
2 2 1 1
f' t ln 3. ln 0, t f t nghịch biến trên (*)
5 5 5 5

♥ Mặt khác f 1 1 (3) có nghiệm t 1 (**)

Từ (*) và (**) ta suy ra phương trình (3) có nghiệm duy nhất t 1


♥ Vậy nghiệm của phương trình (1) là x 2 

NGUYỄN VĂN LỰC  0933.168.309 SP Toán K35 - ĐH Cần Thơ


Hàm số mũ – hàm số logarit FB: http://www.facebook.com/VanLuc168

BÀI TẬP TỰ LUYỆN

Phương trình mũ

Câu 1. Giải phương trình 52 x 1  6.5 x  1  0 .

5 x  1
x  0
52 x 1  6.5 x  1  0  5.52 x  6.5 x  1  0   x 1  
5   x  1
 5

Câu 2. Giải phương trình 3.25 x2  3x  10 5 x2  x  3 .

3.25 x  2   3 x  10  5 x  2  x  3
    
 5 x  2 3.5 x  2  1  x 3.5 x  2  1  3 3.5 x  2  1  0 
 
 3.5 x 2  1 5 x2  x  3  0 
3.5 x2  1  0 1
  x 2
5  x  3  0 2
1 1
+ 1  5x2   x  2  log5  2  log5 3
3 3
2  5   x  3 . Vế trái là hàm đồng biến vế phải là hàm nghịch biến mà (2)
x 2

có nghiệm x = 2 nên là nghiệm duy nhất.


Vậy Phương trình có nghiệm là: x = 2  log5 3 và x = 2.

Câu 3. Giải phương trình: (3  2 2) x  2( 2 1) x  3  0

(3  2 2) x  2( 2  1) x  3  0
 ( 2  1)2 x  2( 2  1) x  3  0
 ( 2  1)3 x  3( 2  1) x  2  0
 ( 2  1) x  2
 x  log 2 1 2

2x 2 6x 6
Câu 4. Giải phương trình: 2 2.4x 1

2x 2 6 x 6 1
(2x 2 6x 6) 2
x 1
2 2.4 2 2 2.22(x 1)
2x 3x 3
22x 3

2
x 3
x 3x 3 2x 3 x2 x 6 0
x 2

NGUYỄN VĂN LỰC  0933.168.309 SP Toán K35 - ĐH Cần Thơ


Hàm số mũ – hàm số logarit FB: http://www.facebook.com/VanLuc168
Câu 5. Giải phương trình: 2 4x 4
17.2 2x 4
1 0

16x 4x
24 x 4
17.22x 4
1 0 17. 1 0 42x 17.4x 16 0
16 16
x
t 1 4 1 x 0
t2 17t 16 0
t 16 4x 16 x 2

Câu 6. Giải phương trình: 25 x  3.5 x  10  0

25x  3.5x  10  0  52 x  3.5 x  10  0


Đặt t  5x , t  0
Phương trình trở thành:
t  2(nhan)
t 2  3t  10  0  
t  5(loai)
t  2  5  2  x  log5 2
x

Vậy phương trình đã cho có nghiệm x  log5 2 .

Câu 7. Giải phương trình 2 x  23 x  2  0

8
2 x  23 x  2  0  2 x   2  0  2 2 x  2.2 x  8  0
2x
Đặt t  2 x , t  0
Phương trình trở thành:
t  4 (nhan)
t 2  2.t  8  0  
t  2 (loai)
t  4  2x  4  x  2
Vậy phương trình có nghiệm x = 2.

Câu 8. Giải phương trình sau: 9x  10.3x  9  0

9x  10.3x  9  0  32 x  10.3x  9  0
Đặt t  3x , t  0 .
t  1 (nhan)
Phương trình trở thành: t 2  10t  9  0  
t  9 (nhan)
t  1  3x  1  x  0
t  9  xx  9  x  2
Vậy phương trình có hai nghiệm x = 0 và x = 2.

NGUYỄN VĂN LỰC  0933.168.309 SP Toán K35 - ĐH Cần Thơ


Hàm số mũ – hàm số logarit FB: http://www.facebook.com/VanLuc168
Câu 9. Giải phương trình sau: 8.3 + 3.2 = 24 + 6 x x x

Phương trình đã cho tương đương (3x -3)(8-2x )= 0


Từ đó tìm được x=1 hoặc x=3

Câu 10. Giải phương trình 2e x  2e x  5  0, x  R .

2e x  2e x  5  0  2e2 x  5e x  2  0.
Đặt t  e x , t  0 . Phương trình trở thành
t  2
2t  5t  2  0   1
2
t 
 2
ex  2  x  ln 2

 x 1
e   x  ln 1
 2  2

Câu 11. Giải phương trình sau:


5.32 x 1  7.3x 1  1  6.3x  9 x 1  0

Đặt t  3x  0 . (1)  5t 2  7t  3 3t  1  0

 x  log 3 ; x   log 5
3 3
5

 2 x 1  2 x 1 4
Câu 12. Giải phương trình (2  3) x  (2  3) x 
2 2

2 3

Phương trình  (2  3) x 2 x  (2  3) x 2 x  4 .
2 2

+) Ta có: (2  3) x  2 x .(2  3) x  2 x  (4  3) x 2 x
 1, x 
2 2 2
.
1
đặt t  (2  3) x 2 x  0  (2  3) x 2 x  .
2 2

t
t  2  3 (TM )
trở thành: 1
t   4  t 2  4t  1  0   .
t t  2  3 (TM )
x  1 2
t  2 3, ta có: (2  3) x 2 x
 2  3  x2  2 x  1  x2  2 x 1  0  
2

 x  1  2
ta có: (2  3) x 2 x  (2  3) 1  x 2  2 x  1  x 2  2 x  1  0  x  1 .
2
t  2 3,
+) KL: ...

Câu 13. Giải phương trình 2 x 1  3x  3x 1  2 x  2


2 2 2 2

Tập xác định .


1
 3x  3x 1
 2x 2
 2x 1
1  8  3x 1 1  3
2 2 2 2 2 2
2x
x 2 1
2 4
    x 2  1  2  x   3.
3 9

NGUYỄN VĂN LỰC  0933.168.309 SP Toán K35 - ĐH Cần Thơ


Hàm số mũ – hàm số logarit FB: http://www.facebook.com/VanLuc168
Câu 14. Giải phương trình: 7  2.7  9  0 . x 1 x

14 t  2
Đặt t  7 x , t  0 . Ta có phương trình: t   9  0  t 2  9t  14  0  
t t  7
Với t  2, suy ra 7 x  2  x  log7 2
Với t  7, suy ra 7 x  7  x  1
Vậy phương trình đã cho có tập nghiệm là S  log 7 2;1 .

Câu 15. Giải phương trình: 34  2 x = 953 x  x


2

Đưa về cùng cơ số 3 khi đó phương trình tương đương với x 2  2 x  3  0

nghiệm cần tìm là x = 1 hoặc x = -3

Câu 16. Giải phương trình 5 2 x 2  26.5 x 2  1  0

Giải phương trình 5 2 x 2  26.5 x 2  1  0


t  1 x  0
Đặt t = 5x >0. Phương trình <=> t2–26t + 25 = 0    .
t  25 x  2

Câu 17. Giải phương trình 2.4x  6x  9x.

Phương trình
 2  x
x x 2x x    1  Loai 
4 6 2 2 3
 2.       1  2.       1  0    x   log 2 2
9 9 3 3  2 x 1
   3

 3  2
Vậy phương trình có nghiệm x   log 2 2
3

x 1
Câu 18. Giải phương trình: 312 x.27 3
 81 .

x 1
3.
Phương trình đã cho tương đương với : 312 x.3 3
 81  312 x.3x 1  34
32 x  34  2  x  4  x  2.

x 1
2
x 1
Câu 19. Giải phương trình 4 x   trên tập số thực.
2

x 1
x2  x 1 2 x
 21 x
2
4    22 x
2

NGUYỄN VĂN LỰC  0933.168.309 SP Toán K35 - ĐH Cần Thơ


Hàm số mũ – hàm số logarit FB: http://www.facebook.com/VanLuc168
 3  17
x 
4
2 x 2  2 x  1  x  2 x 2  3x  1  0  
 3  17
x 
 4

Câu 20. Giải phương trình 5.9 x  2.6 x  3.4 x (1)

Phương trình đã cho xác định với mọi x 


Chia cả hai vế của phương trình (1) cho 4 x  0 ta được :
2x x
3 3
5.9 x  2.6 x  3.4 x  5.    2.    3
2 2
3
2x
3
x
 3  2 x    3  x 
 5.    2.    3  0     1 5.    3  0 (2)
2 2  2     2  
x x
3 3
Vì 5.    3  0 x  nên phương trình (2) tương đương với   1 x  0.
2 2
Vậy nghiệm của phương trình là: x  0

Câu 21. Giải phương trình 22 x5  22 x3  52 x2  3.52 x+1 .

TXĐ D =
Phương trình  2 2 x 3 (4  1)  52 x 1 (5  3)
 2 2 x 3.5  5 2 x 1.8
2x
2
   1
5 .
 2x  0  x  0

   
x x
Câu 22. Giải phương trình: 5 1  5  1  2x1

x x
 5  1  5 1 
PT        2
 2   2 
 5 1
x
1
   t (t  0) t 2 t 1
Đặt  2  ta có phương trình: t
x
 5 1
Với t=1     1  x  0
 2 
Vậy phương trình có nghiệm x=0

Câu 23. Giải phương trình 2log x 3 1


2log 3 x 2
x (1)

Điều kiện: x 0
Đặt t log 3 x x 3t thì phương trình (1) trở thành

NGUYỄN VĂN LỰC  0933.168.309 SP Toán K35 - ĐH Cần Thơ


Hàm số mũ – hàm số logarit FB: http://www.facebook.com/VanLuc168
t
t 1 t t 9 t t 2 4
2.2 .2 3 .2 3 t 2
4 4 3 9

Với t 2 thì x 9 (thỏa điều kiện)


Vậy nghiệm của phương trình là x 9

 3x
Câu 24. Giải phương trình sau: 5x  625
2

3x 3x
 625  5 x  54  x 2  3 x  4
2 2
5x
x 1
 x 2  3x  4  0  
 x  4
Vậy phương trình có nghiệm x = 1 và x = -4.

3 x  6
Câu 25. Giải phương trình sau: 2x  16
2

3 x  6 3 x  6
 16  2 x  24  x 2  3 x  6  4
2 2
2x
x  5
 x 2  3x  10  0  
 x  2
Vậy phương trình có nghiệm x = 5 và x = -2.

Câu 26. Giải phương trình sau: 2x 1.5x  200

2 x 1.5x  200  2.2 x.5 x  200


 10 x  100  x  2
Vậy phương trình có nghiệm x = 2.

Câu 27. Giải phương trình: 23 x  x10  4x  x4  2x  x2  16  0 .


2 2 2

Phương trình tương đương:


 x 10 2 x 8  x2  x 14 2 x 12  x 2
 22 x  2x  16  0  23 x  22 x  2x 1  0
2 2 2 2 2 2
23 x
2 x 12  x 2  2 x 12
 (22 x  1)(2x  1)  0  22 x 1  0
2 2 2

 2 x 12  x  2
 22 x  20  2 x 2  2 x  12  0  
2

x  3
Vậy phương trình có 2 nghiệm x  2, x  3.

    2x
log3 x log3 x
Câu 28. Giải phương trình: 10  1  10  1  .
3

Điều kiện: x > 0

NGUYỄN VĂN LỰC  0933.168.309 SP Toán K35 - ĐH Cần Thơ


Hàm số mũ – hàm số logarit FB: http://www.facebook.com/VanLuc168

    2
log3 x log3 x
Ta có phương trinhg tương đương với: 10  1  10  1  .3log3 x
3
log3 x log3 x log3 x
 10  1   10  1  2  10  1 
     . Đặt t    (t > 0).
 3   3  3  3 
 1  10
t 
3
1 2 
Phương trình trỏ thành: t    3t  2t  3  0  1  10 (loại)
2

t 3 t 
 3

1  10
Với t = ta giải được x = 3
3
Vậy phương trình đã cho có nghiệm duy nhất x =3.

NGUYỄN VĂN LỰC  0933.168.309 SP Toán K35 - ĐH Cần Thơ


Hàm số mũ – hàm số logarit FB: http://www.facebook.com/VanLuc168
Phương trình logarith

Câu 1. Giải phương trình: 2log2 (x - 2) + log0,5 (2x - 1) = 0

2 log2(x 2) log0,5(2x 1) 0 (*)


x 2 0 x 2
 Điều kiện: 1 x 2
2x 1 0 x
2
 Khi đó, (*) log2 (x 2)2 log2 (2x 1) 0 log2 (x 2)2 log2 (2x 1)
x 1 (loai)
(x 2)2 (2x 1) x2 6x 5 0
x 5 (nhan)

Câu 2. Giải phương trình: x  log2 (9  2x )  3 .

Điều kiện: 9  2 x  0 . Phương trình đã cho tương đương: log2 (9  2x )  3  x  9  2x  23 x


8 2x  1  x  0
 9  2  x  2  9.2  8  0   x
x 2x x
 (thỏa điều kiện)
2 2  8  x  3

Câu 3. Giải phương trình log52 x  log0,2 (5x)  5  0.

GPT: log52 x  log 0,2 (5 x)  5  0 (1)


Đk: x>0. PT (1)  log52 x  log5 (5x)  5  0  log52 x  log5 x  6  0
log5 x  3  x  125
 
log5 x  2  x  1/ 25
KL: Vậy tập nghiệm PT (1) là T  1/ 25;125

Câu 4. Giải phương trình: log 2 ( x 2  2 x  8)  1  log 1 ( x  2)


2

log 2 ( x 2  2 x  8)  1  log 1 ( x  2)
2

 log2 ( x  2 x  8)  log 2 2  log 2 ( x  2)  log2 ( x2  2 x  8)  log 2 2( x  2)


2

 x20  x20
   x  6
 x  2 x  8  2( x  2)  x  4 x  12  0
2 2

NGUYỄN VĂN LỰC  0933.168.309 SP Toán K35 - ĐH Cần Thơ


Hàm số mũ – hàm số logarit FB: http://www.facebook.com/VanLuc168
Câu 5. Giải phương trình: log 2 3.log 3  2 x  1  1
3
PT  log 2  2 x  1  1  2x 1  2  x 
2

Câu 6. Giải phương trình: log  x  1  log  3  x   log  2 x  3


3 3 3

x 1  0

Điều kiện 3  x  0  1  x  3 (*)
2 x  3  0

Phương trình tương đương log 3  x  1  log 3  3  x   log 3  2 x  3
 log 3  x  1 (3  x)  log 3  2 x  3
  x  1 (3  x)  2 x  3
  x2  2 x  3  2 x  3   x2  0
 x = 0 , kết hợp với đk (*) phương trình có 1 nghiệm x = 0

Câu 7. Giải phương trình: log 2  3x  1  6  log 0,5  5 x  2 

2
ĐK x 
5
PT đã cho tương đương với log 2  3x  2  5 x  2   6   3x  2  5 x  2   64
x  2
 15 x 2  4 x  68  0  
 x   34
 15
Kết hợp đk ta được tập nghiệm phương trình là: S  2

Câu 8. Giải phương trình: log x  2  log 1 (2  x)  log 27 x3  0


3
3

+ ĐK: 0  x  2 (*)
+PT  log3 ( x  2)  log3 (2  x)  log3 x  0  log3 [( x  2)(2  x)]= log3 x  (2  x)(2  x)  x
1  17
 x2  x  4  0  x 
2

1  17
Kết hợp với (*) ta được nghiệm của phương trình là x 
2

NGUYỄN VĂN LỰC  0933.168.309 SP Toán K35 - ĐH Cần Thơ


Hàm số mũ – hàm số logarit FB: http://www.facebook.com/VanLuc168
Câu 9. Giải phương trình: log2 (4  4).log 2 (4  1)  3 .
x 1 x

 
log 2 (4 x 1  4).log 2 (4 x  1)  3  2  log 2 (4 x  1) .log 2 (4 x  1)  3

t  1
Đặt t  log 2 (4 x  1) , phương trình trở thành:  2  t  t  3  
t  3
 t  1  log2 (4x  1)  1  4x  1  2  x  0 .
1 7
 t  3  log 2 (4 x  1)  3  4 x  1   4 x   : Phương trình vô nghiệm.
8 8
Vậy phương trình đã cho có nghiệm: x  0 .

Câu 10. Giải phương trình log 2  x2  x  1  log2  x2  x  3

ĐK: x 

       
2 2
PT  log 2 x 2  x  1  log 2 x 2  x  3  x 2  x  1  x 2  x  1  2  0

3
Đặt: t  x 2  x  1, t 
4
t  1( L)
Ta được phương trình : t 2  t  2  0  
t  2( N )
 1  5
x 
2
Với t  2  x2  x 1  0  
 1  5
x 
 2

1  5 1  5
Vậy : x  và x  là nghiệm của phương trình.
2 2

Câu 11. Giải phương trình sau:


2log32 x  5log3 (9 x)  3  0

Đk:x>0
 2log 32 x  5(log 3 9  log 3 x)  3  0
Khi đó PT  2log 32 x  5log 3 x  12  0
log 3 x  4  x  81
  3  1 (t/m)
log 3 x  x 
 2  3
9

NGUYỄN VĂN LỰC  0933.168.309 SP Toán K35 - ĐH Cần Thơ


Hàm số mũ – hàm số logarit FB: http://www.facebook.com/VanLuc168
Câu 12. Giải phương trình log2 (x 1) log2 (3x 4) 1 0.

4
Điều kiện xác định: x  (*). Với điều kiện (*), ta có
3
(1)  log2 (x  1)(3x  4)  1  log2(3x 2  7x  4)  log2 2
 3x 2  7x  2  0  x  2 (do điều kiện (*)).
Vâ ̣y phương trình đã cho có nghiê ̣m duy nhấ t x = 2.

Câu 13. log3  x  5  log9  x  22  log 3  x  1  log 3


2.  2 

Tập xác định D  1;   \ 2.


 2   log3  x  5  log3 x  2  2 log3  x  1  log3 2
 x  5 . x  2  2  x  5 . x  2  2 x  1 2
    
 x  1
2

Với x  2 ta có:  x  5 x  2   2  x  1  x 2  3x  10  2 x 2  4 x  2


2

x  3
 x 2  7 x  12  0  
x  4
Với 1  x  2 ta có  x  5 2  x   2  x  1   x 2  3x  10  2 x 2  4 x  2
2

 97
x  1 t / m 
6
 3x  x  8  0  
2

 1  97
x   loai 
 6
1  97 
Vậy phương trình đã cho có ba nghiệm x   
;3; 4 .

 6 

Câu 14. Giải phương trình: log 2 ( x  5)  log 2 ( x  2)  3

Điều kiện x  5 . Phương trình đã cho tương đương với


log 2 ( x  5)( x  2)  3  ( x  5)( x  2)  8
 x  6(t / m)
 x 2  3x  18  0  
 x  3(l )
Vậy phương trình đã cho có nghiệm là x  6.

x3
Câu 15. Giải phương trình:
1
2
log 2 x 2
 2 x  3  log 2
x 3
0

Điều kiện: x  3  x  7

NGUYỄN VĂN LỰC  0933.168.309 SP Toán K35 - ĐH Cần Thơ


Hàm số mũ – hàm số logarit FB: http://www.facebook.com/VanLuc168
x 7
Phương trình  log 2 (x 2  2x  3)  log 2 0
x 3
(x 2  2x  3).(x  3)
 log 2 0
x 7
(x 2  2x  3).(x  3)
 1
x 7
 x 3  5x 2  2x  2  0  (x  1)(x 2  4x  2)  0
 x  1  x  1
 2  
 x  4x  2  0  x  2  6  x  2  6
So với điều kiện, phương trình có nghiệm x  2  6 .

Câu 16. Giải phương trình 2 log8  2 x   log 8  x 2  2 x  1 


4
3

Điều kiện x  0, x  1 .
Với điều kiện đó, PT đã cho tương đương với :
4  2 x  x  1  4
log8  2 x   x  1    2 x  x  1   16  
2
 x2
2 2

3  2 x  x  1  4

4
Câu 17.  2  log3 x  log9 x 3  1
1  log3 x

4
Giải phương trình  2  log3 x  log9 x 3   1 (1)
1  log3 x


x  0
ĐKXĐ: 
 x  3 (*)
 1
x 
 9

1 4 2  log3 x
Với ĐK (*), ta có : (1)   2  log3 x   1  
4
1 (2)
log3 9 x 1  log3 x 2  log3 x 1  log3 x

t  1
Đặt: t  log 3 x ( ĐK:  (**) ). Khi đó phương trình (2) trở thành:
t  2

t  1  1
2t 4  t  1  x 
  t  2   3
2  t 1 t t 2  3t  4  0 t  4 
  x  81

1
So sánh điều kiện được 2 nghiệm x  ; x  81
3

NGUYỄN VĂN LỰC  0933.168.309 SP Toán K35 - ĐH Cần Thơ


Hàm số mũ – hàm số logarit FB: http://www.facebook.com/VanLuc168
Câu 18. Giải phương trình: log 2  x  1  3log 1  3x  2   2  0
8

Điều kiện: x  1
Khi đó phương trình đã cho tương đương với phương trình
log 2  x  1  log 2  3x  2   2  0  log 2  4 x  4   log 2  3x  2 

 4 x  4  3x  2  x  2

Kết hợp với điều kiện phương trình có nghiệm x  2 .

Câu 19. Giải phương trình: log3 x 2  x   log 1 x  4   1 .


3

x  1
Điều kiện: 
 4  x  0
     
log 3 x 2  x  log 3 x  4  1  log 3 x 2  x  log 3 x  4  log 3 3 

 log 3 x  x  log 3
2
 3  x  4    x
 
2
 x  3 x  4 
x   2
 x 2  4x  12  0   (thoả mãn)
x  6
Vậy phương trình có hai nghiệm x  2; x  6 .

Câu 20. Giải phương trình: log3 ( x 2  3x)  log 1 (2 x  2)  0 ; ( x  )


3

Đk: x>0 (*)

Với Đk(*) ta có: (1)  log3 ( x2  3x)  log3 (2 x  2)

 x  1(t / m)
 x2  x  2  0   . Vậy nghiệm của PT là x = 1.
 x  2(loai)

Câu 21. Giải phương trình: log 22 x  4 log 4 4 x  7 .

Đk: x>0, log 22 x  4 log 4 4x  7  0  log 22 x  2 log 2 x  3  0


x  2
log 2 x  1  1
log x  3   1. Đối chiếu điều kiện ta được nghiệm của PT là x  2 và x  .
 2 x 8
 8
NGUYỄN VĂN LỰC  0933.168.309 SP Toán K35 - ĐH Cần Thơ
Hàm số mũ – hàm số logarit FB: http://www.facebook.com/VanLuc168

Câu 22. Giải phương trình: log32 x  log32 x  1  5  0

Điều kiện: x  0.
Đặt t  log 32 x  1, t  1.
t  2
Phương trình trở thành t 2  t – 6  0  
t  3  loai 
 log 3 x  3  x3 3
Với t = 2 thì log32 x  1  2  log 32 x  3    (tmđk).
 3
log x   3  x  3 3

Vậy phương trình có hai nghiệm x  3 3 và x  3 3

4
Câu 23. Giải phương trình:  2  log3 x  log 9 x 3  1
1  log 3 x

Điều kiện x  0, x  3, x  1/ 9
1 4 2  log 3 x 4
Phương trình   2  log3 x   1   1
log 3 9 x 1  log 3 x 2  log 3 x 1  log 3 x

Câu 24. Giải phương trình sau log 2 x  log 4 x  log8 x  11

log 2 x  log 4 x  log8 x  11 (1)


Điều kiện: x > 0.
(1)  log 2 x  log 22 x  log 23 x  11
1 1
 log 2 x  log 2 x  log 2 x  11
2 3
11
 log 2 x  11
6
 log 2 x  6  x  26  64 (nhan)
Vậy phương trình có nghiệm x = 64.

1
Câu 25. Giải phương trình sau log5 x  log 25 x  log 0,2
3

1
log5 x  log 25 x  log 0,2 (1)
3
Điều kiện: x > 0.

NGUYỄN VĂN LỰC  0933.168.309 SP Toán K35 - ĐH Cần Thơ


Hàm số mũ – hàm số logarit FB: http://www.facebook.com/VanLuc168

 3
1 1
(1)  log 5 x  log 52 x  log 51  log5 x  log5 x  log5 3
2
3 2
 log 5 x  log 5 3  log 5 x  log 5 3
2 3

 
2

 log 5 x  log 5 3 3
 log 5 x  log 5 3 3

x33
Vậy phương trình có nghiệm x  3 3 .

Câu 26. Giải phương trình sau log 22 x  log 2 x  6  0

log 22 x  log 2 x  6  0 (1)


Điều kiện: x > 0.
t  3
Đặt t  log 2 x . PT (1) trở thành t 2  t  6  0  
t  2
t  3  log 2 x  3  x  23  8 (thỏa mãn)
t  2  log 2 x  2  x  22  4 (thỏa mãn)
Vậy phương trình có nghiệm x = 4 và x = 8.

Câu 27. Giải phương trình sau 4 log 22 x  log 2 x  2

4 log 22 x  log 2
x  2 (1)
Điều kiện x > 0.
(1)  4 log 22 x  log 1 x  2  4 log 22 x  2 log 2 x  2  0 (1’)
22

t  1
Đặt t  log 2 x . PT (1’) trở thành 4t  2t  2  0   1
2
t 
 2
1
t  1  log 2 x  1  x  21  (t / m)
2
1
1 1
t   log 2 x   x  2  2 (t / m)
2

2 2
1
Vậy phương trình có nghiệm x  và x  2
2

Câu 28. Giải phương trình sau 3log 32 x  10 log 3 x  3

NGUYỄN VĂN LỰC  0933.168.309 SP Toán K35 - ĐH Cần Thơ


Hàm số mũ – hàm số logarit FB: http://www.facebook.com/VanLuc168
3log x  10 log 3 x  3 (1)
2
3

Điều kiện x > 0


t  3
Đặt t  log 3 x ta được 3t  10t  3  3t  10t  3  0   1
2 2
t 
 3
t  3  log 3 x  3  x  33  27 (nhận)
1
1 1
t   log3 x   x  33  3 3
3 3

Vậy phương trình có hai nghiệm x = 27 và x  3 3 .

Câu 29. Giải phương trình sau ln( x2  6 x  7)  ln( x  3)

ln( x 2  6 x  7)  ln( x  3) (1)


 x2  6x  7  0
Điều kiện 
x  3  0
 x  2 (loai)
(1)  x 2  6 x  7  x  3  x 2  7 x  10  0  
 x  5 (nhan)
Vậy phương trình có nghiệm x = 5.

Câu 30. Giải phương trình: log 4  x  1  2  log 4  x  log8  4  x  (1)


2 3
2

x 1  0
  4  x  4
Điều kiện: 4  x  0  
4  x  0  x  1

(1)  log 2 x  1  2  log 2  4  x   log 2  4  x   log 2 x  1  2  log 2 16  x 2 
 log 2 4 x  1  log 2 16  x 2   4 x  1  16  x 2
+ Với 1  x  4 ta có phương trình x 2  4 x  12  0 (2) ;
x  2
(2)  
 x  6  lo¹i 
+ Với 4  x  1 ta có phương trình x2  4 x  20  0 (3);
 x  2  24
  
3 
 x  2  24  lo¹i 
Vậy phương trình đã cho có hai nghiệm là x  2 hoặc x  2 1  6 .  
NGUYỄN VĂN LỰC  0933.168.309 SP Toán K35 - ĐH Cần Thơ
Hàm số mũ – hàm số logarit FB: http://www.facebook.com/VanLuc168
Câu 31. Giải phương trình: log x x  14log16 x x  40.log 4 x x  0
2 3

log x x 2  14log16 x x3  40.log 4 x x  0 (1)


2

Đk: x  0, x  1/ 4, x  1/16, x  2 (*)


Khi đó, phương trình tương đương với 2.log x x  42.log16 x x  20.log 4 x x  0 (2)
2

Nhận thấy x =1 luôn là nghiệm của PT.


2 42 20
Với 0 < x ≠ 1, PT (2)    0
x log x 16 x log x 4 x
log x
2
2 42 20
Đặt t = logx2, phương trình trên trở thành   0 (3)
1  t 1  4t 1  2t
(3)  2t2 + 3t – 2 = 0  t = 1/2 hoặc t = -2(tmđk)
1
* Với t = -2 thì logx2 = -2  x  
2
* Với t = 1/2 thì logx2 = 1/2  x = 4.
1
Kết hợp đk ta được nghiệm của phương trình là x = 4, x =
2

x2  x  1
Câu 32. Giải phương trình log 3 2  x 2  3x  2
2x  2x  3

Đặt u  x2  x  1; v  2 x2  2 x  3  u  0, v  0  suy ra v – u  x2  3x 2.


u
PT đã cho trở thành log3  v  u  log3 u  log3 v  v  u  log3 u  u  log3 v  v (1). Xét
v
hàm đặc trưng: f  t   log3 t  t , t  0 .
1
Ta có f ' (t )   1  0, t  0 nên hàm số đồng biến khi t > 0.
t.ln 3
Từ (1) ta có f(u) = f(v), suy ra u = v hay v-u=0, tức là x2-3x+2=0.
Phương trình có nghiệm x  1, x  2 .

NGUYỄN VĂN LỰC  0933.168.309 SP Toán K35 - ĐH Cần Thơ


Hàm số mũ – hàm số logarit FB: http://www.facebook.com/VanLuc168

IV. BẤT PHƯƠNG TRÌNH MŨ & LOGARIT


Chuyên đề: Hàm số mũ – hàm số logarit

1. CÁC PHƯƠNG PHÁP GIẢI BẤT PHƯƠNG TRÌNH MŨ & LOGARIT:


a. Phương pháp 1: Biến đổi phương trình về dạng cơ bản : aM < aN ( , ,  )
loga M  loga N ( , ,  )

Ví dụ 1: Giải bất phương trình 3x


2
x
9 (1)
Bài giải
♥ Ta có:
2

1 3x x
32

x2 x 2

x2 x 2 0

1 x 2

♥ Vậy tập nghiệm của bất phương trình là S 1; 2 


Tự luyện: Giải các bất phương trình
6 x 3 4 x 2 15 x 13
2 x 1 1
1) 3 x
 27
3
2) 23 x 4

Ví dụ 2: Giải bất phương trình 2 log3  4x  3   log 1 2x  3   2 (1)


3

Bài giải
x  3
4x  3  0  4 3
♥ Điều kiện:   x (*)
2x  3  0 x   3 4

 2
♥ Khi đó:
1  log 3  4x  3 2  2  log 3 2x  3 
 log 3  4x  3   log 3 9  2x  3 
2

  4x  3   9  2x  3 
2

 16x 2  42x  18  0
3
 x3
8
3
♥ So với điều kiện ta được nghiệm của bpt(1) là x3 
4

NGUYỄN VĂN LỰC  0933.168.309 SP Toán K35 - ĐH Cần Thơ


Hàm số mũ – hàm số logarit FB: http://www.facebook.com/VanLuc168
Tự luyện: Giải các bất phương trình sau
1) log 2 2 x 3 log 2 3x 1 2) log 1 5x 10 log 1 x2 6x 8
2 2

x4
3) log 1  log 1 (3  x) 4) log 2 x 3 log 2 x 2 1
3
2x  3 3

5) log 1 (x2  6x  5)  2 log3 (2  x)  0 6) log 1 x  2 log 1  x  1  log2 6  0


3 2 4

7) log 1  x    log 2 ( x  1)  1
1
8) log 1 x2 5x 6 1
2  2  2

x 2  3x  2
Ví dụ 3: Giải bất phương trình log 1 0 (1)
2 x
Bài giải
x 2  3x  2 0  x  1
♥ Điều kiện: 0 (*)
x  x  2
♥ Khi đó:
x 2  3x  2
1  log 1  log 1 1
2 x 2

x  3x  2
2
 1
x
x 2  4x  2
 0
x
x  0

2  2  x  2  2
2  2  x  1
♥ So với điều kiện ta được nghiệm của bpt(1) là  
2  x  2  2

Tự luyện: Giải các bất phương trình sau
2x  1 3x  5
1) log2 0 2) log3 1
x 1 x 1
2x  1 3x  1
3) log 0,5 2 4) log 1 1
x5 3
x2

 x2  x 
Ví dụ 4: Giải bất phương trình: log0,7  log6 0 (1)
 x  4 

Bài giải
♥ Điều kiện:
 x2  x  x2  x
 x  4  0  x  4  0  4  x  2
x2  x x2  4
     1   0   (*)
log x  x  0
2
x  x  1
2
x4 x4 x2

 6
x4  x  4
♥ Khi đó:

NGUYỄN VĂN LỰC  0933.168.309 SP Toán K35 - ĐH Cần Thơ


Hàm số mũ – hàm số logarit FB: http://www.facebook.com/VanLuc168
 x x
2
x2  x
1  log0,7  log6  log 1  log 1
x  4 
0,7 6
 x4
x2  x x2  x
 log6  log6 6  6
x4 x4
x2  5x  24  4  x  3
 0
x4  x  8
 4  x   3
♥ So với điều kiện ta được nghiệm của bpt(1) là  
 x  8
2x  3 
Tự luyện: Giải bất phương trình log 1  log2 0
3  x 1 

b. Phương pháp 2: Đặt ẩn phụ chuyển về bất phương trình đại số.

Ví dụ 5: Giải bất phương trình 9 x 1  36.3x 3  3  0 (1)


Bài giải
♥ Biến đổi bất phương trình (1) ta được
1 2
1 3x 4.3x 1
3 0 (2)

♥ Đặt t 3x 1 t 0 , bất phương trình (2) trở thành t 2 4t 3 0 (3)


3 1 t 3

Suy ra: 1 3x 1
3 0 x 1 1 1 x 2

♥ Vậy tập nghiệm của bất phương trình là S 1; 2 

Bài giải
1) 22x - 3.2x+2 + 32 < 0 2) 2 x 23 x
9

3) 9 x  5.3x  6  0 4) 52x1  5x  4
2x  x2
1
5) 9 x2 2x
 2   3 6) 32x1  22x 1  5.6x  0
3

Ví dụ 6: Giải bất phương trình log22 x  log2 x  2  0 (1)

Bài giải
♥ Điều kiện: x 0
♥ Đặt t log 2 x , bất phương trình (1) trở thành t 2 t 2 0 (2)
3 2 t 1

NGUYỄN VĂN LỰC  0933.168.309 SP Toán K35 - ĐH Cần Thơ


Hàm số mũ – hàm số logarit FB: http://www.facebook.com/VanLuc168
1
Suy ra: 2 log 2 x 1 x 2
4

1
♥ Vậy tập nghiệm của bất phương trình là S ;2 
4

Tự luyện: Giải các phương trình sau


1) log 2 2 x 17 log 2 x  4  0 2) 3.log32 x 14.log 3 x  3  0
4
3) log 2 x  2 log x 4  5  0 4) log 21 ( x  1)  3   log 1 ( x  1)5
3
5 3

5) 3. log 1 x  log 4 x  2  0
2
6) log x log 1 x 2 0
2
1
2 2 2

NGUYỄN VĂN LỰC  0933.168.309 SP Toán K35 - ĐH Cần Thơ


Hàm số mũ – hàm số logarit FB: http://www.facebook.com/VanLuc168

BÀI TẬP TỰ LUYỆN

Bất phương trình mũ


x 1

Câu 1. Giải bất phương trình:  


1
 2 2 x
2  

BPT  2 x1  2 2 x   x 1  2x  x  1

Câu 2. Giải bất phương trình: 3.9 x  10.3x  3  0 .

Đặt t  3x (t  0) . Bất phương trình đã cho trở thành


1
3t 2  10t  3  0  t 3
3
1
Suy ra  3 x  3  1  x  1 .
3
Vậy bất phương trình có tập nghiệm là S  [1;1] .

x 2 1
1 3
Câu 3. Giải bất phương trình: 22x 1    .
8  

Bất phương trình tương đương với


x 2 1
22x 1   
23 3  22x 1  2x
2
1
 2x  1  x 2  1

 x 2  2x  0  2  x  0 . Vậy bất phương trình có tập nghiệm S  2; 0 .  


8 x 3
Câu 4. Giải bất phương trình: 2 x2
4 x 1

8 x 3 4 2 x 6
4 2x  6
2 x 2  4 x 1  2 x 2  2  x 1

x2 x 1
2  x  1 x  4   4  x  1
 0
 x  2  x  1 1  x  2

3x 7
 49
2
Câu 5. Giải bất phương trình sau: 76 x

3x 7 3 x 7
 49  76 x  72  6 x 2  3x  7  2  6 x 2  3x  9  0
2 2
76 x

NGUYỄN VĂN LỰC  0933.168.309 SP Toán K35 - ĐH Cần Thơ


Hàm số mũ – hàm số logarit FB: http://www.facebook.com/VanLuc168
x  1
VT  0  6 x 2  3x  9  0  
 x  3
Xét dấu VT ta được tập nghiệm của bất phương trình S = [-3; 1].

Câu 6. Giải bất phương trình: 4x  3.2x  2  0

Bất phương trình 4x  3.2x  2  0  22 x  3.2x  2  0


Đặt t  2 x , t  0
Bất phương trình trở thành: t 2  3t  2  0  1  t  2  1  2 x  2  0  x  1
Vậy bất phương trình có nghiệm S = (0; 1).

log2 x 2log2 x
Câu 7. Giải bất phương trình 2 2 x  20  0

Điều kiện: x> 0 ; BPT  24log2 x  x2log2 x  20  0


2

Đặt t  log 2 x . Khi đó x  2t .


BPT trở thành 42t  22t  20  0 . Đặt y  22t ; y  1.
2 2 2

BPT trở thành y2 + y - 20  0  - 5  y  4.


Đối chiếu điều kiện ta có: 22t  4  2t 2  2  t 2  1  - 1  t  1.
2

1
Do đó - 1  log 2 x  1   x  2.
2

4
Câu 8. Giải bất phương trình (2  3) x 2 x1  (2  3) x 2 x1 
2 2

2 3

   
x2 2 x x2 2 x
Bpt  2  3  2 3 4

t  2  3
x2 2 x
Đặt (t  0)
1
BPTTT: t   4  t 2  4t  1  0  2  3  t  2  3 (tm)
t

 
x2 2 x
2 3  2 3  2  3  1  x 2  2 x  1  x2  2 x  1  0  1  2  x  1  2

NGUYỄN VĂN LỰC  0933.168.309 SP Toán K35 - ĐH Cần Thơ


Hàm số mũ – hàm số logarit FB: http://www.facebook.com/VanLuc168
Bất phương trình logarith

Câu 1. Giải bất phương trình: log0,2 x  log0,2(x  1)  log0,2(x  2) .

Điều kiện: x  0 (*).


log0,2 x  log0,2 (x  1)  log0,2 (x  2)  log0,2(x2  x)  log0,2(x  2)

 x2  x  x  2  x  2 (vì x > 0).


Vậy bất phương trình có nghiệm x  2 .

Câu 2. Giải bất phương trình : log 1 log 2 (2  x 2 )   0 ( x  R) .


2

Điều kiện: log 2 (2  x 2 )  0  2  x 2  1  1  x  1


 1  x  1
  1  x  1  1  x  1
Khi đó (2)  log 2 (2  x 2 )  1     
2  x  2 x  0  x0
2 2

Vậy tập nghiệm bpt là S  (1;0)  (0;1)

Câu 3. Giải bất phương trình: 2log3 ( x  1)  log 3 (2 x  1)  2

ĐK: x > 1 , 2 log3 ( x  1)  log 3 (2 x  1)  2  log 3[( x  1)(2 x  1)]  1


1
 2 x 2  3x  2  0   x2 => tập nghiệm S = (1;2]
2

Câu 4. Giải bất phương trình: log5  4 x  1  log5  7  2 x   1  log 1 3x  2 


5

1 7
+ Điều kiện:   x 
4 2
+ BPT  log5  4 x  1  log5  3x  2   1  log 5  7  2 x 
 log 5  4 x  1 3 x  2   log 5 5  7  2 x 
  4 x  1 3 x  2   5  7  2 x 
 12 x 2  21x  33  0
33
  x 1
12
1
Giao với điều kiện, ta được:   x  1
4
1
Vậy: nghiệm của BPT đã cho là   x  1
4

Câu 5. Giải bất phương trình sau:


1  log 2 x  log 2  x  2   log 2
6  x

NGUYỄN VĂN LỰC  0933.168.309 SP Toán K35 - ĐH Cần Thơ


Hàm số mũ – hàm số logarit FB: http://www.facebook.com/VanLuc168
ĐK: 0  x  6 . BPT  log2  2 x  4 x   log2  6  x  .
2 2

Hay: BPT  2 x2  4 x   6  x 2  x2  16 x  36  0
Vậy: x  18 hay 2  x
So sánh với điều kiện.
KL: Nghiệm BPT là 2  x  6 .

Câu 6. Giải bất phương trình log 2  2 x  1  log 1  x  2   1 .


2

- ĐK: x  2

- Khi đó bất phương trình có dạng: log 2  2 x  1  log 2  x  2   1

 log 2  2 x  1 x  2    1


 5
 2 x 2  5 x  0  x  0; 
 2

- Kết hợp điều kiện ta có: x   2; 


5
 2

Câu 7. Giải bất phương trình sau: log3 log 2  x  2   3log 25 4.log 8 5

log 3 log 2  x  2    3log 25 4.log 8 5  log 3 log 2  x  2    log 3 3

log 2  x  2   3
  4  x  10
log 2  x  3  0

Câu 8. Giải bất phương trình sau : log 2 ( x 2  1)  log 1 ( x  1) .


2

ĐK: x >1. BPT


log 2 ( x 2  1)  log 1 ( x  1)  log 2 ( x 2  1)  log 2 ( x  1)  0
2

 ( x 2  1)( x  1)  1  x3  x 2  x  1  1  x( x 2  x  1)  0
1 5
 x (do x >1).
2
1  5 
Vậy tập nghiệm của BPT là S=  ;   .
 2 

x
Câu 9. Giải bất phương trình log 22 x  log 2  4
4

x
Giải bất phương trình log 22 x  log 2  4 (1)
4
Điều kiện của bất phương trình (1) là: x  0 (*)

NGUYỄN VĂN LỰC  0933.168.309 SP Toán K35 - ĐH Cần Thơ


Hàm số mũ – hàm số logarit FB: http://www.facebook.com/VanLuc168
Với điều kiện (*),
(1)  log 22 x  log 2 x  log 2 4  4  log 22 x  log 2 x  2  0  (log 2 x  2)(log 2 x  1)  0
 x4
 log 2 x  2
 
 2
log x   1 0  x  1
 2

Kết hợp với điều kiện (*), ta có tập nghiệm của bất phương trình (1) là S   0;    4;  
1
 2

 x3   32 
Câu 10. Giải bất phương trình log x  log    9log 2  2   4log 21 x
4
2
2
1
8 x  2 2

Điều kiện x > 0.


2
Bất phương trình  log 42 ( x)  log 2 x3  log 2 8  9 log 2 32  log 2 x2   4log 22 ( x)
 log 42 ( x)  3log 2 x  3  9 5  2log 2 x  4log 22 ( x)
2

Đặt t = log2(x), bất phương trình trên tương đương với


  1 1
 3  t  2  3  log 2 x  2  x
t - 13t + 36 < 0  4  t  9 
4 2 2
  8 4
 2t 3  2  log 2 x  3 
  4 x8

Vậy bất phương trình có nghiệm  ,    4,8  .


1 1
8 4  

1 
Câu 11. Giải bất phương trình log 2 (4 x 2  4 x  1)  2 x  2  ( x  2)log 1   x 
2 
2

 1
1  1
 x
 x0 x   1
ĐK:  2  2 
2
 x  *
4 x 2  4 x  1  0 (2 x  1) 2  0 x  1 2
   2
Với điều kiện (*) bất phương trình tương đương với:
2log 2 (1  2 x)  2 x  2  ( x  2) log 2 (1  2 x)  1  x log 2 (1  2 x)  1  0
 x  0  x  0  x  0
    1
 log 2 (1  2 x)  1  0  log 2 2(1  2 x)  0  2(1  2 x)  1  x 
    4
 x  0  x  0  x  0 
   x  0
 log 2 (1  2 x)  1  0  log 2 2(1  2 x)  0  2(1  2 x)  1
1 1
Kết hợp với điều kiện (*) ta có:  x  hoặc x < 0.
4 2

Câu 12. Giải bất phương trình: log2 (x  1)  log 1 (x  3)  5.


2

Điều kiện: x  1.
NGUYỄN VĂN LỰC  0933.168.309 SP Toán K35 - ĐH Cần Thơ
Hàm số mũ – hàm số logarit FB: http://www.facebook.com/VanLuc168
BPT  log2 (x  1)  log2 (x  3)  5  log2 (x  2x  3)  5
2

 x2  2x  35  0  7  x  5
Kết hợp điều kiện ta được: 1  x  5 là nghiệm của bất phương trình.
Vậy nghiệm của bất phương trình đã cho là: 1  x  5.

Câu 13. Giải bất phương trình:


log 2 2 ( x  1)  log 2 ( x 2  2 x  1)  3  0

log 22 ( x  1)  log 2 ( x 2  2 x  1)  3  0  log 2 2 ( x  1)  2log 2 ( x  1)  3  0


Đặt t = log2(x+1) ta được : t2 – 2t – 3 > 0 <=> t < -1 hoặc t > 3.

 1  1
log 2 ( x  1)  1 0  x   1  x  
Vậy: log 2 ( x  1)  3   2

2
x 1  8 x  7

Câu 14. Giải bất phương trình sau: log 3 (4 x  3)  2

log 3 (4 x  3)  2
3
Điều kiện 4 x  3  0  x 
4
log3 (4 x  3)  2  4 x  3  32  4 x  12  x  3

Kết hợp điều kiện, bất phương trình có nghiệm S   ;3 


3
4 

Câu 15. Giải bất phương trình sau: log0,5 ( x2  5x  6)  1

log0,5 ( x2  5x  6)  1
x  2
Điều kiện x 2  5 x  6  0  
x  3
log0,5 ( x2  5x  6)  1  x 2  5x  6   0,5  x 2  5x  4  0  1  x  4
1

Kết hợp điều kiện bất phương trình có nghiệm S  1;2    3;4

Câu 16. Giải bất phương trình sau: log 1 (2 x  4)  log 1 ( x 2  x  6)


3 3

log 1 (2 x  4)  log 1 ( x 2  x  6)
3 3

 x  2
2 x  4  0 
Điều kiện:  2    x  2  x  3
x  x  6  0  x  3


NGUYỄN VĂN LỰC  0933.168.309 SP Toán K35 - ĐH Cần Thơ


Hàm số mũ – hàm số logarit FB: http://www.facebook.com/VanLuc168
log 1 (2 x  4)  log 1 ( x  x  6)  2 x  4  x  x  6
2 2

3 3

 x  3x  10  0  2  x  5
2

Kết hợp với điều kiện, bất phương trình có nghiệm S   3;5

Câu 17. Giải bất phương trình sau: l o g(7 x  1)  l o g(10 x 2  11x  1)

 1
 x  
7
7 x  1  0   1 1 
Điều kiện:    1  x   ;   1;  
10 x  11x  1  0   x  10  7 10 
2


 x  1
l o g(7 x  1)  l o g(10 x 2  11x  1)  7 x  1  10 x 2  11x  1
9
 10 x 2  18 x  0  0  x 
5

Kết hợp điều kiện, bất phương trình có nghiệm S  0;


1   9
  1;
 10   5 

Câu 18. Giải bất phương trình:


1
log3 x 2  5 x  6  log 1 x  2  log 1  x  3
3
2 3

Điều kiện: x  3
Bất phương trình đã cho tương đương:
log3  x 2  5 x  6   log 31  x  2   log 31  x  3
1 1 1
2 2 2
 log3  x 2  5 x  6   log3  x  2    log 3  x  3
1 1 1
2 2 2
 log 3  x  2  x  3   log 3  x  2   log 3  x  3
 x2 x2
 log 3  x  2  x  3   log 3     x  2  x  3 
 x3 x3
 x   10
 x2  9  1  
 x  10
Giao với điều kiện, ta được nghiệm của phương trình đã cho là x  10 .

Câu 19. Giải bất phương trình:


log 1 log5
3
 
x 2  1  x  log3 log 1
5
 x2  1  x 
Đk: x  0

NGUYỄN VĂN LỰC  0933.168.309 SP Toán K35 - ĐH Cần Thơ


Hàm số mũ – hàm số logarit FB: http://www.facebook.com/VanLuc168

1  log3 log 1 


5

x 2  1  x  log 3 log 5  
x2  1  x  0


 log 3  log 1
 5
 x 2  1  x .log 5  

x2  1  x   0

 log 52  x2  1  x  1 
 0  log 5  x2  1  x  1 
*) 0  log 5  
x2  1  x  x  0

*) log5  
x 2  1  x  1  x 2  1  x  5  x 2  1  5  x  ...  x 
12
5

Vậy BPT có nghiệm x   0; 


12
 5

Câu 20. Giải bất phương trình x(3log 2 x  2)  9log 2 x  2

Điều kiện: x  0 Bất phương trình  3( x  3) log 2 x  2( x  1)


Nhận thấy x=3 không là nghiệm của bất phương trình.
3 x 1
TH1 Nếu x  3 BPT  log 2 x 
2 x 3
3 x 1
Xét hàm số: f ( x)  log 2 x đồng biến trên khoảng  0;  g ( x)  nghịch biến
2 x 3
f ( x)  f (4)  3
trên khoảng  3;   *Với x  4 :Ta có   Bpt có nghiệm x  4 * Với
g ( x)  g (4)  3 
f ( x)  f (4)  3
x  4 :Ta có   Bpt vô nghiệm
g ( x)  g (4)  3 
3 x 1 3
TH 2:Nếu 0  x  3 BPT  log 2 x  f ( x)  log 2 x đồng biến trên  0;  ;
2 x 3 2
x 1 f ( x)  f (1)  0
g ( x)  nghịch biến trên  0;3 *Với x  1 :Ta có   Bpt vô
x 3 g ( x)  g (1)  0 
nghiệm
f ( x)  f (1)  0
 Với x  1:Ta có   Bpt có nghiệm 0  x  1
g ( x)  g (1)  0 
x  4
Vậy bất phương trình có nghiệm  .
0  x  1

NGUYỄN VĂN LỰC  0933.168.309 SP Toán K35 - ĐH Cần Thơ


Nguyên hàm – Tích phân FB: http://www.facebook.com/VanLuc168

I. NGUYÊN HÀM
Chuyên đề: Nguyên hàm – Tích phân

1. Bảng tính nguyên hàm cơ bản


Bảng 1 Bảng 2
Hàm số f(x) Họ nguyên hàm Hàm số f(x) Họ nguyên hàm F(x)+C
F(x)+C
a ( hằng số) ax + C
x 1 (ax  b) 1 (ax  b) 1
C C
x 1  1 a  1
1 ln x  C 1 1
ln ax  b  C
x ax  b a
ax ax Aax  b 1 Aax b
C . C
ln a A ln a
ex ex  C eax  b 1 ax  b
e C
a
sinx -cosx + C sin(ax+b) 1
 cos(ax  b)  C
a
cosx sinx + C cos(ax+b) 1
sin(ax  b)  C
a
1 tanx + C 1 1
tan(ax  b)  C
cos2 x 2
cos (ax  b) a
1 -cotx + C 1 1
 cot(ax  b)  C
sin2 x 2
sin (ax  b) a
u' ( x ) ln u( x )  C 1 1 xa
ln C
u( x ) x  a2
2
2a x  a
tanx  ln cos x  C
cotx ln sin x  C

2. Các phương pháp tìm nguyên hàm của hàm số

Phương pháp 1: Sử dụng định nghĩa và tính chất kết hợp với bảng tính các
nguyên hàm cơ bản
 Phân tích hàm số đã cho thành tổng, hiệu của các hàm số đơn giản có công
thức trong bảng nguyên hàm cơ bản.

 Cách phân tích : Dùng biến đổi đại số như mũ, lũy thừa, các hằng đẳng thức
... và biến đổi lượng giác bằng các công thức lượng giác cơ bản.

NGUYỄN VĂN LỰC  0933.168.309 SP Toán K35 - ĐH Cần Thơ


Nguyên hàm – Tích phân FB: http://www.facebook.com/VanLuc168
Phương pháp 2: Phương pháp đổi biến số
Định lí cơ bản:
Nếu f u du F u C và u u x là hàm số có đạo hàm liên tục thì
f u x u ' x dx F u x C
Cách thực hiện: Tính  f  u(x) u'(x)dx bằng pp đổi biến số
Bước 1: Đặt u  u(x)  du  u '(x)dx (tính vi phân của u)
Bước 2: Tính  f  u(x) u'(x)dx   f(u)du  F(u)  C  F  u(x)  C

Phương pháp 3: Phương pháp tính nguyên hàm từng phần


Định lí cơ bản:
Nếu hai hàm số u u x và v v x có đạo hàm liên tục trên K thì
u x v ' x dx u x v x u ' x v x dx

Cách thực hiện:


u  u ( x) du  u ' ( x)dx
Bước 1: Đặt 
dv  v' ( x)dx v  v( x)

Bước 2: Thay vào công thức nguyên hàm từng phần :  udv  u.v   vdu

Bước 3: Tính  vdu

B. Bài tập
Bài 1: Tính
x2 3 x  1 2 x3  3x
1) I   dx 2) I   dx 3) I   dx
x2 x 1 x2

Bài 2: Tính
3x  2 x 2 1 x
1)  3  x dx 2) I  
x  x  1
dx 3) I  
x  3x  2
2
dx

Bài 3: Tính
ln x
1) I   x ln xdx 2) I   dx 3) I   x3 ln xdx
x

Bài 4: Tính
1) I   ln  x 2  x  dx 2) I    x  2  e2 x dx 3) I   x s in2xdx

Bài 5: Tính
x sin x ex
1) I   dx 2) I   dx 3) I   cos5 xdx
cos 2 x 1  2e x

NGUYỄN VĂN LỰC  0933.168.309 SP Toán K35 - ĐH Cần Thơ


Nguyên hàm – Tích phân FB: http://www.facebook.com/VanLuc168

II. TÍCH PHÂN


Chuyên đề: Nguyên hàm – Tích phân

A. Tóm tắt lí thuyết


I. CÁC PHƯƠNG PHÁP TÍNH TÍCH PHÂN
1. SỬ DỤNG ĐN VÀ CÁC TÍNH CHẤT TÍCH PHÂN
a. Định nghĩa: Cho hàm số y = f(x) liên tục trên K và a, b  K. Giả sử F(x) là một
nguyên hàm của hàm số f(x) trên K thì:

b
b
 f ( x)dx   F( x)a  F(b)  F(a) ( Công thức NewTon - Leipniz)
a

b. Các tính chất của tích phân


b a
 Tính chất 1:  f ( x )dx    f ( x )dx
a b

 Tính chất 2: Nếu hai hàm số f(x) và g(x) liên tục trên  a; b  thì
b b b

  f ( x)  g( x) dx   f ( x)dx   g( x)dx


a a a
 Tính chất 3: Nếu hàm số f(x) liên tục trên  a; b  và k là một hằng số thì
b b

 k. f ( x)dx  k. f ( x)dx


a a
 Tính chất 4: Nếu hàm số f(x) liên tục trên  a; b  và c là một hằng số thì
b c b

 f ( x)dx   f ( x)dx   f ( x)dx


a a c
 Tính chất 5: Tích phân của hàm số trên  a; b  cho trước không phụ thuộc vào
biến số ,
b b b
nghĩa là:  f ( x )dx   f (t )dt   f (u)du  ...
a a a
2. PHƯƠNG PHÁP ĐỔI BIẾN SỐ
b
a) DẠNG 1: Tính I =  f[u(x)].u' (x)dx bằng cách đặt t = u(x)
a

 f u ( x).u ' ( x)dx   f (t )dt


b u (b )
Công thức đổi biến số dạng 1:
a u(a)

Cách thực hiện:

Bước 1: Đặt t  u ( x)  dt  u ' ( x)dx


NGUYỄN VĂN LỰC  0933.168.309 SP Toán K35 - ĐH Cần Thơ
Nguyên hàm – Tích phân FB: http://www.facebook.com/VanLuc168
xb t  u (b)
Bước 2: Đổi cận: 
xa t  u (a)
Bước 3: Chuyển tích phân đã cho sang tích phân theo biến t ta được
I   f u ( x).u ' ( x)dx   f (t )dt
b u (b )
(tiếp tục tính tích phân mới)
a u(a)
b
b) DẠNG 2: Tính I =  f(x)dx bằng cách đặt x = (t)
a

I   f ( x)dx   f  (t ) ' (t )dt
b
Công thức đổi biến số dạng 2
a 

Cách thực hiện

Bước 1: Đặt x   (t )  dx   ' (t )dt


xb t
Bước 2: Đổi cận: 
xa t 
Bước 3: Chuyển tích phân đã cho sang tích phân theo biến t ta được

I   f ( x)dx   f  (t ) ' (t )dt
b
(tiếp tục tính tích phân mới)
a 

3. PHƯƠNG PHÁP TÍCH PHÂN TỪNG PHẦN

Công thức tích phân từng phần

 u ( x).v' ( x)dx  u ( x).v( x)a   v( x).u ' ( x)dx


b b
b

a a

hay:  udv  u.v   vdu


b b
b
a
a a

Cách thực hiện

u  u ( x) du  u ' ( x)dx
Bước 1: Đặt 
dv  v' ( x)dx v  v( x)

Bước 2: Thay vào công thức tích phân từng từng phần:  udv  u.vba   vdu
b b

a a

Bước 3: Tính u.vba


b
và  vdu
a

II. CÁC VÍ DỤ
2
x2 3x 1
Ví dụ 1: Tính tích phân I 2
dx . (Phân tích & dùng định nghĩa)
1
x x

Bài giải
x2 3x 1 2x 1
♥ Biến đổi hàm số thành dạng 2
1
x x x2 x

NGUYỄN VĂN LỰC  0933.168.309 SP Toán K35 - ĐH Cần Thơ


Nguyên hàm – Tích phân FB: http://www.facebook.com/VanLuc168
2 2 2 2
x 3x 1 2x 1
Khi đó: I 2
dx dx dx
1
x x 1 1
x2 x
2
2
dx x1 1
1

2
2x 1 2
dx ln x 2 x ln 3
1
x2 x 1

♥ Vậy I 1 ln 3 . 

1 2
x 1
Ví dụ 2: Tính tích phân I dx . (Phân tích & dùng định nghĩa)
0
x2 1

Bài giải
2
x 1 x2 2x 1 2x
♥ Biến đổi hàm số thành dạng 2 2
1 2
x 1 x 1 x 1
1 2 1 1
x 1 2x
Khi đó: I 2
dx dx 2
dx
0
x 1 0 0
x 1

1
1
dx x0 1
0

1
2x 1
2
dx ln x 2 1 ln 2
0
x 1 0

♥ Vậy I 1 ln 2 . 

ln 2
2
Ví dụ 3: Tính tích phân I e x 1 e x dx . (Đổi biến số dạng 1)
0

Bài giải
♥ Đặt t ex 1 dt e x dx

x ln 2 t 1
Đổi cận:
x 0 t 0
1 1
2 t3 1
Suy ra: I t dt
0
3 0
3

1
♥ Vậy I . 
3

NGUYỄN VĂN LỰC  0933.168.309 SP Toán K35 - ĐH Cần Thơ


Nguyên hàm – Tích phân FB: http://www.facebook.com/VanLuc168
1

Ví dụ 4: Tính tích phân I x 2 x 2 dx . (Đổi biến số dạng 1)


0

Bài giải
♥ Đặt t 2 x2 t2 2 x2 2tdt 2 xdx tdt xdx

x 1 t 1
Đổi cận:
x 0 t 2

2 2
2 t3 2 2 1
Suy ra: I t dt
1
3 1
3

2 2 1
♥ Vậy I .
3

e
4 5ln x
Ví dụ 5: Tính tích phân I dx . (Đổi biến số dạng 1)
1
x

Bài giải
5
♥ Đặt t 4 5ln x t2 4 5ln x 2tdt dx
x
x e t 3
Đổi cận:
x 1 t 2
3 3
2 2 2 3 2 3 38
Suy ra: I t dt t 3 23
5 2
15 2 15 15

38
♥ Vậy I . 
15

4
Ví dụ 6: Tính tích phân I x 1 sin 2 xdx . (Tích phân từng phần)
0

Bài giải
du dx
u x 1
♥ Đặt 1
dv sin 2 xdx v cos 2 x
2

1 4 1 4
Suy ra: I x 1 cos 2 x sin 2 x
2 0 4 0

1 4 1 4 3
x 1 cos 2 x sin 2 x
2 0 4 0 4

3
♥ Vậy I . 
4
NGUYỄN VĂN LỰC  0933.168.309 SP Toán K35 - ĐH Cần Thơ
Nguyên hàm – Tích phân FB: http://www.facebook.com/VanLuc168

4
Ví dụ 7: Tính tích phân I x 1 sin 2 x dx . (Tích phân từng phần)
0

4 4 4 4
x2 4 2
♥ Ta có: I xdx x sin 2 xdx x sin 2 xdx x sin 2 xdx
0 0
2 0 0
32 0

du dx
u x
Đặt 1
dv sin 2 xdx v cos 2 x
2

4 4 4
1 4 1 1 1 4 1
Suy ra: x sin 2 xdx x cos 2 x cos 2 xdx cos 2 xdx sin 2 x
0
2 0 2 0
2 0
4 0 4

2
1
♥ Vậy I . 
32 4

2
x2 2 ln x
Ví dụ 8: Tính tích phân I dx . (Phân tích + đổi biến số dạng 1)
1
x

Bài giải
2 2
ln x
♥ Ta có: I xdx 2 dx
1 1
x

2 2
x2 3
xdx
0
2 1
2

2
ln x
♥ Tính dx
1
x

1
Đặt t ln x dt dx
x
x 2 t ln 2
Đổi cận:
x 1 t 0
2 ln 2 ln 2
ln x t2 ln 2 2
Suy ra: dx tdt
1
x 0
2 0
2

3
♥ Vậy I ln 2 2 . 
2

NGUYỄN VĂN LỰC  0933.168.309 SP Toán K35 - ĐH Cần Thơ


Nguyên hàm – Tích phân FB: http://www.facebook.com/VanLuc168
2 2
x 1
Ví dụ 9: Tính tích phân I 2
ln xdx . (Tích phân từng phần)
1
x

1
u ln x du dx
♥ Đặt x
x2 1
dv dx 1
x2 v x
x
2 2
1 1 1
Suy ra: I x ln x x dx
x 1 1
x x
2 2
1 1
x ln x x
x 1
x 1

5 3
ln 2
2 2
5 3
♥ Vậy I ln 2 . 
2 2

1 2
Ví dụ 10: Tính tích phân I = 0 (2ex  ex )xdx . (Phân tích + đổi biến dạng 1+ tích

phân từng phần)


Bài giải
1 x2 1
♥ Ta có: I = 0 2xe dx   xe x dx .
0

1 1 x2 1
x2
0 2xe dx  0 e d(x ) = e  0
x2 2
I1 = = e – 1.

1
0 xe dx
x
I2 =
Đă ̣t u = x  du = exdx
dv = exdx  v = ex.
1 1 1
Suy ra: I2 =  xex  0  0 ex dx = e  ex  0 = 1.

♥ Vâ ̣y I = e – 1 + 1 = e. 

NGUYỄN VĂN LỰC  0933.168.309 SP Toán K35 - ĐH Cần Thơ


Nguyên hàm – Tích phân FB: http://www.facebook.com/VanLuc168

III. TÍCH PHÂN HÀM PHÂN THỨC HỮU TỶ

Chuyên đề: Nguyên hàm – Tích phân


P( x)
A. DẠNG: I=  dx  a  0
 ax+b

m m 
* Chú ý đến công thức: 
 ax+b
dx  ln ax+b . Và nếu bậc của P(x) cao hơn hoắc bằng
a 
   
P( x) m 1
2 thì ta chia tử cho mẫu dẫn đến  ax+b dx   Q( x)  ax+b dx   Q( x)dx  m ax+b dx
2
x3
Ví dụ 1: Tính tích phân: I= 1 2 x  3 dx
Giải
3
x 1 3 9 27 1
Ta có: f ( x)   x2  x  
2x  3 2 4 8 8 2x  3
Do đó:
1 2 3 9 27 1  1 3 3 2 9 2
2 2
x3 27 13 27
1 2 x  3 1  2 x  4 x  8  8 2 x  3  dx   3 x  8 x  8 x  16 ln 2x  3  1   6  16 ln 35
dx 

x2  5
3
Ví dụ 2: Tính tích phân: I=  x  1 dx
5

Giải
x 52
4
Ta có: f(x)=  x 1 .
x 1 x 1
3
x2  5
3
 4  1  3  5 1 
Do đó:  x  1 dx    x  1  x  1  dx   2 x  x  4ln x  1   5  1  4ln  
2

5 5  5  4 


P( x)
B. DẠNG: 
 ax
2
 bx  c
dx

1. Tam thức: f ( x)  ax 2  bx  c có hai nghiệm phân biệt


u '( x) 
Công thức cần lưu ý: 
 u ( x)
dx  ln u ( x)

Ta có hai cách
Cách 1: ( Hệ số bất định )
Cách 2: ( Nhẩy tầng lầu )

NGUYỄN VĂN LỰC  0933.168.309 SP Toán K35 - ĐH Cần Thơ


Nguyên hàm – Tích phân FB: http://www.facebook.com/VanLuc168
4 x  11
1
Ví dụ 3: Tính tích phân: I= x
0
2
 5x  6
dx .

Giải
Cách 1: ( Hệ số bất định )
4 x  11 4 x  11 A B A  x  3  B  x  2 
Ta có: f(x)=    
x  5 x  6 ( x  2)( x  3) x  2 x  3
2
( x  2)( x  3)
Thay x=-2 vào hai tử số: 3=A và thay x=-3 vào hai tử số: -1= -B suy ra B=1
3 1
Do đó: f(x)= 
x2 x3
4 x  11  3 1 
1 1
1
Vậy: 0 x2  5x  6 dx  0  x  2  x  3  dx   3ln x  2  ln x  3  0  2 ln 3  ln 2
Cách 2: ( Nhẩy tầng lầu )
2  2 x  5  1 2x  5 1 2x  5 1 1
Ta có: f(x)=  2. 2   2. 2  
x  5x  6
2
x  5 x  6  x  2  x  3 x  5x  6 x  2 x  3
2x  5  x2  1
Do đó: I=  f ( x)dx    2.
1 
1 1
1
    
2
    2 ln 3  ln 2
x  3  0
dx 2 ln x 5 x 6 ln
0 0  x2  5x  6 x  2 x  3  

2. Tam thức: f ( x)  ax 2  bx  c có hai nghiệm kép


u '( x)dx 
Công thức cần chú ý:   ln  u ( x) 
 u ( x) 
Thông thừơng ta đặt (x+b/2a)=t .
3
x3
Ví dụ 4: Tính tích phân sau: I= 0 x 2  2 x  1 dx
Giải
3 3 3 3
x x
Ta có: x
0
2
 2x 1
dx  
0  x  1
2
dx

Đặt: t=x+1 suy ra: dx=dt ; x=t-1 và: khi x=0 thì t=1 ; khi x=3 thì t=4 .
 t  1
3
 3 1 1 1 4
3 4 4
x3 3
Do đó:   x  1
0
2
dx  
1
t2
dt    t  3   2  dt   t 2  3t  ln t    2 ln 2 
1
t t  2 t1 2
1
4x
Ví dụ 5: Tính tích phân sau: I=  4x
0
2
 4x 1
dx

Giải
4x 4x
Ta có: 
4 x  4 x  1  2 x  12
2

1  x  0  t  1
Đặt: t= 2x-1 suy ra: dt  2dx  dx  dt; 
2 x  1  t  1
1
1
4x 4x
1 1 4.  t  1 1 1
1 1   1 1
Do đó:  2 dx   dx   2 dt     2  dt   ln t   1  2
     1 
t t   t
2 2
0
4 x 4 x 1 0 2 x 1 1
t 2

3. Tam thức: f ( x)  ax 2  bx  c vô nghiệm

NGUYỄN VĂN LỰC  0933.168.309 SP Toán K35 - ĐH Cần Thơ


Nguyên hàm – Tích phân FB: http://www.facebook.com/VanLuc168
 b
 u  x
P( x) P( x)  2a
Ta viết: f(x)=  ;
 b      a  u  k  k  
2 2 2 2

a  x        2a
 2a   2a  

Khi đó: Đặt u= ktant

x3  2 x 2  4 x  9
2
Ví dụ 6: Tính tích phân sau: I= 0 x2  4
dx

Giải
x  2x  4x  9
3 2
1
 Ta có:  x2 2
x 4
2
x 4
x  2x  4x  9  1  1 2 2
2 3 2 2 2
dx
 Do đó: 0 dx    x  2  2  dx   x  2 x    2  6 J (1)
x 4
2
0
x 4 2  0 0 x 4
2
1
Tính tích phân J= x
0
2
4
dx

x  0  t  0
 
= 2 dt; 
2
 Đặt: x=2tant suy ra: dx   t  0;   cost>0
cos t  x  2  t   4
 4
 

1
2
1 1 2 14 4
1 
 Khi đó:  2 dx   dt   dt  t 4 
x 4 4 0 1  tan t cos t
2 2
20 2 8
0 0

 Thay vào (1): I  6 
8


P( x)
C. DẠNG: 
 ax
3
 bx 2  cx  d
dx

1. Đa thức: f(x)= ax 3  bx 2  cx  d  a  0  có một nghiệm bội ba


1 

1 1
Công thức cần chú ý: 
 x
m
dx  . m1
1 m x 
1
x
Ví dụ 7: Tính tích phân: I=   x  1
0
3
dx

Giải
Cách 1:
 Đặt: x+1=t , suy ra x=t-1 và: khi x=0 thì t=1 ; khi x=1 thì t=2
t 1 1 1  1 1 12 1
1 2 2
x
 Do đó:   x  1 dx   dt    2  3  dt     2  1 
1
0
3
1
t 3
t t   t 2t  8
Cách 2:
x  x  1  1  1  1
 Ta có: 
 x  1  x  1  x  1  x  1
3 3 2 3

1
x  1 1
1   1 1 1 1 1
 Do đó: 0  x  13 dx  0   x  12  x  13   x  1 2  x  12  0  8
  dx    
   

NGUYỄN VĂN LỰC  0933.168.309 SP Toán K35 - ĐH Cần Thơ


Nguyên hàm – Tích phân FB: http://www.facebook.com/VanLuc168
0 4
x
Ví dụ 8: Tính tích phân: I=  dx .
 x  1
3
1

Giải
 Đặt: x-1=t , suy ra: x=t+1 và: khi x=-1 thì t=-2 và khi x=0 thì t=-1 .
1
 t  1 1 4 1
4
t  4t 3  6t 2  4t  1  6 4 1
0
x4
 Do đó:   x  1 dx   dt   dt    t  4   2  3  dt
2 
1
3
2
t3 2
t 3
t t t 
1
1 33
    t  4   2  3  dt   t 2  4t  6ln t   2    6ln 2
6 4 1 1 4 11
2 
t t t  2 t 2 t  2 8
2. Đa thức: f(x)= ax  bx  cx  d  a  0  có hai nghiệm:
3 2

Có hai cách giải: Hệ số bất định và phương pháp nhẩy tầng lầu
3
1
Ví dụ 9: Tính tích phân sau: I=   x  1 x  1
2
3
dx

Giải
Cách 1. ( Phương pháp hệ số bất định )
A  x  1  B  x  1 x  1  C  x  1
2
1 A B C
 Ta có:    
 x  1 x  1
2
x  1  x  1  x  1 2
 x  1 x  1
2

 1
 A
1  4 A 
 Thay hai nghiệm mẫu số vào hai tử số:  
4 . Khi đó (1)
1  2C C   1
 2


 A  B  x   2 A  C  x  A  B  C  A  B  C  1  B  A  C 1  1  1 1   1
2

 x  1 x  1
2
4 2 4
3
1 
3
1 1 1 1 1 1
 Do đó: 2  x  1 x  12 dx  2  4 x  1 4  x  1 2  x  12  dx
 .  . 
 
1 1 1 3 1 3
 I   ln  x  1 x  1  .   ln 8  ln 2
4 2  x  1  2 4 4
Cách 2:
 Đặt: t=x+1, suy ra: x=t-1 và khi x=2 thì t=3 ; khi x=3 thì t=4 .
1 t  t  2 1 1 
3 4 4 4 4
1 dt 1
 Khi đó: I=  3 t 2  t  2  2 3 t 2  t  2  2  2 t  t  2  3 t dt 
dx   dt   dt 
2  x  1 x  1
2

11  1 1 1  1 t 2 1 4 3
4 4
 I      dt   dt    ln  ln t  3  4 ln 2
2 2 2t 2 t  3
t  4 t 2 

Hoặc: 3
1 
 3
3t 2  4t  1  3t 2  4t  4   3t 2  4t 1  3t  2   3t 2  4t 1  3 2 
  3       
t  2t 2 t  2t 2 4  t  2t 2   t 3  2t 2 4 t 2  t 3  2t 2 4  t t 2 
 3t 2  4t 1  3 2  
4
 1 2  4 3
 Do đó: I=      2   dt   ln t 3  2t 2   3ln t     ln 2
3
t  2t
3 2
4  t t   4 t  3 4
1  t  t  4  1  1
2 2
1 t2 1 1 1 2
Hoặc: 2      2     2
t t  2 4  t t  2  4  t  2
2
t  4t2 t t 
 
1  1 1 2 1 t 2 2 4 1 1 1 1 2 1 1
4
 Do đó: I=     2  dt   ln     ln   ln     ln 3  ln 2  
4 3t 2 t t  4 t t  3 4 2 2 3 3 4 6

NGUYỄN VĂN LỰC  0933.168.309 SP Toán K35 - ĐH Cần Thơ


Nguyên hàm – Tích phân FB: http://www.facebook.com/VanLuc168
3 2
x
Ví dụ 10: Tính tích phân sau: I=   x  1  x  2  dx
2
2

Giải
Đặt: x-1=t , suy ra: x=t+1 , dx=dt và: khi x=2 thì t=1 ; x=3 thì t=2 .
 t  1 dt  2 t 2  2t  1 dt
3 2 2
x2
Do đó:  dx  1 t 2  t  3 1 t 2  t  3
2  x  1  x  2 
2

Cách 1; ( Hệ số bất định )


t 2  2t  1 At  B C
Ta có: 2   
 At  B  t  3  Ct   A  C  t   3 A  B  t  3B
2 2

t  t  3 t 2
t 3 t 2  t  3 t 2  t  3
 1
B  3
A  C 1 
  5 t 2  2t  1 1 t  3 4 1
Đồng nhất hệ số hai tử số: 3 A  B  2   A   2  
3B  1  9 t  t  3 9 t 2 9 t 3
  4
C  9

t  2t  1  1 1 3  4  1  1 3 4  2 17 4
2 2 2
7
Do đó:  2 dt      2      dt    ln t    ln t  3  1   ln 5  ln 2
1
t  t  3 1 
9 t t  9  t  3  9 t 9  6 9 9
Cách 2:
 Ta có:
t 2  2t  1 1  3t 2  6t  3  1  3t 2  6t  1  3t 2  6t  1  t   t  9   
2 2
3
             
t 2  t  3 3  t 3  3t 2  3  t 3  3t 2 t 2  t  3  3  t 3  3t 2  9  t 2  t  3   
  
1  3t 2  6t  1 1 1 t  3 1  3t 2  6t  1 1 1  1 3 
  3 2 
    3 2 
    2 
3  t  3t  9 t  3 9 t 2
3  t  3t  9 t  3 9  t t  
t  2t  1
2 2 2
 1  3t 2  6t  1  1 1 3   1 1  t  3 3  2
 Vậy:  2 dt     3 2 
    2   dt   ln t 3  3t 2   ln  
1
t  t  3 1 
3 t  3t  9  t  3 t t   3 27  t t  1
17 4 7
 Do đó I=  ln 5  ln 2
6 9 9

3. Đa thức: f(x)= ax 3  bx 2  cx  d  a  0  có ba nghiệm:

3
1
Ví dụ 11: Tính tích phân sau: I=  xx
2
2
 1
dx

Cách 1: ( Hệ số bất định )


1 1 A B C A  x 2  1  Bx  x  1  Cx  x  1
 Ta có: f(x)=     
x  x 2  1 x  x  1 x  1 x x  1 x  1 x  x  1 x  1
 Đồng nhất hệ số hai tử số bằng cách thay các nghiệm: x=0;x=1 và x=-1 vào hai

 A  1
x  0  1   A 
 
tử ta có:  x  1  1  2C   B   f ( x)    
1 1 1 1  1 1 
  
 x  1  1  2B  2 x 2  x 1  2  x 1 
  1
C  2

NGUYỄN VĂN LỰC  0933.168.309 SP Toán K35 - ĐH Cần Thơ


Nguyên hàm – Tích phân FB: http://www.facebook.com/VanLuc168
1 1 1  1 1 3 5
3 3

   dx    ln  x  1 x  1   ln x  2  ln 2  ln 3
1 3
 Vậy:  2 dx     
2 x  x  1 2
2  x 1 x 1  x  2  2 2
Cách 2: ( Phương pháp nhẩy lầu )
1 x 2   x 2  1 x 1 1 2x 1
Ta có:   2   
x  x  1 x  x  1 x 1 x 2 x 1 x
2 2 2

1 3 5
3 3 3
dx   ln  x 2  1  ln x   ln 2  ln 3
1 1 2 xdx 1 3
Do đó: 2 x  x 2  1 dx    
2 2 x 1 2 x
2
2 2 2 2

x 1
4
Ví dụ 12: Tính tích phân sau: I=  dx
3 x  x2  4
Cách 1:
x 1 x 1 A B C A  x 2  4   Bx  x  2   Cx  x  2 
Ta có:     
x  x 2  4  x  x  2  x  2  x x  2 x  2 x  x2  4
Thay các nghiệm của mẫu số vào hai tử số:
Khi x=0: 1= -4A suy ra: A=-1/4
Khi x=-2: -1= 8C suy ra C=-1/8
Khi x=2: 3= 8B suy ra: B=3/8 .
Do đó: f(x) =        
1 1 1 1 3 1
  
4 x  8 x2 8 x2
x 1  1 3
4 3 3 3
1 1 1 1 3 1 1 3
Vậy: 3 x  x2  4  dx  
4 2 x
dx 
8 2 x  2
dx 
8 2 x  2
dx   
 4
ln x 
8
ln x  2 
8
ln x  2  2

5 3 1
 ln 3  ln 5  ln 2
8 8 4
Cách 2:
Ta có:
1  1  x   x  4  1  1
2 2
x 1 1 1 1 1 1 1 2x 1
              
x  x 2  4   x 2  4  x  x 2  4  4  x  2 x  2  4  x  x 2  4   4  x  2 x  2 2 x  4 x 
2

x 1 1  1 1 1 x  2 1 4
4 4
 ln  x 2  4   ln x 
1 1 2x
Do đó:  2 dx        dx   ln
3 x  x  4
4 3 x2 x2 2 x 4 x 2
4 x  2 2 3
3
x2
Ví dụ 13: Tính tích phân sau: 2  x 2  1  x  2  dx
Giải
Cách 1: ( Hệ số bất định )
x2 x2 A B C A  x  1 x  2   B  x  1 x  2   C  x 2  1
    
 x 2  1  x  2   x  1 x  1 x  2  x  1 x  1 x  2  x 2  1  x  2 
Thay lần lượt các nghiệm mẫu số vào hai tử số:
Thay: x=1 Ta cớ: 1=2A , suy ra: A=1/2
Thay: x=-1 ,Ta có:1=-2B, suy ra: B=-1/2
Thay x=-2 ,Ta có: 4= -5C, suy ra: C=-5/4
Do đó:

NGUYỄN VĂN LỰC  0933.168.309 SP Toán K35 - ĐH Cần Thơ


Nguyên hàm – Tích phân FB: http://www.facebook.com/VanLuc168
1 1 5 1   1 x 1 5 3 1 3
3 2 3
x 1 1
I=  2 dx      dx   ln  ln x  2   ln
2  x  1  x  2  2
2 x 1 2 x 1 4 x  2   2 x 1 4 2 2 2
Cách 2.( Nhẩy tầng lầu )
Ta có:
x2 x2 1  1 1 1 1 1 x  x  1   x  1 x  2 
    
 x  1  x  2   x  1  x  2  x  2  x  1 x  1 x  2  x  2 2  x  1 x  1 x  2 
2 2

1 1 x 1  1 1  1 1 1  1 
      1    
x  2 2   x  1 x  2  x  1 x  2 2  3  x  1 x  2  x  1 
Từ đó suy ra kết quả .


R  x
D. DẠNG: 
 ax
4
 bx 2  c
dx

Những dạng này , gần đây trong các đề thi đại học ít cho ( Nhưng không hẳn là không
cho ) , nhưng tôi vẫn đưa ra đây một số đề thi đã thi trong những năm các trường ra đề
thi riêng , mong các em học sinh khá ,giỏi tham khảo để rút kinh nghiệm cho bản thân
.
1
1
Ví dụ 1. Tính tích phân sau:  dx
 x 2  3x  2 
2
0

Giải
Ta có:
2
1  1
1 1 
x  3 x  2   x  1 x  2   f ( x) 
2
   
 x 2  3x  2   x  1 x  2    x  1  x  2  
2 2

1 1 2 1 1  1 1 
      2   . Vậy:
 x  1  x  2   x  1 x  2   x  1  x  2   x  1 x  2 
2 2 2 2

1 
 1 1   1 x 1 1 2
1
1 1 1 1
0 x2  3x  2 2 dx  0   x  12  x  2 2  x  1 x  2  dx    x  1  x  2  2 ln x  2
   2   0  3  2 ln 3
    

x2 1
3
Ví dụ 2. Tính các tích phân sau: 
1
x4  x2  1
dx

Giải
Chia tử và mẫu cho x  0 , ta có: 2

1  1 
1 3 3 1  2  dx
 x 
f ( x)  x2 
 f ( x)dx   1
1
x2  2 1  2 1 
 x  2  1
1 1
x  x 
x  1  t  2
 1 
t  x   x  2  t  2, dt  1  2  dx  
1 1
Đặt: 2 2
x  3  t  4
x x  x 
 3

NGUYỄN VĂN LỰC  0933.168.309 SP Toán K35 - ĐH Cần Thơ


Nguyên hàm – Tích phân FB: http://www.facebook.com/VanLuc168
4 4 4

 1 
3 3 3 3
dt 1 1 1
Vậy:  f ( x)dx  t   dt     dt
1 2
2
3 2 t  3 t  3  2 3 t
2 3 t 3

4
t 3 1  1 74 3 
I ln
1
2 3 t 3
3  ln  ln
2 3 7
 
7  2 3
1
ln 7  4 3  )
2

x2 1
2
Ví dụ 3. Tính tích phân sau: 1  x2  5x  1 x2  3x  1 dx .

Ta có:
1  1 
1  1  2  dx
x 1 2 2 2
 x 
1
2
f ( x)  2  x   f ( x)dx  
 x  5x  1 x  3x  1  x  1  5 
2

 x 
1 

1 1 x

1  1 
 5  x   3 
 x  x 3  x  x 

Đặt: t  x   dt  1  2  dx , x  1  t  2, x  2  t 
1 1 5
x  x  2
Vậy (1) trở thành:
5 5
5
2
dt 1  1 2
1  1 t 5 1 1 5
2  t  5 t  3  2 2  t  5  t  3  dt  2 ln t  3 2  2  ln 5  ln 3  2 ln 3
2

5
2
dx
Ví dụ 4. Tính tích phân sau: x
3
4
 4 x2  3
.
2

1 1 1 1 1 
Ta có: f ( x)   2   2  2 
x  4 x  3  x  1 x  3 2  x  3 x  1 
4 2 2

5 5

 1 
2 2
1
Do đó:  f ( x)dx    x  2  dx  I  J 1 Với:
3 3
2
 3 x 1 
2 2

5 5 5 5
2
1 2
1 1 2 1 1  1 x 3 2 1 37  20 3
I  2 dx   dx      dx  ln  ln
3 x 3
2
3 x 3
2

x 3 
2 3 3 x 3 x 3
2

2 3 x  3 3 2 3 65 7  4 3  
2
5 5
1 2 1 1  1 x 1 2 1  3 1  1 15
1 1
1 1
J   2 dx   dx      dx  ln   ln  ln   ln
0 
0
x 1 x  1 x  1 2 3  x 1 x  1  2 x 1 3 2  7 5 2 7
2
2
x7 3
Ví dụ 5. Tính tích phân sau:  dx
2 1  x 8
 2x 4

NGUYỄN VĂN LỰC  0933.168.309 SP Toán K35 - ĐH Cần Thơ


Nguyên hàm – Tích phân FB: http://www.facebook.com/VanLuc168
3 7 3 4
x x
I = dx   x 3dx 1
2 1  x  2x x 
8 4 2
2 4
1
dt  3x3dx, x  2  t  15; x  3  t  80

Đặt: t  x 4  1   1 x4 1  t  1 1 1 1 
f ( x ) dx  3 x 3
dx  dt    2  dt
 3  x  1
4
3 t 2
3 t t 

Vậy: I     2  dt   ln t    ln 
80
1 1 1 1 1 80 1 16 13
15 
3 t t  3 t  15 3 3 720


R  x
E. TRƯỜNG HỢP: 
 Q( x )
dx ( Với Q(x) có bậc cao hơn 4 )

Ở đây tôi chỉ lưu ý: Đối với hàm phân thức hữu tỷ có bậc tử thấp hơn bậc mẫu tới hai
bậc hoặc tinh ý nhận ra tính chất đặc biệt của hàm số dưới dấu tích phân mà có cách
giải ngắn gọn hơn . Phương pháp chung là như vậy , nhưng chúng ta khéo léo hơn thì
cách giải sẽ hay hơn .
2
dx
Ví dụ 1. Tính tích phân sau:  xx
1
4
 1
.

Nếu theo cách phân tích bằng đồng nhất hệ số hai tử số thì ta có:
A Bx 3  Cx 2  Dx  E A  x  1  x  Bx  Cx  Dx  E 
4 3 2
1
f ( x)     
x  x 4  1 x x4  1 x  x 4  1
A  B  0 A 1

 f ( x) 
 A  B  x  Cx  Dx  Ex+A  C  0, D  0   B  1
4 3  2

 
1

x3
  f ( x )
x  x 4  1 E  0 C  0, D  0, x x4  1
 A  1  E  0
Nhưng nếu ta tinh ý thì cách làm sau sẽ hay hơn .
Vì x và x 3 cách nhau 3 bậc , mặt khác x  1; 2  x  0 . Cho nên ta nhân tử và mẫu với
x3
x3  0 . Khi đó f ( x)  . Mặt khác d  x 4   4 x3dx  dt  4 x3dx t  x  , cho nên:
4

x 4  x 4  1
1 3x3dx 1 dt 1 1 1 
f ( x)dx        f (t ) . Bài toán trở nên đơn giản hơn rất nhiều
3 x  x  1 3 t  t  1 3  t t  1 
4 4

. ( Các em giải tiếp )

1
2
x2
Ví dụ 2. Tính tích phân sau:   x  1  x  3 dx
0
2

Nhận xét:
* Nếu theo cách hướng dẫn chung ta làm như sau:
x2  1 A B C D
- f ( x)     
 x  1  x  3  x  1  x  1 x 1 x  3
3 3 2

NGUYỄN VĂN LỰC  0933.168.309 SP Toán K35 - ĐH Cần Thơ


Nguyên hàm – Tích phân FB: http://www.facebook.com/VanLuc168
1 3 5
- Sau đó quy đồng mẫu số , đồng nhất hệ số hai tử số , ta có: A  , B  , C   D 
2 8 32
1
2  1 3 5 5 
Do vậy: I        dx
0
 2  x  13
 8  x  1
2
32  x  1 32  x  3 

 1 3 5 5 1 5 1
    ln x  1  ln x  3  2  ln
 8  x  1 8  x  1 32
2
32  0 32 28

2
dx
Ví dụ 3. Tính tích phân sau:  x 1  x 
1
4

1  d  x  d  x   1  x 4  2 1 32
2 2 2 4 4
dx 1 3x3
1 x 1  x 4   3 1 x 4 1  x 4  dx  3 1  x 4  1  x 4   3 ln  1  x 4  1  3 ln 17
 

1
x3
Ví dụ 4. Tính tích phân sau:  dx
1  x 2 
3
0

1
x3 1
1
x2  x 2  t  1; dt  2 xdx
 1 .
2 0 1  x 2 3
dx  2 xdx Đặt: t  1  x 2  
1  x   x  0  t  1, x  1  t  2
2 3
0

t 1
Do đó I   3 dt    2  3  dt     2  
2 2
1 1 1 1 2 13
1
1
t t t   t 4t  1 16

x 4  3x 2  1
1
Ví dụ 5. Tính tích phân sau:  dx
1  x 2 
3
0

 1  x 2 2 
x 4  3x 2  1
1 1 1 1
 x2  1 x2
 dx    dx  0 1  x 2 0 1  x 2 3 dx  J  K 1
dx 
0 1  x  2 3
0 1  x 


2 3
1  x 
2 3 
  

Tính J: Bằng cách đặt x  tan t  J 
4
1  
1 1
Tính K=     dx  E  F  2 
 1  x 2 2 1  x  
2 3 
0

 1
dx  cos 2t dt
x  tan t  
Tính E: Bằng cách đặt  x  0  t  0; x  1  t  
 4

  
2 2
1  1  1   1
1 4 4
1 1 1 1 14
Vậy: E    2 
2 0  1 x 
dx    2 
2 0  1  tan t  cos t
2
dt  
20 1 cos 2t
dt 
2 0
cos 2tdt

cos 4t


14 1 1  1 1  2
  1  cos2t  dt   t  sin 2t  4     
40 4 2  0 4 4 2 16

NGUYỄN VĂN LỰC  0933.168.309 SP Toán K35 - ĐH Cần Thơ


Nguyên hàm – Tích phân FB: http://www.facebook.com/VanLuc168
Tính F. Tương tự như tính E ;
 1
dx  cos 2t dt
x  tan t  
Bằng cách đặt  x  0  t  0; x  1  t  
 4

  
3 3
1  1  14  1
1
1 14 1 1 14
F  
2 0  1  tan 2 t  cos 2t 2 0 
Vậy:  dx    dt  dt  cos 4tdt
2 0  1  x2  1 cos 2t 20
cos 6t
 

1 4
1 4 1  cos4t 
  1  cos2t  dt   1  2cos2t  dt 4 
2

80 8 0 2  0


1  3  8
 3  4 cos 2t  cos4t  dt   3t  2sin 2t  sin 4t  4   3  2  
1 4 1 1

16 0 16  4  0 16  4  64
1
1
x  x  3 3

Ví dụ 6. Tính tích phân sau: 


1 x4
dx
3
1

x  x 
1 1
3 3
1
 x  x3  3 1
1 1
 1  3 1 dx

1 x4
dx    3  3 dx    2  1 2 .
1 x  x 1 x  x x
3 3 3

 dx
 dt  
Đặt: t   2  1  t  1  2  
1 1 x
x  x  x  1  t  8; x  1  t  0
 3
  3 7 3 48 3
Khi đó I   t 3  t  1 dt    t 3  t 3  dt   t 3  t 3   .27  .24  16    
0 1 8 4 1
3 24 3 468
8 0  7 4 0 7 4  7 4 7
* Chú ý: Còn có cách khác
1
 1 1 3
  3
1

1   1 t t  t 
2 3 3
Vì: x   ;1  x  0 . Đặt x   dx   2 dt; f ( x)dx   t t 4 
1 1
  2  dt   dt
3  t t 1  t  t
 
t 
1
 1 3
1
 t  t  t 
1 1 1
3 3 dt  dt  t 1  2  dt (2)
2
. Đặt: u  1   2  1  u; du  dt
 t 
2
t t t

a
x 3 dx
Ví dụ 7. Tính tích phân sau:  3
.
0
x 2
a 2 2

 dt 
dx=a cos 2t ; x  0  t  0, x  a  t  4

Đặt:
3 3 3
x  atant   f ( x)  x dx  a tan t a dt  a cos t.tan 3 tdt
 3 3
cos 2t
  x  a  a3  2 
2 2 2
 1  2

  cos t 

NGUYỄN VĂN LỰC  0933.168.309 SP Toán K35 - ĐH Cần Thơ


Nguyên hàm – Tích phân FB: http://www.facebook.com/VanLuc168
   

Vậy: I  
a 4
f ( x)dx   a cos t.tan tdt   a cos t.
4
sin t 3
sin t 4
3
 dt 
4 1  cos t sin t 2

   
3
3
dt a. 2
dt a 2
0 0 0
cos t 0
cos t 0
cos t
  1
du   s intdt;t= 4  u  2 ; t  0  u  1

- Đặt: cost=u  
 f (t )dt  1  u  du  1  1  du
2

    2
u2  u 
2
2
2
 1   1 2 2 3 3 2 3 2 4
Vậy: I  
1
1  2  du   u   2 
 u   u 2

2
2
2
2
2
2
2
1
1 1
Ví dụ 8. Tính tích phân sau:  e x e dx   e x ee dx .
x x

0 0

dt  e x dx; x  0  t  1; x  1  t  e
Đặt: t  e   x

 f ( x)dx  e e dx  e dt
x ex t

1 e
e
Vậy: I   f ( x)dx   et dt  et  ee  e
0 1
1

2a 2a
Ví dụ 9. Tính tích phân sau:  x 2ax  x 2 dx  x a 2   x  a  dx
2

0 0

  
dx  a.costdt,x=0  t=- 2 ;x=2a  t= 2
Đặt: x  a  a.sin t  
 f ( x)dx   a  a.sin t  a 2cos 2t .a.costdt


 2 
  2 

2
3 
2
 3  1  cos2t 2

Vậy: I  a  1  sin t  cos tdt  a   cos tdt   cos t sin tdt   a  
3 2 2 2
dt   cos td  cost  
2

 2

2
  2 

2
   2 

2

   
1  1  2 1   1     
2 
 a3   t  sin 2t   cos3t  a3      a3
2  2   3   2  2 2  2
   
2 2

1
x 7 dx
Ví dụ 10. Tính tích phân sau: 0 1  x 
4 2

1 1
x 7 dx 1 x4
 1 .
3 0 1  x 4 2
 3 x 3 dx
0 1  x 
4 2

dt  3x3dx, x  0  t  1; x  1  t  2
Đặt: t  1  x 4   1  t 1  1 1 1 
 f ( x)dx  3  t 2  dt  3  t  t 2  dt
    

Vậy: I     2  dt   ln t     ln 2  
2
1 1 1 1 1 2 1 1
0
3 t t  3 t  1 3 2

NGUYỄN VĂN LỰC  0933.168.309 SP Toán K35 - ĐH Cần Thơ


Nguyên hàm – Tích phân FB: http://www.facebook.com/VanLuc168
x  2x
1 3
Ví dụ 11. Tính tích phân sau:  dx
 x 2  1
2
0

1  x  2
2
x3  2 x
1 1

0 x2  1 2 dx  2 0 x2  1 2 2 xdx 1
   
dt  2 xdx; x  0  t  1; x  1  t  2
Đặt: t  1  x  x  2  t  3  
2 2
1  t 3 1 1 3 
 f ( x)dx  2  t 2  dt  2  t  t 2  dt
    

Vậy: I     2  dt   ln t     ln 2  
2
1 1 3 1 3 2 1 3
1
2t t  2 t  1 2 2

1  x3
2
Ví dụ 12. Tính tích phân sau: 
1
x4
dx

1 x 1 x 2
2 3 2 3

 4
dx   x dx 1 .
1
x 1
x6
2tdt  3x 2 dx; x  1  t  2, x  2  t  3

Đặt: t  1  x3  t 2  1  x3   1 1  x3 2 1 t 2 t2
 f ( x)dx  3 x 6 3x dx  3 2 2tdt  dt
  3  t 2  12
2

 t  1
Vậy:
2
3
 1 1 1 1 
2
2 1 1
3
1   1
2 3  1 1  1 1 
I
3   t  1  2  t  1  t  1   dt  3   4  t  1  t  1    6    t  12  t  12  t  1 t  1   dt
   
2  2  2 
1 1 t  1  3 1  2t t 1  3 8 2  3 1
  
1
6  t  1 t  1
 ln  
t  1  2 6   t 2  1
 ln 
t 1  2

24
 ln 2 2  2
3
 
 

4
dx
Ví dụ 13. Tính tích phân sau: x x2  9
7
4 4
dx xdx
x  x 1 .
7 x 9
2
7 x 2
 92

t 2  x 2  9  tdt  xdx, x 2  t 2  9
Đặt: t  x 2  9  
5 5
dt dt
. Do đó: I   2 
4 t t  9 4 
 x  7  t  4, x  4  t  5
 t t  3 t  3

1 A B C A  t 2  9   Bt  t  3  C  t  3 t
Ta có: f (t )     
t  t  3 t  3 t t  3 t  3 t t 2  9
Đồng nhất hệ số hai tử số bằng cách thay lần lượt các nghiệm vào hai tử số ta có:
1
- Với x=0: -9A=1  A  
9
1
- Với x=-3: 9C=1  C 
9
1
- Với x=3: 9B=1  B 
9
1  1 1 1   1 5 1 t 2  9 5 1 144
5
Vậy: I        
dt  
9 4  t t 3 t 3  9 
ln t 2
 9  ln t   
 4  9 ln t 4  9 ln 35

NGUYỄN VĂN LỰC  0933.168.309 SP Toán K35 - ĐH Cần Thơ


Nguyên hàm – Tích phân FB: http://www.facebook.com/VanLuc168
* Chú ý: Nếu theo phương pháp chung thì đặt: x  3sin t  dx  3cos tdt .
 7
 x  7  7  3sin t  sin t 
 3 . Như vậy ta không sử dụng được phương pháp
Khi: 
 x  4  4  3sin t  sin t  4  1

 3
này được.

1
x 2
 x  dx
Ví dụ 14. Tính tích phân sau: 0 x2  1
1
x 2
 x  dx 1
x2
1
x
  dx   dx  J  K 1
0 x 1
2
0 x 1
2
0 x 1
2

* Để tính J:
 1 
dx  cos 2t dt , x  0  t  0; x  1  t  4

Đặt: x  tan t   1 . Tính tích phân này không đơn giản
 tan 2 t. dt 2
2
cos t  tan t
 f ( x)dx  dt
 1  tan 2
t cost
, vì vậy ta phải có cách khác .
x2  1 1
1 1 1
x2 1 1
- Từ: g ( x)    x 1  2
  g ( x)dx   x  1dx  
2
dx
x 1
2
x 1
2
x 1
2
0 0 0 x2  1
- Hai tích phân này đều tính được .
1
1 2 1 1
1
1
 x2
+/ Tính: E   x 2  1dx x x 2  1   dx  2    x  1dx   dx 
0
0 0 x2  1 0 0 x2  1 

   
1 2 1
 2  E  ln x  x 2  1  2 E  2  ln 1  2  E   ln 1  2
0 2 2

 
1 1
x 1 1 1
* Tính K=  2 dx  x 2  1  2  1 ;  2 dx  ln x  x 2  1  ln 1  2
0 x 1 0 0 x 1 0

Do vậy: I=
2 1

 ln 1  2  ln 1  2 
2 2
2 3
 
 ln 1  2
2 2
  
x5  2 x3
3
Ví dụ 15. Tính tích phân sau: 
0 x2  1
dx

x5  2 x3
3 3 3
x5 x3
 dx   dx  2  dx  J  K 1
0 x2  1 0 x2  1 0 x2  1
 x 2  t 2  1; xdx  tdt ; x  0  t  1, x  3  t  2

- Tính J: Đặt t  x 2  1   x 4 xdx  t  1 tdt
2 2

 f ( x)dx     t 4  2t 2  1 dt
 x 1 t
2

Suy ra: J=   t 4  2t 2  1 dt   t 5  t 3  t  
2
1 2 2 38
1 5 3  1 15
 x 2  t 2  1; xdx  tdt ; x  0  t  1, x  3  t  2

- Tính K: Đặt t  x 2  1   x 2 xdx  t  1 tdt
2

 f ( x)dx     t 2  1 dt
 x 1
2 t

NGUYỄN VĂN LỰC  0933.168.309 SP Toán K35 - ĐH Cần Thơ


Nguyên hàm – Tích phân FB: http://www.facebook.com/VanLuc168
1 2 4
2
Suy ra: K=  t
 1 dt   t 3  t  
2

1 3 1 3
28 4 48 16
Vậy: I=   
15 3 15 5
1

 1  x  dx
2 3
Ví dụ 16. Tính tích phân sau:
0

 
1
 dx  costdt. x=0  t=0;x=1  t=
 1  x  dx . Đặt:
2 3 2
x  sin t  
 
 f ( x)dx  1  x 2 dx  cos6tcostdt=cos 4tdt
0 3


  

1  cos2t  1  cos4t 
2

I=  
2
1 2 2
3 1 1 
Do đó
0
2


dt  
4 0
 1  2 cos 2t 
2


dt  
0
  cos2t+ cos4t  dt
4 2 8 

3 1 1  3
  t  sin 2t  sin 4t  2 
4 4 32  0 8

NGUYỄN VĂN LỰC  0933.168.309 SP Toán K35 - ĐH Cần Thơ


Nguyên hàm – Tích phân FB: http://www.facebook.com/VanLuc168

BÀI TẬP TỰ LUYỆN

1. Tích phân hàm phân thức


1
Câu 1. Tính tích phân: I   6x+7 dx .
0
3x  2

1 1 1
6x+7 (6x+4)+3 3
I dx   dx   (2  )dx
0
3x  2 0
3x  2 0
3x  2
1 1 1 1
3 1
 2  dx   dx  2  dx   d(3x+2)
0 0
3x  2 0 0
3x  2
1 1
 2x  ln 3x  2
0 0

5
 2  ln .
2

2x  3
Câu 2. Tìm họ nguyên hàm  dx
2 x2  x 1

2x  3 2x  3  4 1 5 1 
Ta có: 2 2x
 x 1
dx  
(2 x  1)( x  1)
dx     .  .
 3 2 x  1 3 x  1 
dx

4 1 5 1
  dx   dx
3 2x 1 3 x 1
2 d (2 x  1) 5 d ( x  1)
   
3 2x 1 3 x 1
2 5
  ln 2 x  1  ln x  1  C
3 3

2
 1  x2 
Câu 3. Tính tích phân sau: I    x 2  3 
dx
1 x  x 

 2 1  x2 
2 2 2
1  x2
I   x   dx   x dx  
2
dx
1 x  x3  1 1 xx
2

2 2
1 7
Tính I1   x dx  x3  2

1
3 1 3

NGUYỄN VĂN LỰC  0933.168.309 SP Toán K35 - ĐH Cần Thơ


Nguyên hàm – Tích phân FB: http://www.facebook.com/VanLuc168
1 1 
 1 d   x  2
1 x
2 2 2
x    ln 1  x
2
x2 4
I2   dx  1 1 dx   1 1  ln
1 x x
3
x 5
x x 1
x x
7 4
Vậy I  I1  I 2   ln
3 5

2. Tích phân hàm chứa căn thức

2 dx
Câu 4. Tính tích phân I   .
1
x x3  1

2 dx 2 x 2 dx
I   .
1
x x 1 3 1
x 3
x 1
3

2
Đặt t  x3  1  x3  t 2  1  x 2 dx  t.dt .
3
x 1 t  2 ; x  2  t  3
3 2 t.dt 1 3  1 1 
I       dt
2 3 (t 2  1)t 3  t 1 t 1 
2

1 1 2 1  1 3  2 2
3
1 x 1
I  ln   ln  ln   ln
3 x 1 2
3 2 2 1 3 2

3
1
Câu 5. Tính tích phân I  x
1 x2  1
dx

3 3
1 x
I x
1 x2  1
dx  x
1
2
x2  1
dx

Đặt u  x 2  1  u 2  x2  1  udu  xdx ,  x2  u 2 1


2
u 1
2
u  1   u  1
I   u 2  1 u du  2   u  1 u  1 du
2 2

1 u 1
 
2
 1 1 
2
1 1

2     du  ln
 u 1 u  1  2 u 1 2
  ln 3 3  2 2
2
2

1
Câu 6. Tính tích phân I x 1 xdx
0

 Đặt t 1 x dt dx dx dt và x 1 t
 Đổi cận: x 0 1
t 1 0
1
3 5
1 3
1 0 1 2t2
2t2
4
 Vậy, I x 1 xdx (1 t ) t ( dt ) (t 2
t )dt
2
0 1 0 3 5 0 15

NGUYỄN VĂN LỰC  0933.168.309 SP Toán K35 - ĐH Cần Thơ


Nguyên hàm – Tích phân FB: http://www.facebook.com/VanLuc168

2x  1
1
Câu 7. Tính tích phân sau I  dx
0 1  3x  1

t2 1 2
Đặt 3x  1  t ta được x   dx  tdt
3 3
Đổi cận x  0  t  1; x  1  t  2
2 2t 3  t 2  3 
2 2
28 2 3
Khi đó I  
9 1 1 t
dt    2t 2  2t  3 
9 1
dt 
t 1
 ln
27 3 2

x 3
3
Câu 8. Tính tích phân sau:  3.
0 x 1  x  3
dx

x  0  u  1
Đặt u = x  1  u 2  1  x  2udu  dx ; đổi cận: 
x  3  u  2
x 3 2u  8u
3 2 3 2 2
1
Ta có:  dx   2 du   (2u  6)du  6 du
0 3 x 1  x  3 1
u  3u  2 1 1
u 1

  1  6ln u  1 1  3  6 ln 2
2 2 3
 u 2  6u

9
xdx
Câu 9. Tính tích phân: 
4 x 1

Đặt t  x  t 2  x  2tdt  dx
Đổi cận: x = 4  t  2
x= 9 t 3
3 3 3
t dt  1 
I  2  2  t 2  t  1   dt
2
t  1 2  t  1 
3
 t3 t 2  59
 2    t  ln t  1    2ln 2
3 2 2 3

0
dx
Câu 10. Tính tích phân: I   ( x  1)
1 3  2x  x 2

2

0 0
dx 1
I  ( x  1)
1 3  2x  x 2
  ( x  1)
1 ( x  1)( x  3)
dx
 
2 2
0
1
= dx
 x3
( x  1)

1 2
2
x 1
 x3  x3 4
Đặt t   t2   2tdt  dx
x 1 x 1 ( x  1) 2

NGUYỄN VĂN LỰC  0933.168.309 SP Toán K35 - ĐH Cần Thơ


Nguyên hàm – Tích phân FB: http://www.facebook.com/VanLuc168
3
1 1
I  
2 7
dt  ( 7  3 )
2

Câu 11. Tính tích phân x2 x 1 xdx .


0

3 3 3

Ta có I x2 x 1 xdx x 3dx x x 1 dx .
0 0 0

3 3 3 3
1 81
Đă ̣t J 3
x dx và K x x 1 dx ; ta có J   x dx  x 4 
3

0 0 0
4 0 4
3
K   x x  1 dx . Đặt t x 1 t2 x 1 2tdt dx và x  t 2  1
0

Ta có x  0  t  1; x  3  t  2 .
2
2
1 1  116
2
Khi đó K  2 t (t  1)dt  2  (t  t )dt  2  t 5  t 3  
2 2 4 2

1 1 5 3 1 15

1679
Vậy I  J  K 
60

 
1
Câu 12. Tính tích phân: I   x 2 1  x 1  x 2 dx
0

 
1 1 1
I   x 1  x 1  x dx   x dx   x 3 1  x 2 dx
2 2 2

0 0 0

1
1
x3 1
I1   x dx  2

0
3 3
0
1
I 2   x 3 1  x 2 dx
0

Đặt t  1  x 2  x 2  1  t 2  xdx  tdt


Đổi cận: x  0  t  1; x  1  t  0
1
0
 t3 t5  1
 I 2    1  t  t dt    t  t dt     
2 2 2 2 4

1 0  3 5  0 15

7
Vậy I  I1  I 2 
15

Câu 13. Tính nguyên hàm sau: I   x x 2  3dx

NGUYỄN VĂN LỰC  0933.168.309 SP Toán K35 - ĐH Cần Thơ


Nguyên hàm – Tích phân FB: http://www.facebook.com/VanLuc168

Đặt t  x 2  3  t 2  x 2  3  2tdt  2xdx  xdx  tdt .


t3 ( x 2  3)3
Suy ra I   t.tdt   t dt   C  2
C
3 3

dx
Câu 14. Tính nguyên hàm: I   
2x  1  4

Đặt t  2x  1  t 2  2x  1  tdt  dx
tdt  4 
I   1   dt  t  4 ln t  4  C
t4  t4
 2x  1  4 ln  2x  1  4  C 
Câu 15. Tính I = 
1

0 
x3 x  x 2  1 dx 

 
1
1 1 1 x5 1
 x x  x  1 dx   x dx   x x  1dx  J  J
3 2 4 3 2
I= 0 0 0 5 0 5

t  t 2  dt
1 2
J   x3 x2  1dx  ...   4
0 1

22 2
J  ... 
15
1 1 2 2
I = J 
5 15

6
Câu 16. Tính tích phân sau: I   x x  3dx
1

 Đặt x  3  t ta được x  3  t 2  dx  2tdt


 Đổi cận: x  1  t  2; x  6  t  3
3
2 
3

Khi đó I    2t  6t  dt   t 5  2t 3  
4 2 232
2 5 2 5

3. Tích phân hàm số mũ, hàm số logarith


1
Câu 17. Tính tích phân I   1  x   2  e2 x  dx
0

 du   dx
u  1  x
Đặt  =>  1
dv  (2  e )dx
2x
 v  2 x  e2 x
 2
2
1 1 1
I  (1  x)(2 x  e2 x )   (2  e2 x )dx
2 0 1 2

NGUYỄN VĂN LỰC  0933.168.309 SP Toán K35 - ĐH Cần Thơ


Nguyên hàm – Tích phân FB: http://www.facebook.com/VanLuc168
1 1 1 1 e 1
2
= (1  x)(2 x  e2 x )  ( x 2  e2 x ) 
2 0 4 0 4

x3  2 ln x
2
Câu 18. Tính tích phân I   dx .
1
x2

2 2 2 2 2
ln x x2 ln x 3 ln x
I   xdx  2 2 dx  2 2 dx   2 2 dx
1 1
x 2 1 1 x 2 1 x
2
ln x
Tính J   dx
1
x2
1 1 1
Đặt u  ln x, dv  2
dx . Khi đó du  dx, v  
x x x
2 2
1 1
Do đó J   ln x   2 dx
x 1 1
x
2
1 1 1 1
J   ln 2    ln 2 
2 x1 2 2
1
Vậy I   ln 2
2

1
Câu 19. Tính tích phân I = (1 + x)e x dx
0

1
I (1 x )e xdx
0

u 1 x du dx
 Đặt . Thay vào công thức tích phân từng phần ta được:
dv e xdx v ex
1 1 1
I (1 x )e x 0
e xdx (1 1)e1 (1 0)e 0 ex 0 2e 1 (e1 e0) e
0
1
 Vậy, I (1 x )e xdx e
0

ln 2
e2 x
Câu 20. Tính tích phân: I   0 ex 1
dx

Đặt t  e x  1  t 2  e x  1  2tdt  e x dx
x  0  t  2, x  ln 2  t  3
(t 2  1)2tdt
3 3
I  2  (t 2  1)dt
2
t 2

3
 t3  2 2
 2  t  
3  2
3

NGUYỄN VĂN LỰC  0933.168.309 SP Toán K35 - ĐH Cần Thơ


Nguyên hàm – Tích phân FB: http://www.facebook.com/VanLuc168
1
Câu 21. Tính: I  0 ( x  2)e x dx.

1
I   ( x  2)e x dx.
0

u  x2 du  dx
Đặt   
dv  e dx  ve
x x

1
Khi đó I= ( x  2)e x 0   e x dx
1

1 1
= ( x  2)e x 0  e x 0  2e  1

e 1  3ln x ln x
Câu 22. Tính: I  1 dx.
x

e 1  3ln x ln x
I  dx.
1 x
3
Đặt u= 1  3ln x =>u2= 1+3lnx => 2udu= dx
x
Đổi cận: x=e => u=2
x=1 => u=1
u2 1 2
2
Khi đó I=  u. . udu
1
3 3
2 2
2 2 u5 u3 116
=  u 2 (u 2  1)du  (  ) 
91 9 5 3 1 135

1
Câu 23. Tính tích phân I x (x e x )dx
0

u x du dx
 Đặt x2
dv (x e x )dx v ex
2
1 1 1
x x2 x
1 x2 x 1 x3 x
Ta có I x (x e )dx x( e ) ( e )dx e ( e )
0
2 0 0 2 2 6 0

1 1 4
e ( e) (0 1)
2 6 3

Câu 24. Tính tích phân I  


e
x 2
 1 ln x  x  1
dx.
1
x ln x  1

e
x  x ln x  1   ln x  1 e e
d  x ln x  1
I  dx   xdx   .
1
x ln x  1 1 1
x ln x  1

NGUYỄN VĂN LỰC  0933.168.309 SP Toán K35 - ĐH Cần Thơ


Nguyên hàm – Tích phân FB: http://www.facebook.com/VanLuc168
2 e
x e2 1
  ln  e  1
e
I  ln x ln x  1 1 
2 1
2 2

e
Câu 25. Tính tích phân I    x   ln xdx.
1
1  x

e e e
Ta có: I    x   ln xdx   x ln xdx   ln xdx.
1 1
1
x 1 1
x
e
1 x2
 Tính  x ln xdx . Đặt u  ln x và dv  xdx . Suy ra du  dx và v 
1
x 2
e e e 2
x2 x e2 x 2 e2 1
Do đó,  x ln xdx  ln x   dx    
1
2 1 1
2 2 4 1 4 4
e
1 1
 Tính  x ln xdx. Đặt t  ln x  dt  x dx . Khi
1
x 1 thì t  0 , khi x  e thì t  1 .

e 1 1
1 t2 1
Ta có:  ln xdx   tdt   .
1
x 0
2 0
2
e 3
2
Vậy, I  .
4


4
tan x .ln(cos x )
Câu 26. Tính tích phân: I =  cos x
dx
0

*Đặt t=cosx
 1
Tính dt=-sinxdx , đổi cận x=0 thì t=1 , x  thì t 
4 2
1
1
2
ln t ln t
Từ đó I    dt   dt
1
t2 1 t2
2

1 1 1
*Đặt u  ln t ;dv  dt  du  dt ; v  
t2 t t
1 1 1
1 1 2 1
Suy ra I   ln t 1 
t 1 t 2 dt   2 ln 2  t 1
2 2 2
2
*Kết quả I  2 1  ln 2
2

e
log 23 x
Câu 27. Tính tích phân sau: x
1 1  3ln 2 x
dx

Đặt
Từ
NGUYỄN VĂN LỰC  0933.168.309 SP Toán K35 - ĐH Cần Thơ
Nguyên hàm – Tích phân FB: http://www.facebook.com/VanLuc168
Đổi cận: với

1 1
2 2
u4 2 u 2 1
I    (1u 2 ) 2
du   ( 1  u 4  2 u 2 1
du )
0 0
2 u 1 2
2( u 1) 1
2
2 1
*)   
u  2 u 1
4 2
( u 1) 2 2
u 1
2
( u 1) 2
2

2 1 [( u 1) ( u 1)]2
  .
u 2 1 4 ( u 2 1) 2

12  1 2 1 
 2  
u 1 4  ( u 1) 2  ( u 1)( u 1)  ( u 1) 2 
 

1  1 1  3  1 1 
    
4  ( u 1) 2 ( u 1) 2  4  u 1 u 1 

 
1 1
2 2
2 u 1
2  1 1 1  3 1 1 
I   ( 1  u 4  2 u 2 1
du )        
 4 ( u 1) 2 ( u 1) 2 4  u 1 u 1  
1 



 du
0 0 

1
 1 1 1  3 |u 1| 2  3 5
   u      ln   ln 3 
 4  u 1 u 1  4 |u 1|  4 6
0

1  x 2 ln x
e
Câu 28. Tính tích phân I   dx
1
x

1  x 2 ln x
e e e
1
I  dx   dx   x ln xdx
1
x 1
x 1
e
1 e
A   dx  ln x  1
1
x 1
 1
 du  dx
e
u  ln x  x
B   x ln xdx Dat  
dv  xdx
2
1 v  x
 2
e
x 2
e x x 2
e x 2
e e2 1 e2 5
 B  .ln x   dx  .ln x    I 
2 1 12 2 1 4 1 4 4 4 4

e ln x  1
Câu 29. Tính tích phân: I  1 dx
x 2  ln 2 x

e ln x  1 ln x 1  ln x 1
I . Đặt u   du  dx : u(1)=0; u(e)=
1   ln x   2
x x 2
e
x 2 1    
  x  

NGUYỄN VĂN LỰC  0933.168.309 SP Toán K35 - ĐH Cần Thơ


Nguyên hàm – Tích phân FB: http://www.facebook.com/VanLuc168
1
1
1 1  u 1  1 e 1 e
I e
du  ln   ln
0 u 1
2
2  u  1 0 2 e  1

Câu 30. Tính tích phân (x 3e x )e 2xdx .


0

1 1 1
Ta có I (x 3e x )e 2xdx   xe 2xdx  3 e 3xdx .
0 0 0

1 1 1
1
Đă ̣t J  3 e 3xdx và K   xe 2xdx ; ta có J  3 e 3xdx  e 3x  e 3  1
0
0 0 0

1 u  x du  dx 1 1
 1 1 2x
2 0
K   xe dx . Đặt 
2x
 1 ; khi đó K  xe 2x  e dx
dv  e dx
2x
v  e 2
2x
0 0
 2
1
1 1 1 1 1 1 1 1 3
 K  e 2  e 2x  e 2  e 2   e 2  . Vậy I  e 3  e 2 
2 4 0
2 4 4 4 4 4 4

e
3x  2 ln x  1
Câu 31. Tính tích phân I   dx
1 x 2  x ln x

e e e
3x  2 ln x  1 2( x  ln x) x 1
Phân tích I  dx =  x2  x ln x dx   x 2  x ln x dx
1 x  x ln x
2
1 1
e e
2( x  ln x) 1
Tính  x2  x ln x dx  2  x dx  2.
1 1
1
e e 1 e
ln( x  ln x) 1  ln(e  1)
e
x 1 d ( x  ln x)
 x 2  x ln x dx =  x  ln x dx  
Tính x 
1 1 1 x  ln x

Vậy I = 2 + ln(e+1).

dx
Câu 32. Tính nguyên hàm sau: 
e 1
x

dx ex
Ta có:  e x  1  e x  1)dx
 (1 

d (e x  1)
=  dx   x = x – ln( e x  1 ) + C
e 1

1
Câu 33. Tính tích phân: I   (1  e x ) xdx
0

NGUYỄN VĂN LỰC  0933.168.309 SP Toán K35 - ĐH Cần Thơ


Nguyên hàm – Tích phân FB: http://www.facebook.com/VanLuc168
u  x du  dx
Đặt:  
dv  (1  e )dx v  x  e
x x

1
x2 3
Khi đó: I  x( x  e x ) 10   ( x  e x )dx  I  1  e  (  ex ) 1
0 
0
2 2

e
Câu 34. Tính tích phân I   x3 ln xdx.
1

1
ln x  u  x   x dx  u '  x  dx
Đặt  3 
 x  v '  x  v  x   1 x 4
 4
e4 1 4 e 3e4  1
e e
1 4 1 4 1
I  x .ln x   x . dx   x 
4 1 1
4 x 4 16 1 16

4  x ln 3 x
e
Câu 35. Tính tích phân: I   dx
1
x2

ee
1 ln 3 x 4e 4
I  4 2 dx   2 dx    I1    4  I1
1
x 1
x x1 e
e
ln 3 x
Tính I1   2 dx
1
x

1
Đặt t  ln x  dt  dx
x
Đổi cận: x  1  t  0; x  e  t  1
1
t4 1 1
I1   t dt  3

0
4 0 4
y

Vậy
x
-8 -6 -4 -2 2 4 6 8

-5

1
Câu 36. Tính tích phân sau I   (2x+e x )dx
0

1 1 1
I    2 x  e  dx   2 xdx   e x dx  x 2  e x  1  0  e  1  e
x 1 1

0 0
0 0 0

NGUYỄN VĂN LỰC  0933.168.309 SP Toán K35 - ĐH Cần Thơ


Nguyên hàm – Tích phân FB: http://www.facebook.com/VanLuc168
1
Câu 37. Tính tích phân sau I   2 x  e x  3 dx
0

x 1
 2e 
1
 2e  1  3
1 1 1
2x
I   2  e  3 dx    2e  dx 3 2 dx  3  
x x x x

0 0 0 ln 2e ln 2 0  ln 2e  ln 2
0

 1
e
2 ln x  1 
I   
 x x  ln x  1 
Câu 38. Tính các tích phân sau dx
1 

 
e
1
 Tính I1   dx ta được kết quả I1  2 e 1
1 x
dx
 Đặt ln x  t ta được dt 
x
 Đổi cận x  1  t  0; x  e  t  1
2t  1
1
dt   2t  ln  t  1   2  ln 2
1
 Khi đó K 2  
0
t 1 0

 Vậy ta được I  I1  I 2  2 e  ln 2

 
ln 2
1
Câu 39. Tính các tích phân sau I   x  2e
0
x  dx
1

ln 2
1
 Tính I1   xdx
0
ta được kết quả I1  ln 2 2
2
ln 2
1
 Tính I 2   2e
0
x
1
dx

 Đặt e x  t ta được e x dx  dt
 Đổi cận x  0  t  1; x  ln 2  t  2
2
  ln t  ln  2t  1   ln 2  ln  ln
dt 2 5 6
 Khi đó I 2  
1 t  2t  1 3 5
1

1 6
Vậy ta được L  L1  L2  ln 2 2  ln
2 5

e
Câu 40. Tính các tích phân I   x ln xdx
1

 1
 du  dx
u  ln x  x
 
 dv  xdx v  x
2

 2

NGUYỄN VĂN LỰC  0933.168.309 SP Toán K35 - ĐH Cần Thơ


Nguyên hàm – Tích phân FB: http://www.facebook.com/VanLuc168
e e 2 e
e2  1
e
x2 x x2 x
I ln x   dx  ln x  
2 1 1
2 2 1
4 1
4

1
Câu 41. Tính các tích phân I   xe x dx
0

u  x du  dx 1
. I  xe 0   e x dx  e  e x 0  1
x 1 1
 
dv  e dx v  e
x x
0

4. Tích phân hàm lượng giác



2
Câu 42. Tính tích phân I   ( x  sin 2 x ) cos xdx.
0

  
2 2 2
I   ( x  sin x ) cos xdx   x cos xdx   sin 2 x cos xdx .
2

0 0 0
M N
Tính M
u  x du  dx
Đặt  
dv  cos xdx v  sin x

 
2
 
M  x sin x 2   sin xdx   cos x 2   1.
2 2
0 0 0
Tính N
Đặt t  sin x  dt  cos xdx

x
 t 1
Đổi cận 2
x0t 0
1 3
t 1 1
N   t 2 dt   .
0
3 0 3
 2
Vậy I  M  N   .
2 3


2
Câu 43. Tính tích phân: I   2 x cos 2 xdx
0

 
2 2
I   xdx   x cos 2 xdx
0 0

NGUYỄN VĂN LỰC  0933.168.309 SP Toán K35 - ĐH Cần Thơ


Nguyên hàm – Tích phân FB: http://www.facebook.com/VanLuc168
 
2
x2 2 2
+  xdx  
0
2 0
8
  
2 
12 1 2
+ J   xcos2 xdx  x sin 2 x 02   sin 2 xdx  cos2 x  0
0
20 4 0

2
I
8


2
Câu 44. Tính tích phân I =  ( x  cos2 x) sin xdx .
0

   
2 2 2 2
I   x sin xdx   cos2 x sin xdx . Đặt I1   x sin xdx, I 2   cos 2 x sin xdx
0 0 0 0

u  x du  dx  2 
Đặt    I1   x cos x 2   cos xdx  sin x 2 1
dv  sin xdx v   cos x 0
0
0

  
2 2 3
cos x 2 1
I 2   cos2 x sin xdx    cos2 xd (cos x)    .
0 0
3 0 3
1 4
Vậy I  1   .
3 3

Câu 45. Tính tích phân: I (1 cos x )xdx


0

I (1 cos x )xdx xdx x cos xdx


0 0 0

x2 2
02 2
 Với I 1 xdx
0
2 0 2 2 2

 Với I 2 x cos xdx


0

u x du dx
 Đặt . Thay vào công thức tích phân từng phần ta được:
dv cos xdx v sin x

I2 x sin x 0 sin xdx 0 ( cos x ) 0 cos x 0 cos cos 0 2


0

2
 Vậy, I I1 I2 2
2

NGUYỄN VĂN LỰC  0933.168.309 SP Toán K35 - ĐH Cần Thơ


Nguyên hàm – Tích phân FB: http://www.facebook.com/VanLuc168

2
Câu 46. Tính Tích phân I   x cos xdx
0


2
I   x cos xdx ,
0

u  x du  dx
Đặt  
dv  cos xdx v  sin x

 2
 

I  x sin x 02   sin xdx   cos x 02  1
0
2 2

  
2 cot  x  
Câu 47. Tính tích phân sau: I    6
dx
 
2
 3 cos x  sin x
3

 3 1   
+ Ta có: 3 cos x  sinx  2  cos x  sin x   2cos  x  
 2 2   6
   
cot  x   


2
6 2
1   1  2 1
+ Do đó: I   dx   d tan  x    ln tan  x    ln 3
     6 4  6 4

4 cos 2  x   
4 tan  x   3
3
 6 3
 6


2
Câu 48. Tính các tích phân: I   sin 2 x. sin 3 x.dx
0


2
Tính các tích phân: I   sin 2 x. sin 3 x.dx
0

2
I =  2 sin 4 x. cos x.dx .
0

Đặt t=sinx => dt=cosxdx


1 1
t5 2
▪ I   2t dt = 2
4
= .
0
5 0 5

Câu 49. Cho hàm số f ( x)  tan x2 cot x  2 cos x  2 cos 2 x  có nguyên hàm là F (x ) và
  
F    . Tìm nguyên hàm F (x ) của hàm số đã cho.
4 2

Tìm nguyên hàm F (x )


  
F ( x)   tan x 2 cot x  2 cos x  2 cos 2 x dx =  2  2 sin x  sin 2 x dx 

NGUYỄN VĂN LỰC  0933.168.309 SP Toán K35 - ĐH Cần Thơ


Nguyên hàm – Tích phân FB: http://www.facebook.com/VanLuc168
cos 2 x
 2 x  2 cos x  C
2
   2 
F    2.  2.  0  C   C  1
4 4 2 2
cos 2 x
Vậy F ( x)  2 x  2 cos x  1
2


2
Câu 50. Tính tích phân I   2x  1  sin x  dx .
0

   
2 2 2 2
I   2x  1  sin x  dx   2x.dx   dx   sin xdx  A  B  C
0 0 0 0
 
 
2
2 2

A   2x .dx  x 2 2
 ; B   dx  x 02 
0
0 4 0
2

2 
C   sin xdx  cosx   2
0
1
0

2 
Vậy I  A  B  C   1
4 2


4

 x tan
2
Câu 51. Tính tích phân I = xdx
0

  
4 44
1 1
I=  x(
0
cos 2 x
 1) dx  0 cos 2 x
x. dx  0 xdx
 
4
x2 4 2
0 xdx 
2

32
0

u  x
du  dx
4

Đặt 
1
. x dx  I1 dx  
cos 2 x  dv  v  tan x

0
cos 2 x

 4
 

I1 = x tan x 04   tanxdx   ln cos x 4
0
  ln 2 .
0
4 4
 2
Vậy I=  ln 2  .
4 32

Câu 52. Tính nguyên hàm I    x  2  sin 3xdx

Tính nguyên hàm I    x  2  sin 3xdx

NGUYỄN VĂN LỰC  0933.168.309 SP Toán K35 - ĐH Cần Thơ


Nguyên hàm – Tích phân FB: http://www.facebook.com/VanLuc168
du  dx
u  x  2
Đặt  , ta được  cos 3 x
dv  sin 3xdx  v
 3
 x  2  cos 3x  1 cos 3xdx    x  2  cos 3x  1 sin 3x  C
3
Do đó: I  
3 3 9


Câu 53. Tính tích phân sau: I    s inx+ cos x  dx
0

  
I    s inx  cos x  dx   s inxdx   cos xdx   cos x 0  sin x 0  2
 

0 0 0


Câu 54. Tính tích phân sau: I    x  sin 2 x  dx
0

    
1 1 2
I    x  sin 2 x  dx   xdx   sin 2 xdx  x 2  cos 2 x 
0 0 0 2 0 2 0 2


Câu 55. Tính tích phân sau: I   1  sin 3 x  cos xdx
0


2
I   1  sin 3 x  cos xdx
0

 Đặt sin x  t  dt  cos xdx



 Đổi cận x  0  t  0; x   t  1
2
1
1
 t4 
 Khi đó I   1  t dt   t   
3 3
0  4 0 4


4
1
Câu 56. Tính tích phân sau I   dx
 sin x cos 4 x
2

1
 Đặt cot x  t  dt  dx
sin 2 x
 
 Đổi cận x   t  3; x   t 1
6 4
2 3
 1  2 1  2 1 
3 3
8 3 4
Khi đó I   1  2  dt  1  t 2 t 4   t 3t 3  1 27  3
 1   dt   t    
1  t 

NGUYỄN VĂN LỰC  0933.168.309 SP Toán K35 - ĐH Cần Thơ


Nguyên hàm – Tích phân FB: http://www.facebook.com/VanLuc168

Câu 57. Tính tích phân sau: I    s inx  x  sin xdx
0

  
I    s inx  x  sin xdx   sin 2 xdx   x sin xdx
0 0 0
 
1  cos 2 x 1
 Đặt I1   sin 2 xdx   dx  
0 0 2 2

 I 2   x sin xdx
0

u  x du  dx
  
dv  sin xdx v   cos x

 
 I 2   x cos x 0   cos xdx    s inx 0  
0
3
Khi đó I  
2


2
Câu 58. Tính các tích phân I   x sin xdx
0

u  x du  dx
Đặt  
dv  sin xdx v   cos x

 2 
I   x cos x 02   cos xdx  0  0  sinx 02  1
0

NGUYỄN VĂN LỰC  0933.168.309 SP Toán K35 - ĐH Cần Thơ


Nguyên hàm – Tích phân FB: http://www.facebook.com/VanLuc168

III. ỨNG DỤNG CỦA TÍCH PHÂN


Chuyên đề: Nguyên hàm – Tích phân

A. Tóm tắt lí thuyết


I. CÔNG THỨC
1. Công thức tính diện tích hình phẳng

y (C 2 ) : x  g ( y)
(C1 ) : x  f ( y )
y xb (C ) : x  g ( y )
xa b yb  2
(C1 ) : y  f ( x) (C1 ) : y  f ( x) (H ) : 
(C ) : y  g ( x)
 2 (H )  1 : y  a
(H ) :  (C 2 ) : y  g ( x) (H ) 
 2 : y  b
 1 : x  a a ya

 2 : x  b x
x
O a b O
(C1 ) : x  f ( y)

b b

S f ( x) g ( x) dx S f ( y) g ( y ) dy
a a

2. Công thức tính thể tích vật thể tròn xoay


y
y xb
xa (C ) : y  f ( x)
b yb
x0 (C ) : x  f ( y )

a ya
x x
O a y0 b O

2 2
V     f ( x) dx V     f ( y ) dy
b b

a a

NGUYỄN VĂN LỰC  0933.168.309 SP Toán K35 - ĐH Cần Thơ


Nguyên hàm – Tích phân FB: http://www.facebook.com/VanLuc168
II. CÁC VÍ DỤ
Ví dụ 1: Tính diện tích hình phẳng giới hạn bởi đường cong y x2 x 3 và đường
thẳng y 2 x 1.

Bài giải
♥ Phương trình hoành độ giao điểm của hai đường
x 1
x2 x 3 2x 1 x2 3x 2 0
x 2

♥ Diện tích hình phẳng cần tìm là


2

S x2 3x 2 dx
1

2 2
x3 3x 2 1
x 2
3x 2 2x . 
1
3 2 1
6

Ví dụ 2: Tính thể tích khối tròn xoay tạo thành khi quay hình phẳng được giới hạn bởi
1
các đường y  , y  0, x  0 và x 1 xung quanh trục hoành.
1  4  3x
Bài giải
1
dx
♥ Thể tích khối tròn xoay là V  .
1  
2
0 4  3x
4  t2 2t
♥ Đặt t  4  3x , ta có khi x0 thì t  2, khi x 1 thì t 1 và x nên dx   dt .
3 3

2t 2 2  1 1 
1 2 2
1 t
Khi đó ta có V  . dt   dt      dt
2
(1  t ) 3
2
3 1 (t  1) 2
3 1  t  1 (t  1) 2 
2  1  2 2  3 1    3 
  ln | t  1|     ln     6ln  1 . 
3  t 1 1 3  2 6 9  2 

NGUYỄN VĂN LỰC  0933.168.309 SP Toán K35 - ĐH Cần Thơ


Nguyên hàm – Tích phân FB: http://www.facebook.com/VanLuc168

BÀI TẬP TỰ LUYỆN

Câu 1. Tính diện tích hình phẳng giới hạn bởi các đường y  e x  1 ,trục hoành, x = ln3
và x = ln8.

ln8
Diện tích S  
ln 3
e x  1dx ; Đặt t  e x  1  t 2  e x  1  e x  t 2  1

2t
Khi x = ln3 thì t = 2 ; Khi x = ln8 thì t = 3; Ta có 2tdt = e xdx  dx  dt
t 1
2

 t 1  3
3 3
Do đó S   2 dt    2  2  dt   2t  ln  3
2t 2 2
  2  ln   (đvdt)
2
t 1 2
t 1  t 1  2 2

x 1
Câu 2. Tính diện tích hình phẳng giới hạn bởi đồ thị hàm số y  và các trục tọa độ.
x2

x 1
0
Đồ thị hàm số cắt trục hoành tại (-1; 0). Do đó S   dx
1
x2

x 1
0 0
3 2 3
1 x  2dx =  (1  x  2 )dx  ( x  3ln x  2 )|
0
Ta có S  1
 1  3ln  3ln  1 .
1
3 2

Câu 3. Tính diện tích hình phẳng giới hạn bởi đường cong y  ( x  1) ln x và đường thẳng
y  x  1.

+) Xét phương trình: (x-1)lnx = x-1 x = 1 hoặc x = e.


y

x
-8 -6 -4 -2 2 4 6 8

-5

+ Diện tích cần tìm là:


e e e
x2
S   ( x  1)(ln x  1) dx   ( x  1)(ln x  1)dx   (ln x  1)d (  x ) 
2
1 1 1
e
x2 x 1 1 
 (  x )(ln x  1) |1e   (  1)dx     x 2  x  |1e
2 2 2 4 
1
e 2  4e  5
 (đvdt).
4

Câu 4. Tính diện tích hình phẳng giới hạn bởi các đường sau y  x2 , trục hoành và hai
đường thẳng x=0, x=2.

y  x2 , trục hoành và hai đường thẳng x= 0, x=2.


Trên [0; 2] ta có x2  0  x  0  [0;2]
Diện tích của hình phẳng đã cho:

NGUYỄN VĂN LỰC  0933.168.309 SP Toán K35 - ĐH Cần Thơ


Nguyên hàm – Tích phân FB: http://www.facebook.com/VanLuc168
2 2
1 8
S   x 2 dx  x 3 
0
3 0 3

Câu 5. Tính diện tích hình phẳng giới hạn bởi các đường sau y  x2 , y  2 x  3 và hai
đường thẳng x =0, x=2.

Đặt f1 ( x)  x 2 , f 2 ( x)  2 x  3
 x  1 [0;2]
Ta có: f1 ( x)  f 2 ( x)  0  x 2  (2 x  3)  0  x 2  2 x  3  0  
 x  3  [0;2]
Diện tích hình phẳng đã cho
2
S   | x 2  2 x  3 | dx
0
1 2
  ( x  2 x  3)dx   ( x 2  2 x  3)dx
2

0 1
1 2
 x3   x3 
   x 2  3x     x 2  3x 
 3 0  3 1
1 8 1 5 7
  2   4  6  1 3    4
3 3 3 3 3

Câu 6. Tính diện tích hình phẳng giới hạn bởi các đường sau y  x 2 , y  x  2

 x  1
Ta có: x 2  ( x  2)  0  x 2  x  2  0  
x  2
Diện tích hình phẳng
2
2
 x3 x 2  8 1 1 9
S   | x  x  2 | dx     2x    2  4    2 
2

1  3 2  1 3 3 2 2

Câu 7. Tính diện tích hình phẳng giới hạn bởi các đường sau đây:
y x 3 4x 2 3x 1 và y 2x 1 .

x 1
Cho x 3 4x 2 3x 1 2x 1 x3 4x 2 5x 2 0
x 2
2
 Diện tích cần tìm là: S x3 4x 2 5x 2 dx
1
2
2 x4 4x 3 5x 2 1 1
hay S (x 3 4x 2 5x 2)dx 2x (đvdt)
1 4 3 2 1 12 12

Câu 8. Tính diện tích hình phẳng giới hạn bởi đồ thị C : y  x 3  3x 2  4 và đường thẳng
: y   x  1
NGUYỄN VĂN LỰC  0933.168.309 SP Toán K35 - ĐH Cần Thơ
Nguyên hàm – Tích phân FB: http://www.facebook.com/VanLuc168

Phương trình hoành độ giao điểm của C  và  là:


 x 3  3x 2  4  x  1  x 3  3x 2  x  3  0
 x  1
  x  3
 x  1
Diện tích hình phẳng phải tìm:

  x  3x  4   x  1 dx 
3 3

S  x  3x 2  x  3 dx
3 2 3

1 1
1 3

  x  3x 2  x  3 dx    x 3  3x 2  x  3 dx
3

1 1

  x  3x 2  x  3dx    x  3x 2  x  3dx
1 3

 3 3

1 1
1 3
 x4 x2   x4 x2 
   x 
3
 3x     x 
3
 3x 
 4 2  1  4 2 1
  4  4  8 (đvdt)

Câu 9. Tính diện tích hình phẳng giới hạn bởi đồ thị hàm số y  ( x  1).( x 2  2 x) với trục
hoành.

Ta có
x  0
( x  1).( x 2  2 x)  0   x  1 
 x  2
Do đó diện tích cần tìm là
2
S   ( x  1).( x 2  2 x) dx
0
1 2
  ( x  1).( x 2  2 x) dx   ( x  1).( x 2  2 x) dx
0 1
1 2
  ( x  1).( x 2  2 x)dx   ( x  1).( x 2  2 x)dx
0 1
1 2
1 1
  ( x 2  2 x)d ( x 2  2 x)   ( x 2  2 x)d ( x 2  2 x)
20 21
1 1 2

  ( x 2  2 x) 2  ( x 2  2 x) 2 
4  0 1 
1
 1  (0  1)
4
1
 .
2

Câu 10. Tính thể tích vật thể tròn xoay sinh ra khi quay hình (D) quanh trục Ox biết (D)
giới hạn bởi y  1  x 2 , y  0

NGUYỄN VĂN LỰC  0933.168.309 SP Toán K35 - ĐH Cần Thơ


Nguyên hàm – Tích phân FB: http://www.facebook.com/VanLuc168

Ta có: 1  x2  0  x  1
b
Áp dụng công thức: V    f 2 ( x)dx
a
1
1
 2x 3 x5 
1
Ta có: V    (1  x ) dx    1  2x  x  dx    x 
2 2 2 4
 
1 1  3 5  1

 2 1   2 1   4 2  16
  1      1        2    
 3 5   3 5   3 5  15

Câu 11. Gọi (H) là hình phẳng giới hạn bởi đồ thị (C): y  x sin x , các trục Ox, Oy và

đường thẳng x  . Tính thể tích khối tròn xoay sinh ra khi cho (H) quay quanh Ox.
4

Thể tích khối tròn xoay cần tính là



 
4
1  cos 2 x   

V=   ( x sin x) 2 dx =  04 x.sin 2 xdx   04 x. dx    4 xdx   4 x cos 2 xdx 
0 2 2 0 0

 
4
x 2 4
2
+  xdx =  .
0
2 0
32

1
+  0
4
x cos 2 xdx . Đặt từng phần u = x, dv = cos 2xdx. Ta có du = dx, v =
2
sin 2x.

 1  2
Từ đó, tính được  0
4
x cos 2 xdx =  . Do đó, V =
8 4 64
(  4  8) .

Câu 12. Tính thể tích vật thể tròn xoay thu được khi quay hình phẳng giới hạn bởi các

đường y = tanx ; y=0 ; x=0; x = quanh trục Ox.
4

Gọi V là thể tích của vật thể cần tìm:


  
4 4 1 
V    tan 2 xdx   (  1) dx   (tan x  1) 04   (1  )
0 2
0 cos x 4

Câu 13. Tính thể tích của vật thể tròn xoay sinh ra bởi hình phẳng giới hạn bởi các đường
y  x3  1 ,y =0,x =0,x =1 khi quay xung quanh trục Ox.

Phương trình hoành độ giao điểm của đồ thị y  x3  1 và y=0:


x3  1  0  x  1  0;1
Gọi V là thể tích của vật thể cần tìm:
1
1 1
 x7 1 4   1 1  23
V    ( x  1) dx    ( x  2 x  1)dx
3 2
    x  x       1   (đvtt )
6 3

0 0  7 2 0  7 2  14

NGUYỄN VĂN LỰC  0933.168.309 SP Toán K35 - ĐH Cần Thơ


Số phức FB: http://www.facebook.com/VanLuc168

CHUYÊN ĐỀ: SỐ PHỨC

A. Tóm tắt lí thuyết


I. SỐ PHỨC & CÁC PHÉP TOÁN
1. Số phức là một biểu thức dạng a + bi, trong đó a, b là các số thực và số i thỏa mãn
i 2  1 . Kí hiệu z  a  bi
 i: đơn vị ảo,  a: phần thực,  b: phần ảo.
Chú ý:
 z  a  0i  a được gọi là số thực (a   )
 z  0  bi  bi được gọi là số ảo (hay số thuần ảo)
 0  0  0i vừa là số thực vừa là số ảo y
M
2. Biểu diễn hình học của số phức. b

 M(a;b) biểu diễn cho số phức z  z = a + bi a


O x
3. Hai số phức bằng nhau. Cho hai số phức z  a  bi và z '  a ' b 'i với a, b, a ', b '
a  a '
z  z'  
b  b '
4. Cộng và trừ số phức. Cho hai số phức z  a  bi và z '  a ' b 'i với a, b, a ', b '
z  z '   a  a '   b  b ' i

z  z '   a  a '   b  b ' i


5. Nhân hai số phức. Cho hai số phức z  a  bi và z '  a ' b 'i với a, b, a ', b '
z.z '   aa ' bb '    ab ' a ' b  i

y
6. Môđun của số phức z = a + bi b
M

 z  a 2  b2  OM
O a x
7. Số phức liên hợp của số phức z = a + bi là z  a  bi

 zz
 zz
 z  z  2a
 z.z  a 2  b 2  z
2

NGUYỄN VĂN LỰC  0933.168.309 SP Toán K35 - ĐH Cần Thơ


Số phức FB: http://www.facebook.com/VanLuc168
8. Chia hai số phức.
Cho hai số phức z  a  bi và z '  a ' b 'i với a, b, a ', b '
z ' z ' z z ' z ac  bd ad  bc
o Thương của z’ chia cho z (z  0) :   2  2  i
z zz z a  b2 a 2  b2

II. PHƯƠNG TRÌNH BẬC NHẤT, BẬC HAI TRÊN TẬP SỐ PHỨC
1. Căn bậc hai của số phức
o z0 có một căn bậc hai là 0
o za là số thực dương có 2 căn bậc 2 là  a
o za là số thực âm có 2 căn bậc hai là  a .i

2. Phương trình bậc nhất ax + b = 0 (a, b là số phức cho trước, a  0 ).


Giải tương tự phương trình bậc nhất với hệ số thực
3. Phương trình bậc hai ax2 + bx + c = 0 (a, b, c là số thực cho trước, a  0 ).
Tính   b 2  4ac
b  
o   0: Phương trình có hai nghiệm phân biệt thực x1 ,2 
2a
b  i 
o   0 : Phương trình có hai nghiệm phân biệt phức x1 ,2 
2a
b
o   0 : Phương trình có 1 nghiệm kép là x
2a

III. CÁC VÍ DỤ
Dạng 1: Tìm số phức thỏa mãn các điều kiện cho trước.

Ví dụ 1: Cho số phức z thỏa mãn điều kiện 2 z iz 2 5i . Tìm phần thực và phần ảo
của z
Bài giải
♥ Đặt z a bi , a, b ta có:

2 z iz 2 5i 2 a bi i a bi 2 5i

2a b 2b a i 2 5i

2a b 2 a 3
a 2b 5 b 4

♥ Vậy số phức z cần tìm có phần thực bằng 3 và phần ảo bằng 4 

NGUYỄN VĂN LỰC  0933.168.309 SP Toán K35 - ĐH Cần Thơ


Số phức FB: http://www.facebook.com/VanLuc168
Ví dụ 2: Cho số phức z thỏa mãn điều kiện 3z z 1 i 5z 8i 1 . Tính môđun của z

Bài giải
♥ Đặt z a bi , a, b ta có:

3z z 1 i 5z 8i 1 3 a bi a bi 1 i 5 a bi 8i 1

3a 4b 2a b i 1 8i

3a 4b 1
2a b 8

a 3
b 2

♥ Vậy môđun của z là z 


2
a2 b2 32 2 13

Ví dụ 3: Cho số phức z thỏa mãn điều kiện 2 z 3 1 i z 1 9i . Tính môđun của z

Bài giải
♥ Đặt z a bi , a, b ta có:

2z 31 i z 1 9i 2 a bi 3 1 i a bi 1 9i

5a 3b 3a b i 1 9i

5a 3b 1 a 2
3a b 9 b 3

♥ Vậy môđun của z là z a2 b2 22 32 13 

Ví dụ 4: Cho số phức z thỏa mãn điều kiện z 2 i z 3 5i . Tìm phần thực và phần

ảo của z
Bài giải
♥ Đặt z a bi , a, b ta có:

z 2 i z 3 5i a bi 2 i a bi 3 5i

3a b a b i 3 5i

3a b 3 a 2
a b 5 b 3

♥ Vậy số phức z cần tìm có phần thực bằng 2 và phần ảo bằng 3 

NGUYỄN VĂN LỰC  0933.168.309 SP Toán K35 - ĐH Cần Thơ


Số phức FB: http://www.facebook.com/VanLuc168
z 1
Ví dụ 5: Tìm số phức z thỏa mãn ( z  1)(1  i )   | z |2 .
1 i

Bài giải
Đặt z x yi, (x, y ) ta có:
z 1 2 x 1 yi (1 i )
( z  1)(1  i )   z x 1 yi (1 i ) x2 y2
1 i 2
3x 1 y (3 x 1 y )i 2( x 2 y2 )

3x  1  y  2( x 2  y 2 )

3x  1  y  0
y (3x 1)
2
10 x 3x 0
x 0, y 1
3 1
x , y .
10 10
3 1
♥ Vậy số phức z cần tìm là z  i hoặc z  i. 
10 10

9
Ví dụ 6: Tìm số phức z thỏa mãn z  3i  1  i z và z  là số thuần ảo.
z

Bài giải
♥ Đặt z a bi (a, b ) ta có:
| z  3i |  | 1  iz | |a (b 3)i | |1 i ( a bi ) |
| a (b 3)i | |1 b ai |
a2 (b 3)2 (1 b)2 ( a) 2
.
b 2
9 9 9(a  2i ) a 3  5a  (2a 2  26)i
z  a  2i   a  2i  2  là số ảo
z a  2i a 4 a2  4
a  5a  0
3

a  0, a   5 .
♥ Vậy các số phức cần tìm là z  2i, z  5  2i, z   5  2i .
z 2i
Ví dụ 7: Tìm số phức z thỏa mãn z z 2 2i và là số thuần ảo.
z 2

Bài giải
♥ Đặt z x yi (x, y ) và z 2 ta có:
z z 2 2i x yi x 2 (y 2)i
2 2
x y (x 2) 2 (y 2) 2
x y 2 y 2 x. (1)
z  2i x  ( y  2)i [ x  ( y  2)i].[( x  2)  yi]
Ta có  
z  2 ( x  2)  yi ( x  2) 2  y 2

NGUYỄN VĂN LỰC  0933.168.309 SP Toán K35 - ĐH Cần Thơ


Số phức FB: http://www.facebook.com/VanLuc168
x( x  2)  ( y  2) y ( x  2)( y  2)  xy
  i là số ảo khi và chỉ khi
( x  2) 2  y 2 ( x  2) 2  y 2
x( x  2)  ( y  2) y 
 x  y  2( x  y )
2 2
 0   (2)
( x  2) 2  y 2 
( x  2)  y  0
2 2


(2) ta được ( x  1)  1  x  0 . Suy ra y  2 .
2
Thay (1) vào
x  2
♥ Vậy các số phức cần tìm là z  2i. 

4
Ví dụ 8: Cho số phức z thỏa mãn z   i. Tính A  1  (1  i) z .
z 1

Bài giải
♥ Đặt z a bi, (a, b ) ta có:
4
z i a 2  b 2  a  4  bi  b  (a  1)i
z 1
a 2  b 2  a  4  b

 b  a  1
a  1, b  2

a  2, b  1
♥ Vậy
Với a  1, b  2 ta có A  1  (1  i)(1  2i)  3i  3.
Với a  2, b  1 ta có A  1  (1  i)(2  i)   3i  3. 

Ví dụ 9: Tìm số phức z thỏa mãn đẳ ng thức z  1  i  2z  z  5  3i sao cho z  2  2i đa ̣t


giá tri ̣nhỏ nhấ t.
Bài giải
♥ Đặt z x yi , x, y ta có:
z  1  i  (x  1)  (y  1)i = (x  1)2  (y  1)2
2z  z  5  3i = 3x  5  (y  3)i = (3x  5)2  (y  3)2 .
Do đó: z  1  i  2z  z  5  3i  (x + 1)2 + (y + 1)2 = (3x – 5)2 + (y – 3)2.
 8y = 8x2 – 32x + 32
 y = x2 – 4x + 4.
2
2  3 7 7
♥ Ta có z  2  2i = (x  2)  (y  2) = y   y  2  = y  3y  4 =
2 2 2
y   ≥
 2 4 2

3 3 6
Dấ u "=" xảy ra  y =  (x – 2)2 =  x  2 .
2 2 2
6 3 6 3
♥ Vâ ̣y z  2   i hay z  2   i thì z  2  2i đa ̣t GTNN. 
2 2 2 2

NGUYỄN VĂN LỰC  0933.168.309 SP Toán K35 - ĐH Cần Thơ


Số phức FB: http://www.facebook.com/VanLuc168
Dạng 2: Tìm tập hợp điểm biểu diễn số phức thỏa điều kiện cho trước.

Ví dụ 10: Trong mặt phẳng tọa độ, tìm tập hợp điểm biểu diển các số phức z thỏa mãn
z i 1 i z

Bài giải
♥ Đặt z x yi x, y và M x; y là điểm biểu diễn của z trên mặt phẳng Oxy ta
có:
z i 1 i z x yi i 1 i x iy

x y 1i x y x y i

2 2 2
x2 y 1 x y x y

x2 y2 2y 1 x2 2 xy y2 x2 2 xy y2

x2 y2 2y 1 0

♥ Tập hợp điểm M biểu diển của số phức z là đường tròn có phương trình
x2 y2 2y 1 0 

BÀI TẬP TỰ LUYỆN

1. Tính toán với số phức

Câu 1. Cho số phức z thỏa mãn hệ thức: (1  2i) z  (2  3i) z  2  2i . Tính mô đun của z.

Gọi z=x+yi  x, y  R  . Phương trình đã cho trở thành: 1  2i  x  yi    2  3i  x  yi   2  2i


  x  2 y    2 x  y  i   2 x  3 y    3x  2 y  i  2  2i
  3x  5 y     x  y  i  2  2i
3x  5 y  2 x 1
 
  x  y  2 y 1
Do đó z  12  12  2

Câu 2. Tìm số phức z có môđun nhỏ nhất thỏa : z  1  5i  z  3  i .

Giả sử : z  x  yi,  x, y  
từ gt ,ta có : x  1   y  5 i  x  3   y  1 i ;

NGUYỄN VĂN LỰC  0933.168.309 SP Toán K35 - ĐH Cần Thơ


Số phức FB: http://www.facebook.com/VanLuc168
  x  1   y  5    x  3   y  1  x  3 y  4  0  x  4  3 y
2 2 2 2

Khi đó z  x2  y 2  10 y 2  24 y  16
8 2 6
z nhỏ nhất bằng khi và chỉ khi: z   i
5 5 5

Câu 3. Cho số phức z thỏa mãn z   2  3i  z  1  9i . Tìm môđun của số phức z.

Gọi z  a  bi, a, b  ; Khi đó z   2  3i  z  1  9i


 a  bi   2  3i  a  bi   1  9i  a  3b   3a  3b   1  9i
a  3b  1 a  2
  . Vậy môđun của số phức z là : z  22  (1)2  5
3a  3b  9 b  1

Câu 4. Cho số phức z thỏa mãn hệ thức: (2  i )(1  i )  z  4  2i . Tính môđun của z .

Đặt z  a  bi , ( a, b  ), khi đó z  a  bi . Theo bài ra ta có


(2  i )(1  i )  a  bi  4  2i  a  3  (1  b)i  4  2i
a  3  4 a  1
  . Do đó z  1  3i , suy ra z  12  32  10
1  b   2 b  3

Câu 5. Cho số phức z thỏa mãn điều kiện z   2  i  z  5  i . Tính mô đun của số phức
w  1  iz  z 2 .

3a  b  5 a  1
Đặt z  a  bi  a, b   . Từ giả thiết ta có:   .
a  b  1 b  2
Do đó z  1  2i .
Suy ra w  1  iz  z 2  1  i 1  2i   1  2i   3i . Vậy w  3 .
2

z2  2z  3
Câu 6. Tìm môđun của số phức z, biết z 
z 1

z2  2z  3
Tìm môđun của số phức z, biết z 
z 1
+ Điều kiện z  1 .
+ Gọi z  a  bi  a, b   ,
z2  2z  3
ta có : z    a  bi  a  bi  1   a  bi   2  a  bi   3
2

z 1
  2b 2  a  3   2ab  3b  i  0
 3
2b2  a  3  0 a  3 a   2
  hay 
2ab  3b  0 b  0 b   3
 2
Với a  3, b  0 , ta có z  a 2  b2  3 .

NGUYỄN VĂN LỰC  0933.168.309 SP Toán K35 - ĐH Cần Thơ


Số phức FB: http://www.facebook.com/VanLuc168
3 3 9 3
Với a   , b   , ta có z  a 2  b 2    3 .
2 2 4 4
Vậy môđun của số phức z là 3 hay 3 .

z  6  2i
Câu 7. Tìm môđun của số phức z thỏa mãn số phức là số thuần ảo và đồng thời
z  2  4i
z 6i  5

Đặt z=a+bi : Đk : z  2  4i
a 2  b 2  4a  2b  12  0 a  2 a  2
Theo đề bài :   (L)V   z  2  2i  z  2 2
 a  6    b  1  25 b  4 b  2
2 2

Câu 8. Cho số phức z thỏa mãn (1 i )z z 2 i . Tính môđun của số phức z .

Đặt z  a  bi,(a,b  ); khi đó z  a  bi . Do đó


(*) (1 i )(a bi ) (a bi 2)i (a b) (a b)i b (a 2)i
a  b  b a  4
   z  4 2  22  2 5
a  b  a  2 b  2
 

Câu 9. Cho số phức z thỏa mãn hệ thức: z  3  i   3  i . Tính môđun của số phức
1 1
 2  2
w 1 i  z .

1
3 i
 1  1 2  35  12 i
z3  i  3  i z 
 2  2 1
3 i 37 37
2

2 2
72 49  72   49  7585
w  1 i  z   i  w      
37 37  37   37  37

3
Câu 10. Trong các số phức thỏa mãn z  2  3i  . Tìm số phức z có môđun nhỏ nhất.
2

3 9
* Gọi z=x+yi. z  2  3i   …   x  2 2   y  32  .
2 4
26  3 13 78  9 13
* Vẽ hình |z|min z. ĐS: z   i.
13 26

z  11 z  4i
Câu 11. Cho số phức z thỏa mãn điều kiện  z  1 . Hãy tính .
z2 z  2i

NGUYỄN VĂN LỰC  0933.168.309 SP Toán K35 - ĐH Cần Thơ


Số phức FB: http://www.facebook.com/VanLuc168
z  11  z  2  3i
 z  1  z 2  4 z  13  0 ,  '  9  9i 2   z  2  3i
z2 
z  4i 2  i
 z  2  3i  = 1
z  2i 2  i
z  4i 2  7i 53
 z  2  3i  = 
z  2i 2  5i 29

Câu 12. Cho số phức z thỏa mãn điều kiện iz + 2 z  1  i . Tính mô đun số phức
w = iz+ 4

Gọi z  a  bi, (a, b  )

ta có:
 (2a  b)  (a  2b)i  1  i

 2a  b  1 a  1
 
a  2b  1 b  1
 z  1  i  w  5  i | w | 26

Câu 13. Gọi x1 , x2 là hai nghiệm trên tập số phức của phương trình x2  2x  5  0 .
Tính x1  x2

   4  4i 2 ,
x1  1  2i , x2  1  2i , x1  x2  2 5

Câu 14. Gọi z1 , z2 là hai nghiệm phức của phương trình z 2  4 z  29  0 .


Tính A  z1  z2 .
4 4

 '  25  0 . Phương trình đã cho có hai nghiệm phức z1  2  5i, z2  2  5i .


Khi đó z1  z2  29  A  1682 .

Câu 15. Cho z là số phức. Tìm m để phương trình mz 2  (m  1)z  i  0 có hai nghiệm
phân biệt z1 ; z 2 sao cho | z1 |  | z 2 | 2

Để pt có 2 nghiệm (*)
Với thì pt đã cho là pt bậc hai có nên pt có 2 nghiệm

Theo bài ra :
Kết hợp với điều kiện (*) ta được thỏa mãn bài toán

NGUYỄN VĂN LỰC  0933.168.309 SP Toán K35 - ĐH Cần Thơ


Số phức FB: http://www.facebook.com/VanLuc168

Câu 16. Gọi z1; z2 là 2 nghiệm phức của phương trình sau:
z 2  z  1  0,( z  C ) . Tính A= z1  z2

1 8 1 8
z1   i; z2   i
2 2 2 2
1 8 3
z1  z2   i   z1  z2  3
2 2 2

Câu 17. Cho z1 , z 2 là các nghiệm phức của phương trình 2 z 2  4 z  11  0 .


2 2
z  z2
Tính giá trị của biểu thức A = 1 .
( z1  z2 ) 2

3 2 3 2
Giải pt đã cho ta được các nghiệm: z1  1  i, z2  1  i
2 2
2
3 2  22
Suy ra | z1 || z2 | 1  
2
  ; z1  z2  2
 2  2
2 2
z  z2 11
Đo đó 1  ... 
( z1  z2 ) 2 4

 
Câu 18. Tính mô đun của số phức z biết rằng:  2 z  11  i   z  1 1  i   2  2i

Gọi z= a+ bi (a, b  R )
Ta có
 2 z  11  i    z  1 1  i   2  2i
  2a  1  2bi  1  i    a  1  bi  1  i   2  2i
  2a  2b  1   2a  2b  1 i   a  b  1   a  b  1 i  2  2i
 1
 a 
3a  3b  2
  3a  3b    a  b  2  i  2  2i  
3

a  b  2  2 b   1
 3
2
Suy ra mô đun: z  a 2  b2 
3

2. Tìm số phức Z

Câu 19. Cho số phức z thỏa mãn điều kiện (1  i ) z  1  3i  0 . Tìm phần ảo của số phức
w  1  zi  z

1  3i
(1  i) z  1  3i  0  z   2i
1 i
NGUYỄN VĂN LỰC  0933.168.309 SP Toán K35 - ĐH Cần Thơ
Số phức FB: http://www.facebook.com/VanLuc168
 w = 2 – i. Số phức w có phần ảo bằng - 1

Câu 20. Tìm phần thực và phần ảo của số phức w  ( z  4i)i biết z thỏa mãn điều kiện
1  i  z   2  i  z  1  4i.

Giả sử z  x  yi,  x. y   , suy ra z  x  yi.


Thế vào gt ta tìm được x= 3, y = 4.
Vậy z = 3 +4i. Do đó w = 3i
w có phần thực 0; phần ảo 3.

Câu 21. Tìm phần thực và phần ảo của số phức z thoả mãn điều kiện
z  (2  i) z  3  5i

Giả sử ,z=x+yi(x,y  R ).Ta có


z  (2  i) z  3  5i x+yi +(2+i)(x-yi)=3+5i
3x+y+(x-y)i=3+5i
3x  y  3  x2
  
 x y 5  y  3
Vậy phần thực và phần ảo của số phức z lần lượt bằng 2,-3

3  4i
Câu 22. Tìm phần thực và phần ảo của số phức: z   (3  5i )(6  i )
3  2i

Ta có
(3  4i )(3  2i )
z  18  3i  30i  5i 2
3 2
2 2

9  6i  12i  8i 2
  23  27i
32  22
1  18i 298 333
  23  27i    i
13 13 13
298 333
Vậy phần thực:  , phần ảo:
13 13

Câu 23. Cho số phức z 1 3i . Tìm số nghịch đảo của số phức: z2 z .z

Với z 1 3i , ta có
 z2
z.z (1 3i)2 (1 3i)(1 3i) 1 6i 9i 2 12 9i 2 2 6i
1 1 2 6i 2 6i 2 6i 1 3
 2
i
2 6i (2 6i)(2 6i) 2 36i 2 40 10 10

Câu 24. Cho số phức: z  3  2i .Xác định phần thực và phần ảo của số phức z 2  z .

z 2  z   3  2i    3  2i   8  14i
2

Phần thực a=8; phần ảo b=-14

NGUYỄN VĂN LỰC  0933.168.309 SP Toán K35 - ĐH Cần Thơ


Số phức FB: http://www.facebook.com/VanLuc168

Câu 25. Tìm phần ảo của z biết: z  3z   2  i   2  i  .


3

z  3z   2  i   2  i  (1)
3

Giả sử z=a+bi

(1)  a  bi  3a  3bi  8  12i  6i 2  i 3   2  i    2  11i . 2  i 


15
 4a  2bi  4  2i  22i  11i 2  20i  15  a  ; b  10 .
4
Vậy phần ảo của z bằng -10.

Câu 26. Tìm số phức liên hợp của


1
z  (1  i )(3  2i ) 
3i

3i 3i
Ta có z  5  i   5i  .
(3  i )(3  i ) 10
53 9
Suy ra số phức liên hợp của z là: z  i
10 10

Câu 27. Tìm phần thực và phần ảo của số phức z biết: z  2 z  3  2i .

Gọi z  a  bi (a, b  R )  z  a  bi
Ta có : 3a + bi = 3-2i
Suy ra : a=1 và b = -2
Vậy phần thực là 1 và phần ảo là -2

Câu 28. Cho số phức z  3  2i . Tìm phần thực và phần ảo của số phức w  iz  z .

z  3  2i
w  i  3  2i    3  2i 
 1  i
Phần thực là -1, phần ảo là 1.

 z  z  10.
Câu 29. Tìm phần thực, phần ảo của các số phức z, biết: 
 z  13.

 z  z  10.
Tìm phần thực, phần ảo của các số phức z, biết: 
 z  13.
Giả sử z = x + yi => z = x– yi. (x, yIR)
2 x  10. x  5
Theo đề bài ta có :   .
 x  y  13.  y  12
2 2

NGUYỄN VĂN LỰC  0933.168.309 SP Toán K35 - ĐH Cần Thơ


Số phức FB: http://www.facebook.com/VanLuc168
3  5i
Câu 30. Tìm phần thực và phần ảo của số phức sau: z    5  2i  3  i 
1  4i

Tìm phần thực và phần ảo của số phúc sau:


3  5i
z   5  2i  3  i 
1  4i
 3  5i 1  4i   15  2  5i  6i
  
1  16
 1  i   17  i 
 18

Kết luận phần thực bằng -18, phần ảo bằng 0

Câu 31. Cho số phức z 1 2i . Tìm phần thực và phần ảo của số phức w z2 2iz .

Ta có z  1  2i , khi đó w  (1  2i)2  2i(1  2i)  1  4i  4i 2  2i  4i 2


7 2i
Do đó, phần thực của số phức w là: -7 và phần ảo của số phức w là: -2.

Câu 32. Cho số phức z thỏa mãn 1  i  z   3  i  z  2  6i . Tìm phần thực, phần ảo của số
phức w  2 z  1 .

Giả sử z  a  bi  a, b    z  a  bi , khi đó:

1  i  z  3  i  z  2  6i  1  i  a  bi   3  i  a  bi   2  6i  4a  2b  2bi  2  6i
4a  2b  2 a  2
   z  2  3i
2b  6 b  3

Do đó w  2 z  1  2  2  3i   1  5  6i

Vậy số phức w có phần thực là 5, phần ảo là 6.

3. Giải phương trình nghiệm phức

Câu 33. Giải phương trình sau đây trên tập số phức: 2z 2 - 2z + 5 = 0

2z 2 2z 5 0 (*)
 Ta có, ( 2)2 4.2.5 36 (6i)2
 Vậy, phương trình (*) có 2 nghiệm phức phân biệt:
2 6i 1 3 2 6i 1 3
z1 i ; z2 i
4 2 2 4 2 2

NGUYỄN VĂN LỰC  0933.168.309 SP Toán K35 - ĐH Cần Thơ


Số phức FB: http://www.facebook.com/VanLuc168
Câu 34. Giải phương trình 3z  6 z  15  0 trên tập hợp số phức.
2

+ Tính đúng  '  36  0


3  6i 3  6i
+ Nêu được hai nghiệm z1   1  2i , z2   1  2i
3 3

Câu 35. Giải phương trình sau trên tập số phức z 2  z  1  0

Ta có:   1  4  3  3i 2 căn bậc hai của  là i 3


1 i 3 1 3 1 3
Phương trình có nghiệm: z1    i, z2   i
2 2 2 2 2

 zi
4

Câu 36. Giải phương trình nghiệm phức:    1, ( z  C )


 z i

Đk: khi đó, pt đã cho tương đương

(1) (t/m)
(2) (t/m)
Vậy pt có tập nghiệm z={-1;0;1}

Câu 37. Giải phương trình nghiệm phức: z  i  0,( z  C )


2

 2 2
1 1 1 z   i
i  .(2i)  (1  i)2 z2  (1  i)2   2 2
2 2 2   2 2
 z  2

2
i

Câu 38. Giải các phương trình sau trên tập số phức x2  2 x  5  0

  4  20  16  16i 2
Căn bậc hai của  là 4i .
Phương trình có nghiệm: x1  1  2i, x2  1  2i

Câu 39. Giải các phương trình sau trên tập số phức z 4  2 z 2  3  0

Đặt t = z2.
Phương trình trở thành:
t  1 z2  1  z  1
t  2t  3  0  
2
 2 
t  3  z  3  z  i 3
Vậy phương trình có 4 nghiệm: -1, 1, i 3, i 3

NGUYỄN VĂN LỰC  0933.168.309 SP Toán K35 - ĐH Cần Thơ


Số phức FB: http://www.facebook.com/VanLuc168

Câu 40. Giải phương trình sau đây trên tập số phức: z2 2z 5 0.

- Ta có, 22 4.( 1).( 5) 16 (4i )2


2 4i 2 4i
 Vậy, pt (*) có 2 nghiệm phức phân biệt z1 1 2i và z 2 1 2i
2 2

Câu 41. Giải phương trình trong tập số phức: z 2  3z  7  0

  19   19i  2

Phương trình đã cho có hai nghiệm phức:


3  19i 3 19 3  19i 3 19
z1    i; z2    i
2 2 2 2 2 2

Câu 42. Giải phương trình sau trên tập số phức: 3x 2 2 3x 2 0

Ta có: ( 2 3)2 4.3.2 12 24 12 (2 3i)2


Phương trình có 2 nghiệm phức
3 3 3 3
x1 i; x 2 i
3 3 3 3

Câu 43. Giải phương trình sau trên tập số phức: x2 – 6x + 29 = 0

  20
Phương trình có 2 nghiệm phức: x  3  2i 5

Câu 44. Giải phương trình sau trên tập số phức: x 2  4 x  11  0.

 '  4  11  7  ( 7i ) 2
x1,2  2  7i.

NGUYỄN VĂN LỰC  0933.168.309 SP Toán K35 - ĐH Cần Thơ


Hình học không gian FB: http://www.facebook.com/VanLuc168

ÔN TẬP: KIẾN THỨC CƠ BẢN HÌNH HỌC


LỚP 9 – 10 – 11 - 12
Chuyên đề: Hình học không gian

ÔN TẬP 1. KIẾN THỨC CƠ BẢN HÌNH HỌC LỚP 9-10

1. Hệ thức lượng trong tam giác vuông : Cho ABC vuông ở A ta có :


a) Định lý Pitago : BC 2  AB 2  AC 2 A
b) BA 2  BH .BC ; CA 2  CH .CB
c) AB. AC = BC. AH c b
1 1 1
d) 2
 2

AH AB AC 2 H M
B C
e) BC = 2AM a
b c b c
f) sin B  , cosB  , tan B  , cot B 
a a c b
b b
g) b = a. sinB = a.cosC, c = a. sinC = a.cosB, a =  ,
sin B cos C
b = c. tanB = c.cot C
2. Hệ thức lượng trong tam giác thường:
* Định lý Côsin: a2 = b2 + c2 - 2bc.cosA
a b c
* Định lý Sin:    2R
sin A sin B sin C
3. Các công thức tính diện tích.
a/ Công thức tính diện tích tam giác:
1 1 a.b.c abc
S a.ha = a.b sin C   p.r  p.( p  a )( p  b)( p  c ) với p 
2 2 4R 2
1 a2 3
Đặc biệt :* ABC vuông ở A : S  AB. AC ,* ABC đều cạnh a: S
2 4
b/ Diện tích hình vuông : S = cạnh x cạnh
c/ Diện tích hình chữ nhật : S = dài x rộng
1
d/ Diện tích hình thoi : S = (chéo dài x chéo ngắn)
2
1
d/ Diện tích hình thang : S  (đáy lớn + đáy nhỏ) x chiều cao
2
e/ Diện tích hình bình hành : S = đáy x chiều cao
f/ Diện tích hình tròn : S   .R
2

NGUYỄN VĂN LỰC  0933.168.309 SP Toán K35 - ĐH Cần Thơ


Hình học không gian FB: http://www.facebook.com/VanLuc168

4. Các hệ thức quan trọng trong tam giác đều:

ÔN TẬP 2. KIẾN THỨC CƠ BẢN HÌNH HỌC LỚP 11

A. QUAN HỆ SONG SONG

§1. ĐƯỜNG THẲNG VÀ MẶT PHẲNG SONG SONG

I. Định nghĩa:
Đường thẳng và mặt phẳng gọi a

là song song với nhau nếu a / /(P)  a  (P)  


chúng không có điểm nào
(P)
chung.

II.Các định lý:


ĐL1:Nếu đường thẳng d không d
nằm trên mp(P) và song song d  (P)
với đường thẳng a nằm trên 
d / / a  d / /(P)
a
mp(P) thì đường thẳng d song a  (P) (P)
song với mp(P) 
ĐL2: Nếu đường thẳng a song
a / /(P) (Q)
song với mp(P) thì mọi mp(Q) 
a

chứa a mà cắt mp(P) thì cắt a  (Q)  d / /a d


theo giao tuyến song song với (P)  (Q)  d

a. (P)

ĐL3: Nếu hai mặt phẳng cắt


nhau cùng song song với một (P)  (Q)  d d
đường thẳng thì giao tuyến của 
chúng song song với đường (P) / /a  d / /a a
(Q) / /a
thẳng đó.  P
Q

NGUYỄN VĂN LỰC  0933.168.309 SP Toán K35 - ĐH Cần Thơ


Hình học không gian FB: http://www.facebook.com/VanLuc168

§2.HAI MẶT PHẲNG SONG SONG

I. Định nghĩa:

Hai mặt phẳng được gọi là song


song với nhau nếu chúng không (P) / /(Q)  (P)  (Q)   P

có điểm nào chung.


Q

II.Các định lý:


ĐL1: Nếu mp(P) chứa hai a,b  (P)
đường thẳng a, b cắt nhau và  a

cùng song song với mặt phẳng a  b  I  (P) / /(Q) P b I


a / /(Q),b / /(Q)
(Q) thì (P) và (Q) song song  Q
với nhau.
ĐL2: Nếu một đường thẳng a
nằm một trong hai mặt phẳng (P) / /(Q) P
  a / /(Q)
song song thì song song với a  (P)

mặt phẳng kia. Q

ĐL3: Nếu hai mặt phẳng (P) và R


(Q) song song thì mọi mặt
(P) / /(Q)
phẳng (R) đã cắt (P) thì phải cắt  P a
(Q) và các giao tuyến của (R)  (P)  a  a / / b
chúng song song. (R)  (Q)  b Q b

B. QUAN HỆ VUÔNG GÓC

§1.ĐƯỜNG THẲNG VUÔNG GÓC VỚI MẶT PHẲNG

I.Định nghĩa:

Một đường thẳng được gọi là


a  mp(P)  a  c, c  (P)
a
vuông góc với một mặt phẳng
nếu nó vuông góc với mọi
đường thẳng nằm trên mặt P c
phẳng đó.

II. Các định lý:


ĐL1: Nếu đường thẳng d
d
vuông góc với hai đường thẳng d  a ,d  b
cắt nhau a và b cùng nằm trong 
mp(P) thì đường thẳng d vuông a ,b  mp(P)  d  mp(P)
góc với mp(P). a,b caét nhau b
 P a

NGUYỄN VĂN LỰC  0933.168.309 SP Toán K35 - ĐH Cần Thơ


Hình học không gian FB: http://www.facebook.com/VanLuc168

ĐL2: (Ba đường vuông góc)


Cho đường thẳng a không
vuông góc với mp(P) và đường a
thẳng b nằm trong (P). Khi đó, a  mp(P), b  mp(P)
điều kiện cần và đủ để b vuông b  a  b  a'
góc với a là b vuông góc với b
a'
hình chiếu a’ của a trên (P). P

§2.HAI MẶT PHẲNG VUÔNG GÓC

I.Định nghĩa:

Hai mặt phẳng được gọi là vuông góc với nhau nếu góc giữa chúng bằng 900.

II. Các định lý:

ĐL1:Nếu một mặt phẳng chứa Q


một đường thẳng vuông góc với a  mp(P) a
một mặt phẳng khác thì hai mặt a  mp(Q)  mp(Q)  mp(P)
phẳng đó vuông góc với nhau. 
P

ĐL2:Nếu hai mặt phẳng (P) và P


(Q) vuông góc với nhau thì bất (P)  (Q) a
cứ đường thẳng a nào nằm 
trong (P), vuông góc với giao (P)  (Q)  d  a  (Q)
tuyến của (P) và (Q) đều vuông a  (P),a  d
 d Q
góc với mặt phẳng (Q).
ĐL3: Nếu hai mặt phẳng (P) và P
(Q) vuông góc với nhau và A là (P)  (Q) a
một điểm trong (P) thì đường  A

thẳng a đi qua điểm A và vuông A  (P)


  a  (P)
góc với (Q) sẽ nằm trong (P)  A  a Q

a  (Q)
ĐL4: Nếu hai mặt phẳng cắt
nhau và cùng vuông góc với (P)  (Q)  a
P
a
Q

mặt phẳng thứ ba thì giao tuyến 


của chúng vuông góc với mặt (P)  (R)  a  (R)
phẳng thứ ba. (Q)  (R)

R

NGUYỄN VĂN LỰC  0933.168.309 SP Toán K35 - ĐH Cần Thơ


Hình học không gian FB: http://www.facebook.com/VanLuc168

§3.KHOẢNG CÁCH

1. Khoảng cách từ 1 điểm tới 1 đường O


thẳng , đến 1 mặt phẳng:
Khoảng cách từ điểm M đến đường
O
thẳng a (hoặc đến mặt phẳng (P)) là
khoảng cách giữa hai điểm M và H,
H
trong đó H là hình chiếu của điểm M trên a P
H

đường thẳng a ( hoặc trên mp(P))

d(O; a) = OH; d(O; (P)) = OH

2. Khoảng cách giữa đường thẳng và


mặt phẳng song song: a O
Khoảng cách giữa đường thẳng a và
mp(P) song song với a là khoảng cách từ
H
một điểm nào đó của a đến mp(P). P
d(a;(P)) = d(O; (P)) = OH
3. Khoảng cách giữa hai mặt phẳng O
song song: P
là khoảng cách từ một điểm bất kỳ trên
mặt phẳng này đến mặt phẳng kia. Q
H
d((P);(Q)) = d(O; (P)) = OH
4.Khoảng cách giữa hai đường thẳng a
A
chéo nhau:
là độ dài đoạn vuông góc chung của hai
đường thẳng đó. b
B

d(a;b) = AB

§4.GÓC

1. Góc giữa hai đường thẳng a và b


là góc giữa hai đường thẳng a’ và b’ cùng a a'

đi qua một điểm và lần lượt cùng phương


với a và b.
b'
b

2. Góc giữa đường thẳng a không vuông a


góc với mặt phẳng (P)
là góc giữa a và hình chiếu a’ của nó trên
mp(P).
Đặc biệt: Nếu a vuông góc với mặt phẳng P
a'
(P) thì ta nói rằng góc giữa đường thẳng a
và mp(P) là 900.

NGUYỄN VĂN LỰC  0933.168.309 SP Toán K35 - ĐH Cần Thơ


Hình học không gian FB: http://www.facebook.com/VanLuc168

3. Góc giữa hai mặt phẳng


là góc giữa hai đường thẳng lần lượt
vuông góc với hai mặt phẳng đó.
Hoặc là góc giữa 2 đường thẳng nằm a b a b

trong 2 mặt phẳng cùng vuông góc với Q


P
giao tuyến tại 1 điểm P Q

4. Diện tích hình chiếu: Gọi S là diện S

tích của đa giác (H) trong mp(P) và S’ là


diện tích hình chiếu (H’) của (H) trên
mp(P’) thì S'  Scos 
trong đó  là góc giữa hai mặt phẳng A 
C

(P),(P’). B

ÔN TẬP 3. KIẾN THỨC CƠ BẢN HÌNH HỌC LỚP 12

A. THỂ TÍCH KHỐI ĐA DIỆN


I/ Các công thức thể tích của khối đa diện:

1. THỂ TÍCH KHỐI LĂNG TRỤ:


V= B.h
h

với B: diện tích đáy B


h: chiều cao
a) Thể tích khối hộp chữ nhật:
V = a.b.c
với a,b,c là ba kích thước
a

b) Thể tích khối lập phương: a


c
b

V = a3 a

với a là độ dài cạnh


a

2. THỂ TÍCH KHỐI CHÓP:


1
V= Bh h
3
với B: diện tích đáy
B
h: chiều cao
3. TỈ SỐ THỂ TÍCH TỨ DIỆN: S
Cho khối tứ diện SABC và A’, B’, C’ là
các điểm tùy ý lần lượt thuộc SA, SB,
C'
A'

SC ta có:
VSABC SA SB SC
A B'


C

VSA ' B' C ' SA ' SB' SC' B

NGUYỄN VĂN LỰC  0933.168.309 SP Toán K35 - ĐH Cần Thơ


Hình học không gian FB: http://www.facebook.com/VanLuc168

4. THỂ TÍCH KHỐI CHÓP CỤT: A'


B'


h
V  B  B' BB'
3
 A
C'

B, B' : dieän tích hai ñaùy


với 
 h : chieàu cao
C

Chú ý:
1/ Đường chéo của hình vuông cạnh a là d = a 2 ,
Đường chéo của hình lập phương cạnh a là d = a 3 ,
Đường chéo của hình hộp chữ nhật có 3 kích thước a, b, c là d = a 2  b2  c 2 ,
a 3
2/ Đường cao của tam giác đều cạnh a là h =
2
3/ Hình chóp đều là hình chóp có đáy là đa giác đều và các cạnh bên đều bằng
nhau ( hoặc có đáy là đa giác đều, hình chiếu của đỉnh trùng với tâm của đáy).
4/ Lăng trụ đều là lăng trụ đứng có đáy là đa giác đều.

NGUYỄN VĂN LỰC  0933.168.309 SP Toán K35 - ĐH Cần Thơ


Hình học không gian FB: http://www.facebook.com/VanLuc168

I. THỂ TÍCH LĂNG TRỤ

Chuyên đề: Hình học không gian

1) Dạng 1: Khối lăng trụ đứng có chiều cao hay cạnh đáy

Ví dụ 1: Đáy của lăng trụ đứng tam giác ABC.A’B’C’ là tam giác ABC vuông cân
tại A có cạnh BC = a 2 và biết A'B = 3a. Tính thể tích khối lăng trụ.

A' C' Lời giải:


Ta có
B'
ABC vuông cân tại A nên AB = AC = a
3a ABC A'B'C' là lăng trụ đứng  AA'  AB
AA'B  AA'2  A'B2  AB2  8a2
A C  AA'  2a 2
a 2 Vậy V = B.h = SABC .AA' = a3 2
a
B

Ví dụ 2: Cho lăng trụ tứ giác đều ABCD.A’B’C’D' có cạnh bên bằng 4a và đường
chéo 5a. Tính thể tích khối lăng trụ này.

C' Lời giải:


D'
ABCD A'B'C'D' là lăng trụ đứng nên
A' BD2 = BD'2 - DD'2 = 9a2  BD  3a
B' 3a
ABCD là hình vuông  AB 
2
4a
5a
D C 9a 2
Suy ra B = SABCD =
4
A B Vậy V = B.h = SABCD.AA' = 9a3

Ví dụ 3: Đáy của lăng trụ đứng tam giác ABC.A’B’C’ là tam giác đều cạnh a = 4 và
biết diện tích tam giác A’BC bằng 8. Tính thể tích khối lăng trụ.

NGUYỄN VĂN LỰC  0933.168.309 SP Toán K35 - ĐH Cần Thơ


Hình học không gian FB: http://www.facebook.com/VanLuc168

A' C' Lời giải:


Gọi I là trung điểm BC .Ta có
B' ABC đều nên
AB 3
AI   2 3 & AI  BC
2
 A 'I  BC(dl3 )
1 2S
A C SA'BC  BC.A 'I  A 'I  A'BC  4
2 BC
I
B AA '  (ABC)  AA '  AI .
A 'AI  AA '  A 'I2  AI2  2
Vậy : VABC.A’B’C’ = SABC .AA'= 8 3

Ví dụ 5: Cho hình hộp đứng có đáy là hình thoi cạnh a và có góc nhọn bằng 600
Đường chéo lớn của đáy bằng đường chéo nhỏ của lăng trụ. Tính thể tích hình hộp .

Lời giải:
D' C' Ta có tam giác ABD đều nên : BD = a
a2 3
và SABCD = 2SABD =
B' 2
A' a 3
Theo đề bài BD' = AC = 2 a 3
D C 2
DD'B  DD'  BD'2  BD2  a 2
a3 6
A 60
B Vậy V = SABCD.DD' =
2

2) Dạng 2: Lăng trụ đứng có góc giữa đường thẳng và mặt phẳng.

Ví dụ 1: Cho lăng trụ đứng tam giác ABC A'B'C' có đáy ABC là tam giác vuông cân
tại B với BA = BC = a ,biết A'B hợp với đáy ABC một góc 60 0. Tính thể tích lăng
trụ.

C'
A' Lời giải:
Ta có A 'A  (ABC)  A 'A  AB& AB là hình
chiếu của A'B trên đáy ABC .
B'
Vậy góc[A'B,(ABC)]  ABA'  60o
ABA'  AA'  AB.tan 600  a 3
1 a2
A C
SABC = BA.BC 
2 2
60o a3 3
Vậy V = SABC.AA' =
2
B

NGUYỄN VĂN LỰC  0933.168.309 SP Toán K35 - ĐH Cần Thơ


Hình học không gian FB: http://www.facebook.com/VanLuc168

Ví dụ 2: Cho lăng trụ đứng tam giác ABC A'B'C' có đáy ABC là tam giác vuông tại
A với AC = a , ACB = 60 o biết BC' hợp với (AA'C'C) một góc 300. Tính AC' và thể
tích lăng trụ.

Lời giải:
A' C'
ABC  AB  AC.tan60o  a 3 .
Ta có:
AB  AC;AB  AA'  AB  (AA'C'C)
nên AC' là hình chiếu của BC' trên (AA'C'C).
B' o
30 Vậy góc[BC';(AA"C"C)] = BC'A = 30o
AB
AC'B  AC'   3a
t an30o
A a C V =B.h = SABC.AA'
o AA'C'  AA'  AC'2  A'C'2  2a 2
60
a2 3
B ABC là nửa tam giác đều nên SABC 
2
Vậy V = a3 6

Ví dụ 3: Cho lăng trụ đứng ABCD A'B'C'D' có đáy ABCD là hình vuông cạnh a và
đường chéo BD' của lăng trụ hợp với đáy ABCD một góc 300. Tính thể tích và tổng
diên tích của các mặt bên của lăng trụ .

C' B' Lời giải:


D' A' Ta có ABCD A'B'C'D' là lăng trụ đứng nên ta có:
DD'  (ABCD)  DD'  BD và BD là hình chiếu của
BD' trên ABCD.
o Vậy góc [BD';(ABCD)] = DBD'  300
C 30 B
a 6
D
A
BDD'  DD'  BD.tan 300 
3
3
a a 6 4a 2 6
Vậy V = SABCD.DD' = S = 4SADD'A' =
3 3

Ví dụ 4: Cho hình hộp đứng ABCD A'B'C'D' có đáy ABCD là hình thoi cạnh a và
BAD = 60o biết AB' hợp với đáy (ABCD) một góc 30 o. Tính thể tích của hình hộp.

B' C' Lời giải:


a2 3
ABD đều cạnh a  SABD 
A'
D'
4
a2 3
 SABCD  2SABD 
B C 2
o
30
ABB' vuông tạiB  BB'  ABt an30o  a 3
o
3a3
V B.h S .BB'
60
A D
Vậy   ABCD 
a
2

NGUYỄN VĂN LỰC  0933.168.309 SP Toán K35 - ĐH Cần Thơ


Hình học không gian FB: http://www.facebook.com/VanLuc168

3) Dạng 3: Lăng trụ đứng có góc giữa 2 mặt phẳng

Ví dụ 1: Cho lăng trụ đứng tam giác ABC A'B'C' có đáy ABC là tam giác vuông cân
tại B với BA = BC = a , biết (A'BC) hợp với đáy (ABC) một góc 600 .Tính thể tích
lăng trụ.

A' C' Lời giải:


Ta có A 'A  (ABC)& BC  AB  BC  A 'B
B' Vậy góc[(A'BC),(ABC)]  ABA'  60o
ABA'  AA'  AB.tan 600  a 3
1 a2
A C
SABC = BA.BC 
o 2 2
60
a3 3
B
Vậy V = SABC.AA' =
2

Ví dụ 2: Đáy của lăng trụ đứng tam giác ABC.A’B’C’ là tam giác đều . Mặt (A’BC)
tạo với đáy một góc 300 và diện tích tam giác A’BC bằng 8. Tính thể tích khối lăng
trụ.

Lời giải:
A' C'
ABC đều  AI  BC mà AA'  (ABC) nên
A'I  BC (đl 3  ).
Vậy góc[(A'BC);)ABC)] = A'IA = 30o
B'
2x 3
Giả sử BI = x  AI   x 3 .Ta có
2
2 AI 2 x 3
A' AI : A' I  AI : cos 30 0    2x
3 3
A 30o C 3
A’A = AI.tan 300 = x 3. x
3
xI Vậy VABC.A’B’C’ = CI.AI.A’A = x3 3
B
Mà SA’BC = BI.A’I = x.2x = 8  x  2
Do đó VABC.A’B’C’ = 8 3

Ví dụ 3: Cho lăng trụ tứ giác đều ABCD A'B'C'D' có cạnh đáy a và mặt phẳng
(BDC') hợp với đáy (ABCD) một góc 60 o.Tính thể tích khối hộp chữ nhật.

NGUYỄN VĂN LỰC  0933.168.309 SP Toán K35 - ĐH Cần Thơ


Hình học không gian FB: http://www.facebook.com/VanLuc168

C' D' Lời giải:


Gọi O là tâm của ABCD . Ta có
B'
A' ABCD là hình vuông nên OC  BD
CC'  (ABCD) nên OC'  BD (đl 3  ). Vậy
góc[(BDC');(ABCD)] = COC' = 60o
Ta có V = B.h = SABCD.CC'
C 60 0
D ABCD là hình vuông nên SABCD = a2
a 6
O OCC' vuông nên CC' = OC.tan60o =
A 2
a
a 6
3
B
Vậy V =
2

Ví dụ 4: Cho hình hộp chữ nhật ABCD A'B'C'D' có AA' = 2a ; mặt phẳng (A'BC)
hợp với đáy (ABCD) một góc 60 o và A'C hợp với đáy (ABCD) một góc 30 o. Tính thể
tích khối hộp chữ nhật.

Lời giải:
A' D' Ta có AA'  (ABCD)  AC là hình chiếu
của A'C trên (ABCD)
B' C'
Vậy góc[A'C,(ABCD)] = A 'CA  30o
2a BC  AB  BC  A'B (đl 3  ) .
Vậy góc[(A'BC),(ABCD)] = A 'BA  60o
o
A D A'AC  AC = AA'.cot30o = 2a 3
60 o
C 2a 3
B
30
A'AB  AB = AA'.cot60o =
3
4a 6
ABC  BC  AC2  AB2 
3
3
16a 2
Vậy V = AB.BC.AA' =
3

4) Dạng 4: Khối lăng trụ xiên

Ví dụ 1: Cho lăng trụ xiên tam giác ABC A'B'C' có đáy ABC là tam giác đều cạnh a ,
biết cạnh bên là a 3 và hợp với đáy ABC một góc 60 o. Tính thể tích lăng trụ.

Lời giải:
Ta có C'H  (ABC)  CH là hình chiếu của CC'
A'
C'
B'
trên (ABC)
Vậy góc[CC',(ABC)]  C'CH  60o
3a
A C 60
o CHC'  C'H  CC'.sin 600 
2
2
a 3 3a 3 3
a B H
SABC =  .Vậy V = SABC.C'H =
4 8

NGUYỄN VĂN LỰC  0933.168.309 SP Toán K35 - ĐH Cần Thơ


Hình học không gian FB: http://www.facebook.com/VanLuc168

Ví dụ 2: Cho lăng trụ xiên tam giác ABC A'B'C' có đáy ABC là tam giác đều cạnh a .
Hình chiếu của A' xuống (ABC) là tâm O đường tròn ngoại tiếp tam giác ABC biết
AA' hợp với đáy ABC một góc 60 .
1) Chứng minh rằng BB'C'C là hình chữ nhật.
2) Tính thể tích lăng trụ .

A' Lời giải:


C'
1) Ta có A 'O  (ABC)  OA là hình chiếu của
AA' trên (ABC)
Vậy góc[AA',(ABC)]  OAA'  60o
B'
Ta có BB'CC' là hình bình hành ( vì mặt bên
của lăng trụ)
60 o AO  BC tại trung điểm H của BC nên
A
C
BC  A'H (đl 3  )
O  BC  (AA 'H)  BC  AA ' mà AA'//BB' nên
a H
BC  BB' Vậy BB'CC' là hình chữ nhật.
B 2 2a 3 a 3
2) ABC đều nên AO  AH  
3 3 2 3
o
AOA '  A 'O  AO t an60  a
a3 3
Vậy V = SABC.A'O =
4

Ví dụ 3: Cho hình hộp ABCD.A’B’C’D’ có đáy là hình chữ nhật với AB= 3 ,
AD= 7 .Hai mặt bên (ABB’A’) và (ADD’A’) lần lượt tạo với đáy những góc 450 và
600. Tính thể tích khối hộp nếu biết cạnh bên bằng 1.

D'
Lời giải:
C' Kẻ A’H  ( ABCD ) ,HM  AB , HN  AD
 A' M  AB , A' N  AD (đl 3  )
A'  A'MH  45o ,A'NH  60o
B'
Đặt A’H = x . Khi đó
2x
A’N = x : sin 600 =
3
D
3  4x 2
C AN = AA'  A' N 
2 2
 HM
N 3
H
Mà HM = x.cot 450 = x
A
M 3  4x 2 3
B Nghĩa là x = x
3 7
3
Vậy VABCD.A’B’C’D’ = AB.AD.x = 3. 7. 3
7
NGUYỄN VĂN LỰC  0933.168.309 SP Toán K35 - ĐH Cần Thơ
Hình học không gian FB: http://www.facebook.com/VanLuc168

II. THỂ TÍCH KHỐI CHÓP

Chuyên đề: Hình học không gian

1) Dạng 1: Khối chóp có cạnh bên vuông góc với đáy

Ví dụ 1: Cho hình chóp SABC có SB = SC = BC = CA = a . Hai mặt (ABC) và


(ASC) cùng vuông góc với (SBC). Tính thể tích hình chóp .

A Lời giải:
Ta có
(ABC)  (SBC)

a_

  AC  (SBC)
C B 
 (ASC)  (SBC)
/
1 1 a2 3 a3 3
/ \
Do đó V  SSBC.AC  a
S 3 3 4 12

Ví dụ 2: Cho hình chóp SABC có đáy ABC là tam giác vuông cân tại B với AC = a
biết SA vuông góc với đáy ABC và SB hợp với đáy một góc 60 o.
1) Chứng minh các mặt bên là tam giác vuông .
2) Tính thể tích hình chóp.

Lời giải:
S
1) SA  (ABC)  SA  AB &SA  AC
mà BC  AB  BC  SB ( đl 3  ).
Vậy các mặt bên chóp là tam giác vuông.
2) Ta có SA  (ABC)  AB là hình chiếu của
SB trên (ABC).
C
A a Vậy góc[SB,(ABC)] = SAB  60o .
a
ABC vuông cân nên BA = BC =
60o 2
1 a 2
SABC = BA.BC 
B 2 4
a 6
SAB  SA  AB.t an60o 
2
1 1 a a 6 a3 6
2
Vậy V  S .SA  
3 ABC 34 2 24

Ví dụ 3: Cho hình chóp SABC có đáy ABC là tam giác đều cạnh a biết SA vuông
góc với đáy ABC và (SBC) hợp với đáy (ABC) một góc 60 o. Tính thể tích hình chóp
NGUYỄN VĂN LỰC  0933.168.309 SP Toán K35 - ĐH Cần Thơ
Hình học không gian FB: http://www.facebook.com/VanLuc168

S Lời giải: M là trung điểm của BC,vì tam giác


ABC đều nên
AM  BC  SA  BC (đl3  ) .
Vậy góc[(SBC);(ABC)] = SMA  60o .
1 1
Ta có V = B.h  SABC .SA
A C

60 o 3 3
3a
a M
SAM  SA  AM tan 60o 
B
2
1 1 a3 3
Vậy V = B.h  SABC .SA 
3 3 8

Ví dụ 4: Cho hình chóp SABCD có đáy ABCD là hình vuông có cạnh a và SA


vuông góc đáy ABCD và mặt bên (SCD) hợp với đáy một góc 60 o.
1) Tính thể tích hình chóp SABCD.
2) Tính khoảng cách từ A đến mặt phẳng (SCD).

Lời giải:
S
1) Ta có SA  (ABC) và CD  AD  CD  SD
H
( đl 3  ).(1)
Vậy góc[(SCD),(ABCD)] = SDA = 60o .
SAD vuông nên SA = AD.tan60o = a 3
1 1 a3 3
60
o Vậy V  SABCD .SA  a2a 3 
A D 3 3 3
2) Ta dựng AH  SD ,vì CD  (SAD) (do (1) )
nên CD  AH  AH  (SCD)
Vậy AH là khoảng cách từ A đến (SCD).
B a
C 1 1 1 1 1 4
SAD      
AH2 SA2 AD2 3a2 a2 3a2
a 3
Vậy AH =
2

2) Dạng 2 : Khối chóp có một mặt bên vuông góc với đáy

Ví dụ 1: Cho hình chóp S.ABCD có đáy ABCD là hình vuông có cạnh a. Mặt bên
SAB là tam giác đều nằm trong mặt phẳng vuông góc với đáyABCD.
1) Chứng minh rằng chân đường cao khối chóp trùng với trung điểm cạnh AB.
2) Tính thể tích khối chóp SABCD.

Lời giải:
1) Gọi H là trung điểm của AB.
SAB đều  SH  AB

NGUYỄN VĂN LỰC  0933.168.309 SP Toán K35 - ĐH Cần Thơ


Hình học không gian FB: http://www.facebook.com/VanLuc168

S mà (SAB)  (ABCD)  SH  (ABCD)


Vậy H là chân đường cao của khối chóp.
a 3
2) Ta có tam giác SAB đều nên SA =
A
D 2
1 a 3
3
suy ra V  SABCD .SH 
B H 3 6
a C

Ví dụ 2: Cho tứ diện ABCD có ABC là tam giác đều ,BCD là tam giác vuông cân tại
D, (ABC)  (BCD) và AD hợp với (BCD) một góc 60 o. Tính thể tích tứ diện ABCD.

A Lời giải:
Gọi H là trung điểm của BC.
Ta có tam giác ABC đều nên AH  (BCD) , mà
(ABC)  (BCD)  AH  (BCD) .
a
Ta có AH  HD  AH = AD.tan60o = a 3
a 3
B & HD = AD.cot60o =
60 o D
3
H
2a 3
BCD  BC = 2HD = suy ra
C 3
V = 1 SBCD .AH  1 . 1 BC.HD.AH  a 3
3

3 3 2 9

Ví dụ 3: Cho hình chóp S.ABC có đáy ABC là tam giác vuông cân tại B, có BC = a.
Mặt bên SAC vuông góc với đáy, các mặt bên còn lại đều tạo với mặt đáy một góc
450.
a) Chứng minh rằng chân đường cao khối chóp trùng với trung điểm cạnh AC.
b) Tính thể tích khối chóp SABC.

S Lời giải:
a) Kẻ SH  BC vì mp(SAC)  mp(ABC) nên
SH  mp(ABC).
Gọi I, J là hình chiếu của H trên AB và BC 
SI  AB, SJ  BC, theo giả thiết SIH  SJH  45o
H Ta có: SHI  SHJ  HI  HJ nên BH là đường
phân giác của ABC ừ đó suy ra H là trung điểm
A
45 C

I
J
của AC.
a 1 a3
B b) HI = HJ = SH =  VSABC= S ABC .SH 
2 3 12

NGUYỄN VĂN LỰC  0933.168.309 SP Toán K35 - ĐH Cần Thơ


Hình học không gian FB: http://www.facebook.com/VanLuc168

3) Dạng 3 : Khối chóp đều

Ví dụ 1: Cho chóp tam giác đều SABC cạnh đáy bằng a và cạnh bên bằng 2a.
Chứng minh rằng chân đường cao kẻ từ S của hình chóp là tâm của tam giác đều
ABC.Tính thể tích chóp đều SABC .

Lời giải:
S Dựng SO  (ABC) Ta có SA = SB = SC suy ra OA
= OB = OC
2a Vậy O là tâm của tam giác đều ABC.
Ta có tam giác ABC đều nên
C 2 2a 3 a 3
A
AO = AH  
3 3 2 3
O 11a2
SAO  SO2  SA 2  OA 2 
a H

B
3
a 11 1 a3 11
 SO  .Vậy V  SABC .SO 
3 3 12

Ví dụ 2:Cho khối chóp tứ giác SABCD có tất cả các cạnh có độ dài bằng a .
1) Chứng minh rằng SABCD là chóp tứ giác đều.
2) Tính thể tích khối chóp SABCD.

Lời giải:
S Dựng SO  (ABCD)
Ta có SA = SB = SC = SD nên
OA = OB = OC = OD  ABCD là hình thoi có
đường tròn ngoại tiếp nên ABCD là hình
vuông .
C
D Ta có SA2 + SB2 = AB2 +BC2 = AC2 nên
a 2
ASC vuông tại S  OS 
O 2
A 1 1 2 a 2 a3 2
a B  V  S ABCD .SO  a 
3 3 2 6
a3 2
Vậy V 
6

Ví dụ 3: Cho khối tứ diện đều ABCD cạnh bằng a, M là trung điểm DC.
a) Tính thể tích khối tứ diện đều ABCD.
b) Tính khoảng cách từ M đến mp(ABC). Suy ra thể tích hình chóp MABC.

Lời giải:
a) Gọi O là tâm của ABC  DO  ( ABC )
1
V  S ABC .DO
3
NGUYỄN VĂN LỰC  0933.168.309 SP Toán K35 - ĐH Cần Thơ
Hình học không gian FB: http://www.facebook.com/VanLuc168

D a2 3 2 a 3
S ABC  , OC  CI 
4 3 3
M a 6
DOC vuông có : DO  DC 2  OC 2 
3
1 a 2 3 a 6 a3 2
A V  . 
C 3 4 3 12
O
H b) Kẻ MH// DO, khoảng cách từ M đến
mp(ABC) là MH
I a 1 a 6
MH  DO 
B 2 6
1 1 a 2 3 a 6 a3 2
 VMABC  S ABC .MH  . 
3 3 4 6 24
a3 2
Vậy V 
24

4) Dạng 4 : Khối chóp & phương pháp tỷ số thể tích

Ví dụ 1: Cho hình chóp S.ABC có tam giác ABC vuông cân ở B, AC  a 2 , SA


vuông góc với đáy ABC , SA  a
1) Tính thể tích của khối chóp S.ABC.
2) Gọi G là trọng tâm tam giác ABC, mặt phẳng (  ) qua AG và song song
với BC cắt SC, SB lần lượt tại M, N. Tính thể tích của khối chóp S.AMN

Lời giải:
1
a)Ta có: VS . ABC  S ABC .SA và SA  a
S
3
+ ABC cân có : AC  a 2  AB  a
1 1 1 a3
N  S ABC  a 2 Vậy: VSABC  . a 2 .a 
2 3 2 6
A G C b) Gọi I là trung điểm BC.
SG 2
M G là trọng tâm,ta có : 
I SI 3
SM SN SG 2
B  // BC  MN// BC    
SB SC SI 3
VSAMN SM SN 4
  . 
VSABC SB SC 9
4 2a 3
Vậy: VSAMN  VSABC 
9 27

NGUYỄN VĂN LỰC  0933.168.309 SP Toán K35 - ĐH Cần Thơ


Hình học không gian FB: http://www.facebook.com/VanLuc168

Ví dụ 2: Cho tam giác ABC vuông cân ở A và AB  a . Trên đường thẳng qua C và
vuông góc với mặt phẳng (ABC) lấy điểm D sao cho CD  a . Mặt phẳng qua C
vuông góc với BD, cắt BD tại F và cắt AD tại E.
a) Tính thể tích khối tứ diện ABCD.
b) Chứng minh CE  ( ABD )
c) Tính thể tích khối tứ diện CDEF

Lời giải:
1 a 3
D a) Tính V ABCD : VABCD  SABC .CD 
F 3 6
b) Tacó:
a AB  AC , AB  CD  AB  ( ACD )  AB  EC
E DB  EC  EC  ( ABD )
V DE DF
B c) Tính VDCEF :Ta có: DCEF  . (*)
C VDABC DA DB
Mà DE.DA  DC , chia cho DA2
2

a DE DC 2 a2 1
   
A DA DA 2
2a 2
2
2
DF DC a2 1
Tương tự:   
DB DB 2
DC  CB
2 2
3
VDCEF 1 1 a3
Từ (*)   .Vậy VDCEF  VABCD 
VDABC 6 6 36

Ví dụ 3: Cho khối chóp tứ giác đều SABCD. Một mặt phẳng ( ) qua A, B và trung
điểm M của SC . Tính tỉ số thể tích của hai phần khối chóp bị phân chia bởi mặt
phẳng đó.

S
Lời giải:
Kẻ MN // CD (N  SD) thì hình thang ABMN là
thiết diện của khối chóp khi cắt bởi mặt phẳng
N
(ABM).
VSAND SN 1 1 1
+    VSANB  VSADB  VSABCD
VSADB SD 2 2 4
M D
A VSBMN SM SN 1 1 1 1 1
 .  .   VSBMN  VSBCD  VSABCD
VSBCD SC SD 2 2 4 4 8
O
3
Mà VSABMN = VSANB + VSBMN = VSABCD .
8
5
C B Suy ra VABMN.ABCD = VSABCD
8
VSABMN 3
Do đó : 
V ABMN . ABCD 5

NGUYỄN VĂN LỰC  0933.168.309 SP Toán K35 - ĐH Cần Thơ


Hình học không gian FB: http://www.facebook.com/VanLuc168

Ví dụ 4: Cho hình chóp tứ giác đều S.ABCD, đáy là hình vuông cạnh a, cạnh bên tạo
với đáy góc 60 . Gọi M là trung điểm SC. Mặt phẳng đi qua AM và song song với
BD, cắt SB tại E và cắt SD tại F.
a) Hãy xác định mp(AEMF)
b) Tính thể tích khối chóp S.ABCD
c) Tính thể tích khối chóp S.AEMF

Lời giải:
S a) Gọi I  SO  AM . Ta có (AEMF)//BD  EF // BD
1
b) VS . ABCD  S ABCD .SO với S ABCD  a
2
3
M  a 6
E + SOA có : SO  AO.tan 60 
2
a3 6

I
B C Vậy : VS . ABCD
F 6
c) Phân chia chóp tứ giác ta có
O
VS . AEMF = VSAMF + VSAME =2VSAMF
A
D
VS . ABCD = 2VSACD = 2 VSABC
Xét khối chóp S.AMF và S.ACD
SM 1
Ta có :  
SC 2
SAC có trọng tâm I, EF // BD nên:
SI SF 2 V SM SF 1
    SAMF  . 
SO SD 3 VSACD SC SD 3
1 1 a3 6
 VSAMF  VSACD  VSACD 
3 6 36
a3 6 a3 6
 VS . AEMF 2 
36 18

Ví dụ 5: Cho hình chóp S.ABCD có đáy ABCD là hình vuông cạnh a, SA vuông góc
đáy, SA  a 2 . Gọi B’, D’ là hình chiếu của A lần lượt lên SB, SD. Mặt phẳng
(AB’D’) cắt SC tại C’.
a) Tính thể tích khối chóp S.ABCD.
b) Chứng minh SC  ( AB ' D ')
c) Tính thể tích khối chóp S.AB’C’D’

NGUYỄN VĂN LỰC  0933.168.309 SP Toán K35 - ĐH Cần Thơ


Hình học không gian FB: http://www.facebook.com/VanLuc168

Lời giải:
1 a3 2
S
a) Ta có: VS . ABCD  S ABCD .SA 
3 3
b) Ta có BC  ( SAB)  BC  AB '
& SB  AB ' Suy ra: AB '  ( SBC )
nên AB'  SC .Tương tự AD'  SC.
Vậy SC  (AB'D')
C' B'
c) Tính VS . AB 'C ' D '
D' I VSAB 'C ' SB ' SC '
+ Tính VS . AB ' C ' : Ta có:  . (*)
A B VSABC SB SC
SC ' 1
O SAC vuông cân nên 
SC 2
2 2
D C SB ' SA 2a 2a 2 2
Ta có:    
SB SB 2 SA2  AB 2 3a 2 3
VSAB 'C ' 1
Từ (*)  
VSABC 3
1 a3 2 a3 2
 VSAB 'C '  . 
3 3 9
2a 3 2
+ VS . AB 'C ' D '  2VS . AB 'C ' 
9

5) Dạng 5 : Ôn tập khối chóp và lăng trụ

Ví dụ 1: Cho hình chóp S.ABCD có ABCD là hình vuông cạnh 2a, SA vuông góc

đáy. Góc giữa SC và đáy bằng 60 và M là trung điểm của SB.
1) Tính thể tích của khối chóp S.ABCD.
2) Tính thể tích của khối chóp MBCD.

Lời giải:
. 1
a)Ta có V  S ABCD .SA
3
+ S ABCD  (2a)2  4a 2
+ SAC có : SA  AC tan C  2a 6
1 2 8a3 6
 V  4a .2a 6 
3 3
b) Kẻ MH / / SA  MH  ( DBC )
1 1
Ta có: MH  SA , S BCD  S ABCD
2 2

NGUYỄN VĂN LỰC  0933.168.309 SP Toán K35 - ĐH Cần Thơ


Hình học không gian FB: http://www.facebook.com/VanLuc168

S 1 2a 3 6
 VMBCD  V 
4 3

H
A B

60o

D
C
2a

Ví dụ 2: Cho hình hộp chữ nhật ABCD.A’B’C’D’ có AB  a 3 , AD = a, AA’ = a,


O là giao điểm của AC và BD.
a) Tính thể tích khối hộp chữ nhật, khối chóp OA’B’C’D’
b) Tính thể tích khối OBB’C’.
c) Tính độ dài đường cao đỉnh C’ của tứ diện OBB’C’.

Lời giải:
A B a) Gọi thể tích khối hộp chữ nhật là V.
Ta có : V  AB. AD.AA '  a 3.a  a 3
2 3
O
M
D
C ABD có : DB  AB2  AD2  2a
* Khối OA’B’C’D’ có đáy và đường cao
1 a3 3
A' B'
giống khối hộp nên:  VOA' B 'C ' D '  V 
3 3
D'
C'
b) M là trung điểm BC  OM  ( BB ' C ')
1 1 a 2 a 3 a3 3
 VO BB 'C '  S BB 'C ' .OM  . . 
3 3 2 2 12
c) Gọi C’H là đường cao đỉnh C’ của tứ
3VOBB 'C '
diện OBB’C’. Ta có : C ' H 
SOBB '
ABD có : DB  AB2  AD2  2a
1 2
 SOBB '  a  C ' H  2a 3
2

Ví dụ 3: Cho hình lập phương ABCD.A’B’C’D’có cạnh bằng a. Tính thể tích khối
tứ diện ACB’D’.

Lời giải:
Hình lập phương được chia thành: khối ACB’D’
và bốn khối CB’D’C’, BB’AC, D’ACD,
NGUYỄN VĂN LỰC  0933.168.309 SP Toán K35 - ĐH Cần Thơ
Hình học không gian FB: http://www.facebook.com/VanLuc168

A B AB’A’D’.
+ Các khối CB’D’C’, BB’AC, D’ACD,
D C AB’A’D’ có diện tích đáy và chiều cao bằng
nhau nên có cùng thể tích.
1 1 2 1 3
Khối CB’D’C’ có V1  . a .a  a
A' B'

3 2 6
+ Khối lập phương có thể tích: V2  a
C' 3
D'
a
1 3 1 3
 VACB ' D '  a  4. a  a
3

6 3

Ví dụ 4: Cho hình lăng trụ đứng tam giác có các cạnh bằng a.
a) Tính thể tích khối tứ diện A’B’ BC.
b) E là trung điểm cạnh AC, mp(A’B’E) cắt BC tại F. Tính thể tích khối
CA’B’FE.

Lời giải:
E B
a) Khối A’B’ BC:Gọi I là trung điểm AB,
A
1 2 3
I F VA ' B ' BC  S A ' B ' B .CI  1 a . a 3  a 3
C 3 3 2 2 12
b) Khối CA’B’FE: phân ra hai khối CEFA’ và
CFA’B’.
+ Khối A’CEFcó đáy là CEF, đường cao A’A
1
A'
B'
nên VA 'CEF  SCEF . A ' A
3
J
1 a2 3 a3 3
C'
SCEF  S ABC   VA 'CEF 
4 16 48
+ Gọi J là trung điểm B’C’. Ta có khối
A’B’CF có đáy là CFB’, đường cao JA’ nên
1
VA ' B 'CF  SCFB' . A ' J
3
1 a2
SCFB'  SCBB ' 
2 4
1 a 2 a 3 a3 3
 VA ' B 'CF  
3 4 2 24
a3 3
+ Vậy : VCA'B'FE 
16

NGUYỄN VĂN LỰC  0933.168.309 SP Toán K35 - ĐH Cần Thơ


Hình học không gian FB: http://www.facebook.com/VanLuc168

BÀI TẬP TỰ LUYỆN

1. Hình chóp tam giác


Câu 1. Cho hình chóp S . ABC có tam giác ABC vuông tại A , AB  AC  a , I là trung
điểm của SC , hình chiếu vuông góc của S lên mặt phẳng  ABC  là trung điểm H của
BC , mặt phẳng  SAB  tạo với đáy 1 góc bằng 60 . Tính thể tích khối chóp S . ABC và
tính khoảng cách từ điểm I đến mặt phẳng  SAB  theo a .

Sj

Gọi K là trung điểm của AB  HK  AB (1)


Vì SH   ABC  nên SH  AB (2)
Từ (1) và (2) suy ra  AB  SK
M
B Do đó góc giữa  SAB  với đáy bằng góc giữa SK
C H
và HK và bằng SKH  60
K
a 3
Ta có SH  HK tan SKH 
A 2

1 1 1 a3 3
Vậy VS . ABC  S ABC .SH  . AB. AC.SH 
3 3 2 12
Vì IH / / SB nên IH / /  SAB  . Do đó d  I ,  SAB    d  H ,  SAB  

Từ H kẻ HM  SK tại M  HM   SAB   d  H ,  SAB    HM

. Vậy d  I ,  SAB   
1 1 1 16 a 3 a 3
Ta có 2
 2
 2
 2  HM 
HM HK SH 3a 4 4

Câu 2. Cho hình chóp S.ABC có SA vuông góc với mặt phẳng (ABC), SA = 8a, tam
giác ABC đều cạnh bằng 4a; M, N lần lượt là trung điểm của cạnh SB và BC. Tính
theo a thể tích khối chóp S.ABC và khoảng cách từ điểm B đến mặt phẳng (AMN).

NGUYỄN VĂN LỰC  0933.168.309 SP Toán K35 - ĐH Cần Thơ


Hình học không gian FB: http://www.facebook.com/VanLuc168

*) Ta có: AN  AB 2  BN 2  2a 3
S
Diện tích tam giác ABC là:
1
S ABC  BC. AN  4a 2 3 .
2
Thể tích hình chóp S.ABC là: M

1 1
VS . ABC  S ABC .SA  4a 2 3.8a A C
3 3
H
32a 3 3 N
 (đvtt).
3 B

*) Ta có:
VB. AMN BA BM BN 1
 . . 
VS . ABC BA BS BC 4

1 8a 3 3
VB. AMN  VS . ABC  .
4 3
1 1
Mặt khác, SB  SC  4 5a  MN  SC  2 5a ; AM  SB  2 5a .
2 2

Gọi H là trung điểm AN thì MH  AN ,  MH  AM 2  AH 2  a 17 .


1 1
Diện tích tam giác AMN là SAMN  AN .MH  2a 3.a 17  a 2 51 .
2 2
Vậy khoảng cách từ B đến (AMN) là:
3VB. AMN 8a 3 3 8a 8a 17
d ( B, ( AMN ))   2   .
SAMN a 51 17 17

Câu 3. Cho hình chóp S.ABC có đáy ABC là tam giác vuông tại B, cạnh SA vuông
góc với mặt đáy. Góc SCB 600 , BC = a, SA a 2 . Gọi M là trung điểm
SB.Tính thể tích khối chóp MABC

BC SA (SAB ) S
BC (SAB ) (do SA cắt BC)
BC AB (SAB )
Mà BC (SBC ) nên (SBC ) (SAB )
a 2
M
0
Ta có, SB BC .tan SCB a.tan 60 a 3 60
2 2 2 2 A C
AB SB SA (a 3) (a 2) a
a
1 1 1 a2 2 B
S MAB
S SAB
SA AB
2 2 2 4

NGUYỄN VĂN LỰC  0933.168.309 SP Toán K35 - ĐH Cần Thơ


Hình học không gian FB: http://www.facebook.com/VanLuc168

1 1
V B h S MAB BC
Thể tích khối chóp M.ABC: 3 3 (đvdt)
1 a2 2 a3 2
a
3 4 12

Câu 4. Cho hình chóp S. ABC có tam giác ABC vuông tại C , AC  a, AB  2a , SA
vuông góc với đáy. Góc giữa mặt phẳng  SAB  và mặt phẳng  SBC  bằng 60 . Gọi
H , K lần lượt là hình chiếu của A lên SB và SC . Chứng minh rằng AK vuông góc
HK và tính thể tích khối chóp S. ABC .

SA  BC, AC  BC  BC   SAC   BC  AK .
Mà AK  SC  AK   SBC   AK  HK .
a2 3 3
S ABC  , AK  AH sin 60  AH
2 2
1 1 1 1 1
2
 2 2
 2  2 (1),
AH SA AB SA 4a
1 1 1 4 1 1
2
 2 2
 2
 2 2
AK SA AC 3 AH SA a
1 3 3
    2
AH 2 4SA2 4a 2
1 2 a 2
Từ (1) và (2) suy ra  2  SA  .
SA 2
a 2
a3 6
VS . ABC  .
12
Câu 5. Cho hình chóp S.ABC có đáy ABC là tam giác vuông tại A, AB = a; AC = 2a.
Mặt bên (SBC) là tam giác cân tại S và nằm trong mặt phẳng vuông góc với đáy. Biết
góc giữa hai mặt (SAB) và (ABC) bằng 300. Tính thể tích khối chóp SABC và khoáng
cách giữa hai đường thẳng SC và AB theo a

Tính VS.ABC
Gọi H là trung điểm BC.

Do SBC cân tại S nên SH  BC .


(SBC)  (ABC)

Ta có: (SBC)  (ABC)  BC  SH  (ABC)
SH  BC

Gọi K là trung điểm của AB  HK // AC mà
NGUYỄN VĂN LỰC  0933.168.309 SP Toán K35 - ĐH Cần Thơ
Hình học không gian FB: http://www.facebook.com/VanLuc168

AC  AB
 HK  AB và SH  AB (do SH  (ABC) )
 AB  (SHK)  AB  SK
(SAB)  (ABC)  AB

SK  AB  Góc giữa (SAB) và (ABC) là SKH  30o
 HK  AB

SH a 3 1 a3 3
tan 30o   SH   VS.ABC  .SH.SABC 
HK 3 3 9
 Tính d(SC,AB)
Vẽ hình chữ nhật BKEC  CE // AB mà AB  (SHK)  CE  (SHK)
d(AB,SC) = d(AB,(SEC)) = d(K,(SEC)) = 2 d(H,(SEC))
Kẻ HF  SE và HF  CE  HF  (SEC)
1 1 1 3 1 4 a a
Ta có: 2
 2
 2
 2  2  2  HF   d(H,(SEC)) =  d(AB,SC) = a.
HF HE SH a a a 2 2

Câu 6. Cho hình chóp S . ABC có đường cao SA bằng 2a , tam giác ABC vuông ở C có
AB  2a, CAB  30 . Gọi H là hình chiếu vuông của A trên SC . Tính theo a thể tích của
khối chóp H . ABC . Tính cô-sin của góc giữa hai mặt phẳng  SAB  ,  SBC  .

S
Trong mặt phẳng  SAC  , kẻ HI song song với SA thì
HI   ABC  .
K

Ta có CA  AB cos 30  a 3. Do đó
1 1 a2 3
S ABC  AB. AC.sin 30  .2a.a 3.sin 30  .
H
2 2 2
A B Ta có
HI HC HC.SC AC 2 AC 2 3a 2 3 6
I        HI  a .
C
SA SC SC 2
SC 2
SA  AC
2 2
4a  3a
2 2
7 7
2 3
1 1 a 3 6 a 3
Vậy VH . ABC  S ABC .HI  . . a .
3 3 2 7 7
1
(Cách khác: VH . ABC  VB. AHC  S AHC .BC )
3

Gọi K là hình chiếu vuông góc của A lên SB . Ta có AH  SC , AH  CB (do


CB   SAC  ), suy ra AH   SBC   AH  SB .
Lại có: SB  AK , suy ra SB   AHK  . Vậy góc giữa giữa hai mặt phẳng  SAB  ,  SBC  là
HKA .
1 1 1 1 1 7 a.2 3
2
 2 2
 2 2 2
 AH  ;
AH SA AC 4a 3a 12a 7
1 1 1 1 1 1
2
 2 2
 2  2  2  AK  a 2 .
AK SA AB 4a 4a 2a
Tam giác HKA vuông tại H (vì AH   SBC  ,  SBC   HK ).

NGUYỄN VĂN LỰC  0933.168.309 SP Toán K35 - ĐH Cần Thơ


Hình học không gian FB: http://www.facebook.com/VanLuc168

a.2 3
AH 7  6  cos HKA  7
sin HKA  
AK a 2 7 7

Câu 7. Cho hình chóp tam giác đều S.ABC có cạnh đáy bằng a và cạnh bên bằng
a 3 . Tính thể tích khối chóp S.ABC và diện tích của mặt cầu ngoại tiếp hình chóp
S.ABC theo a.

S +) Từ giả thiết suy ra tam giác ABC đều


cạnh a và SH(ABC) với H là tâm của
a 3
tam giác đều ABC => AH = và SH
M 3
là đường cao của hình chóp S.ABC
Từ giả thiết => SA = a 3 => trong tam
I giác vuông SAH vuông tại H có
A 2 6a
C SH  SA2  AH 2  .
H 3

a2 3 1 a3 2
+) Diện tích tam giác ABC bằng: S ABC   VS . ABC  S ABC .SH 
4 3 6
+) SH là trục của đường tròn ngoại tiếp tam giác ABC, trong mặt phẳng (SAH) kẻ
đường trung trực của cạnh SA cắt SH tại I => I là tâm của mặt cầu ngoại tiếp hình
chóp S.ABC có bán kính R = IS. Hai tam giác vuông SMI và SHA đồng dạng =>
SM .SA 3 6
SI   a
SH 8
27 2
+) Diện tích mặt cầu là: S  4 R 2  a .
8

Câu 8. Cho hình chóp S.ABC có tam giác ABC vuông tại A, BC = 2a, Góc
ACB  600 . Mặt phẳng (SAB) vuông góc với mp(ABC), tam giác SAB cân tại S, tam
giác SBC vuông tại S. Tính thể tích khối chóp S.ABC và khoảng cách từ điểm A tới
mp(SBC).

NGUYỄN VĂN LỰC  0933.168.309 SP Toán K35 - ĐH Cần Thơ


Hình học không gian FB: http://www.facebook.com/VanLuc168

a) Gọi H là trung điểm của cạnh AB, từ gt có


1
SH  ( ABC ) . VS . ABC  S ABC .SH . Tam giác ABC vuông tại
S 3
A có: AB  2a sin 60  3a; AC  2acos600  a
0

1 3
Nên S ABC  AB. AC  a 2
2 2
Gọi K là trung điểm của cạnh BC thì
1 1 1
A C SK  BC  a; HK  AC  a cos 600  a
600 2 2 2
H K 3
SH 2  SK 2  KH 2  a 2
B 4
3 1
 SH  a . Suy ra VS . ABC  a 3 .
2 4

6
b) Ta có SB  SH 2  HB 2  a
2
3a 2 7a 2
HC  AC  AH  a 
2 2 2 2

4 4
3a 2 7a 2 10
SC  SH 2  HC 2    a
4 4 2
1 1 6 10 15 2
SSBC  SB.SC  . a. a a
2 2 2 2 4
3 3
a
3V 3
Vậy d ( A;( SBC ))  S . ABC  4  a
S SBC 15 2 15
a
4

Câu 9. Cho hình chóp S . ABC có SA   ABC  , ABC  900 , AB  a, BC  a 3, SA  2a Chứng


minh trung điểm I của cạnh SC là tâm của mặt cầu ngoại tiếp hình chóp S . ABC và tính
diện tích mặt cầu đó theo a.

S Vì SA   ABC   SA  BC
Mặt khác theo giả thiết AB  BC , nên
I BC   SAB  và do đó BC  SB

Ta có tam giác SBC vuông đỉnh B; tam


A C giác SAB vuông đỉnh A nên
SC
IA  IB   IS  IC (*)
B 2
Vậy điểm I cách đều bốn đỉnh của hình
chóp, do đó I là tâm mặt cầu ngoại tiếp
của hình chóp S . ABC

NGUYỄN VĂN LỰC  0933.168.309 SP Toán K35 - ĐH Cần Thơ


Hình học không gian FB: http://www.facebook.com/VanLuc168

SC
Từ (*) ta có bán kính của mặt cầu là R 
2
Ta có AC  AB 2  BC 2  2a
SC  SA2  AC 2  2 2a  R  a 2
Diện tích mặt cầu là 4 R 2  8 a 2

NGUYỄN VĂN LỰC  0933.168.309 SP Toán K35 - ĐH Cần Thơ


Hình học không gian FB: http://www.facebook.com/VanLuc168

2. Hình chóp tứ giác


Câu 1. Cho hình chóp đều S.ABCD có cạnh đáy 2a, góc giữa cạnh bên và mặt đáy
bằng 600. Tính thể tích của hình chóp.

A D
60
O
B 2a C
 Gọi O là tâm của mặt đáy thì SO (ABCD ) do đó SO là đường cao
của hình chóp và hình chiếu của SB lên mặt đáy là BO,
do đó SBO 600 (là góc giữa SB và mặt đáy)
SO BD
 Ta có, tan SBO SO BO. tan SBO . tan SBO
BO 2
a 2.tan 600 a 6
 Vậy, thể tích hình chóp cần tìm là
1 1 1 4a 3 6
V B.h AB.BC .SO 2a.2a.a 6
3 3 3 3

Câu 2. Cho hình chóp S.ABCD có đáy ABCD là hình thang vuông tại A và B,
AB  BC  a , CD  2a , SA vuông góc với mặt phẳng (ABCD) và SA  a . Tính thể tích
khối chóp S.ABCD và khoảng cách từ điểm D đến mặt phẳng (SBC).

Kẽ đường thẳng qua C và song song với AB cắt AD tại E.


Ta có: AE  BC  a ; DE= DE  (2a)2  a 2  a 3
Suy ra diện tích hình thang ABCD là: SABCD  1 a2 2  3
2
 
Vậy: VS . ABCD  SA.S SABCD  a 3  2  3 
1 1
3 6
Vì AD//(SBC) nên d ( D, ( SBC ))  d ( A, ( SBC ))
Kẻ AI vuông góc SB tại I, chứng minh được AI vuông góc (SBC).
NGUYỄN VĂN LỰC  0933.168.309 SP Toán K35 - ĐH Cần Thơ
Hình học không gian FB: http://www.facebook.com/VanLuc168

Nên d ( A, ( SBC ))  AI
Trong tam giác SAB vuông tại A có AI là đường cao nên: 1 2  1 2  1 2
AI SA AB
Suy ra: AI  SA.AB  a
SB 2

Câu 3. Cho hình chóp S . ABCD có đáy ABCD là hình vuông cạnh bằng 2a . E , F lần lượt
là trung điểm của AB và BC , H là giao điểm của AF và DE . Biết SH vuông góc với
mặt phẳng ( ABCD ) và góc giữa đường thẳng SA và mặt phẳng ( ABCD ) bằng 60 0 . Tính
thể tích khối chóp S . ABCD và khoảng cách giữa hai đường thẳng SH , DF .

Do ABCD là hình vuông cạnh 2a nên S ABCD  4a 2 .

SH  ( ABCD )  HA là hình chiếu vuông góc của SA trên mp  ABCD 

 SAH  600  SH  AH 3
ABF  DAE  c.g.c   BAF  ADE

Mà: AED  ADE  900 Nên BAF  AED  900  AHE  900  DE  AF
2a
Trong ADE có: AH .DE  AD. AE  AH 
5

1 2a 3 2 8a3 15
Thể tích của khối chóp S . ABCD là: V  . .4a  (đvtt)
3 5 15

Trong mp  ABCD  kẻ HK  DF tại K .  d  SH , DF   HK .

4a
Trong ADE có: DH .DE  DA2  DH  Có : DF  a 5
5

16a 2 9a 2 3a
Trong DHF có: HF 2  DF 2  DH 2  5a 2    HF 
5 5 5
HF .HD 12a 5 12a 5
 HK   Vậy d  SH , DF  
DF 25 25

NGUYỄN VĂN LỰC  0933.168.309 SP Toán K35 - ĐH Cần Thơ


Hình học không gian FB: http://www.facebook.com/VanLuc168

Câu 4. Cho hình chóp S.ABCD có đáy ABCD là hình thang vuông tại A và B biết
AB=AC=a, AD=2a, SA vuông góc với đáy và (SCD) hợp với đáy một góc 60 0. Tính thể
tích khối chóp S.ABCD.

Hình chiếu của SB và SC trên (ABC) là AB và AC , mà SB=SC nên AB=AC


a
Ta có : BC2 = 2AB2 – 2AB2cos1200  a2 = 3AB2  AB =
S 3
2
a a 2
SA2 = a 2   SA = a
3 3
1 1 a2 3 a2 3 a
SABC = AB. AC.sin1200 = =
2 2 3 2 12
2 3 C
1a 2 a 3 a 2
V = = (đvtt) A
3 3 12 36
a
B

Câu 5. Cho hình chóp S . ABCD có đáy ABCD là hình vuông cạnh a 2 , cạnh bên SA
vuông góc với mặt phẳng đáy. Đường thẳng SB tạo với mặt phẳng ( SAD ) một góc 60 0 .
Tính thể tích của khối chóp S . ABCD theo a .

Ta có SA  ( ABCD )  SA là chiều cao


Đáy ABCD là hình vuông cạnh a
nên S ABCD  (a 2) 2  2a 2
Ta có góc [SB,(SAD)] = BSA = 60o

Tam giác SAB vuông tại A có

AB a 2 a 6
AB  a 2  SA  o
 
tan 60 3 3

1 1 a 6 2a 3 6
Vậy V = SABCD .SA  2a 2 . 
3 3 3 9

Câu 6. Cho hình chóp S.ABCD có đáy ABCD là hình thoi ; hai đường chéo AC= 2 3a ,
BD = 2a và cắt nhau tại O; hai mặt phẳng (SAC) và (SBD) cùng vuông góc với mặt
a 3
phẳng (ABCD). Biết khoảng cách từ điểm O đến mặt phẳng (SAB) bằng , tính thể
4
tích khối chóp S.ABCD theo a.

Từ giả thiết AC = 2a 3 ; BD = 2a và AC ,BD vuông góc với nhau tại trung điểm O của
NGUYỄN VĂN LỰC  0933.168.309 SP Toán K35 - ĐH Cần Thơ
Hình học không gian FB: http://www.facebook.com/VanLuc168

mỗi đường chéo.Ta có tam giác ABO vuông tại O và AO = a 3 ; BO = a , do đó


AB D  600
Hay tam giác ABD đều.
Từ giả thiết hai mặt phẳng (SAC) và (SBD) cùng vuông góc với mặt phẳng (ABCD)
nên giao tuyến của chúng là SO  (ABCD).

Do tam giác ABD đều nên với H là trung điểm của AB, K là trung điểm của HB ta có
1 a 3
DH  AB và DH = a 3 ; OK // DH và OK  DH   OK  AB  AB  (SOK)
2 2
Gọi I là hình chiếu của O lên SK ta có OI  SK; AB  OI  OI  (SAB) , hay OI là
khoảng cách từ O đến mặt phẳng (SAB).
1 1 1 a
Tam giác SOK vuông tại O, OI là đường cao  2
 2
 2
 SO 
OI OK SO 2
Diện tích đáy S ABCD  4S ABO  2.OA.OB  2 3a 2 ; S
a
đường cao của hình chóp SO  .
2
Thể tích khối chóp S.ABCD:
1 3a 3
VS . ABCD  S ABCD .SO  D
I
3 3 3a A
O
H
a K
C
B

Câu 7. Cho hình chóp S.ABCD có đáy là hình thoi, tam giác SAB đều và nằm trong
mặt phẳng vuông góc với mặt phẳng (ABCD). Biết AC  2a, BD  4a , tính theo a thể
tích khối chóp S.ABCD và khoảng cách giữa hai đường thẳng AD và SC.

A D
K

H O
B
E C

NGUYỄN VĂN LỰC  0933.168.309 SP Toán K35 - ĐH Cần Thơ


Hình học không gian FB: http://www.facebook.com/VanLuc168

Gọi O  AC  BD , H là trung điểm của AB, suy ra SH  AB .


Do AB  ( SAB )  ABCD ) và ( SAB )  ( ABCD ) nên SH  ( ABCD )
AC 2a BD 4a
+) Ta có OA    a , OB    2a .
2 2 2 2
AB  OA 2  OB 2  a 2  4a 2  a 5

AB 3 a 15
+) SH  
2 2
1 1
S ABCD  AC .BD  2a.4a  4a 2 .
2 2
1 1 a 15 2a 3 15
V  SH .S ABCD   .4a 2 
Thể tích khối chóp S ABCD là : 3 3 2 3 .

Ta có BC // AD nên AD //(SBC)  d ( AD , SC )  d ( AD , ( SBC ))  d ( A, ( SBC )) .


Do H là trung điểm của AB và B = AH  (SBC ) nên d ( A, ( SBC ))  2d ( H , ( SBC )).
Kẻ HE  BC , H  BC , do SH  BC nên BC  (SHE ) .
Kẻ HK  SE , K  SE , ta có BC  HK  HK  ( SBC )  HK  d ( H , ( SBC )) .
2S BCH S ABC S ABCD 4a 2 2a 5
HE     
BC BC 2. AB 2a 5 5 .
1 1 1 5 4 91 2a 15 2a 1365
2
 2
 2
 2 2
 2
 HK  
HK HE SH 4a 15a 60a 91 91
4a 1365
d ( AD, SC )  2 HK 
Vậy 91 .

Câu 8. Cho hình chóp S.ABCD có đáy là hình vuông cạnh a, SA  a, SA  (ABCD).
Gọi M là trung điểm của SA. Mặt phẳng (BCM) cắt SD tại N. Tính thể tích khối chóp
S.BCMN và khoảng cách giữa SB và AC.

NGUYỄN VĂN LỰC  0933.168.309 SP Toán K35 - ĐH Cần Thơ


Hình học không gian FB: http://www.facebook.com/VanLuc168

Do (BCM) // AD nên mp này cắt mp (SAD) theo giao


tuyến MN // AD.
 BC  AB
  BC  BM
Ta có  BC  SA
Tứ giác BCMN là hình thang vuông có BM là
a a 5
đường cao, MN  ; BM  .
2 2
 a a 5
 a  . 3a 2 5
Diện tích hình thang BCMN là S BCMN   2  2  .
2 8
Dụng SK  BM , do BC  ( SAB)  BC  SK  SK  ( BCMN ) .
a 5 1 3a 2 5 a 5 a3
Có SK  d  A, BM   . Vậy VS .BCMN  . .  .
5 3 8 5 8
Trong mặt phẳng (ABCD) dựng  qua B song song với AC. Đặt (P) = (  , SB).
Khi đó, AC // (P) và d(AC; SB) = d(AC; (P)) = d(A; (P)).
Từ A hạ AI   tại I; Từ A hạ AH  SI tại H suy ra AH = d(A; (P)).
a a 3
Ta có AI   AH  .
2 3

Câu 9. Cho khối chóp S.ABCD có đáy ABCD là hình chữ nhật, biết AB=2a , AD= a .
a
Trên cạnh AB lấy điểm M sao cho AM  , cạnh AC cắt MD tại H . Biết SH vuông
2
góc với mặt phẳng (ABCD) và SH = a . Tính thể tích khối chóp S. HCD và tính khoảng
cách giữa hai đường thẳng SD và AC theo a.

* Tính thể tích khối chóp S.HCD:


AM AD 1
Hai tam giác vuông AMD và DAC có   nên đồng dạng,
AD DC 2
Suy ra ADH  DCH , mà ADH  HDC  90  DHC  90
 ADC vuông tại D: AC2  AD2  DC2  AC  a 5
Hệ thức lượng  ADC: DH.AC = DA.DC

NGUYỄN VĂN LỰC  0933.168.309 SP Toán K35 - ĐH Cần Thơ


Hình học không gian FB: http://www.facebook.com/VanLuc168

DC.DA 2a
Suy ra: DH  
AC 5
4a
 DHC vuông tại H: HC  DC2  DH 2 
5

1 4a 2
Do đó diện tích  HCD: SHCD  DH.HC 
2 5
1 4a 3
Thể tích khối chóp SHCD: VS.HCD  SH.SHCD 
3 15

Tính khoảng cách giữa SD và AC:


Dựng HE  SD
Ta có SH  (ABCD) nên SH  AC và DH  AC , do đó AC  (SHD)
Mà HE  (SHD) nên HE  AC
Từ đó HE là đoạn vuông góc chung của SD và AC.
nên HE  d SD; AC 
 SHD vuông tại H nên:
1 1 1 2a
   HE 
HE 2 SH2
HD2 3
2a
Vậy d SD; AC   HE 
3

Câu 10. Cho hiǹ h chóp S.ABCD có đáy ABCD là hiǹ h thang với đáy lớn là AD; các
đường thẳng SA, AC và CD đôi mô ̣t vuông góc với nhau; AC  CD  a 2 và AD  2 BC .
Tính thể tích của khố i chóp S.ABCD và khoảng cách giữa hai đường thẳ ng SB và CD.

Ta có: SA  AC và SA  CD S

 SA  (ABCD).
 ACD vuông cân ta ̣i C
 AD = 2a  BC = a. K

Go ̣i I là trung điể m AD  AI = BC, AI // BC và CI 


D
A I

AD  ABCI là hin


H
̀ h vuông. B C

 AB  AD.
2 2 3
Do đó SABCD = (AD  BC).AB  3a . Vâ ̣y VSABCD = 1 .SABCD .SA  1 . 3a .a 2  a 2 .
2 2 3 3 2 2

Ta có CD // BI  CD // (SBI)  d(SB, CD) = d(CD, (SBI)) = d(C, (SBI))


Go ̣i H = AC  BI và AK  SH tại K. Ta có AK  (SBI)  d(A, (SBI)) = AK.

1 1 1 1 4 5 a 10
Ta có       AK = .
AK 2 SA 2 AH2 2a2 2a2 2a2 5

a 10
 d(A; (SBI)) = AK = . Vì H là trung điể m AC nên d(C; (SBI)) = d(A; (SBI)) =
5
NGUYỄN VĂN LỰC  0933.168.309 SP Toán K35 - ĐH Cần Thơ
Hình học không gian FB: http://www.facebook.com/VanLuc168

a 10 a 10
. Vâ ̣y d(CD, SB) = .
5 5

Câu 11. Cho hình chóp S .ABCD có đáy ABCD là hình vuông cạnh bằng a , tâm O và
SO vuông góc với mặt phẳng ABCD . Trên cạnh SA lấy điểm M sao cho MA = 2MS.

Gọi N là trung điểm của CD , SNO 600 . Tính thể tích khối chóp S .ABCD theo a và
cosin góc giữa MN với mặt phẳng (ABCD ) .

a
Xét SON vuông ta ̣i O, cóON  , SNO  600
S 2
a 3
SO  ON . tan 600 
2
M
Diện tić h hiǹ h vuông ABCD là SABCD  a 2
1 a3 3
A  VS .ABCD  SO.SABCD 
D 3 6
H Kẻ MH SO (H  BD )  MH  (ABCD )
O N
B C Khi đó, ta có hình chiế u vuông góc của MN trên
(ABCD) là HN suy ra góc giữa MN và (ABCD) là   MNH
2 2
Vì MH SO, MA  2MS  BH  2HO nên ta có HD  BD  a 2
3 3
17 2 a 17
Xét HND , ta có HN 2  HD 2  DN 2  2HD.DN .cos 450  a  HN 
16 4
MH 2 51 17
Xét MHN vuông ta ̣i H, ta có tan     cos  
HN 17 29

Câu 12. Cho hình chóp S.ABCD có đáy ABCD là hình chữ nhật. Tam giác SAB đều và
nằm trong mặt phẳng vuông góc với mặt phẳng đáy (ABCD). Biết SD  2a 3 và góc tạo
bởi đường thẳng SC với mặt phẳng (ABCD) bằng 300 . Tính theo a thể tích khối chóp
S.ABCD và khoảng cách từ điểm B đến mặt phẳng (SAC).

Gọi H là trung điểm của AB. Suy ra


S
SH  ( ABCD )
và SCH  300 .
K
Ta có: SHC  SHD  SC  SD  2a 3 .
A
D Xét tam giác SHC vuông tại H ta có:
I
H
SH  SC.sin SCH  SC.sin 300  a 3
B C HC  SC.cos SCH  SC.cos 300  3a
NGUYỄN VĂN LỰC  0933.168.309 SP Toán K35 - ĐH Cần Thơ
Hình học không gian FB: http://www.facebook.com/VanLuc168

Vì tam giác SAB đều mà SH  a 3 nên AB  2a . Suy ra


BC  HC 2  BH 2  2a 2 . Do đó, S ABCD  AB.BC  4a 2 2 .
1 4a 3 6
Vậy, VS . ABCD  S ABCD .SH  .
3 3
Vì BA  2HA nên d  B,  SAC    2d  H ,  SAC  
Gọi I là hình chiếu của H lên AC và K là hình chiếu của H lên SI. Ta có:
AC  HI và AC  SH nên AC   SHI   AC  HK . Mà, ta lại có: HK  SI .
Do đó: HK   SAC  .
HI AH AH .BC a 6
Vì hai tam giác SIA và SBC đồng dạng nên   HI   .
BC AC AC 3
HS .HI a 66
Suy ra, HK   .
HS  HI
2 2 11

Vậy , d  B,  SAC    2d  H ,  SAC    2HK 


2a 66
11

Câu 13. Cho hình chóp S . ABCD có đáy ABCD là hình thoi cạnh a, mặt bên SAD là tam
a 6
giác đều nằm trong mặt phẳng vuông góc với đáy, SC  . Tính thể tích khối chóp
2
S . ABCD và khoảng cách giữa hai đường thẳng AD, SB theo a.

D C

A B

Gọi H là chân đường cao hạ từ S của tam giác đều SAD


Suy ra:
a 3
SH  và SH   ABCD 
2
a 3
Trong tam giác vuông HSC có HC 
2
2
a 3a 2
 a2 
DH  DC  CH
2 2 2
4  1  HDC  600
cos HDC   4
2 DH .DC a 2
2. .a
2

NGUYỄN VĂN LỰC  0933.168.309 SP Toán K35 - ĐH Cần Thơ


Hình học không gian FB: http://www.facebook.com/VanLuc168

a2 3
Suy ra S ABCD  DA.DC.sin ADC 
2
2
1 1a 3 a 3 1 3
VS . ABCD  SH .S ABCD  .  a
3 3 2 2 4
Ta có ADC đều cạnh a  CH  AD  CH  BC
hay BC   SHC   BC  SC  CSB vuông tại C
1 1 a3 a3
Lại có VD.SBC  VS .BCD  VS . ABCD  . 
2 2 4 8
a3 3a3
 d  D;  SBC   .SSBC   d  D;  SBC   
1
3 8 8.SSBC
3a3 3a3
 d  D;  SBC   
a 6
  .
1 a 6 4
8. CS .CB 4. .a
2 2
Vậy d  AD; SB   d  D;  SBC   
a 6
.
4

Câu 14. Cho hình chóp S.ABCD có đáy ABCD là hình vuông, cạnh AB  2a . Hình
chiếu vuông góc của S lên mặt phẳng (ABCD) trùng với trọng tâm G của tam giác
ABC, góc giữa SA và mặt phẳng ( ABCD ) bằng 30 0 . Tính theo a thể tích khối chóp
S.ABCD và cosin của góc giữa đường thẳng AC và mặt phẳng (SAB).

K
A
I B

G
O M

D C

Gọi M là trung điểm BC, O là giao điểm của AC và BD. Ta có


2 2 5a
AM  AB 2  BM 2  a 5  AG  AM  . Vì SG vuông góc với mặt đáy, nên góc
3 3
giữa SA và mặt đáy là SAG  300 . Xét tam giác vuông SGA, ta có
1 SG 2 5a
tan SAG  tan 300    SG  .
3 AG 3 3
1 1 2 5a 2 8 15a3
S ABCD  4a 2 . Suy ra VS . ABCD  SG.S ABCD  . .4a  (đvtt)
3 3 3 3 27

NGUYỄN VĂN LỰC  0933.168.309 SP Toán K35 - ĐH Cần Thơ


Hình học không gian FB: http://www.facebook.com/VanLuc168

Hạ GI vuông góc với AB, I thuộc AB. Nối S với I, hạ GK vuông góc với SI, K thuộc
2 2a
SI. Khi đó K là hình chiếu vuông góc của G trên (SAB). Ta có GI  MB  , do đó
3 3
GS .GI 10a
GK   .
GS  GI
2 2 6

3 10a
Gọi H là hình chiếu vuông góc của O lên (SAB), ta có OH  GK  . Khi đó AH là
2 4
hình chiếu của AO lên (SAB) suy ra góc giữa AC và (SAB) là OAH . Xét tam giác
OH 10a 5 11
vuông OHA, ta có sin OAH     cos OAH  .
OA 4. 2.a 4 4

3a
Câu 15. Cho hình chóp S . ABCD có đáy là hình vuông cạnh a, SD  . Hình chiếu
2

vuông góc H của đỉnh S lên mặt phẳng (ABCD) là trung điểm của đoạn AB . Gọi K là
trung điểm của đoạn AD . Tính theo a thể tích khối chóp S . ABCD và khoảng cách giữa
hai đường thẳng HK và SD .

F
B C
E
H
O
A K D

Từ giả thiết ta có SH là đường cao của hình chóp S.ABCD và


3a 2 a 2
SH  SD 2  HD 2  SD 2  ( AH 2  AD 2 )  ( )  ( )  a2  a
2 2
1 1 a3
Diện tích của hình vuông ABCD là a 2 , VS . ABCD  SH .S ABCD  a.a 2 
3 3 3
Từ giả thiết ta có HK / / BD  HK / /( SBD)
Do vậy: d ( HK , SD )  d ( H ,( SBD )) (1)
Gọi E là hình chiếu vuông góc của H lên BD, F là hình chiếu vuông góc của H lên SE
Ta có BD  SH , BD  HE  BD  ( SHE )  BD  HF mà HF  SE nên suy ra
HF  ( SBD)  HF  d ( H , ( SBD)) (2)

NGUYỄN VĂN LỰC  0933.168.309 SP Toán K35 - ĐH Cần Thơ


Hình học không gian FB: http://www.facebook.com/VanLuc168

a a 2
+) HE  HB.sin HBE  .sin 450 
2 4
+) Xét tam giác vuông SHE có:
a 2
a.
SH .HE 4 a
HF .SE  SH .HE  HF    (3)
SE a 2 2 3
( )  a2
4
a
+) Từ (1), (2), (3) ta có d ( HK , SD )  .
3

Câu 16. Cho hình chóp S.ABCD có đáy ABCD là hình vuông cạnh a. Gọi I là trung
điểm AB, H là giao điểm của BD với IC. Các mặt phẳng (SBD) và (SIC) cùng vuông
góc với đáy. Góc giữa (SAB) và (ABCD) bằng 60 0 . Tính thể tích khối chóp S.ABCD và
khoảng cách giữa hai đường thẳng SA và IC.

1
Ta có VS.ABCD  SH.SABCD , trong đó SABCD  a 2
3
Do (SIC),(SBD) cùng vuông với đáy suy ra
SH  (ABCD)
F
D A Dựng HE  AB   SHE   AB , suy ra SEH là góc
K
M
giữa (SAB) và (ABCD)  SEH  600
P I Ta có SH  HE.tan 600  3HE
H
C E HE HI 1 a
B
   HE 
CB IC 3 3
a 3
 SH 
3
1 1 a 3 2 3a 3
Suy ra VS.ABCD  SH.SABCD  . .a 
3 3 3 9

Gọi P là trung điểm của CD, suy ra AP song song vớiCI


 d  SA, CI   d  CI, SAP    d  H, SAP  

Dựng HK  AP , suy ra  SHK    SAP 


Dựng HF  SK  HF  SPA   d  H, SPA    HF
1 1 1
Do SHK vuông tại H  2
 2
 (1)
HF HK HS2
1 1 1 1
Dựng DM  AP , ta thấy DM  HK  2
 2
 2

HK DM DP DA 2
1 1 1 1 4 1 3 8 a
Thay vào (1) ta có  2  2  2
 2
 2  2  2  2  HF  .
HF DP DA HS a a a a 2 2
a
Vậy d  SA, CI   .
2 2
NGUYỄN VĂN LỰC  0933.168.309 SP Toán K35 - ĐH Cần Thơ
Hình học không gian FB: http://www.facebook.com/VanLuc168

Câu 17. Cho hình chóp S.ABCD có SA vuông góc với mặt đáy (ABCD), đáy ABCD là
hình chữ nhật có AD = 3a, AC = 5a, góc giữa hai mặt phẳng (SCD) và (ABCD) bằng
450. Tính theo a thể tích khối chóp S.ABCD và tính góc giữa đường thẳng SD và mặt
phẳng (SBC).

- Tính thể tích

+) Ta có: AB  AC 2  BC 2  4a
S K

+) Mà   SCD  ,  ABCD    SDA  450


H
nên SA = AD = 3a
A D
1
Do đó: VS . ABCD  SA.S ABCD  12a 3 (đvtt)
3
B C

- Tính góc…

+) Dựng điểm K sao cho SK  AD

Gọi H là hình chiếu vuông góc của

D lên CK, khi đó: DK   SBC  . Do đó:  SD,  SBC    DSH

DC.DK 12a
+) Mặt khác DH   , SD  SA2  AD 2  3a 2
KC 5

3a 34
SH  SD 2  DH 2 
5

Do đó:  SD,  SBC    DSH  arccos


SH 17
 arccos  340 27 '
SD 5

Câu 18. Cho hình chóp S.ABCD có đáy ABCD là hình thoi tâm O, hai đường chéo
AC  2 3a, BD  2a , BD = 2a; hai mặt phẳng (SAC) và (SBD) cùng vuông góc với mặt

phẳng (ABCD). Biết góc giữa đường thẳng SB và mặt phẳng (SAC) bằng 30 0. Tính thể
tích khối chóp S.ABCD và khoảng cách giữa 2 đường thẳng SB và AC.

Tính VS . ABCD và d(SB , AC)

NGUYỄN VĂN LỰC  0933.168.309 SP Toán K35 - ĐH Cần Thơ


Hình học không gian FB: http://www.facebook.com/VanLuc168

Cm được góc giữa SB và mp (SAC) là góc BSO  300


1 1 1
VS . ABCD  S ABCD .SO  . AC.BD.SO  2a 3
3 3 2
Vẽ OH vuông góc SB . Chứng minh được d(SB , AC) = OH (đường vuông góc chung)
a 3
Tính được: d(SB , AC) = OH 
2


Câu 19. Cho hình chóp S.ABCD có đáy ABCD là hình thoi cạnh a, ABC  60 0 . Cạnh
bên SA vuông góc với mặt đáy và cạnh bên SC tạo với mặt đáy một góc 600 . Gọi I là
trung điểm BC, H là hình chiếu vuông góc của A lên SI.
a) Tính thể tích khối chóp S.ABCD.
b) Tính khoảng cách từ điểm H đến mặt phẳng (SCD) theo a.

S

a) Do ABC =600 nên tam giác ABC đều, suy ra
3
SABCD  a 2 và AC  a
2

K Mặt khác SA  ( ABCD)  SCA  60 0
H
1 a3
 SA  AC.tan 600  a 3  VS.ABCD  SA.SABCD  .
A D 3 2
2 2
HS HS.IS AS AS 4
E
b)Ta có   2  2 
IS IS 2
IS IA  AS 2
5
B I C
 d  H,  SCD    d  I,  SCD    d  B,  SCD    d  A,  SCD  
4 2 2
5 5 5
( vì I là trung điểm BC và AB//(SCD))

Gọi E là trung điểm CD, K là hình chiếu của A lên SE, ta


có AE  DC  DC  (SAE)  AK  (SCD)
Suy ra d  H, SCD    d  A, SCD    AK 
2 2 2 SA.AE 2a 15
 .
5 5 5 SA 2  AE 2 25

Câu 20. Cho hình chóp S. ABCD có đáy ABCD là hình chữ nhật .Biết SA  ( ABCD ) , SC
4
hợp với mặt phẳng ( ABCD ) một góc  với tan   , AB  3a và . Tính thể tích
y

x
-8 -6 -4 -2 2 4 6 8

-5

5
của khối chóp và khoảng cách từ điểm D đến mặt phẳng (SBC ) .
y

x
-8 -6 -4 -2 2 4 6 8

-5

NGUYỄN VĂN LỰC  0933.168.309 SP Toán K35 - ĐH Cần Thơ


Hình học không gian FB: http://www.facebook.com/VanLuc168

A
D
3a
α
B
4a C


Xác định đúng góc SCA  
1 1 4
Thể tích VSABCD  S ABCD .SA  .3a.4a. .5a  16a 3
3 3 5
Khoảng cách từ điểm D đến mặt phẳng (SBC)
Xác định dược khoảng cách d D, (SBC   d  A, (SBC   AH
Tính đúng d D, ( SBC )   AH 
12a
5
Vậy : D(0; 0; 0) và D( 6; 0; 0 )

3. Hình lăng trụ đứng


Câu 1. Cho hình lăng trụ ABC . A ' B ' C ' , ABC đều có cạnh bằng a , AA '  a và đỉnh
A ' cách đều A, B, C . Gọi M , N lần lượt là trung điểm của cạnh BC và A ' B . Tính
theo a thể tích khối lăng trụ ABC . A ' B ' C ' và khoảng cách từ C đến mặt phẳng
( AMN ) .

 Gọi O là tâm tam giác đều ABC  A’O  (ABC)


a 3 2 a 3
Ta có AM  , AO  AM 
2 3 3
a2 a 6 a2 3
A ' O  AA '2  AO 2  a 2   ; SABC 
3 3 4
a2 3 a 6 a2 2
Thể tích khối lăng trụ ABC . A ' B ' C ' : V  SABC . A ' O  . 
4 3 4

NGUYỄN VĂN LỰC  0933.168.309 SP Toán K35 - ĐH Cần Thơ


Hình học không gian FB: http://www.facebook.com/VanLuc168

1 3VNAMC
 Ta có VNAMC  SAMC .d  N ,( ABC )  d C ,( AMN ) 
3 SAMC
1 a2 3 1 a 6
S AMC  S ABC  ; d  N ,( ABC )  A ' O 
2 8 2 6
2 2
1a 3 a 6 a 2
Suy ra: VNAMC  . 
3 8 6 48
a 3
lại có : AM  AN  , nên AMN cân tại A
2
A 'C a
Gọi E là trung điểm AM suy ra AE  MN , MN  
2 2
3a 2 a 2 a 11 1 a 2 11
 AE  AN 2  NE 2    ; S AMN  MN . AE 
4 16 4 2 16
2
3a 2 a 11 a 22
 d C ,( AMN )  :  (đvđd)
48 16 11

Câu 2. Cho hình lăng trụ ABC.A’B’C’ có đáy là tam giác đều cạnh a, hình chiếu
vuông góc của A’ lên măt phẳng (ABC) trùng với tâm O của tam giác ABC. Tính thể
a 3
tích khối lăng trụ ABC.A’B’C’ biết khoảng cách giữa AA’ và BC là
4

Gọi M là trung điểm BC ta thấy:


AM  BC 
  BC  ( A' AM )
A' O  BC 

Kẻ MH  AA' , (do A nhọn nên H thuộc trong đoạn
AA’.)
BC  ( A' AM ) 
Do   HM  BC .Vậy HM là đọan vuông
HM  ( A' AM )
góc chung của

NGUYỄN VĂN LỰC  0933.168.309 SP Toán K35 - ĐH Cần Thơ


Hình học không gian FB: http://www.facebook.com/VanLuc168

3
AA’và BC, do đó d ( AA' , BC)  HM  a .
4

A' O HM
Xét 2 tam giác đồng dạng AA’O và AMH, ta có: 
AO AH
AO.HM a 3 a 3 4 a
 suy ra A' O   
AH 3 4 3a 3
1 1aa 3 a3 3
Thể tích khối lăng trụ: V  A' O.S ABC  A' O.AM.BC  a
2 23 2 12

Câu 3. Cho hình lăng trụ đứng ABCD.A ' B'C' D ' có đáy là hình thoi cạnh a, BAD  120o và
AC'  a 5. Tính thể tích khối lăng trụ ABCD.A ' B'C' D ' và khoảng cách giữa hai đường
thẳng AB' và BD theo a.

A'
Gọi O là tâm hình thoi ABCD. D'

Do hình thoi ABCD có


BAD  120o
 ABC, ACD đều.
B' C'

 AC  a.
A D
a2 3 120o H
Ta có: SABCD  2SABC 
2 O

B C

Mà ABCD.A ' B'C' D ' là lăng trụ đứng.


 ACC ' vuông tại C  CC'  AC'2  AC2  5a2  a2  2a.
a2 3
Vậy VABCD.A'B'C'D'  CC'.SABCD  2a   a3 3.
2
Tứ giác AB'C' D là hình bình hành  AB' // C' D  AB' // (BC ' D).
 d(AB',BD)  d(AB',(BC' D))  d(A,(BC' D))  d(C,(BC ' D)).
Vì BD  AC,BD  CC'  BD  (OCC')  (BC' D)  (OCC').
Trong (OCC'), kẻ CH  OC' (H  OC').
 CH  (BC' D)  d(C,(BC' D))  CH
1 1 1 4 1 2a
OCC' vuông tại C     2  2  CH 
CH 2
CO CC'
2 2
a 4a 17
2a
Vậy d(AB',BD)  
17

Câu 4. Cho lăng trụ đứng ABC. A ' B ' C ' có đáy ABC là tam giác vuông tại A , AB  a,
ABC  600. Góc giữa đường thẳng A ' C và mặt phẳng ( ABC ) bằng 450 . Tính thể tích của
khối lăng trụ theo a và cosin của góc giữa đường thẳng AB ' và mặt phẳng ( A ' BC ) .
NGUYỄN VĂN LỰC  0933.168.309 SP Toán K35 - ĐH Cần Thơ
Hình học không gian FB: http://www.facebook.com/VanLuc168

Theo giả thiết, ta có


a
AB  BC.cos ABC  BC   2a  AC  3a . Góc giữa
A'
C'
cos 600
A ' C và (ABC) là A ' CA  450  AA '  AC  3a .
B'

I
1 3a 3
K Vậy thể tích khối lăng trụ là V  AA '. AB. AC  .
A
C
2 2

Gọi I là tâm hình chữ nhật ABB’A’, H là hình chiếu vuông góc của A trên BC, K là hình
 BC  AH
chiếu vuông góc của A trên A’H. Ta có   BC  AK . Do đó
 BC  A ' A
AK  ( A ' BC ) . Vậy IK là hình chiếu của IA lên ( A ' BC ) , hay góc giữa AB’ và mặt
phẳng ( A ' BC ) là AIK .
Dễ thấy AB '  AA '2  A ' B '2  2a  IA  a. Ứng dụng hệ thức trong tam giác vuông, ta có
1 1 1 1 1 1 5 3a
2
 2
 2
 2
 2
 2
 2 . Suy ra AK  .
AK AA ' AH AA ' AB AC 3a 5
AK 3 2
Xét tam giác vuông AKI. ta có sin AIK    cos AIK  .
AI 5 5

Câu 5. Cho lăng trụ tam giác ABC.A1B1C1 có tất cả các cạnh bằng a , góc tạo bởi cạnh
bên và mặt phẳng đáy bằng 300. Hình chiếu H của điểm A trên mặt phẳng (A1B1C1)
thuộc đường thẳng B1C1. Tính thể tích khối lăng trụ ABC.A1B1C1 và tính khoảng cách
giữa hai đường thẳng AA1 và B1C1 theo a.

Do AH  ( A1 B1C1 ) nên góc AA1 H là góc giữa AA1 và (A1B1C1), theo giả thiết thì góc
AA1 H bằng 300.

NGUYỄN VĂN LỰC  0933.168.309 SP Toán K35 - ĐH Cần Thơ


Hình học không gian FB: http://www.facebook.com/VanLuc168

a
Xét tam giác vuông AHA1 có AA1 = a, góc AA1 H =300  AH  .
2

a a 2 3 a3 3
VABCA1B1C1  AH .S A1B1C   
2 4 8

a 3
Xét tam giác vuông AHA1 có AA1 = a, góc AA1 H =300  A1 H  . Do tam giác
2

a 3
A1B1C1 là tam giác đều cạnh a, H thuộc B1C1 và A1 H  nên A1H vuông góc với
2
B1C1. Mặt khác AH  B1C1 nên B1C1  ( AA1 H )
Kẻ đường cao HK của tam giác AA1H thì HK chính là khoảng cách giữa AA1 và B1C1
A1 H . AH a 3
Ta có AA1.HK = A1H.AH  HK  
AA1 4

NGUYỄN VĂN LỰC  0933.168.309 SP Toán K35 - ĐH Cần Thơ


Lượng giác FB: http://www.facebook.com/VanLuc168

I. CÔNG THỨC LƯỢNG GIÁC


Chuyên đề: Lượng giác

I. Đơn vị đo góc và cung:


1. Độ: 180 o
Goùc 10  1 goùc beït
180
. x
y O
2. Radian: (rad)
180 0   rad

3. Bảng đổi độ sang rad và ngược lại của một số góc (cung ) thông dụng:

Độ 00 300 450 600 900 1200 1350 1500 1800 3600


Radian 0     2 3 5  2
6 4 3 2 3 4 6

II. Góc lượng giác & cung lượng giác:


1. Định nghĩa: y
(tia ngọn)
y

(điểm ngọn) 
 B
 t
t  M
x
 O
x A (điểm gốc)
O (tia gốc)
(Ox, Oy )    k 2 (k  Z)
AB    k 2
2. Đường tròn lượng giác:

Số đo của một số cung lượng giác đặc biệt: AM k2


y
B 

C O A x

D 

NGUYỄN VĂN LỰC  0933.168.309 SP Toán K35 - ĐH Cần Thơ


Lượng giác FB: http://www.facebook.com/VanLuc168
A  2k 
B    2k
2
C    2k 
-   2k
M
D 
2
A, C  k
B, D    k
2

III. Định nghĩa hàm số lượng giác:

1. Đường tròn lượng giác: y t


 A: điểm gốc B u
 x'Ox : trục côsin ( trục hoành ) u' 1

 y'Oy : trục sin ( trục tung )
1 R 1 1
 t'At : trục tang C O A x
x'
 u'Bu : trục cotang

1 D
2. Định nghĩa các hàm số lượng giác: y'
t'
a. Định nghĩa: Trên đường tròn lượng giác cho AM .
Gọi P, Q lần lượt là hình chiếu vuông góc của M trên x'Ox và y'Oy
T, U lần lượt là giao điểm của tia OM với t 'At và u'Bu
Ta định nghĩa: t y t
Trục sin
Trục cotang
u' B U
u
Q
M
T  cos   OP
t  
x sin   OQ
x' O P
A tan  AT

cot   BU
Trục cosin 1
Trục tang
y' t'

b. Các tính chất :


 Với mọi  ta có :
1  sin   1 hay sin  1
1  cos  1 hay cos   1

 tan xác định    k
2
 cot xác định   k

NGUYỄN VĂN LỰC  0933.168.309 SP Toán K35 - ĐH Cần Thơ


Lượng giác FB: http://www.facebook.com/VanLuc168
c. Tính tuần hoàn

sin(  k 2 )  sin 
cos(  k 2 )  cos 
(k  Z )
tan(  k )  tan 
cot(  k )  cot 

IV. Giá trị các hàm số lượng giác của các cung (góc ) đặc biệt:
Ta nên sử dụng đường tròn lượng giác để ghi nhớ các giá trị đặc biệt

y
t

- 3 -1 - 3 /3 B /2 3 /3 1 3
u' 2/3
1 /3 u
3 /2 /4
3/4
2 /2 /6
5/6 3 /3
1/2


x'  - 3 /2 - 2 /2 -1/2 1/2 2 /2 3 /2 1 A (Ñieåm goác) x
-1 O

-1/2
- 3 /3
-/6
- 2 /2 
- 3 /2 -/4
-1 -/3 -1
- /2

- 3
y' t'

Góc 00 300 450 900 1200 1350 1500 1800 3600


600
Hslg 0     2 3 5  2
6 4 3 2 3 4 6
sin  0 1 2 3 1 3 2 1 0 0
2 2 2 2 2 2
cos  1 3 2 1 0 
1 2 3 -1 1
 
2 2 2 2 2 2
tan  0 3 1 3 kxđ  3 -1 3 0 0

3 3
cot  kxđ 3 1 3 0 3 -1  3 kxđ kxđ

3 3
NGUYỄN VĂN LỰC  0933.168.309 SP Toán K35 - ĐH Cần Thơ
Lượng giác FB: http://www.facebook.com/VanLuc168
V. Hàm số lượng giác của các cung (góc) có liên quan đặc biệt:
Đó là các cung :
 
1. Cung đối nhau:  vaø - (tổng bằng 0) (Vd: & ,…)
6 6
 5
2. Cung bù nhau:  vaø  - ( tổng bằng  ) (Vd: & ,…)
6 6

   
3. Cung phụ nhau:  vaø  ( tổng bằng ) (Vd: & ,…)
2 2 6 3

   2
4. Cung hơn kém :  vaø   (Vd: & ,…)
2 2 6 3

 7
5. Cung hơn kém  :  vaø    (Vd: & ,…)
6 6

1. Cung đối nhau: 2. Cung bù nhau:

cos( )  cos  cos(   )   cos 


sin( )   sin  Đối cos Bù sin sin(   )  sin 
tan( )   tan  tan(   )   tan 
cot( )   cot  cot(   )   cot 


3. Cung phụ nhau: 4. Cung hơn kém :
2

 
cos(   )  sin  cos(   )   sin 
2  2
 Hơn kém 
sin(   )  cos  Phụ chéo 2 sin(   )  cos 
2 sin bằng cos 2
 cos bằng trừ sin 
tan(   )  cot tan(   )  cot
2 2
 
cot(   )  tan  cot(   )   tan 
2 2

5. Cung hơn kém  :

cos(   )   cos 
sin(   )   sin  Hơn kém 
tan(   )  tan tang, cotang
cot(   )  cot 

NGUYỄN VĂN LỰC  0933.168.309 SP Toán K35 - ĐH Cần Thơ


Lượng giác FB: http://www.facebook.com/VanLuc168
VI. Công thức lượng giác:
1. Các hệ thức cơ bản:
1
2 2 1  tan 2 =
cos   sin   1 cos2
sin 1
tan = 1  cot 2 =
cos sin 2 
cos tan . cot = 1
cot =
sin

2. Công thức cộng:


cos(   )  cos  .cos   sin  .sin 
cos(   )  cos  .cos   sin  .sin 
sin(   )  sin  .cos   sin  .cos 
sin(   )  sin  .cos   sin  .cos 
tan +tan
tan( + ) =
1  tan  .tan 
tan  tan
tan(   ) =
1  tan  .tan 

3. Công thức nhân đôi: cos2


1 cos 2
2
cos 2  cos2   sin 2 
 2 cos2   1 1 cos 2
2 sin2
 1  2 sin  2
 cos4   sin 4 
sin 2  2 sin  .cos 
2 tan  1
tan 2  sin  cos   sin 2
2
1  tan 2 

4 Công thức nhân ba:


cos 3  3 cos 
cos 3  
cos3  4cos3   3cos  4
sin 3  3sin   4sin 3 
3 sin   sin 3
sin 3  
4
5. Công thức hạ bậc:

1 cos 2 1 cos 2 1 cos 2


cos2 ; sin2 ; tan 2
2 2 1 cos 2

6. Công thức tính sin  ,cos  ,tg theo t  tan  :


2

2t 1 t2 2t
sin ; cos ; tan
1 t2 1 t2 1 t2

NGUYỄN VĂN LỰC  0933.168.309 SP Toán K35 - ĐH Cần Thơ


Lượng giác FB: http://www.facebook.com/VanLuc168

7. Công thức biến đổi tích thành tổng :


1
cos .cos    cos(   )  cos(   )
2
1
sin  .sin    cos(   )  cos(   )
2
1
sin  .cos   sin(   )  sin(   )
2

8. Công thức biến đổi tổng thành tích:

   
cos   cos   2 cos .cos
2 2
   
cos   cos   2 sin .sin
2 2
   
sin   sin   2 sin .cos
2 2
   
sin   sin   2 cos .sin
2 2
sin(   )
tan   tan  
cos  cos 
sin(   )
tan   tan  
cos  cos 

9. Các công thức thường dùng khác:

  3 cos 4
cos  sin   2 cos(  )  2 sin(  ) cos4 sin 4
4 4 4
  5 3 cos 4
cos  sin   2 cos(  )   2 sin(  ) cos6 sin6
4 4 8

NGUYỄN VĂN LỰC  0933.168.309 SP Toán K35 - ĐH Cần Thơ


Lượng giác FB: http://www.facebook.com/VanLuc168

II. BIẾN ĐỔI LƯỢNG GIÁC


Chuyên đề: Lượng giác

1
Ví dụ 1: Cho góc ; mà sin . Tính sin
2 5 6
Bài giải
♥ Từ hệ thức: cos2 sin 2 1 và ;
2
1 2
Suy ra: cos 1 sin2 1 (2)
5 5
3 2
Thay (2) vào (1) ta được: sin
6 2 5
3 1
Ví dụ 2: Cho góc ;2 mà sin cos . Tính sin 2
2 2 2 2
Bài giải
1 1 3
♥ Từ sin cos 1 sin sin
2 2 2 4 4
9 7
cos2 1 sin 2 1
16 16 7
♥ Do cos
3 4
;2
2
3 7
♥ Vậy sin 2 2sin .cos
8
3 9
Ví dụ 3: Cho góc ; mà cos . Tính tan
2 41 4
Bài giải
3 92 40 40
♥ Do ; sin 1 cos2 1 tan
2 412 41 9
40
1
tan 1 9 31
♥ Do đó tan .
4 1 tan 40 49
1
9
1
Ví dụ 4: Cho là góc mà sin . Tính sin 4 2sin 2 cos
4
Bài giải
♥ Ta có: sin 4 2sin 2 cos cos2 1 .2sin 2 .cos 2 cos2 .4sin .cos2
2
2 1 1 225
8 1 sin 2
.sin 81 .
16 4 128
sin
Ví dụ 5: Cho là góc mà tan 2. Tính P 3
sin 3cos3

NGUYỄN VĂN LỰC  0933.168.309 SP Toán K35 - ĐH Cần Thơ


Lượng giác FB: http://www.facebook.com/VanLuc168
Bài giải
♥ Vì tan 2 nên sin 0, do đó:
1
sin sin2 1 cot 2 tan 2 1 .tan 22 1 .2 10
P
sin 3
3cos3 1 3cot 3 1 3cot 3 tan 3
3 2 3
3 11

BÀI TẬP TỰ LUYỆN

4 cot   tan 
Câu 1. Biết cos   và 00    900 . Tính giá trị của biểu thức A  .
5 cot   tan 

1
+ Biến đổi được A 
2 cos 2   1
4 25
+ Thay cos   , ta được A 
5 7
Lưu ý. HS có thể tính sin  , suy ra tan  ,cot  , thay vào A.

3 1 sin 
Câu 2. Cho góc  thõa mãn :     và cos =- . Tính P 
2 3 sin   3cos 3 
3

Ta có
1 8
sin 2   1  cos 2   1  
9 9
3
Vì     nên sin  <0
2
2 2
Do đó sin   
3
2 2

sin  3 18 2
Vậy P  = 
sin   3cos 3 
3
 2 2
3
 1
3
16 2  3
   3.   
 3   3


Câu 3. Cho cos   ,      0  .
4
5  2 
 
Tính giá trị biểu thức A  sin     cos    
4 4    

2
4 9
sin   cos   1  sin   1  cos   1    
2 2 2 2

5 25
3
 sin   
5
NGUYỄN VĂN LỰC  0933.168.309 SP Toán K35 - ĐH Cần Thơ
Lượng giác FB: http://www.facebook.com/VanLuc168
   1  
. A  sin     cos      sin 2  sin    
3
Vì     0 nên sin   
2 5  4   4 2   2 
1 49
  2sin  cos   1  
2 50
12
Câu 4. Cho góc thỏa mãn và sin   . Tính A cos
2 13 4

  2
Ta có A  cos     
4 2
cos  sin  

144 25 5 5 
cos2  1  sin2   1    cos    cos   (do     )
169 169 13 13 2
12 5 7 2
Thay sin   , cos   vào A ta được A 
13 13 26

4 cos 2
Câu 5. Cho góc thỏa mãn và sin . Tính A
2 5 1 cos

cos 2 1 2 sin2
Ta có A
1 cos 1 cos
16 9 3 3 
cos2  1  sin2   1    cos    cos   (do     )
25 25 5 5 2
4 3 7
Thay sin   , cos   vào A ta được A  
5 5 40

. Tính sin  α   .
3π 2π
Câu 6. Cho tan α  2 và π  α 
2  3 

Ta có
1 1 1 5
Cos 2 α     cosα  
1  tan α 1  4 5
2
5

3π 5
Do π  α   cosα  0 nên cosα  
2 5
 5 2 5
sin α  cosα.tan α  .2 
5 5

Vậy
 2π  2π 2π
sin  α    sin α.cos  cosα.sin
 3  3 3
2 5 1  5 3 2 5  15
 .  . 
5 2 5 2 10

NGUYỄN VĂN LỰC  0933.168.309 SP Toán K35 - ĐH Cần Thơ


Lượng giác FB: http://www.facebook.com/VanLuc168

Câu 7. Cho    

. Tính giá trị P 
cos  cos    sin   sin  2
2

6 sin   cos  2  sin   cos 2

2  2cos  cos   sin  sin   2  2 cos   


P 
2  2sin  cos   sin  cos   2  2 sin    

2  2 cos
P 6  2 3

2  2 sin
6

  
Câu 8. Cho     0 và cos   . Tính giá trị: P  cos      sin     .
3
2 5  3  6

 4
Vì     0 nên sin    1  cos 2    . Suy ra
2 5
   
P  cos  cos  sin  .sin  sin  .cos  cos  .sin
3 3 6 6
3 1 4 3 4 3 3 1 3
P .  .  .  .  .
5 2 5 2 5 2 5 2 5

 7
và sin(   )   . Tính tan     .
1
Câu 9. Cho góc  thỏa mãn  
2 3  2 

1 1
Ta có: sin(   )    s inx 
3 3
 7      
tan      tan  3      tan      cot 
 2   2  2 
 1 1
Vì      cot   0 . Do đó 1  cot 2    cot     1  2 2
2 sin 
2
sin 2 
7
Vậy tan      2 2 .
 2 

1 
Câu 10. Cho sin   . Tính giá trị biểu thức P  2 (1  cot  ).cos(   ) .
2 4

sin   cos 1  2 sin 2 


P (cos  sin  ) 
sin  sin 
1
thay sin   vào ta tính được P =1
2

sin 4 a  cos 4 a
Câu 11. Cho cot a  2 . Tính giá trị của biểu thức P  .
sin 2 a  cos 2 a

sin 4 a  cos 4 a sin 4 a  cos 4 a sin 4 a  cos 4 a


P   .
sin 2 a  cos 2 a  sin 2 a  cos 2 a  sin 2 a  cos 2 a  sin 4 a  cos 4 a

NGUYỄN VĂN LỰC  0933.168.309 SP Toán K35 - ĐH Cần Thơ


Lượng giác FB: http://www.facebook.com/VanLuc168
1  cot a 1  2
4 4
17
Chia tử và mẫu cho sin 4 a , ta được P   
1  cot a 1  2
4 4
15

Câu 12. Cho sin   2 cos   1 . Tính giá trị biểu thức P  2sin 2  2 cos 2  sin 2  .

P  4sin  cos   4 cos 2   sin 2   2


P  4sin  cos   4 cos 2   sin 2   2   2 cos   sin    2  12  2  1
2

3 
Câu 13. Cho cos   . Tính giá trị của biểu thức P  cos 2  cos 2
5 2

1  cos 
Ta có: P    2 cos 2   1
2

1 3  9  27
 1     2.  1 
2  5   25  25

Câu 14. Cho góc lượng giác  , biết tan   2 .


cos2 -3
Tính giá trị biểu thức P  .
sin2 

cos2 -3 2cos2  4
P  
sin2  1  cos2
1 1 1 9
1  tan2    cos2   . Suy ra P  
cos 
2
1  tan  5
2
2

3
Câu 15. Cho góc  thỏa mãn:     và tan   2 .
2

Tính giá trị A  sin 2  cos(  ) .
2

3 sin   0 1 1 2
Vì     nên  . Do đó: cos     sin   cos .tan   
2 cos  0 1  tan 2  5 5

42 5
Ta có: A  2sin  .cos  sin  
5

1 
Câu 16. Cho tan    với     0.
2 2
Tính giá trị của biểu thức: A  5 cos   5 sin 2.


Do     0  sin   0, cos   0.
2
NGUYỄN VĂN LỰC  0933.168.309 SP Toán K35 - ĐH Cần Thơ
Lượng giác FB: http://www.facebook.com/VanLuc168
1 1 1 2
Ta có: 1  tan2    1   cos  
cos 
2
4 cos 2
5
1
 sin   tan .cos   
5
2 1 2
Do đó: A  5 cos   10 sin  cos   5   10    2  4  6.
5 5 5

Câu 17. Cho tan   1 (  (0;  )) .


2 2
 
2sin  3cos
Tính giá trị biểu thức P  2 2 1 .
 
sin  2cos 5
2 2


 2 tan

1
tan   (   (0; )) nên 2  1  tan 2   4 tan   1  0
2 2  2 2 2
1  tan 2
2
  
Suy ra tan  2  5 hoặc tan  2  5 (l ) . Do tan  0 .
2 2 2

2 tan 3
2 1 2 5 1 1
Thay vào ta có P     2
 5 5 5
tan 2
2

NGUYỄN VĂN LỰC  0933.168.309 SP Toán K35 - ĐH Cần Thơ


Lượng giác FB: http://www.facebook.com/VanLuc168

III. PHƯƠNG TRÌNH LƯỢNG GIÁC


Chuyên đề: Lượng giác

Các bước giải một phương trình lượng giác

Bước 1: Tìm điều kiện (nếu có) của ẩn số để hai vế của pt có nghĩa
Bước 2: Sử dụng các phép biến đổi tương đương để biến đổi pt đến một pt đã biết
cách giải
Bước 3: Giải pt và chọn nghiệm phù hợp ( nếu có)
Bước 4: Kết luận
1. Định lý cơ bản: ( Quan trọng )

 u = v+k2
sinu = sinv  
 u =  -v+k2
 u = v+k2
cosu = cosv    u =  v + k2
 u = -v+k2

tanu = tanv  u = v+k (u;v   k )
2
cotu = cotv  u = v+k (u;v  k )

( u; v là các biểu thức chứa ẩn và k  Z )

2. Các phương pháp giải phương trình lượng giác thường sử dụng :
a. Phương pháp 1: Biến đổi pt đã cho về một trong các dạng pt lượng giác đã
biết cách giải.

b. Phương pháp 2: Biến đổi pt đã cho về dạng tích số.


Cơ sở của phương pháp là dựa vào các định lý sau đây:
 A=0
 A=0
A.B  0   hoặc A.B.C  0   B=0
 B=0 C=0

c. Phương pháp 3: Biến đổi pt về dạng có thể đặt ẩn số phụ.

NGUYỄN VĂN LỰC  0933.168.309 SP Toán K35 - ĐH Cần Thơ


Lượng giác FB: http://www.facebook.com/VanLuc168
Một số dấu hiệu nhận biết :
 Phương trình chứa cùng một một hàm số lượng giác ( cùng cung khác lũy
thừa).
 Phương trình có chứa (cos x  sin x ) vaø sinx.cosx .

3. Các phương trình lượng giác thường gặp:


a. Dạng 1: sinx = m ; cosx = m ; tanx = m ; cotx = m ( m  R )
(Phương trình lượng giác cơ bản)
* Gpt : sinx = m (1)
 Nếu m  1 thì pt(1) vô nghiệm
 Nếu m  1 thì ta đặt m = sin  và ta có
 x =  +k2
(1)  sinx = sin  
 x = ( - )+k2
* Gpt : cosx = m (2)
 Nếu m  1 thì pt(2) vô nghiệm
 Nếu m  1 thì ta đặt m = cos  và ta có
 x =  +k2
(2)  cosx = cos  
 x =   +k2
* Gpt: tanx = m (3) ( pt luôn có nghiệm m  R )
 Đặt m = tan  thì
(3)  tanx = tan  x =  +k

* Gpt: cotx = m (4) ( pt luôn có nghiệm m  R )

 Đặt m = cot  thì


(4)  cotx = cot  x =  +k

Các trường hợp đặc biệt: y



sin x  1  x =   k 2 B
2 
sinx = 0  x = k

sin x  1  x =  k 2 C O A x
2
cosx  1  x =   k 2
 D 
cosx = 0  x= + k
2
cos x  1  x = k 2

NGUYỄN VĂN LỰC  0933.168.309 SP Toán K35 - ĐH Cần Thơ


Lượng giác FB: http://www.facebook.com/VanLuc168
b. Dạng 2:
a sin 2 x  b sin x  c  0
a cos2 x  b cos x  c  0
( a  0)
a tan 2 x  b tan x  c  0
a cot 2 x  b cot x  c  0

(Phương trình bậc hai đối với một hàm số lượng giác)

Cách giải:

Đặt ẩn phụ : t = sinx ( t = cosx; t = tanx; t = cotx)


Ta được phương trình : at 2  bt  c  0 (1)
Giải phương trình (1) tìm t, rồi suy ra x
Chú ý : Phải đặt điều kiện thích hợp cho ẩn phụ (nếu có)

c. Dạng 3:

a cos x  b sin x  c (1) ( a;b  0)

(Phương trình bậc nhất đối với sinx và cosx)


Cách giải:

 Chia hai vế của phương trình cho a2  b2 thì pt


a b c
(1)  cos x  sin x  (2)
a2  b2 a2  b2 a2  b2

a b
 Đặt  cos vaø  sin  với    0;2  thì :
2 2 2 2
a b a b
c
(2)  cosx.cos + sinx.sin =
a2  b 2
c
 cos(x- ) = (3)
2 2
a b
Pt (3) có dạng 1. Giải pt (3) tìm x.

Chú ý :
Pt acosx + bsinx = c coù nghieäm  a2  b2  c2

d. Dạng 4:

a sin2 x  b sin x.cos x  c cos2 x  0 (a;c  0) (1)


NGUYỄN VĂN LỰC  0933.168.309 SP Toán K35 - ĐH Cần Thơ
Lượng giác FB: http://www.facebook.com/VanLuc168

(Phương trình đẳng cấp bậc hai đối với sinx và cosx)
Cách giải 1:

1  cos 2 x 1  cos 2 x
Áp dụng công thức hạ bậc : sin2 x  vaø cos2 x  và công thức
2 2
1
nhân đôi : sin x.cos x  sin 2 x thay vào (1) ta sẽ biến đổi pt (1) về dạng 3
2
Cách giải 2: ( Quy về pt theo tang hoặc cotang )

Chia hai vế của pt (1) cho cos2 x ta được pt:


a tan2 x  b tan x  c  0
Đây là pt dạng 2 đã biết cách giải.


Chú ý: Trước khi chia phải kiểm tra xem x   k có phải l nghiệm của (1) không?
2

e. Dạng 5:

a(cos x  sin x )  b sin x.cos x  c  0 (1)

(Phương trình đối xứng đối với sinx và cosx)


Cách giải :


 Đặt t  cos x  sin x  2 cos( x  ) vôùi - 2  t  2
4
t2  1
Do (cos x  sin x )2  1  2sin x.cos x  sinx.cosx=
2
 Thay vào (1) ta được phương trình :
t2  1
at  b c  0 (2)
2

 Giải (2) tìm t . Chọn t thỏa điều kiện rồi giải pt: 2 cos( x  )  t tìm x.
4

Chú ý : Ta giải tương tự cho pt có dạng :

a(cos x  sin x )  b sin x.cos x  c  0

NGUYỄN VĂN LỰC  0933.168.309 SP Toán K35 - ĐH Cần Thơ


Lượng giác FB: http://www.facebook.com/VanLuc168
CÁC VÍ DỤ
Ví dụ 1: Giải phương trình sin 5 x 2 cos 2 x 1 (1)
Bài giải

♥ Ta có: 1 cos 5 x cos 2 x 0


2

cos 5 x cos 2 x (Biến đổi về pt cơ bản)


2

5x 2x k2
2
k
5x 2x k2
2

k2
x
6 3
k
k2
x
14 7
k2 k2
♥ Vậy nghiệm của phương trình đã cho là x , x k 
6 3 14 7

Ví dụ 2: Giải phương trình sin 3x 3 cos 3 x 2sin 2 x (1)


Bài giải
1 3
♥ Ta có: 1 sin 3x cos 3x sin 2 x
2 2

sin 3x sin 2 x (Biến đổi về pt cơ bản)


3

3x 2x k2
3
3x 2x k2
3

x k2
3
k
4 k2
x
15 5
4 k2
♥ Vậy nghiệm của phương trình đã cho là x k2 , x + k .
3 15 5

NGUYỄN VĂN LỰC  0933.168.309 SP Toán K35 - ĐH Cần Thơ


Lượng giác FB: http://www.facebook.com/VanLuc168
5x 3x
Ví dụ 3: Giải phương trình 4 cos cos 2 8sin x 1 cos x 5 (1)
2 2

Bài giải
♥ Ta có: 1 2 cos 4 x cos x 8sin 2 x 2 cos x 5

2 cos 4 x 8sin 2 x 5 0

4sin 2 2 x 8sin 2 x 3 0 (Biến đổi về pt bậc hai theo sin2x)


3
sin 2 x : phương trình vô nghiệm
2

2x k2 x k
1 6 12
sin 2 x sin 2 x sin k
2 6 5 5
2x k2 x k
6 12
5
♥ Vậy nghiệm của phương trình đã cho là x k , x +k k .
12 12

Ví dụ 4: Giải phương trình 2 cos 5 x.cos 3 x sin x cos 8 x (1)


Bài giải

♥ Ta có: 1 cos8 x cos 2 x sin x cos8 x

2sin 2 x sin x 1 0 0

sin x 1
1 (Biến đổi về pt bậc hai theo sinx)
sin x
2

sin x 1 x k2
2

x k2
1 6
sin x sin x sin k
2 6 7
x k2
6

♥ Vậy nghiệm của phương trình đã cho là


7
x k2 ; x k2 , x +k 2 k .
2 6 6

Ví dụ 5: Giải phương trình 2 sin x 2cos x 2 sin 2 x (1)

NGUYỄN VĂN LỰC  0933.168.309 SP Toán K35 - ĐH Cần Thơ


Lượng giác FB: http://www.facebook.com/VanLuc168
Bài giải
♥ Ta có: 1 2 sin x 2 2 cos x 2sin x cos x 2 0

sin x 2 cos x 2 2 2 cos x 2 0

sin x 2 2 cos x 2 0 (Biến đổi về pt tích số)

sin x 2 0 sin x 2 : phương trình vô nghiệm

2 3 3
2cos x 2 0 cos x cos x cos x k2 k
2 4 4
3
♥ Vậy nghiệm của phương trình đã cho là x k2 k .
4

Ví dụ 6: Giải phương trình sin x 4 cos x 2 sin 2 x (1)


Bài giải
♥ Ta có: 1 sin x 4 cos x 2sin x cos x 2 0

sin x 2 2 cos x 1 0 (Biến đổi về pt tích số)


sin x 2 0 sin x 2 : phương trình vô nghiệm
1
2 cos x 1 0 cos x cos x cos x k2 k
2 3 3

♥ Vậy nghiệm của phương trình đã cho là x k2 k .


3

Ví dụ 7: Giải phương trình cos x sin 2 x 0 (1)


2

Bài giải
♥ Ta có: 1 sin x 2sin x cos x 0

sin x 1 2 cos x 0 (Biến đổi về pt tích số)


sin x 0 x k
1 2 2
1 2 cos x 0 cos x cos x cos x k2
2 3 3
2
♥ Vậy nghiệm của phương trình đã cho là x k , x k2 k .
3

Ví dụ 8: Giải phương trình sin 3x cos 2 x sin x 0 (1)


Bài giải
NGUYỄN VĂN LỰC  0933.168.309 SP Toán K35 - ĐH Cần Thơ
Lượng giác FB: http://www.facebook.com/VanLuc168
♥ Ta có: 1 2 cos 2 x sin x cos 2 x 0 0
cos 2 x 2sin x 1 0 (Biến đổi về pt tích số)
k
cos 2 x 0 2x k x k
2 4 2
x k2
1 6
2sin x 1 0 sin x sin x sin k
2 6 7
x k2
6
♥ Vậy nghiệm của phương trình đã cho là
k 7
x ,x k2 , x k2 k .
4 2 6 6

Ví dụ 9: Giải phương trình 2 cos 2 x sin x sin 3 x (1)


Bài giải
♥ Ta có: 1 2 cos 2 x sin x sin 3 x 0
2 cos 2 x 2 cos 2 x sin x 0
cos 2 x sin x 1 0 (Biến đổi về pt tích số)
k
cos 2 x 0 2x k x k
2 4 2
sin x 1 0 sin x 1 x +k 2 k
2
k
♥ Vậy nghiệm của phương trình đã cho là x , x +k 2 k .
4 2 2

Ví dụ 10: Giải phương trình 1 2sin x cos x 1 sin x cos x


2
(1)

Bài giải
♥ Ta có: 1 2 1 sin x sin 2 x 1 sin x 0
1 sin x 2sin 2 x 1 0 (Biến đổi về pt tích số)
sin x 1 x k2 k
2
2x k2 x k
1 6 12
2sin 2 x 1 0 sin 2 x sin 2 x sin k
2 6 5 5
2x k2 x k
6 12
♥ Vậy nghiệm của phương trình đã cho là
5
x k2 , x k , x +k k .
2 12 12

Ví dụ 11: Giải phương trình 1 tan x 2 2 sin x (1) (Phương trình lượng giác có
4

điều kiện)
NGUYỄN VĂN LỰC  0933.168.309 SP Toán K35 - ĐH Cần Thơ
Lượng giác FB: http://www.facebook.com/VanLuc168
Bài giải

♥ Điều kiện: cos x 0 x k


2
sin x
♥ Ta có: 1 1 2 sin x cos x
cos x

sin x cos x 2 cos x 1 0 (Biến đổi về pt tích số)

sin x cos x 0 tan x 1 x k k


4
1
2 cos x 1 0 cos x cos x cos x k2 k
2 3 3

Đối chiếu điều kiện: các nghiệm tìm được đều thỏa điều kiện.

♥ Vậy nghiệm của phương trình đã cho là x k , x k2 k .


4 3

NGUYỄN VĂN LỰC  0933.168.309 SP Toán K35 - ĐH Cần Thơ


Lượng giác FB: http://www.facebook.com/VanLuc168

BÀI TẬP TỰ LUYỆN

1. Phương trình lượng giác bậc nhất

Câu 1. Giải phương trình: cos 2 x  (1  2 cos x)(sin x  cos x)  0

cos 2 x  (1  2 cos x)(sin x  cos x)  0


sin x  cos x  0
 (sin x  cos x)(sin x  cos x  1)  0  
sin x  cos x  1
 
 x   k
  4
sin( x  4 )  0 

   x   k 2 ( k  )
  2  2
sin( x  4 )  2  x    k 2



Câu 2. Giải phương trình: sin 2 x  1  6sin x  cos 2 x .

sin 2 x  1  6sin x  cos 2 x


 (sin 2 x  6sin x)  (1  cos 2 x)  0
 2sin x  cos x  3  2sin 2 x  0
 2sin x  cos x  3  sin x   0
sin x  0

sin x  cos x  3(Vn)
 x  k . Vậy nghiệm của PT là x  k , k  Z

Câu 3. Giải phương trình: sin 4 x  2cos 2 x  4  sin x  cos x   1  cos 4 x .

sin 4 x  2 cos 2 x  4sin x  cos x   1  cos 4 x


 2 sin 2 x cos 2 x  2 cos 2 x  2 cos 2 2 x  4sin x  cos x   0
 cos 2 xsin 2 x  1  cos 2 x   2sin x  cos x   0
 cos 2 x2 sin x cos x  2 sin 2 x   2sin x  cos x   0
 sin x  cos x cos 2 x sin x  1  0

Với sin x  cos x  0  x    k , k  Z
4
Với cos 2 x sin x  1  0  1  2 sin 2 x sin x  1  0  sin x  1 2 sin 2 x  1  0

 sin x  1  x   2m , m  Z
2

NGUYỄN VĂN LỰC  0933.168.309 SP Toán K35 - ĐH Cần Thơ


Lượng giác FB: http://www.facebook.com/VanLuc168
Câu 4. Giải phương trình: cos2x  2sin x  1  2sin x cos 2x  0 .

 PT  cos2 x 1  2sin x   1  2sin x   0


  cos2 x  11  2sin x   0
+ Khi cos2x=1<=> x  k , k  Z
1  5
Khi s inx   x   k 2 hoặc x   k 2 , k  Z
2 6 6

Câu 5. Giải phương trình: sin 2 x  cos x  sin x  1 (x  R)

sin 2 x  cos x  sin x  1 (1)


(1)  (sin x  cos x)(1  sin x  cos x)  0
 
 x   k
 sin x  cos x  0 4
  (k  Z )
1  sin x  cos x  0  x  2k   x  3
 2k 
 2

Câu 6. Giải phương trình: s inx  cos x  cos2 x

Ta có: s inx  cos x  cos2 x  s inx  cos x  cos 2 x  sin 2 x


 (s inx  cos x) 1  (cos x  s inx)   0
 
s inx  cos x  0  2cos( x  4 )  0
 
cos x  s inx  1  2cos( x   )  1
 4

  
     x  4  2  k 
x
3
 k
2cos( x  )  0 cos( x  )  0  
    
4
  x    k 2   x  k 2
4 4
 
    2  4 4
2cos( x  )  1 cos( x  )    
 4     x    k 2
4 2  x     k 2  2
 4 4


Câu 7. Giải phương trình: lượng giác: 2 sin  2 x    3sin x  cos x  2 (x  ).
 4

PT (1)  sin 2 x  cos 2 x  3sin x  cos x  2


 2sin x cos x  3sin x  2cos2 x  cos x  3  0 .
  2cos x  3 sin x   cos x  1 2cos x  3  0
  sin x  cos x  1 2cos x  3  0
 3  
 cos x  (VN )   1  x    k 2
 2  sin  x      2 (k  )
  4 2 
sin x  cos x  1  x    k 2

Phương trình có các nghiê ̣m: x    k 2 , x    k 2 (k  ).
2
NGUYỄN VĂN LỰC  0933.168.309 SP Toán K35 - ĐH Cần Thơ
Lượng giác FB: http://www.facebook.com/VanLuc168

Câu 8. Giải phương trình: 3cos5x  2sin 3x.cos2 x  s inx  0

  
 x  k

PT  cos5x  sin 5x  sinx  sin   5x   sinx   18 3
3 1
2 2 3  x     k 
 6 2

Câu 9. Giải phương trình: 1  sin 2x  cos 2x

 x  k
sin x  0
1  sin 2x  cos 2 x  2sin x cos x   2sin x   
 x     k
2

 cos x   sin x
 4

Câu 10. Giải phương trình: cos2 x  3sin x  2  0

cos2 x  3sin x  2  0
 1  2sin 2 x  3sin x  2  0  2sin 2 x  3sin x  1  0
 
 x   k 2
2
 sin x  1 

 1   x   k 2 , k 
sin x  6
 2  5
x   k 2
 6

Câu 11. Giải phương trình: sin 2 x  cos2 x  2sin x  1 .

Biến đổi phương trình về dạng: 2s inx(cos x  1)  2sin x  0


2

s inx  0
s inx(sin x  cos x  1)  0  
sin x  cos x  1  0
Với s inx  0  x  k 2
 x  k 2
1
Với cos2x = 1  sin x  cos x  1  0  sin( x  )   , k Z
4 2  x    k 2
 2

Vậy phương trình có 2 họ nghiệm. x  k , x   k 2 , k  Z
2

Câu 12. Giải phương trình: cos 2 x  cos x  sin x  1  0

cos 2 x  0
cos 2 x  cos x  sin x  1  0   
sin  x     1
  4 2
NGUYỄN VĂN LỰC  0933.168.309 SP Toán K35 - ĐH Cần Thơ
Lượng giác FB: http://www.facebook.com/VanLuc168
k 
+) Với cos 2 x  0  x    k  
4 2
 x  k 2
  1
+) Với sin  x     (k  )
 4 2  x    k 2
 2
Câu 13. Giải phương trình: 2(cos x  sin 2 x)  1  4sin x(1  cos 2 x)

Phương trình đã cho tương đương với: 2 cos x  2sin 2 x  1  4sin 2 x.cos x
 (1  2 cos x)(2sin 2 x  1)  0
 
 1  x   3  k 2
cos x  2 
 (k Z )
   x   k
sin 2 x  1  12

 x  5  k
 2
 12
  5
Vậy pt có nghiệm là: x    k 2 ; x   k ; x  k (k Z )
3 12 12

Câu 14. Giải phương trình: sin 2x   sin x  cos x  1 2sin x  cos x  3  0

PT   sin x  cos x   1   sin x  cosx  1 2sin x  cos x  3


2

  sin x  cos x   1  sin x  cos x   1   sin x  cosx  1 2sin x  cos x  3 
 x  k2
sin x  cos x  1
 
 sin x  2 cos x  4(VN)  x    k2
 2

Câu 15. Giải phương trình: 3sin x  cos x  2  cos2 x  sin 2 x  0

sin x  cos x  1  2sin x  2sin 2 x  2sin x cos x  0


 (1+2sinx)(sinx - cosx +1) = 0

   2
s inx  cos x  1 sin(x  ) 
  4 2
s inx  1  1
 2 s inx  2

 7
 x  6  k 2

 x    k 2
 6 k

 x  3  k 2
 2
 x  k 2

NGUYỄN VĂN LỰC  0933.168.309 SP Toán K35 - ĐH Cần Thơ


Lượng giác FB: http://www.facebook.com/VanLuc168

Câu 16. Giải phương trình: cos 2 x  5  2(2  cos x)(sin x  cos x)

(cos x – sin x )2  4(cos x – sin x ) – 5  0



 x   k 2  x    k 2
2

Câu 17. Giải phương trình: : 3  cos 2 x - sin x   cos x  2sin x  1  0 .

 sin 2 x  3 cos 2 x  3 sin x  cos x


1 3 3 1
 sin 2 x  cos 2 x  sin x  cos x
2 2 2 2
   
 sin 2 x cos  cos 2 x sin  sin x cos  cos x sin
3 3 6 6
 
 sin(2 x  )  sin( x  )
3 6

    
 2 x  3  x  6  k 2  x   2  k 2
 (k  )   (k  )
 2 x      ( x   )  k 2  x  5  k 2
 3 6  18 3

Câu 18. Giải phương trình: sin 2 x  4  8cosx  s inx

s inx  4 (vn)
Biến đổi phương trình về dạng: (s inx-4)(2 cos x  1)  0  
cos x  1
 2

1 
Với cosx   x    k 2
2 3

Kl: phương trình có 2 họ nghiệm: x    k 2 ,
3

Câu 19. Giải phương trình: 2sin x  1  cos x  sin 2 x.

 2sin x  1  cos x  2sin x.cos x.


cos x  1
 2sin x  1  cos x(1  2sin x)  
sin x  1
 2
-Với cos x  1  x    k 2 , k  .
 
 x   k 2
1 6
-Với sin x    ,k  .
2  5
x  k 2
 6

NGUYỄN VĂN LỰC  0933.168.309 SP Toán K35 - ĐH Cần Thơ


Lượng giác FB: http://www.facebook.com/VanLuc168
Câu 20. Giải phương trình: cos x  sin 4x  cos3x  0 .

cos x  sin 4x  cos3x  0  2sin 2x.sin x  2sin 2x.cos 2x  0


 2sin 2x(s inx  cos2x)  0  sin 2x( 2sin 2 x  sin x  1)  0
 kπ
x  2
 
sin 2x  0  x  π  k2π
  2
 s inx  1  
 1  x   π  k2π
s inx   6
 2  7π
x   k2π
 6

Câu 21. Giải phương trình: sin 3x  cos 2 x  1  2sin x cos 2 x


sin 3x  cos 2 x  1  2sin x cos 2 x  sin 3 x  cos 2 x  1  sin x  sin 3 x
 cos 2 x  1  sin x

 x  k
sin x  0 
 
 1  2sin x  1  sin x 
2
1   x   k 2
sin x   6
 2  5
x   k 2
 6

Câu 22. Giải phương trình sau: 1 3cos x  cos 2x  2cos3x  4sin x.sin 2x

Giải phương trình: 1 3cos x  cos 2x  2cos3x  4sin x.sin 2x (1)


(1)  1  3cos x  cos 2 x  2 cos  2 x  x   4sin x.sin 2 x

 1  3cos x  cos 2 x  2  cos x.cos 2 x  sin x.sin 2 x   4sin x.sin 2 x

 1  3cos x  cos 2 x  2  cos x.cos 2 x  sin x.sin 2 x   0

 1  3cos x  cos 2 x  2 cos x  0  1  cos x  cos 2 x  0

 
 cos x  0  x  2  k
 2 cos x  cos x  0    ;k  .
2
 cos x   1  x   2  k 2
 2  3

Câu 23. Giải phương trình: sin 2x  2 sinx  0.


 x  k
s inx  0 
Pt     x    k 2
cosx  2
 4
 
2 
 x    k 2
 4

NGUYỄN VĂN LỰC  0933.168.309 SP Toán K35 - ĐH Cần Thơ


Lượng giác FB: http://www.facebook.com/VanLuc168
Câu 24. Giải phương trình: 3 sin 2 x  cos 2 x  4sin x  1.

3 sin 2 x  cos 2 x  4sin x  1  2 3 sin x cos x  1  cos 2 x  4sin x  0


 2 3 sin x cos x  2sin 2 x  4sin x  0  2sin x  
3 cos x  sin x  2  0

sin x  0  x  k
sin x  0 
   ,k  .
  sin  x     1  x    k 2
 3 cos x sin x 2
  3  6

Câu 25. Giải phương trình: cos 2 x(4sinx  1)  3 sin 2 x  1

pt  4cos 2 x.sin x  cos 2 x  1  2 3 sin x cos x  0


 4 cos 2 x.sin x  2sin 2 x  2 3 sin x cos x  0
 2sin x(2cos 2 x  sin x  3 cos x)  0
sin x  0 sin x  0
  
cos x.cos   sin x.sin   cos 2 x cos( x   )  cos 2 x
 6 6  6

 x  k


  x    k 2 , (k  )
 6
  2
x   k
 18 3

Câu 26. Giải phương trình: cos 2 x  3sin x  2  0 .

- Ta có phương trình cos 2 x  3sin x  2  0  2sin 2 x  3sin x  1  0

 
 x   2  k 2
sin x  1 

  1   x    k 2 , k  .
sin x    6
 2 
 x  7  k 2
 6

- KL: Phương trình có ba họ nghiệm…

Câu 27. Giải phương trình: sin 3 x  sinx  2 3 cos x.cos 2x .


y

x
-8 -6 -4 -2 2 4 6 8

-5

NGUYỄN VĂN LỰC  0933.168.309 SP Toán K35 - ĐH Cần Thơ


Lượng giác FB: http://www.facebook.com/VanLuc168

cos x  0  x   k
2
 
sin 2x  3 cos 2x  0  sin  2x    0
 3
  
Pt có nghiệm x   k , x   k
2 6 2

Câu 28. Giải phương trình: 2 3 sin x  cos x  sin 2x  3 .

2 3 sin x  cos x  sin 2x  3  2 3 sin x  cos x  2sin x cos x  3  0


y

x
-8 -6 -4 -2 2 4 6 8

-5

* cos x  3  0 : Vô nghiệm.
 
 x  6  k2
* 2 sin x  1  0   .
 x  5  k2
 6
 5
Vậy nghiệm của phương trình là x   k2 ; , x   k2
6 6

Câu 29. Giải phương trình: sin 2x  1  4 cos x  cos 2x.

PT  sin 2x  1  cos 2x  4 cos x  0


 2 sin x cos x  2 cos2 x  4 cos x  0
 cos x(sin x  cos x  2)  0
 cos x  0 
  x   k
sin x  cos x  2 (VN do 1  1  2 )
2 2 2
2

Vậy nghiệm của phương trình đã cho là: x   k.
2

Câu 30. 2 s inx cos x + 6 s inx  cosx  3  0 ;

TXĐ D =
Phương trình đã cho  (2s inx  1)(cos x + 3)  0
  
sin x 
1
 x   k 2
 2 
6 , với k, l là số nguyên. Kết luận.
  5
cosx = 3(v« nghiÖm) x  l 2
 6

Câu 31. Giải phương trình: sinx+sin2x+sin3x+sin4x+sin5x+sin6x=0 (1)

1   sin 6 x  sin x    sin 5x  sin 2 x   sin 4 x  sin 3x   0

NGUYỄN VĂN LỰC  0933.168.309 SP Toán K35 - ĐH Cần Thơ


Lượng giác FB: http://www.facebook.com/VanLuc168
7 x  5x x 3x  7x 3x
 2sin   cos  cos   cos   0  4sin cos  2cos x  1  0
2  2 2 2 2 2
 7x  k 2
sin 2  0 x  7
 
3x  k 2
 cos  0   x  
 ;k  Z
 2  3 3
 2cos x  1  0  2
 x    k 2
  3


Câu 32. Giải phương trình lượng giác: 2 cos(2x  )  4s inx.sin3x - 1  0
3


Giải phương trình : 2 cos(2x  )  4s inxsin3x  1  0 (1)
3
 
 2(cos2xcos  sin 2x sin )  4sin x sin 3x  1  0
3 3
 cos2x  3 s in2x+4sin x sin 3x  1  0
 1  2s in 2 x-2 3 sin x cos x  4sin x sin 3x  1  0
 s inx(2s in3x-sin x- 3 cos x)  0
sinx  0

sinx  3 cos x  2sin 3x
*s inx  0  x  k (k  z)
1 3
*s inx  3 cos x  2sin 3x  s inx  cos x  sin 3x
2 2
   
 3x  x  3  k2  x  6  k
 sin(x  )  sin 3x    (k  z)
3 3x    x    k2 x    k 
 3  6 2
 
Vậy phương trình đã cho có nghiệm x  k ; x   k (k  z)
6 2

17 x 
Câu 33. Giải phương trình sin(2x  )  16  2 3.sin x cos x  20sin 2 (  )
2 2 12

*Biến đổi phương trình đã cho tương đương với



c os2x  3 sin 2x  10cos(x  )  6  0
6
 
 c os(2x  )  5c os(x  )  3  0
3 6
 
 2c os 2 (x  )  5cos(x  )  2  0
6 6
 1 
Giải được cos(x  )   và cos(x  )  2 (loại)
6 2 6
 1  5
*Giải c os(x  )   được nghiệm x   k 2 và x    k 2
6 2 2 6

NGUYỄN VĂN LỰC  0933.168.309 SP Toán K35 - ĐH Cần Thơ


Lượng giác FB: http://www.facebook.com/VanLuc168

Câu 34. Giải phương trình sau: cos    x   sin  2 x     1 .


4   4 2

Pt đã cho cos    x   sin  2 x     1  2 cos    x   2 sin  2 x     1


4   4 2 4   4
 cos x  sin x  sin 2 x  cos2 x  1
 sin x(1  2 cos x)  cos x(1  2 cos x)  0.
 (sin x  cos x)(1  2 cos x)  0.
 
cos x  sin x  0  tan x  1  x    k
  
4
(k  )
1  2 cos x  0 cos x  1 
 x    k 2
 2
 3
 
Vậy phương trình đã cho có 3 họ nghiệm: x    k , x    k 2 , (k  ) .
4 3

2. Phương trình bậc hai đối với sin, cos

Câu 35. Giải phương trình: (sinx  cosx)2  1  cosx .

Ta có: (s inx  cosx)2  1  cosx  1  2 sin xcosx  1  cosx


 cosx(2 sin x-1)  0
 
cosx  0  x   k
2
 

s inx= 1   x=  k2 (k  Z).
 2  6
 5
 x  6  k2

Câu 36. Giải phương trình: 2cos 2 2 x  3cos 3 x  4 cos 2 x  3cos x  0

Khi đó , phương trình tương đương với :


 cos2 x  cos 2 x  3cos x  2   0
   
 x   k  x   k
  4 4
cos2 x  0  2 x   k 2  
  2  cos x  1   x  k (k  )
cos2 x  3cos x  2  0  2cos 2 x  3cos x  1  0  
 1 2
cos x    x    k 2
 2  3
 2
Vậy nghiệm phương trình là: x   k ; x    k 2
4 3

Câu 37. Giải phương trình 3  2 cos2 x  cos x  2   s inx  3  2 cos x   0.


Phương trình đã cho tương đương với 3 3s inx  cos x  2sin x 3s inx  cos x  0   
  3  2sin x  
3s inx  cos x  0
NGUYỄN VĂN LỰC  0933.168.309 SP Toán K35 - ĐH Cần Thơ
Lượng giác FB: http://www.facebook.com/VanLuc168
 
 x   k 2
 3 3
s inx  
2
 x 
2
  k 2
    3
 cos  x  3   0  5
   x   k , k  .
 6

Câu 38. Giải phương trình: sin 2 x  2 cos 2 x  3sin x  cos x .

Phương trình đã cho tương đương 2sin 2 x  3sin x  2  2sin x cos x  cos x  0
  2sin x  1 sin x  cos x  2   0
 sin x  cos x  2  0 : Phương trình vô nghiệm
 
 x   6  k 2
 2sin x  1  0   (k  )
 x  7  k 2
 6
 7
Vậy phương trình đã cho có nghiệm: x    k 2 , x   k 2 (k  ).
6 6

Câu 39. Giải phương trình: 2sin2x + 3cosx – 2 = 0

2sin2x + 3cosx – 2 = 0 (1)


 Pt (1)  2(1 – cos2x) + 3cosx – 2 = 0  2cos2x – 3cosx = 0 (*)
 đặt t = cosx (t ≤ 1)
t = 0
 Pt (*) trở thành : 2t2 – 3t = 0   3 .So sánh điều kiện t = 0 thỏa mãn
t =
 2
 Với t = 0  cosx = 0  x = k2 (k  Z)
Vậy nghiệm của phương trình là : x = k2 (k  Z)

Câu 40. Giải phương trình lượng giác: cos2 x  3 cos x  3sin x  3sin 2 x  0

2 2
 3  3 
cos2 x  3 cos x  3sin x  3sin 2 x  0   cos x     3 sin x 
 2   2 
 3 3
cos x    3 sin x  3 sin x  cos x  0 (1)
2 2
 
 3 3  3 sin x  cos x  3 (2)
cos x    3 sin x
 2 2

(1)  tan x   1  x    k
3 6

NGUYỄN VĂN LỰC  0933.168.309 SP Toán K35 - ĐH Cần Thơ


Lượng giác FB: http://www.facebook.com/VanLuc168
 
 x  2  k2
(2)  sin  x     sin  
 6 3  x  5  k2
 6

 
Vậy phương trình có hai họ nghiệm là x    k hay x   k2 .
6 2

Câu 41. Giải phương trình 2 3 cos 2 x  6sin x.cos x  3  3

Tập xác định .


*  3 1  cos 2 x   3sin 2 x  3  3  3 cos 2 x  3sin 2 x  3
1 3 3   3
 cos 2 x  sin 2 x   sin  2 x   
2 2 2  6 2
    
 2 x  6  3  k 2  x  12  k
  k .
 2 x    2  k 2  x    k
 6 3  4

Câu 42. Giải phương trình: 2sin 2 x  3 sin 2 x  2  0 .

3 1 1
2sin 2 x  3 sin 2 x  2  0  3 sin 2 x  cos 2 x  1  sin 2 x  cos 2 x 
2 2 2

 
 x   k
  
 sin  2 x    sin  
6
k  
 6 6  x    k
 2

Câu 43. Giải phương trình: 2 cos 2 x  sin x  1  0 .

Ta có: 2 cos 2 x  sin x  1  0  2sin 2 x  sin x  3  0  (sin x  1)(2sin x+3)=0


 sin x  1 (do 2sin x  3  0 x  )

 s inx  1  x   k 2  k  
2

Vậy nghiệm của phương trình đã cho là x   k 2  k  
2

Câu 44. Giải phương trình trình sau trên tập số thực:
3
sin2x - 2 3 cos2x = 0 với x  (o; )
2

sin2x - 2 3 cos2x = 0 <=> cosx(sinx- 3cosx)=0

NGUYỄN VĂN LỰC  0933.168.309 SP Toán K35 - ĐH Cần Thơ


Lượng giác FB: http://www.facebook.com/VanLuc168
 
cos x  0  x  2  k
<=>  
 tan x  3  x    k
 3
  4
Trên (0,3π/2) ta có tập nghiệm là:  , ,  .
3 2 3 


Câu 45. Giải phương trình : 2cos 2   2 x   3 cos 4 x  4cos 2 x  1 (1)
4 

1  1  cos  
 4 x   3 cos 4 x  4 cos 2 x  1  sin 4 x  3 cos 4 x  2  2 cos 2 x  1
2 
1 3  
 sin 4 x  cos 4 x  cos 2 x  cos  4 x    cos 2 x
2 2  6
  k
x  k  x   ,k 
12 36 3

3. Phương trình chứa mẫu

1  cos x(2cos x  1)  2 s inx


Câu 46. Giải phương trình: 1
1  cos x

Điều kiện: cos x  1  x  k 2 , k 


Với điều kiện trên phương trình đã cho tương đương:
1  cos x(2cos x  1)  2 sinx  1  cos x  2sin 2 x  2 sin x  2  0
2  5
sin x    x    k , k  ; x   k , k  (thỏa điều kiện)
2 4 4

3(2.cos 2 x  cos x  2)  (3  2cos x).sin x


Câu 47. Giải phương trình: 0
2cos x  1

ĐK:
Pt đã cho tương đương với pt:

Vậy pt có 2 họ nghiệm hoặc

NGUYỄN VĂN LỰC  0933.168.309 SP Toán K35 - ĐH Cần Thơ


Lượng giác FB: http://www.facebook.com/VanLuc168
 
2 sin  x  
Câu 48. Giải phương trình:  4
 tan 2 x  cos 2 x  0
sin x  cos x

ĐK : cos2x  0.
Biến đổi phương trình    sin x  cos x   sin x2 x  cos 2 x.cos 2 x  0
2

pt  cos 2 x.cos 2 x  1  0

pt  cos 2 2 x  cos 2 x  2  0  cos 2 x  1 (thỏa mãn ĐK), cos2x = -2 (vn)


 k
Vậy cos2x = 1  x   , k  Z
4 2
 k
Vậy phương trình có 1 họ nghiệm. x   , k Z
4 2

2cos 4 x
Câu 49. Giải phương trình: cot x  tan x  (1)
sin 2 x

sin x  0
 k
ĐK: cos x  0  sin 2 x  0  x  , k  Z
sin 2 x  0 2

 x  l
2cos4 x 2 cos 2 x 2cos4 x
1  cot x  tan x     cos4 x  cos2 x   ,l Z
sin 2 x sin 2 x sin 2 x  x  l
 3

Kiểm tra điều kiện ta được x    l , l  Z
3

Câu 50. Giải phương trình:


4cos3 x  2cos 2 x  2sin x  1  sin 2 x  2  sin x  cos x 
 0 (1)
2sin 2 x  1

 k
ĐK: 2sin 2 x  1  0  cos2 x  0  x   ,k Z
4 2
1  4cos x  sin x  cos x   2 cos x  sin x  cos x   2  sin x  cos x   0
2

 
 x   4  m

 2  sin x  cos x  cos x  1 2 cos x  1  0   x  m2 ,mZ
 2
x    m 2
 3
m2
Kiểm tra điều kiện ta được nghiệm x  ,m Z
3

NGUYỄN VĂN LỰC  0933.168.309 SP Toán K35 - ĐH Cần Thơ


PP tọa độ trong không gian FB: http://www.facebook.com/VanLuc168

I. KIẾN THỨC TỌA ĐỘ ĐIỂM – TỌA ĐỘ VÉCTƠ


Chuyên đề: Phương pháp tọa độ trong không gian

I. Hệ trục toạ độ ĐỀ-CÁC trong không gian


 x'Ox : trục hoành z

 y'Oy : trục tung


 z'Oz : trục cao x'
 O : gốc toạ độ
k
 i, j, k : véc tơ đơn vị y' y
O
(hay i; j; k : véc tơ đơn vị ) j
i
Quy ước : Không gian mà trong đó có chọnx hệ trục toạ độ Đề-Các vuông góc Oxyz
được gọi là không gian Oxyz và ký hiệu là : kg(Oxyz) z '

II. Toạ độ của một điểm và của một véc tơ:


1. Định nghĩa 1: Cho M  kg(Oxyz) . Khi đó véc tơ OM được biểu diển một cách duy
nhất theo i, j, k bởi hệ thức có dạng : OM  xi  y j + yk vôùi x,y,z  .
z Bộ số (x;y;z) trong hệ thức trên được gọi là toạ độ của điểm M.
M
Ký hiệu: M(x;y;z)
y
O

x ( x: hoành độ của điểm M; y: tung độ của điểm M, z: cao độ của điểm M )

ñ/n
M ( x; y; z)  OM  xi  y j  zk
 Ý nghĩa hình học:
z
R M2

M3 z M
O y y x  OP ; y= OQ ; z = OR
x Q

p M1
x

2. Định nghĩa 2: Cho a  kg(Oxyz) . Khi đó véc tơ a được biểu diển một cách duy nhất
theo i, j, k bởi hệ thức có dạng : a  a1 i  a2 j + a3 k vôùi a1,a2 ,a3  .
Bộ số (a1;a2;a3) trong hệ thức trên được gọi là toạ độ của véc tơ a .
Ký hiệu: a  (a1; a2 ; a3 )

ñ/n
a=(a1;a2 ;a3 )  a  a1 i  a2 j  a3 k

NGUYỄN VĂN LỰC  0933.168.309 SP Toán K35 - ĐH Cần Thơ


PP tọa độ trong không gian FB: http://www.facebook.com/VanLuc168

II. Các công thức và định lý về toạ độ điểm và toạ độ véc tơ :


Định lý 1: Nếu A( x A ; y A ; zA ) vaø B(x B; yB ; zB ) thì

AB  ( xB  x A ; yB  y A ; zB  zA )

Định lý 2: Nếu a  (a1; a2 ; a3 ) vaø b  (b1; b2 ; b3 ) thì


a1  b1

* a  b  a2  b2
a  b
 3 3

* a  b  (a1  b1; a2  b2 ; a3  b3 )
* a  b  (a1  b1; a2  b2 ; a3  b3 )
* k.a  (ka1; ka2 ; ka3 ) (k  )

III. Sự cùng phương của hai véc tơ:


Nhắc lại
 Hai véc tơ cùng phương là hai véc tơ nằm trên cùng một đường thẳng
hoặc nằm trên hai đường thẳng song song .
 Định lý về sự cùng phương của hai véc tơ:
 Định lý 3 : Cho hai véc tơ a vaø b vôùi b  0

a cuøng phöông b  !k  sao cho a  k.b

Nếu a  0 thì số k trong trường hợp này được xác định như sau:
k > 0 khi a cùng hướng b
k < 0 khi a ngược hướng b
a
k 
b

 Định lý 4 : A, B, C thaúng haøng  AB cuøng phöông AC

 Định lý 5: Cho hai véc tơ a  (a1; a2 ; a3 ) vaø b  (b1; b2 ; b3 ) ta có :


a1  kb1

a cuøng phöông b  a2  kb2  a 1 : a2 : a3  b1 : b2 : b3
a  kb
 3 3

IV. Tích vô hướng của hai véc tơ:


Nhắc lại:
a.b  a . b .cos(a, b)
2 2
a a
ab  a.b  0

NGUYỄN VĂN LỰC  0933.168.309 SP Toán K35 - ĐH Cần Thơ


PP tọa độ trong không gian FB: http://www.facebook.com/VanLuc168

 Định lý 6: Cho hai véc tơ a  (a1; a2 ; a2 ) vaø b  (b1; b2 ; b3 ) ta có :

a.b  a1b1  a2 b2  a3b3

Định lý 7: Cho hai véc tơ a  (a1; a2 ; a3 ) ta có :

a  a12  a22  a32


 Định lý 8: Nếu A( x A ; y A ; zA ) vaø B(x B; yB ; zB ) thì

AB  ( xB  x A )2  ( yB  y A )2  (zB  zA )2
 Định lý 9: Cho hai véc tơ a  (a1; a2 ; a3 ) vaø b  (b1; b2 ; b3 ) ta có :

ab  a1b1  a2 b2  a3b3  0

 Định lý 10: Cho hai véc tơ a  (a1; a2 ; a3 ) vaø b  (b1; b2 ; b3 ) ta có :

a.b a1b1  a2 b2  a3b3


cos(a, b)  
a.b a12  a22  a32 . b12  b22  b32

V. Điểm chia đoạn thẳng theo tỷ số k:


Định nghĩa : Điểm M được gọi là chia đoạn AB theo tỷ số k ( k  1 ) nếu như :
MA  k.MB
  
A M B

Định lý 11 : Nếu A( x A ; y A ; zA ) , B(x B; yB ; zB ) và MA  k.MB ( k  1 ) thì

 x A  k .x B
 xM  1  k

 y A  k .y B
 yM 
 1 k
 zA  k .zB
 zM  1  k

 x A  xB
 xM  2

 y y
Đặc biệt : M là trung điểm của AB   yM  A B
 2
 zA  zB
 zM  2

NGUYỄN VĂN LỰC  0933.168.309 SP Toán K35 - ĐH Cần Thơ


PP tọa độ trong không gian FB: http://www.facebook.com/VanLuc168

Định lý 12: Cho tam giác ABC biết A( x A ; y A ; zA ) , B(x B; yB ; zB ), C(xC ; yC ; zC )


 x A  x B  xC
 xG  3

 y y y
G là trọng tâm tam giác ABC   yG  A B C
 3
 zA  zB  zC
 zG  3

VI. Tích có hướng của hai véc tơ:


1. Định nghĩa: Tích có hướng của hai véc tơ a  (a1; a2 ; a3 ) vaø b  (b1; b2 ; b3 ) là một véc
tơ được
ký hiệu :  a; b  có tọa độ là : 1 2 3

a a3 a3 a1 a1 a2  a  (a1; a2 ; a3 )
 a; b    2 ; ;  Cách nhớ:
 
 b2 b3 b3 b1 b1 b2  b  (b1; b2 ; b3 )
2. Tính chất:

  a; b   a vaø  a; b   b
   
A
1
 SABC  .  AB; AC 
2 B C
D C D'
 S ABCD   AB; AD  C'
A A'
B'
B
D
 VABCD. A'B'C'D'   AB; AD  .AA' C
D A
B
1
 VABCD  .  AB; AC  . AD C
6 A

B
 a cuøng phöông b   a; b   0

 a, b, c ñoàng phaúng   a, b  .c  0

 A, B, C, D đồng phẳng  AB, AC, AD đồng phẳng   AB,AC .AD  0

NGUYỄN VĂN LỰC  0933.168.309 SP Toán K35 - ĐH Cần Thơ


PP tọa độ trong không gian FB: http://www.facebook.com/VanLuc168

II. MẶT PHẲNG TRONG KHÔNG GIAN

Chuyên đề: Phương pháp tọa độ trong không gian

I. Các định nghĩa:


1. Véc tơ chỉ phương (VTCP) của đường thẳng:

ñn  a  0

a là VTCP của đường thẳng (  )  
a coù giaù song song hoaëc truøng vôùi ( )

a

a ( )
Chú ý:
 Một đường thẳng có vô số VTCP, các véc tơ này cùng phương với nhau.
 Một đường thẳng (  ) hoàn toàn được xác định khi biết một điểm thuộc nó
và một VTCP của nó.
2. Cặp VTCP của mặt phẳng: 
a 
b

 b

Cho mặt phẳng  xác định bởi hai đường thẳng cắt nhau a và b . Gọi a là
VTCP của đường thẳng a và b là VTVP của đường thẳng b. Khi đó :
Cặp (a,b) được gọi là cặp VTCP của mặt phẳng 
Chú ý :
 Một mặt phẳng  hoàn toàn được xác định khi biết một điểm thuộc nó và
một cặp VTCP của nó.
3. Véc tơ pháp tuyến ( VTPT) của mặt phẳng : 
n

ñn  n  0

n là VTPT của mặt phẳng   
n coù giaù vuoâng goùc vôùi mp
Chú ý :
 Một mặt phẳng có vô số VTPT, các véc tơ này cùng phương với nhau.

NGUYỄN VĂN LỰC  0933.168.309 SP Toán K35 - ĐH Cần Thơ


PP tọa độ trong không gian FB: http://www.facebook.com/VanLuc168

 Một mặt phẳng hoàn toàn được xác định khi biết một điểm thuộc nó và
một cặp VTPT của nó.

4. Cách tìm tọa độ một VTPT của mặt phẳng khi biết cặp VTCP của nó:
 a  (a1; a2 ; a3 )
Định lý: Giả sử mặt phẳng  có cặp VTCP là :  thì mp  có một VTPT
 b  (b1; b2 ; b3 )
là :

a a3 a3 a1 a1 a2 
n   a; b    2 ; ; 
 b2 b3 b3 b1 b1 b2 

  
n  [a , b ]

a 
b

Ví dụ: Tìm một VTPT của mặt phẳng  biết  đi qua ba điểm A(-2;0;1), B(0;10;3),
C(2;0;-1)

II. Phương trình của mặt phẳng :


Định lý 1: Trong Kg(Oxyz) . Phương trình mặt phẳng  đi qua điểm M0 ( x0 ; y0 ; z0 ) và
có một VTPT n  ( A; B; C) là:

n  ( A; B; C )

M  x; y;z   A( x  x0 )  B( y  y0 )  C ( z  z0 )  0
M 0 ( x0 ; y 0 ; z 0 )
 
n  ( A; B; C )
z

M0
Định lý 2: Trong Kg(Oxyz) . Phương trình dạng :
y
Ax  By  Cz  D  0 với A2  B2  C 2  0

là phương trình tổng quát của một mặt phẳng . x


Chú ý :
 Nếu ( ) : Ax  By  Cz  D  0 thì ( ) có một VTPT là n  ( A; B; C)
 M0 ( x0 ; y0 ; z0 )  ( ) : Ax  By  Cz  D  0  Ax 0  By0  Cz0  D  0 (Oyz )
z

Các trường hợp đặc biệt: y


1. Phương trình các mặt phẳng tọa độ: O
 (Oxy):z = 0 (Oxz )
x
 (Oyz):x = 0
 (Oxz):y = 0
(Oxy )

NGUYỄN VĂN LỰC  0933.168.309 SP Toán K35 - ĐH Cần Thơ


PP tọa độ trong không gian FB: http://www.facebook.com/VanLuc168

2. Phương trình mặt phẳng theo đoạn chắn:


 A(a; 0; 0)

 Phương trình mặt phẳng cắt các trục Ox, Oy, Oz tại  B(0; b; 0) (a,b,c  0)
C (0; 0; c)

x y z C

là:   1
a b c
c
O b
a B

Ví dụ 1: Trong Kg(Oxyz) cho ba điểm A(3;1;0), B(-1;2;-1), C(2;-1;3)


Viết phương trình mặt phẳng (ABC)
Ví dụ 2: Trong Kg(Oxyz) cho A 1;2;3 , B  2; 3;1 . Viết phương trình mặt phẳng  P 
đi qua A và vuông góc với đường thẳng AB.
Ví dụ 3: Trong Kg(Oxyz) cho hai mặt phẳng  P  : x  2 y  3z  4  0 và
 R  : 3x  2y  z  1  0 . Viết phương trình mặt phẳng  R  đi qua A 1;1;1 đồng thời vuông
góc với cả  P  và  Q  .
Ví dụ 4: Viết phương trình mặt phẳng đi qua điểm M(9;1;1) , cắt các tia Ox, Oy, Oz
lần lượt tại A, B, C sao cho thể tích tứ diện OABC có giá trị nhỏ nhất.

III. Vị trí tương đối của hai mặt phẳng :


1. Một số quy ước và ký hiệu:
a1  tb1
a  tb
(a1 , a2 ,..., an )  2 2
Hai bộ n số :  được gọi là tỷ lệ với nhau nếu có số t  0 sao cho .
(b1 , b2 ,..., bn ) .

an  tbn
a1 a2 a
Ký hiệu: a1 : a2 : ... : an  b1 : b2 : ... : bn hoặc   ...  n
b1 b2 bn
2. Vị trí tương đối của hai mặt phẳng:
Định lý: Trong Kg(Oxyz) cho hai mặt phẳng  ,  xác định bởi phương trình :
( ) : A1x  B1y  C1z  D1  0 coù VTPT n1  ( A1; B1; C1 )
(  ) : A2 x  B2 y  C2 z  D2  0 coù VTPT n2  ( A2 ; B2 ; C2 )

 n1
n2

  n2
n1 a  n1
n 2
b
a
a
b

NGUYỄN VĂN LỰC  0933.168.309 SP Toán K35 - ĐH Cần Thơ


PP tọa độ trong không gian FB: http://www.facebook.com/VanLuc168

A1 B1 B C C A
( ) caét ( )  A1 : B1 : C1  A2 : B2 : C2 (hay:  hoaëc 1  1 hoaëc 1  1 )
A 2 B2 B2 C2 C2 A2
A1 B1 C1 D1
( ) // ( )    
A 2 B2 C2 D2
A1 B1 C1 D1
( )  ( )    
A 2 B2 C2 D2
Đặc biệt:
    A1 A2  B1B2  C1C2  0

NGUYỄN VĂN LỰC  0933.168.309 SP Toán K35 - ĐH Cần Thơ


PP tọa độ trong không gian FB: http://www.facebook.com/VanLuc168

II. ĐƯỜNG THẲNG TRONG KHÔNG GIAN

Chuyên đề: Phương pháp tọa độ trong không gian

I. Phương trình của đường thẳng:


1.Phương trình tham số của đường thẳng:
Định lý: Trong Kg(Oxyz) . Phương trình tham số của đường thẳng ( ) đi qua điểm
M 0 ( x0 ; y0 ; z0 ) và nhận a  (a1; a2 ; a3 ) làm VTCP là :


z a  x  x0  ta1

( ) () :  y  y0  ta2 (t  )
 z  z  ta
M0 M ( x, y , z ) y  0 3

2. Phương trình chính tắc của đường thẳng:


Định lý: Trong Kg(Oxyz) . Phương trình chính tắc của đường thẳng ( ) đi qua
điểm M0 ( x0 ; y0 ; z0 ) và nhận a  (a1; a2 ; a3 ) làm VTCP là :

x  x0 y  y0 z  z0
( ) :  
a1 a2 a3

Ví dụ 1: Trong không gian với hệ tọa độ Oxyz , cho các điểm A 2; 2;1 , B 0; 2;5 . Viết

phương trình tham số của đường thẳng đi qua A và B .


Ví dụ 2: Trong không gian với hệ tọa độ Oxyz , cho tam giác ABC có
A 1;1;0 , B 0; 2;1 và trọng tâm G 0; 2; 1 . Viết phương trình đường thẳng đi qua điểm
C và vuông góc với mặt phẳng ABC
Ví dụ 3:
 x  1  2t

Cho điểm M(-2;1;1) và đường thẳng (d) : y  1  t . Lập phương trình mặt phẳng (P)
z  3  t

qua điểm M và vuông góc với đường thẳng (d).


x z z
Ví dụ 4: Cho điểm M(1;2;3) và đường thẳng (d) :   . Lập phương trình mặt
1 1 1

phẳng (P) chứa điểm M và đường thẳng (d)

NGUYỄN VĂN LỰC  0933.168.309 SP Toán K35 - ĐH Cần Thơ


PP tọa độ trong không gian FB: http://www.facebook.com/VanLuc168

II. Vị trí tương đối của đường thẳng và mặt phẳng :


1.Vị trí tương đối của đường thẳng và mặt phẳng :

M a ( )
( )  
a n  
n n
M M 
a a a a ( )

x  x0 y  y0 z  z0
Định lý: Trong Kg(Oxyz) cho: đường thẳng () :   có VTCP
a1 a2 a3
a  (a1; a2 ; a3 ) và qua M0 ( x0 ; y0 ; z0 ) và mặt phẳng ( ) : Ax  By  Cz  D  0 có VTPT
n  ( A; B; C )
Khi đó :
() caét ( )  Aa1  Ba2  Ca3  0
Aa1  Ba2  Ca3  0
() // ( )  
 Ax0  By0  Cz0  D  0
Aa1  Ba2  Ca3  0
(  )  ( )  
 Ax0  By0  Cz0  D  0


a 
Đặc biệt: ()  ( )  a1 : a2 : a3  A : B : C n

a
 pt()
Chú ý: Muốn tìm giao điểm M của (  ) và (  ) ta giải hệ phương trình :  tìm
 pt( )

x,y,z. Suy ra: M(x,y,z)


Ví dụ 1: Cho hai điểm A(0;0;-3) , B(2;0;-1) và mặt phẳng (P): 3x - 8y + 7z -1 = 0
Tìm toạ độ giao điểm I của đường thẳng AB và mặt phẳng (P).
Ví dụ 2: Cho điểm M(1;1;1) và mặt phẳng (P) có phương trình: x  2y  3z  14  0 .
Tìm tọa độ hình chiếu vuông góc của M trên mặt phẳng (P).
x 1 y  2 z  2
Ví dụ 3: Cho đường thhẳng (d) :   và mặt phẳng
1 5 4

(P) : x  3y  4m 2z  m  0 . Tìm m để đường thẳng (d) nằm trong mặt phẳng (P).
2. Vị trí tương đối của hai đường thẳng :
1  1
 u
M ' a M0  M0
0 u 1
 '  
b 
2 1 M 0 M 0 u u' 2 
2 u'
'
u'
M
M0 M 0' 0
2

NGUYỄN VĂN LỰC  0933.168.309 SP Toán K35 - ĐH Cần Thơ


PP tọa độ trong không gian FB: http://www.facebook.com/VanLuc168

Định lý: Trong Kg(Oxyz) cho hai đường thẳng :


x  x0 y  y0 z  z0
(1 ) :   coù VTCP u  (a; b; c) vaø qua M 0 ( x0 ; y0 ; z0 )
a b c
x  x0 y  y0 z  z0
( 2 ) : '
 '
 ' coù VTCP u'  (a' ; b' ; c' ) vaø qua M'0 ( x0' ; y0' ; z0' )
a b c

 (1 ) vaø ( 2 ) ñoàng phaúng  u, u'  .M0 M0'  0


 
 u, u'  .M M '  0

 (1 ) caét ( 2 )     0 0
a : b : c  a' : b' : c'

 (1 ) // ( 2 )  a : b : c  a' : b' : c'  ( x0'  x0 ) : ( y0'  y0 ) : ( z0'  z0 )


 (1 )  ( 2 )  a : b : c  a' : b' : c'  ( x0'  x0 ) : ( y0'  y0 ) : (z0'  z0 )

 (1 ) vaø ( 2 ) cheùo nhau  u, u'  .M0 M0'  0


 
 pt(1 )
Chú ý: Muốn tìm giao điểm M của (1 ) vaø ( 2 ) ta giải hệ phương trình :  tìm
 pt(2 )
x,y,z. Suy ra: M(x,y,z)

III. Góc trong không gian:


1. Góc giữa hai mặt phẳng:
Định lý: Trong Kg(Oxyz) cho hai mặt phẳng  ,  xác định bởi phương trình :
( ) : A1 x  B1y  C1z  D1  0
(  ) : A2 x  B2 y  C2 z  D2  0
Gọi  là góc giữa hai mặt phẳng ( ) & (  ) ta có công thức: 
n1  ( A1 ; B1 ; C1 )

A1 A2  B1 B2  C1C2 
cos   n2  ( A2 ; B2 ; C 2 )
A12  B12  C12 . A22  B22  C22

0 0    90 0
b

Ví dụ: Cho hai mặt phẳng (P) : x  y  2  0 &(Q) : x  z  3  0 . Xác định góc giữa hai
mặt phẳng (P) và (Q).

2. Góc giữa đường thẳng và mặt phẳng:


x  x0 y  y0 z  z0
Định lý: Trong Kg(Oxyz) cho đường thẳng () :  
a b c ( )
và mặt phẳng ( ) : Ax  By  Cz  D  0 
a  (a; b; c)
Gọi  là góc giữa hai mặt phẳng () & ( ) ta có công thức: 
n  ( A; B; C )
Aa  Bb  Cc
sin  
A2  B 2  C 2 . a 2  b 2  c 2 a
0 0    90 0

NGUYỄN VĂN LỰC  0933.168.309 SP Toán K35 - ĐH Cần Thơ


PP tọa độ trong không gian FB: http://www.facebook.com/VanLuc168


a1  (a; b; c)
3.Góc giữa hai đường thẳng :
Định lý: Trong Kg(Oxyz) cho hai đường thẳng :
1
x  x0 y  y0 z  z0
(1 ) :  
a b c 2 
x  x0 y  y0 z  z0 a 2  ( a ' ; b' ; c ' )
( 2 ) :   '
a' b' c
0 0    90 0
Gọi  là góc giữa hai mặt phẳng (1 ) & ( 2 ) ta có công thức:

aa'  bb'  cc'


cos  
a 2  b2  c 2 . a'2  b'2  c'2

IV. Khoảng cách:


1. Khoảng cách từ một điểm đến một mặt phẳng:
Định lý: Trong Kg(Oxyz) cho mặt phẳng ( ) : Ax  By  Cz  D  0 và điểm M0 ( x0 ; y0 ; z0 )
Khoảng cách từ điểm M0 đến mặt phẳng ( ) được tính bởi công thức:
M 0 ( x0 ; y 0 ; z 0 )

Ax0  By0  Cz0  D


d ( M0 ;  ) 
H A2  B 2  C 2
a

Ví dụ: Cho hình tứ diện ABCD biết tọa độ các đỉnh A(2,3,1) ; B(4,1,-2) ; C(6,3,7) ;
D(-5,-4,8). Tính độ dài đường cao hình tứ diện xuất phát từ D.

2. Khoảng cách từ một điểm đến một đường thẳng:

Định lý: Trong Kg(Oxyz) cho đường thẳng (  ) đi qua điểm M0 ( x0 ; y0 ; z0 ) và có


VTCP u  (a; b; c) . Khi đó khoảng cách từ điểm M1 đến ( ) được tính bởi công thức:
M1
  M 0 M1; u 
 
u ( ) d ( M1 ,  ) 
M 0 ( x0 ; y 0 ; z 0 ) H u

x y 1 z  3
Ví dụ: Cho đường thẳng : (d ) :   và điểm A(1;2;1)
3 4 1
Tính khoảng cách từ điểm A đến đường thẳng (d).

NGUYỄN VĂN LỰC  0933.168.309 SP Toán K35 - ĐH Cần Thơ


PP tọa độ trong không gian FB: http://www.facebook.com/VanLuc168

3. Khoảng cách giữa hai đường thẳng chéo nhau:

Định lý: Trong Kg(Oxyz) cho hai đường thẳng chéo nhau :
(1 ) coù VTCP u  (a; b; c) vaø qua M 0 ( x0 ; y0 ; z0 )
( 2 ) coù VTCP u'  (a' ; b' ; c' ) vaø qua M'0 ( x0' ; y0' ; z0' )
Khi đó khoảng cách giữa (1 ) vaø ( 2 ) được tính bởi công thức

u 1
M0

u, u ' .M0 M0'


 
 d (1 ,  2 ) 
u' u; u '
M 0' 2  

Ví dụ: Cho hai đường thẳng :


 x  9  6t
x  5 y  5 z 1 
(d1 ) :   vaø (d 2 ) :  y  2t
3 2 2 z  2  t

Tính khoảng cách giữa hai đường thẳng (d1) và (d2).

NGUYỄN VĂN LỰC  0933.168.309 SP Toán K35 - ĐH Cần Thơ


PP tọa độ trong không gian FB: http://www.facebook.com/VanLuc168

III. MẶT CẦU TRONG KHÔNG GIAN

Chuyên đề: Phương pháp tọa độ trong không gian

I. Phương trình mặt cầu:


1. Phương trình chính tắc:
Định lý: Trong Kg(Oxyz). Phương trình của mặt cầu (S) tâm I(a;b;c), bán kính
R là :
z
(S )
(S) : ( x  a)2  ( y  b)2  (z  c)2  R2 (1)
I R
M ( x; y; z ) Phương trình (1) được gọi là phương trình
O y chính tắc của mặt cầu

Đặc biệt: Khi I  O thì (C) : x2  y2  z2  R2


x

2. Phương trình tổng quát:


Định lý : Trong Kg(Oxyz). Phương trình :

x 2  y2  z2  2ax  2by  2cz  d  0

với a2  b2  c2  d  0 là phương trình của mặt cầu (S) có


tâm I(a;b;c), bán kính R  a2  b2  c2  d .
Ví dụ: Cho 4 điểm A(-1;-2;0), B(2;-6;3), C(3;-3;-1), D(-1;-5;3)
Viết phương trình mặt cầu đi qua bốn điểm A, B, C, D. Xác định tâm và bán
kính của mặt cầu
II. Giao của mặt cầu và mặt phẳng:
Định lý: Trong Kg(Oxyz) cho mặt phẳng ( ) và mặt cầu (S) có phương trình :
( ) : Ax  By  Cz  D  0
(S ) : ( x  a)2  ( y  b)2  (z  c)2  R2
Gọi d(I;  ) là khoảng cách từ tâm mặt cầu (S) đến mặt phẳng 
Ta có :
1. ( ) caét maët caàu (S)  d(I; ) < R
2. ( ) tieáp xuùc maët caàu (S)  d(I; ) =R
3. ( ) khoâng caét maët caàu (S)  d(I; ) > R

(S )
(S )
I
(S )
NGUYỄN VĂN LỰC  0933.168.309 I SP Toán K35 - ĐH Cần Thơ
R
R (C )
PP tọa độ trong không gian FB: http://www.facebook.com/VanLuc168

Chú ý:
Khi  cắt mặt cầu (S) thì sẽ cắt theo một đường tròn (C). Đường tròn (C) nầy có:

 Tâm là hình chiếu vuông góc của tâm mặt cầu trên mặt phẳng 
 Bán kính r  R2  d 2 (I , )

Ví dụ: Cho mặt cầu (S) : x2  y 2  z 2  4x  2y  2z  3  0 . Viết phương trình tiếp diện của
mặt cầu tại
điểm M(0;1;-2).

NGUYỄN VĂN LỰC  0933.168.309 SP Toán K35 - ĐH Cần Thơ


PP tọa độ trong không gian FB: http://www.facebook.com/VanLuc168

IV. BÀI TẬP VÍ DỤ

Chuyên đề: Phương pháp tọa độ trong không gian

Ví dụ 1: Trong không gian Oxyz cho mặt phẳng (P): 2x  y  z  8  0 và đường thẳng
x  2 y 1 z 1
(d):   . Tìm phương trình    , hình chiếu vuông góc của (d) trên (P).
2 3 5

Bài giải

 Gọi A  (d)  P  , tọa độ A là nghiệm của hệ phương trình:


2x  y  z  8  0  x  6
 x  2

y  1

z  1   y  5  A  6;5; 9 
 2 3 5 z  9
 Lấy B  2; 1;1   d  , gọi (d') là đường thẳng qua B và vuông góc với (P)
Phương trình tham số của (d') là:
 x  2  2t
 y  1  t
 z  1  t
 Gọi H  (d ') (P) , tọa độ H là nghiệm của hệ phương trình:
 2
t  3
 x  2  2t  10
 y  1  t  x 
z  1  t  3  H  10 ;  1 ; 5 
1  
2x  y  z  8  0 y    3 3 3
  3
z  5
 3
    chính là đường thẳng đi qua hai điểm A, H. Ta có
 8 16 32  8
AH    ;  ;    1; 2; 4 
 3 3 3  3
x 6 y5 z 9
 Vậy phương trình    :  
1 2 4
x 1 y 1 z  3
Ví dụ 2: Trong không gian Oxyz cho M  1; 2; 3 ;a   6; 2; 3 ,  d  :   .
3 2 5
Tìm phương trình đường thẳng    qua M, vuông góc a và cắt (d).
Bài giải
NGUYỄN VĂN LỰC  0933.168.309 SP Toán K35 - ĐH Cần Thơ
PP tọa độ trong không gian FB: http://www.facebook.com/VanLuc168

 Lấy điểm N  (d) , tọa độ N có dạng N 1  3t; 1  2t;5  3t  , ta có:


MN   2  3t; 3  2t;6  5t 
 MN  a  MN.a  0  6  2  3t   2  3  2t   3  6  5t   0  t  0
 Đường thẳng cần tìm đi qua M có VTCP là MN   2; 3;6 có phương trình
là:
x 1 y  2 z  3
 
2 3 6
Ví dụ 3: Trong không gian Oxyz, lập phương trình chính tắc của đường thẳng d đi
x 1 y  2 z
qua A  0;1;1 , vuông góc (d1 ) :   và cắt  d 2  là giao tuyến của hai mặt phẳng
3 1 1
có phương trình: x  y  z  2  0, x  1  0 .
Bài giải

 x  1
 Viết phương trình tham số của đường thẳng  d 2  :  y  1  t
 z  t
 Xét điểm B  1; 1  t, t   (d 2 ) . Tìm t để AB.a d  0
1

AB.a d1  0  t  3  B  1;2;3
x y 1 z 1
 Phương trình (d):  
1 2 3
x  3 y  2 z 1
Ví dụ 4: Trong không gian Oxyz, cho đường thẳng (d):   và mặt
2 1 1
phẳng (P): x  y  z  2  0 . Gọi M là giao điểm của (d) và (P). Viết phương trình đường

NGUYỄN VĂN LỰC  0933.168.309 SP Toán K35 - ĐH Cần Thơ


PP tọa độ trong không gian FB: http://www.facebook.com/VanLuc168

thẳng    nằm trong (P) saocho    vuông góc với (d) và khoảng cách từ M đến   
bằng 42 .

Bài giải

 Do M  (d) (P) nên tọa độ M là nghiệm của hệ phương trình:


 x  3 y  2 z  1  x  1
 
 2 1 1   y  3  M 1; 3;0 
 x  y  z  2  0 z  0
 (d) có VTCP a   2;1  1 và (P) có VTPT n P  1;1;1 .
Mặt phẳng (Q) chứa (d) và vuông góc với (P) có VTPT n Q  a; n P    2; 3;1
Phương trình mp(Q): 2x  3y  z  11  0
 Gọi (d') là hình chiếu vuông góc của (d) trên mặt phẳng (P) thì
(d)   P  Q
VTCP của (d') là a d '   n P ; n Q    4;1; 5  , phương trình tham số của (d') là:
 x  1  4t
 y  3  t
 z  5t
 Ta tìm N   d ' sao cho MN  42 , đặt N 1  4t; 3  t; 5  , ta có:
MN  42  42t 2  42  t  1
+ Với t  1 ta có N1  5; 2; 5  .  1  qua N1 nằm trong (P) và vuông góc với (d')
có VTCP là
a 1   n P ; n d '    6;9; 3  3  2; 3;1 . Phương trình đường thẳng cần tìm là:
x 5 y 2 z 5
 1  :  
2 3 1
x 3 y 4 z5
+ Với t  1 ta có:   2  :  
2 3 1

NGUYỄN VĂN LỰC  0933.168.309 SP Toán K35 - ĐH Cần Thơ


PP tọa độ trong không gian FB: http://www.facebook.com/VanLuc168

Ví dụ 5: Trong không gian Oxyz cho ba điểm A 1;0;1 , B 1; 2;1 ;C  4;1; 2  và mặt
phẳng (P): x  y  z  0 . Tìm trên (P) điểm M sao cho MA 2  MB2  MC2 đạt giá trị nhỏ
nhất.

Bài giải

 Gọi G là trọng tâm của tam giác ABC, ta có G  2;1;0  , ta có


MA 2  MB2  MC 2  3MG 2  GA 2  GB2  GC2 (1)
 Từ hệ thức (1) ta suy ra :
MA 2  MB2  MC 2 đạt GTNN  MG đạt GTNN  M là hình chiếu vuông
góc của G trên (P)
 Gọi (d) là đường thẳng qua G và vuông góc với (P) thì (d) có phương
trình tham số là:
 x  2  t
y  1  t
z  t
 Tọa độ M là nghiệm của hệ phương trình:
x  2  t  t  1
y  1  t 
z  t   x  1  M 1, 0, 1
y0
x  y  z  0 z  1
 
 Vậy M 1; 0; 1 .

Ví dụ 6: Trong không gian Oxyz, cho hai đường thẳng


x 1 y  2 z x  2 y 1 z 1
 d1  :   ; d2  :   và mặt phẳng  P  : x  y  2z  5  0 . Lập
1 2 1 2 1 1
phương trình đường thẳng song song với mặt phẳng (P) và cắt  d1  ,  d 2  lần lượt tại A,
B sao cho độ dài đoạn AB nhỏ nhất.

Bài giải

NGUYỄN VĂN LỰC  0933.168.309 SP Toán K35 - ĐH Cần Thơ


PP tọa độ trong không gian FB: http://www.facebook.com/VanLuc168

 Đặt A  1  a; 2  2a;a  , B  2  2b;1  b;1  b  , ta có


AB   a  2b  3; 2a  b  3; a  b  1
 Do AB song song với (P) nên:
AB  n P  1;1; 2   b  a  4
Suy ra: AB   a  5; a  1; 3
 Do đó: AB   a  5   a  1   3  2a 2  8a  35  2  a  2   27  3 3
2 2 2 2

Suy ra: min AB  3 3  ab  22


x 1 y  2 z  2
 Vậy phương trình đường thẳng cần tìm là:  
1 1 1

Ví dụ 7: Trong không gian Oxyz, cho A  0;0; 4  , B  2;0;0  và mặt phẳng (P) có phương
trình 2x  y  3  0 . Lập phương trình mặt cầu  S  đi qua ba điểm O, A, B và tiếp xúc
mặt phẳng (P).

Bài giải

 Phương trình mặt cầu (S) có dạng:


x 2  y 2  z 2  2ax  2by  2cz  d  0
d  0 d  0
 Do O, A, B   S   16  8c  0  c  2
4  4a  0 a  1
Suy ra: (S) có tâm I 1; b;2 , R  1  b2  4  b2  5
 Do (S) tiếp xúc với (P) nên:
2b3 b  0
d  I;(P)   R   b 2  4  4b 2  10b  0   5
4 1  b   2
 Vậy có hai mặt cầu là:
S1  : x 2  y2  z 2  2x  4z  0
S2  : x 2  y2  z 2  2x  5y  4z  0

Ví dụ 8: Trong không gian Oxyz, cho ba điểm A  0;1; 2  , B 1;1;1 , C  2; 2;3 và mặt
phẳng (P): x  y  z  3  0 . Tìm điểm M trên (P) sao cho MA  MB  MC đạt giá trị nhỏ
nhất.

NGUYỄN VĂN LỰC  0933.168.309 SP Toán K35 - ĐH Cần Thơ


PP tọa độ trong không gian FB: http://www.facebook.com/VanLuc168

Bài giải

 Gọi G là trọng tâm của tam giác ABC, suy ra: G 1;0; 2 
 Xét điểm M  (P) . Ta có:
MA  MB  MC  3 MG  3MG

Suy ra: MA  MB  MC đạt GTNN  MG đạt GTNN  M là hình chiếu của G


trên (P)
 Tìm M
+ Gọi (d) là đường thẳng qua G vuông góc với mặt phẳng (P)
x  1  t
Phương trình đường thẳng (d):  y   t
z  2  t
+ Tọa độ M là nghiệm của hệ phương trình:
x  1  t  t  2
 y  1 
z  2  t   x  1  M  1; 2;0 
y2
x  y  z  3  0 z  0
 
 Vậy M  1; 2; 0 

Ví dụ 9: Trong không gian Oxyz , cho đường thẳng (d) là giao tuyến của hai mặt
phẳng 5x  4y  3z  20  0;3x  4y  z  8  0 . Viết phương trình mặt cầu (S) có tâm
I  2;3; 1 và cắt (d) tại hai điểm A, B sao cho AB  16 .

Bài giải

 4 3 3 5 5 4 
 Đường thẳng (d) có VTCP là: u   ; ;   8; 4; 8  4  2;1; 2 
 4 1 1 3 3 4 

NGUYỄN VĂN LỰC  0933.168.309 SP Toán K35 - ĐH Cần Thơ


PP tọa độ trong không gian FB: http://www.facebook.com/VanLuc168

 Kẻ IH  AB thì HA  HB  8 và IH  d  I, (d)  , R  IH2  AH2


 Xét điểm M 11;0; 25  , ta có:
IM   9; 3; 24 
   u; IM    30;30; 15 
n
 d   2;1; 2 
 u; IM   30   302   15
2 2
 
 d  I;(d)     15
u 3

 Do đó: R  IH 2  AH 2  225  64  17
 Vậy phương trình mặt cầu (S) là:  x  2    y  3   z  1  289
2 2 2

x  2 y  3 z 1
Ví dụ 10: Trong không gian với hệ trục Oxyz, cho đường thẳng d :   .
1 2 2
Xét hình bình hành ABCD có A(1 ; 0 ; 0), C (2 ; 2 ; 2), D  d . Tìm tọa độ B biết diện tích
hình bình hành ABCD bằng 3 2.

Bài giải

x  2 y  3 z 1
 Do D  d :    D(t  2 ;  2t  3 ;  2t  1)
1 2 2
3 2
 Vì S ABCD  3 2  S ACD  . (1)
2
 Ta có AC  (1 ; 2 ; 2); AD  (t  3 ;  2t  3 ;  2t  1) .
Suy ra [ AC , AD ]  (4 ; 4t  7 ;  4t  9)
 Khi đó:
S ACD 
1
2

AC , AD  
1
2
16  (4t  7) 2  (4t  9) 2 
1
2
32t 2  128t  146 . (2)
Từ (1) và (2) ta có 32t  128t  128  0  t  2 . Suy ra D (0 ;  1 ;  3) .
2

 Do ABCD là hình bình hành nên AB  DC . Suy ra B(3 ; 3 ; 5)


 Vậy B  3;3;5  .

NGUYỄN VĂN LỰC  0933.168.309 SP Toán K35 - ĐH Cần Thơ


PP tọa độ trong không gian FB: http://www.facebook.com/VanLuc168

BÀI TẬP TỰ LUYỆN

1. Tìm tọa độ điểm

Câu 1. Trong không gian với hệ toạ độ Oxyz , cho điểm A  4;1;3 và đường thẳng
x 1 y 1 z  3
d:   . Viết phương trình mặt phẳng ( P) đi qua A và vuông góc với đường
2 1 3
thẳng d . Tìm tọa độ điểm B thuộc d sao cho AB  27 .

Đường thẳng d có VTCP là ud   2;1;3


Vì  P   d nên  P  nhận ud   2;1;3 làm VTPT
Vậy PT mặt phẳng  P  là : 2  x  4   1 y  1  3  z  3  0
 2 x  y  3 z  18  0
Vì B  d nên B  1  2t;1  t; 3  3t 
AB  27  AB 2  27   3  2t   t 2   6  3t   27  7t 2  24t  9  0
2 2

t  3
Vậy B  7; 4;6  hoặc B   ; ;  
13 10 12
 3
t   7 7 7
 7

Câu 2. Trong không gian với hệ tọa độ Oxyz, cho điểm A(-1;0;0) và đường thẳng d có
x  2 y 1 z 1
phương trình   . Lập phương trình mặt phẳng (P) đi qua A và vuông
1 2 1
góc với đường thẳng d. Từ đó suy ra tọa độ điểm H là hình chiếu vuông góc của A lên
đường thẳng d.

+) d có 1 VTCP là u  1; 2;1 .


+) (P) qua A(-1;0;0) và có VTPT n  u  1; 2;1 có pt : x + 2y + z +1 = 0.
+) H là giao điểm của (d) và (P) nên tọa độ H là nghiệm của hệ pt
 x  2 y 1 z 1 x  1
   
 1 2 1   y  1. Vậy H(1;-1;0).
 x  2 y  z  1  0 z  0

Câu 3. Trong không gian với hệ tọa độ Oxyz cho hai điểm A(2;-1;4), B(0;1;0) và đường
x 2t
thẳng : y 1 t , t . Viết phương trình mặt phẳng (P) đi qua điểm A
z 4 t
và vuông góc với đường thẳng và tìm tọa độ điểm M thuộc đường thẳng sao cho tam
giác ABM vuông tại M.

NGUYỄN VĂN LỰC  0933.168.309 SP Toán K35 - ĐH Cần Thơ


PP tọa độ trong không gian FB: http://www.facebook.com/VanLuc168

a) * Mp(P) có vtpt n a (2; 1;1)


*Ptmp(P) là: 2x – y + z - 9 = 0.
*Xét ptgđ của đt và mp(P) 4t – 1(1-t) + (4 + t) - 9 = 0 t = 1.
* Gọi N là gđ cần tìm
Thay t = 1 vào đt ta được N(2 ; 0 ; 5)

b) Ta có M nên tọa độ M(2t ; 1- t ; 4 + t)


Vì tam giác ABM vuông tại M nên ta có
t=0
AM BM AM .BM 0 1
t=
3
2 2 13
* Vậy ta có hai điểm M cần tìm là M(0;1;4), M( ; ; )
3 3 3

Câu 4. Trong không gian với hệ tọa độ Oxyz cho ba điểm A(1;-2;1), B(-1;0;3), C(0;2;1).
Lập phương trình mặt cầu đường kính AB và tìm tọa độ điểm H là chân đường cao kẻ từ
A của tam giác ABC.

Tìm được tọa độ tâm I của mặt cầu I(0;-1;2), bán kính mặt cầu: R  3
Phương trình mặt cầu (S): x 2  ( y  1)2  ( z  2)2  3
Giả sử H(x;y;z), AH  (x 1; y 2; z1), BC  (1; 2; 2), BH  ( x  1; y; z  3)
AH  BC  AH .BC  0  x  2 y  2 z  5
2 x  y  2 7 4 23
BH cùng phương BC   , Tìm được H(  ; ; )
y  z  3 9 9 9

Câu 5. Trong không gian với hệ tọa độ Oxyz, cho điểm A  2;5;1 và mặt phẳng
( P) : 6 x  3 y  2 z  24  0 . Tìm tọa độ điểm H là hình chiếu vuông góc của A trên mặt phẳng
(P). Viết phương trình mặt cầu (S) có diện tích 784 và tiếp xúc với mặt phẳng (P) tại H,
sao cho điểm A nằm trong mặt cầu.

 x  2  6t

Gọi d là đường thẳng đi qua A và vuông góc với (P). Suy ra: d :  y  5  3t
 z  1  2t

Vì H là hình chiếu vuông góc của A trên (P) nên H  d  ( P) .
Vì H  d nên H  2  6t;5  3t;1  2t  .

Mặt khác, H  ( P ) nên ta có: 6  2  6t   3  5  3t   2 1  2t   24  0  t  1


Do đó, H  4; 2;3 .
Gọi I , R lần lượt là tâm và bán kính mặt cầu.
Theo giả thiết diện tích mặt cầu bằng 784 , suy ra 4 R 2  784  R  14 .
Vì mặt cầu tiếp xúc với mặt phẳng (P) tại H nên IH  ( P )  I  d .
Do đó tọa độ điểm I có dạng I  2  6t;5  3t;1  2t  , với t  1 .
Theo giả thiết, tọa độ điểm I thỏa mãn:
NGUYỄN VĂN LỰC  0933.168.309 SP Toán K35 - ĐH Cần Thơ
PP tọa độ trong không gian FB: http://www.facebook.com/VanLuc168

 6  2  6t   3  5  3t   2 1  2t   24
  14  t  1
d ( I , ( P ))  14 
 6  3  (2)   t  3  t  1
2 2 2

 AI  14   2  t  2
  6t    3t    2t   14 
2 2 2

Do đó, I  8;8;  1 .
Vậy, mặt cầu ( S ) :  x  8   y  8   z  1  196
2 2 2

Câu 6. Trong không gian Oxyz, cho các điểm A(1;0;0); B(0;2;0); C(0;0;-2) tìm tọa độ
điểm O’ đối xứng với O qua (ABC).

*Từ phương trình đoạn chắn suy ra pt tổng quát của mp(ABC) là:2x+y-z-2=0
*Gọi H là hình chiếu vuông góc của O l ên (ABC), OH vuông góc với
(ABC) nên OH // n(2;1;1) ; H   ABC 
1 2 1 1
Ta suy ra H(2t;t;-t) thay vào phương trình( ABC) có t=
suy ra H ( ; ; )
3 3 3 3
4 2 2
*O’ đối xứng với O qua (ABC)  H là trung điểm của OO’  O ' ( ; ; )
3 3 3

Câu 7. Trong không gian với hệ tọa độ Oxyz, cho mặt phẳng (P ) : x y z 3 0 và
x 2 y 1 z
đường thẳng d : . Tìm tọa độ giao điểm của (P) và d; tìm tọa độ điểm
1 2 1
A thuộc d sao cho khoảng cách từ A đến (P) bằng 2 3 .

x  2  t

Ta có phương trin
̀ h tham số của d là  y  1  2t
 z  t

I  d  ( P )  Ta có phương trình: (2  t )  ( 1  2t )  ( t )  3  0
 t  1  I (1;1;1)
Ta có A  d  A(2  t; 1  2t; t )
(2  t )  (1  2t )  (t )  3 2t  2
Khi đó, ta có d ( A;( P))  
1 1 1
2 2 2
3
2t  2 t  4
Vâ ̣y d ( A;( P))  2 3   2 3  t  1  3  
3 t  2
Khi đó t  4  A(2;7; 4); t  2  A(4; 5; 2)

Câu 8. Trong không gian Oxyz cho các điểm A(3;  4; 0) , B (0; 2; 4) , C (4; 2; 1) . Tính diện
tích tam giác ABC và tìm tọa độ điểm D trên trục Ox sao cho AD  BC .

Tính diện tích tam giác ABC


AB; AC    18; 7;  24
1 494
S 18 2  7 2  24 2 
2 2

NGUYỄN VĂN LỰC  0933.168.309 SP Toán K35 - ĐH Cần Thơ


PP tọa độ trong không gian FB: http://www.facebook.com/VanLuc168

Tìm tọa độ điểm D trên trục Ox sao cho AD  BC .


Gọi D(x; 0; 0) . Ta có AD  BC ( x 3 )2 42 02 42 02 32

Câu 9. Trong không gian với hệ tọa độ Oxyz, cho mặt phẳng  P  : x  y  z  1  0 và hai
điểm A 1; 3;0  , B  5; 1; 2  . Tìm tọa độ điểm M trên mặt phẳng  P  sao cho MA  MB đạt
giá trị lớn nhất.

Kiểm tra thấy A và B nằm khác phía so với mặt phẳng  P  .


Gọi B '  x; y; z  là điểm đối xứng với B  5; 1; 2 
Suy ra B '  1; 3; 4  . Lại có MA  MB  MA  MB '  AB '  const
Vậy MA  MB đạt giá trị lớn nhất khi M , A, B ' thẳng hàng hay M là giao điểm của đường
thẳng AB ' với mặt phẳng  P 
A

B’
M
P
B

x  1 t

AB ' có phương trình  y  3
 z  2t

x  1 t t  3
 y  3  x  2
 
Tọa độ M  x; y; z  là nghiệm của hệ  
 z  2t  y  3
 x  y  z  1  0  z  6
Vậy điểm M  2; 3;6 

2. Phương trình đường thẳng


Câu 10. Trong không gian với hệ trục Oxyz , cho hai điểm A(7;2;1), B( 5; 4; 3) và mặt
phẳng (P ) : 3x 2y 6z 3 0 . Viết phương trình đường thẳng AB và chứng minh rằng
AB song song với (P).

 x  7  12t

+ Đường thẳng AB đi qua A, VTCP AB   12; 6; 4 có PTTS là  y  2  6t
 z  1  4t

 x  7  12t
 y  2  6t

+ Xét hệ phương trình  và CM được hệ VN
 z  1  4t
3 x  2 y  6 z  3  0
NGUYỄN VĂN LỰC  0933.168.309 SP Toán K35 - ĐH Cần Thơ
PP tọa độ trong không gian FB: http://www.facebook.com/VanLuc168

x2 y z 3
Câu 11. Trong khoâng gian Oxyz cho đđường thẳng (d1) :   và đường
1 2 2
x 1 y 1 z  2
thẳng (d2) :   .Tìm tọa độ giao điểm của( d1 )và ( d2).Viết phương trình
2 1 3
đường thẳng (d) đối xứng (d1) qua (d2).

Tọa độ giao điểm I(1;2;-1)


13 17 16
Trên (d1) lấy M1(2;0;-3).tọa độ hình chiếu của M1lên (d2) là H( ; ; )
7 7 7
22 34 11
Điểm đối xứng của M1 qua (d2) là M’1( ; ; )
7 7 7
15 20 4
.đường thẳng (d) đi qua I có VTCP IM  ( ; ; )
7 7 7
 15
x  1 7 t

PTTS(d):  y  2 
20
t (t  )
 7
 4
 z  1  7 t

Câu 12. Trong không gian oxyz cho điểm A(0;2;2) . Viết phương trình đường thẳng 
 x  2
x 1 y2 z 
qua A và vuông góc đường thẳng d1 :   ; đồng thời cắt d :  y  t .
2
3 2 2 z  1  t

Giả sử  cắt d 2 tại B(-2;t;1+t)


Ta có AB   2; t  2; t  1
Đường thẳng d1 có VTCP u   3; 2; 2 
 vuông d1  AB.u  0  t  3  AB   2;1; 2  .
 x  2u

Vậy  qua A có VTCP AB   2;1; 2  có PTTS:  y  2  u
 z  2  2u

Câu 13. Viết phương trình đường thẳng  đi qua A  3; 2; 4  , song song với mặt phẳng
x2 y  4 z 1
 P  : 3x  2 y  3z  7  0 và cắt đường thẳng  d  :   .
3 2 2

Ta có nP  3; 2; 3 . Giả sử B(2 + 3t ; –4 – 2t ; 1 + 2t) là giao điểm của  và d .


Khi đó AB  1  3t; 2  2t;5  2t  , AB ||  P   AB  nP  AB.nP  0  t  2 .
Vậy B(8; 8;5) và AB  5; 6;9 .
x 3 y  2 z  4
Vậy phương trình đường thẳng    :   .
5 6 9

NGUYỄN VĂN LỰC  0933.168.309 SP Toán K35 - ĐH Cần Thơ


PP tọa độ trong không gian FB: http://www.facebook.com/VanLuc168

Câu 14. Viết phương trình đường thẳng d’ là hình chiếu vuông góc của đường thẳng (d)
x y 1 z 1
  trên mặt phẳng (P): x + y – z +1 =0.
2 1 3

 x  2t

PTTS của d :  y  1  t
 z  1  3t

Thay x, y, z của phương trình đường thẳng d vào phương trình mặt phẳng (P) ta được:
2t – 1 +t – 1 – 3t + 1 = 0
 Phương trình vô nghiệm  d // (P).
Lấy điểm A(0; 1;1)  d .
x  t

Gọi  là đường thẳng qua A và vuông góc với mp(P)   :  y  1  t
z  1  t

Gọi H là hình chiếu của A lên mặt phẳng (P)  H    (P)
Thay x, y, z của phương trình  vào phương trình mặt phẳng (P) ta được:
1 1 2 2
t–1+t–1+t+1=0 t  H ; ; 
3 3 3 3
Gọi d’ là hình chiếu của d lên mặt phẳng (P)  d ' qua H và song song với d
 x  1  2t
 3

 d ' : y   2  t
3

z  2 3  3t

Câu 15. Trong không gian với hệ tọa độ Oxyz, cho hai đường thẳng d1:
x 1 y 1 z 1 x 1 y  2 z 1
  ; d2:   và mặt phẳng (P): x - y - 2z + 3 = 0. Viết
2 1 1 1 1 2
phương trình chính tắc của đường thẳng , biết  nằm trên mặt phẳng (P) và  cắt hai
đường thẳng d1 , d2 .

Gọi A = d1(P) suy ra A(1; 0 ; 2) ; B = d2  (P) suy ra B(2; 3; 1)


Đường thẳng  thỏa mãn bài toán đi qua A và B.
Một vectơ chỉ phương của đường thẳng  là u  (1;3; 1)
x 1 y z  2
Phương trình chính tắc của đường thẳng  là:  
1 3 1

x 1 y  2 z  2
Câu 16. Trong không gian với hệ toạ độ Oxyz, cho đường thẳng  :   và
3 2 2
mặt phẳng (P): x + 3y + 2z + 2 = 0. Lập phương trình đường thẳng song song với mặt
phẳng (P), đi qua M(2; 2; 4) và cắt đường thẳng ().

NGUYỄN VĂN LỰC  0933.168.309 SP Toán K35 - ĐH Cần Thơ


PP tọa độ trong không gian FB: http://www.facebook.com/VanLuc168

 x  1  3t

Đường thẳng () có phương trình tham số:  y  2  2t t 
 z  2  2t

Mặt phẳng (P) có VTPT n  (1; 3; 2)

Giả sử N(1 + 3t ; 2  2t ; 2 + 2t)    MN  (3t  3; 2t ;2t  2)

Để MN // (P) thì MN .n  0  t  7  N(20; 12; 16)


x2 y2 z4
Phương trình đường thẳng cần tìm :  
9 7 6

Câu 17. Trong không gian với hệ trục tọa độ Oxyz, cho đường thẳng
x3 y 9 z 6
d:   và mặt phẳng (P): . Lập phương trình đường
2 3 2
3
thẳng nằm trong mặt phẳng (P), vuông góc với d và cách d một khoảng bằng
238

Gọi
chứa . Giả sử tại H. Hạ HK , thì . Vậy
góc AKH nhọn là góc giữa (P) và (Q). Và HK là đoạn vuông góc chung của d và nên
. Do (Q) vuông góc với d nên (Q) có dạng:
Với
Với

Câu 18. Trong không gian với hệ tọa độ Oxyz cho hai mặt phẳng
 P  : x  y  z  6  0 , mặt phẳng (Q) : 2x  y  2z  1  0 và đường thẳng
x 2 y 3 z 4
D:   . Tìm điểm M thuộc D , N thuộc mặt phẳng (P) sao cho MN
1 1 1
vuông góc với mặt phẳng (Q) và MN = 3

NGUYỄN VĂN LỰC  0933.168.309 SP Toán K35 - ĐH Cần Thơ


PP tọa độ trong không gian FB: http://www.facebook.com/VanLuc168

VTPTn Q  (2;1; 2)


M  D  M  2  t;3  t; 4  t 
MN   Q   MN  kn Q   2k; k; 2k   N  2k  t  2; k  t  3; 2k  t  4 
N   P   k  t  3
MN  3  k 2  1  k  1
k  1  t  4 : M  6; 1;0  ; N(8;0; 2)
k  1  t  2 : M  4;1; 2  ; N  2;0; 4 

3. Phương trình mặt phẳng


Câu 19. Trong không gian Oxyz , cho điểm A( 3;2; 3) và hai đường thẳng
x -1 y + 2 z - 3 x - 3 y -1 z - 5
d1 : = = và d 2 : = =
1 1 -1 1 2 3

a/ Chứng minh rằng d1 và d2 cắt nhau.

b/ Viết phương trình mặt phẳng (P) chứa d1 và d2 . Tính khoảng cách từ A đến mp(P).

a/  d1 đi qua điểm M1(1; 2; 3) , có vtcp u1 (1;1; 1)


 d2 đi qua điểm M 2 (3;1;5) , có vtcp u2 (1;2; 3)
1 1 1 1 1 1
 Ta có [u1, u2 ] ; ; (5; 4;1)
2 3 3 1 1 2

và M1M 2 (2; 3;2)

 Suy ra, [u1, u2 ].M1M 2 5.2 4.3 1.2 0 , do đó d1 và d2 cắt nhau.


b/ Mặt phẳng (P) chứa d1 và d2 .
 Điểm trên (P): M1(1; 2; 3)
 vtpt của (P): n [u1, u2 ] (5; 4;1)
 Vậy, PTTQ của mp(P) là: 5(x 1) 4(y 2) 1(z 3) 0
5x 4y z 16 0
 Khoảng cách từ điểm A đến mp(P) là:
5.( 3) 4.2 ( 3) 16 42
d (A,(P )) 42
2 2 2 42
5 ( 4) 1

Câu 20. Trong không


gian với hệ tọa độ Oxyz, cho mặt cầu
 S  : x  y  z  4 x  2 y  4 z  7  0 và mặt phẳng (α) : x - 2y + 2z + 3 = 0
2 2 2

a. Tính khoảng cách từ tâm I của mặt cầu (S) tới mặt phẳng (α).
b. Viết phương trình mặt phẳng (β) song song với mặt phẳng (α) và tiếp xúc với mặt cầu
(S).

a. (S) có tâm I(2;-1;-2) và bán kính R=4


NGUYỄN VĂN LỰC  0933.168.309 SP Toán K35 - ĐH Cần Thơ
PP tọa độ trong không gian FB: http://www.facebook.com/VanLuc168

Do đó d(I,(  ))=1

b. Viết phương trinh mặt phẳng (β) song song với mặt phẳng (α) và tiếp xúc với mặt cầu
(S).
Vì mặt phẳng (β) song song với mặt phẳng (α) nên pt của (β) có dạng
x-2y+2z+D=0
Ta có d(I, (β))=R
D  D  12
 4 
3  D  12
Vậy (β) có pt là x-2y+2z+12=0 hoặc x-2y+2z-12=0

Câu 21. Trong không gian tọa độ Oxyz, cho hai điểm A 1;3;  1 , B   1;1;3 và đường
x y 1 z  2
thẳng d có phương trình   . Viết phương trình mặt phẳng trung trực của đoạn
2 1 1
AB và tìm điểm C trên đường thẳng d sao cho CAB là tam giác cân tại C.

Tọa độ trung điểm M của đoạn AB: M  0; 2; 1 , AB    2;  2; 4 


Mặt phẳng trung trực (P) của đoạn AB đi qua M, nhận n  1; 1;  2  làm VTPT nên có
phương trình:
x  y  2  2  z  1  0  x  y  2 z  0
CAB cân tại C  CA  CB  C   P 
 x y 1 z  2
 
Vậy C là giao điểm của d với (P), tọa độ C là nghiệm:  2 1 1  C   6; 4;  1
x  y  2z  0

Câu 22. Trong không gian với hệ tọa độ Oxyz, cho mặt cầu
x y 1 z  2
 S  : x 2  y 2  z 2  4 x  2 y  4 z  7  0 , đường thẳng d :  
1 2 1
a. Viết phương trình mặt phẳng (P) vuông góc với đường thẳng d và tiếp xúc với mặt cầu
(S).
b. Viết phương trình đường thẳng đi qua tâm của mặt cầu (S), cắt và vuông góc với
đường thẳng d.

d có một vtcp u  (1; 2; 1) , (S) có tâm I(2;-1;-2) và bán kính R=4
Vì (P) vuông góc với d nên (P) nhận u  (1; 2; 1) làm vtpt .Do đó pt của (P) có dạng
x+2y-z+D=0
Mặt khác (P) tiếp xúc với (S) nên ta có
2 D  D  2  4 6
d(I,(P))=R 4
6  D  2  4 6
Vậy pt của (P) là x+2y-z-2+ 4 6 =0 hoặc x+2y-z-2- 4 6 =0
 xt

Pt của d được viết dưới dạng tham số  y  1  2t
 z  2t

NGUYỄN VĂN LỰC  0933.168.309 SP Toán K35 - ĐH Cần Thơ


PP tọa độ trong không gian FB: http://www.facebook.com/VanLuc168

Gọi d’ là đt cần tìm,và H(t ;1+2t ;2-t) là giao điểm của d và d’


Ta có IH  (t  2; 2  2t; 4  t )
Và IH .u  0 t-2+2(2+2t)-(4-t)=0t=1/3
Vậy H(1/3 ;5/3 ;5/3)
Do đó d’ đi qua 2 điểm I(2;-1;2) và H(1/3 ;5/3 ;5/3)
 x  2  5t

Vậy pt đt cần tìm  y  1  8t
 z  2  11t

Câu 23. Trong không gian tọa độ Oxyz , cho A 1;1;1 , B  2;1;0  , C  2;0; 2  . Viết phương
trình mặt phẳng   đi qua hai điểm B, C và cách A một khoảng lớn nhất.

Lập luận để được mặt phẳng cần tìm là mặt phẳng cần tìm là mặt phẳng qua BC và vuông
góc với (ABC)
BC   0; 1;2 , AB  1;0; 1 .Vectơ pháp tuyến của (ABC) là: n ABC    BC , AB   1; 2;1

Suy ra VTPT của   là : n   BC , n ABC     5; 2;1


Pt   : 5 x  2 y  z  8  0

Câu 24. Trong không gian với hệ tọa độ Oxyz cho hai điểm A 1; 1; 2  , B  3;0; 4  và mặt
phẳng (P) : x  2 y 2 z  5  0 . Tìm tọa độ giao điểm của đường thẳng AB và mặt phẳng (P).
Viết phương trình mặt phẳng chứa đường thẳng AB và vuông góc với mặt phẳng (P).

AB   2;1; 6  là vtcp của đường thẳng AB.

 x  1  2t

Ptts AB:  y  1  t t  R 
 z  2  6t

Gọi M là giao điểm của AB và (P). Khi đó M 1  2t; 1  t; 2  6t  .


1
M  (P)  1  2t   2  1  t   2  2  6t   5  0  t 
6

4 5 
 M  ;  ;1
3 6 

Vtpt nQ    AB, n P     10; 10; 5 .

 Q  : 2 x  2 y  z  2  0.

Câu 25. Trong không gian với hệ trục Oxyz, cho mặt cầu (S) có phương trình:
x2 + y2 + z2 – 2x + 4y + 2z – 3 = 0 và mặt phẳng (P): 2x – y + 2z – 14 = 0. Viết phương
trình mặt phẳng (Q) chứa trục Ox và cắt mặt cầu (S) theo một đường tròn có bán kính
bằng 3.

(S) có tâm I(1; –2; –1), bán kính R = 3. (Q) chứa Ox  (Q): ay + bz = 0.
NGUYỄN VĂN LỰC  0933.168.309 SP Toán K35 - ĐH Cần Thơ
PP tọa độ trong không gian FB: http://www.facebook.com/VanLuc168

Mặt khác đường tròn thiết diện có bán kính bằng 3 cho nên (Q) đi qua tâm I.
Suy ra: –2a – b = 0  b = –2a (a  0)  (Q): y – 2z = 0.

Câu 26. Trong không gian với hệ tọa độ Oxyz, cho hai điểm A(2; 1;2), B(0; 0;2) và
x 3 y 6 z 1
đường thẳng d : . Viết phương trình mặt phẳng (P) đi qua A và
2 2 1
vuông góc với d và phương trình mặt cầu có tâm B, tiếp xúc với (P).

Véc tơ chỉ phương của d là u  (2; 2;1)


(P)  d  (P) nhận u  (2; 2;1) là véc tơ pháp tuyến
 Phương trình của (P) : 2( x  2)  2( y  1)  ( z  2)  0  2 x  2 y  z  4  0
Gọi (S) là mặt cầu tâm B, có bán kính là R
Ta có (S) tiếp xúc với (P) nên ta có R  d (B;(P))  2
 phương trình mặt cầu (S): x 2  y 2  ( z  2) 2  4

Câu 27. Trong không gian Oxyz ,cho điểm M(0;2;0) và hai đường thẳng d1 ; d 2 có
x 1 y  2 z 1 x  3 y 1 z
phương trình: d1 :   ; d2 :   . Viết phương trình mặt
2 2 1 2 2 1
phẳng (P) đi qua M , song song với trục Ox , sao cho (P) cắt hai đường thẳng d1 ; d2 lần
lượt tại A, B sao cho AB = 1 .

Giả sử có mặt phẳng (P) thỏa yêu cầu đề bài


A  d1  A 1  2t ;2  2t ; 1  t 
B  d 2  B  3  2l ; 1  2l ; l 
AB   2(l  t )  2; 2(l  t )  3;(l  t )  1
l  t  1
AB  9(l  t )  22(l  t )  14  1  
2 2
13
l  t  
 9
*l  t  1
 AB   0; 1;0   VTPT n( P )   AB; i   (0;0;1)
Pt mặt phẳng (P): z = 0 ( loại vì (P) chứa Ox)
*l  t  13 / 9
 8 1 4   4 1
 AB   ; ;   VTPT n ( P )   AB; i    0;  ; 
 9 9 9   9 9
Pt mặt phẳng (P): - 4 y + z + 8 = 0 ( thỏa đề bài nhận)

NGUYỄN VĂN LỰC  0933.168.309 SP Toán K35 - ĐH Cần Thơ


PP tọa độ trong không gian FB: http://www.facebook.com/VanLuc168

Câu 28. Trong không gian với hệ tọa độ Oxyz, cho mặt cầu (S):
x 2  y 2  z 2  2 x  4 y  6 z  2  0 và mặt phẳng (P): x + y + z + 2015 = 0
a) Xác định tọa độ tâm I và tính bán kính của mặt cầu (S). Viết phương trình đường thẳng
qua I và vuông góc với mặt phẳng (P)
b) Viết phương trình mặt phẳng (Q) song song mặt phẳng (P) và tiếp xúc (S)

(S): x 2  y 2  z 2  2 x  4 y  6 z  2  0 và (P): x + y + z + 2015 = 0


a) (S) có tâm I(1; -2; 3) và R = 4
x  1  t
(D) qua I(1; -2; 3) và có VTCP u = (1; 1; 1;) có ptts :  y  2  t
z  3  t

b) (Q)// (P) => (Q): x + y + z + D = 0 (D  2015)
d  I ,  Q    4  D  2  4 3
Vậy (Q) : x + y + z 2  4 3  0

x 1 y  3 z
Câu 29. Trong không gian với hệ tọa độ Oxyz, cho hai đường thẳng 1 :  
2 3 2
x 3 y z 2
và  2 :   . Tìm tọa độ giao điểm của  1 và  2 và viết phương trình mặt
6 4 5
phẳng (P) sao cho đường thẳng  2 là hình chiếu vuông góc của đường thẳng  1 lên mặt
phẳng (P).

Viết lại  1 và  2 dưới dạng tham số

Giải hệ phương trình tìm được giao điểm A(3; 0; 2)

Đường thẳng  1 có VTCP u1   2; 3;2


Đường thẳng  2 có VTCP u2   6; 4; 5

Gọi (Q) là mặt phẳng chứa 1 ,  2 thì (Q) có VTPT là n  u1 , u2   (7; 22; 26)

Vì  2 là hình chiếu vuông góc của đường thẳng  1 lên mặt phẳng (P)  (P) chứa  2 và
( P)  (Q)
Do đó (P) cũng đi qua A và có VTPT là n1   n , u2   (214;191; 104)
(P) có phương trình là: 214 x  191 y  104 z  850  0

Câu 30. Trong không gian Oxyz cho các điểm A  2;3;0  , B  0;1  2  , C 1, 4, 1 .Viết phương
trình mặt phẳng (P) chứa B, C và song song với đường thẳng OA. Tính khoảng cách từ
điểm A đến đường thẳng BC.

n  BC  1;3;1
Theo đề bài mặt phẳng (P) có VTPT   n   BC ; OA   3; 2; 3
n  OA   2;3;0 
NGUYỄN VĂN LỰC  0933.168.309 SP Toán K35 - ĐH Cần Thơ
PP tọa độ trong không gian FB: http://www.facebook.com/VanLuc168

mp(P)có VTPT n và qua B suy ra


 P  :  3  x  0   2  y  1  3  z  2   0
 3x  2 y  3z  8  0
 AB, AC    4;0; 4   S ABC  2 2
 
2S 4 2 4 22
d  A, BC   ABC  
BC 11 11

Câu 31. Trong không gian Oxyz cho điểm A(3; -2; -2) và mặt phẳng
 P : x  y  z 1  0 .
a) Viết phương trình mặt cầu (S) có tâm A và tiếp xúc với mp (P).
b) Viết phương trình mặt phẳng (Q) đi qua A, vuông góc với mp (P) biết rằng mp (Q)
cắt hai trục Oy, Oz lần lượt tại điểm phân biệt M và N sao cho OM = ON.

8
a) Vì (S) có tâm A và tiếp xúc (P) nên bán kính của (S) là R = d(a, (P)) = .Vậy pt của
3
64
(S) là: ( x  3) 2  ( y  2) 2  ( z  2) 2 
3
b) Gọi nQ là VTPTcủa (Q), n P = (1;-1;-1) là VTPT của (P). Khi đó nQ  nP Mp(Q) cắt
hai trục Oy và Oz tại M  0; a;0  , N  0;0; b  phân biệt sao cho
a  b  0
OM = ON nên a  b  
 a  b  0

+ a = b thì MN   0; a; a  nQ  u => nQ  u, nP    2;1;1 . Khi đó mp


u  0; 1;1 và
(Q): 2 x  y  z  2  0 và M  0; 2;0  ; N  0;0; 2  (thỏa mãn)

+ a = - b thì MN   0; a; a  u  0;1;1 và nQ  u => nQ  u , nP    0;1; 1

Khi đó mp (Q): y  z  0 và M  0;0;0  và N  0;0;0  (loại).


Vậy  Q  : 2 x  y  z  2  0 .

Câu 32. Trong không gian với hệ tọa độ Oxyz cho điểm A(10; 2; -1) và đường thẳng d có
x 1 y z 1
phương trình   . Lập phương trình mặt phẳng (P) đi qua A, song song với d
2 1 3
và khoảng cách từ d tới (P) là lớn nhất.

Gọi H là hình chiếu của A trên d, mặt phẳng (P) đi qua A và (P)//d, khi đó khoảng
cách giữa d và (P) là khoảng cách từ H đến (P).

Giả sử điểm I là hình chiếu của H lên (P), ta có AH  HI => HI lớn nhất khi A  I
Vậy (P) cần tìm là mặt phẳng đi qua A và nhận AH làm véc tơ pháp tuyến.
H  d  H (1  2t ; t ;1  3t ) vì H là hình chiếu của A trên d nên
AH  d  AH .u  0 (u  (2;1;3) là véc tơ chỉ phương của d)  H (3;1;4)  AH (7;1;5) Vậy
(P): 7(x – 10) + (y – 2) – 5(z + 1) = 0
 7x + y -5z -77 = 0
NGUYỄN VĂN LỰC  0933.168.309 SP Toán K35 - ĐH Cần Thơ
PP tọa độ trong không gian FB: http://www.facebook.com/VanLuc168

Câu 33. Trong không gian với hệ tọa độ Oxyz , cho mặt cầu ( S ) có phương trình
x2  y 2  z 2  4 x  6 y  2 z  2  0 . Lập phương trình mặt phẳng ( P) chứa truc Oy và cắt
mặt cầu ( S ) theo một đường tròn có bán kính r  2 3 .

(S ) : x2  y 2  z 2  4 x  6 y  2 z  2  0  ( x  2)2  ( y  3)2  ( z  1)2  16


 ( S ) có tâm I (2; 3;1) bán kính R  4 ; trục Oy có VTCP j  (0;1;0)
Gọi n  (a; b; c) là VTPT mp(P) ,
( P) chứa Oy  n  j  b  0  n  (a;0; c) (a 2  c 2  0)
Phương trình mp(P): ax  cz  0
(P) cắt mặt cầu (S) theo đường tròn có bán kinh r  2 3
2a  c
 d  I ,( P)  R 2  r 2  2   2  4a 2  4ac  c 2  4a 2  4c 2
a2  c2
c  0
 3c 2  4ac  0  
3c  4a
Vậy phương trình mp(P) : x  0 hoặc 3x  4 z  0 .

Câu 34. Trong không gian với hê ̣ to ̣a đô ̣ Oxyz, cho điể m M(1;-1;1) và hai đường thẳng
x y 1 z x y 1 z  4
(d ) :   và (d ') :   . Chứng minh: điể m M, (d), (d’) cùng nằ m trên
1 2 3 1 2 5
mô ̣t mă ̣t phẳ ng. Viết phương trình mă ̣t phẳ ng đó.

*(d) đi qua M 1 (0; 1; 0) và có vtcp u1  (1; 2; 3)


(d’) đi qua M 2 (0;1; 4) và có vtcp u 2  (1; 2;5)
*Ta có u1; u 2   (4; 8; 4)  O , M1M 2  (0; 2; 4)
Xét u1; u 2  .M1M 2  16  14  0
 (d) và (d’) đồng phẳng .
*Gọi (P) là mặt phẳng chứa (d) và (d’) => (P) có vtpt n  (1; 2; 1) và đi qua M1 nên có
phương trình x  2y  z  2  0
*Dễ thấy điểm M(1;-1;1) thuộc mf(P) , từ đó ta có đpcm

Câu 35 Cho mặt cầu (S): x 2  y 2  z 2  2 x  6 y  8 z  1  0 .


a) Xác định tọa độ tâm I và bán kính r của mặt cầu (S).
b) Viết phương trình mp(P) tiếp xúc với mặt cầu tại M(1;1;1).

 2 a  2 a  1
 2b  6 b   3
 
a) Từ phương trình mặt cầu ta có:  
 2 c  8 c  4
 d  1  d  1
Tọa độ tâm I(1; -3; 4).
Bán kính: r  1  9  16  1  5

b) Mặt phẳng tiếp xúc mặt cầu tại M nên IM vuông với mp.
NGUYỄN VĂN LỰC  0933.168.309 SP Toán K35 - ĐH Cần Thơ
PP tọa độ trong không gian FB: http://www.facebook.com/VanLuc168

IM  (0; 4; 3)
Mp(P) qua M(1;1;1), có VTPT IM  (0; 4; 3) có phương trình:
A( x  x0 )  B( y  y0 )  C ( z  z0 )  0
 0( x  1)  4( y  1)  3( z  1)  0  4 y  3z  1  0

Câu 36. Trong không gian to ̣a đô ̣ Oxyz cho hai điể m A(1; 1; 1), B(2; 2; 2), mă ̣t phẳ ng
(P): x + y  z + 1 = 0 và mă ̣t cầ u (S): x2 + y2 + z2  2x + 8z  7 = 0. Viế t phương triǹ h
mă ̣t phẳ ng (Q) song song với đường thẳ ng AB, vuông góc với mă ̣t phẳ ng (P) và cắ t (S)
theo mô ̣t đường tròn (C) sao cho diê ̣n tić h hiǹ h tròn (C) bằ ng 18.

Mp(Q) // AB, (Q)  (P), cắ t (S) theo đường tròn có bán kính 3 2 .
Ta có x2 + y2 + z2  2x + 8z  7 = 0  (x 1)2 + y2 + (z +4)2 = 24.
Suy ra (S) có tâm I(1 ; 0 ;  4), bán kiń h R = 2 6 .
Go ̣i n P , nQ lầ n lươṭ là vecto pháp tuyế n của mp(P), mp(Q). Ta có
n P = (1; 1; 1), AB = (1; 3; 1), [ n P , AB ] = (4;  2; 2)  0 .
(Q ) / / AB nQ  AB 1
Ta có   nên có thể cho ̣n nQ = [ n P , AB ]
(Q )  ( P)  nQ  n P 2

Hay nQ = (2; 1; 1). Suy ra pt mp(Q): 2x  y + z + d = 0


Gọi r, d lầ n lươ ̣t là bán kin
́ h của (C), khoảng cách từ tâm I của (S) đế n mp(Q).
Ta có diê ̣n tić h hình tròn (C) bằ ng 18 nên r2 = 18.
Do đó d2 = R2  r2 = 24  18 = 6  d = 6 .
Ta có d = 6  |d 2| = 6  d = 8 hoă ̣c d =  4.
Từ đó, có 2 mp là (Q1): 2x  y + z + 8 = 0, (Q2): 2x  y + z  4 = 0
Mp(Q) có pt trên có thể chứa AB.
Kiể m tra trực tiế p thấ y
A(1; 1; 1)  (Q1) nên AB // (Q1); A(1; 1; 1)  (Q2) nên AB  (Q2).
KL: pt mp(Q): 2x  y + z + 8 = 0.

4. Phương trình mặt cầu

Câu 37. Trong không gian với hệ tọa độ Oxyz, cho hai điểm A(0; 0; 3), B(2; 0; 1) và mặt
phẳng ( P) : 3 x  y  z  1  0 . Viết phương trình mặt cầu (S) có tâm nằm trên đường thẳng
AB, bán kính bằng 2 11 và tiếp xúc với mặt phẳng (P).

Đường thẳng AB đi qua A(0;0;-3) có VTCP AB  (2;0; 2)


 x  2t

Nên phương trình tham số của đường thẳng AB là:  y  0
 z  3  2t

Gọi I là tâm của mặt cầu thì I(2t;0;-3+2t).
NGUYỄN VĂN LỰC  0933.168.309 SP Toán K35 - ĐH Cần Thơ
PP tọa độ trong không gian FB: http://www.facebook.com/VanLuc168

Mặt phẳng (P) tiếp xúc với mặt cầu (S) khi và chỉ khi:
6t   3  2t   1
d ( I ;( P))  2 11   2 11
11
 9
 t
 4t  4  22 2
 4t  4  22   
 4t  4  22 t   13
 2
9
t   I (9;0;6) . Phương trình mặt cầu ( S ) : (x  9) 2  y 2  (z  6) 2  44
2
13
t    ( I  13;0; 16) Phương trình ( S )  (x  13) 2  y 2  (z  16) 2  44
2

Câu 38. Trong không gian Oxyz, cho 4 điểm A( 1;1;1), B(5;1; 1), C (2;5;2), D(0; 3;1) . Viết
phương trình mặt cầu (S) có tâm là điểm D, tiếp xúc với mặt phẳng (ABC).Viết phương
trình mp tiếp diện với mặt cầu (S) song song với mp(ABC).

Ta có AB (6; 0; 2) , AC (3; 4;1)


0 2 2 6 6 0
vtpt của mp(ABC): n [AB, AC ] ; ; (8; 12;24)
4 1 1 3 3 4

 PTTQ của mp(ABC): 8(x 1) 12(y 1) 24(z 1) 0


8x 12y 24z 4 0 2x 3y 6z 1 0
- Mặt cầu (S ) có tâm D, tiếp xúc mp(ABC)
Tâm của mặt cầu: A(0; 3;1)
2.0 3.( 3) 6.1 1 14
 Bán kính mặt cầu: R d (D,(ABC )) 2
2 2
( 3) 2
62 7
 Phương trình mặt cầu (S ) : x (y 3) (z 1) 4
2 2 2

 Gọi (P) là tiếp diện của (S ) song song với mp(ABC) thì (P) có phương trình
2x 3y 6z D 0 (D 1)
2.0 3.( 3) 6.1 D
 Vì (P) tiếp xúc với (S ) nên d(I ,(P )) R 2
2 2 2
2 ( 3) 6
15 D 14 D 1 (loai)
15 D 14
15 D 14 D 29(nhan)
 Vậy, phương trình mp(P) cần tìm là: 2x 3y 6z 29 0

Câu 39. Trong không gian với hệ tọa độ ÕOxyz , cho đường thẳng :
x

y 3

z6
1 1 1
và hai mặt phẳng  P  : x  2 y  2 z  6  0 , Q  : 2 x  y  2 z  7  0 . Viết phương trình mặt
cầu  S  có tâm thuộc  đồng thời tiếp xúc với hai mặt phẳng  P  , Q  .

Gọi I là tâm mặt cầu  S  , khi đó I  t;3  t; 6  t  .


5t  12 5t  8 5t  12 5t  8
d  I ;( P)   , d  I ;(Q )   , theo giả thiết 
3 3 3 3

NGUYỄN VĂN LỰC  0933.168.309 SP Toán K35 - ĐH Cần Thơ


PP tọa độ trong không gian FB: http://www.facebook.com/VanLuc168

2
 t  2  I  2;1; 4  , R  .
3
4
Mặt cầu  S  :  x  2    y  1   z  4  
2 2 2
.
9

Câu 40. Trong không gian với hệ tọa độ Oxyz , cho điểm A(1;3;2) , đường thẳng
x 1 y  4 z
d:   và mặt phẳng ( P) : 2 x  2 y  z  6  0 . Tìm tọa độ giao điểm của d với
2 1 2
(P) và viết phương trình mặt cầu (S) đi qua A, có tâm thuộc d đồng thời tiếp xúc với (P).

 x  1  2t

d có phương trình tham số  y  4  t .
 z  2t

Gọi B  d  (P) , do B  d nên B(1  2t ;4  t ;2t )
Do B  (P ) nên 2(1  2t )  2(4  t )  2t  6  0  t  4  B(7;0;8)
Gọi I là tâm mặt cầu (S), do I thuộc d nên I (1  2a;4  a;2a)
Theo bài ra thì (S) có bán kính R  IA  d ( I , ( P ))
2(1  2a)  2(4  a)  2a  6
 (2  2a) 2  (a  1) 2  (2  2a) 2 
22  22  12
4a  16 35
 9a 2  2a  9   9(9a 2  2a  9)  (4a  16) 2  65a 2  110a  175  0  a  1; a   .
3 13
+) Với a  1  I  (1;3;2), R  4  ( S ) : ( x  1)  ( y  3)  ( z  2)  16
2 2 2

2 2 2

+) Với a    I    ; ; ; R  
35 83 87 70 116 83   87   70  13456
 (S ) :  x     y     z   
13  13 13 13  13  13   13   13  169

Câu 41. Trong không gian tọa độ Oxyz, cho mp(P): x + y + z – 3 = 0 và hai đường thẳng
x 1 y  2 z 1 x  2 y 1 z 1
d1 :   ; d2 :  
2 1 1 1 2 5
Viết phương trình mặt cầu có tâm thuộc d1, tiếp xúc với d2 và cắt mp(P) theoo một
đường tròn có bán kính r = 3 ,biết rằng tâm mặt cầu có cao độ dương

d2 qua A(2;1;-1) có vtcp ud  1;2;5 2

 I  d1  I  1  2t ; 2  t ;1  t 
 AI   2t  3; t  3; 2  t  ,  AI , ud    7t  19; 11t  17;3t  3
2

 AI , ud  179t 2  658t  659


 2 
 d I ,d 2   
ud 2 30

179t 2  658t  659


 d2 tiếp xúc với (S) nên d I ,d   R  2
 R(1)
30
2t  5
 d I , P  
3
2
 2t  5  4t 2  20t  34 4t 2  20t  34
 Ta có: R  d 2 2
 I ,P r  R 
2 2
  3  R 2
  R  (2)
 3  3 3
NGUYỄN VĂN LỰC  0933.168.309 SP Toán K35 - ĐH Cần Thơ
PP tọa độ trong không gian FB: http://www.facebook.com/VanLuc168

t  1
179t 2  658t  659 4t 2  20t  34
 Từ (1) và (2), ta có:   139t  458t  319  0   319
2

30 3 t 
 139
599 41 180 
Suy ra: I(1;-1;0) (nhận) hoặc I  ; ;  (loại do zI > 0)
 139 139 139 
 Với I(1;-1;0)  R  6   S  :  x  1   y  1  z 2  6
2 2

Kết luận: phương trình mặt cầu cần tìm là  S  :  x  12   y  12  z 2  6

Câu 42. Trong không gian với hệ tọa độ Oxyz, cho điểm A(2;1;1) và mặt phẳng
 P  : 2 x – y  2 z  1  0 . Viết phương trình mặt cầu tâm A tiếp xúc với mặt phẳng (P) và tìm
tọa độ các giao điểm của mặt cầu đó với trục Ox.

4 1 2  1
+) Mặt cầu tâm A tiếp xúc với mặt phẳng (P) có bán kính R  d ( A, ( P))  2
2 2 2
2 1  2
+) Phương trình mặt cầu là: (x – 2) + (y – 1) + (z – 1) = 4.
2 2 2

+) Tọa độ giao điểm của mặt cầu và trục Ox là nghiệm của hệ pt:
( x  2)2  ( y  1)2  ( z  1) 2  4
 x  2  2
y  0 
z  0  x  2  2

+) Các giao điểm: M (2  2;0;0), N (2  2;0;0).

Câu 43. Trong không gian với hệ trục 0xyz, cho hai điểm A(1; -2; 3), B(-1; 0; 1) và mặt
phẳng (P): x + y + z + 4 = 0. Tìm tọa độ hình chiếu vuông góc của A trên (P) và viết
phương trình mặt cầu (S) có tâm thuộc đường thẳng AB, bán kính bằng 1 và tiếp xúc với
mặt phẳng (P).

+ Gọi H là hình chiếu vuông góc của A lên (P), AH   x  1; y  2; z  3

x  1  t
AH có ptts là :  y  2  t
z  3  t

+ H  AH nên H 1  t; 2  t ;3  t 
Mặt khác: H∈ (P) nên suy ra: 1  t  2  t  3  t  4  0  t  2
Vậy H(-1;-4;1)
 x  1  2t

+ Đường thẳng AB có ptts là :  y  2  2t
 z  3  2t

+Gọi I là tâm mặt cầu (S):


I  AB  I(1  2t ; 2  2t ;3  2t )

Mặt phẳng (P) tiếp xúc với (S) có bán kính R=1 nên : d(I,(P))=1
NGUYỄN VĂN LỰC  0933.168.309 SP Toán K35 - ĐH Cần Thơ
PP tọa độ trong không gian FB: http://www.facebook.com/VanLuc168

 3
t  3 
2
 6  2t  3  
 3
t  3 
 2
Vậy có hai phương trình mặt cấu cần tìm là :

     
2 2 2
x 5 3  y 4 3  z 3 3 1

     
2 2 2
và x  5  3  y  4  3  z  3  3  1.

Câu 44. Trong không gian Oxyz, cho hình lăng trụ tam giác ABC.A'B'C' có A(1;1; 1),
B(1; 2; 1), C(1; 1; 2) và A'(2; 2; 1). Tìm tọa độ các đỉnh B', C' và viết phương trình mặt
cầu đi qua bốn điểm A, B, C, A'.

- Do ABC.A'B'C' là hình lăng trụ nên BB '  AA '  B '  2;3;1

Tương tự: CC '  AA '  C '  2; 2; 2

- Gọi phương trình mặt cầu (S) cần tìm dạng

x 2  y 2  z 2  2ax  2by  2cz  d  0, a 2  b 2  c 2  d  0

Do A, B, C và A' thuộc mặt cầu (S) nên:

 2a  2b  2c  d  3
 2a  4b  2c  d  3
  6 a  b  c  
  2
 2a  2b  4c  d  6  d  6
 4a  4b  2c  d  9

- Do đó phương trình mặt cầu (S): x 2  y 2  z 2  3x  3 y  3z  6  0

Câu 45. Trong không gian với hệ tọa độ Oxyz, cho ba điểm A(2;1;-3), B(4;3;-2),
C(6;-4;-1). Chứng minh rằng A, B,C là ba đỉnh của một tam giác vuông và viết phương
trình mặt cầu tâm A đi qua trọng tâm G của tam giác ABC.

2 2
Ta có: AB(2; 2;1); AC (4; 5; 2)    AB; AC không cùng phương  A; B; C lập
4 5

thành tam giác. Mặt khác: AB.AC  2.4  2.(5)  1.2  0  AB  AC suy ra ba điểm A; B; C là
ba đỉnh của tam giác vuông.

Vì G là trọng tâm của tam giác ABC nên G(4;0; -2). Ta có: AG  6
NGUYỄN VĂN LỰC  0933.168.309 SP Toán K35 - ĐH Cần Thơ
PP tọa độ trong không gian FB: http://www.facebook.com/VanLuc168

Mặt cầu cần tìm có tâm A và bán kính AG  6 nên có pt: ( x  2)2  ( y  1) 2  ( z  3) 2  6

Câu 46. Trong không gian với hệ tọa độ Oxyz cho 4 điểm A(1; 1; 0); B(1; 0; 2);
C(2;0; 1), D(-1; 0; -3). Chứng minh A, B, C, D là 4 đỉnh của một hình chóp và viết
phương trình mặt cầu ngoại tiếp hình chóp đó .

Ta có AB  (0; 1;2); AC  (1; 1;1); AD  (2; 1;  3) .


 AB , AC   1; 2;1 ;  AB , AC  . AD  7
   
Do  AB , AC  . AD  7  0 , nên 3 véc tơ AB , AC , AD không đồng phẳng suy ra A, B, C, D
là 4 đỉnh của một hình chóp.
Gọi phương trình mặt cầu có dạng x 2  y 2  z 2  2ax  2by  2cz  d  0
( với a 2  b 2  c 2  d  0 ).
 2a  2b  d  2
 2 a  4 c  d  5
Do mặt cầu đi qua 4 điểm A, B, C, D nên ta có hệ 
 4 a  2 c  d  5
 2a  6c  d  10
5 31 5 50
Giải hệ suy ra a  ; b  ; c  ; d  
14 14 14 7
5 31 5 50
Vậy phương trình mc là: x 2  y 2  z 2  x  y  z   0 .
7 7 7 7

Câu 47. Trong không gian với hệ tọa độ Oxyz , cho hai điểm A  3;0; 4  , B 1;0;0  . Viết
phương trình mặt cầu đường kính AB và tìm điểm M trên tia Oy sao cho MA  MB 13 .

+ Gọi  S  là mặt cầu có đường kính AB và I là trung điểm của AB.

Ta có I  1;0;2 , AB  4 2
AB
Khi đó mặt cầu  S  có tâm I và có bán kính R   2 2 nên có phương trình
2

 x  1  y 2   z  2  8
2 2

+ M  Oy  M  0; t;0 
khi đó

MA  MB 13   3   t   42  12   t   02 . 13  25  t 2  13 1  t 2   t  1
2 2 2

Với t  1  M  0;1;0 

t  1  M  0; 1;0 

NGUYỄN VĂN LỰC  0933.168.309 SP Toán K35 - ĐH Cần Thơ


PP tọa độ trong không gian FB: http://www.facebook.com/VanLuc168

Câu 48. Trong không gian với hệ trục toạ độ Oxyz , cho mặt phẳng P  : x  y  2z  1  0
và hai điểm A 2; 0; 0  , B  3; 1;2  . Viết phương trình mặt cầu S  tâm I thuộc mặt phẳng
P  và đi qua các điểm A, B và điểm gốc toạ độ O .

Giả sử I  x , y, z  . Ta có I  P   x  y  2z  1  0 1
x  y  2z  5
Do A, B,O  S   IA  IB  IO . Suy ra  2 
x  1
x  y  2z  1  0 x  1
 
Từ (1) và (2) ta có hệ x  y  2z  5  y  2  I 1; 2;1  
x  1 z  1
 
Bán kính mặt cầu (S) là R  IA  6
Vậy phương trình mặt cầu (S) là: x  1  y  2   z  1  6
2 2 2

NGUYỄN VĂN LỰC  0933.168.309 SP Toán K35 - ĐH Cần Thơ


Tổ hợp – Xác suất FB: http://www.facebook.com/VanLuc168

I. KIẾN THỨC CƠ BẢN


Chuyên đề: Tổ hợp & Xác suất

1.1. Đại số tổ hợp


1.1.1. Quy tắc cộng:
Có n1 cách chọn đối tượng A1.
n2 cách chọn đối tượng A2.
A1  A2 = 
 Có n1 + n2 cách chọn một trong các đối tượng A1, A2.
1.1.2. Quy tắc nhân:
Có n1 cách chọn đối tượng A1. Ứng với mỗi cách chọn A1, có n2 cách chọn đối
tượng A2.
 Có n1.n2 cách chọn dãy đối tượng A1, A2.
1.1.3. Hoán vị:
 Mỗi cách sắp thứ tự n phần tử gọi là một hoán vị của n phần tử.
 Số hoán vị: Pn = n!.
1.1.4. Chỉnh hợp:
 Mỗi cách lấy ra k phần tử từ n phần tử (0 < k  n) và sắp thứ tự của chúng gọi
là một chỉnh hợp chập k của n phần tử.
n!
 Số các chỉnh hợp: Ank 
(n  k )!
1.1.5. Tổ hợp:
 Mỗi cách lấy ra k phần tử từ n phần tử (0  k  n) gọi là một tổ hợp chập k
của n phần tử.
n!
 Số các tổ hợp: Cnk 
k !(n  k )!
 Hai tính chất: Cnk  Cnnk , Cnk11  Cnk1  Cnk
1.1.6. Nhị thức Newton
n
(a  b)n   Cnk a nk b k  Cn0a n  Cn1a n1b  ...  Cnnb n
k 0

 Số hạng tổng quát (Số hạng thứ k + 1): Tk 1  Cnk a nk b k


 Đặc biệt: (1  x) n  Cn0  xCn1  x 2Cn2  ...  x nCnn
1.2. Xác suất
A
1.2.1. Tính xác suất bằng định nghĩa cổ điển: P  A 

+ 0  P(A)  1 + P     1, P    0
NGUYỄN VĂN LỰC  0933.168.309 SP Toán 35 - ĐH Cần Thơ
Tổ hợp – Xác suất FB: http://www.facebook.com/VanLuc168
1.2.2. Tính xác suất theo các quy tắc:
a) Quy tắc cộng xác suất
Nếu A và B là hai biến cố xung khắc, thì:
P  A  B   P  A  P  B 
c) Quy tắc nhân xác suất
Nếu hai biến cố A và B độc lập với nhau thì:
P  AB   P  A  P  B 

II. BÀI TẬP VÍ DỤ


Chuyên đề: Tổ hợp & Xác suất

Ví dụ 1: Một hộp đựng 5 viên bi trắng, 3 viên bi xanh. Lấy ngẫu nhiên 2 bi.
a) Có bao nhiêu cách lấy được 2 bi ?
b) Có bao nhiêu cách lấy được 2 bi trắng ?
c) Có bao nhiêu cách lấy được 1 bi trắng, 1 bi xanh ?

Lời giải
a) Có C82 28 cách lấy
b) Có C52 10 cách lấy
c) Có C51C31 15 cách lấy

Ví dụ 2: Một hộp đựng 5 viên bi trắng, 3 viên bi xanh. Lấy lần lượt 2 bi.
a) Có bao nhiêu cách lấy được 2 bi ?
b) Có bao nhiêu cách lấy được 2 bi trắng ?
c) Có bao nhiêu cách lấy được 1 bi trắng, 1 bi xanh ?

Lời giải
a) Có C81C71 56 cách lấy (hoặc A82 56 )

b) Có C51C41 20 cách lấy (hoặc A52 20 )

c) Có C51C31 C31C51 30 cách lấy


NGUYỄN VĂN LỰC  0933.168.309 SP Toán 35 - ĐH Cần Thơ
Tổ hợp – Xác suất FB: http://www.facebook.com/VanLuc168

Ví dụ 3: Một hộp đựng 5 viên bi trắng, 6 viên bi xanh và 7 viên bi đỏ. Lấy ngẫu
nhiên 4 bi.
a) Có bao nhiêu cách lấy được 4 bi ?
b) Có bao nhiêu cách lấy được 4 bi có đủ cả ba màu ?

Lời giải
a) Có C184 3060 cách lấy
b) Có C52C61C61 C51C62C71 C51C61C72 1575 cách lấy

Ví dụ 4: Có bao nhiêu số tự nhiên chẵn gồm 4 chữ số đôi một khác nhau sao cho
trong mỗi số đều có mặt các chữ số 8 và 9 ?

Lời giải
Giả sử số cần lập là abcd , d {0, 2, 4, 6, 8}. Xét các trường hợp sau
d  0. Số cách lập abc trong đó có các chữ số 8 và 9 là C71 .3!  42.
d  8. Số cách lập abc trong đó có chữ số 9 là C82 .3! C71 .2!  154.
d  {2, 4, 6}. Số cách lập abc trong đó có các chữ số 8 và 9 là  
3. C71 .3! 2  120.

Vậy số các số lập được là 42  154  120  316. 

Ví dụ 5: Có bao nhiêu số tự nhiên gồm 5 chữ số, các chữ số đôi một khác nhau sao
cho chữ số đầu và chữ số cuối của mỗi số đó đều là số chẵn?

Lời giải
+ Chữ số đầu tiên là chữ số chẵn, khác 0 nên có 4 cách chọn.
+ Chữ số tận cùng cũng là chữ số chẵn, khác với chữ số đầu tiên nên cũng có 4 cách
chọn.
+ Ba chữ số ở giữa có số cách sắp xếp là A83 .

Suy ra số các số thỏa mãn yêu cầu bài toán là 4  4  A83  5376. 

NGUYỄN VĂN LỰC  0933.168.309 SP Toán 35 - ĐH Cần Thơ


Tổ hợp – Xác suất FB: http://www.facebook.com/VanLuc168
18
1
Ví dụ 6: Tìm hệ số của số hạng chứa x12 trong khai triển nhị thức Niutơn của x
x2

Lời giải
♥ Khai triển nhị thức Niutơn ta có:
18 18 k 18
1 k 18 k 1 k
x C .x18
. C18k . 1 .x18 3k

x2 k 0 x2 k 0

♥ Chọn k thỏa mãn: 18 3k 12 k 2

♥ Vậy hệ số của số hạng chứa x12 trong khai triển là 


2
1 C182 153 .

Ví dụ 7: Tìm hệ số của số hạng không chứa x trong khai triển nhị thức Niutơn của
18
1
x
x2

Lời giải
♥ Khai triển nhị thức Niutơn ta có:
18 18 k 18
1 k 18 k 1 k
x C .x18
. C18k . 1 .x18 3k

x2 k 0 x2 k 0

♥ Chọn k thỏa mãn: 18 3k 0 k 6

♥ Vậy số hạng không chứa x trong khai triển là 


6
1 C186 18564 .

Ví dụ 8: Tìm hệ số của số hạng chứa x8 trong khai triển nhị thức Niutơn của
n
1
x5 , biết rằng Cnn 1
4
Cnn 3
7 n 3 (1)
x3

Lời giải
♥ Giải phương trình (1) tìm n, ta có:
n 4! n 3!
Cnn 1
4
Cnn 3
7 n 3 7 n 3
n 1 !3! n !3!

n 2 n 3 n 4 n 1 n 2 n 3 42 n 3

n 2 n 4 n 1 n 2 42

3n 6 42 n 12

NGUYỄN VĂN LỰC  0933.168.309 SP Toán 35 - ĐH Cần Thơ


Tổ hợp – Xác suất FB: http://www.facebook.com/VanLuc168
♥ Khai triển nhị thức Niutơn ta có:
12 12 12 k 12 60 11k
1 5 k 1 k
x C . 3
12
. x5 C12k .x 2

x3 k 0 x k 0

60 11k
♥ Chọn k thỏa mãn: 8 k 4
2

♥ Vậy hệ số của số hạng chứa x8 trong khai triển là C124 495 . 

Ví dụ 9: Để kiểm tra chất lượng sản phẩm từ một công ty sữa, người ta đã gửi đến bộ
phận kiểm nghiệm 5 hộp sữa cam, 4 hộp sữa dâu và 3 hộp sữa nho. Bộ phận kiểm
nghiệm chọn ngẫu nhiên 3 hộp sữa để phân tích mẫu. Tính xác suất để 3 hộp sữa
được chọn có cả 3 loại. (Khối A-2014)

Bài giải
♥ Số phần tử của không gian mẫu là: C123 220

♥ Gọi A là biến cố: “3 hộp sữa được chọn có cả 3 loại”


Số kết quả thuận lợi cho biến cố A là: A
C51C41C31 60

60 3
♥ Vậy xác suất cần tính là P A A
. 
220 11

Ví dụ 10: Từ một hộp chứa 16 thể được đánh số từ 1 đến 16, chọn ngẫu nhiên 4 thẻ.
Tính xác suất để 4 thẻ được chọn đều được đánh số chẵn. (Khối B-2014)

Bài giải
♥ Số phần tử của không gian mẫu là: C164 1820

♥ Gọi A là biến cố: “4 thẻ được chọn đều được đánh số chẵn”
Số kết quả thuận lợi cho biến cố A là: A
C84 70

70 1
♥ Vậy xác suất cần tính là P A A
. 
1820 26

NGUYỄN VĂN LỰC  0933.168.309 SP Toán 35 - ĐH Cần Thơ


Tổ hợp – Xác suất FB: http://www.facebook.com/VanLuc168
Ví dụ 11: Có hai chiếc hộp chứa bi. Hộp thứ nhất chứa 4 viên bi đỏ và 3 viên bi trắng,
hộp thứ hai chứa 2 viên bi đỏ và 4 viên bi trắng. Lấy ngẫu nhiên từ mỗi hộp ra 1
viên bi, tính xác suất để hai viên bi được lấy ra có cùng màu.
(Khối B-2013)

Bài giải
♥ Số phần tử của không gian mẫu là: C71C61 42

♥ Gọi A là biến cố: “hai viên bi được lấy ra có cùng màu”


Số kết quả thuận lợi cho biến cố A là: A
C41C21 C31C41 10

20 10
♥ Vậy xác suất cần tính là P A A
. 
42 21

Ví dụ 12: Trong một lớp học gồm có 15 học sinh nam và 10 học sinh nữ. Giáo viên
gọi ngẫu nhiên 4 học sinh lên bảng giải bài tập. Tính xác suất để 4 học sinh được gọi
có cả nam và nữ. (Khối B-2012)

Lời giải
♥ Số phần tử của không gian mẫu là: C254 12650

♥ Gọi A là biến cố: “4 học sinh được gọi có cả nam và nữ”


Số kết quả thuận lợi cho biến cố A là: A
C151 C103 C152 C102 C153 C101 11075

11075 443
♥ Vậy xác suất cần tính là P A A
. 
12650 506

Ví dụ 13: Gọi S là tập hợp tất cả các số tự nhiên gồm ba chữ số phân biệt được chọn
từ các chữ số 1, 2, 3, 4, 5, 6, 7. Chọn ngẫu nhiên 1 số từ S, tính xác suất để số được
chọn là số chẵn. (Khối A-2013)

Bài giải
♥ Số phần tử của không gian mẫu là: A73 210

♥ Gọi A là biến cố: “số được chọn là số chẵn”


Số kết quả thuận lợi cho biến cố A là: A
3.6.5 90
NGUYỄN VĂN LỰC  0933.168.309 SP Toán 35 - ĐH Cần Thơ
Tổ hợp – Xác suất FB: http://www.facebook.com/VanLuc168
90 3
♥ Vậy xác suất cần tính là P A A
. 
210 7

Ví dụ 14: Gọi S là tập hợp tất cả các số tự nhiên gồm bốn chữ số phân biệt được chọn
từ các chữ số 1, 2, 3, 4, 5, 6. Chọn ngẫu nhiên 1 số từ S, tính xác suất để số được
chọn có mặt chữ số 6.

Bài giải
♥ Số phần tử của không gian mẫu là: A64 360

♥ Gọi A là biến cố: “số được chọn có mặt chữ số 6”


Số kết quả thuận lợi cho biến cố A là: A
4. A53 240

240 2
♥ Vậy xác suất cần tính là P A A
. 
360 3

Ví dụ 15: Gọi S là tập hợp tất cả các số tự nhiên gồm ba chữ số phân biệt được chọn
từ các chữ số 1, 2, 3, 4, 5, 6. Chọn ngẫu nhiên 1 số từ S, tính xác suất để số được
chọn có tổng các chữ số bằng 8.

Bài giải
♥ Số phần tử của không gian mẫu là: A63 120

♥ Gọi A là biến cố: “số được chọn có mặt chữ số 6”


Số kết quả thuận lợi cho biến cố A là: A
12

12 1
♥ Vậy xác suất cần tính là P A A
. 
120 10

Ví dụ 16: Cho tập hợp E  1, 2, 3, 4, 5. Gọi M là tập hợp tất cả các số tự nhiên có ít
nhất 3 chữ số, các chữ số đôi một khác nhau thuộc E. Lấy ngẫu nhiên một số
thuộc M. Tính xác suất để tổng các chữ số của số đó bằng 10.

Phân tích
Số các số thuộc M có 3 chữ số là A53  60.
Số các số thuộc M có 4 chữ số là A54  120.
NGUYỄN VĂN LỰC  0933.168.309 SP Toán 35 - ĐH Cần Thơ
Tổ hợp – Xác suất FB: http://www.facebook.com/VanLuc168
Số các số thuộc M có 5 chữ số là A  120. 5
5

Số phần tử của không gian mẫu là: 60 120 120 300.

Gọi A là tập con của M mà mỗi số thuộc A có tổng các chữ số bằng 10.
Các tập con của E có tổng các phần tử bằng 10 gồm
E1  {1,2,3,4}, E2  {2,3,5}, E3  {1,4,5}.
Từ E1 lập được số các số thuộc A là 4!
Từ mỗi tập E2 và E3 lập được số các số thuộc A là 3!
Suy ra số phần tử của A là 4! 2.3!  36.
36 3
Vậy xác suất cần tính là P  .
300 25

Bài giải
♥ Số phần tử của không gian mẫu là: 60 120 120 300.

♥ Gọi A là biến cố: “số được chọn có tổng các chữ số của số đó bằng 10”
Số kết quả thuận lợi cho biến cố A là: A
4! 2.3! 36.

36 3
♥ Vậy xác suất cần tính là P A A
.
300 25

Ví dụ 17: Một hộp chứa 4 quả cầu màu đỏ, 5 quả cầu màu xanh và 7 quả cầu màu
vàng. Lấy ngẫu nhiên cùng lúc ra 4 quả cầu từ hộp đó. Tính xác suất sao cho 4 quả
cầu được lấy ra có đúng một quả cầu màu đỏ và không quá hai quả cầu màu
xanh.

Lời giải
♥ Số phần tử của không gian mẫu là: C164 1820

♥ Gọi A là biến cố:


“4 quả cầu được lấy ra có đúng một quả cầu màu đỏ và không quá hai quả
cầu màu xanh”
Số kết quả thuận lợi cho biến cố A là: A
C41C53 C41C71C52 C41C72C51 740

740 37
♥ Vậy xác suất cần tính là P A A

1820 91

NGUYỄN VĂN LỰC  0933.168.309 SP Toán 35 - ĐH Cần Thơ


Tổ hợp – Xác suất FB: http://www.facebook.com/VanLuc168
Ví dụ 18: Một đội ngũ cán bộ khoa học gồm 8 nhà toán học nam, 5 nhà vật lý nữ và 3
nhà hóa học nữ. Chọn ra từ đó 4 người. Tính xác suất trong 4 người được chọn phải
có nữ và có đủ ba bộ môn.

Lời giải
♥ Số phần tử của không gian mẫu là: C164 1820

♥ Gọi A là biến cố: “4 người được chọn phải có nữ và có đủ ba bộ môn”


Số kết quả thuận lợi cho biến cố A là:
A
C82C51C31 C81C52C31 C81C51C32 420 240 120 780

780 3
♥ Vậy xác suất cần tính là P A A

1820 7

Ví dụ 19: Có 40 tấm thẻ được đánh số từ 1 đến 40. Chọn ngẫu nhiên ra 10 tấm thẻ.
Tính xác suất để trong 10 thẻ được chọn ra có 5 tấm thẻ mang số lẻ, 5 tấm thẻ
mang số chẵn trong đó chỉ có đúng một tấm thẻ mang số chia hết cho 10.

Lời giải
♥ Số phần tử của không gian mẫu là: C4010 847660528

♥ Gọi A là biến cố: “10 thẻ được chọn ra có 5 tấm thẻ mang số lẻ, 5 tấm thẻ mang
số chẵn trong đó chỉ có
đúng một tấm thẻ mang số chia hết cho 10”
Số kết quả thuận lợi cho biến cố A là: A
C205 .C164 .C41 112869120

112869120 1680
♥ Vậy xác suất cần tính là P A A

847660528 12617

Ví dụ 20: Một chiếc hộp đựng 6 quả cầu trắng, 4 quả cầu đỏ và 2 quả cầu đen. Chọn
ngẫu nhiên 6 quả từ hộp. Tính xác suất để 6 quả cầu được chọn có 3 quả cầu
trắng, 2 quả cầu đỏ và 1 quả cầu đen.

Lời giải
♥ Số phần tử của không gian mẫu là: C126 924
NGUYỄN VĂN LỰC  0933.168.309 SP Toán 35 - ĐH Cần Thơ
Tổ hợp – Xác suất FB: http://www.facebook.com/VanLuc168
♥ Gọi A là biến cố: “6 quả cầu được chọn có 3 quả cầu trắng, 2 quả cầu đỏ và 1
quả cầu đen”
Số kết quả thuận lợi cho biến cố A là: A
C63C42C21 240

240 20
♥ Vậy xác suất cần tính là P A A

294 77

Ví dụ 21: Một tổ học sinh gồm có 5 học sinh nam và 7 học sinh nữ. Chọn ngẫu
nhiên 2 học sinh đi chăm sóc bồn hoa. Tính xác suất để 2 học sinh được chọn đi
chăm sóc bồn hoa có cả nam và nữ.

Lời giải
♥ Số phần tử của không gian mẫu là: C122 66

♥ Gọi A là biến cố: “2 học sinh được chọn đi chăm sóc bồn hoa có cả nam và nữ”
Số kết quả thuận lợi cho biến cố A là: A
C51C71 35

35
♥ Vậy xác suất cần tính là P A A

66

Ví dụ 22: Một hộp đựng 3 viên bi xanh, 4 viên bi đỏ và 5 viên bi vàng. Chọn ngẫu
nhiên 3 viên bi. Tính xác xuất để 3 viên bi được chọn có đủ cả ba màu.

Lời giải
♥ Số phần tử của không gian mẫu là: C123 220

♥ Gọi A là biến cố: “3 viên bi được chọn có đủ cả ba màu”


Số kết quả thuận lợi cho biến cố A là: A
C31C41C51 60

60 3
♥ Vậy xác suất cần tính là P A A

220 11

Ví dụ 23: Một hộp đựng 8 viên bi đỏ và 6 viên bi xanh. Chọn ngẫu nhiên 4 viên bi
từ hộp trên. Tính xác xuất để 4 viên bi được lấy ra có cả bi xanh và bi đỏ.

Bài giải
NGUYỄN VĂN LỰC  0933.168.309 SP Toán 35 - ĐH Cần Thơ
Tổ hợp – Xác suất FB: http://www.facebook.com/VanLuc168
♥ Số phần tử của không gian mẫu là: C 4
14
1001

♥ Gọi A là biến cố: “4 viên bi được lấy ra có cả bi xanh và bi đỏ”


Số kết quả thuận lợi cho biến cố A là: A
C81C63 C82C62 C83C61 916

916
♥ Vậy xác suất cần tính là P A A

1001

Ví dụ 24: Có 6 học sinh nam và 4 học sinh nữ. Người ta chọn ra một cách ngẫu
nhiên 4 học sinh. Tìm xác suất để trong 4 học sinh được chọn ra có ít nhất 2 học
sinh nữ.

Lời giải
♥ Số phần tử của không gian mẫu là: C104 210

♥ Gọi A là biến cố: “4 học sinh được chọn ra có ít nhất 2 học sinh nữ”
Khi đó biến cố A là: “4 học sinh được chọn ra có nhiều nhất 1 học sinh nữ”
Số kết quả thuận lợi cho biến cố A là: A
C64 C41C63 95

95 19
Suy ra: P A A

210 42

19 23
♥ Vậy xác suất cần tính là P A 1 P A 1 
42 42

Ví dụ 25: Một hộp đựng 4 viên bi đỏ, 5 viên bi trắng, 6 viên bi vàng (các viên bi có
kích thức giống nhau, chỉ khác nhau về màu). Người ta chọn ngẫu nhiên 4 viên bi từ
hộp đó. Tính xác suất để 4 viên bi chọn ra không có đủ cả ba màu.

Lời giải
♥ Số phần tử của không gian mẫu là: C154 1365

♥ Gọi A là biến cố: “4 viên bi chọn ra không có đủ cả ba màu”


Khi đó biến cố A là: “4 bi chọn ra có đủ cả ba màu”
Số kết quả thuận lợi cho biến cố A là: A
C42C51C61 C41C52C61 C41C51C62 720

720 48
Suy ra: P A A

1365 91

NGUYỄN VĂN LỰC  0933.168.309 SP Toán 35 - ĐH Cần Thơ


Tổ hợp – Xác suất FB: http://www.facebook.com/VanLuc168
48 43
♥ Vậy xác suất cần tính là P A 1 P A 1 
91 91

Ví dụ 26: Xếp ngẫu nhiên 3 học sinh nam và 2 học sinh nữ thành một hàng
ngang. Tính xác suất để có 2 học sinh nữ đứng cạnh nhau.

Lời giải
♥ Số phần tử của không gian mẫu là: 5!

♥ Gọi A là biến cố: “2 học sinh nữ đứng cạnh nhau”


Số kết quả thuận lợi cho biến cố A là: A
4.2!.3!

2
♥ Vậy xác suất cần tính là P A A

5

Ví dụ 27: Trong giải cầ u lông kỷ niê ̣m ngày truyề n thố ng ho ̣c sinh sinh viên có 8
người tham gia trong đó có hai ba ̣n Viê ̣t và Nam. Các vâ ̣n đô ̣ng viên đươ ̣c chia làm
hai bảng A và B, mỗi bảng gồm 4 người. Giả sử việc chia bảng thực hiê ̣n bằ ng cách
bố c thăm ngẫu nhiên, tính xác suất để cả hai bạn Viê ̣t và Nam nằ m chung mô ̣t bảng
đấ u.
Lời giải
♥ Số phần tử của không gian mẫu là: C84 70

♥ Gọi A là biến cố: “cả hai ba ̣n Viê ̣t và Nam nằ m chung mô ̣t bảng đấ u”
Số kết quả thuận lợi cho biến cố A là: A
C21C62 30

30 3
♥ Vậy xác suất cần tính là P A A
. 
70 7

Ví dụ 28: Người ta phân chia một cách ngẫu nhiên 8 bạn học sinh Kì, Thi, Trung,
Học, Phổ, Thông, Quốc, Gia thành 2 nhóm, mỗi nhóm 4 bạn, để chơi trò kéo co. Tính
xác xuất để hai bạn Quốc và Gia ở trong cùng một nhóm.

Lời giải
C84
♥ Số phần tử của không gian mẫu là: 35
2
♥ Gọi A là biến cố: “hai bạn Quốc và Gia ở trong cùng một nhóm”

Số kết quả thuận lợi cho biến cố A là: A


C62 15

NGUYỄN VĂN LỰC  0933.168.309 SP Toán 35 - ĐH Cần Thơ


Tổ hợp – Xác suất FB: http://www.facebook.com/VanLuc168
15 3
♥ Vậy xác suất cần tính là P A A
.
35 7

Ví dụ 29: Viết ngẫu nhiên một số tự nhiên chẵn gồm 4 chữ số đôi một khác nhau
lên bảng. Tính xác suất để số vừa viết thỏa mãn trong số đó mỗi chữ số đều lớn
hơn chữ số đứng trước nó.

Phân tích
Gọi A là biến cố số tự nhiên chẵn gồm 4 chữ số được viết ra thỏa mãn mỗi chữ số lớn
hơn chữ số đứng trước nó. Khi đó
  {abcd : a  0, d {0, 2, 4, 6, 8}};
 A  {abcd : 0  a  b  c  d , d là số chẵn}.
Để tính  ta xét các trường hợp sau
+) d  0. Trường hợp này có A93 số.
+) d {2, 4, 6, 8}. Trường hợp này có ( A93  A82 ).4 số.
Suy ra   A93  4( A93  A82 )  2296.
Để tính  A ta xét các trường hợp sau
+) d  4. Trường hợp này có 1 số.
+) d  6. Trường hợp này có C53 số.
+) d  8. Trường hợp này có C73 số.
Suy ra  A  1  C53  C73  46.
A 46
Do đó P( A)    0,02.
 2296

Lời giải
♥ Số phần tử của không gian mẫu là: A93 4( A93 A82 ) 2296.

♥ Gọi A là biến cố: “số vừa viết thỏa mãn trong số đó mỗi chữ số đều lớn hơn chữ
số đứng trước nó”
Số kết quả thuận lợi cho biến cố A là: A
1 C53 C73 46.

46 23
♥ Vậy xác suất cần tính là P( A) A
.
2296 1146

NGUYỄN VĂN LỰC  0933.168.309 SP Toán 35 - ĐH Cần Thơ


Tổ hợp – Xác suất FB: http://www.facebook.com/VanLuc168

BÀI TẬP TỰ LUYỆN

1. Bài toán đếm

Câu 1. Từ các chữ số 0; 1; 2; 3; 4; 5 có thể lập được bao nhiêu số tự nhiên có 5 chữ số
và số đó chia hết cho 3?

Số có 5 chữ số cần lập là abcde ( a  0 ; a, b, c, d, e  {0; 1; 2; 3; 4; 5})


abcde 3  (a  b  c  d  e) 3
- Nếu (a  b  c  d ) 3 thì chọn e = 0 hoặc e = 3
- Nếu (a  b  c  d ) chia 3 dư 1 thì chọn e = 2 hoặc e = 5
- Nếu (a  b  c  d ) chia 3 dư 2 thì chọn e = 1 hoặc e = 4
Như vậy với mỗi số abcd đều có 2 cách chọn e để được một số có 5 chữ số chia hết
cho 3
Số các số dạng abcd lập được từ tập A là: 5x6x6x6= 1080 số
Số các số cần tìm là 2 x 1080 = 2160 số

Câu 2. Từ các chữ số 1,2,3,4,5,6,7,8,9 có thể lập được bao nhiêu số tự nhiên mỗi số
gồm 6 chữ số khác nhau và tổng các chữ số hàng chục, hàng trăm, hàng ngàn bằng 8.

Giả sử số cần lập có dạng a1a2a3a4a5a6


a , a , a5  1; 2;5 
Theo đề a3  a4  a5  8   3 4 .
 
a3 , a4 , a5  1;3; 4
TH1: a3 , a4 , a5  1; 2;5 .
Có 6 cách chọn a1; 5 cách chọn a2; 3! Cách chọn a3,a4,a5 và 4 cách chọn a6
Vậy có 6.5.3!.4=720 số
TH2: a3 , a4 , a5  1;3; 4 . Tương tự có 720 số
Vậy có 1440 số thỏa đề.

Câu 3. Cho tập A  0;1;2;3;4;5 , từ A có thể lập được bao nhiêu số tự nhiên gồm 5 chữ
số khác nhau, trong đó nhất thiết phải có chữ số 0 và 3.

-Gọi số cần tìm là abcde  a  0


-Tìm số các số có 5 chữ số khác nhau mà có mặt 0 và 3 không xét đến vị trí a.
Xếp 0 và 3 vào 5 vị trí có: A52 cách
3 vị trí còn lại có A43 cách
Suy ra có A52 A43 số
-Tìm số các số có 5 chữ số khác nhau mà có mặt 0 và 3 với a = 0.
NGUYỄN VĂN LỰC  0933.168.309 SP Toán 35 - ĐH Cần Thơ
Tổ hợp – Xác suất FB: http://www.facebook.com/VanLuc168
Xếp 3 có 4 cách
3 vị trí còn lại có A43 cách
Suy ra có 4.A43 số
Vậy số các số cần tìm tmycbt là: A52 A43 - 4.A43 = 384

Câu 4. Cho tập A= {0; 1; 2; 3; 4; 5; 6}. Có bao nhiêu số tự nhiên có 5 chữ số khác nhau
chọn trong A sao cho số đó chia hết cho 15.

Nhận xét: Số chia hết cho 15 thì chia hết 3 và chia hết 5.
 Các bộ số gồm 5 số có tổng chia hết cho 3 là: (0; 1; 2; 3; 6), (0; 1; 2; 4; 5), (0; 1; 3;
5; 6), (0; 2; 3; 4; 6), (0; 3; 4; 5; 6),(1; 2; 3; 4; 5), (1; 2; 4; 5; 6).
 Mỗi số chia hết cho 5 khi và chỉ khi số tận cùng là 0 hoặc 5.
+ Trong các bộ số trên có 4 bộ số có đúng một trong hai số 0 hoặc 5  4.P4 = 96
số chia hết cho 5
+ Trong các bộ số trên có 3 bộ số có cả 0 và 5.
Nếu tận cùng là 0 thì có P4= 24 số chia hết cho 5.
Nếu tận cùng là 5 vì do số hàng chục nghìn không thể là số 0, nên có 3.P 3=18 số
chia hết cho 5.
Trong trường hợp này có: 3(P4+3P3) = 126 số.
Vậy số các số theo yêu cầu bài toán là: 96 + 126 = 222 số.

Câu 5. Có bao nhiêu số tự nhiên có 5 chữ số khác nhau mà trong mỗi số luôn luôn có
mặt hai chữ số chẵn và ba chữ số lẻ.

Từ giả thiết bài toán ta thấy có C52  10 cách chọn 2 chữ số chẵn (kể cả số có chữ số 0
đứng đầu) và C53 =10 cách chọn 2 chữ số lẽ => có C52 . C53 = 100 bộ 5 số được chọn.
Mỗi bộ 5 số như thế có 5! số được thành lập => có tất cả C52 . C53 .5! = 12000 số.
Mặt khác số các số được lập như trên mà có chữ số 0 đứng đầu là C41 .C53 .4!  960 .
Vậy có tất cả 12000 – 960 = 11040 số thỏa mãn bài toán.

Câu 6. Có 12 học sinh giỏi gồm 3 học sinh khối 12, 4 học sinh khối 11, 5 học sinh khối
10. Hỏi có bao nhiêu cách chọn ra 6 học sinh sao cho mỗi khối có ít nhất 1 học sinh.

Tổng số cách chọn 6 học sinh trong 12 học sinh là C126


Số học sinh được chọn phải thuộc ít nhất 2 khối
Số cách chọn chỉ có học sinh khối 12 và khối 11 là: C76
Số cách chọn chỉ có học sinh khối 11 và khối 10 là: C96
Số cách chọn chỉ có học sinh khối 12 và khối 10 là: C86
Số cách chọn thoả mãn đề bài là: C126  C76  C96  C86  805 (cách)

NGUYỄN VĂN LỰC  0933.168.309 SP Toán 35 - ĐH Cần Thơ


Tổ hợp – Xác suất FB: http://www.facebook.com/VanLuc168
Câu 7. Trên các cạnh AB, BC, CD, DA của hình vuông ABCD lần lượt cho 1, 2, 3 và n
điểm phân biệt khác A, B, C, D. Tìm n biết số tam giác có ba đỉnh lấy từ n + 6 điểm đã
cho là 439.

Nếu n  2 thì n + 6  8. Do đó số tam giác có ba đỉnh được lấy từ n + 6 điểm đó không


vượt qua C83  56  439 (loại). Vậy n  3
Vì mỗi tam giác được tạo thành ứng với 1 tổ hợp 3 chập n + 6 phần tử. Nhưng trên
cạnh CD có 3 đỉnh, trên cạnh DA có n đỉnh nên số tam giác tạo thành là:

Cn36  C33  Cn3 


 n  4 n  5 n  6   1   n  2  n  1 n  439
6 6
 (n + 4)(n + 5)(n + 6) – (n – 2)(n – 1)n = 2540
 n2 + 4n – 140 = 0
Từ đó tìm được n = 10.

Câu 8. Trong mp có bao nhiêu hình chữ nhât đươc̣ ta ̣o thành từ 6 đường thẳ ng song
song với nhau và 8 đường thẳ ng vuông góc với 6 đường thẳ ng song song đó.

Gọi A là tập hợp gồm 6 đường thẳng song song


B là tập hợp gồm 8 đường thẳng vuông góc
Mỗi hình chữ nhật được tạo thành gồm 2 đường thẳng trong tập hợp A và 2 đường
thẳng trong tập hợp B.
Như vậy số hình chữ nhật được tạo thành là C82 .C62  420

Câu 9. Trong không gian cho n điểm phân biệt (n  , n  4) , trong đó không có 4 điểm
nào đồng phẳng. Tìm n, biết rằng số tứ diện có đỉnh là 4 trong n điểm đã cho nhiều gấp
4 lần số tam giác có đỉnh là 3 trong n điểm đã cho.

Số tứ diện có đỉnh là 4 trong n điểm đã cho là Cn4 , số tam giác có đỉnh là 3 trong n
điểm đã cho là Cn3 .
Theo giả thiết, ta có
n! n!
Cn4  4Cn3   4.  n  3  16  n  19.
4!(n  4)! 3!(n  3)!

NGUYỄN VĂN LỰC  0933.168.309 SP Toán 35 - ĐH Cần Thơ


Tổ hợp – Xác suất FB: http://www.facebook.com/VanLuc168
2. Nhị thức Newton

Câu 10. Giải phương trình: Cn1  3Cn2  7Cn3  ...  (2n  1)Cnn  32 n  2n  6480

Cn1  3Cn2  7Cn3  ...  (2n  1)Cnn  32 n  2n  6480


Xét 1  x n  Cn0  Cn1 .x  Cn2 .x 2  Cn3 .x3  ...  Cnn .x n
 Với x = 2 ta có: 3n  Cn0  2Cn1  4Cn2  8Cn3  ...  2n Cnn (1)
Với x = 1 ta có: 2n  Cn0  Cn1  Cn2  Cn3  ...  Cnn (2)
 Lấy (1) – (2) ta được: Cn1  3Cn2  7Cn3  ...   2n  1 Cnn  3n  2n
 PT  3n  2n  32n  2n  6480  32n  3n  6480  0  3n  81  n  4

Câu 11. Giải phương trình Cxx  2Cxx1  Cxx2  Cx2x23 ( Cnk là tổ hợp chập k của n phần tử)

2  x  5
ĐK : 
x  N
Ta có Cxx  Cxx1  Cxx1  Cxx2  Cx2x23  Cxx1  Cxx11  Cx2x23  Cxx2  Cx2x23
 (5  x)!  2!  x  3

 1 nCnn
n
Cn1 2Cn2 3Cn3
Câu 12. Tính tổng S    ... 
2.3 3.4 4.5  n  1 n  2 

Tính tổng S  Cn  2Cn  3Cn  ...    n


1 nC n
1 2 3 n

2.3 3.4 4.5  n  1 n  2 

Ta có
Cnk

n!

1
.
 n  1!

Cnk11
(3)
k  1 k ! k  1 n  k ! n  1  k  1!  n  1   k  1 ! n  1

 1 kCnk   1 kCnk22


k k

Áp dụng 2 lần công thức (3) ta được:


 k  1 k  2  n  1 n  2 
Cho k chạy từ 1 đến n rồi cộng vế các đẳng thức trên ta có
 n  1 n  2  S  Cn32  2Cn42  3Cn52  ...   1 nCnn22
n

   Cn21  Cn31   2  Cn31  Cn41   3  Cn41  Cn51   ...   1 nCnn11


n

 Cn21  Cn31  Cn41  ...   1 Cnn11


n


 Cn01  Cn11  Cn01  Cn11  Cn21  Cn31  Cn41  Cn51  ...   1
n 1

Cnn11 

1   n  1  1  1
n 1
 n
n
Vậy S  .
 n  1 n  2 

Câu 13. Cho khai triển đa thức: 1  2 x   ao  a1x  a2 .x 2  ...  a2013 .x 2013
2013

S  a0  2 a1  3 a2  ...  2014 a2013


Tính tổng:

NGUYỄN VĂN LỰC  0933.168.309 SP Toán 35 - ĐH Cần Thơ


Tổ hợp – Xác suất FB: http://www.facebook.com/VanLuc168

Ta có:  x(1  2 x)2013   a0  2a1 x  3a2 x 2  ...  2014a2014 x 2013.


 (1  2 x) 2013  4026 x(1  2 x)1012  a0  2a1 x  3a2 x 2  ...  2014a2013 x 2013 (*).
Nhận thấy: ak x k  ak ( x)k do đó thay x  1 vào cả hai vế của (*) ta có:
S  a0  2 a1  3 a2  ...  2014 a2013  1343.32213

Câu 14. Tính giá trị biểu thức: A  4C100


2
 8C100
4
 12C100
6
 ...  200C100
100
.

Ta có: 1  x   C100  C100 x  C100 x 2  ...  C100


100 0 1 2 100 100
x (1)
1  x   C100  C100 x  C100 x 2  C100 x3  ...  C100
100 0 1 2 3 100 100
x (2)
Lấy (1)+(2) ta được: 1  x   1  x   2C100  2C100 x 2  2C100 x 4  ...  2C100
100 100 0 2 4 100 100
x
Lấy đạo hàm hai vế theo ẩn x ta
được:100 1  x   100 1  x   4C100 x  8C100 x3  ...  200C100
99 2 4 99 100 99
x
Thay x=1 vào => A  100.299  4C100
2
 8C100
4
 ...  200C100
100

3. Hệ số khai triển nhị thức

Câu 15. Tìm hệ số của số hạng chứa x 6 trong khai triển của: x3  2  x5  , biết tổng
1
x 
các hệ số trong khai triển trên bằng 4096 ( trong đó n là số nguyên dương và x  0 ).

Xét khai triển :


n
3 1 
n
1 5 
5
x  3  x   x  3  x2 
3

x  x 
  1 n n 1
 52 
nk
 52 
k 5 n
1 1  k  1  n 2 
 x Cn  3   Cn  3   x   ...  Cn  3   x   ...  Cn  x  
3 0

  x  x    x      
Thay x  1 vào khai triển ta được:
2n  Cn0  Cn1  ...  Cnk  ...  Cnn 
Theo giả thiết ta có:
Cn0  Cn1  ...  Cnk  ...  Cnn  4096  2n  212  n  12
12
1 
x  2  x5 
3

Với n  12 ta có khai triển: x 


k  1 0  k  12, k  Z  6
Gọi số hạng thứ là số hạng chứa x .

 x
12  k
1 k 5k
2 k  21
Tk 1  x C  2 
3 k
12
5
C x
k
12
2

Ta có : x 
5k 2  21  6 
6 2k  21  6k  6
Vì số hạng có chứa x nên : 2 9 .
Với k  6 ta có hệ số cần tìm là : C12  924 .
6

NGUYỄN VĂN LỰC  0933.168.309 SP Toán 35 - ĐH Cần Thơ


Tổ hợp – Xác suất FB: http://www.facebook.com/VanLuc168

Câu 16. Tìm hệ số của x 4 trong khai triển Niutơn của biểu thức : P  (1  2 x  3x 2 )10 .

10 10 k
+ Ta có P  (1  2 x  3x 2 )10   C10k (2 x  3x 2 ) k   ( C10k Cki 2k i3i x k i )
k 0 k 0 i 0

k  i  4
 i  0 i  1 i  2
Theo giả thiết ta có 0  i  k  10    
i, k  N  k  4  k  3 k  2

+Vậy hệ số của x 4 là: C104 24  C103 C31 223  C102 C22 32  8085 .

Câu 17. Cho số nguyên dương n thõa điều kiện C21n1  C23n1  ...  C22nn11  1023 . Tìm hệ số
của x13 trong khai triển (x+3)3n

Đặt S = C20n1  C21n1  C22n1  ...  C22nn1  C22nn11  22n1


Ta có C21n1  C23n1  ...  C22nn11  C22nn11  C20n1  C22n1  ...  C22nn1
1 1
Do đó C21n 1  C23n 1  ...  C22nn11  C22nn11  S  .2n1  2n
2 2
2 n 1 2 n 1
=> C2n1  C2n1  ...  C2n1  2  C2n1  2  1
1 3 n n

Vậy C21n1  C23n1  ...  C22nn11  1023  2n  1  1023  n  5


15
Với n=5 , ta có (x+3)3n=(x+3)15  C1515k 315k x k .
k 0

Vậy hệ số của x trong khai triển (x+3)15 là 32.C1513  945


13

Câu 18. Tìm hệ số của x9 trong khai triển 2 3x , trong đó n là số nguyên dương thỏa
2n

mãn: C21n 1 C23n 1 C25n 1 ... C22nn 11 4096 .

Ta có
2n 1
1 x C20n 1 C21n 1 x C22n 1 x 2 ... C22nn 11 x 2 n 1

Cho x=1, ta có 22n 1 C20n 1 C21n 1 C22n 1 ... C22nn 11 (1)


Cho x= -1, ta có : 0 C20n 1 C21n 1 C22n 1 ... C22nn 11 (2)
Lầy (1) trừ (2), ta được : 22 n 1 2 C21n 1 C23n 1 C25n 1 ... C22nn 11
 22 n C21n 1 C23n 1 C25n 1 ... C22nn 1
1

Từ giả thiế t ta có 22 n 4096 22 n 212 2n 12


12
Do đó ta có 2 3x ( 0 ≤ k ≤ 12, k nguyên)
12
( 1 )k C12k 212 k ( 3x )k
k 0

 hệ số của x9 là : - C129 39 23 .

Câu 19.  3x  1 ,  k , n  N ;0  k  2n 
2n
 a0  a1x  a2 x 2  ...  ak x k  ...  a2n x 2n

Biết rằng: a0  a1  a2  ...   1 ak  ...  a2n  4096 . Tìm hệ số của x8 trong khai triển.
k

NGUYỄN VĂN LỰC  0933.168.309 SP Toán 35 - ĐH Cần Thơ


Tổ hợp – Xác suất FB: http://www.facebook.com/VanLuc168
Ta có:
 3x + 1 = a 0 + a1x + a 2 x 2 +...+ a k x k +...+ a 2n x 2n
2n

Thay x = -1, ta có: (-2)2n = a0 – a1 + a2 - … + (-1)kak +…+ a2n


Từ giả thiết suy ra: (-2)2n = 4096  n = 6
Với n = 6, ta có khai triển:
1+3x  =C120 + C112 .(3x) + C122 (3x)2 +...+ C1212 (3x)12
12

8 8
 Hệ số của x8 trong khai triển là: C12 .3

Câu 20. Cho n là số nguyên dương thỏa 4Cnn11  2Cn2  25n  120
n

Tìm hệ số của số hạng chứa x7 trong khai triển  x 2   ,(x > 0)


2
x  

4Cnn11  2Cn2  25n  120  4


 n  1!  2 n !  25n  120
 n  1!2! 2! n  2 !
 2n  n  1   n  1 n  25n  120  n 2  22n  120  0  10  n  12
Mà n nguyên dương nên n = 11
n 11 44 5 k
 2   2 2  k
k 22  2 k 2
  x2  có số hạng tổng quát là:    11
    11

k k

  x   Tk 1 1 C x k /2
1 C k k
2 x 2

 x  x x
44  5k
Tk+1 là số hạng chứa x7 khi  7  k  6  T7  C116 26 x 7
2
 Hệ số cấn tìm là: C116 26

n
 3 1 
Câu 21. Tìm hệ số của số hạng chứa x trong khai triển biểu thức
10
x  2  biết n là số
 x 
tự nhiên thỏa mãn Cn4  13Cnn2 .

n  3
Điều kiện  . Phương trình đã cho tương đương với
n  N
n! n!
 13.
4!(n  4)! ( n  2)!2!
 n  15(t / m)
Vậy n  15.  n2  5n  150  0  
 n  10(l )
Với n = 15 ta có
15
 3 1  3 15 k  1 k
 x  2    C15  x  .   2 
15
k

 x  k 0  x 
15
  C15k (1) k .x 455 k
k 0

Để trong khai triển đã cho có số hạng chứa x10 thì 45  5k  10  k  7(t / m)


Vậy hệ số của x10 trong khai triển đã cho là C157 .(1)7  6435 .

NGUYỄN VĂN LỰC  0933.168.309 SP Toán 35 - ĐH Cần Thơ


Tổ hợp – Xác suất FB: http://www.facebook.com/VanLuc168
14
 2
Câu 22. Tìm hệ số của số hạng chứa x trong khai triển : 5
x 2  .
 x 

14

= x  2 x  2    C14k x14 3k .2k
2 14
x 2 
 x 
số hạng chứa x5 trong khai triển ứng với k thoả mãn 14 - 3k = 5 => k=3
Hệ số cần tìm là C143 2 3  2912

Câu 23. Tìm số hạng chứa x trong khai triển của  x  2  .


3 2
 x 

2
9 k

Ta có  x  2    C9k x 9k  2    C9k x 93k  2 k


9 9
2
 x  k 0 x  k 0

Số hạng chứa x tương ứng giá trị k thoả mãn 9  3k  3  k  2


3

Suy ra số hạng chứa x 3 bằng C92 x 3  2   144x 3


2

n
 2 2
Câu 24. Tìm số hạng chứa x trong khai triển nhị thức Niu-tơn của
4
x  x với
 
x ≠ 0, biết rằng: Cn1  Cn2  15 với n là số nguyên dương.

n(n+ 1)
Ta có Cn1  Cn2  15  Cn+2 1  15   15
2
 n  5 (t / m)
 n2 + n  30  0  
 n  6 (lo¹i)
5
 2 2 5
2 5
Với n = 5 và x  0 ta có  x     C 5k ( x2 )k ( )5k   C 5k x3k 5 (2)5k
 x  k 0 x k 0

Số hạng chứa x4 trong khai triển trên thỏa mãn 3k – 5 = 4  k = 3, suy ra số hạng
chứa x4 trong khai triển trên là 40x4.

Câu 25. Tìm số hạng chứa x 3 trong khai triển nhị thức Niu - tơn của biểu thức
n
 2
 x   , x  0. Trong đó n là số tự nhiên thỏa mãn An2  2Cn1  180 .
 x

- ĐK: n  , n  2

 n  15
- Khi đó: An2  2Cn1  180  n2  3n  180  0   
DK
n  15
 n  12

15 15 3 k
 2 15
     
k k k
- Khi n = 15 ta có:  x  C15 1 2 x 2

 x  k 0

15  3k
Mà theo bài ra ta có: 3 k 3
2
NGUYỄN VĂN LỰC  0933.168.309 SP Toán 35 - ĐH Cần Thơ
Tổ hợp – Xác suất FB: http://www.facebook.com/VanLuc168
Do đó số hạng chứa x trong khai triển trên là: C  1 23 x3  3640 x3
3 3 3
15

n
 1 
Câu 26. Tìm số hạng không chứa x trong khai triển nhị thức  2.x   , biết rằng
 x
An2  Cnn11  4n  6

Giải phương trình An2  Cnn11  4n  6 ; Điều kiện: n ≥ 2 ; n  N.


(n  1)! n(n  1)
Phương trình tương đương với n(n  1)   4n  6  n(n  1)   4n  6
2!(n  1)! 2
 n2 – 11n – 12 = 0  n = - 1 (Loại) v n = 12.
12
 1 
Với n = 12 ta có nhị thức Niutơn:  2x   .
 x
k
 1 
Số hạng thứ k + 1 trong khai triển là: Tk +1 = C (2 x) k
12
12  k
  ; k  N, 0 ≤ k ≤ 12
 x
k 24 3 k

Hay Tk+ 1 = C12k  2 x 
12  k
.x 2
= C12k .212 k .x 2
.
k  N , 0  k  12
Số hạng này không chứa x khi   k  8.
24  3k  0
Vậy số hạng thứ 9 không chứa x là T9 = C128 24  7920

Câu 27. Tìm hệ số của x8 trong khai triển (x2 + 2)n, biết: An3  8Cn2  Cn1  49 .

Điều kiện n  4. Ta có  x 2  2    Cnk x 2 k 2nk


n n

k 0

Hệ số của số hạng chứa x8 là Cn4 2n  4


Hệ số của số hạng chứa x8 là Cn4 2n  4
Ta có: An3  8Cn2  Cn1  49
 (n – 2)(n – 1)n – 4(n – 1)n + n = 49
 n3 – 7n2 + 7n – 49 = 0  (n – 7)(n2 + 7) = 0  n = 7
Nên hệ số của x8 là C74 23  280

Câu 28. Cho khai triển: 1  2 x  x  x  1  ao  a1x  a2 x 2  ...  a14 x14 . Hãy tìm giá trị
10 2 2

của a6 .

1 3
Ta có x 2  x  1  (2 x  1) 2  nên
4 4
1 3 9
1  2 x  ( x 2  x  1)2  (1  2 x)14  (1  2 x)12  (1  2 x)10
10

16 8 16
NGUYỄN VĂN LỰC  0933.168.309 SP Toán 35 - ĐH Cần Thơ
Tổ hợp – Xác suất FB: http://www.facebook.com/VanLuc168
Trong khai triển 1  2x  hệ số của x là: 2 C ; Trong khai triển 1  2x  hệ số của
14 6 6 6 12
14

x 6 là: 26 C126
Trong khai triển 1  2x  hệ số của x 6 là: 26 C106
10

1 6 6 3 6 6 9 6 6
Vậy hệ số a6  2 C14  2 C12  2 C10  41748.
16 8 16

Câu 29. Cho khai triển (1 2x )n a0 a1x a2x 2 ... an x n . Tìm số nguyên dương n
biết a0 8a1 2a2 1 .

n n
Ta có (1  2x )n  C nk (2x )k  C nk 2k x k . Khi đó, suy ra ak  Cnk 2k
k 0 k 0

Do đó, ta có a0  Cn0 ;a1  2C n1 ;a2  4C n2


8n(n  1)
Vâ ̣y a 0  8a1  2a2  1  C n0  16C n1  8C n2  1  1  16n  1
2!
16n  4n(n  1)  4  n  1(n  0)  n  5

NGUYỄN VĂN LỰC  0933.168.309 SP Toán 35 - ĐH Cần Thơ


Tổ hợp – Xác suất FB: http://www.facebook.com/VanLuc168
4. Xác suất

Câu 30. Một hộp đựng 9 thẻ được đánh số 1,2,3,..,9. Rút ngẫu nhiên 3 thẻ và nhân 3
số ghi trên ba thẻ với nhau. Tính xác suất để tích nhận được là một số lẻ.

Số phần tử của không gian mẫu là n(  ) = C 39 = 84


Số cách chọn 3 thẻ có tích là số lẻ là n(A) = C53 = 10
10 5
=> Xác suất cần tính là P(A) = =
84 42

Câu 31. Một tổ có 5 học sinh nam và 6 học sinh nữ. Giáo viên chọn ngẫu nhiên 3 học
sinh để làm trực nhật . Tính xác suất để 3 học sinh được chọn có cả nam và nữ.

n     C113  165
Số cách chọn 3 học sinh có cả nam và nữ là C52 .C61  C51.C62  135
135 9
Do đó xác suất để 3 học sinh được chọn có cả nam và nữ là 
165 11

Câu 32. Từ các chữ số 1;2;3;4;5 có thể lập được bao nhiêu số tự nhiên có năm chữ số,
trong đó chữ số 3 có mặt đúng ba lần, các chữ số còn lại có mặt không quá một lần.
Trong các số tự nhiên nói trên, chọn ngẫu nhiên một số, tìm xác suất để số được chọn
chia hết cho 3.

Gọi a1a2 a3a4 a5 là số tự nhiên cần tìm, a1 , a2 , a3 , a4 , a5 thuộc 1; 2;3; 4;5
Sắp chữ số 3 vào ba vị trí, có C53  10 (cách)
Còn lại hai vị trí, 4 chữ số. Chọn hai chữ số xếp vào hai vị trí đó, có C42  12 (cách)
Vậy không gian mẫu có 10.12  120 phần tử
Gọi A là biến cố: “số được chọn chia hết cho 3”, có hai phương án:
Hai chữ số còn lại là 1 và 5, có C53.2!  20 số
Hai chữ số còn lại là 2 và 4, có C53.2!  20 số
40 1
Vậy biến cố A có 40 phần tử. Xác suất của biến cố A là: P  
120 3

Câu 33.Xét các số tự nhiên có 5 chữ số khác nhau. Tìm xác suất để số tự nhiên có 5
chữ số khác nhau lấy ra từ các số trên thảo mãn: Chữ số đứng sau lớn hơn chữ số đứng
trước.

Các số tự nhiên có 5 chữ số khác nhau: a1a2 a3 a4 a5 trong đó ai  a j với i  j


a1  0  Có 9 cách chọn a1
Mỗi cách chọn a1 có 9 cách chọn a2
Mỗi cách chọn a1, a2 có 8 cách chọn a3
Mỗi cách chọn a1, a2, a3 có 7 cách chọn a4
Mỗi cách chọn a1, a2, a3, a4 có 6 cách chọn a5
   9.9.8.7.6  27216

NGUYỄN VĂN LỰC  0933.168.309 SP Toán 35 - ĐH Cần Thơ


Tổ hợp – Xác suất FB: http://www.facebook.com/VanLuc168
Xét biến cố A: “ Số có năm chữ số lấy ra thoả mãn chữ số đứng sau lớn hơn chữ số
đứng trước”. Vì chữ số 0 không thể đứng trước bất kỳ số nào nên xét tập hợp:
X= 1; 2;3; 4;5;6;7;8;9 . Mỗi bộ gồm 5 chữ số khác nhau lấy ra từ X có một cách sắp
xếp theo thứ tự tăng dần   A  C9
5

126 1
 P ( A)  
27216 216

Câu 34. Một người gọi điện thoại, quên hai chữ số cuối và chỉ nhớ rằng hai chữ số đó
phân biệt. Tính xác suất để người đó gọi một lần đúng số cần gọi.

+ Hai chữ số cuối phân biệt nên gọi  là tập hợp tất cả các cách chọn 2 số phân biệt
trong 10 chữ số 0,1, 2,3, 4,5, 6, 7,8,9 , ta có được   A102  90
+ Gọi A là biến cố “Gọi 1 lần đúng số cần gọi”, ta có A  1 . Vậy xác suất cần tìm là
1
P  A 
90

Câu 35. Đề cương ôn tập cuối năm môn Toán lớp 12 có 40 câu hỏi. Đề thi cuối năm
gồm 3 câu hỏi trong số 40 câu đó. Một học sinh chỉ ôn 20 câu trong đề cương. Giả sử
các câu hỏi trong đề cương đều có khả năng được chọn làm câu hỏi thi như nhau. Hãy
tính xác suất để có ít nhất 2 câu hỏi của đề thi cuối năm nằm trong số 20 câu hỏi mà
học sinh nói trên đã ôn.

Không gian mẫu  có n(  )= C403  9880 (phần tử)


Gọi A là biến cố “có ít nhất 2 câu hỏi của đề thi nằm trong số 20 câu đã ôn”.Ta thấy
xảy ra một trong hai TH sau
TH1: Trong đề thi có đúng 2 câu hỏi trong 20 câu đã ôn
TH2: Trong đề thi có đúng 3 câu hỏi trong 20 câu đã ôn
Do đó n(X)= C202 .C201  C201  1330 (phần tử)
n( A) 1330 7
Vậy xác suất cần tìm: P(X)=  
n() 9880 52

Câu 36. Giải bóng chuyền VTV Cup gồm 12 đội bóng tham dự, trong đó có 9 đội
nước ngoài và 3 đội của Việt Nam. Ban tổ chức cho bốc thăm ngẫu nhiên để chia
thành 3 bảng A, B, C mỗi bảng 4 đội. Tính xác suất để 3 đội bóng của Việt Nam ở ba
bảng khác nhau.

Số phần tử không gian mẫu là n()  C124 .C84 .C44  34.650


Gọi A là biến cố “3 đội bong của Việt nam ở ba bảng khác nhau”
Số các kết quả thuận lợi của A là n( A)  3C93 .2C63 .1.C33  1080
n( A) 1080 54
Xác xuất của biến cố A là P( A)    0,31
n( 34650 173

NGUYỄN VĂN LỰC  0933.168.309 SP Toán 35 - ĐH Cần Thơ


Tổ hợp – Xác suất FB: http://www.facebook.com/VanLuc168
Câu 37. Gieo đồng thời ba con xúc sắc.Tính xác suất để tổng số chấm xuất hiện trên
ba con là 10.

Gọi  là tập hợp tất cả các khả năng xảy ra.Ta có n(  ) = 6.6.6=216
Gọi A là biến cố:” tổng số chấm xuất hiện trên ba con là 10”.
Các khả năng thuận lợi của A chính là tổ hợp có tổng bằng 10 là: (1;3;6), (1;4;5),
(2;2;6), (2;3;5), (3;3;4) và các hoán vị có thể của các tổ hợp này.
Ta có n(A) = 6+6+3+6+3 = 24 ( do (2;2;6), (3;3;4) chỉ có 3 hoán vị)
n ( A) 24 1
Vậy xác suất P(A) = = 
n ( ) 216 9

Câu 38. Một hộp có 5 viên bi đỏ, 3 viên bi vàng và 4 viên bi xanh. Lấy ngẫu nhiên lấy
4 viên bi từ hộp. Gọi A là biến cố “ trong số 4 viên bi lấy được có số bi đỏ lớn hơn số
bi vàng. Tính xác suất của biến cố A.

  C124  495
Các khả năng:
+4 bi lấy được không có bi vàng:4bi đỏ; 1 bi đỏ +3bi xanh;
+4 bi lấy được có đúng 1 bi vàng:gồm 2bi đỏ, 1 bi vàng, 1 bi xanh hoặc 3 bi đỏ , 1 bi
vàng.
  C54  C51.C43  C52 .C42  C53 .C41  C52 .C31.C41  C53 .C31 = 275
275 5
P  A  
495 9

Câu 39. Gọi M là tập hợp các số tự nhiên gồm 9 chữ số khác nhau. Chọn ngẫu nhiên
một số từ M, tính xác suất để số được chọn có đúng 4 chữ số lẻ và chữ số 0 đứng giữa
hai chữ số lẻ (các chữ số liền trước và liền sau của chữ số 0 là các chữ số lẻ).

Xét các số có 9 chữ số khác nhau:


- Có 9 cách chọn chữ số ở vị trí đầu tiên.
- Có A98 cách chọn 8 chữ số tiếp theo
Do đó số các số có 9 chữ số khác nhau là: 9. A98 = 3265920
Xét các số thỏa mãn đề bài:
- Có C54 cách chọn 4 chữ số lẻ.
- Đầu tiên ta xếp vị trí cho chữ số 0, do chữ số 0 không thể đứng đầu và cuối nên có 7
cách xếp.
- Tiếp theo ta có A42 cách chọn và xếp hai chữ số lẻ đứng hai bên chữ số 0.
- Cuối cùng ta có 6! cách xếp 6 chữ số còn lại vào 6 vị trí còn lại.
Gọi A là biến cố đã cho, khi đó n( A)  C54 .7.A42 .6! 302400.
302400 5
Vậy xác suất cần tìm là P( A)   .
3265920 54

Câu 40. Có 20 tấm thẻ được đánh số từ 1 đến 20. Chọn ngẫu nhiên ra 5 tấm thẻ. Tính
xác suất để trong 5 tấm thẻ được chọn ra có 3 tấm thẻ mang số lẻ, 2 tấm thẻ mang số
chẵn trong đó chỉ có đúng một tấm thẻ mang số chia hết cho 4.
NGUYỄN VĂN LỰC  0933.168.309 SP Toán 35 - ĐH Cần Thơ
Tổ hợp – Xác suất FB: http://www.facebook.com/VanLuc168

Số phần tử của không gian mẫu là: n     C205  15504 .


Trong 20 tấm thẻ, có 10 tấm thẻ mang số lẻ, có 5 tấm thẻ mang số chẵn và chia hết
cho 4, 5 tấm thẻ mang số chẵn và không chia hết cho 4.
Gọi A là biến cố cần tính xác suất. Ta có: n  A  C103 .C51.C51  3000 .
n  A  3000 125
Vậy, xác suất cần tính là: P  A    .
n    15504 646

Câu 41. Từ các chữ số của tập T  0;1; 2;3; 4;5 , người ta ghi ngẫu nhiên hai số tự
nhiên có ba chữ số khác nhau lên hai tấm thẻ. Tính xác suất để hai số ghi trên hai tấm
thẻ đó có ít nhất một số chia hết cho 5.

+ Có 5. A52  100 số tự nhiên có 3 chữ số khác nhau


+ Có A52  4. A41  36 số tự nhiên có 3 chữ số khác nhau và chia hết cho 5.
+ Có 64 số tự nhiên có 3 chữ số khác nhau và không chia hết cho 5.
+ n    C100
1 1
.C99  9900

+ Gọi A là biến cố : “Trong hai số được ghi trên 2 tấm thẻ có ít nhất 1 số chia hết cho
5”
Ta có: n  A  C36
1 1
.C64  C36
1 1
.C35  3564
n  A 3564 9
Vậy : P  A     0,36
n    9900 25

Câu 42. Hai người cùng bắn vào một mục tiêu. Xác suất bắn trúng của từng người là
0,8 và 0,9. Tìm xác suất của các biến cố sao cho chỉ có một người bắn trúng mục tiêu.

Gọi A là biến cố của người bắn trúng mục tiêu với xác suất là 0.8
B là biến cố của người bắn trúng mục tiêu với xác suất là 0.9
Gọi C là biến cố cần tính xác suất thì C= AB
.  AB
.
Vậy xác suất cần tính là P(C)=0,8.(1-0,9)+(1-0,8).0,9=0,26

Câu 43. Một người có 10 đôi giày khác nhau và trong lúc đi du lịch vội vã lấy ngẫu
nhiên 4 chiếc . Tính xác suất để trong 4 chiế c giày lấ y ra có ít nhất một đôi.

Số cách lấ y 4 chiế c giày tù y ý : C420 = 4845


Số cách chọn 4 chiế c giày từ 4 đôi ( mỗi chiếc lấy từ một đôi )là :
(số cách chọn 4 đôi từ 10 đôi)( số cách chọn 4 chiếc)= C41024
C420 - C10
4
.24 672
Xác suất cần tìm là : =
C420 969

NGUYỄN VĂN LỰC  0933.168.309 SP Toán 35 - ĐH Cần Thơ


Tổ hợp – Xác suất FB: http://www.facebook.com/VanLuc168
Câu 44. Trong kỳ thi học sinh giỏi quốc gia, lớp 12A Có 2 học sinh đạt giải môn Toán
đều là học sinh nam và 4 học sinh đạt giải môn Vật lí trong đó có 2 học sinh nam và 2
học sinh nữ. Chọn ngẫu nhiên 4 học sinh trong các học sinh đạt giải đó đi dự lễ tổng
kết năm học của tỉnh. Tính xác suất để 4 học sinh được chọn có 2 nam và 2 nữ, đồng
thời còn có cả học sinh đạt giải môn Toán và học sinh đạt giải môn Vật lí.

Không gian mẫu Ω là tập hợp gồm tất cả các cách chọn ra 3 học sinh trong các học
sinh đạt giải của kỳ thi học sinh giỏi quốc gia, do đó ta có n()  C36  20
Kí hiệu A là biến cố ‘‘4 học sinh được chọn có 2 nam và 2 nữ, đồng thời còn có cả
học sinh đạt giải môn Toán và học sinh đạt giải môn Vật lí’’.
Vì chỉ có đúng 2 học sinh nữ đạt giải đều thuộc môn Vật lí, do đó phải chọn tiếp ra 2
học sinh nam lại phải có mặt ở hai môn khác nhau thì chỉ có thể là 2 học sinh nam
đạt giải môn Toán hoặc 1 học sinh nam đạt giải môn Toán và 1 học sinh nam đạt
n(A) 1
giải môn Vật lí. Vậy ta có n(A)  1  C21 .C21  5  P(A)  
n ( ) 4

Câu 45. Có 30 tấm thẻ được đánh số từ 1 đến 30. Chọn ngẫu nhiên ra 10 tấm thẻ. Tính
xác suất để có 5 tấm thẻ mang số lẻ,5 tấm thẻ mang số chẵn trong đó chỉ có duy nhất
1 tấm mang số chia hết cho 10.

Gọi A là biến cố lấy được 5 tấm thẻ mang số lẻ, 5 tấm thẻ mang số chẵn trong đó chỉ
có 1 tấm thẻ mang số chia hết cho 10.
Chọn 10 tấm thẻ trong 30 tấm thẻ có : C1030 cách chọn
Ta phải chọn :
5 tấm thẻ mang số lẻ trong 15 tấm mang số lẻ có C 155 cách chọn.
1 tấm thẻ chia hết cho 10 trong 3 tấm thẻ mang số chia hết cho 10, có : C 13 cc
4 tấm thẻ mang số chẵn nhưng không chia hết cho 10 trong 12 tấm như vậy, có :
C412
C155 .C124 .C31 99
Vậy xác suất cần tìm là : P(A) = 10

C30 667

Câu 46. Có 5 hộp bánh, mỗi hộp đựng 8 cái bánh gồm 5 cái bánh mặn và 3 bánh ngọt.
Lấy ngẫu nhiên từ mỗi hộp ra hai bánh. Tính xác suất biến cố trong năm lần lấy ra đó
có bốn lần lấy được 2 bánh mặn và một lần lấy được 2 bánh ngọt.

Gọi  là không gian mẫu của phép thử.


Gọi A là biến cố “Trong năm lần lấy ra có bốn lần lấy được 2 bánh mặn và một lần
lấy được 2 bánh ngọt”.
 n()  (C82 )5 , n(A)  5.(C52 ) 4 .C32
5.(C52 )4 .C32 9375
 P(A)    0,0087
(C82 )5 1075648

Câu 47. Cho tập A  0;1; 2; 4;5;7;8 .Gọi X là tập hợp các số tự nhiên có 4 chữ số phân
biệt lấy từ A.Tính số phần tử của X.Lấy ngẫu nhiên một số từ tập X,tính xác suất để số
lấy được là số chẵn.
NGUYỄN VĂN LỰC  0933.168.309 SP Toán 35 - ĐH Cần Thơ
Tổ hợp – Xác suất FB: http://www.facebook.com/VanLuc168

+) Xét các số tự nhiên có 4 chữ số phân biệt lấy từ A, giả sử các số đó có dạng:
abcd , a  0.
Chọn a  0 , có 6 cách chọn, chọn các chữ số b, c, d  a và xếp thứ tự có: A63  120 cách.
 có tất cả: 6.120 = 720 số tự nhiên như vậy.
Vậy số phần tử của X là: 720. Số phần tử của không gian mẫu là: n()  720 .
+) Gọi B là biến cố: “Số tự nhiên được chọn là số chẵn”.
+) Xét các số tự nhiên chẵn có 4 chữ số phân biệt lấy từ A, giả sử các số đó có dạng:
a1a2 a3a4 , a1  0, a4 0; 2; 4;8 .
+) TH1: a4  0 , có 1 cách chọn; chọn các chữ số a1 , a2 , a3  0 và xếp thứ tự có A63  120
cách chọn  TH1 có: 1.120 = 120 số tự nhiên như vậy.
+) TH2: a4  2; 4; 6 , có 3 cách chọn; chọn a1  A \ 0; a4  , có 5 cách chọn; chọn các
chữ số a2 , a3  A \ a1; a4  và xếp thứ tự có A52  20 cách chọn  TH2 có: 3.5.20 = 300 số
tự nhiên như vậy.
 có tất cả: 120 + 300 = 420 số tự nhiên như vậy  Số phần tử thuận lợi cho biến cố
B là: n(B) = 420.
n( B) 420 7
+) Vậy: P( B)    .
n() 720 12

Câu 48. Một nhóm gồm 6 học sinh có tên khác nhau, trong đó có hai học sinh tên là
An và Bình. Xếp ngẫu nhiên nhóm học sinh đó thành một hàng dọc. Tính xác suất sao
cho hai học sinh An và Bình đứng cạnh nhau.

Mỗi cách xếp ngẫu nhiên 6 học sinh thành 1 hàng dọc là một hoán vị của 6 phần tử
 n()  6!  720 (phần tử)

Gọi A là biến cố: "An và Bình đứng cạnh nhau".


 n( A)  5!.2!  240 (phần tử)
n( A) 240 1
 P( A)    (phần tử)
n() 720 3

Câu 49. Trong đợt thi học sinh giỏi của tỉnh Nam Định trường THPT Xuân Trường
môn Toán có 5 em đạt giải trong đó có 4 nam và 1 nữ, môn Văn có 5 em đạt giải trong
đó có 1 nam và 4 nữ, môn Hóa học có 5 em đạt giải trong đó có 2 nam và 3 nữ, môn
Vật lí có 5 em đạt giải trong đó có 3 nam và 2 nữ. Hỏi có bao nhiêu cách chọn mỗi
môn một em học sinh để đi dự đại hội thi đua? Tính xác suất để có cả học sinh nam và
nữ để đi dự đại hội?

Có tất cả 5.5.5.5=625 cách  n(Ω)  625


Gọi A là biến cố “có cả HS nam và nữ đi dự đại hội”
 A là biến cố “Cả bốn HS nam hoặc cả 4 HS nữ đi dự ĐH”

NGUYỄN VĂN LỰC  0933.168.309 SP Toán 35 - ĐH Cần Thơ


Tổ hợp – Xác suất FB: http://www.facebook.com/VanLuc168

 n(A)  4.1.2.3  1.4.3.2  48  P A    n(A) 48



n(Ω) 625
Vậy P(A)  1  P  A   1 
48 577

625 625

Câu 50. Một xí nghiệp có 50 công nhân, trong đó có 30 công nhân tay nghề loại A, 15
công nhân tay nghề loại B, 5 công nhân tay nghề loại C. Lấy ngẫu nhiên theo danh
sách 3 công nhân. Tính xác suất để 3 người được lấy ra có 1 người tay nghề loại A, 1
người tay nghề loại B, 1 người tay nghề loại C.

Số phần tử của không gian mẫu n     C503  19600.


Số kết quả thuận lợi cho biến cố “trong 3 người được lấy ra, mỗi người thuộc 1 loại”
2250 45
là C301 .C151 .C51  2250 . Xác suất cần tính là p   .
19600 392

Câu 51. Có 30 tấm thẻ đánh số từ 1 đến 30. Chọn ngẫu nhiên ra 10 tấm thẻ. Tìm xác
suất để có 5 tấm thẻ mang số lẻ, 5 tấm thẻ mang số chẵn, trong đó chỉ có đúng 1 tấm
thẻ mang số chia hết cho 10.

Gọi  là tập hợp các cách chọn ra 10 tấm thẻ từ 30 tấm thẻ đã cho
Suy ra   C3010
Trong 30 tấm thẻ có 15 tấm thẻ mang số lẻ, 15 tấm thẻ mang số chẵn trong đó có 3
tấm thẻ mang số chia hết cho 10.
Gọi  A là tập hợp các cách chọn ra có 5 tấm thẻ mang số lẻ, 5 tấm thẻ mang số chẵn,
trong đó chỉ có đúng 1 tấm thẻ mang số chia hết cho 10
Suy ra  A  C155 .C124 .C31
C155 .C124 .C31 99
Vậy P  A  10
 .
C30 667

Câu 52. Đội văn nghệ của một lớp có 5 bạn nam và 7 bạn nữ. Chọn ngẫu nhiên 5 bạn
tham gia biểu diễn, tìm xác suất để trong 5 bạn được chọn có cả nam và nữ, đồng thời
số bạn nam nhiều hơn số bạn nữ.

Đội văn nghệ của một lớp có 5 bạn nam và 7 bạn nữ. Chọn ngẫu nhiên 5 bạn tham gia
biểu diễn, tìm xác suất để trong 5 bạn được chọn có cả nam và nữ, đồng thời số bạn
nam nhiều hơn số bạn nữ.
Số cách chọn 5 bạn bất kì là: C125  729 . Để chọn được 5 bạn thỏa mãn yêu cấu bài toán,
ta có hai khả năng sau:
-TH1: Chọn 4 bạn nam và 1 bạn nữ, có C54 .C71  35 cách chọn.
-TH2: Chọn 3 bạn nam và 2 bạn nữ, có C53 .C72  210 cách chọn.
35  210 245
Vậy xác suất cần tìm là: P   .
729 729

NGUYỄN VĂN LỰC  0933.168.309 SP Toán 35 - ĐH Cần Thơ


Tổ hợp – Xác suất FB: http://www.facebook.com/VanLuc168
Câu 53. Trong cuộc thi “Rung chuông vàng” có 20 bạn lọt vào vòng chung kết, trong
đó có 5 bạn nữ và 15 bạn nam. Để sắp xếp vị trí chơi, ban tổ chức chia các bạn thành 4
nhóm A, B, C, D, mỗi nhóm có 5 bạn. Việc chia nhóm được thực hiên bằng cách bốc
thăm ngẫu nhiên. Tính xác suất để 5 bạn nữ thuộc cùng một nhóm.

Chia 20 học sinh thành 4 nhóm nên số phần tử của không gian mẫu là
  C20
5
.C155 .C105 .C55
Gọi A là biến cố “ Chia 20 học sinh thành 4 nhóm sao cho 5 bạn nữ thuộc cùng một
nhóm”
Xét 5 bạn nữ thuộc một nhóm có C155 .C105 .C55 cách chia 15 nam vào 3 nhóm còn lại
Vì 5 bạn nữ có thể thuộc nhóm A,B,C hay D nên ta có  A  4.C155 .C105 .C55 .
A 4.C155 .C105 .C55 1
Vậy xác suất của biến cố A là P( A)    .
 5 5 5 5
C20 .C15 .C10 .C5 3876

Câu 54. Trong bộ môn Toán, thầy giáo có 40 câu hỏi khác nhau gồm 5 câu hỏi khó,
15 câu hỏi trung bình, 20 câu hỏi dễ. Một ngân hàng đề thi mỗi đề thi có 7 câu hỏi
đựơc chọn từ 40 câu hỏi đó. Tính xác suất để chọn được đề thi từ ngân hàng đề nói
trên nhất thiết phải có đủ 3 loại câu hỏi (khó, trung bình, dễ) và số câu hỏi dễ không ít
hơn 4.

Không gian mẫu của việc tạo đề thi là :   C 407  18643560


Gọi A là biến cố chọn đựợc đề thi có đủ 3 loại câu hỏi(khó, trung bình, dễ) và số câu
hỏi dễ không ít hơn 4.
 A  C 204 .C52 .C151  C 204 .C51 .C152  C 20
5
.C51C151  4433175
A 915
Xác suất cần tìm là P( A)  
 3848

Câu 55. Một đội văn nghệ gồm có 20 người trong đó có 12 nam và 8 nữ. Chọn ngẫu
nhiên 8 người để hát đồng ca. Tính xác suất để 8 người được chọn có cả nam và nữ và
số nữ nhiều hơn số nam.

+) Xét phép thử chọn ngẫu nhiên 8 người từ 20 người, mỗi kết quả của phép thử ứng
với một cách chọn được 8 người từ 20 người => Số phần tử của không gian mẫu là:
8
n()  C20  125970 .
+) Gọi biễn cố A: “8 người được chọn có cả nam và nữ và số nữ nhiều hơn số nam”
n( A)  C85.C12
3
 C86 .C12
2
 C87 .C12
1
 14264
Ta có n( A) 14264 7132
 P( A)    .
n() 125970 62985

Câu 56. Một hộp chứa 4 viên bi trắng, 5 viên bi đỏ và 6 viên bi xanh. Lấy ngẫu nhiên
từ hộp ra 4 viên bi. Tính xác xuất để 4 viên bi được chon có đủ 3 màu và số bi đỏ
nhiều nhất.

Ta có: n     C 15  1365
4

NGUYỄN VĂN LỰC  0933.168.309 SP Toán 35 - ĐH Cần Thơ


Tổ hợp – Xác suất FB: http://www.facebook.com/VanLuc168
Gọi A là biến cố “4 viên bi được chọn có đủ 3 màu và số bi đỏ nhiều nhất’
Khi đó n  A  C 14C 52C 16  240
n  A  16
Vậy p  A   
n    91

Câu 57. Đội văn nghệ của nhà trường gồm 4 học sinh lớp 12A, 3 học sinh lớp 12B và
2 học sinh lớp 12C. Chọn ngẫu nhiên 5 học sinh từ đội văn nghệ để biểu diễn trong lễ
bế giảng năm học. Tính xác suất sao cho lớp nào cũng có học sinh được chọn và có ít
nhất 2 học sinh lớp 12A.

Gọi không gian mẫu của phép chọn ngẫu nhiên là 


Số phần tử của không gian mẫu là: C95  126
Gọi A là biến cố “Chọn 5 học sinh từ đội văn nghệ sao cho có học sinh ở cả ba lớp và
có ít nhất 2 học sinh lớp 12A”.
Chỉ có 3 khả năng xảy ra thuận lợi cho biến cố A là :
+ 2 học sinh lớp 12A, 1 học sinh lớp 12B, 2 học sinh lớp 12C
+ 2 học sinh lớp 12A, 1 học sinh lớp 12B, 2 học sinh lớp 12C
+ 3 học sinh lớp 12A, 1 học sinh lớp 12B, 1 học sinh lớp 12C
Số kết quả thuận lợi cho biến cố A là: C42 .C31.C22  C42 .C32 .C21  C43.C31.C21  78 .
78 13
Xác suất cần tìm là P   .
126 21

Câu 58. Một ngân hàng đề thi gồm 20 câu hỏi. Mỗi đề thi gồm 4 câu được lấy ngẫu
nhiên từ 20 câu hỏi trên. Thí sinh A đã học thuộc 10 câu trong ngân hàng đề thi. Tìm
xác suất để thí sinh A rút ngẫu nhiên được 1 đề thi có ít nhất 2 câu đã thuộc.

Lấy ngẫu nhiên từ ngân hàng đề thi 4 câu hỏi để lập một đề thi có C204  4845 đề thi.
Thí sinh A rút ngẫu nhiên được 1 đề thi có 2 câu đã thuộc, có C102 .C102  2025 trường hợp.
Thí sinh A rút ngẫu nhiên được 1 đề thi có 3 câu đã thuộc, có C103 .C101  1200 trường hợp.
Thí sinh A rút ngẫu nhiên được 1 đề thi có 4 câu đã thuộc, có C104  210 trường hợp.
Do đó, thí sinh A rút ngẫu nhiên được 1 đề thi có ít nhất 2 câu đã thuộc, có
2025  1200  210  3435 trường hợp
Vậy xác suất để thí sinh A rút ngẫu nhiên được 1 đề thi có ít nhất 2 câu đã thuộc là
3435 229
 .
4845 323

Câu 59. Một hộp đựng 4 viên bi trắng, 5 viên bi đỏ và 6 bi xanh. Lấy ngẫu nhiên từ
hộp đó ra 4 viên bi, tính xác suất để 4 viên bi được chọn có đủ ba màu và số bi xanh
nhiều nhất.

+ Số phần tử của không gian mẫu là:


+ Gọi A là biến cố “ 4 viên bi được chọn có đủ cả ba màu và bi xanh nhiều nhất”

NGUYỄN VĂN LỰC  0933.168.309 SP Toán 35 - ĐH Cần Thơ


Tổ hợp – Xác suất FB: http://www.facebook.com/VanLuc168
Số phần tử của biến cố A là n( A)  C .C .C  300
1 1 2
4 5 6

+ Vậy xác suất của biến cố A là :

Câu 60. Đội dự tuyển học sinh giỏi giải toán trên máy tính cầm tay môn toán của một
trường phổ thông có 4 học sinh nam khối 12, 2 học sinh nữ khối 12 và 2 học sinh nam
khối 11. Để thành lập đội tuyển dự thi học sinh giỏi giải toán trên máy tính cầm tay
môn toán cấp tỉnh nhà trường cần chọn 5 em từ 8 em học sinh trên. Tính xác suất để
trong 5 em được chọn có cả học sinh nam và học sinh nữ, có cả học sinh khối 11 và
học sinh khối 12.

- Số cách chọn 5 em học sinh từ 8 học sinh trên là C85 = 56 cách

- Để chọn 5 em thỏa mãn bài ra, ta xét các trường hợp sau

+) 1 nam khối 11, 1 nữ khối 12 và 3 nam khối 12 có: C21C21C43 cách

+) 1 nam khối 11, 2 nữ khối 12 và 2 nam khối 12 có: C21C22C42 cách

+) 2 nam khối 11, 1 nữ khối 12 và 2 nam khối 12 có: C22C21C42 cách

+) 2 nam khối 11, 2 nữ khối 12 và 1 nam khối 12 có: C22C22C41 cách

Số cách chọn 5 em thỏa mãn bài ra là:

C21C21C43 + C21C22C42 + C22C21C42 + C22C22C41 = 44 cách

44 11
Vậy xác suất cần tính là: 
56 14

Câu 61. Trong kì thi học sinh giỏi cấp tỉnh của trường THPT Phù Cừ có 10 học sinh
đạt giải trong đó có 4 học sinh nam và 6 học sinh nữ. Nhà trường muốn chọn một
nhóm 5 học sinh trong 10 học sinh trên để tham dự buổi lễ tuyên dương khen thưởng
cuối học kỳ 1 năm học 2015 – 2016 do huyện uỷ Phù Cừ tổ chức. Tính xác suất để
chọn được một nhóm gồm 5 học sinh mà có cả nam và nữ, biết số học sinh nam ít hơn
số học sinh nữ.

Không gian mẫu n     C 105  252


Gọi A là biến cố 5 học sinh được chọn có cả nam và nữ đồng thời số học sinh nam ít
hơn học sinh nữ.
Trường hợp 1: Chọn 1 học sinh nam và 4 học sinh nữ nên ta có C 41.C 64
Trường hợp 2: Chọn 2 học sinh nam và 3 học sinh nữ nên ta có C 42 .C 63
Suy ra n A  C 41.C 64  C 42 .C 63  180
NGUYỄN VĂN LỰC  0933.168.309 SP Toán 35 - ĐH Cần Thơ
Tổ hợp – Xác suất FB: http://www.facebook.com/VanLuc168
5
Vậy xác suất cần tìm là P  A 
7

Câu 62. Trong cụm thi để xét công nhận tốt nghiệp THPT thí sinh phải thi 4 môn
trong đó có 3 môn bắt buộc là Toán, Văn, Ngoại ngữ và một môn do thí sinh tự chọn
trong số các môn: Vật lí, Hóa học, Sinh học, Lịch sử và Địa lí. Trường A có 30 học
sinh đăng kí dự thi, trong đó có 10 học sinh chọn môn Lịch sử. Lấy ngẫu nhiên 5 học
sinh bất kỳ của trường A, tính xác suất để trong 5 học sinh đó có nhiều nhất 2 học sinh
chọn môn Lịch sử.

Số phần tử của không gian mẫu là: n()  C305  142506

Gọi A là biến cố : “5 học sinh được chọn có nhiều nhất 2 học sinh chọn môn lịch sử”

Số phần tử của biến cố A là: n( A)  C205  C204 C101  C203 C102  115254

115254
Vậy xác suất cần tìm là: P( A)   0,81 .
142506

Câu 63. Gọi A là tập hợp tất cả các số tự nhiên gồm 4 chữ số phân biệt được chọn từ
các chữ số 0; 1; 2; 3; 4; 5; 6. Chọn ngẫu nhiên một số từ tập A, tính xác suất để số chọn
được là số chia hết cho 5.

Số phần tử của A là 6.A36  720


Số cách chọn một số có hàng đơn vị là số 0 có 1.A36  120 cách
Số cách chọn một số có hàng đơn vị là số 5 có 1.5.A52  100 cách
Suy ra số cách chọn một số chia hết cho 5 là 120  100  220 cách
220 11
Vậy xác suất cần tìm bằng  .
720 36

Câu 64. Cho X là tập hợp gồm 6 số tự nhiên lẻ và 4 số tự nhiên chẵn. Chọn ngẫu
nhiên từ tập X ba số tự nhiên. Tính xác suất chọn được ba số tự nhiên có tích là một số
chẵn.

Phép thử T: “Chọn ngẫu nhiên từ tập X ba số tự nhiên”.


 Số phần tử của không gian mẫu là: n()  C10
3
 120.
Gọi A là biến cố “Chọn được ba số tự nhiên có tích là một số chẵn”.
 A là biến cố “Chọn được ba số tự nhiên có tích là một số lẻ”
Chọn được 3 số tự nhiên lẻ có C36 cách.
 n(A)  C36  20.
n(A) 20 1
Do đó: P(A)    
n() 120 6

NGUYỄN VĂN LỰC  0933.168.309 SP Toán 35 - ĐH Cần Thơ


Tổ hợp – Xác suất FB: http://www.facebook.com/VanLuc168
1 5
Vậy P(A)  1  P(A)  1   
6 6

Câu 65. Gọi M là tập hợp các số có 4 chữ số đôi một khác nhau lập từ các chữ số 1, 2,
3, 4, 5, 6, 7. Lấy ra từ tập M một số bất kỳ. Tính xác suất để lấy được số có tổng các
chữ số là số lẻ ?

Gọi A là biến cố "Số chọn được là số có 4 chữ số đôi một khác nhau và tổng các chữ
số là một số lẻ". Số các số có 4 chữ số đôi một khác nhau lập từ 7 chữ số đã cho là
A74  840 (số), suy ra:   840
Gọi số 4 chữ số đôi một khác nhau và tổng các chữ số là một số lẻ có dạng abcd . Do
tổng a  b  c  d là số lẻ nên số chữ số lẻ là lẻ
Trường hợp 1 : có 1 chữ số lẻ , 3 chữ số chẵn : có C41.C33  4 bộ số
Trường hợp 2 : có 3 chữ số lẻ , 1 chữ số chẵn : có C43.C31  12 bộ số
Từ mỗi bộ số trên ta lập được P4  24 số
Tất cả có 16.24= 384 số , suy ra:  A  384 .
A 384 48
Vậy P( A)    .
 840 105

Câu 66. Gọi A là tập hợp tất cả các số tự nhiên gồm các chữ số đôi một khác nhau
được lập từ các chữ số 1, 2, 3, 4, 5. Lấy ngẫu nhiên một số trong A , tính xác suất để
lấy được số có chứa chữ số 3.

+ Số các số có một, hai, ba, bốn, năm chữ số phân biệt lần lượt là:
A51 , A52 , A53 , A54 , A55 . Vậy tập A có A51 + A52 + A53 + A54 + A55 = 325 số.
+ Tương tự, số các số của A không có chữ số 3 là: A41  A42  A43  A44  64 số.
Vậy số các số có chứa chữ số 3 là: 325 – 64 = 261 số

Từ đó xác suất cần tìm là P = 261/325

NGUYỄN VĂN LỰC  0933.168.309 SP Toán 35 - ĐH Cần Thơ


PT – BPT – HPT FB: http://www.facebook.com/VanLuc168

I. KIẾN THỨC TRỌNG TÂM


Chuyên đề: PT – BPT - HPT

Nhắc lại:

1) Một số phép biến đổi tương đương phương trình thường sử dụng
a) Chuyển vế một biểu thức từ vế này sang vế kia (nhớ đổi dấu của biểu thức).
b) Nhân hoặc chia hai vế của phương trình với một hằng số (khác 0) hoặc với
một biểu thức
(khác không).
c) Thay thế một biểu thức bởi một biểu thức khác bằng với biểu thức đó.
Lưu ý:
+ Chia hai vế của phương trình cho biểu thức chứa ẩn đề phòng mất nghiệm.
+ Bình phương hai vế của phương trình đề phòng dư nghiệm.

2) Các bước giải một phương trình


Bước 1: Tìm điều kiện (nếu có) của ẩn số để hai vế của pt có nghĩa

Bước 2: Sử dụng các phép biến đổi tương đương để biến đổi pt đến một pt đã biết
cách giải

Bước 3: Giải pt và chọn nghiệm phù hợp ( nếu có)

Bước 4: Kết luận

3. Các phương pháp giải phương trình đại số thường sử dụng


a) Phương pháp 1: Biến đổi phương trình đã cho về phương trình đã biết cách giải

b) Phương pháp 2: Biến đổi phương trình đã cho về dạng tích số : A.B=0; A.B.C=0.
A  0
A  0
Định lý: A.B  0   ; A.B.C  0   B  0
B  0 C  0
c) Phương pháp 3: Đặt ẩn phụ đưa phương trình đã cho về dạng đã biết cách giải.

NGUYỄN VĂN LỰC  0933.168.309 SP Toán K35 - ĐH Cần Thơ


PT – BPT – HPT FB: http://www.facebook.com/VanLuc168

A. PHƯƠNG TRÌNH ĐẠI SỐ

I. Giải và biện luận phương trình bậc nhất:

x : aån soá
1. Dạng : ax + b = 0 (1) 
a, b : tham soá

2. Giải và biện luận:

Ta có : (1)  ax = -b (2)
Biện luận:
b
 Nếu a  0 thì (2)  x  
a
 Nếu a = 0 thì (2) trở thành 0.x = -b
* Nếu b  0 thì phương trình (1) vô nghiệm
* Nếu b = 0 thì phương trình (1) nghiệm đúng với mọi x

Tóm lại :
b
 a  0 : phương trình (1) có nghiệm duy nhất x  
a
 a = 0 và b  0 : phương trình (1) vô nghiệm
 a = 0 và b = 0 : phương trình (1) nghiệm đúng với mọi x

3. Điều kiện về nghiệm số của phương trình:

Định lý: Xét phương trình ax + b = 0 (1) ta có:

 (1) có nghiệm duy nhất  a 0


a  0
 (1) vô nghiệm  
b  0
a  0
 (1) nghiệm đúng với mọi x  
b  0

II. Giải và biện luận phương trình bậc hai:

x : aån soá
1. Dạng: ax 2  bx  c  0 (1) 
a, b , c : tham soá

NGUYỄN VĂN LỰC  0933.168.309 SP Toán K35 - ĐH Cần Thơ


PT – BPT – HPT FB: http://www.facebook.com/VanLuc168

2. Giải và biện luận phương trình :

Xét hai trường hợp


Trường hợp 1: Nếu a  0 thì (1) là phương trình bậc nhất : bx + c = 0
c
 b  0 : phương trình (1) có nghiệm duy nhất x  
b
 b = 0 và c  0 : phương trình (1) vô nghiệm
 b = 0 và c = 0 : phương trình (1) nghiệm đúng với mọi x
Trường hợp 2: Nếu a  0 thì (1) là phương trình bậc hai có
b
Biệt số   b2  4ac ( hoặc  '  b '2  ac vôùi b'  )
2
Biện luận:
 Nếu   0 thì pt (1) vô nghiệm

b b'
 Nếu   0 thì pt (1) có nghiệm số kép x1  x2   ( x1  x2   )
2a a

b    b'   '
 Nếu   0 thì pt (1) có hai nghiệm phân biệt x1,2  ( x1,2  )
2a a

3. Điều kiện về nghiệm số của phương trình bậc hai:


Định lý : Xét phương trình : ax 2  bx  c  0 (1)

a  0
 a  0
 Pt (1) vô nghiệm  b  0 hoặc 
c  0   0

a  0
 Pt (1) có nghiệm kép  
  0

a  0
 Pt (1) có hai nghiệm phân biệt  
  0

a  0
 Pt (1) có hai nghiệm  
  0

a  0

 Pt (1) nghiệm đúng với mọi x  b  0
c  0

Đặc biệt
Nếu pt(1) có hệ số a,c thoả a.c < 0 thì pt(1) luôn có hai nghiệm phân biệt.

NGUYỄN VĂN LỰC  0933.168.309 SP Toán K35 - ĐH Cần Thơ


PT – BPT – HPT FB: http://www.facebook.com/VanLuc168

4. Định lý VIÉT đối với phương trình bậc hai:


 Định lý thuận: Nếu phương trình bậc hai : ax 2  bx  c  0 ( a  0 ) có hai nghiệm x1,
x2 thì

 b
S  x1  x 2   a

 P  x .x  c

1 2
a

 Định lý đảo : Nếu có hai số x, y mà x  y  S và x.y  P ( S 2  4 P) thì x, y là nghiệm


của phương trình
X2 S.X P 0

 Ý nghĩa của định lý VIÉT:


Cho phép tính giá trị các biểu thức đối xứng của các nghiệm ( tức là biểu thức chứa
x1, x2 và không thay đổi giá trị khi ta thay đổi vai trò x1,x2 cho nhau .Ví dụ:
x12  x22 1 1
A  2  2 ) mà không cần giải pt tìm x1, x2 , tìm hai số khi biết tổng và
x1 x2 x1 x2
tích của chúng,...
Chú ý:
c
 Nếu pt (1) có các hệ số thoả mãn a b c 0 thì pt (1) có hai nghiệm là x1  1 vaø x 2 
a
 Nếu pt (1) có các hệ số thoả mãn a b c 0 thì pt (1) có hai nghiệm là
c
x1  1 vaø x 2  
a

5. Dấu nghiệm số của phương trình bậc hai:


Dựa vào định lý Viét ta có thể suy ra định lý sau:

Định lý: Xét phương trình bậc hai : ax 2  bx  c  0 (1) ( a  0)


 > 0

 Pt (1) có hai nghiệm dương phân biệt  P > 0
S > 0

 > 0

 Pt (1) có hai nghiệm âm phân biệt  P > 0
S < 0

 Pt (1) có hai nghiệm trái dấu  P<0

II. Phương trình trùng phương:

1.Dạng : ax 4  bx 2  c  0 (a  0) (1)
2.Cách giải:
NGUYỄN VĂN LỰC  0933.168.309 SP Toán K35 - ĐH Cần Thơ
PT – BPT – HPT FB: http://www.facebook.com/VanLuc168

 Đặt ẩn phụ : x2= t ( t  0 ). Ta được phương trình: at 2  bt  c  0 (2)


Giải pt (2) tìm t. Thay t tìm được vào x2= t để tìm x.
Tùy theo số nghiệm của phương trình (2) mà ta suy ra được số nghiệm
của phương trình (1)

Định lý:
Phương trình (1) có 4 nghiệm phân biệt  Phương trình (2) có 2 nghiệm dương phân biệt

III . Phương trình bậc ba:

1. Dạng: ax 3  bx 2  cx  d  0 (1) (a  0)

2 .Cách giải: Áp dụng khi biết được một nghiệm của phương trình (1)

Bước 1: Nhẩm một nghiệm của phương trình (1). Giả sử nghiệm là x = x 0
Bước 2: Sử dụng phép CHIA ĐA THỨC hoặc sơ đồ HOÓCNE để phân tích vế
trái thành nhân tử và đưa pt (1) về dạng tích số :
(1)  (x-x0)(Ax2+Bx+C) = 0
 x  x0
  2
 Ax  Bx  C  0 (2)
Sơ đồ Hoocne:
a b c d
x0 A B C 0 (số 0)
Trong đó:
a  A, x 0 .A  b  B, x 0 .B  c  C, x 0 .C  d  0

Bước 3: Giải phương trình (2) tìm các nghiệm còn lại ( nếu có)

Định lý: Phương trình (1) có 3 nghiệm phân biệt  Phương trình (2) có 2 nghiệm phân
biệt khác x0

Chú ý Ta có thể áp dụng phương pháp phân tích đa thức thành nhân tử bằng kỹ thuật sử
dụng sơ đồ HOÓCNE, để giải các phương trình đa thức bậc cao (với điều kiện nhẩm được
một nghiệm của đa thức).
IV. PHƯƠNG TRÌNH BẬC BỐN QUY VỀ BẬC HAI BẰNG PHÉP ĐẶT ẨN PHỤ

1.Dạng I: ax 4  bx 2  c  0 (a  0)

 Đặt ẩn phụ : t = x2

NGUYỄN VĂN LỰC  0933.168.309 SP Toán K35 - ĐH Cần Thơ


PT – BPT – HPT FB: http://www.facebook.com/VanLuc168

2. Dạng II. ( x  a)( x  b)( x  c)( x  d )  k (k  0) trong đó a+b = c+d

 Đặt ẩn phụ : t = (x+a)(x+b)

3.Dạng III: ( x  a )4  ( x  b) 4  k (k  0)

ab
 Đặt ẩn phụ : t = x 
2

4.Dạng IV: ax 4  bx 3  cx 2  bx  a  0

Chia hai vế phương trình cho x2


1
 Đặt ẩn phụ : t = x 
x

NGUYỄN VĂN LỰC  0933.168.309 SP Toán K35 - ĐH Cần Thơ


PT – BPT – HPT FB: http://www.facebook.com/VanLuc168

B. BẤT PHƯƠNG TRÌNH ĐẠI SỐ

Nhắc lại:
Các phép biến đổi tương đương bất phương trình thường sử dụng:
1) Chuyển vế một biểu thức của bpt từ vế này sang vế kia (nhớ đổi dấu biểu thức)
2) Nhân hoặc chia hai vế của bpt với một hằng số hoặc một biểu thức khác 0
Ghi nhớ quan trọng:
+ Âm thì đổi chiều
+ Dương thì không đổi chiều
3) Thay thế một biểu thức trong bpt bởi một biểu thức khác bằng với biểu thức đó.

I. Bất phương trình bậc nhất:


1. Dạng : ax  b  0 (1) (hoặc , ,  )
2. Giải và biện luận:

Ta có : (1)  ax  b (2)
Biện luận:
b
 Nếu a  0 thì ( 2)  x  
a
b
 Nếu a  0 thì ( 2)  x  
a
 Nếu a  0 thì (2) trở thành : 0.x  b
* b  0 thì bpt vô nghiệm
* b  0 thì bpt nghiệm đúng với mọi x

II. Dấu của nhị thức bậc nhất:


1. Dạng: f ( x)  ax  b (a  0)
2. Bảng xét dấu của nhị thức:

x b
  
a
ax+b Trái dấu với a 0 Cùng dấu với a

NGUYỄN VĂN LỰC  0933.168.309 SP Toán K35 - ĐH Cần Thơ


PT – BPT – HPT FB: http://www.facebook.com/VanLuc168

III. Dấu của tam thức bậc hai:


1. Dạng: f ( x)  ax 2  bx  c (a  0)
2. Bảng xét dấu của tam thức bậc hai:
x  
f(x) Cùng dấu a
0

x b
0   
2a
  b2  4ac Cùng dấu a 0 Cùng dấu a
f(x)

0 x  x1 x2 
f(x) Cùng dấu a 0 Trái dấu a 0 Cùng dấu a

Chú ý:
 Nếu tam thức bậc hai f(x) ax2 bx c (a 0) có hai nghiệm x1, x2 thì tam thức
luôn có thể phân tích thành

f(x) ax2 bx c a x x1 x x2

 Mọi tam thức bậc hai f(x) = ax2+bx+c (a0) điều có thể biểu diển thành

b 2 
f ( x )  ax 2  bx  c  a( x  ) 
2a 4a

3. Điều kiện không đổi dấu của tam thức:


Định lý: Cho tam thức bậc hai: f ( x)  ax 2  bx  c (a  0)
  0   0
 f (x)  0 x  R    f (x)  0 x  R  
a  0 a  0
  0   0
 f (x)  0 x  R    f (x)  0 x  R  
a  0 a  0

IV. Bất phương trình bậc hai:

1. Dạng: ax 2  bx  c  0 ( hoặc , ,  )
2. Cách giải: Xét dấu tam thức bậc hai ở vế trái rồi chọn nghiệm thích hợp.

NGUYỄN VĂN LỰC  0933.168.309 SP Toán K35 - ĐH Cần Thơ


PT – BPT – HPT FB: http://www.facebook.com/VanLuc168

C. PHƯƠNG TRÌNH & BẤT PT CHƯA DẤU GIÁ TRỊ TUYỆT ĐÔI

TRỌNG TÂM KIẾN THỨC

I. Định nghĩa và các tính chất cơ bản:


A neáu A  0
1. Định nghĩa: A 
 A neáu A < 0
2. Tính chất :
2
A 0 , A  A2

Lưu ý: A2 A

II. Các định lý cơ bản:


a) Định lý 1 : Với A  0 và B  0 thì A = B  A2 = B2

b) Định lý 2 : Với A  0 và B  0 thì A > B  A2 > B2

III. Các phương trình và bất phương trình chứa giá trị tuyệt đối cơ bản & cách
giải:
Phương pháp chung để giải loại này là KHỬ DẤU GIÁ TRỊ TUYỆT ĐỐI bằng
định nghĩa hoặc nâng lũy thừa.
* Dạng 1 : A  B  A 2  B 2 , A  B  A   B
 A  0

B  0 B  0 A  B
* Dạng 2 : A  B   , A B , A B
A  B  A  0
A  B
2 2

 A  B

 A  0

B  0 B  0 A  B
* Dạng 4: A B 2 , A  B   , AB
 2
     A  0
 A B B A B

 A  B
B  0 B  0
 
* Dạng 5: A  B    B  0 , A  B   B  0
 A 2  B 2  A  B  A  B


NGUYỄN VĂN LỰC  0933.168.309 SP Toán K35 - ĐH Cần Thơ


PT – BPT – HPT FB: http://www.facebook.com/VanLuc168

D. PHƯƠNG TRÌNH & BẤT PT CHỨA CĂN

TRỌNG TÂM KIẾN THỨC

I. Các điều kiện và tính chất cơ bản:

* A có nghĩa khi A  0
* A  0 với A  0
 A neáu A  0
* A2  A & A 
- A neáu A  0
*  A
2
A với A  0
* A.B  A. B khi A , B  0
* A.B   A.  B khi A , B  0

II. Các định lý cơ bản: (quan trọng)

a) Định lý 1 : Với A 0 và B 0 thì A=B A2 = B2


b) Định lý 2 : Với A 0 và B 0 thì A>B A2 > B2
c) Định lý 3: Với A và B bất kỳ thì A=B A2 = B2

III. Các phương trình và bất phương trình căn thức cơ bản & cách giải :

Phương pháp chung để giải loại này là KHỬ CĂN THỨC bằng pháp nâng lũy thừa.

A  0 (hoaëc B  0 )
* Dạng 1 : A B
A  B
B  0

* Dạng 2 : A  B  2
A  B

A  0

* Dạng 3 : A  B  B  0
 2
A  B
A  0

 B  0
* Dạng 4: A B 
B  0
 
  A  B2

NGUYỄN VĂN LỰC  0933.168.309 SP Toán K35 - ĐH Cần Thơ


PT – BPT – HPT FB: http://www.facebook.com/VanLuc168

CÁC HỆ PHƯƠNG TRÌNH CƠ BẢN

I. Hệ phương trình bậc nhất nhiều ẩn

1. Hệ phương trình bậc nhất hai ẩn

a1 x  b1y  c1
a. Dạng :  (1) a12 b12 0, a22 b22 0
a2 x  b2 y  c2

Cách giải đã biết: Phép thế, phép cộng, sử dụng MTBT...

b. Giải và biện luận phương trình : Quy trình giải và biện luận
Bước 1: Tính các định thức :
a1 b1
 D  a1b2  a 2 b1 (gọi là định thức của hệ)
a2 b2
c1 b1
 Dx   c1b2  c2 b1 (gọi là định thức của x)
c2 b2
a1 c1
 Dy   a1c2  a 2 c1 (gọi là định thức của y)
a2 c2
Bước 2: Biện luận
 Dx
 x  D
 Nếu D  0 thì hệ có nghiệm duy nhất 
 y  Dy
 D
 Nếu D = 0 và D x  0 hoặc Dy  0 thì hệ vô nghiệm.
 Nếu D = Dx = Dy = 0 thì hệ có vô số nghiệm.

2. Hệ phương trình bậc nhất ba ẩn


a1 x  b1 y  c1z  d1

Dạng : a2 x  b2 y  c2 z  d2
a x  b y  c z  d
 3 3 3 3

Cách giải: Sử dụng phép cộng để khử một ẩn đưa về hệ bậc nhất hai ẩn, sử dụng MTBT.

II. Hệ phương trình bậc hai hai ẩn:

1. Hệ gồm một phương trình bậc nhất và một phương trình bậc hai hai ẩn:
Cách giải: Giải bằng pháp thế

2. Hệ phương trình đối xứng :

NGUYỄN VĂN LỰC  0933.168.309 SP Toán K35 - ĐH Cần Thơ


PT – BPT – HPT FB: http://www.facebook.com/VanLuc168

1. Hệ phương trình đối xứng loại I:

a. Định nghĩa: Đó là hệ chứa hai ẩn x,y mà khi ta thay đổi vai trò x,y cho nhau thì hệ
phương trình không thay đổi.
b. Cách giải:

Bước 1: Đặt x+y=S và xy=P với S 2  4 P ta đưa hệ về hệ mới chứa hai ẩn S,P.
Bước 2: Giải hệ mới tìm S,P . Chọn S,P thoả mãn S 2  4 P .
Bước 3: Với S,P tìm được thì x,y là nghiệm của phương trình :
X 2  SX  P  0 ( định lý Viét đảo ).

Chú ý: Do tính đối xứng, cho nên nếu (x0;y0) là nghiệm của hệ thì (y0;x0) cũng là nghiệm
của hệ.

2. Hệ phương trình đối xứng loại II:


a. Định nghĩa: Đó là hệ chứa hai ẩn x,y mà khi ta thay đổi vai trò x,y cho nhau
thì phương trình nầy trở thành phương trình kia của hệ.
b. Cách giải:

 Trừ vế với vế hai phương trình và biến đổi về dạng phương trình tích số.
 Kết hợp một phương trình tích số với một phương trình của hệ để suy ra nghiệm của
hệ .

III. Hệ phương trình đẳng cấp bậc hai:


2 2
a1 x  b1 xy  c1 y  d1

a. Dạng :  2 2
a2 x  b2 xy  c2 y  d2

b. Cách giải:

x y x
Đặt ẩn phụ t hoặc t. Giả sử ta chọn cách đặt t.
y x y

Khi đó ta có thể tiến hành cách giải như sau:


Bước 1: Kiểm tra xem (x,0) có phải là nghiệm của hệ hay không ?
x
Bước 2: Với y  0 ta đặt t x ty . Thay vào hệ ta được hệ mới chứa 2 ẩn t, y.
y
Từ 2 phương trình ta khử y để được 1 phương trình chứa t .
Bước 3: Giải phương trình tìm t rồi suy ra x,y.

NGUYỄN VĂN LỰC  0933.168.309 SP Toán K35 - ĐH Cần Thơ


PT – BPT – HPT FB: http://www.facebook.com/VanLuc168

II. PHƯƠNG TRÌNH


Chuyên đề: PT – BPT - HPT

§ 1. CÁCH GIẢI PHƯƠNG TRÌNH DẠNG


x4 ax 2 bx c
(Tách bậc – đưa về phương trình tích)

Phương pháp giải


Với mọi m ta luôn có:
x4 ax 2 bx c x4 2mx 2 m 2 ax 2 bx c 2mx 2 m 2
( x 2 m) 2 (2m a) x 2 bx c m 2
(1)
Đặt f ( x) (2m a) x 2
bx c m . Ta tìm m sao cho f ( x ) trở thành bình phương của
2

một nhị thức


0
Điều nầy được thỏa khi: . Suy ra: f ( x) ( Ax B) 2
2m a 0
Khi đó: (1) ( x2 m) 2 ( Ax B)2 x2 m ( Ax B) . Đây là phương trình bậc hai.

Ví dụ 1: Giải phương trình x 4 3x 2 10 x 4 (1)

Lời giải

Với mọi m ta có: (1) ( x 2 m) 2 (3 2 m) x 2 10 x 4 m2 (2)


Đặt f ( x) (3 2m) x 2 10 x 4 m 2
25 (3 2m)(4 m2 ) 0
f ( x) là bình phương của một nhị thức
3 2m 0
2m3 3m2 8m 13 0
m 1
3 3m 0
Khi đó: (2) ( x2 1) 2 5x2 10 x 5 ( x2 1) 2 5( x 1) 2
x2 1 5( x 1) 5 1 4 5
x
x2 1 5( x 1) 2

5 1 4 5
Tập nghiệm của phương trình (1) là S 
2

NGUYỄN VĂN LỰC  0933.168.309 SP Toán K35 - ĐH Cần Thơ


PT – BPT – HPT FB: http://www.facebook.com/VanLuc168

Chú ý: Việc tìm ra m 1 có thể làm trên nháp, không cần trình bày trên bài làm.
Có thể trình bày ngắn gọn như ví dụ sau

7
Ví dụ 2: Giải phương trình x 4 2x2 3x (1)
16

Lời giải
9 3 2
Ta có: (1) ( x2 1) 2 4x2 3x ( x2 1) 2 (2 x )
16 4
3 1
x2 1 2x x2 2x 0
4 4 3
x 1
3 7 2
x2 1 2x x2 2x 0
4 4
3
Tập nghiệm của phương trình (1) là S 1 
2
Bài tập tương tự

Giải các phương trình

1) x 4 19 x 2 10 x 8 0

2) 2 x 4 3x 2 10 x 3 0

3) 3x 4 2 x 2 16 x 5 0

NGUYỄN VĂN LỰC  0933.168.309 SP Toán K35 - ĐH Cần Thơ


PT – BPT – HPT FB: http://www.facebook.com/VanLuc168

§ 2. CÁCH GIẢI PHƯƠNG TRÌNH DẠNG


ax 4 bx3 cx 2 dx e 0 a 0

Phương pháp giải


b
Đặt ẩn phụ x t để đưa phương trình về dạng t 4 t2 t
4a

Ví dụ : Giải phương trình x 4 8 x3 20 x 2 12 x 9 0 (1)

Lời giải

Đặt x t 2 . Thay vào phương trình (1) ta được


(t 2) 4 8(t 2)3 20(t 2) 2 12(t 2) 9 0
4 2
t 4t 4t 1 0
4 2
t (2t 1) 0
2 2
(t 2t 1)(t 2t 1) 0
t2 2t 1 0 t 1
t2 2t 1 0 t 1 2

Tập nghiệm của phương trình (1) là S 1 2;1 2;3 

2. Một số bài toán tự luyện

Giải các phương trình

1) x 4 14 x3 54 x 2 38 x 11 0

2) x 4 6 x3 9 x 2 2 x 7 0

3) 4 x 4 4 x3 11x 2 7 x 7 0

Nhắc lại:
(a b) 4 C40 a 4b0 C41a3b1 C42 a 2b 2 C43a1b3 C44 a 0b 4
a4 4a3b 6 a 2b 2 4ab3 b4

NGUYỄN VĂN LỰC  0933.168.309 SP Toán K35 - ĐH Cần Thơ


PT – BPT – HPT FB: http://www.facebook.com/VanLuc168

§ 3. CÁCH GIẢI PHƯƠNG TRÌNH DẠNG


(ax b)n pn a' x b' qx r
( x là ẩn số; p, q, r , a, b, a ', b ' là các hằng số; paa ' 0 ; n 2;3
Dạng thường gặp: (ax b)2 p a ' x b ' qx r

Phương pháp giải


Đặt ẩn phụ:
+ Đặt n a ' x b ' ay b nếu pa ' 0
+ Đặt n a ' x b ' (ay b) nếu pa ' 0
Bài toán dẫn đến giải hệ phương trình hai ẩn đối với x và y :
h( x ) Ay Bx C
(*)
h( y ) ( A ' B) x C'
(*) thường là hệ đối xứng loại 2 đối với x và y .
Chú ý: Có thể sử dụng phương pháp nâng lũy thừa để đưa về phương trình bậc bốn.

Ví dụ 1: Giải phương trình 2 x 15 32 x 2 32 x 20 (1)

Lời giải
15
Điều kiện: 2 x 15 0 x
2
Phương trình (1) viết lại thành: 2(4 x 2)2 2 x 15 28
1
Đặt 2 x 15 4y 2 (y ) , ta được hệ phương trình:
2
(4 y 2)2 2 x 15 (2)
(4 x 2)2 2 y 15 (3)
Trừ theo từng vế của (2) và (3) ta được:
(4 y 4x 4)(4 y 4 x) 2( x y) (x y ) 1 8( x y 1) 0
+ Khi x y, thay vào (3) ta được:
1
x
2
(4 x 2) 2 2 x 15 16 x 2 14 x 11 0
11
x
8
1
So với điều kiện của x và y ta chọn x .
2
9
+ Khi 1 8( x y 1) 0 y x , thay vào (3) ta được:
8
9 9 221
(4 x 2)2 2x 15 64 x 2 72 x 35 0 x
4 16

NGUYỄN VĂN LỰC  0933.168.309 SP Toán K35 - ĐH Cần Thơ


PT – BPT – HPT FB: http://www.facebook.com/VanLuc168

9 221
So với điều kiện của x và y ta chọn x .
16
1 9 221
Tập nghiệm của (1) là S ; 
2 16

Ví dụ 2: Giải phương trình 4 x 2 3x 1 5 13 x (1)

Lời giải
1
Điều kiện: 3 x 1 0 x
3
Phương trình (1) viết lại thành: (2 x 3)2 3x 1 x 4
3
Đặt 3x 1 (2 y 3) ( y ) , ta được hệ phương trình:
2
(2 x 3)2 2 y x 1 (2)
(2 y 3)2 3x 1 (3)
Trừ theo từng vế của (2) và (3) ta được:
2(2 x 2y 6)( x y) 2y 2x (x y )(2 x 2y 5) 0
+ Khi x y, thay vào (3) ta được:
15 97
4 x 2 12 x 9 3x 1 4 x 2 15 x 8 0 x
8
15 97
So với điều kiện của x và y ta chọn x .
8
+ Khi 2 x 2 y 5 0 2y 5 2x , thay vào (3) ta được:
11 73
(2 2 x)2 3x 1 4 x 2 11x 3 0 x
8
11 73
So với điều kiện của x và y ta chọn x .
8
11 73 15 97
Tập nghiệm của (1) là S ; 
8 8
Bài tập tương tự

Giải các phương trình


1) x 6 x 2 4 x 2) x 2 4 x 3 x 5
3) 2 x 1 x 2 3x 1 0 4) 4 x 2 14 x 11 4 6 x 10
5) 2 x 2 6 x 1 4 x 5 6) 9 x 2 12 x 2 3x 8
7) 9 x 2 6 x 5 3x 5 8) 4 x 2 4 x 3 2x 5 9) 2 x 2 x 3 2 x

NGUYỄN VĂN LỰC  0933.168.309 SP Toán K35 - ĐH Cần Thơ


PT – BPT – HPT FB: http://www.facebook.com/VanLuc168

§ 4. GIẢI PHƯƠNG TRÌNH CĂN THỨC BẰNG KỸ THUẬT


NHẨM NGHIỆM VÀ PHÂN TÍCH THÀNH NHÂN TỬ

Phương pháp chung


Bước 1: Đặt điều kiện cho hai vế của phương trình có nghĩa và dựa vào điều kiện để
nhẩm nghiệm
Giả sử phương trình có một nghiệm là x x0

Bước 2: Sử dụng các phép biến đổi tương đương để biến đổi phương trình về dạng
x x0
(x x0 ). f ( x) 0
f ( x) 0
Chú ý: Đối với phương trình vô tỷ ta thường sử dụng biến đổi
+ Nhân lượng liên hợp
+ Tách thành các biểu thức liên hợp

Bước 3: Giải phương trình f ( x) 0

Chú ý: Nếu phương trình có hai nghiệm x x1 và x x2 thì ta định hướng biến đổi về
dạng ( x x1 ).( x x 2 ). f ( x) 0

Ví dụ 1: Giải phương trình 3x 1 x 3 x 5 0 (1)

Bài giải
1
♥ Điều kiện: x
3
♥ Phương trình có một nghiệm là x 1 nên ta định hướng biến đổi về dạng
( x 1). f ( x) 0
Ta có: (1) ( 3x 1 2) ( x 3 2) x 1 0 (Tách thành các biểu thức liên
hợp)
3 x 1 x 1
x 1 0 (Nhân liên hợp)
3x 1 2 x 3 2

3 1
( x 1) ( 1) 0 x 1
3x 1 2 x 3 2
0

♥ Vậy phương trình (1) có nghiệm là x 1

Ví dụ 2: Giải phương trình 2 x 2 x 3 x 2 x 21x 17 (1)


NGUYỄN VĂN LỰC  0933.168.309 SP Toán K35 - ĐH Cần Thơ
PT – BPT – HPT FB: http://www.facebook.com/VanLuc168

Bài giải
17
♥ Điều kiện: x
21
♥ Phương trình có hai nghiệm là x 1 và x 2 nên ta định hướng biến đổi về dạng
( x 1).( x 2). f ( x) 0 hay ( x 2 3 x 2). f ( x) 0
Ta có: (1) ( 2 x2 x 3 x 1) (3x 1 21x 17) x 2 3x 2 0
x2 3x 2 9( x 2 3x 2)
x2 3x 2 0
2
2x x 3 x 1 3x 1 21x 17

1 9
( x2 3x 2)( 1) 0
2
2x x 3 x 1 3x 1 21x 17
0

x 1
x2 3x 2 0
x 2
♥ Vậy phương trình (1) có nghiệm là x 1; x 2 

Bài tập tương tự:

1) Giải phương trình 3x 1 6 x 3x 2 14 x 8 0

2) Giải phương trình 4x 1 3 x 2x2 3x 4 0

7
2) Giải phương trình x2 15 x2 8
3x 2

Thực hành giải toán


Bài 1: Giải các phương trình sau
1) x x x2 x 1 1 2) x 2 2 x 3 x 2 x2 1 1

3) x 2 5 x 5 x 2 x 2 3x 2 4) 3x 1 2 x x 4 5

5) 3x 1 x 3 x 5 0 6) 3x 1 6 x 3x 2 14 x 8 0

Bài 2: Giải các phương trình sau


1) 2 x 2 4 x 9 5x 6 7 x 11 0 2) 2 x 2 9 x 3 3x 2 7x 1 3x 2 0

3) 3x 2 x 3 3x 1 5x 4 4) 3 3x 4x 1 9x 4

NGUYỄN VĂN LỰC  0933.168.309 SP Toán K35 - ĐH Cần Thơ


PT – BPT – HPT FB: http://www.facebook.com/VanLuc168

§ 5. ỨNG DỤNG SỰ BIẾN THIÊN CỦA HÀM SỐ


ĐỂ GIẢI MỘT SỐ PHƯƠNG TRÌNH

I. CÁC KIẾN THỨC CƠ BẢN

1. Các định lý
 Cho hàm số y  f (x) có đạo hàm trên khoảng (a; b) .
a) Nếu f '(x)  0 với mọi x  (a; b) thì hàm số f (x) đồng biến trên (a; b) .
b) Nếu f '(x)  0 với mọi x  (a; b) thì hàm số f (x) nghịch biến trên (a; b) .
 Nếu hàm số liên tục trên đoạn  a; b  và có đạo hàm f '(x)  0 trên khoảng
(a; b) thì hàm số f đồng biến trên đoạn  a; b  .
 Nếu hàm số liên tục trên đoạn đọan  a; b  và có đạo hàm f '(x)  0 trên
khoảng (a; b) thì hàm số f nghịch biến trên đoạn  a; b  .

2. Các tính chất


 Tính chất 1: Giả hàm số y  f (x) đồng biến (nghịch biến) trên khoảng
(a; b) và u; v  (a; b) khi đó:
f (u)  f (v)  u  v
 Tính chất 2: Nếu hàm số y  f (x) đồng biến trên (a; b) và y  g(x) làm
hàm hằng hoặc là một hàm số nghịch biến trên (a; b) thì phương trình f (x)  g(x)
có nhiều nhất một nghiệm thuộc khoảng (a; b) .
Dựa vào tính chất trên ta suy ra:
Nếu có x 0  (a; b) sao cho f (x 0 )  g(x 0 ) thì phương trình f (x)  g(x) có nghiệm duy
nhất x0 trên (a; b) .
Chú ý: Khoảng (a; b) nêu trong tính chất có thể thay bởi các miền
(; a);  ; a  ;  a; b  ;  a; b  ;  a; b  ;(b; ); b;   ;( ; )

II. CÁC VÍ DỤ

Ví dụ 1. Giải phương trình 15  x  3  x  6 (1)


Lời giải
 TXĐ: D   ;3
 Xét hàm số f ( x)  15  x  3  x với x   ;3 , khi đó:
1  f  x   f  1 (2)
 Khảo sát tính đơn điệu của hàm số f trên nữa khoảng  ;3
1 1
Ta có: f '( x)    0 x   ;3
2 15  x 2 3  x

NGUYỄN VĂN LỰC  0933.168.309 SP Toán K35 - ĐH Cần Thơ


PT – BPT – HPT FB: http://www.facebook.com/VanLuc168

 Do f liên tục trên nữa khoảng  ;3 và f '  x   0 x   ;3 nên f đồng
biến trên nữa khoảng  ;3
 Suy ra:  2   x  1
 Vậy phương trình (1) có nghiệm duy nhất là x  1 .

Ví dụ 2. Giải phương trình 3x  5  2 x  3  2  12  x (1)

Lời giải
TXĐ: D   ;12
5

3 

Ta có: 1  3x  5  2 x  3  12  x  2 (2)


Xét hàm số f ( x)  3x  5  2 x  3  12  x với x   ;12 , khi đó:
5

3 
1  f  x   f  3 (3)
Khảo sát tính đơn điệu của hàm số f trên đoạn  ;12 
5

3 
3 1 1 5 
Ta có: f '( x)     0 x   ;12 
2 3x  5 2 x  3 2 12  x 3 
Do f liên tục trên đoạn  ;12  và f '  x   0 x   ;12  nên f đồng biến trên đoạn
5 5
3  3 
5 
 3 ;12 
 
 Suy ra:  3  x  3
 Vậy phương trình (1) có nghiệm duy nhất là x  3 .

Ví dụ 3. Giải phương trình 3x 7  5  4 x  3  x3 (1)

Lời giải
TXĐ: D   ; 
5

 4 
Ta có: 1  3x7  x3  5  4 x  3 (2)
Xét hàm số f ( x)  3x7  x3  5  4 x với x   ;  , khi đó:
5

 4 
1  f  x   f 1 (3)
Khảo sát tính đơn điệu của hàm số f trên nữa khoảng  ; 
5

 4
2  5
Ta có: f '( x)  21x6  3x 2  0 x   ; 
5  4x  4

NGUYỄN VĂN LỰC  0933.168.309 SP Toán K35 - ĐH Cần Thơ


PT – BPT – HPT FB: http://www.facebook.com/VanLuc168

Do f liên tục trên đoạn  ;   và f '  x   0 x   ;  nên f đồng biến trên
5 5
 4  4

nữa khoảng  ;  


5
 4
 Suy ra:  3  x  1
 Vậy phương trình (1) có nghiệm duy nhất là x  3 .

Ví dụ 4. Giải phương trình 2 x 2  23  4 x  2  2 x 2  7 (1)

Lời giải
 Ta có: 1  2 x  23  2 x2  7  4 x  2
2
(2)
1
Do VT(2) luôn dương với mọi x nên với x  thì (1) vô nghiệm
2
1
 Điều kiện: x 
2
Xét hàm số f ( x)  4 x  2  2 x2  7  2 x2  23 với x   ;   , khi đó:
1

2 
 2  4 x  2  2 x2  7  2 x2  23  0  f  x   f 1 (3)
Khảo sát tính đơn điệu của hàm số f trên khoảng  ;  
1

2 
 1  1 1 
Ta có: f '( x)  4  2 x  0 x   ;  
 2x  7
2
2 x 2  23  2 

Do đó f đồng biến trên khoảng  ;  


1
2 
 Suy ra:  3  x  1
 Vậy phương trình (1) có nghiệm duy nhất là x  3 .

Ví dụ 5. Giải phương trình 4 x3  x   x  1 2 x  1  0 (1)

Lời giải
TXĐ: D    ;  
1

 2 
1   2x   
3

3
Ta có:  2x  2x 1  2x 1 (2)
 Xét hàm đặc trưng f (t )  t 3  t với t  , khi đó:
 2  f  2x   f  2x 1  (3)
 Khảo sát tính đơn điệu của hàm số f trên
Ta có: f '(t )  3t 2  1  0 t 
Do đó f đồng biến trên

NGUYỄN VĂN LỰC  0933.168.309 SP Toán K35 - ĐH Cần Thơ


PT – BPT – HPT FB: http://www.facebook.com/VanLuc168

x  0
x  0  1 5
 Suy ra:  3  2 x  1  2 x   2  1 5  x  4
4 x  2 x  1  0 x 
 4
1 5
 Vậy phương trình (1) có nghiệm là x  .
4


Ví dụ 6. Giải phương trình  2 x  1 2  4 x 2  4 x  4  3x 2  9 x 2  3  0    (1)

Lời giải
 TXĐ: D 
 1   2 x  1  2   2 x  1  3    3x   2   3x   3 
2 2
Ta có:
   
(2)
 
Xét hàm đặc trưng f (t )  t 2  t 2  3 với t  , khi đó:
 2   f  2 x  1  f  3x  (3)
 Khảo sát tính đơn điệu của hàm số f trên
t2
Ta có: f '(t )  2  t 2  3  0 t 
t2  3
Do đó f đồng biến trên
1
 Suy ra:  3  2 x  1  3x  x  
5
1
 Vậy phương trình (1) có nghiệm là x   .
5
Bài tập tương tự:
Giải phương trình 2 x x 1 x2 2x 3 x 2 x2 4x 6 3 0

NGUYỄN VĂN LỰC  0933.168.309 SP Toán K35 - ĐH Cần Thơ


PT – BPT – HPT FB: http://www.facebook.com/VanLuc168

§ 6. SỬ DỤNG BẤT ĐẲNG THỨC


ĐỂ GIẢI MỘT SỐ PHƯƠNG TRÌNH CĂN THỨC

Ví dụ 1: Giải phương trình 3x2 6 x 7 5x2 10 x 14 4 2x x2 (1)

Bài giải
♥ Đánh giá hai vế của (1) bằng bất đẳng thức ta có
VP(1) 4 2x x2 5 ( x 1)2 5
2 2
VT (1) 3x 6x 7 5x 10 x 14
3( x 1)2 4 5( x 1) 2 9 5
4 2x x2 5
Nên: 1 x 1
3x 2 6x 7 5x2 10 x 14 5
♥ Vậy nghiệm của phương trình là x 2 

Ví dụ 2: Giải phương trình x 2 4 x x2 6 x 11 (1)

Bài giải
♥ Điều kiện: 2 x 4
♥ Đánh giá hai vế của (1) bằng bất đẳng thức ta có
VP(1) x2 6 x 11 ( x 3)2 2 2
VT (1) x 2 4 x 2( x 2 4 x) 2
x2 6 x 11 2
Nên: 1 x 2
x 2 4 x 2
♥ Vậy nghiệm của phương trình là x 2 

Bài tập tương tự


1 x 7
Giải phương trình 2x 3 4x2 4x
2 2

x2 4x 9
Ví dụ 3: Giải phương trình x 2
x 9 3 3x x 2
(1)
3

x2 x 9 0
♥ Điều kiện: 2
(*)
3 3x x 0
♥ Khi đó: 1 3( x 2 x 9) 3(3 3x x2 ) x2 4x 9 (1)

Theo BĐT Cauchy ta có:

NGUYỄN VĂN LỰC  0933.168.309 SP Toán K35 - ĐH Cần Thơ


PT – BPT – HPT FB: http://www.facebook.com/VanLuc168

2 x2 x 9 3 x2 x 6
3( x x 9)
2 2
3 3x x 3 6 3x x 2
2

3(3 3x x2 )
2 2
Suy ra: 3( x2 x 9) 2 2
3(3 3x x ) x 4 x 9
x2 x 9 3 x2 x 12 0
Dấu “=” xảy ra khi: x 3 [thỏa (*)]
3 3x x2 3 x2 3x 0
♥ Vậy nghiệm của phương trình là x 3 

Ví dụ 4: Giải phương trình 16 x4 5 6 3 4 x3 x (1)

Bài giải
♥ Do 16 x 5 0 nên 6 4 x x 0 , suy ra: x 0
4 3 3

♥ Áp dụng BĐT Cauchy cho ba số dương 4 x2 1, 4 x, 2 ta có


VP(1) 6 3 4 x3 x 3 3 4 x(4 x2 1).2 4 x 4 x2 1 2 4 x2 4x 3 (2)
1
Dấu “=” xảy ra khi 4 x 4 x 2 1 2 x
2
Mặt khác: VT (1) 16 x4 5 4 x2 4 x 3 8x4 2 x2 2 x 1 0
(2 x 1)2 (2 x 2 2 x 1) 0 : thỏa x 0
1
Dấu “=” xảy ra khi 2 x 1 0 x
2
1
Nên: (1) x
2
1
♥ Vậy nghiệm của phương trình là x 
2

NGUYỄN VĂN LỰC  0933.168.309 SP Toán K35 - ĐH Cần Thơ


PT – BPT – HPT FB: http://www.facebook.com/VanLuc168

BÀI TẬP TỰ LUYỆN

Câu 1. Giải phương trình: x 2  x 2  3x  5  3x  7.

Ta đặt x 2  3x  5  t (t  0)
Ta được t 2  t  12  0 , giải được t = 3 , t = -4 ( loại)
Với t = 3 , giải tìm được : x  1, x  4.

Câu 2. Giải phương trình: 4(2 10  2x  3 9x  37)  4x 2  15x  33

ĐK: x  5 .
Phương trình  4  4  3 9 x  37   8  4  10  2 x   4 x 2  15 x  81  0
4  27  9 x  8(6  2 x)
   ( x  3)(4 x  27)  0
  4  10  2 x
2
16  4 3 9 x  37  3
9 x  37
- TH1 x  3  0  x  3 (TMPT)
- TH 2. x  3
36 16
phương trình    4 x  27  0
  4  10  2 x
2
16  4 3 9 x  37  3
9 x  37
36 16
   4 x  27  0
  4  10  2 x
2
12  3
9 x  37  2
36 16
Do x  5 nên VT    4.5  27  0 . Đẳng thức xảy ra  x  5
12 4
Vậy phương trình có 2 nghiệm là x  3, x  5.

Câu 3. Giải phương trình: x 4  x 2  1  x  x (1  x 2 )

x  1
ĐK: 
0  x  1
TH1: Với x = 0 không phải nghiệm của phương trình
TH2: Với x  0 .
* Với 0  x  1

NGUYỄN VĂN LỰC  0933.168.309 SP Toán K35 - ĐH Cần Thơ


PT – BPT – HPT FB: http://www.facebook.com/VanLuc168

1 1 1 1
Khi đó phương trình  x 2
 x2  1  x  x x  2
 x2  1  1  x
x x x x

1 1
Đặt t  x  t 4  2  x2  2 . Khi đó ta được phương trình
x x
t  1
t2  3 1  t   4 2  t  1(loai)
t  t  2t  2  0
1 1
* Với x  1 . Ta có  2  x 2  1  1    x
x x

1 1
Đặt t   x  t 4  2  x2  2 . Khi đó ta được t 4  3  t  1  t  1
x x

1  5
Khi đó ta được x 2  x  1  0  x  .
2

1  5 1  5
So sánh đk ta được nghiệm x  .Vậy phương trình đã cho có nghiệm x  .
2 2

 
2
Câu 4. Giải phương trình: x  x 4  x  4 x  4  2x  x  4  50.

Điều kiện x  4

 x x4  2
 x  4  2  2 x  x  4  50
 x  x  4  
2
 2 x  x  4  48  0
Giải phương trình : x  x  4  5  x  5.

Câu 5. Giải phương trình: x x  1   2x  3  2x  2  x  2.


2

TXĐ D = 1;  
Phương trình  ( x 1) x  1  ( x 1)  x  1  (2 x  3)3  (2 x  3)2  2 x  3 (1)
Xét hàm số f (t )  t 3  t 2  t  f' (t )  3t 2  2t  1  f' (t )  0, t  suy ra hàm số f(t) đồng biến
trên .
Phương trình (1) có dạng f ( x  1)  f (2 x  3) . Từ hai điều trên phương trình (1)
 x 1  2x  3
x  3 / 2 x  3 / 2
   2  x= 2
 x  1  4 x  12 x  9 4 x  13 x  10  0
2

Câu 6. Giải phương trình: 4 x  2  22  3x  x 2  8 trên tập số thực.

4 x  2  22  3 x  x 2  8

NGUYỄN VĂN LỰC  0933.168.309 SP Toán K35 - ĐH Cần Thơ


PT – BPT – HPT FB: http://www.facebook.com/VanLuc168

 x4 x  14
pt  4  x  2    22  3x   x2  x  2
 3  3
4
  x2  x  2    x2  x  2
1
9  9  x2  x  2
x4 x  14
x2 22  3x 
3 3
 x  x  2  0 1
2

  x  2
 4 1
   với đk  22

9  9  1  2  x  3
x4 x  14
 x2 22  3 x 
 3 3
Chứng minh được vế trái âm suy ra pt(2) vô nghiệm
Kết luận phương trình có 2 nghiệm x  1, x  2.

Câu 7. Giải phương trình:


2x 5  3x 4  14x 3  2 
  4x 4  14x 3  3x 2  2  1  
x 2  x 2

Điền kiện: x  2 (*).



PT  x3 (2x 2  3x  14)  (4x 4  14x3  3x 2  2) x  2  2 
 x3 (x  2)(2x  7) 
x  2  2  (4x 4  14x 3  3x 2  2)(x  2  4)

 x (x  2)(2x  7) 
3
x  2  2   ( 4x 4
 14x 3  3x 2  2)(x  2)
 x  2  0  x  2 (thoûa maõn (*))
 3
  
 x (2x  7) x  2  2  4x  14x  3x  2
4 3 2
(1)

(1)  x3 (2x  7) x  2  4x4  14x3  4x4  14x3  3x 2  2


 x3 (2x  7) x  2  3x2  2
Nhận thấy x  0 không là nghiệm của phương trình  x  0.
3 2 2 3
Khi đó, PT  (2x  4  3) x  2    2(x  2) x  2  3 x  2  3  (2)
x x 3
x x
Xét hàm số: f(t)  2t 3  3t với t  .
Ta có: f '(t)  6t 2  3  0 t   Hàm số f(t) đồng biến trên .

Do đó (2)  f   1 1
x  2  f    x  2   x x  2 1
x x
x  0
 1  5
 x (thỏa mãn (*))
(x  1)(x  x  1)  0

2
2

1  5
Vậy nghiệm của phương trình đã cho là: x  , x  2.
2

NGUYỄN VĂN LỰC  0933.168.309 SP Toán K35 - ĐH Cần Thơ


PT – BPT – HPT FB: http://www.facebook.com/VanLuc168

Câu 8. Giải phương trình sau trên tập số thực


7x 2  25x  19  x 2  2x  35  7 x  2.

Điều kiện x  7
Phương trình tương đương 7 x 2  25 x  19  7 x  2  x 2  2 x  35 .
Bình phương 2 vế suy ra: 3x 2  11x  22  7 ( x  2)( x  5)( x  7)
3( x 2  5 x  14)  4( x  5)  7 ( x  5)( x 2  5 x  14)
Đặt a  x2  5x  14; b  x  5 .( a ,b  0) Khi đó ta có phương trình
a  b
3a 2  4b2  7ab  3a 2  7ab  4b2  0  
3a  4b
Với a = b suy ra x  3  2 7 (t / m); x  3  2 7 (l ) .
61  11137 61  11137
Với 3a = 4b suy ra x  (t / m); x  (l ) .
18 18
61  111237
Đs: x  3  2 7, x  .
18

Câu 9. Giải phương trình: 2x 2  15x  34  3 3 4x  8 1 .

Ta có 2 x2  15x  34  0  3 3 4 x  8  0  x  2
Cách 1:(Liên hợp thành phần)
12  x  4 
1  2 x 2  15 x  28  3  3 4 x  8  2    x  4  2 x  7  
 4 x  8  2 3 4x  8  4
2
3

x  4

  2x  7   0  *
12
 
 3
 4 x  8
2
 2 3 4x  8  4

+ Nếu x  4  VT *  0  phương trình (*) vô nghiệm
+ Nếu x  4  VT *  0  phương trình (*) vô nghiệm
+ Nếu x  4 . Thỏa mãn phương trình (*)
Vậy phương trình đã cho có nghiệm duy nhất x  4 .
Cách 2:(Liên hợp hoàn toàn)
1  2 x 2  16 x  32  3 3 4 x  8   x  2 
 x  4   x  14 
2

 2  x  4  0
2

9 3  4 x  8  3 3 4 x  8  x  2    x  2 
2 2

x  4


2
 x  14   0 
*
 9 3  4 x  8  3 3 4 x  8  x  2    x  2 
2 2

NGUYỄN VĂN LỰC  0933.168.309 SP Toán K35 - ĐH Cần Thơ
PT – BPT – HPT FB: http://www.facebook.com/VanLuc168

Vậy phương trình đã cho có nghiệm duy nhất x  4 .


Cách 3:(Phương pháp đánh giá)
Ta có: 3 3  4 x  8 .8.8  4 x  8  3  4 x  8   x  2 ( Theo bất đẳng thức Cô si)
Do đó 2 x2  15x  34  x  2  2  x  4   0  x  4 . Thử lại thấy thỏa mãn.
2

Vậy phương trình đã cho có nghiệm duy nhất x  4.

Câu 10. Giải phương trình: 2 x  2   3



x  5  2 2x  5  3x  1 (x  ) .

5
Điều kiện xác định: x  .
2
Phương trình đã cho tương đương:
3x  1 3x  1
3
x  5  2 2x  5   3 x  5  2 2x  5 
0
2x  4 2x  4
3x  1
với x thuộc  ;  
5
Đặt f ( x)  3 x  5  2 2 x  5 
2x  4 2 
1 2 10 5
 f '( x)     0 với x 
3 3  x  5 2x  5  2x  4
2
2
2

5 
 hàm số f ( x) đồng biến trên  ;   .
2 
 phương trình f ( x)  0 có tối đa một nghiệm (1)
Ta có f (3)  0 (2)
Từ (1) và (2) suy ra phương trình đã cho có nghiệm duy nhất x  3.

Câu 11. Giải phương trình: 4 x 2  x  1  1  5x  4x 2  2x 3  x 4

3
Đặt t  x 2  x  1, t  . Khi đó phương trình trở thành:
2
4t  t 4  7t 2  5  t 4  6t 2  9   t 2  4t  4   0
t 2  t  1  0
  t  3   t  2  0   t  t  1 t  t  5  0 (*)   2
2 2 2 2 2

t  t  5  0
3 1 5
 Với t  thì t 2  t  1  0 có một nghiệm là t 
2 2
3 1  21
 Với t  thì t 2  t  5  0 có một nghiệm là t 
2 2

NGUYỄN VĂN LỰC  0933.168.309 SP Toán K35 - ĐH Cần Thơ


PT – BPT – HPT FB: http://www.facebook.com/VanLuc168
2
1 5 1 5 
 Khi t  thì x 2  x  1     2x  2x 1  5  0
2

2  2 
1  3  2 5 1  3  2 5
x hoặc x  .
2 2
2
1  21  1  21 
Khi t  thì x 2  x  1     2 x  2 x  9  21  0
2

2  2 
1  19  2 21 1  19  2 21
x hoặc x  .
2 2
1  19  2 21 1  19  2 21
Vậy phương trình đã cho có nghiệm x  , x .
2 2

Câu 12. Giải phương trình:


2x  8  2 x 2  4x  12  3  x 2  x 6 .
x  2  0
Điều kiện   x6
x  6  0
Đặt t = x  2  x  6 (Đk: t > 0)
 t 2  2 x  4  2 x 2  4 x  12
 t 2  4  2 x  8  2 x 2  4 x  12
 t  1  l 
Phương trình đã cho trở thành t 2
 3t  4  0  
t  4  n 
Với t  4  x  2  x  6  4
 2 x  4  2 x2  4 x  12  16  x2  4 x  12  10  x
10  x  0
 2
 x  4 x  12  100  20 x  x
2

 x  10
  x  7 (Thoả đk x  6 )
16 x  112  0
Vậy phương trình đã cho có nghiệm x  7.

Câu 13. Giải phương trình: 15x 2  12x  12  10  2x  1 x 2  3

15x 2  12 x  12  10  2 x  1 x 2  3 1
1
Điều kiện: x  
2
Với điều kiện trên phương trình 1 tương đương:
NGUYỄN VĂN LỰC  0933.168.309 SP Toán K35 - ĐH Cần Thơ
PT – BPT – HPT FB: http://www.facebook.com/VanLuc168

3  2 x  1  3  x 2  3  10  2 x  1 x 2  3
2

Đặt a  2 x  1, b  x2  3 b  3  phương trình trở thành: 3a 2


 3b 2  10ab
a
2
b  3 3b  a
a
b
a
 3    10    3  0
b

do b  3  
a  1


b  3a
 b 3
 1
x  
Với 3b  a , a  3b ta được: 3 x  3  2 x  1  
2
2
5 x 2  4 x  26  0
 VN 
 1
x   114  18
Với b  3a , a  3b ta được: x  3  6 x  3  
2
2 x
35 x 2  36 x  6  0 35

114  18
So điều kiện ta được x  .
35

Câu 14. Giải phương trình: x  2x  1  2(3  x ) 2

1
x  2 x  1  2(3  x) 2 . Ñk: x  Vôùi ñk treân, pt töông ñöông
2
2( x  5)
2 x  1  3  2 x 2  13 x  15   ( x  5)(2 x  3)
2x  1  3
 x5

(2 x  3)( 2 x  1  3)  2
Giaûi (2 x  3)( 2 x  1  3)  2 (1)
Ñaët t= 2 x  1, t  0  t 2  2 x  1
(1) trôû thaønh: t 3  3t 2  2t  8  0
 1  17
t  (nhaän)
t  2 (loaïi) 2
 (t  2)(t  t  4)  0   2
2
. Giaûi t  t  4  0 
2

t  t  4  0  1  17
t  (loaïi)
 2
1  17 1  17 11  17
Vôùi t   2x  1  x (nhaän)
2 2 4
11  17
Vaäy pt coù nghieäm laø x  5, x  .
4

NGUYỄN VĂN LỰC  0933.168.309 SP Toán K35 - ĐH Cần Thơ


PT – BPT – HPT FB: http://www.facebook.com/VanLuc168

III. BẤT PHƯƠNG TRÌNH


Chuyên đề: PT – BPT - HPT

1. BẤT PHƯƠNG TRÌNH CHỨA MỘT CĂN BẬC HAI

Ví dụ 1. (Đề thi đại học  Khối D năm 2002): Giải bất phương trình:
x 2
 3x  2x 2  3x  2  0, x  R.

Chúng ta cần sử dụng phép biến đổi tương đương sau:


g(x)  0

f (x). g(x)  0 , với f(x) và g(x) có nghĩa   g(x)  0 . .
 f (x)  0

 Giải
Bất phương trình tương đương với:
 1
x  2  x   2
 2x 2  3x  2  0  x  3
   x  2
 2x  3x  2  0     x  1/ 2
2
  x  2 .
  x 2  3x  0  
  x  1/ 2
   x  3

   x  0

Vậy, tập nghiệm của bất phương trình là  ;    2  3;    .
1
 2

Ví dụ 2. Giải bất phương trình:


(x  1) 2x  1  3(x  1), x  R.

 Giải
Điều kiện:
1
2x1  0  x  . (*)
2
1 2
Đặt t = 2x  1 , t0x= (t + 1).
2
Khi đó, bất phương trình có dạng:

NGUYỄN VĂN LỰC  0933.168.309 SP Toán K35 - ĐH Cần Thơ


PT – BPT – HPT FB: http://www.facebook.com/VanLuc168

[ 1 (t2 + 1)1]t  3[ 1 (t2 + 1)1]  t33t2t + 3  0


2 2
 (t + 1)(t1)(t3)  0  1  t  3  1  2x  1  3  1  x  5.
Vậy, tập nghiệm của bất phương trình là [1; 5].
 Chú ý: Ta không thể bình phương hai vế của bất phương trình ban đầu vì chưa khẳng
định được dấu của hai vế.
Hoàn toàn có thể sử dụng phép biến đổi tương đương để thực hiện thí dụ trên,
cụ thể:
(x1)( 2x  1 3)  0
  x  1  0   x  1 x  1
  
  2x  1  3  0  2x  1  3  0  2x  1  9
     x  1
  x  1  0   x  1 

  2x  1  3  0   2x  1  3
    2x  1  9

 1  x  5.
Vậy, tập nghiệm của bất phương trình là [1; 5].

Ví dụ 3. Giải bất phương trình:


(x 2  1)  (x  1)  3x x  1  0, x  R.

Hướng dẫn: Điều kiện x  1.


Biến đổi tương đương bất phương trình:
x2(x + 1) + 3x x  1 + 2 > 0.
Đặt t = x x  1 .
Từ đó, ta nhận được nghiệm x  1.

DẠNG CƠ BẢN 1

Với bất phương trình f(x)  g(x) ta có phép biến đổi tương đương:
f(x)  0

g(x)  0 .
f(x)  g 2 (x)
 (*)
Các em học sinh cần biết đánh giá tính giải được của bất phương trình (*).

Ví dụ 1. Giải bất phương trình:


x  1  2(x 2  1), x  R.

Sử dụng lược đồ trong “DẠNG CƠ BẢN 1” bới trong trường hợp này (*) là một bất
phương trình bậc hai  Giải được.
NGUYỄN VĂN LỰC  0933.168.309 SP Toán K35 - ĐH Cần Thơ
PT – BPT – HPT FB: http://www.facebook.com/VanLuc168

 Giải
Bất phương trình tương đương với:
2(x 2  1)  0  x 1  x 1
    x  1
x  1  0   x  1   x  1  1  x  3.
2(x 2  1)  (x  1) 2  x 2  2x  3  0 1  x  3 
  
Vậy, tập nghiệm của bất phương trình là [1; 3]  {1}.

Ví dụ 2. Giải các bất phương trình:


x 2  3x  10  x  2, x  R.

Bất phương trình tương đương với hệ:


 x 2  3x  10  0 (x  2)(x  5)  0 x  5
  
x  2  0  x  2  0  x  2
 x 2  3x  10  (x  2) 2  x  14  0  x  14
  
 5  x < 14.
Vậy, tập nghiệm của bất phương trình là [5; 14).

Ví dụ 3. Giải các bất phương trình:


x 2  2x  15  x  3, x  R.

 Giải
Bất phương trình tương đương với hệ:
 x 2  2x  15  0  x 2  2x  15  0  x  5 hoac x  3
  
x  3  0  x  3  0  x  3
 x 2  2x  15  (x  3) 2 4x  24  0 x  6
  
 5  x  6.
Vậy, tập nghiệm của bất phương trình là [5; 6].

Ví dụ 4. Giải bất phương trình:


x2  3  3x2  1, x  R.

Sử dụng lược đồ trong “DẠNG CƠ BẢN 1” bới trong trường hợp này (*) là một bất
phương trình trùng phương  Giải được.
Ngoài ra, bất phương trình còn được giải theo các cách khác:
 Nhẩm nghiệm x0 rồi chuyển bất phương trình về dạng tích (x  x0)h(x) bằng
phép nhân liên hợp. Cụ thể:
o Nhận xét rằng x0 = 1 là nghiệm của bất phương trình.
NGUYỄN VĂN LỰC  0933.168.309 SP Toán K35 - ĐH Cần Thơ
PT – BPT – HPT FB: http://www.facebook.com/VanLuc168

o Biến đổi bất phương trình về dạng:


x2  3  4
x 2  3  2  3x 2  3  
 3 x2  1 
x 3 2 2

 1 
 (x2  1)   3   0.
 x 3 2 
2

 Sử dụng phương pháp đạt ẩn phụ, với t  x2  3, t  3.


 Giải
Ta có thể trình bày theo các cách sau:
1
Cách 1: Với điều kiện 3x2  1  0 tức x  , ta biến đổi phương trình về dạng:
3
    
2
x 2  3  3x 2  1  9x4  7x2  2  0  x2  1 9x2  2  0
 x  1  0  x  1.
2

Vậy, tập nghiệm của bất phương trình là (; 1]  [1; +).
Cách 2: Biến đổi phương trình về dạng:
x2  3  4
x 2  3  2  3x 2  3  
 3 x2  1 
x 3 2 2

 1 
 (x2  1)   3   0. (*)
 x 3 2 
2

Nhận xét rằng:


1 1 1
  3 0
x 3 2
2 2 x 3 2
2

nên (*) được biến đổi về dạng:


x 2  1  0  x  1.
Vậy, tập nghiệm của bất phương trình là (; 1]  [1; +).
Cách 3: Đặt t  x2  3, t  3. Suy ra x2 = t2  3.
Bất phương trình có dạng:
t  3(t2  3)  1  3t2  t  10  0  (3t + 5)(t  2)  0
t 3
 t  2  0  x2  3  2  x2 + 3  4  x2  1  x  1.
Vậy, tập nghiệm của bất phương trình là (; 1]  [1; +).

Ví dụ 5. Giải bất phương trình:


x2  8  4x2  1, x  R.

 Giải
Ta có thể trình bày theo các cách sau:
1
Cách 1: Với điều kiện 4x2  1  0 tức x  , ta biến đổi bất phương trình về dạng:
2
    
2
x 2  8  4x 2  1  16x4  9x2  7  0  x2  1 16x2  7  0
 x  1  0  x  1.
2

NGUYỄN VĂN LỰC  0933.168.309 SP Toán K35 - ĐH Cần Thơ


PT – BPT – HPT FB: http://www.facebook.com/VanLuc168

Vậy, tập nghiệm của bất phương trình là (; 1]  [1; +).
Cách 2: Biến đổi bất phương trình về dạng:
x2  8  9
x 2  8  3  4x 2  4  
 4 x2  1 
x 8 3 2

 1 
 (x2  1)   4   0. (*)
 x 8 3 
2

Nhận xét rằng:


1 1 1
  40
x 8 3
2 3 x 8 3
2

nên (*) được biến đổi về dạng:


x 2  1  0  x  1.
Vậy, tập nghiệm của bất phương trình là (; 1]  [1; +).
Cách 3: Đặt t  x2  8, t  2 2. Suy ra x2 = t2  8.
Bất phương trình có dạng:
t  4(t2  8)  1  4t2  t  33  0  (4t + 11)(t  3)  0
t 2 2
 t  3  0  x2  8  3  x2 + 8  9  x2  1  x  1.
Vậy, tập nghiệm của bất phương trình là (; 1]  [1; +).

Ví dụ 6. Giải bất phương trình:


x  1  5  x, x  R.

 Giải
Ta có thể trình bày theo các cách sau:
Cách 1: Biến đổi bất phương trình về dạng:
x  1  0 x  1  1  x  5
  
5  x  0  x  5  x  3  1  x < 3.
 x 2  11x  24  0 x  8
 x  1   5  x  
2

Vậy, tập nghiệm của bất phương trình là [1; 3).

Cách 2: Với điều kiện x + 1  0 tức x  1, ta biến đổi bất phương trình về dạng:
x 1 4 x 3
x 1  2  3  x   3x   x3 0
x 1  2 x 1  2
 1 
  x  3   1  0  x  3 < 0  x < 3.
 x 1  2 
1

Vậy, tập nghiệm của bất phương trình là [1; 3).

Cách 3: Điều kiện x + 1  0 tức x  1.


Đặt t  x  1, (t  0) . Suy ra x = t2  1.
NGUYỄN VĂN LỰC  0933.168.309 SP Toán K35 - ĐH Cần Thơ
PT – BPT – HPT FB: http://www.facebook.com/VanLuc168

Bất phương trình có dạng:


t < 5  (t2  1)  t2 + t  6 < 0  3 < t < 2  x  1  2
 x + 1 < 4  x < 3.
Vậy, tập nghiệm của bất phương trình là [1; 3).

Cách 4: Điều kiện x + 1  0 tức x  1.


Nhận xét rằng:
 VT là hàm đồng biến.
 VP là hàm nghịch biến.
Hai đồ thị cắt nhau tại điểm có hoành độ x = 3.
Vậy, tập nghiệm của bất phương trình là [1; 3).

Ví dụ 7. Giải bất phương trình:


1  x3  x  5, x  R.

Sử dụng lược đồ trong “DẠNG CƠ BẢN 1” bới trong trường hợp này (*) là một bất
phương trình bậc ba  Giải được.
Ngoài ra, bất phương trình còn được giải theo cách:
 Nhẩm nghiệm x0 rồi chuyển bất phương trình về dạng tích (x  x0)h(x) bằng
phép nhân liên hợp. Cụ thể:
o Nhận xét rằng x0 = 2 thoả mãn VT = VP.
o Biến đổi bất phương trình về dạng:
1  x3  9
1  x3  3  x  2  x2
1  x3  3
x3  8  x2  x  1 
 x2  0  (x  2)  1  0
1  x3  3  1  x3  3 

 Sử dụng phương pháp hàm số, với điều kiện x  1 nhận xét:
o VP là hàm đồng biến.
o VT là hàm nghịch biến.
Hai đồ thị cắt nhau tại điểm có hoành độ x = 2.
Vậy, tập nghiệm của bất phương trình là [2; 1].
 Giải
Ta có thể trình bày theo các cách sau:
Cách 1: Bất bất phương trình tương đương với:
1  x3  0 x3  1 x  1
  
x  5  0  x  5  0  x  5
1  x3  (x  5)2 x3  x 2  10x  24  0 (x  2)(x 2  x  12)  0
  

NGUYỄN VĂN LỰC  0933.168.309 SP Toán K35 - ĐH Cần Thơ


PT – BPT – HPT FB: http://www.facebook.com/VanLuc168

x  1 x  1
 
  x   5   x  5  2  x  1.
x  2  0 x  2
 
Vậy, tập nghiệm của bất phương trình là [2; 1].

Cách 2: Với điều kiện 1  x3  0 tức x  1, ta biến đổi bất phương trình về dạng:
1  x3  9 x3  8
1  x3  3  x  2  x2  x2 0
1  x3  3 1  x3  3
 x2  x  1 
 (x  2)  1   0 x + 2  0  x  2.
 1  x3  3 
Vậy, tập nghiệm của bất phương trình là [2; 1].

Cách 3: Với điều kiện x  1 nhận xét:


 VP là hàm đồng biến.
 VT là hàm nghịch biến.
Hai đồ thị cắt nhau tại điểm có hoành độ x = 2.
Vậy, tập nghiệm của bất phương trình là [2; 1].
 Nhận xét: Như vậy, để giải một bất phương trình chứa căn ta có thể lựa chọn một
trong các cách:
Cách 1: Biến đổi tương đương. Lưu ý cách nhẩm nghiệm x0 rồi chuyển
bất phương trình về dạng tích (x  x0)h(x) bằng phép nhân liên hợp,
bởi trong nhiều trường hợp sẽ nhận được cách giải hay.
Cách 2: Đặt ẩn phụ. Một hoặc nhiều ẩn phụ.
Cách 3: Sử dụng phương pháp hàm số. Sử dụng đạo hàm.
Cách 4: Đánhgiá.

Ví dụ 8. Giải các bất phương trình:


x3  3  3x  1, x  R.

 Giải
Bất bất phương trình tương đương với:
x3  3  0 x3  3  0 x3  3  0
  
3x  1  0  3x  1  0  3x  1  0
x 3  3  (3x  1)2 x3  9x 2  6x  2  0 (x  1)(x 2  8x  2)  0
  
x   33


 1
 x   1  x  4  3 2.
 3
x  4  3 2 hoac 1  x  4  3 2

Vậy, tập nghiệm của bất phương trình là 1; 4  3 2  .
 
NGUYỄN VĂN LỰC  0933.168.309 SP Toán K35 - ĐH Cần Thơ
PT – BPT – HPT FB: http://www.facebook.com/VanLuc168

Ví dụ 9. Giải các bất phương trình:


x  2  3x  4, x  R.

 Giải
Ta có thể trình bày theo các cách sau:
Cách 1: Biến đổi bất phương trình về dạng:
 4
 x
x  2  0 x  2 3
  
3x  4  0  3x  4  x  2  x > 2.
x  2  (3x  4)2 9x 2  25x  14  0 
  x 
7
 9
Vậy, tập nghiệm của bất phương trình là (2; +).

Cách 2: Với điều kiện x + 2  0 tức x  2 , ta biến đổi bất phương trình về dạng:
x24
x  2  2  3x  6   3x  6
x2 2
 1 
 (x  2)   3   0. (*)
 x2 2 
Nhận xét rằng:
1 1 1
  3 0
x2 2 2 x2 2
nên (*) được biến đổi về dạng:
x  2 > 0  x > 2.
Vậy, tập nghiệm của bất phương trình là (2; +).

Cách 3: Điều kiện x + 2  0 tức x  2.


Đặt t  x  2, (t  0) . Suy ra x = t2  2.
Phương trình có dạng:
t < 3(t2  2)  4  3t2  t  10 > 0
t  2
  x  2  2  x > 2.
 t   5 (loai)
 3
Vậy, tập nghiệm của bất phương trình là (2; +).

Ví dụ 10. Với a > 0, giải bất phương trình:


x  a 2  x2  a, x  R.

Sử dụng lược đồ trong “DẠNG CƠ BẢN 1” bới trong trường hợp này (*) là một bất
phương trình bậc hai  Giải được.
NGUYỄN VĂN LỰC  0933.168.309 SP Toán K35 - ĐH Cần Thơ
PT – BPT – HPT FB: http://www.facebook.com/VanLuc168

Ngoài ra, bất phương trình còn được giải theo cách lượng giác hoá với:
x = a.cost, t  [0; ].
 Giải
Ta có thể trình bày theo các cách sau:
Cách 1: Biến đổi bất phương trình về dạng:
a  x  0  a  x  a
 2 xa
a x
2 2
 ax  a  x  0
2
 
x  a  
 2 x  0  a  x  0
a  x 2  (a  x)2 
Vậy, nghiệm của bất phương trình là a  x  0 hoặc x = a
Cách 2: Điều kiện a  x  a.
Đặt x = a.cost, với t  [0, ]  a2  x2 = a.sint.
Khi đó, bất phương trình có dạng:
a.cost + a.sint  a  cost + sint  1  cos(t  )  1
4 2

2  t    1  cos t  0  a  a. cos t  0  a  x  0
 
      .
cos t  1 a. cos t  a x  a
t  0
Vậy, nghiệm của bất phương trình là a  x  0 hoặc x = a.

Ví dụ 11. Giải bất phương trình:


2a 2
x2  a2  x  , x  R.
x2  a2

 Giải
Ta có thể trình bày theo các cách sau:
Cách 1: Đặt x = atgt, với t  (  ,  ) suy ra:
2 2
|a|
x2  a 2 = .
cos t
Khi đó, bất phương trình có dạng:
2a 2 . cos t
|a|
 atgt +  1  sint + 2cos2t  2sin2tsint1  0
cos t |a|
  1  sint  1  tgt  1
 x  | a | .
2 3 3

Vậy, nghiệm của bất phương trình là x  |a|


.
3

Cách 2: Biến đổi bất phương trình về dạng:


x2 + a2  x x2  a2 + 2a2  x2a2  x x2  a2 . (2)
Xét hai trường hợp:
 Nếu x  0, thì (2) được viết lại dưới dạng:

NGUYỄN VĂN LỰC  0933.168.309 SP Toán K35 - ĐH Cần Thơ


PT – BPT – HPT FB: http://www.facebook.com/VanLuc168

x 2  a 2  0 | x || a |
 
x2a2  x 2 (x 2  a 2 )  x 2  a 2  0  | x || a |
  |a|
( x 2  a 2 )2  x 2 ( x 2  a 2 ) | x |
 3
x 0
x  0.

 Nếu x < 0, thì (2) được viết lại dưới dạng:


x 2  a 2  0  | a | x | a |

  | a |  x < 0
x0
 2  |a| |a| 
(x  a 2 )2  x 2 (x 2  a 2 )  3  x  3 3

Vậy, nghiệm của bất phương trình là x  | a | .


3

NGUYỄN VĂN LỰC  0933.168.309 SP Toán K35 - ĐH Cần Thơ


PT – BPT – HPT FB: http://www.facebook.com/VanLuc168

DẠNG CƠ BẢN 2

Với bất phương trình f(x)  g(x) ta có phép biến đổi tương đương:
f(x)  0 g(x)  0
(I) :  hoặc (II) : 
g(x)  0 f(x)  g (x). (*)
2

Các em học sinh cần biết đánh giá tính giải được của bất phương trình (*).

Ví dụ 1. Giải bất phương trình:


2x  1  1  x, x  R.

Sử dụng lược đồ trong “DẠNG CƠ BẢN 2” bới trong trường hợp này (*) là một bất
phương trình bậc hai  Giải được.
Ngoài ra, phương trình còn được giải theo các cách khác:
 Nhẩm nghiệm x0 rồi chuyển bất phương trình về dạng tích (x  x0)h(x) bằng
phép nhân liên hợp. Cụ thể:
o Nhận xét rằng x0 = 0 thoả mãn VT = VP.
o Biến đổi bất phương trình về dạng:
2x  1  1  
 
2x  1  1  x  0 
2x  1  1
 x  0  x
2
 1  0
 2x  1  1 
 Sử dụng phương pháp hàm số, với nhận xét:
o VT là hàm đồng biến.
o VP là hàm nghịch biến.
Hai đồ thị cắt nhau tại điểm có hoành độ x = 0.
Vậy, tập nghiệm của bất phương trình là (0; +).
 Giải
Ta có thể trình bày theo các cách sau:
Cách 1: Bất phương trình tương đương với:
2x  1  0 1  x  0
(I) :  hoặc (II) :  2.
1  x  0 2x  1  1  x 
Ta lần lượt:
 Giải (I) ta được:
 1
x  
 2  x > 1. (1)
 x  1
 Giải (II) ta được:
x  1 x  1
 2   0  x  1. (2)
 x  4x  0  0  x  4
Từ (1) và (2) suy ra tập nghiệm của bất phương trình là (0; +).

NGUYỄN VĂN LỰC  0933.168.309 SP Toán K35 - ĐH Cần Thơ


PT – BPT – HPT FB: http://www.facebook.com/VanLuc168

1
Cách 2: Với điều kiện 2x + 1  0 tức x, ta biến đổi bất phương trình về dạng:
2
 
 
2x  1  1  x  0 
2x  1  1
2x  1  1
 x  0  x
2
 1  0
 2x  1  1 
 x  0.
Vậy, tập nghiệm của bất phương trình là (0; +).

1
Cách 3: Điều kiện 2x + 1  0 tức x .
2
t 1
2
Đặt t  2x  1, (t  0) . Suy ra x .
2
Bất phương trình có dạng:
t2 1 t  1
t  1  t2 + 2t  3 > 0 
2  t  3 (loai)
 2x  1  1  2x + 1 > 1  x > 0.
Vậy, tập nghiệm của bất phương trình là (0; +).

Cách 4: Nhận xét rằng:


 VT là hàm đồng biến.
 VP là hàm nghịch biến.
Hai đồ thị cắt nhau tại điểm có hoành độ x = 0.
Vậy, tập nghiệm của bất phương trình là (0; +).

Ví dụ 2. Giải bất phương trình:


x  2  4  x, x  R.

 Giải
Ta có thể trình bày theo các cách sau:
Cách 1: Biến đổi bất phương trình về dạng:
x  2  0 4  x  0
(I) :  hoặc (II) :  2.
 4  x  0  x  2   4  x 
Ta lần lượt:
 Giải (I) ta được:
 x  2
  x > 4. (1)
x  4
 Giải (I) ta được:
x  4 x  4
 2   2 < x  4. (2)
x  9x  14  0 2  x  7
Từ (1) và (2) suy ra tập nghiệm của bất phương trình là (2; +).

NGUYỄN VĂN LỰC  0933.168.309 SP Toán K35 - ĐH Cần Thơ


PT – BPT – HPT FB: http://www.facebook.com/VanLuc168

Cách 2: Với điều kiện x + 2  0 tức x  2 , ta biến đổi bất phương trình về dạng:
x24
x2 2 2x  2x
x2 2
 1 
 (x  2)   1  0  x  2 > 0  x > 2.
 x2 2 
Vậy, tập nghiệm của bất phương trình là (2; +).

Cách 3: Điều kiện x + 2  0 tức x  2.


Đặt t  x  2, (t  0) . Suy ra x = t2  2.
Bất phương trình có dạng:
t  2
t > 4  (t2  2)  t2 + t  6 > 0   x2  2
 t  3 (loai)
 x > 2.
Vậy, tập nghiệm của bất phương trình là (2; +).

Cách 4: Nhận xét rằng:


 VT là hàm đồng biến.
 VP là hàm nghịch biến.
Hai đồ thị cắt nhau tại điểm có hoành độ x = 2.
Vậy, tập nghiệm của bất phương trình là (2; +).

Ví dụ 3. Giải bất phương trình:


1 1
 x  x  , x  R.
4 2

Sử dụng lược đồ trong “DẠNG CƠ BẢN 2” bới trong trường hợp này (*) là một bất
phương trình bậc hai có chứa dấu giá trị tuyệt đối  Giải được bằng phương pháp chia
khoảng.
 Giải
Bất phương trình tương đương với:
 1
x  2  0
1 
(I) : x  0 hoặc (II) :  2 .
2 1
 x  x  1 
 4   (*)
 2
1
Giải (I) ta được x   . (1)
2
Giải (II): Ta có biến đổi cho (*):
1 1
 Với x 0 tức x  thì:
4 4
2
1  1
 x   x    x2 + 2x  0  2  x  0, thoả mãn.
4  2
NGUYỄN VĂN LỰC  0933.168.309 SP Toán K35 - ĐH Cần Thơ
PT – BPT – HPT FB: http://www.facebook.com/VanLuc168

1 1
 Với x 0 tức x  thì:
4 4
2
1  1 1
x    x    x2   0 , vô nghiệm
4  2 2
Suy ra, nghiệm của (*) là 2  x  0.
Và hệ (II) có dạng:
 1
x   1
 2    x  0. (2)
 2  x  0 2

Từ (1) và (2) suy ra tập nghiệm của bất phương trình là (; 0].

Ví dụ 4. Giải bất phương trình:


1
x  1  x  , x  R.
4

 Giải
Bất phương trình tương đương với:
 1
x  4  0
1 
(I) : x  0 hoặc (II) :  2 .
4  x  1   x  1  (*)
  
 4
1
Giải (I) ta được x  . (1)
4
Giải (II): Ta có biến đổi cho (*):
 Với x  1  0 tức x  1 thì:
2
 1 3 17
x 1   x    x2  x   0 , vô nghiệm.
 4 2 16
 Với x  1 < 0 tức x < 1 thì:
2
 1 1 15 3 1
1 x   x   x   0    x  , thoả mãn.
 x2 
 4 2 16 4 4
3 1
Suy ra, nghiệm của (*) là   x  . Và dễ thấy hệ (II) vô nghiệm.
4 4
tập nghiệm của bất phương trình là  ;  .
1
Vậy,
 4

Ví dụ 5. Giải bất phương trình:


x 2  3x  6  3x 2  9x  8, x  R.

Nếu sử dụng lược đồ trong “DẠNG CƠ BẢN 2” thì (*) là một bất phương trình bậc bốn
 Để giải được bất phương trình này cần có kỹ năng phân tích đa thức thành nhân tử.
Ngoài ra, phương trình còn được giải theo các cách khác:
NGUYỄN VĂN LỰC  0933.168.309 SP Toán K35 - ĐH Cần Thơ
PT – BPT – HPT FB: http://www.facebook.com/VanLuc168

 Sử dụng phương pháp đạt ẩn phụ, với t  x2  3x  6, t  0.


 Nhẩm nghiệm x0 rồi chuyển phương trình về dạng tích (x  x0)h(x) bằng
phép nhân liên hợp. Cụ thể:
o Nhận xét rằng x0 = 1 là nghiệm của phương trình.
o Biến đổi phương trình về dạng:
x 2  3x  6  4
x 2  3x  6  2  3x 2  9x  6   3(x 2  3x  2)
x  3x  6  2
2

 1 
 (x 2  3x  2)   3  0
 x  3x  6  2 
2

 Giải
Ta có thể trình bày theo các cách sau:
Cách 1: Biến đổi phương trình về dạng:
 
x2  3x  6  3 x2  3x  6  10.

Đặt t  x 2  3x  6, (t  0) ta được:
5
t  3t 2  10  3t 2  t  10  0   t 2  t2
3
 1  x  2.
 x 2  3x  6  2  x2  3x  2  0
Vậy, tập nghiệm của bất phương trình là [1; 2].

Cách 2: Ta có biến đổi:


x 2  3x  6  4
x 2  3x  6  2  3x 2  9x  6   3(x 2  3x  2)
x  3x  6  2
2

 1 
 (x2  3x  2)   3   0. (*)
 x  3x  6  2 
2

Nhận xét rằng:


1 1 1
  3 0
x  3x  6  2
2 2 x  3x  6  2
2

nên (*) được biến đổi về dạng:


 x2  3x  2  0  1  x  2.
Vậy, tập nghiệm của bất phương trình là [1; 2].

Ví dụ 6. Giải bất phương trình:


x 2  3x  5  2x 2  6x  5, x  R.

 Giải
Ta có thể trình bày theo các cách sau:
Cách 1: Biến đổi bất phương trình về dạng:
 
x2  3x  5  2 x2  3x  5  15.

NGUYỄN VĂN LỰC  0933.168.309 SP Toán K35 - ĐH Cần Thơ


PT – BPT – HPT FB: http://www.facebook.com/VanLuc168

Đặt t  x 2  3x  5, (t  0) ta được:
5
t  2t 2  15  2t 2  t  15  0   t3
2
 x 2  3x  5  3  x2  3x  4  0   4  x  1.
Vậy, tập nghiệm của bất phương trình là (4; 1).

Cách 2: Ta có biến đổi:


x 2  3x  5  9
x 2  3x  5  3  2x 2  6x  8   2(x 2  3x  4)
x  3x  5  3 2

 1 
 (x2  3x  4)   2   0. (*)
 x  3x  5  3 
2

Nhận xét rằng:


1 1 1
  2 0
x  3x  5  3
2 3 x  3x  5  3
2

nên (*) được biến đổi về dạng:


x 2  3x  4  0   4  x  1.
Vậy, tập nghiệm của bất phương trình là (4; 1).

Ví dụ 7. Giải bất phương trình:


2x
 2x  2, x  R.
2x  1  1

Thiết lập điều kiện có nghĩa cho bất phương trình, rồi sử dụng phép nhân liên hợp để biến
đổi bất phương trình về dạng cơ bản.
 Giải
Điều kiện:
2x  1  0 1
  0  2x + 1  1    x  0. (*)
 2x  1  1  0 2
Trục căn thức, ta biến đổi bất phương trình về dạng:
2x  2x  1  1   2x  2  2x  1  1  2x  2
 2x  1  1  2x  1  1 
(*)
 2x  1  2x  1  2x  1  (2x  1) 2
1
 4x2 + 2x < 0    x  0.
2
 1 
Vậy, tập nghiệm của bất phương trình là T =  2 ; 0 .
 

NGUYỄN VĂN LỰC  0933.168.309 SP Toán K35 - ĐH Cần Thơ


PT – BPT – HPT FB: http://www.facebook.com/VanLuc168

Ví dụ 8. Giải bất phương trình:


1  1  4x 2
 3, x  R.
x

 Giải
Điều kiện:
 1
1  4x 2  0  2  x  0
   .
x  0 0  x  1
  2

Cách 1: Thực hiện phép nhân liên hợp:


(1  1  4x2 )(1  1  4x2 )
(1)  < 3(1 + 1  4x2 )
x

 4x < 3 + 3 1  4x2 3 1  4x2 > 4x3


 3
x 
4 x  3  0  4
   1
 2  x  0
1
1  4 x 2  0 | x | 2
   
x 0
  .
4 x  3  0  3 0  x  1
  x  4  2
9(1  4 x )  (4 x  3)
2 2

9(1  4 x 2 )  (4 x  3)2

Cách 2: Xét hai trường hợp dựa trên điều kiện.


 Với  1  x < 0 thì:
2
1  3x  0
(1)  1  4x2 < 13x  
1  4x2  (1  3x)2
 1
x 
  3  x < 0.
13x2  6x  0

Kết hợp với điều kiện đang xét được nghiệm là  1  x < 0.
2

 Với 0 < x  1
thì:
2
(1)  1  4x2 > 13x
 1
x 
1  3x  0  3
  1 1 1
3  x  2
1
1  4 x 2  0  2  x  2
   0<x 1
.
1  3x  0  1 0  x  1 2
   x   3
1  4 x  (1  3x)
2 2
  3
13x 2  6x  0

NGUYỄN VĂN LỰC  0933.168.309 SP Toán K35 - ĐH Cần Thơ


PT – BPT – HPT FB: http://www.facebook.com/VanLuc168

Kết hợp với điều kiện đang xét được nghiệm là 0 < x  1
.
2
Vậy, bất phương trình có tập nghiệm là [ 1 ; 0)  (0 ; 1
].
2 2

Ví dụ 9. Giải bất phương trình:


4x 2
 2x  2, x  R.
(1  1  2x )2

Thiết lập điều kiện có nghĩa cho bất phương trình, rồi sử dụng phép nhận liên hợp để biến
đổi bất phương trình về dạng cơ bản.
 Giải
Điều kiện:
2x  1  0
 1
  0  2x + 1  1    x  0. (*)
1  2x  1  0
 2
Trục căn thức, ta biến đổi bất phương trình về dạng:
 
2
 2x 2x  1  1 
 
2
   2x  9  2x  1  1  2x  9
 
 2x  1  1 2x  1  1 
  
 2x  1  1  2 2x  1  2x  9

(*)
 2x  1  2x  1  2x  1  (2x  1) 2
x  0
 4x + 2x > 0  
2
.
x   1
 2
Vậy, bất phương trình có tập nghiệm là (0; +).

Ví dụ 10. Giải bất phương trình:


2x 2
 x  21, x  R.
(3  9  2x ) 2

 Giải
Điều kiện:
9  2x  0
  0  9 + 2x  9   9  x  0. (*)
3  9  2x  0 2

Cách 1: Biến đổi bất phương trình về dạng:


2x2 x2
< x + 21  < x + 21
9  9  2x  6 9  2x 9  x  3 9  2x
 x < (9 + x3
2
9  2x )(x + 21)  (x + 21) 9  2x < 10x + 63

NGUYỄN VĂN LỰC  0933.168.309 SP Toán K35 - ĐH Cần Thơ


PT – BPT – HPT FB: http://www.facebook.com/VanLuc168
(*)
 (x + 21)2(9 + 2x) < (10x + 63)2  2x37x2 < 0
(*)
 2x7 < 0  x < 7
.
2
Kết hợp với điều kiện (*), nghiệm của bất phương trình là x  [ 9 , 7
) \ {0}
2 2

Cách 2: Trục căn thức, ta biến đổi bất phương trình về dạng:
(3 + 9  2x )2 < 2(x + 21)  9  2x < 4  x < 7 .
2
Kết hợp với điều kiện (*), nghiệm của bất phương trình là x  [ 9 , 7
) \ {0}
2 2

Cách 3: Sử dụng phương pháp đặt ẩn phụ.


Đặt t = 9  2x , 0  t  3 suy ra
2x = t29  x = 1 (t29)
2
Khi đó bất phương trình có dạng:
4x2
< 2x + 42  (t29)2 < (3t)2(t29 + 42)
(3  9  2x ) 2

 (t + 3)2 < t2 + 33  t < 4  9  2x <4x< 7


.
2
Kết hợp với điều kiện (*), nghiệm của bất phương trình là x  [ 9 , 7
) \ {0}
2 2

Ví dụ 11. Giải bất phương trình:


2x
x+ >3 5. (1)
x2  4

Thiết lập điều kiện có nghĩa cho bất phương trình.


Dựa vào tập xác định để thực hiện phương pháp chia khoảng.
Ẩn phụ xuất hiện khi bình phương hai vế của bất phương trình.
 Giải
Điều kiện:
x24 > 0  x > 2. (*)
Trường hợp 1: Với x < 2 thì bất phương trình vô nghiệm (do vế trái âm).
Trường hợp 2: Với x > 2 thì bình phương 2 vế phương trình (1) ta được:
4x 2 4x 2 x4 x2
x2 + + > 45  + 4. > 45 . (2)
x2  4 x2  4 x2  4 x2  4
x2
Đặt t = , t > 0.
x2  4
Khi đó, bất phương trình (2) có dạng:
t  5 x2
t2 + 4t45 > 0  t>5 > 5  x425x2 + 100 > 0
 t  9 x 4
2

NGUYỄN VĂN LỰC  0933.168.309 SP Toán K35 - ĐH Cần Thơ


PT – BPT – HPT FB: http://www.facebook.com/VanLuc168

 x 2  20 | x | 20
  2   .
 x  5 | x | 5
Kết hợp với trường hợp đang xét, ta được tập nghiệm của bất phương trình là:
(;  20 )  ( 5 ; 5 )  ( 20 ; +).

Ví dụ 12. Giải bất phương trình:


x21  2x x 2  2x .

Bất phương trình được mở rộng từ dạng cơ bản f(x)  g(x) thành h(x) f(x)  g(x) nên
chưa thể sử dụng phép khai phương.
Trước tiên, hãy đi đặt điều kiện có nghĩa cho bất phương trình.
Nhận xét rằng với ẩn phụ t  x 2  2x, (t  0) , ta được:
x2  2tx  1  0.
suy ra, bất phương trình bậc hai ẩn x và tham số t.
 Giải
Đặt t = x 2  2x , điều kiện t  0.
Bất phương trình có dạng:
f(x) = x22tx1  0. (1)
Coi vế trái là một tam thức bậc 2 theo x, ta có:
’ = t2 + 1 = x2 + 2x + 1 = (x + 1)2
khi đó f(x) = 0 có các nghiệm:
x  t  x  1
x  t  x  1

tức là (1) được biến đổi về dạng:
(xtx1)(xt + x + 1)  0  ( x 2  2x + 1)( x 2  2x 2x1)  0
 x 2  2x 2x1  0  x 2  2x  2x + 1
 1
 2x  1  0 x   2
2x  1  0  2 
    x  2x  0   x0  x  0.
0  x  2x  (2x  1) 
2 2
 2
3x  2x  1  0   x  2
Vậy, bất phương trình có nghiệm x  0.

Ví dụ 13. Giải bất phương trình:


x2 + 4x  (x + 4) x 2  2x  4 .

 Giải

Đặt t = x 2  2x  4 , điều kiện t  0.


Bất phương trình có dạng:
f(x) = x2(t4)x4t  0. (1)

NGUYỄN VĂN LỰC  0933.168.309 SP Toán K35 - ĐH Cần Thơ


PT – BPT – HPT FB: http://www.facebook.com/VanLuc168

Coi vế trái là một tam thức bậc 2 theo x, ta có:


 = (t4)2 + 16t = (t + 4)2
khi đó f(x) = 0 có các nghiệm:
 x  4
x  t

tức là (1) được biến đổi về dạng:
(x + 4)(xt)  0  (x + 4)(x x 2  2x  4 ) 0
  x  4  0   x  4
  2
 x  x  2x  4  0  x  2x  4  x
2

   
 x  4  0  x  4
  
  x  x  2x  4  0   x  x  2x  4  0
2 2

  x  4

x  0 x  2
 
 0  x  2x  4  x
  x  4 .

2 2
 
 x  4

Vậy, tập nghiệm của bất phương trình là (; 4]  [2; +).

NGUYỄN VĂN LỰC  0933.168.309 SP Toán K35 - ĐH Cần Thơ


PT – BPT – HPT FB: http://www.facebook.com/VanLuc168

2. BẤT PHƯƠNG TRÌNH CHỨA HAI CĂN BẬC HAI

Ví dụ 1. Giải bất phương trình:


x  9  5  2x  4, x  R.

Dễ thấy chưa thể sử dụng ngay phép khai phương cho bất phương trình này, suy ra cần
biến đổi:
x  9  2x  4  5.
Tới đây, ta sẽ nhận được bất phương trình dạng cơ bản.
Ngoài ra, cũng có thể sử dụng phương pháp hàm số.
 Giải
Ta có thể trình bày theo các cách sau:
Cách 1: Điều kiện:
x  9  0
  x 2. (*)
2x  4  0
Biến đổi bất phương trình về dạng:
x  9  2x  4  5  x  9  2x  4  2 (x  9)(2x  4)  25
12  3x  0

 2 (x  9)(2x  4)  12  3x   12  3x  0
 4(x  9)(2x  4)  (12  3x) 2

 x > 0.
Vậy, tập nghiệm của bất phương trình là (0; +).

Cách 2: Điều kiện:


x  9  0
  x 2. (*)
2x  4  0
Biến đổi bất phương trình về dạng:
x  9  2x  4  5.
Nhận xét rằng:
 VT là hàm đồng biến.
 VP là hàm hằng.
Hai đồ thị cắt nhau tại điểm có hoành độ x = 0.
Vậy, tập nghiệm của bất phương trình là (0; +).

Ví dụ 2. Giải các bất phương trình:


x  1  5  2x  3, x  R.

NGUYỄN VĂN LỰC  0933.168.309 SP Toán K35 - ĐH Cần Thơ


PT – BPT – HPT FB: http://www.facebook.com/VanLuc168

 Giải
Ta có thể trình bày theo các cách sau:
Cách 1: Điều kiện:
x  1  0
  x 1. (*)
2x  3  0
Biến đổi bất phương trình về dạng:
x  1  2x  3  5  x  1  2x  3  2 (x  1)(2x  3)  25
 21  3x  0

 2 (x  1)(2x  3)  21  3x   21  3x  0
 4(x  1)(2x  3)  (21  3x) 2

 x > 3.
Vậy, tập nghiệm của bất phương trình là (3; +).

Cách 2: Điều kiện:


x  1  0
  x 1. (*)
2x  3  0
Biến đổi bất phương trình về dạng:
x  1  2x  3  5
Nhận xét rằng:
 VT là hàm đồng biến.
 VP là hàm hằng.
Hai đồ thị cắt nhau tại điểm có hoành độ x = 3.
Vậy, tập nghiệm của bất phương trình là (3; +).

Ví dụ 3. Giải các bất phương trình:


3  x  x  2  1, x  R.

 Giải
Ta có thể trình bày theo các cách sau:
Cách 1: Điều kiện:
3  x  0
  2 ≤ x ≤ 3.
x  2  0
Biến đổi bất phương trình:
3  x  1  x  2  3  x  1  (x  2)  2 x  2  x  2  x
x  0
 x  0 x  0 
  2    x  1  x < 1.
 x  2  ( x) x  x  2  0
2
 x  2

Vậy, tập nghiệm của bất phương trình là [2; 1).

NGUYỄN VĂN LỰC  0933.168.309 SP Toán K35 - ĐH Cần Thơ


PT – BPT – HPT FB: http://www.facebook.com/VanLuc168

Cách 2: Điều kiện:


3  x  0
  2 ≤ x ≤ 3.
x  2  0
Biến đổi bất phương trình về dạng:
3  x  1  x  2.
Nhận xét rằng:
o VT là hàm nghịch biến.
o VP là hàm đồng biến.
Hai đồ thị cắt nhau tại điểm có hoành độ x = 1.
Vậy, tập nghiệm của bất phương trình là [1; 3).

Ví dụ 4. Giải bất phương trình:


x  2  x2  5  3, x  R.

Bất phương trình chứa hai căn bậc hai với lõi là các hàm số bậc hai. Nên không thể sử
dụng phương pháp bình phương.
Bất phương trình được giải theo cách "Nhẩm nghiệm x0" rồi chuyển về dạng tích (x 
x0)h(x) bằng phép nhân liên hợp. Cụ thể:
o Nhận xét rằng x0 = 3 thoả mãn VT = VP..
o Biến đổi bất phương trình về dạng:
  
x  2 1  
x2  5  2  0 
x  2 1
x  2 1

x2  5  4
x2  5  2
0

x 3 x2  9
  0
x  2 1 x2  5  2
 1 x3 
 (x  3)     0.
 x  2 1 x2  5  2 
 Giải
Ta có thể trình bày theo các cách sau:
Cách 1: Điều kiện:
x  2  0 x  2

 2    x  5. (*)
 x  5  0 
 x  5
Biến đổi phương trình về dạng:
  
x  2 1  
x2  5  2  0 
x  2 1
x  2 1

x2  5  4
x2  5  2
0

x 3 x2  9
  0
x  2 1 x2  5  2
 1 x3 
 (x  3)     0.
 x  2 1 x2  5  2 
 x  3 > 0.  x > 3.
Vậy, bất phương trình có tập nghiệm là (3; +).
NGUYỄN VĂN LỰC  0933.168.309 SP Toán K35 - ĐH Cần Thơ
PT – BPT – HPT FB: http://www.facebook.com/VanLuc168

Cách 2: Điều kiện:


x  2  0 x  2

 2    x  5. (*)
 x  5  0 
 x  5
Xét hàm số f(x)  x  2  x2  5 trên D   5;    :
1 x
f '(x)   > 0, xD  Hàm số đồng biến trên D.
2 x2 x 5
2

Nhận xét rằng phương trình có:


o VT là hàm đồng biến.
o VP là hàm hằng.
Hai đồ thị cắt nhau tại điểm có hoành độ x = 3.
Vậy, bất phương trình có tập nghiệm là (3; +).

Ví dụ 5. Giải bất phương trình:


x2  3x  3  x2  3x  6  3, x  R.

 Giải
Ta có thể trình bày theo các cách sau:
Cách 1: Đặt t = x23x + 3, ta có:
2
 3 3 3
t  x    
 2 4 4
3
do đó, điều kiện cho ẩn phụ t là t .
4
Khi đó, bất phương trình có dạng:
t + t3 < 3  t + t + 3 + 2 t(t  3) <9 t(t  3) < 3t
3  t  0 t  3
     t < 1  x23x + 3 < 1
t(t  3)  (3  t) t  1
2

 x23x + 2 < 0  1 < x < 2.


Vậy, bất phương trình có tập nghiệm là (1; 2).

Cách 2: Biến đổi phương trình về dạng:


  
x 2  3x  3  1  x 2  3x  6  2  0 
x 2  3x  3  1 x 2  3x  6  4
  0
x 2  3x  3  1 x 2  3x  6  2
x 2  3x  2 x 2  3x  2
  0
x 2  3x  3  1 x 2  3x  6  2
 1 1 
 (x2  3x  2)   0
 x  3x  3  1 x 2  3x  6  2 
2

NGUYỄN VĂN LỰC  0933.168.309 SP Toán K35 - ĐH Cần Thơ


PT – BPT – HPT FB: http://www.facebook.com/VanLuc168

 x23x + 2 < 0  1 < x < 2.


Vậy, bất phương trình có tập nghiệm là (1; 2).

Ví dụ 6. (Đề thi đại học  Khối B năm 2012): Giải bất phương trình:
x  1  x 2  4x  1  3 x, (x  R).

Dễ thấy không thể sử dụng ngay phép khai phương cho bất phương trình này, suy ra cần
sử dụng ẩn phụ.
Câu hỏi được đặt ra là ẩn phụ kiểu gì ?
 Ẩn phụ dễ nhận thấy nhất là t  x (t  0) và khi đó ta nhận được bất phương trình
dạng:
t 4  4t 2  1   t 2  3t  1.
Trong trường hợp này cần phải giải một bất phương trình cao hơn 2.
 Từ việc đánh giá hệ số và x hoàn toàn được đưa vào căn bậc hai nên nếu chia cả
1 1
hai vế của phương trình cho x  0 sẽ thấy xuất hiện x và x , từ đó nhận
x x
1
được ẩn phụ t x (t  2) . Và khi đó, ta nhận được bất phương trình dạng:
x
t 2  6  3  t.
 Nhận xét: 1. Với bất phương trình đã cho, trước tiên chúng ta cần đặt điều kiện có
nghĩa.
2. Trong cả hai lựa chọn chúng ta đều gặp bất phương trình dạng cơ bản:
g  0

f  g   g  0 .
 f  g2

 Giải
Ta có thể trình bày theo các cách sau:
Cách 1: Điều kiện:
x 2  4x  1  0 x  2  3
  . (*)
x  0 0  x  2  3
Đặt t  x (t  0) bất phương trình được chuyển về dạng:
t 2  1  t 4  4t 2  1  3t  t 4  4t 2  1  t 2  3t  1. (1)
Ta xét hai trường hợp:
Trường hợp 1: Bất phương trình (1) đúng khi:
3 5 3 5
t 2  3t  1  0  t 2  3t  1  0  t  hoặc t .
2 2
Trường hợp 2: Với điều kiện:
3 5 3 5
t2 + 3t  1  0  t . (**)
2 2
Bất phương trình (1) được chuyển về dạng:

NGUYỄN VĂN LỰC  0933.168.309 SP Toán K35 - ĐH Cần Thơ


PT – BPT – HPT FB: http://www.facebook.com/VanLuc168

 
2
t 4  4t 2  1  t 2  3t  1  6t 3  15t 2  6t  0
 1 (**)  3  5 1
t  0  t  t
 3t(2t  5t  2)  0   2  2  2
2
 2.
2t  5t  2  0
t  2  t  2
Kết hợp hai trường hợp 1 và trường hợp 2, ta được:
 1  1  1 (*)  1
x
t  2   2   x
4   0x
4.
   
t  2  x  2 x  4 x  4

bất phương trình có tập nghiệm là 0;    4;   .


1
Vậy,
 4

Cách 2: Điều kiện:


x 2  4x  1  0 x  2  3
  . (*)
x  0 0  x  2  3
Nhận xét rằng x = 0 là nghiệm của bất phương trình.
Với x > 0, biến đổi bất phương trình về dạng:
1 1
x  x4 3
x x
1 1 2
Đặt t x (t  2) suy ra  t  2 nên bất phương trình
x được chuyển về dạng:
x x
3  t  0

t  t 2  6  3  t 2  6  3  t   3  t  0
 t 2  6  (3  t)2

t  3
 5 1 5 u  x 0 1 5
  t  3  t   x   u 
 6t  15
2 x 2 u 2

u  2  x 2 x  4
 2u  5u + 2  0 
2     .
u  1  x1 0  x  1
 2  2  4

Vậy, bất phương trình có tập nghiệm là 0;    4;   .


1
 4

Ví dụ 7. Giải bất phương trình:


5 1
5 x  2x  , x  R.
2 x 2x

 Giải
Điều kiện x > 0. (*)
Viết lại phương trình dưới dạng:

NGUYỄN VĂN LỰC  0933.168.309 SP Toán K35 - ĐH Cần Thơ


PT – BPT – HPT FB: http://www.facebook.com/VanLuc168

1 1
5( x + ) < 2(x + ) +4 (2)
2 x 4x
1
Đặt t = x + , ta có nhận xét:
2 x
1 C «si 1
x +  2 x = 2 ,
2 x 2 x
vậy điều kiện là t  2 . (**)
Mặt khác:
2
 1 
2
t =  x   = x+ 1
+1x+ 1
= t21.
 2 x 4x 4x
Khi đó, bất phương trình có dạng:
5t < 2(t21) + 4  2t25t + 2 > 0
t  2 (**)
   t>2 x + 1
> 2.
t  1 / 2 2 x
Đặt X = x , X > 0, khi đó:
X + 1 > 2  2X24X + 1 > 0
2X
 2 2  2 2  3
X   x x  2  2
 

2  

2   .
2 2 2 2 0  x  3  2
X   x
 2  2  2

Vậy, bất phương trình có nghiệm (0, 3  2 )(3 + 2 , + ).


2 2

Ví dụ 8. Giải bất phương trình:


2x 2  6x  8  x  x  2, x  R.

Biến đổi bất phương trình về dạng:


2(x  2) 2  2x  x2 + x .
Sử dụng hai ẩn phụ:
u  x  0
 .
v  x  2

 Giải
Điều kiện x  0. (*)
Biến đổi bất phương trình về dạng:
2(x  2) 2  2x  x2 + x . (2)
Đặt:
u  x  0
 .
v  x  2

Khi đó, bất phương trình có dạng:


u  v  0 u  v  0
2u 2  2 v 2 u+v   2  
2 u  2 v 2  (u  v )2 (u  v )2  0

NGUYỄN VĂN LỰC  0933.168.309 SP Toán K35 - ĐH Cần Thơ


PT – BPT – HPT FB: http://www.facebook.com/VanLuc168

x  2  0 x  2
u=v0    2 x=4
 x  x  2 x  5x  4  0
Vậy, nghiệm của bất phương trình là x = 4.

Ví dụ 9. Giải bất phương trình:


2x 2  12x  6  2x 1  x  2, x  R.

 Giải
Điều kiện:
2 x 2  12 x  6  0
  x 1
. (*)
2 x  1  0 2

Biến đổi bất phương trình về dạng:


2(x  2) 2  2(2x  1) > x + 2 + 2x  1 . (2)
Đặt
u  2 x  1  0
 .
v  x  2

Khi đó, bất phương trình có dạng:


u  v  0 u  v  0
2u 2  2 v 2 >u+v   2    u  v.
2 u  2 v 2  (u  v )2 (u  v )2  0

Xét trường hợp u = v


x  1
 2x  1 = x + 2  x26x + 5 = 0  
x  5

Suy ra, để u  v, ta phải có x  [ 1 , + ) \ {1, 5}.


2
Vậy, nghiệm của bất phương trình là x  [ 1 ; +) \ {1; 5}.
2

Ví dụ 10. Giải bất phương trình:


2x 2  10x  16  x 1  x  3, x  R.

 Giải
Hướng dẫn: Viết lại bất phương trình dưới dạng:
2(x  1)  2(x  3)2  x  1 + x3.
Sử dụng phép biến đặt ẩn phụ u = x  1 và v = x3.

Ví dụ 11. (Đề thi đại học  Khối A năm 2010): Giải bất phương trình:
x x
 1.

1  2 x2  x  1 

NGUYỄN VĂN LỰC  0933.168.309 SP Toán K35 - ĐH Cần Thơ


PT – BPT – HPT FB: http://www.facebook.com/VanLuc168

Đây là bất phương trình không mẫu mực chứa căn bậc hai và được cho dưới dạng phân
P(x)
thức  k (k lµ h»ng sè) , do vậy để giải nó chúng ta cần có những đánh giá dần như sau:
Q(x)
 Nhận xét về dấu của Q(x) đề chuyển bất phương trình về dạng:
P(x)  k.Q(x) hoặc P(x) ≤ k.Q(x).
Với bài toán này, ta có:
Q(x) < 0  1  2  x2  x  1  0  2  x2  x  1  1
 
 2 x2  x  1  1  2x2  2x + 1 > 0, luôn đúng.
hoặc:
 
2 x2  x  1  x2   x  1  1
2
> 1  MS  0
hoặc:
2
 1 3
 
2 x  x 1  2 x    
2

 2 2
3
2
 1  MS  0

 Suy ra, bất phương trình được biến đổi về dạng:


x  x  1  2  x2  x  1  2  x2  x  1  1  x  x. (1)
Tới đây, việc lựa chọn phương pháp giải cho bất phương trình (1) sẽ được dựa theo
dạng xuất phát cơ bản là f  g . Tuy nhiên, như đã trình bày trong phần cấu trúc đề
thi đại học môn toán thì đây luôn là câu hỏi khó nên các em học sinh cần có kiến thức
rất tốt mới có thể tiếp tục được. Cụ thể, chúng ta sẽ lựa chọn một trong các hướng
sau:
Hướng 1: Sử dụng phép biến đổi tương đương:
g  0
f g  .
f  g
2

Và với hướng này cần có kinh nghiệm tốt trong việc biến đổi đại số.
Hướng 2: Sử dụng ẩn phụ t  x (t  0) và phép biến đổi tương đương giống như
hướng 1 để nhận được một bất phương trình bậc 4 theo t.
Hướng 3: Sử dụng ẩn phụ t là tổ hợp của x và phép biến đổi tương đương giống
như hướng 1 để nhận được một bất phương trình bậc 2 theo t. Cụ thể trong
1
bài toán này chúng ta sẽ đặt t  x.
x
Hướng 4: Sử dụng phương pháp đánh giá (nếu có thể). Cụ thể trong bài toán này
chúng ta sử dụng bất đẳng thức 2(a2 + b2)  (a + b)2 bởi ta có biến đổi:
 
2 x 2  x  1  2 1  x    x    1  x  x 
2 2 2

  1  x  x.

 Giải
Nhận xét rằng: 2  x2  x  1  x2   x  12  1 > 1  MS  1  2  x2  x  1  0.
Điều kiện x  0.
Bất phương trình được biến đổi về dạng:

NGUYỄN VĂN LỰC  0933.168.309 SP Toán K35 - ĐH Cần Thơ


PT – BPT – HPT FB: http://www.facebook.com/VanLuc168

  
x  x  1  2 x2  x  1  2 x2  x  1  1  x  x.  (1)
Tới đây, chúng ta có thể trình bày theo các cách sau:
Cách 1: Biến đổi tiếp (1) về dạng:
1  x  x  0

 2

x   x  1  1  1  x  x 
2 2


1  x  x  0 
1  x  x  0
   2
   
2 2

 x 2
 x  1  1  1  x  2(1  x) x  x x  1  2(1  x) x  x

1  x  x  0 1  x  x  0

 2 
   
2
 x  2x  1  2(1  x) x  x  0  1  x  x  0

1  x  x  0 2 x  0 1  x  0
 x  1
   2   2
1  x  x  0
  x  1  x x  1  x 
 x  3x  1  0
3 5
x .
2
Vậy, bất phương trình có nghiệm x  3  5 .
2

Cách 2: Đặt t  x (t  0) , khi đó (1) có dạng:


1  t 2  t  0
 4 2

2 t  t 1  1 t  t   4 2
2

   
2
2 t  t  1  1  t  t
2

1  t 2  t  0
 4 2
2  t  t  1  1  t  t  2t  2t  2t
4 2 2 3

1  t 2  t  0 
1  t  t  0
2
1  t 2  t  0
 
 4    2
 
2
t  2t  t  2t  1  0

3 2

 t 2
 t  1  0 t  t  1  0

2t  0
 5 1 5 1 3 5
  1  5  t   x x .
t  2 2 2
 2

Vậy, bất phương trình có nghiệm x  3  5 .


2

Cách 3: Nhận xét rằng x = 0 không phải là nghiệm của (1) nên ta có biến đổi:
 1 1
2 x 1    x  1. (2)
 x x
1 1
Đặt t  x thì x   t 2  2 , khi đó (2) có dạng:
x x
t  1  0  t  1 t  1
 
2 t2  1  t  1   2 2 
2(t  1)  (t  1)  t  2t  1  0  t  1  0
2 2

NGUYỄN VĂN LỰC  0933.168.309 SP Toán K35 - ĐH Cần Thơ


PT – BPT – HPT FB: http://www.facebook.com/VanLuc168

1 1  x  0

t=1   x  1  x  1 x  
x  1  x 
2
x 
x  1 3 5
 2 x .
x  3x  1  0 2
3 5
Vậy, bất phương trình có nghiệm x .
2

Cách 4: Sử dụng bất đẳng thức 2(a2 + b2)  (a + b)2, ta thấy:


 
2 x 2  x  1  2 1  x    x    1  x  x 
2 2 2


 1  x  x  1  x  x. (3)
Từ (1) và (3) suy ra bất phương trình được biến đổi về dạng:
1  x  0
1  x  x  x  1 3 5
 1  x   x   2
2
 x .
1  x  x  0  x  3x  1  0 2
2 x  0
3 5
Vậy, bất phương trình có nghiệm x .
2

Ví dụ 12. Giải bất phương trình:


x  2ax  a 2  x  2ax  a 2  2a, a  0, x  R.

Hẳn bước đặt điều kiện có nghĩa cho bất phương trình đã phức tạp. Do đó, bài toán cần
có cách giải khác bằng việc đánh giá dạng đặc thù của căn thức:
2ax  a 2  a 2
x  2ax  a 2 =  2ax  a 2
2a

2ax  a 2  a
2ax  a 2  2a 2ax  a 2  a 2
  .
2a 2a
Tới đây, cần sử dụng đúng tính chất giá trị tuyệt đối.
 Giải
Ta có nhận xét:
2ax  a 2  a 2
x  2ax  a 2 =  2ax  a 2
2a

2ax  a 2  a
2ax  a 2  2a 2ax  a 2  a 2
  ,
2a 2a
2ax  a 2  a
x  2ax  a 2  .
2a
Khi đó, bất phương trình được biến đổi tương đương thành:

NGUYỄN VĂN LỰC  0933.168.309 SP Toán K35 - ĐH Cần Thơ


PT – BPT – HPT FB: http://www.facebook.com/VanLuc168

| 2ax  a 2  a | | 2ax  a 2  a |
+  2a
2a 2a
 2ax  a 2 + a +  2ax  a 2 a 2a
a 0
 + a +  2ax  a2 a 2a
2ax  a 2
  2ax  a2 a a 2ax  a2  2ax  a2 a  0
2ax  a 2  0 a 0 a
 2ax  a 2 a    x  a.
2ax  a 2  a 2 2

Vậy, bất phương trình có nghiệm a  x  a.


2

Ví dụ 13. Giải bất phương trình:


3
x  2 x  1  x  2 x  1  , x  R.
2

 Giải
Ta có thể trình bày theo các cách sau:
Cách 1: Viết lại bất phương trình dưới dạng:
3
x 1 2 x 1 1 + x 1  2 x 1  1 >
2

 ( x  1  1)2 + ( x  1  1)2 > 3


.
2
Điều kiện: x1  0  x  1. (*)
Khi đó, phương trình trở thành:
 x  1  1  0

 3
2 x  1  2 x  2
x 1 +1+ x 1 1 > 3
     x.
2  x  1  1  0 x  2

2  3
 2
Kết hợp với điều kiện (*) được x  1 là nghiệm của bất phương trình.

Cách 2: Điều kiện:


x  1  0

 x  x  1  0  x  1. (*)

x  x  1  0
Bình phương hai vế của bất phương trình, ta được:
2x  2 x  2 
x 1 x  2 x 1   9
4
9 9
 2x  2 x 2  4(x  1) 
 2x  2 x 2  4x  4 
4 4
9 9
 2 (x  2) 2   2x  2 x  2   2x. (1)
4 4

NGUYỄN VĂN LỰC  0933.168.309 SP Toán K35 - ĐH Cần Thơ


PT – BPT – HPT FB: http://www.facebook.com/VanLuc168

Ta có biến đổi cho (1):


 Với x  2  0 tức x  2 thì:
9 25 25
(1)  2(x  2)   2x  4x   x .
4 4 16
Suy ra, nghiệm trong trường hợp này là x  2.
 Với x  2 < 0 tức x < 2 thì:
9 9
(1)  2(2  x)   2x  4  , luôn đúng.
4 4
1
 x2  0, vô nghiệm
2
Suy ra, nghiệm trong trường hợp này là x < 22.
Suy ra (1) nghiệm đúng với mọi x.
Vậy, bất phương trình có tập nghiệm là [1; +).

Ví dụ 14. Giải bất phương trình:


x  x 2  1  x  x 2  1  2, x  R.

Ta có thể trình bày theo các cách sau:


Cách 1: Điều kiện:
x 2  1  0


x  x  1  0  x  1.
2
(*)

x  x 2  1  0

Nhận xét rằng:
VT = x  x2  1 + x  x2  1  2 x  x2  1 . x  x2  1 = 2.
Vậy, bất phương trình có nghiệm khi và chỉ khi
VT = 2  x  x2  1 = x  x2  1  x  x 2  1  x  x 2  1
x  1
 2 x 2 1  0  x 2  1  0   .
 x  1 (loai)
Vậy, nghiệm của bất phương trình là x = 1.
Cách 2: Điều kiện:
x 2  1  0


x  x  1  0  x  1.
2
(*)

x  x 2  1  0

Bình phương hai vế của bất phương trình, ta được:
2x  2 x   
x2 1 x  x2 1  4

 2x  2 x 2   x 2  1  4  2x  2  4  x  1.
Vậy, nghiệm của bất phương trình là x = 1.

NGUYỄN VĂN LỰC  0933.168.309 SP Toán K35 - ĐH Cần Thơ


PT – BPT – HPT FB: http://www.facebook.com/VanLuc168

3. BẤT PHƯƠNG TRÌNH CHỨA NHIỀU CĂN BẬC HAI

Ví dụ 1. (Đề thi đại học  Khối A năm 2005): Giải bất phương trình:
5x  1  x  1  2x  4, x  R.

Đây là bất phương trình vô tỉ và có thể nhận thấy ngay rằng sau phép chuyển vế được
bất phương trình dạng f (x) > g(x) + h(x) , do đó các bước thực hiện bao gồm:
 Thiết lập điều kiện có nghĩa cho bất phương trình.
(*)
 Biến đổi bất phương trình về dạng:
f(x) > g(x) + h(x) + 2 g(x).h(x)
p(x)  0
 g(x).h(x) < p(x)    nghiệm.
g(x).h(x)  p (x)
2

 Kết hợp với (*), nhận được nghiệm của bất phương trình.

 Giải
Điều kiện:
5x  1  0

x  1  0  x  2. (*)
 2x  4  0

Biến đổi bất phương trình về dạng:
5x  1 > 2x  4 + x  1
 5x  1 > 2x  4 + x  1 + 2 (2x  4)(x  1)
(*)
 (2x  4)(x  1) < x + 2  (2x  4)(x  1) < (x + 2)2
 x2  10x < 0  0 < x < 10.
Kết hợp với (*), ta được nghiệm của bất phương trình là 2  x < 10.

Ví dụ 2. Giải bất phương trình:


x 2  3x  2  x 2  4x  3  2 x 2  5x  4, x  R. (1)

Dể thấy không thể sử dụng phép khai phương để giải bất phương trình này.
Nhận thấy nhân tử chung x  1 , nên ta sẽ thực hiện theo các bước:
 Đặt điều kiện có nghĩa cho bất phương trình.
 Sử dụng phương pháp chia khoảng.
 Giải
Điều kiện:

NGUYỄN VĂN LỰC  0933.168.309 SP Toán K35 - ĐH Cần Thơ


PT – BPT – HPT FB: http://www.facebook.com/VanLuc168

 x 2  3x  2  0
 2 x  4
 x  4x  3  0  .
 2 x  1
 x  5x  4  0
Trường hợp 1: Với x  4 thì:
(1)  (x  1)(x  2)  (x  1)(x  3)  2 (x  1)(x  4)
 x 2  x 3  2 x 4
 x 2  x 4  x 4  x 3
luôn đúng vì với x  4 ta được VT > 0 và VP < 0.
Vậy x  4 là nghiệm bất phương trình.
Trường hợp 2: Với x  1 thì:
(1)  (1  x)(2  x)  (1  x)(3  x)  2 (1  x)(4  x)
Với x = 1, bất phương trình nghiệm đúng.
Với x < 1, bất phương trình có dạng:
 2 x  3 x  2 4 x
 2 x  4 x  4 x  3 x
Nhận xét rằng với x < 1 thì VT < 0 và VP > 0, phương trình vô nghiệm.
Vậy, bất phương trình có nghiệm x = 1 hoặc x  4.

Ví dụ 3. Giải bấ t phương trình:


x 11 x 4 2x 1

x 11 0 x 11
● Điề u kiê ̣n: x 4 0 x 4 x 4.
2x 1 0 x 0, 5

x 11 3x 5 2 x 4 2x 1 x 4 2x 1 8 x

x 8 0 x 8
2 12 x 5.
x 4 2x 1 8 x x2 7x 60 0

● Kế t hơp̣ với điề u kiê ̣n, tâ ̣p nghiê ̣m của bấ t phương trình là: S 4;5 .

Ví dụ 4. Giải bấ t phương trình:


x 2 x 1 2x 3

NGUYỄN VĂN LỰC  0933.168.309 SP Toán K35 - ĐH Cần Thơ


PT – BPT – HPT FB: http://www.facebook.com/VanLuc168

3
● Điề u kiê ̣n: x .
2

x 2 2x 3 x 1 x 2 3x 4 2 x 1 2x 3

3
x 3
2 x 3
2
2x 5x 3 3 x 3 x 0 2
2 x2 x 6
2x 2 5x 3 3 x

3
x 3 3
2 x 2.
3 x 2 2

3
● Tâ ̣p nghiê ̣m của bấ t phương trình là x ;2 .
2

Ví dụ 5. Giải bấ t phương trình:


5x 1 4x 1 3 x

5x 1 0
1
● Điề u kiê ̣n: 4x 1 0 x .
4
x 0

5x 1 4x 1 3 x 5x 1 9x 4x 1 6 4x2 x

6 4x2 x 2 8x

1
● Do x 2 8x 0 luôn thỏa.
4

1
● Vâ ̣y tâ ̣p nghiê ̣m của bấ t phương trình là x ; .
4

Ví dụ 6. Giải bấ t phương trình:


x 2 3 x 5 2x

NGUYỄN VĂN LỰC  0933.168.309 SP Toán K35 - ĐH Cần Thơ


PT – BPT – HPT FB: http://www.facebook.com/VanLuc168

x 2 0
● Điề u kiê ̣n: 3 x 0 2 x 3.
5 2x 0

x 2 5 2x 3 x x 2 8 3x 2 5 2x 3 x

2x 3 0
5 2x 3 x 0
5 2x 3 x 2x 3
2x 3 0
2
5 2x 3 x 2x 3

3 3 3
x x 3 x
2 2 x 2 x 2.
5 2x2 x 6 0 2 3
x x 3 x 2
2 2

● Kế t hơp̣ với điề u kiê ̣n, tâ ̣p nghiê ̣m của bấ t phương trình là x 2;2 .

Ví dụ 7. Giải bấ t phương trình:

x2 8x 15 x2 2x 15 4x2 18x 18

x2 8x 15 0 x 5 x 3 x 5
● Điề u kiê ̣n: x2 2x 15 0 x 3 x 5 x 5.
2
4x 18x 18 0 3 x 3
x 3 x
2

● Với x 3 thì đươc̣ thỏa x 3 là mô ̣t nghiê ̣m của bấ t phương trình 1

x 5 x 3 x 5 x 3 x 3 4x 6 2

● Với x 5 x 3 2 0 hay x 3 0 thì

2 x 5 x 5 4x 6 2x 2 x2 25 4x 6

17
x2 25 x 3 x2 25 x2 6x 9 x .
3

17
5 x 3
3

NGUYỄN VĂN LỰC  0933.168.309 SP Toán K35 - ĐH Cần Thơ


PT – BPT – HPT FB: http://www.facebook.com/VanLuc168

● Với x 5 x 5 3 x 8 0 hay 3 x 0 thì

2 5 x 3 x x 5 3 x 3 x 6 4x

5 x x 5 6 4x 2x 2 5 x x 5 6 4x

17
x2 25 3 x x2 25 x2 6x 9 x .
3

x 5 4

17
● Từ 1 , 3 , 4 tâ ̣p nghiê ̣m của bấ t phương trình là x ; 5 3 5; .
3

NGUYỄN VĂN LỰC  0933.168.309 SP Toán K35 - ĐH Cần Thơ


PT – BPT – HPT FB: http://www.facebook.com/VanLuc168

BÀI TẬP TỰ LUYỆN

Câu 1. Giải bất phương trình: (4x 2  x  7) x  2  10  4x  8x 2

Điều kiện: x  2 , bất phương trình đã cho tương đương:


(4 x2  x  7) x  2  2(4 x 2  x  7)  2 ( x  2)  4
(4 x2  x  7)( x  2  2)  2( x  2  2)( x  2  2)
 4 x 2  x  7  2 x  2  4  4 x 2  ( x  2)  2 x  2  1
 (2 x) 2  ( x  2  1) 2  ( x  2  1  2 x)( x  2  1  2 x)  0
 x  2  2 x  1  x  2  2x 1
 hoặc  
 x  2  2 x  1  x  2  2 x  1
5  41
 2  x   1 hoặc  x 
8
 5  41 
Vậy tập nghiệm T   2; 1   ;   .
 8 

Câu 2. Giải bất phương trình: 2x  7  5  x  3x  2 .

2
+ ĐK:  x  5. Biến đổi bất phương trình về dạng
3
2 x  7  3x  2  5  x
+ Bình phương hai vế, đưa về được 3x 2  17 x  14  0
14
+ Giải ra được x  1 hoặc x 
3
2 14
+ Kết hợp với điều kiện, nhận được  x  1 hoặc  x  5.
3 3

Câu 3. Giải bất phương trình: x 3  1  x 2  1  3x x  1  0 .

+ x 3
    
 1  x 2  1  3x x  1  0  x 3  x 2  3 x 3  x 2  2  0
2
+ Đặt t  x x  1   . Bất phương trình  t 2  3t  2  0
3
14
+ Giải ra được x  1 hoặc x 
3

NGUYỄN VĂN LỰC  0933.168.309 SP Toán K35 - ĐH Cần Thơ


PT – BPT – HPT FB: http://www.facebook.com/VanLuc168

 2
t   3
 2 2
+   t    x x  1    x  1 .
t  1
 3 3

 t  2

Câu 4. Giải bất phương trình: 3x  2  x  3  2x  1

2
ĐK: x  
3
3x  2  x  3  2 x  1  3x  2  x  3  ( 3x  2  x  3)( 3x  2  x  3)
 1  3x  2  x  3 (vì 3x  2  x  3 >0)
 1  x  3  3x  2
 1  x  3  2 x  3  3x  2
 x  3  x 1
 x 1  0

   x 1  0
  x  3  x 2  2 x  1
 x 1
 
1  x  3  17
 2
2 3  17
So sánh với điều kiện , ta có nghiệm của bất phương trình là   x  .
3 2


Câu 5. Giải bất phương trình: x 2  5x  4 1  x (x 2  2x  4) (x R). 
 1  5  x  0
ĐK: x(x2 + 2x − 4) ≥ 0  
 x  1  5
Khi đó (*)  4 x( x2  2 x  4)  x2  5x  4
 4 x( x2  2 x  4)  ( x2  2 x  4)  3x (**)
TH 1: x  1  5 , chia hai vế cho x > 0, ta có:
x2  2 x  4 x2  2 x  4
(**)  4  3
x x
x2  2 x  4
Đặt t  , t  0 , ta có bpt: t 2  4t  3  0  1  t  3
x
x2  2 x  4  x 2  7 x  4  0 1  17 7  65
1 3  2  x
x  x  x  4  0 2 2
TH 2: 1  5  x  0 , x2  5 x  4  0 , (**) luôn thỏa

NGUYỄN VĂN LỰC  0933.168.309 SP Toán K35 - ĐH Cần Thơ


PT – BPT – HPT FB: http://www.facebook.com/VanLuc168

 1  17 7  65 
Vậy tập nghiệm bất phương trình (*) là S   1  5;0   ; .
 2 2 

Câu 6. Tìm tất cả các giá trị của tham số m để bất phương trình sau có nghiệm
 
x  x  2 , x  R  .
3
x 3  2x 2  8  m

Điều kiện x  2 . Bất phương trình đã cho tương đương với


x3  2 x2  8
 m   x 3  2 x 2  8  x  x  2   8m .
3

 
3
x  x2


Xét hàm số f  x   x 3  2 x 2  8  
3
x  x  2 có f '  x   0, x  2 nên hàm số f  x  đồng biến
trên 2;  .
Bất phương trình f  x   8m có nghiệm
 8m  m in f  x   f  2   16 2 .
x 2; 

Vậy m  2 2.

Câu 7. Giải bất phương trình: x  1  x 2  2  3x  4x 2 .

x  0 0  x  1
  3  41
Điềukiện: 1  x  0
2
  3  41 3  41  0  x  . (*)
  x 8
2  3x  4 x  0 
2
8 8
Bất phương trình đã cho tương đương với
x  1  x 2  2 x(1  x 2 )  2  3x  4 x 2  3( x 2  x)  (1  x)  2 ( x  x 2 )(1  x)  0
 5  34
 x
x x x x
2 2
x x 1
2
9
3 2 1  0    9 x 2  10 x  1  0  
1 x 1 x 1 x 3  5  34
x  .
 9
Kết hợp điều kiện (*), ta suy ra nghiệm của bất phương trình là
5  34 3  41
x  .
9 8

Câu 8. Giải bất phương trình:  x  1 x 2  5  x  x 2  1.

 x  1 : loại

x2  x  1 1 1
 x  1: x 2  5   x2  5  x   x2  5  x 
x 1 x 1 x 1

NGUYỄN VĂN LỰC  0933.168.309 SP Toán K35 - ĐH Cần Thơ


PT – BPT – HPT FB: http://www.facebook.com/VanLuc168

5 1
   5  x  1  x 2  5  x  4x  5  x 2  5
x2  5  x x 1
 5
x 
 4  x  2 . Vậy x  2.
15x 2  40x  20  0

Câu 9. Giải bất phương trình


(x 2)(x 2 2x 5) 9 (x 2)(3 x 2 5 x2 12) 3
5x 2 7

5
Điều kiện xác định: x   . Khi đó ta có
2
(1)  x 3  3x 2  14x  15  2(x  2) 2x  5  3(x  2) x 2  5  3 5x 2  7  0

 x 3  3x 2  x  18  2(x  2)( 2x  5  3)  3(x  2)( x 2  5  3)  3  3 5x 2  7  0


2(x  2)(2x  4) 3(x  2)(x 2  4) 5(4  x 2 )
 (x  2)(x  5x  9) 
2
  0
 
2
2x  5  3 x2  5  3 9  3 3 5x 2  7  3 5x 2  7

 
 2 4(x  2) 3(x  2)2 5(x  2) 
 (x  2)  x  5x  9      0(*)
 
2
 2x  5  3 x 2  5  3 9  3 3 5x 2  7  3 5x 2  7 
 
 
 4(x  2) 4 3(x  2)2 3
  (x  2);  (x  2)2
5  2x  5  3 3 x2  5  3 5
Ta có với x   5(x  2) 5(x  2)
2  
 
2
 9  3 3 5x 2  7  3 5x 2  7 9

4(x  2) 3(x  2)2 5(x  2) 18x 2  57x  127


 x  5x  9 
2
   0
  45
2
2x  5  3 x2  5  3 9  3 3 5x 2  7  3
5x 2  7

5
Do đó (*)  x  2  0  x  2 , kết hợp với điều kiện x   ta suy ra bất phương trình
2
5
đã cho có nghiệm là   x  2.
2

Câu 10: Giải bất phương trình: 2x  5  3x  2  4x  1  5x  6

BPT  2x  5  4x  1  3x  2  5x  6  0
1 1
 (2x  4)[  ]0
2x  5  4x  1 3x  2  5x  6
x 2

NGUYỄN VĂN LỰC  0933.168.309 SP Toán K35 - ĐH Cần Thơ


PT – BPT – HPT FB: http://www.facebook.com/VanLuc168

Câu 11. Giải bất phương trình: 3x (2  9x 2  3)  (4x  2)(1  1  x  x 2 )  0 1

Viết lại phương trình dưới dạng:


3x(2  (3x) 2  3)  (2 x  1)(2  [(2 x  1) 2 ]  3
t2
Xét hàm số f  t   t (2  t 2  3),  t   ; f (t )  2  t  3 
' 2
0
t2  3
hàm số f  t  luôn đồng biến
1
Do đó (1)  f  3x   f  2 x  1  3x  2 x  1  x   .
5

Vậy tập nghiệm của bất phương trình là T    ;   .


1
 5 

2 8
Câu 12. Giải bất phương trình: 2 1 2x x.
x x

 2  x  2
1
 x   0 
 x  0  2  x  0
Điều kiện của bất phương trình:   
2 x  8  0  x  2 x  2
 x 
  2  x  0

Với 2  x  0  bất phương trình đã cho luôn đúng

Với x  2  bất phương trình đã cho  2 x  2  2( x  2)( x  2)  x x


 4( x  2)  2( x 2  4)  4 ( x  2)2 ( x  2)  x3
 x3  2 x 2  4 x  16  4 2( x3  2 x 2  4 x  8)  0
 2( x3  2 x 2  4 x  8)  8 2( x3  2 x 2  4 x  8)  16  0

 
2
2( x3  2 x 2  4 x  8)  4  0  2( x3  2 x 2  4 x  8)  4

x  0

 x3  2 x 2  4 x  0   x  1  5  x  1  5 (do x  2 )
x  1 5

Vậy bất phương trình đã cho có tập nghiệm là T   2;0   1  5 .  
x2 x  2 2
Câu 13. Giải bất phương trình: x2   1 trên tập số thực.
x 3 x2 3

Điều kiện x  3. Bất phương trình đã cho tương đương với
NGUYỄN VĂN LỰC  0933.168.309 SP Toán K35 - ĐH Cần Thơ
PT – BPT – HPT FB: http://www.facebook.com/VanLuc168

x2  x  2 4
 2
x x2
2
2 x3 x  3  x2 1  0
  x2 1  0 
x3 x 3
2
x x2
2
2

x3 x 3
2

x 2
 1 x 2  x  6 
 x  3  x 2  3
  x2 1  0
x x2
2
2

x3 x 3
2

 
 
 2
  
  x 2  1 
x x 6
 1  0
 x  3 x2  3  x  x  2  2  
 
2

   

 

 x  3 x 2
 3  

 x  1  0  1  x  1 (Với x  3 thì biểu thức trong ngoặc
2
vuông luôn dương). Vậy tập
nghiệm của bất phương trình là S   1;1 .

Câu 14. Giải bất phương trình: 1  4x 2  20  x  4x 2  9.

Giải bất phương trình: 1  4 x 2  20  x  4 x 2  9. (1)


Bất phương trình đã cho tương đương với:
4 x 2  16 16  4 x 2
4 x  9  5  6  4 x  20  x  2  0 
2 2
  x20
4 x 2  9  5 6  4 x 2  20
 4x  8 4x  8 
  x  2    1  0
 4 x  9  5 6  4 x  20 
2 2

Từ (1) suy ra x  1  4 x 2  20  4 x 2  9  0  x  1 .
4x  8 4x  8 1  4 x 2  20  4 x 2  9
Do đó   1   4 x  8 . 1  0
4 x2  9  5 6  4 x 2  20  
4 x 2  9  5 6  4 x 2  20 
Vậy nghiệm của bất phương trình là x  2.

Câu 15. Giải bất phương trình: 9x 2  3  9x  1  9x 2  15.

1
Nhận xét : 9 x  1  9 x 2  15  9 x 2  3  0  x 
9
bpt   9x 2

 3  2  3(3x  1)  9 x 2  15  4
9x  1
2
9x 2  1
  3(3x  1)  0
9x 2  3  2 9 x 2  15  4

NGUYỄN VĂN LỰC  0933.168.309 SP Toán K35 - ĐH Cần Thơ


PT – BPT – HPT FB: http://www.facebook.com/VanLuc168

 3x  1 3x  1 
3x  1   3  0
 9x  3  2 9 x  15  4 
2 2

   
3x  13x  1 1

1
  3  0  3x  1  0  x 
1
  9x  3  2 9 x 2  15  4   3
2

1
Kết hợp các điều kiện suy ra nghiệm của bất phương trình là x  là nghiệm của bất
3
phương trình

1 1 2
Câu 16. Giải bất phương trình   trên tập số thực.
x 2 1 3x 2  5 x 2  2 1

1 1 2
+) Đặt t = x2 – 2, bất phương trình trở thành:   ĐK: t  0 với đk trên,
t 3 3t  1 t 1
bất phương trình tương đương
1 1
( t  1)(  )  2 . Theo Cô-si ta có:
t 3 3t  1
t t t 1 1  t t 1 
 .    
t 3 t 1 t  3 2  t 1 t  3 
1 1 2 11 2 
 .    
t 3 2 t 3 2 2 t 3
t 1 2t 11 2t 
 .    
3t  1 2 3t  1 2  2 3t  1 
1 1 t 1 1  1 t 1 
 .    
3t  1 t  1 3t  1 2  t  1 3t  1 
 VT  2t  0.

+) Thay ẩn x được x2  2  x  (;  2]  [ 2; )  T  (;  2]  [ 2; ).

x 2  x  2 3 2x  1
Câu 17. Giải bất phương trình x  1  trên tập hợp số thực.
3
2x  1  3

- ĐK: x  1, x  13

- Khi đó: x 1 
x2  x  2 3 2x  1
 x  1  2 
x2  x  6
 1 
 x  2 x  1  2,  *

3
2x 1  3 3
2x 1  3 3
2x 1  3

- Nếu 3 2 x  1  3  0  x  13 (1) thì (*)   2 x  1  3 2 x  1   x  1 x  1  x  1

Do hàm f (t )  t 3  t là hàm đồng biến trên , mà (*):


NGUYỄN VĂN LỰC  0933.168.309 SP Toán K35 - ĐH Cần Thơ
PT – BPT – HPT FB: http://www.facebook.com/VanLuc168

f  3
2x 1  f  
x  1  3 2 x  1  x  1  x3  x 2  x  0

 1 5   1 5 
Suy ra: x   ;   0;  
DK(1)
 VN
 2   2 

- Nếu 3 2 x  1  3  0  1  x  13 (2) thì (*)   2 x  1  3 2 x  1   x  1 x  1  x  1

Do hàm f (t )  t 3  t là hàm đồng biến trên , mà (*):

 1
 1  x   2

f  3
 
2x 1  f 
x  1  3 2 x  1  x  1    1  x  13
 2
Suy ra:

  2 x  1   x  1
2 3

1  5  DK(2) 1  5  1  5 
x   1;0   ;     x   1;0   ;13  . KL: x   1;0   ;13 
 2   2   2 

Câu 18. Giải bất phương trình sau trên tập :


5x  13  57  10x  3x 2
 x 2  2x  9
x  3  19  3x

 19
 3  x 
Điều kiện  3
x  4

Bất phương trình tương đương


 x  3  19  3x 2 x  3  19  3x  x 2
 2x  9
x  3  19  3x
 2 x  3  19  3x  x 2  2x  9
 x  5  13  x 
 2 x  3     19  3x    x x 2
2

 3   3 



2 x 2  x  2  
 x2  x  2
x 2  x  2
 x 5   13  x 
9 x  3   9  19  3x  
 3   3 
 
 
 x x 2
2
 
 
2

x  5


1 0
13  x  
*
9  x  3   9  19  3x  
  3   3  

NGUYỄN VĂN LỰC  0933.168.309 SP Toán K35 - ĐH Cần Thơ


PT – BPT – HPT FB: http://www.facebook.com/VanLuc168

 19 


2

1
0 với mọi x   3;
3

\ 4
x  5  13  x  
9 x  3   9  19  3x  
 3   3 
Do đó *  x 2  x  2  0  2  x  1 (thoả mãn)
Vậy tập nghiệm của bất phương trình là S   2;1 .

Câu 19. Giải bất phương trình sau trên tập R


1 1 x 1
x  1 
x x x

Gọi bất phương trình đã cho là (1).


+ ĐK: x  [-1; 0)  [1; +  )
Lúc đó:VP của (1) không âm nên (1) chỉ có nghiệm khi:
1 1 1 1
x  1   x   1   x  1 . Vậy (1) chỉ có nghiệm trên (1; +  ).
x x x x
x 1 x 1
Trên (1; +  ): (1) <=> x 1 1   x 1  1.
x x
x 1 x2  1
Do x  1    0 khi x > 1 nên:
x x
x 1 x2 1 1 x2 1
(1)  x 1 2 1 x   2 1  0
x x x x
x2  1 x2 1 x2 1 1 5
 2 1  0  (  1)2  0  x  .
x x x 2
x  1

Vậy nghiệm bất phương trình là:  1  5
x 
 2

2(x 2  16) 7 x
Câu 20. Giải bất phương trình:  x 3  .
x 3 x 3

ĐK: x  4
  x 2  16  0
 
Bất phương trình  10  2 x  0
 2( x  16)  x  3  7  x  2( x  16)  10  2 x 
2 2
 10  2 x  0
 2
 2( x  16)  (10  2 x)
2

 x5
  x  10  34
10  34  x  5
NGUYỄN VĂN LỰC  0933.168.309 SP Toán K35 - ĐH Cần Thơ
PT – BPT – HPT FB: http://www.facebook.com/VanLuc168

2
VT(*) < 0 (do x  ) nên (*) vô nghiệm.
3

Câu 21. Giải bất phương trình: 2x 3  (1  2x  3x 2 ). 2x  1 .

y  0
Đặt y  2 x  1   , ta được bất phương trình
 y  2x  1
2

2 x3   y 2  3x 2  . y  2 x3  3x 2 y  y 3  0 (2)
1 1
*TH1: Xét y = 0 khi đó  x   thay vào bất phương trình thỏa mãn  x   là
2 2
nghiệm
3 2
 x  x
*TH2: Xét y > 0 khi đó Bất phương trình (2)  2    3    1  0
 y  y
2
 x  x  x 1
  2  1  1  0    y  2 x
 y  y  y 2
 x  0  1
  2  x  0
2x  1  0
suy ra 2x  1  2x    .
 x  0  1 5
 0  x  4
 2 x  1  4 x 2
 1 1 5 
Vậy tập nghiệm của bất phương trình là S =   ; .
 2 2 

NGUYỄN VĂN LỰC  0933.168.309 SP Toán K35 - ĐH Cần Thơ


PT – BPT – HPT FB: http://www.facebook.com/VanLuc168

IV. HỆ PHƯƠNG TRÌNH


Chuyên đề: PT – BPT - HPT

§ 1. PHÂN TÍCH ĐA THỨC HAI BIẾN THÀNH NHÂN TỬ


BẰNG PHƯƠNG PHÁP TAM THỨC BẬC HAI

Kiến thức cơ bản


 Nếu tam thức bậc hai f(x) ax2 bx c (a 0) có hai nghiệm x1, x2 thì tam thức
luôn có thể phân tích thành

f(x) ax2 bx c a(x x1 )(x x2 )

VD: f ( x ; y ) x 2 4 y 2 8 x 4 y 15

♥ Xem (1) là một tam thức bậc hai theo x : f ( x; y ) x 2 8 x 4 y 2 4 y 15


Ta có: ' 16 ( 4 y 2 4 y 15) 4 y 2 4 y 1 (2 y 1) 2 0
x 5 2y
Suy ra (1) có hai nghiệm:
x 3 2y
♥ Vậy: f ( x; y ) ( x 5 2 y )( x 3 2 y) 

Thực hành kỹ năng giải toán


Bài 1: Phân tích các đa thức sau thành nhân tử

1) f ( x; y ) y2 5x2 4 xy 16 x 8 y 16
2
2) f ( x; y ) xy 3y x 4y 7
2 2
3) f ( x; y ) 2x xy y 5x y 2
2 2
4) f ( x; y ) 2x y xy y 5x 2

Bài 2: Phân tích các đa thức sau thành nhân tử

1) f ( x; y ) 6 x 2 3xy x y 1

2) f ( x; y ) 2 x 2 xy y2 5x y 2

3) f ( x; y ) ( x y )(2 x y ) 6 x 3 y 4

4) f ( x; y )  ( x  y )( x  4 y 2  y )  3 y 4
NGUYỄN VĂN LỰC  0933.168.309 SP Toán K35 - ĐH Cần Thơ
PT – BPT – HPT FB: http://www.facebook.com/VanLuc168

§ 2. CÁC PHƯƠNG PHÁP GIẢI HỆ PHƯƠNG TRÌNH


Yêu cầu:
Học sinh đã thành thạo việc giải các hệ cơ bản: bậc nhất hai ẩn, đối xứng loại 1, đối xứng
loại 2, đẳng cấp. Các phương trình một ẩn: bậc nhất, bậc hai, bậc ba, các bậc bốn đặc
biệt,...Thành thạo các phép biến đổi tương đương một phương trình: chuyển vế, nhân chia
hai vế, thay thế biểu thức, bình phương hai vế,...
Chú ý:
Các bài toán giải hệ 2 ẩn đa phần đều quy về việc tìm một pt một ẩn giải được.

1. PHƯƠNG PHÁP THẾ

Kỹ thuật 1: Rút một biến để thế


Cụ thể: Rút một ẩn từ phương trình nầy, thay vào phương trình kia để được phương trình
một ẩn giải được.

Kỹ thuật 2: Rút một biểu thức để thế


Cụ thể: Rút một biểu thức từ phương trình nầy, thay vào phương trình kia để được
phương trình một ẩn giải được.

Kỹ thuật 3: Thế hằng số bởi biểu thức

2. PHƯƠNG PHÁP CỘNG

Có thể: Cộng vế với vế, trừ vế với vế hoặc nhân cho một hằng số thích hợp rồi cộng hoặc
trừ vế với vế mục đích để tạo ra một phương trình mới có thể hỗ trợ cho việc giải hệ đã
cho như: pt một ẩn, pt bậc nhất hai ẩn, phương trình tích số,...

Kỹ thuật 1: Tạo ra pt một ẩn

Kỹ thuật 2: Tạo ra pt bậc nhất hai ẩn

Kỹ thuật 3: Nhân hệ số thích hợp và cộng hoặc trừ vế với vế để tạo ra pt bậc nhất hai ẩn
Chú ý: Các hằng đẳng thức cơ bản sau
  a  b   a 2  2ab  b 2
2

  a  b   a3  3a 2b  3ab 2  b3
3

  a  b   a3  3a 2b  3ab 2  b3
3

3. PHƯƠNG PHÁP ĐẶT ẨN PHỤC

Kỹ thuật: Biến đổi mỗi hệ sao cho có hai biểu thức giống nhau
Chú ý: Các phép biến đổi tương đương một phương trình: chuyển vế, nhân chia hai vế,
thay thế biểu thức,...
NGUYỄN VĂN LỰC  0933.168.309 SP Toán K35 - ĐH Cần Thơ
PT – BPT – HPT FB: http://www.facebook.com/VanLuc168

4. PHƯƠNG PHÁP BIẾN ĐỔI VỀ TÍCH SỐ

Chú ý: Các phép biến đổi: tạo các biểu thức có nhân tử giống nhau, phân tích tam thức
bậc hai thành thừa số, bình phương,...

Kỹ thuật 1: Biến đổi một pt của hệ thành tích số

Kỹ thuật 2: Cộng hoặc trừ vế với vế để biến đổi về pt tích số

5. PHƯƠNG PHÁP HÀM SỐ

Kỹ thuật 1: Sử dụng tính đơn điệu kết hợp nhẩm nghiệm

Kỹ thuật 2: Tìm hàm đặc trưng và sử dụng tính chất f(u) = f(v)

6. KẾT HỢP CÁC PHƯƠNG PHÁP

NGUYỄN VĂN LỰC  0933.168.309 SP Toán K35 - ĐH Cần Thơ


PT – BPT – HPT FB: http://www.facebook.com/VanLuc168

I. PHƯƠNG PHÁP TÍCH SỐ


(PHÂN TÍCH THÀNH NHÂN TỬ)

PHƯƠNG PHÁP GIẢI

Bước 1: Tìm điều kiện cho các biến x, y của hệ phương trình (nếu có)

Bước 2: Biến đổi một phương trình của hệ về dạng phương trình tích số để được
các hệ thức đơn giản chứa x,y

Các kỹ thuật thường sử dụng:

+ Nhóm nhân tử chung

+ Phân tích tam thức bậc hai thành nhân tử

+ Nhẩm nghiệm + nhân liên hợp

Bước 3: Thay hệ thức đơn giản tìm được vào phương trình còn lại của hệ để được
phương trình 1 ẩn

Bước 4: Giải phương trình 1 ẩn (cần ôn tập tốt các phương pháp giải phương
trình 1 ẩn).

2x2 y2 3 xy 3x 2y 1 0 (1)
Ví dụ 1. Giải hệ phương trình
4x2 y2 x 4 2x y x 4y (2)

(Khối B – 2013)

Bài giải
2x y 0
♥ Điều kiện: (*)
x 4y 0

♥ Biến đổi phương trình (1) về dạng tích số. Xem (1) là một phương trình bậc hai theo
ẩn y, phân tích tam thức bậc hai thành nhân tử. Ta được
y x 1
(1) y2 (3x 2) y 2x2 3x 1 0 y ( x 1) y (2 x 1) 0
y 2x 1

NGUYỄN VĂN LỰC  0933.168.309 SP Toán K35 - ĐH Cần Thơ


PT – BPT – HPT FB: http://www.facebook.com/VanLuc168

(các hệ thức đơn giản chứa x, y)


♥ Thế y x 1, thay vào (2) ta được phương trình một ẩn:
3x 2 x 3 3x 1 5x 4 (3)
♣ Do phương trình (3) có hai nghiệm x 0 và x 1 nên ta định hướng phân tích (3)
thành dạng ( x 2 x). f ( x) 0 (biến đổi về phương trình tích số bằng kỹ thuật nhân liên hợp).
3 3( x2 x) (x 1 3x 1) (x 2 5x 4) 0 (Tách thành các biểu thức liên hợp)

x2 x x2 x
3( x 2 x) 0 (Nhân liên hợp)
x 1 3x 1 x 2 5x 4

2 1
( x2 x) 3 0
x 1 3x 1 x 2 5x 5
0

x 0
x2 x 0
x 1

♠ Với x 0 y 1 [thỏa (*)]


♠ Với x 1 y 2 [thỏa (*)]
♥ Thế y 2 x 1, thay vào (2) ta được phương trình một ẩn:
3 3x 4x 1 9x 4 (4)
♣ Do phương trình (4) có hai nghiệm x 0 nên ta định hướng phân tích (4) thành
dạng x. f ( x) 0 (biến đổi về phương trình tích số bằng kỹ thuật nhân liên hợp).
4 3x ( 4 x 1 1) ( 9x 4 2) 0 (Tách thành các biểu thức liên hợp)
4x 9x
3x 0 (Nhân liên hợp)
4x 1 1 9x 4 2

4 9
x 3 0
4x 1 1 9x 4 2
0

x 0 [thỏa (*)]
♠ Với x 0 y 1 [thỏa (*)]
♥ Vậy hệ phương trình có hai nghiệm ( x; y ) là (0;1) và (1; 2) 
(Xem lại phần kỹ thuật nhân liên hợp)

NGUYỄN VĂN LỰC  0933.168.309 SP Toán K35 - ĐH Cần Thơ


PT – BPT – HPT FB: http://www.facebook.com/VanLuc168

BÀI TẬP TƯƠNG TỰ


2x2 x y 2 2x 2y 3 4x 2y 6
Giải hệ phương trình
x2 y2 x 3y 2

Ví dụ 2. Giải hệ phương trình


(1 y) x y x 2 (x y 1) y (1)
2 y2 3x 6y 1 2 x 2y 4x 5 y 3 (2)

(Khối B – 2014)

Bài giải
y 0
♥ Điều kiện : x 2 y (*)
4x 5y 3

♥ Biến đổi phương trình (1) về dạng tích số. Do y 1 luôn thỏa (1) nên định hướng phân
tích theo nhân tử y 1 hoặc 1 y . Ta được:
1 (1 y)( x y 1) (x y 1)(1 y) 0

1 1 y 1
(1 y )( x y 1) 0
x y 1 1 y y x 1
0

♥ Thế y 1 , thay vào (2) ta được phương trình một ẩn: 9 3x 0 x 3 [thỏa (*)]
♥ Thế y x 1, thay vào (2) ta được phương trình một ẩn: 2 x 2 x 3 2 x (3)
Điều kiện: 1 x 2
3 3
2 x2 x 3 0 x 1 x x 1 x
Khi đó: (3) 2 2
(2 x 2 x 3)2 2 x 4 x4 4 x3 11x 2 7x 7 0 ( x2 x 1)(4 x 2 7) 0

3
x 1 x
2 1 5
x
1 5 2 1 5
x . So với điều kiện (*) ta chỉ nhận x
2 7 2
x
7 2
x
2

1 5 1 5
♠ Với x y [thỏa (*)]
2 2
NGUYỄN VĂN LỰC  0933.168.309 SP Toán K35 - ĐH Cần Thơ
PT – BPT – HPT FB: http://www.facebook.com/VanLuc168

1 5 1 5
♥ Vậy hệ phương trình có hai nghiệm ( x; y ) là (3;1) và ; 
2 2

(Xem lại phần giải phương trình bậc bốn)


Ví dụ 3. Giải hệ phương trình
( y 1) 2 ( 3x 2)3 1 y 3x 2 3 xy (1)
3 2
x 3x 12 x (3 x 1) y 6 0 (2)

(Thi thử của Sở Giáo Dục Bắc Ninh)

Bài giải
2
♥ Điều kiện : x (*)
3

♥ Biến đổi phương trình (1) về dạng tích số.


(1) y2 2 y 1 (3x 2) 3x 2 1 y 3x 2 3xy

y( y 3x 2) ( y 3x 2) 3x 2 0

y 3x 1
(y 3x 2)( y 3x 2) 0
y 3x 2

♥ Thế y 3x 2, thay vào (2) ta được phương trình một ẩn:


x3 12 x 2 15 x 4 0

x 1
2
( x 1)( x 11x 4) 0 11 105
x
2

11 105
So với điều kiện (*) ta chọn x 1; x
2

♠ Với x 1 y 1

11 105 29 3 105
♠ Với x y
2 2

♥ Thế y 3x 2 , thay vào (2) ta được phương trình một ẩn:

x3 3x 2 12 x (3x 1) 3x 2 6 0

( 3x 2) 3x 2 3( 3x 2) 3 3x 1 1 x3 3x 2 3x 1

( 3x 2 1)3 ( x 1)3
NGUYỄN VĂN LỰC  0933.168.309 SP Toán K35 - ĐH Cần Thơ
PT – BPT – HPT FB: http://www.facebook.com/VanLuc168

x 0 x 1
3x 2 1 x 1 2
x 3x 2 0 x 2

♠ Với x 1 y 1

♠ Với x 2 y 2

111 105 11 3 105


♥ Vậy hệ phương trình có ba nghiệm ( x; y ) là ( 1;1) , ( 2; 2) , ; 
2 2

x y 2 x 2 y2 2 (1)
Ví dụ 4. Giải hệ phương trình
2( x 2 4 y) 8 y xy 2y 34 15 x (2)

(Thi thử của THPT Chuyên Lê Hồng Phong TP HCM)


Bài giải
♥ Điề u kiê ̣n: –2 ≤ x ≤ 2 và y ≥ 0
♥ Biến đổi phương trình (1) về dạng tích số. Xem (1) là một phương trình bậc hai theo
ẩn 2 x , phân tích tam thức bậc hai thành nhân tử. Ta được:
 2x  y
(1)  (2  x)  2  x.y  2y2  0  
 2  x  2y

♥ Thế y 2 x , thay vào (2) ta được phương trình một ẩn:

2( x  2  4 2  x )  8 4  x2  34  15x (3)
♦ Đă ̣t t = x  2  4 2  x  t 2  34  15x  8 4  x2 .
t  0
Do đó: (3)  2t = t2  
t  2

 x2 4 2x  0 4 2  x  x  2
Suy ra:   
 x  2  4 2  x  2  4 2  x  2  x  2

 30
16(2  x)  x  2 17x  30  x
    17 .
16(2  x)  4  16 2  x  x  2 16 2  x  17(x  2) 
x  2
30 2 17
Khi x = y= và khi x = 2  y = 0.
17 17

♥ Với 2  x  2y ≤ 0 mà y ≥ 0  y = 0 và x = 2. Thử la ̣i ta có x = 2, y = 0 là nghiê ̣m.

 30 2 17 
♥ Vậy hệ phương trình có hai nghiệm ( x; y ) là (2; 0),  ;  
 17 17 

NGUYỄN VĂN LỰC  0933.168.309 SP Toán K35 - ĐH Cần Thơ


PT – BPT – HPT FB: http://www.facebook.com/VanLuc168

x2 2 ( y2 y 1) x 2 2 y3 y 0 (1)
Ví dụ 5. Giải hệ phương trình
2x xy 2 (x 2) y 2 4x 4 0 (2)

Bài giải
♥ Điều kiện: y 2 4 x 4 0
♥ Biến đổi phương trình (1) về dạng tích số. Xem (1) là một phương trình bậc hai theo
ẩn x 2 2 , phân tích tam thức bậc hai thành nhân tử. Ta được:
1 x2 2 ( y2 y 1) x 2 2 y3 y 0
( x2 2 y )( x 2 2 y2 1) 0

y x2 2 (vì x2 2 y2 1 0)

♥ Thế y x2 2 , thay vào (2) ta được phương trình một ẩn:

2x x x2 2 2 (x 2) x 2 4x 6 0

x x x2 2 x 2 (x 2) ( x 2)2 2 0

(x 2) (x 2) ( x 2)2 2 ( x) ( x) ( x) 2 2 (3)

♦ Xét hàm đặc trưng f (t ) t t t 2 2 với t


t2
Ta có: f '(t ) 1 t2 2 0, t f t  đồng biến trên .
t2 2

Nên: 3 f (x 2) f ( x) x 2 x x 1

♦ Với x 5 y 3 (thỏa điều kiện (*))

♥ Vậy hệ phương trình có nghiệm ( x; y ) là (5; 3 ) 


BÀI TẬP
Giải các hệ phương trình
y2 (5 x 4)(4 x) xy x 2 0
1) 2) 3 2
y 2
5x 2
4 xy 16 x 8 y 16 0 2x x y x2 y2 2 xy y 0

x2 3x y2 y 2 x3 18 x y 1( y 19) 0
3) 4)
(x y) x2 4x 5 (2 x) ( x y)2 1 x3 2 x2 7y xy 12

(x y)2 x3 2 y2 x2 y 2 xy
2x 1 2y 1
5) 2 6)
(x y )( x 2 y) 3x 2y 4 2 x2 2y 1 3
y 3 14 x 2

NGUYỄN VĂN LỰC  0933.168.309 SP Toán K35 - ĐH Cần Thơ


PT – BPT – HPT FB: http://www.facebook.com/VanLuc168

II. PHƯƠNG PHÁP HÀM SỐ


PHƯƠNG PHÁP GIẢI
Bước 1: Tìm điều kiện cho các biến x, y của hệ phương trình (nếu có)

Bước 2: Tìm một hệ thức liên hệ đơn giản của x và y bằng phương pháp hàm số
+ Biến đổi một phương trình của hệ về dạng f(u) = f(v) (u, v là các biểu thức
chứa x,y)
+ Xét hàm đặc trưng f(t), chứng minh f(t) đơn điệu, suy ra: u = v (đây là hệ thức
đơn giản chứa x, y)

Bước 3: Thay hệ thức đơn giản tìm được vào phương trình còn lại của hệ để được phương
trình 1 ẩn

Bước 4: Giải phương trình 1 ẩn (cần ôn tập tốt các phương pháp giải phương trình 1 ẩn).


Ví dụ 1. Giải hệ phương trình  x  y  1  y  1  x  0 (1)
 x  1 y  2 (2)

Bài giải

 Điều kiện 00  xy  11
 Khi đó: 1  x  1  x  y  1  y (a)
 Xét hàm đặc trưng: f  t   t  1  t với t  0;1
1 1
Ta có: f '  t     0 t   0;1 và f liên tục trên đoạn  0;1
2 t 2 1 t
Suy ra: f  t  đồng biến trên đoạn  0;1
 Do đó: a   f  x   f  y  x  y
 Thay x  y vào phương trình (2) ta được phương trình:
1
x  1  x  2  x  1  x  2 x(1  x)  2  2 x(1  x)  1  x 
2
 Vậy nghiệm của hệ phương trình là (x; y)   ;  .
1 1
2 2

8 x 3  y 3  6 y 2  6 x  9 y  2  0 (1)
Ví dụ 2. Giải hệ phương trình  2
4 x  1  4 x  3 ( y  1)(3  y )  1  0
2
(2)

Bài giải

NGUYỄN VĂN LỰC  0933.168.309 SP Toán K35 - ĐH Cần Thơ


PT – BPT – HPT FB: http://www.facebook.com/VanLuc168

1 1
 Điều kiện   x  , 1  y  3
2 2
 Khi đó: (1)  8 x  6 x  y 3  6 y 2  9 y  2  (2 x)3  3(2 x)  ( y  2)3  3( y  2)
3
(a)
1 1
 Do   x  nên 1  2 x  1 và 1  y  3 nên 1  y  2  1 .
2 2
 Xét hàm đặc trưng f (t )  t 3  3t , với t   1;1 .
Ta có f '(t )  3t 2  3  3(t 2  1)  0 , với mọi t   1;1 .
Suy ra f  t  nghịch biến trên đoạn  1;1 .
 Do đó:  a   f (2 x)  f ( y  2)  2 x  y  2  y  2 x  2 .
 Thay y  2x  2 vào phương trình (2) ta được phương trình:
2 3 3
4 x 2  2 1  4 x 2  1  0  4 x 2  1  2 1  4 x 2  16 x 4  24 x 2  3  0  x  
2
 Vậy nghiệm của hệ phương trình là
 2 3 3   2 3 3 
( x; y )   ; 2  2 3  3   ( x; y )    ; 2  2 3 3.
 2   2 
   
 x 3  x 2 y  x 2  x  y  1 (1)
Ví dụ 3. Giải hệ phương trình  3
 x  9 y  6( x  3 y )  15  3 6 x  2
2 3 2
(2)

Bài giải
 Ta có: 1  x  x y  x  x  y  1  x 2 ( x  y )  ( x  y )  x 2  1  ( x  y )( x 2  1)  x 2  1
3 2 2

 x  y  1  0 (vì x 2  1  0, x )
 Thay y  x  1 vào phương trình (2) ta được phương trình
x3  9 x2  6 x  6  3 3 6 x2  2  ( x  1)3  3( x  1)  (6 x2  2)  3 3 6 x2  2 (a)
 Xét hàm đặc trưng f (t )  t  3t , với t  .
3

Ta có f '(t )  3t 2  3  0 , với mọi t  . Suy ra f  t  đồng biến trên .


 Do đó: (a)  f ( x  1)  f ( 3 6 x 2  2)  x  1  3 6 x 2  2  x3  9 x 2  3x  3  0 .
3
2 1
 ( x  1)3  2( x  1) 2  x  1  3 2( x  1)  x  3
2 1
3
2 1 2
 Với x  y 3
3
2 1 2 1
 3 2 1 
 Vậy nghiệm của hệ phương trình là ( x; y )   3 ;3
2
 .
 2 1 2 1 
(6 x  5) 2 x  1  2 y  3 y 3  0
Ví dụ 4. Giải hệ phương trình 
(1)
 y  x  2 x  4 x  23
2
(2)

Bài giải

NGUYỄN VĂN LỰC  0933.168.309 SP Toán K35 - ĐH Cần Thơ


PT – BPT – HPT FB: http://www.facebook.com/VanLuc168

2 x  1  0
 2  5 2
 Điều kiện x  0  x
2 x 2  4 x  23  0 2

 Khi đó: (1)  2 x  1  2  3(2 x  1)  y(2  3 y 2 ) (a)
 Xét hàm đặc trưng f (t )  t (2  3t 2 )  3t 3  2t , với t   0;   .
Ta có f '(t )  9t 2  2  0 , với mọi t   0;   .
Suy ra f  t  đồng biến trên  0;   .
 Do đó: (a)  f ( 2 x  1)  f ( y)  2 x  1  y .
 Thay y  2x  1 vào phương trình (2) ta được phương trình:
2 x  1  x  2 x 2  4 x  23  3 x  1  2 2 x 2  x  2 x 2  4 x  23
 2 x 2  x  2 2 x 2  x  24  0
 2x2  x  6 x  4
  2 x  x  36  0  
2
 x4
 2 x  x  4
2 x   9
 2
 Với x  4  y  3
 Vậy nghiệm của hệ phương trình là ( x; y )  (4;3) .

(4 x 2  1) x  ( y  3) 5  2 y  0
Ví dụ 5. Giải hệ phương trình  2 (1)
4 x  y  2 3  4 x  7
2

Bài giải
3 5
 Điều kiện x  , y 
4 2
 Khi đó: (1)  (4 x  1).2 x  (5  2 y  1) 5  2 y
2
(a)
 Xét hàm đặc trưng f (t )  (t 2  1)t  t 3  t , với t  .
Ta có f '(t )  3t 2  1  0 , với mọi t  .
Suy ra f  t  đồng biến trên .
x  0

 Do đó: (a)  f (2 x)  f ( 5  2 y )  2 x  5  2 y   5  4 x2 .
 y 
 2

5  4x 2
 Thay y  vào phương trình (2) ta được phương trình:
2
2
5 
4 x2    2 x2   2 3  4 x  7  0 (b)
2 
3
 Nhận thấy x  0 và x  không là nghiệm của phương trình (b)
4

NGUYỄN VĂN LỰC  0933.168.309 SP Toán K35 - ĐH Cần Thơ


PT – BPT – HPT FB: http://www.facebook.com/VanLuc168
2

Xét hàm số g ( x)  4 x    2 x 2   2 3  4 x  7 với x   0;  , khi đó:


5 3
 2

2   4
 b   g  x   g  
1
(3)
2
Khảo sát tính đơn điệu của hàm số g trên khoảng  0; 
3

 4
Ta có: g '( x)  8 x  8 x   2 x 2      3
5 4 4
 4 x 4 x2  3   0 x   0; 
2  3  4x 3  4x  4
Do đó f đồng biến trên khoảng  0; 
3
 4
1
 Suy ra:  3  x 
2

1
 Với x   y  2
2
 Vậy nghiệm của hệ phương trình là  x; y    ; 2  .
1
2  

  
 x x2  y 2  y 4 y 2  1
  (1)
Ví dụ 6. Giải hệ phương trình 
 4 x  5  y 2  8  6 (2)

Bài giải
5
 Điều kiện x  
4
 Nhận thấy y  0 không thỏa mãn hệ
3
x x
 Khi đó: (1)      y 3  y (a)
 y y
 Xét hàm đặc trưng f (t )  t 3  t ,
với t  .
Ta có f '(t )  3t  1  0 , với mọi t  .
2

Suy ra f  t  đồng biến trên .


x x
 Do đó: a  f    f  y    y  x  y2 .
 y y
 Thay x  y 2 vào phương trình (2) ta được phương trình:
4x  5  x  8  6  2  4 x  5 x  8  23  5 x
 5 23
 23  23   x
x  5 x   4 5
  5   x 1
 x 1
4  4 x  5  x  8    23  5 x  2  x  42 x  41  0
2 
    x  41
 Với x  1  y  1

NGUYỄN VĂN LỰC  0933.168.309 SP Toán K35 - ĐH Cần Thơ


PT – BPT – HPT FB: http://www.facebook.com/VanLuc168

 Vậy nghiệm của hệ phương trình là  x; y   1;  1   x; y   1;1 .

Bài tập tương tự

 x 5  xy 4  y10 +y 6
Giải hệ phương trình 
 4 x  5  y  8  6
2

2 y3 12 y 2 25 y 18 (2 x 9) x 4 (1)
Ví dụ 7. Giải hệ phương trình
3x 1 3x 2 14 x 8 6 4y y2 (2)
(Thi thử của THPT Nghi Sơn – Thanh Hóa)

Bài giải
1
x
♥ Điều kiện: 3 (*)
6 4 y y2 0
♥ Khai thác phương trình (1) để tìm hệ thức liên hệ đơn giản của x và y
(sử dụng phương pháp hàm số kiểu f (u ) f (v) )
♦ 2 y3 12 y 2 25 y 18 (2 x 9) x 4 2( y 2)3 ( y 2) 2( x 4)3 x (3)
4
[Tại sao ?]
♦ Xét hàm đặc trưng f (t) 2t 3 t trên ta có:
f '(t) 6t 2 1 0, t f (t ) đồng biến trên
y 2 y 2
Nên: 3 f (y 2) f( x 4) y 2 x 4
(y 2)2 x 4 x 4y y2 (4)
♥ Thế (4) vào (2) để được phương trình một ẩn
3x 1 6 x 3x 2 14 x 8 0 (5)
♦ Phương trình (5) có một nghiệm là x 5 nên có thể biến đổi về phương trình tích số
bằng kỹ thuật nhân liên hợp.
5 ( 3x 1 4) ( 6 x 1) 3x2 14 x 5 0 [Tại sao ?]
3( x 5) x 5
( x 5)(3x 1) 0
3x 1 4 6 x 1

3 1
( x 5) 3x 1 0 x 5
3x 1 4 6 x 1
0

♦ Với x 5 y 1 (thỏa điều kiện (*))


♥ Vậy hệ phương trình có nghiệm duy nhất ( x; y ) (5;1) 

NGUYỄN VĂN LỰC  0933.168.309 SP Toán K35 - ĐH Cần Thơ


PT – BPT – HPT FB: http://www.facebook.com/VanLuc168

x3 y3 17 x 32 y 6 x2 9 y2 24 (1)
Ví dụ 8. Giải hệ phương trình
(y 2) x 4 (x 9) 2 y x 9 x2 9y 1 (2)
(Thi thử của THPT Chuyên Vĩnh Phúc)

Bài giải
x 4
♥ Điều kiện: (*)
2y x 9 0
♥ Khai thác phương trình (1) để tìm hệ thức liên hệ đơn giản của x và y
(sử dụng phương pháp hàm số kiểu f (u ) f (v) )
♦ x3 y 3 17 x 32 y 6 x 2 9 y 2 24 x3 6 x 2 17 x 18 y 3 9 y 2 32 y 42 [Tại sao ?]
3 3
(x 2) 5( x 2) (y 2) 5( y 2) (3)
♦ Xét hàm đặc trưng f (t ) t 3 5t trên ta có:
f '(t ) 3t 2
5 0, t f (t ) đồng biến trên
Nên: 3 f x 2 f y 3 x 2 y 3 y x 1 (4)
♥ Thế (4) vào (2) để được phương trình một ẩn
9) x 11 x 2 9 x 10
(x 3) x 4 (x (5)
♦ Phương trình (5) có một nghiệm là x 5 nên có thể biến đổi về phương trình tích số
bằng kỹ thuật nhân liên hợp.
5 (x 3)( x 4 3) (x 9)( x 11 4) x2 2 x 35 [Tại sao ?]
x 5 x 5
(x 3). (x 9). ( x 5)( x 7)
x 4 3 x 11 4
x 3 9 x
( x 5) (x 7) 0
x 4 3 x 11 4

x 5 0
x 3 x 9
(x 7) 0 (6)
x 4 3 x 11 4
♦ Chứng minh (6) vô nghiệm
x 3 x 5 x 9 x 9
6 0 [Tại sao ?]
x 4 3 2 x 11 4 2
1 1 1 1 2
(x 5) (x 9) 0 : phương trình VN
x 4 3 2 x 11 4 2 x 4 3
0 0 0

♦ Với x 5 y 6 (thỏa điều kiện (*))


♥ Vậy hệ phương trình có nghiệm duy nhất ( x; y ) (5;6) 

BÀI TẬP TƯƠNG TỰ


Giải các hệ phương trình
x3 y3 3x 2 6x 3y 4 (53 5 x) 10 x (5 y 48) 9 y 0
1) 2 2
2)
x y 6x y 10 y 5 4x y 2x y 6 x2 2x 66 2x y 11
NGUYỄN VĂN LỰC  0933.168.309 SP Toán K35 - ĐH Cần Thơ
PT – BPT – HPT FB: http://www.facebook.com/VanLuc168

3
(2012 3 x) 4 x (6 y 2009) 3 2 y 0 x x y y 1 0
3) 4)
2 7x 8y 3 14 x 18 y x2 6 x 13 x4 x3 x2 1 x ( y 1)3 1
(17 3 x) 5 x (3 y 14) 4 y 0
5)
2 2x y 5 3 3x 2 y 11 x2 6 x 13

x 3 4
x 2 y4 5 y (1)
Ví dụ 9. Giải hệ phương trình
x2 2 x( y 2) y2 8y 4 0 (2)
(Phạm Trọng Thư GV THPT Chuyên Nguyễn Quang Diêu, Đồng Tháp – THTT số 2)

Bài giải
♥ Điều kiện: x 2 (*)
♥ Khai thác phương trình (1) để tìm hệ thức liên hệ đơn giản của x và y
♦ x 3 4x 2 y4 5 y 4
x 2 ( x 2) 5 y y4 5 (3)
♦ Xét hàm đặc trưng f (t ) t t4 5 trên nữa khoảng 0; .
2t 3
f liên tục trên 0; và f '(t ) 1 0, t 0; f (t ) đồng biến trên 0;
t4 5
Do 4
x 2 0 và 4 y ( x y 2) 2 y 0 nên
3 4
f ( x 2) f ( y) 4
x 2 y x y4 2 (4)
♥ Thế (4) vào (2) để được phương trình một ẩn
y 0
4y ( y4 y)2 y( y 7 2 y4 y 4) 0 7
y 2 y4 y 4 0 (5)
♦ Giải phương trình (5) bằng phương pháp hàm số
Xét hàm số g ( y ) y 7 2 y 4 y 4 trên nữa khoảng 0; .
Do g liên tục trên 0; 6 3
và g'( y) 7 y 8 y 1 0, y 0; g y đồng biến
trên 0;
Nên: 5 g ( y ) g (1) y 1
♣ Với y 0 x 2 [thỏa (*)]
♣ Với y 1 x 3 [thỏa (*)]
♥ Vậy hệ phương trình có hai nghiệm ( x; y ) là (2; 0) và (3;1) 

 3 x 1
Ví dụ 10. Giải hệ phương trình:
3 2
 x  3 x  y  6 y  9 y  2  ln
y 1
0 1 .

 y  log  x  3  log y   x  1 2
  2 3 

(Thi thử của THPT Chuyên Vĩnh Phúc)

Bài giải

NGUYỄN VĂN LỰC  0933.168.309 SP Toán K35 - ĐH Cần Thơ


PT – BPT – HPT FB: http://www.facebook.com/VanLuc168

 x 1
 y 1  0
 x  3
♥ Điều kiện: x  3  0  
y  0 y  0


♥ Khai thác phương trình (1) để tìm hệ thức liên hệ đơn giản của x và y
♦ 1 ( x  1)3  3( x  1) 2  ln( x  1)  ( y  1)3  3( y  1) 2  ln( x  1) (3)
♦ Xét hàm đặc trưng f (t )  t 3  3t 2  ln t trên khoảng (0; )
1
f (t )  3t 2  6t   0 t  0  f (t ) đồng biến trên khoảng  0;  
t
Do x 1 0 và y 1 0 nên
 3  f ( x  1)  f ( y  1)  x  1  y  1  y  x  2 (4)
♥ Thế (4) vào (2) để được phương trình một ẩn
( x  2)  log 2 ( x  3)  log3 ( x  2)   x  1 (5)

♦ Giải phương trình (5) bằng phương pháp hàm số


x 1 x 1
5  log 2 ( x  3)  log 3 ( x  2)   log 2 ( x  3)  log 3 ( x  2)   0 6 
x2 x2
x 1
♣ Xét hàm số g ( x)  log 2 ( x  3)  log 3 ( x  2)  trên khoảng (3; )
x2
1 1 3
g ( x)     0 x  3
( x  3) ln 2 ( x  2) ln 3 ( x  2) 2
 g ( x ) đồng biến trên khoảng (3; ) .
 4
Nên  6   g ( x)  g (5)  x  5  y3
♥ Vậy hệ phương trình có nghiệm duy nhất  x; y    5;3

Ví dụ 11. Giải hệ phương trình:. 




3x 2  3y 2  8   y  x  y 2  xy  x 2  6
 1

 x  y  13 3y  14  x  1  5
  2
(Thi thử của THPT Chuyên Vĩnh Phúc)

Bài giải
 x  1
x 1  0 
♥ Điều kiện:   14  *
3 y  14  0  y  3

♥ Khai thác phương trình (1) để tìm hệ thức liên hệ đơn giản của x và y
♦ 1  x  13  3 x  1   y  13  3 y  1 (3)
♦ Xét hàm đặc trưng f  t   t  3t , t  3

f   t   3t 2  3  0, t   f  t  đồng biến trên .


Do x 1 0 và y 1 0 nên
 3 f  x  1  f  y  1  x  1  y  1  x  2  y (4)

NGUYỄN VĂN LỰC  0933.168.309 SP Toán K35 - ĐH Cần Thơ


PT – BPT – HPT FB: http://www.facebook.com/VanLuc168

♥ Thế (4) vào (2) để được phương trình một ẩn


 2x  11  3x  8  x  1   5 5
♦ Giải phương trình (5) bằng phương pháp hàm số
Ta nhận thấy x  11 không là nghiệm của phương trình  5  nên
2
5
 5  3x  8  x  1   0. 6
2 x  11
Xét hàm số
5  8 11   11 
g  x   3x  8  x  1  , x   ;    ;  
2 x  11 3 2   2 

3 1 10 3 x  1  3x  8 10  8 11   11 
g x       0 x   ;  &  ;  
2 3x  8 2 x 1  2 x  11 2
2  3x  8 x  1  2 x  11 2
 3 2   2 
 8 11   11 
 g  x  đồng biến trên các khoảng  ;  &  ;  
3 2   2 
 8 11   8 11 
♣ Trên khoảng  3 ; 2  thì g  x  đồng biến, 3   ;  , g  3  0 nên
  3 2 
 4
6 g  x   g  3  x  3  y  5 thoả mãn (*)
 11   11 
♣ Trên khoảng  ;   thì g  x  đồng 8   ;   , g 8   0 nên
biến,
 2   2 
 4
6 g  x   g  8   x  8  y  10 thoả mãn (*)
Vậy hệ phương trình có hai nghiệm  x, y    3;5 ,  x, y   8;10  

NGUYỄN VĂN LỰC  0933.168.309 SP Toán K35 - ĐH Cần Thơ


PT – BPT – HPT FB: http://www.facebook.com/VanLuc168

BÀI TẬP TƯƠNG TỰ


Giải các hệ phương trình
(4 x 2 1) x (y 3) 5 2 y 0
1)
4x2 y2 2 3 4x 7
x3 y3 3y2 4y x 2
2)
x y 3 x 3 y 19 105 y3 xy
4x 2 2y 4 6
3)
2(2 x 1)3 2x 1 (2 y 3) y 2

2 y3 y 2x 1 x 3 1 x (1)
Ví dụ 12. Giải hệ phương trình
9 4 y2 2 x2 6 y2 7 (2)
(Thi thử của THPT Trần Phú – Thanh Hóa)

Bài giải
x 1
♥ Điều kiện: 3 3 (*)
y
2 2
♥ Khai thác phương trình (1) để tìm hệ thức liên hệ đơn giản của x và y
(sử dụng phương pháp hàm số kiểu f (u ) f (v) )
♦ 2 y3 y 2 x 1 x 3 1 x 2 y3 y 2 1 x 2 x 1 x 1 x
2 y3 y 2(1 x) 1 x 1 x (3)
♦ Xét hàm đặc trưng f (t ) 2t 3
t trên ta có:
f '(t ) 6t 2
1 0, t f đồng biến trên
y 0
Nên: 3 f ( y) f ( 1 x) y 1 x 2
(4)
y 1 x
♥ Thế (4) vào (2) để được phương trình một ẩn

4x 5 2x2 6x 1 (5)

♦ Giải phương trình (5) bằng phương pháp đặt ẩn phụ chuyển về hệ đối xứng loại II
Phương trình (5) viết lại thành: (2 x 3)2 2 4 x 5 11
3
Điều kiện Đặt 4 x 5 2t 3 t , ta được hệ phương trình:
2
[Tại sao ?]
(2 x 3)2 4t 5 (6)
(2t 3)2 4x 5 (7)
Trừ theo từng vế của (6) và (7) ta được:
4( x t 3)( x t ) 4t 4x ( x t )( x t 2) 0
+ Khi x t , thay vào (7) ta được:
NGUYỄN VĂN LỰC  0933.168.309 SP Toán K35 - ĐH Cần Thơ
PT – BPT – HPT FB: http://www.facebook.com/VanLuc168

4x2 12 x 9 4x 5 x2 4x 1 0 x 2 3
So với điều kiện của x và t ta chọn x 2 3 . [không thỏa (*)]
+ Khi x t 2 0 t 2 x , thay vào (7) ta được:
(1 2 x)2 4 x 5 x 2 2 x 1 0 x 1 2 (loại)
So với điều kiện của x và t ta chọn x 1 2 .
♦ Với x 1 2 y 4
2 . [thỏa (*)]
♥ Vậy hệ phương trình có hai nghiệm ( x; y ) là (1 2; 4 2) và (1 2; 4 2) 

NGUYỄN VĂN LỰC  0933.168.309 SP Toán K35 - ĐH Cần Thơ


PT – BPT – HPT FB: http://www.facebook.com/VanLuc168

III. PHƯƠNG PHÁP ĐÁNH GIÁ


PHƯƠNG PHÁP GIẢI
Bước 1: Tìm điều kiện cho các biến x, y của hệ phương trình (nếu có)
Bước 2: Tìm một hệ thức liên hệ đơn giản của x và y bằng phương pháp đánh giá
Thường là sử dụng các bất đẳng thức cơ bản: Cô-si, bất đẳng thức về giá trị tuyệt đối,...
Bước 3: Thay hệ thức đơn giản tìm được vào phương trình còn lại của hệ để được phương
trình 1 ẩn
Bước 4: Giải phương trình 1 ẩn (cần ôn tập tốt các phương pháp giải phương trình 1 ẩn).

x 12 y y (12 x2 ) 12 (1)
Ví dụ 1. Giải hệ phương trình
x3 8x 1 2 y 2 (2)

(Khối A - 2014)

Bài giải
2 3 x 2 3
♥ Điều kiện: (*)
2 y 12

♥ Đánh giá phương trình (1) để tìm hệ thức đơn giản liên hệ giữa x và y. Sử dụng BĐT
Cô-si ta có:
x2 12 y
x 12 y
2 x 12 y y (12 x2 ) 12 nên
2 y 12 x 2
y 12 x
2
x 0
1
y 12 x2

♥ Thế y 12 x 2 vào phương trình (2) ta được phương trình một ẩn:

x3 8x 1 2 10 x2 (3)
♠ Phương trình (3) có một nghiệm là x 3 nên ta định hướng phân tích (3) thành dạng
( x 3). f ( x) 0

(3) x3 8x 3 2(1 10 x2 ) 0

NGUYỄN VĂN LỰC  0933.168.309 SP Toán K35 - ĐH Cần Thơ


PT – BPT – HPT FB: http://www.facebook.com/VanLuc168

2 2( x 2 9)
( x 3)( x 3 x 1) 0
1 10 x2

2( x 3)
( x 3) x 2 3x 1 0 x 3
1 10 x2
0

♠ Với x 3 y 3 [thỏa (*)]


♥ Vậy hệ phương trình có một nghiệm ( x; y ) là (3;3) 

 5x 2  2xy  2y 2  2x 2  2xy  5y 2  3(x  y) (1)


Ví dụ 2. Giải hệ phương trình  .
 2x  y  1  2 3 7x  12y  8  2xy  y  5 (2)

Bài giải
5 x 2  2 xy  2 y 2  0
 2
♠ Điề u kiê ̣n: 2 x  2 xy  5 y  0  x  2 y  1  0 .
2

x  2 y 1  0

♠ Khi hê ̣ có nghiê ̣m  x; y  


1
x y  0

Ta thấ y 5x2  2 xy  2 y 2  2 x  y * dấ u bằ ng khi x  y . Thâ ̣t vâ ̣y:

*  5 x 2  2 xy  2 y 2  (2 x  y)2  ( x  y)2  0 luôn đúng với mo ̣i x, y 

Tương tự 2 x2  2 xy  5 y 2  x  2 y ** dấ u bằ ng khi x  y

Từ * & **  VT 1  5x2  2 xy  2 y 2  2 x2  2 xy  5 y 2  3( x  y)  VP1

Dấ u đẳ ng thức xẩ y ra khi x  y  3


♠ Thế y = x vào (2), ta được: 3x  1  2. 3 19 x  8  2 x 2  x  5 (3)
Ta có: (3)  3x  1  ( x  1)  2  3 19 x  8  ( x  2)   2 x 2  2 x

 x2  x 2( x3  6 x 2  7 x)
   2 x2  2 x
3x  1  x  1 3
(19 x  8)  ( x  2) 19 x  8  ( x  2)
2 3 2

x2  x 2( x 2  x)( x  7)
   2( x 2  x)  0
3x  1  x  1 3
(19 x  8)  ( x  2) 19 x  8  ( x  2)
2 3 2

NGUYỄN VĂN LỰC  0933.168.309 SP Toán K35 - ĐH Cần Thơ


PT – BPT – HPT FB: http://www.facebook.com/VanLuc168

 x2  x  0

  1 2( x  7)
  2  0 (*)
 3x  1  x  1 3 (19 x  8) 2  ( x  2) 3 19 x  8  ( x  2) 2

Vì x ≥ 0 nên (*) vô nghiê ̣m. Do đó (3)  x = 0 hay x = 1.


♠ Vậy hệ phương trình có nghiệm ( x, y)  (0;0), (1;1) 

8 xy 17 x y 21
(1)
Ví dụ 3. Giải hệ phương trình x 2
y 2 6 xy 8 y x 4
x 16 y 9 7 (2)

Bài giải
x 16
♥ Điều kiện:
y 9

♥ Đánh giá phương trình (1) để tìm hệ thức đơn giản liên hệ giữa x và y.
8 17 x y 3
Ta có: 1 6
x y 8 y x 4
6
y x

x y x y
Đặt t 2 . 2 t 2 x y và sử dụng BĐT Cô-si ta có:
y x y x

8 17 x y 3 8 17 3 8 1
t (t 6) 2t 2 2.2 6
x y 8 y x 4 t 6 8 4 t 6 8
6
y x

8 1
Dấu “=” xảy ra khi (t 6) t 2 x y
t 6 8

♠ Thế y = x vào (2), ta được: x 16 x 9 7

( x 16)( x 9) 37 x

x 25

Với x 25 y 25 [thỏa (*)]


♠ Vậy hệ phương trình có nghiệm ( x, y )  (25; 25) 

NGUYỄN VĂN LỰC  0933.168.309 SP Toán K35 - ĐH Cần Thơ


PT – BPT – HPT FB: http://www.facebook.com/VanLuc168

IV. PHƯƠNG PHÁP ẨN PHỤ


PHƯƠNG PHÁP GIẢI
Bước 1: Tìm điều kiện cho các biến x, y của hệ phương trình (nếu có).
Bước 2: Biến đổi hai phương trình của hệ sao cho có hai biểu thức giống nhau.
Bước 3: Thay hai biểu thức đó bởi hai biến mới u, v, chuyển sang hệ mới và giải tìm u, v.
Bước 4: Với u, v tìm được ta sẽ tìm được x, y.

x2 1 y( x y) 4y (1)
Ví dụ 1. Giải hệ phương trình (*)
( x2 1)( x y 2) y (2)

Bài giải
♥ Biến đổi sao cho hai phương trình của hệ xuất hiện hai biểu thức giống nhau
Do y 0 không thỏa mãn hệ trên nên
x2 1
(x y) 4
y
*
x2 1
(x y 2) 1
y

x2 1
♥ Đặt ẩn phụ u và v x y 2 , hệ trở thành
y

u v 2 u 1
u.v 1 v 1

u 1
♥ Với ta được hệ phương trình
v 1

x2 1
1 x 1 x 2
y
y 2 y 5
x y 1 1

♥ Vậy hệ phương trình có hai nghiệm là ( x; y ) (1; 2);( 2;5) 

NGUYỄN VĂN LỰC  0933.168.309 SP Toán K35 - ĐH Cần Thơ


PT – BPT – HPT FB: http://www.facebook.com/VanLuc168

Bài tập tương tự


Giải các hệ phương trình
x2 xy 3x y 0 y2 x xy 6 y 1 0
1) 2)
x4 3x 2 y 5 x 2 y2 0 y3 x 9 y 2 x2 y x 0

( x2 x) y 2 4 y2 y 1 0 x 2 ( y 1) 6y 2
3) 4)
xy x2 y 2 1 (4 x3 ) y 3 0 x4 y 2 2x2 y 2 y( x 2 1) 12 y 2 1

x2 y2 xy 3x 2 ( x2 y 2 )( x y 1) 25( y 1)
5) 2 4 2 4 4
6) 2 2
(x xy ) (y 2) 17 x x xy 2y x 8y 9

x2 2 y2 3 x y 5
7)
x2 2 y2 3 x y 2

x2 y2 6 0 (1)
Ví dụ 2. Giải hệ phương trình 4 (*)
(x y 1) 2 2
3 (2)
(x y)

Bài giải
♥ Điều kiện: x y 0
♥ Biến đổi hệ phương trình thành dạng có chứa hai biểu thức x y và x y
(x y )( x y) 6
4
(x y 1) 2 3
(x y)2

♥ Đặt u x y và v x y hệ phương trình trở thành


6
uv 6 v 6 u 3; v 2
u v
4 u 3
(u 1) 2 3 u2 u ;v 8
v2 u2 2u 1 3 8u 2 18u 18 0 4
9
5 35
x 3 x
x y 3 2 x y 8
♥ Suy ra: và 4
x y 2 1 29
y x y 8 y
2 8
5 1 35 29
♥ Vậy hệ phương trình có hai nghiệm là ( x; y) ; ; ; 
2 2 8 8

NGUYỄN VĂN LỰC  0933.168.309 SP Toán K35 - ĐH Cần Thơ


PT – BPT – HPT FB: http://www.facebook.com/VanLuc168

BÀI TẬP TỰ LUYỆN

 x  y  x  y  2
Câu 1. Giải hệ phương trình:  (x,y  )
 x  y  1  3  x  y
2 2 2 2

Điều kiện: x+y  0, x-y  0


 u  v  2 (u  v)  u  v  2 uv  4
u  x  y  
Đặt:  ta có hệ:  u 2  v 2  2   u 2  v2  2
v  x  y   uv  3   uv  3
 2  2
 u  v  2 uv  4 (1)

  (u  v) 2  2uv  2 . Thế (1) vào (2) ta có:
  uv  3 (2)
 2
uv  8 uv  9  uv  3  uv  8 uv  9  (3  uv ) 2  uv  0 .
 uv  0
Kết hợp (1) ta có:   u  4, v  0 (vì u>v).
u  v  4
Từ đó ta có: x =2; y =2.(Thỏa đ/k)
Vậy nghiệm của hệ là: x  2, y  2.

 x  3 xy  x  y 2  y  5 y  4
Câu 2. Giải hệ phương trình: 
 4 y  x  2  y  1  x  1
2

 xy  x  y 2  y  0

Đk: 4 y 2  x  2  0
 y 1  0

Ta có (1)  x  y  3  x  y  y  1  4( y  1)  0
Đặt u  x  y , v  y  1 ( u  0, v  0 )
u  v
Khi đó (1) trở thành : u 2  3uv  4v 2  0  
u  4v(vn)

Với u  v ta có x  2 y  1 , thay vào (2) ta được : 4 y 2  2 y  3  y  1  2 y


 4 y 2  2 y  3   2 y  1   
y 1 1  0

NGUYỄN VĂN LỰC  0933.168.309 SP Toán K35 - ĐH Cần Thơ


PT – BPT – HPT FB: http://www.facebook.com/VanLuc168

2  y  2 y2  2 1 
  0   y  2   0
4 y2  2 y  3  2 y 1 y 1 1  4 y  2 y  3  2 y 1
2
y  1  1 

2 1
 y  2( vì    0y  1 )
4 y  2 y  3  2 y 1
2
y 1 1

Với y  2 thì x  5 .
Đối chiếu Đk ta được nghiệm của hệ phương trình là x  5, y  2.

Câu 3. Giải hệ phương trình:


( x  y )( x 2  xy  y 2  3)  3( x 2  y 2 )  2
  x, y   .
4 x  2  16  3 y  x  8
2

16
ĐK: x  2, y 
3
(1)  ( x  1)  ( y  1)3  y  x  2 Thay y=x-2 vào (2) được
3

4( x  2) 3( x  2)
4 x  2  22  3 x  x 2  8   ( x  2)( x  2) 
x22 22  3 x  4
x  2
 4 3
  ( x  2)   0(*)
 x  2  2 22  3 x  4
Xét f(x)=VT(*) trên [-2;21/3],có f’(x)>0 nên hàm số đồng biến. suy ra x=-1 là nghiệm duy
nhất của (*)
KL: Hệ phương trình có 2 nghiệm x ; y    2;0  , x ; y    1; 3 .

Câu 4. Giải hệ phương trình:


x  y  x  y  3  (x  y)2  2 x  y (1)

 2 (x, y  R)
 x  x  y  2  x  y  3 (2)

x  y  0
Điều kiện:  (*)
x  y  0

Đặt t  x  y  0 , từ (1) ta có: t  t  3  t2  2 t


 t  t2  t  3  2 t  0
3(1  t)  3 
 t(1  t)   0  (1  t)  t  0
t3 2 t  t3 2 t

NGUYỄN VĂN LỰC  0933.168.309 SP Toán K35 - ĐH Cần Thơ


PT – BPT – HPT FB: http://www.facebook.com/VanLuc168

3
 t  1 (Vì t   0, t  0 ).
t3 2 t
Suy ra x  y  1  y  1  x (3).

Thay (3) vào (2) ta có: x2  3  2x  1  3


x2  1 2x  2
 ( x  3  2)  ( 2x  1  1)  0 
2
 0
x 3 2
2
2x  1  1
 x 1 2 
 (x  1)     0
 2  
 x  3  2 2x 1 1 
x 1 2 1
 x  1 (Vì   0, x  ).
x2  3  2 2x  1  1 2
Suy ra (x = 1; y = 0), thoả mãn (*).
Vậy hệ đã cho có nghiệm duy nhất x  1, y  0.

 x  y  2  x  y  2(x 2  y 2 )
Câu 5. Giải hệ phương trình:  1 1 1 1
   2 2
x y x y

 x  y  2  x  y  2( x 2  y 2 ) (1)

1 1 1 1
 x  y  x 2  y 2 (2)

 x  y  2
Điều kiện:  .
 xy  0
Ta thấy x + y = 0 không là nghiệm của hpt. Do đó ta có thể xét hai trường hợp sau:
TH1: 2  x  y  0
2
1 1 1 1 1 1
Từ phương trình (2 ) ta suy ra xy < 0. pt (2)          2 .  0 (3) .
x y x y x y
Giả sử hệ phương trình đã cho có nghiệm x, y.
1 1 1 1
Khi đó phương trình (3) có nghiệm   1  8 .  0  xy  8  0  xy  8 .
x y x y
Khi đó ta có x 2  y 2  2 xy  16 .

Đặt t  x  y  2  0  t  2 .
Từ phương trình (1) ta có t  t 2  2  32  t 2  t  34  0 điều này vô lí .
Vậy TH1 hệ phương trình vô nghiệm.
TH2: x + y >0.
NGUYỄN VĂN LỰC  0933.168.309 SP Toán K35 - ĐH Cần Thơ
PT – BPT – HPT FB: http://www.facebook.com/VanLuc168

Từ (2) suy ra xy > 0, do đó x và y đều dương.


Ta có (2)  ( x  y ) xy  x 2  y 2
( x  y)2 ( x  y)2
Do x 2  y 2  và xy  nên ta có
2 4
( x  y)2 ( x  y)2
 x  y  ( x  y ) xy  ( x  y )
2 2
 x y  2
2 4
Đặt t  x  y  2  t  2 .
Từ (1)  t  t 2  2  (t 2  2)2  t 4  5t 2  t  6  0  (t  2)(t 3  2t 2  t  3)  0 (4) .
Ta có t 3  2t 2  t  3  0  t  2 , do đó, từ (4)  t  2  0  t  2.
Từ đó suy ra: t = 2  x  y  2 , thay vào hphương trình ta có xy=1  x  y  1 .
Vậy hệ phương trình có nghiệm duy nhất là x  1, y  1.

 x  x  2  x  4  y 1  y  3  y  5
Câu 6. Giải hệ phương trình:  trên
 x  y  x  y  44
2 2

Xét hàm số f  t   t  t  2  t  4 trên  0;    , có


1 1 1
f  t      0, t   0;   
2 t 2 t2 2 t4
Nên (1)  x  x  2  x  4   y  5  4   y  5  2  y  5  x  y  5 (*)
Thay (*) vào (2): y  3  y  2  1 (3)
Nhân (3) với lượng liên hợp: 5  y  3  y  2 (4)
(3), (4)  y  3  3  y  6 .
ĐS: x  1, y  6.


x  y  1
3 3

Câu 7. Giải hệ phương trình:  2 .



 x y  2 xy  y  2
2 3


x  y  1
3 3

x  y  1
3 3
(1)
Ta có .  2  

 x y  2 xy  y  2
2 3

2 x  y  x y  2 xy  0
3 3 2 2
(2)
x 3  y 3  1 (3)

y  0 . Ta có:   x  3  x  2  x 
2      2   1  0 (4)
  y  y  y
x 1
Đặt :  t (4) có dạng : 2t3 – t2 – 2t + 1 = 0  t =  1 , t = .
y 2
x 3  y 3  1 1
a) Nếu t = 1 ta có hệ  x y3
x  y 2
x 3  y 3  1
b) Nếu t = -1 ta có hệ   hệ vô nghiệm.
x   y

NGUYỄN VĂN LỰC  0933.168.309 SP Toán K35 - ĐH Cần Thơ


PT – BPT – HPT FB: http://www.facebook.com/VanLuc168

1 x 3  y 3  1 3
3 23 3
Nếu t = ta có hệ  x  , y
2 y  2x 3 3

 xy  2 x  5 y  3  x 2  2 y 2
Câu 8. Giải hệ phương trình : 
 x 2 y  2  y x  1  x  1  2 x  2 y  2

 y  1
ĐK : 
x  1
Phương trình đầu của hệ tương đương với  x  y  1 2 y  x  3  0  2 y  x  3  0 (do đk)
Thay vào phương trình thứ hai, được:  2 y  3 2 y  2  y 2 y  2  2 y  2  2 y  4
  y  2  
2 y  2  2  0  2 y  2  2  0  y  1 (thỏa đk )
Hệ phương trình có nghiệm duy nhất: x  5, y  1.

7 x 3  y 3  3 xy ( x  y )  12 x 2  6 x  1
Câu 9. Giải hệ phương trình: 3 ( x, y  )
 4 x  y  1  3 x  2 y  4

Điều kiện: 3x+2y  0


(1)  8 x3  12 x 2  6 x  1  x3  3x 2 y  3xy 2  y 3
 (2 x  1)3  ( x  y)3  2 x  1  x  y  y  1  x
Thế y = 1 x vào (2) ta được: 3 3x  2  x  2  4
Đặt a  3 3x  2, b  x  2 (b  0)
a  b  4
Ta có hệ 
a  3b  4
3 2

b  4  a b  4  a b  4  a
 3     3
a  3(4  a)  4 a  3(16  8a  a )  4 a  3a  24a  44  0
2 3 2 2

b  4  a a  2
  
(a  2)(a  a  22)  0 b  2
2

 3 3 x  2  2
  x  2  y =  1 (thỏa ĐK)
 x  2  2
Kết luận: Nghiê ̣m của hệ phương trình là (x; y) = (2;1).


x y  x  1  2x x y  2
2 2 2
Câu 10. Giải hệ phương trình:  ( x, y  ) .

 y 3
( x 6
 1)  3 y ( x 2
 2)  3 y 2
 4  0

NGUYỄN VĂN LỰC  0933.168.309 SP Toán K35 - ĐH Cần Thơ


PT – BPT – HPT FB: http://www.facebook.com/VanLuc168

Điều kiện: x 2 y  2 . Gọi hai phương trình lần lượt là (1) và (2)
( 2)  x 6 y 3  3x 2 y  y 3  3 y 2  3 y  1  3( y  1)
 ( x 2 y )3  3x 2 y  ( y  1)3  3( y  1) (3)
Xét hàm số f (t )  t 3  3t có f '(t )  3t 2  3  0, t 
Do đó (3)  f ( x 2 y )  f ( y  1)  x 2 y  y  1, ( y  1).
Thế vào (1) ta được x 2 y  x 2  1  2 x y  1
 x 2 ( y  1)  2 x y  1  1  0  ( x y  1  1) 2  0  x y  1  1
Do đó hệ đã cho tương đương với
x 2 y  x2  1  y  2  x 2
 x y  1  1  2 
 2   x y  y  1   x 2 (2  x 2 )  x 2  1 (4)
 x y  y  1 x  0 x  0
 
(4)  x  3x  1  0  ( x  1)  x  0  ( x 2  x  1)( x 2  x  1)  0
4 2 2 2 2

 1 5
x  1 5  1 5
2
 . Do x > 0 nên x  hoặc x 
 1 5 2 2
x 
 2
1 5 1 5 1 5 1 5
Với x  y . Với x  y .
2 2 2 2
1 5 1 5   1  5 1  5 
Vậy hệ đã cho có nghiệm ( x; y)   ;  , (x ; y )  
 ;  .
 2 2   2 2 

 x 3  y 3  3 y 2  x  4 y  2  0
Câu 11. Giải hệ phương trình:  3 ( x, y  ) .
 x  x  3  2 x  2  y

 x  y  3 y  x  4 y  2  0 (1)
3 3 2

 3 Điều kiện: x  2 .
 x  x  3  2 x  2  y (2)

(1)  x3  x  2  y 3  3 y 2  4 y  x3  x  2   y  1   y  1  2 .
3

Xét hàm số f  t   t 3  t  2 trên  2;   .


Ta có: f '  t   3t 2  1  0, t   2;   . Suy ra hàm số f  t  đồng biến trên  2;   .
Do đó: x  y  1 .
Thay y  x  1 và phương trình (2) ta được: x3  3  2 x  2  1
2  x2 2  x2 2 
 x3  8  2   
x  2  2   x  2 x2  2 x  4  
 x22 
2  x  2  

  x  2 x2  2 x  4     x  2  x2  2x  4 
2 0
 x22  
  x2 2 
 
NGUYỄN VĂN LỰC  0933.168.309 SP Toán K35 - ĐH Cần Thơ
PT – BPT – HPT FB: http://www.facebook.com/VanLuc168

 x20 x  2 y 3
2 2
 x2  2 x  4   0  x2  2x  4  (*)
 x22   x2 2 
2
Ta có VT  x 2  2 x  4   x  1  3  3;VP   1, x   2;  
2

x22
Do đó phương trình (*) vô nghiệm.
Vậy hệ phương trình đã cho có nghiệm duy nhất x  2, y  3.

Câu 12. Giải hệ phương trình


 xy  2  y x 2  2
 2 (với x; y  )
 y  2  
x  1 x 2
 2 x  3  2 x 2
 4 x.

ĐKXĐ: x  ; y  .

Ta có xy  2  y x 2  2  y  
x2  2  x  2  y 
2
x 2x
2

 y  x 2  2  x (1). Thế vào phương trình thứ hai trong hệ, ta có :

 
2
x 2  2  x  2  x  1 x 2  2 x  3  2 x 2  4 x

 1  x x 2  2  2 x   x  1 x 2  2 x  3  0 .

  x  1 1   x  1  2     x  1  x  2  (*)
2 2

   

 
Xét hàm số f (t )  t 1  t 2  2 với t  . Ta có f '(t )  1  t 2  2 
t2
t 2
2
 0, t   f (t )

đồng biến trên .


1
Mặt khác, phương trình (*) có dạng f ( x  1)  f ( x)  x  1   x  x   .
2
1
Thay x   vào (1) ta tìm được y  1 .
2
1
Vậy hệ đã cho có nghiệm là x   , y  1.
2

Câu 13. Tìm m để hệ phương trình


 x3  y 3  3 y 2  3x  2  0
 2 có nghiệm thực.
 x  1  x  3 2 y  y  m  0
2 2

NGUYỄN VĂN LỰC  0933.168.309 SP Toán K35 - ĐH Cần Thơ


PT – BPT – HPT FB: http://www.facebook.com/VanLuc168

 x3  y 3  3 y 2  3x  2  0 (1)
 2
 x  1  x  3 2 y  y  m  0
2 2
(2)

1  x  0 1  x  1
2
Điều kiện:   
2 y  y  0 0  y  2
2

Đặt t = x + 1  t[0; 2]; ta có (1)  t3  3t2 = y3  3y2.


Hàm số f(u) = u3  3u2 nghịch biến trên đoạn [0; 2] nên:
(1)  y = t  y = x + 1  (2)  x2  2 1  x2  m  0
Đặt v  1  x2  v[0; 1]  (2)  v2 + 2v  1 = m.
Hàm số g(v) = v2 + 2v  1 đạt min g (v)  1; m ax g (v)  2
[ 0;1] [ 0;1]

Vậy hệ phương trình có nghiệm khi và chỉ khi 1  m  2.

x 4  x 3y  x 2y 2  1
Câu 14. Giải hê ̣ phương trình:  3
x y  x  xy  1
2


(x  xy )  1  x y
2 2 3

*Biến đổi hệ tương đương với  3


x y  (x  xy )  1

2

x 2  xy  u u 2  1  v
*Đặt ẩn phụ  3 , ta được hệ 
x y  v v  u  1
*Giải hệ trên được nghiệm (u;v) là (1;0) và (-2;-3)

*Từ đó giải được nghiệm x ; y   1;0 , x ; y    1;0  .

 x y4
 2 
Giải hệ phương trình:  y  1 x  y
2 2
Câu 15.

 4x  5  y  8  6
2

Hệ phương trình đã cho tương đương

NX: Nếu y=0 thì từ phương trình (1) Thay x=0; y=0 vào phương trình (2) ta
được:
(vô lý). Vậy y=0 không thỏa mãn bài toán
*) chia cả 2 vế của phương trình (1) cho ta được:

NGUYỄN VĂN LỰC  0933.168.309 SP Toán K35 - ĐH Cần Thơ


PT – BPT – HPT FB: http://www.facebook.com/VanLuc168

Xét
Có . Vậy đồng biến trên R.
Từ (*)
Thay vào phương trình (2) ta được
Vậy hphương trình có cặp nghiệm duy nhất x  1, y  1.

Câu 16. Giải hệ phương trình:



 2 x  6 xy  5 y  2 x  2 xy  13 y  2( x  y ) (1)
2 2 2 2


( x  2 y ) x  2  4 y . y  8 y . y  2 x  2

2 4
(2)

 x  2

Điều kiện: y  0
x  y  0

Xét y = 0, hệ vô nghiệm nên y khác 0 . Chia cả 2 vế của (1) cho y ta được:
2 2
x x x x x
2    6  5  2    2  13  2(  1)
 y y  y y y
x
Dat t= (t  1)
y
PT : 2t 2  6t  5  2t 2  2t  13  2(t  1)
 t 4  2t 3  3t 2  4t  4  0
t  1(loai)
  t  1  t  2   0  
2 2

t  2(t / m)
Với t = 2 => x=2y, thế vào (2) ta được:
4 y 2 y  2  4 y 2 . y  8 y 4 . y  2 2 y  2  4 y 2 y  2  2 2 y  2  8 y 4. y  4 y 2. y
2 2 2 2  2 2
4 2  2  8 y3  4 y    2  22  2   2 y   2.  2 y 
3
(3)
y y y y  y y
Xét hàm số f(u)=u3+2u với u>0; có f’(u) = 3u2 +2>0, mọi u>0 => hàm số đồng biến
 
Từ (3)  f  2  2   f  2 y   2  2  2 y  4 y 3  2 y  2  0  y  1
 y  y
Hệ có nghiệm duy nhất x  2, y  1.

 x 6  3 x 2  4  y 3  3 y 2  6 y
Câu 17. Giải hệ phương trình: 
2 y   x  1 x  y  8  7  x
2

Điều kiện: x 2  y  8  0
 PT(1)  x 6  3x 2   y  1  3  x  1   x 2   3x 2   y  1  3  y  1
3 3 3

 f ( x 2 )  f  y  1 với f(t) = t3 + 3t

NGUYỄN VĂN LỰC  0933.168.309 SP Toán K35 - ĐH Cần Thơ


PT – BPT – HPT FB: http://www.facebook.com/VanLuc168

 Ta có: f’(t) = 3t2 + 3 > 0 t  R  f (t ) đồng biến trên R


Do đó: f ( x 2 )  f  y  1  x 2  y  1
 Với y = x2 – 1 , phương trình (2) trở thành: 2( x2 1)   x  1 2 x2  7  x  7  0
2 x2  7   x  1 2 x 2  7  x  2  0(*)
Đặt t  2 x2  7,(t  7) , pt(*) trở thành: t 2   x  1 t  x  2  0 (**)
Ta có:    x  32 nên (**) có hai nghiệm: t = x + 2 hoặc t = -1 (loại)
 x  2  x  2 x  1
Với t = x + 2  2x2  7  x  2   2    x  3
2 x  7  x  4 x  4  x  4 x  3  0
2 2

Với x = 1  y = 0 (nhận)
Với x = 3  y  8 (nhận)
Kết luận: hệ có hai nghiệm x ; y   1;0 , x ; y    3;8 .

3 x  2 y  3  x  y  5
Câu 18. Giải hệ phương trình:   x, y  
2 3  x  y  2 x  3 y  4  2

u  x  2 y  x  2 y  u
2
 y  u  v  3
2 2

Đặt      2x  3 y  4  u 2  v2  7
v  3  x  y 3  x  y  v  x  6  u  2v
2 2 2

Kết luận nghiệm là (- 3; 2)


3u  v  5
Khi đó hệ ban đầu trở thành:  thế v = 5 – 3u vào phương trình (*)
2v  u  v  7  2
2 2
 *
giải tìm được u = 1, từ đó v = 2
Suy ra x  3, y  2.

 y  x  y  1  x 3  3 y ( x 2  xy  y  1)  1
Câu 19. Giải hệ phương trình:  2
 y  y  5 x  5

 y  x  y  1  x 3  3 y ( x 2  xy  y  1)  1
Giải hệ phương trình :  2
 y  y  5 x  5
y  0
Điều kiện :  ( vì y=0 không thỏa hpt)
 x  y  1
( x  1)
(1)   ( x  1)( x 2  x  1)  3 y ( x  1)( x  y  1)
y  x  y 1
1
 ( x  1)[ x 2  x  3xy  3 y 2  3 y  1  ]
y  x  y 1

NGUYỄN VĂN LỰC  0933.168.309 SP Toán K35 - ĐH Cần Thơ


PT – BPT – HPT FB: http://www.facebook.com/VanLuc168

1
 ( x  1)[ x 2  (3 y  1) x  3 y 2  3 y  1  ] (3)
y  x  y 1
Xét A = x2 + (3y – 1 )x + 3y2 – 3y + 1
 = -3(y - 1)2  0 x  R => A  0 x, y  R
(3)  x = -1
Thay x = -1 vào (2) ta có : y 2  y  5  5
 1  17
y 
2

 1  17
y  (l )
 2
1  17
Vậy hệ phương trình có nghiệm x  1, y  .
2

 12 
1   x 2
y  3x 
Câu 20. Giải hệ phương trình: 

1  12 
  y 6
 y  3x 

Điều kiện: x > 0 và y > 0.


 12 
 1  
 x 2
12 2
 y  3x  1  y  3x  x (1)

 
 1  12  y  6 1  12  6 (2)
 y  3x   y  3x y

2 6 1 3
(1) + (2): 2    1  (*)
x y x y
12 3 1 (*)
12  3 1  3 1 
(2) – (1):        
y  3x y x y  3x  y x 
 y x 
12 9 1
  
y  3x y x  y  3x
 y2  6xy  27x 2  0 
 y  9x
So với điều kiện, nhận y = 3x
(*)  x  4  2 3  y  12  6 3
Vậy hệ phương trình có nghiệm x  4  2 3, y  12  6 3.


 x  x2  4 y  y 2  1  2

Câu 21. Giải hệ phương trình: 
  ( x; y  )

12 y  10 y  2  2 x  1
2 3 3

NGUYỄN VĂN LỰC  0933.168.309 SP Toán K35 - ĐH Cần Thơ


PT – BPT – HPT FB: http://www.facebook.com/VanLuc168

Ta có: (1)  x  x 2  4  (2 y)2  4  (2 y) (*) .

t t  t2  4 t t
Xét hàm số đặc trưng f (t )  t 2  4  t  f '(t )  1  0. 
t 4 t 4
2
t2  4 2

Suy ra f(t) là hàm số đồng biến trên R. Từ (*) suy ra: f ( x)  f (2 y )  x  2 y .
Thay vào phương trình (2) ta được:
3x 2  5 x  2  2 3 x3  1
  x  1  2  x  1   x 3  1  2 3 x 3  1 (**)
3

Xét hàm số g (t )  t 3  2t ta thấy g(t) đồng biến trên R nên từ (**) suy ra
x  0
. Vậy hệ có hai nghiệm là  x ; y    1;  , x ; y    0; 0  .
1
x  1  3 x3  1  
 x  1  2

 y 3  y 2  4( x  y  1)  xy 2
Câu 22. Giải hệ phương trình:  2
( x  1) y  x (2 y  1)  x  3x  2

2 2 2

y  2
Biến đổi phương trình ban đầu về dạng ( y  2)( y  2)( y  1  x)  0   y  2
 y  x  1

TH 1: Với y = 2 thay vào phương trình (2) : 8 x 2  3x  6  0 vô nghiệm

TH 2: Với y = - 2 thay vào (2): 3x  6  0  x  2 suy ra nghiệm (x; y) =(-2;-2)

1 1 5
TH 3: Với y  x  1 thay vào (2): x 4  x  3  0  ( x 2  ) 2  ( x  ) 2   0 (vn)
2 2 2
Kl: hệ phương trình có nghiệm x  2, y  2.

 x 3  y 3  6 y 2  3  x  5 y   14
Câu 23. Giải hệ phương trình:   x, y  
 3  x  y  4  x 3
 y 2
 5

 x  y  6 y  3  x  5 y   14 (1)
3 3 2


 3  x  y  4  x  y  5 (2)
3 2

Đkxđ  x  3
 y  4

Từ (1) ta có x3  3x   y  2 3  3  y  2    x  y  2   x 2  x  y  2    y  2 2  3  0
 y  x  2  3

NGUYỄN VĂN LỰC  0933.168.309 SP Toán K35 - ĐH Cần Thơ


PT – BPT – HPT FB: http://www.facebook.com/VanLuc168

Thế (3) vào (2) ta được


x  2  3  x  x3  x 2  4 x  1  x3  x 2  4 x  4  2  x  2  1  3  x  0
x2 x2
  x  2  x  2  x  1   0
2  x  2 1 3  x
 1 1 
  x  2    x  2  x  1   0
 2  x  2 1 3  x 
 1 1 1 1
  x  2    x  2  x  1     0
 3 2  x  2 1 3  x 3 

 
x 1 x 1
  x  2    x  2  x  1   0

  3 2 x2   
x  2 1 3 1 3  x 2  3  x   

 
1 1
  x  2  x  1   x  2    0

  3 2 x2   
x  2 1 3 1 3  x 2  3  x   

x 1 x 1
y  4  x  2  x  2   0
Vì 
3 2 x2   31 
x  2 1 
3 x 2 3 x 
Từ đó phương trình trên tương đương với
x  2
 x  2  x  1  0  
 x  1

Với x  2  y  0; x  1  y  3 .
Thử lại ta thấy thỏa mãn hệ phương trình. Vậy hệ phương trình đã cho có tập nghiệm là
x ; y    1; 3 , x ; y    2;0 .

 x  3xy  x  1  y  2 xy  x
Câu 24. Giải hệ phương trình:  3
3 2 2 2

2

 y  3 yx  y  1  x  2 xy  y
2 2

Từ (1) và (2) ta có x 3  3xy 2  x  1  ( y 3  3 yx 2  y  1)i  y 2  2 xy  x 2  ( x 2  2 xy  y 2 )i


 x 3  3x 2 yi  3xy 2 i 2  y 3i 3  ( x  yi)  1  i  (1  i ) y 2  2 xy(1  i )  x 2 (1  i )
 ( x  yi) 3  ( x  yi)  1  i  (1  i )( y 2  2 xyi  i 2 x 2 )
 ( x  yi) 3  ( x  yi)  1  i  (1  i )( y  ix) 2  z 3  (1  i ) z 2  z  (1  i )  0
 z  1; z  1; z  1  i .
Do đó x ; y   1;0 , x ; y    1;0  , x ; y    1; 1 .

 x  1   x  1 y  2   x  5  2 y  y  2

Câu 25. Giải hệ phương trình:   x  8 y  1
 2
 x  4x  7
  y  2 x  1  3  

NGUYỄN VĂN LỰC  0933.168.309 SP Toán K35 - ĐH Cần Thơ


PT – BPT – HPT FB: http://www.facebook.com/VanLuc168

Điều kiện x  1; y  2 .


Đặt x  1  a; y  2  b  a, b  0 , từ (1) ta có:
a  ab  a 2  1  5  2  b 2  2   b  a  b  ab  b 2  a 2  b 2  0
  a  b 1  2a  b   0
ab (do a, b  0  1  2a  b  0
 x 1  y  2  y  x  3 .
Thế vào (2) ta được:
 x  8 x  4  x  8 x  4   x  1 x  8
x  4x  7
2
  x  1  x 1  3  x2  4 x  7

x 1  3
x  8
  x4 x 1
 2   *
 x  4 x  7 x 1  3
+ x  8  y  11;
+ *   x  1  3  x  4    x  1  x 2  4 x  7 

 
x 1  3    3   x  2   3 .  x  2   3
2
 x 1
2
   
(**)
Xét hàm số f  t    t  3  t 2  3 với t  có f '  t   3  t  1  0 t  nên f  t  đồng biến
2

trên .
x  2
Do đó **  f  x  1   f  x  2   x  1  x  2  
x  1  x  4x  4
2

x  2 5  13
 2 x (T/M)
 x  5x  3  0 2

5  13 11  13
x y
2 2
 5  13 11  13 
Vậy hệ đã cho có nghiệm là  x ; y   8;11 ,  x ; y    ;  .
 2 2 

2 x3  4 x 2  3x  1  2 x3  2  y  3  2 y 1

Câu 26. Giải hệ phương trình: 
 x  2  14  x 3  2 y  1
 3
 2

Ta thấy x  0 không phải là nghiệm của hệ, chia cả hai vế của (1) cho x 3 ta được
4 3 1
1  2     22  y 3  2 y
x x 2 x3
3
 1  1
 1    1     3  2 y  3  2 y  3  2 y *
 x  x
Xét hàm f  t   t 3  t luôn đồng biến trên

NGUYỄN VĂN LỰC  0933.168.309 SP Toán K35 - ĐH Cần Thơ


PT – BPT – HPT FB: http://www.facebook.com/VanLuc168

1
 *  1   3 2y  3
x
Thế (3) vào (2) ta được x  2  3 15  x  1  x  2  3  2  3 15  x  0
 
 
 1 1 
  x  7   2 
0


x  2  3 4  2 3 x  15   3

x  15 

 0 
111
Vậy hệ đã cho có nghiệm x  7, y  .
98

 x 2  xy  2 y 2  3 y  1  y  1  x

Câu 27. Giải hệ phương trình:  .
 
3 6  y  2 x  3 y  7  2 x  7

x  0

Điều kiện 1  y  6 .
2 x  3 y  7  0

Với điều kiện trên ta có :
y 1 x
(1)   ( y  1  x)( y  1  x)  y ( y  1  x)  0
y 1  x
 1 
 ( y  1  x)   y 1 x  y   0
 y 1  x 
 
 y  x 1
  1
 y  1  x  y  0 (*)
 y  1  x

x  0
+ Với  , suy ra phương trình (*) vô nghiệm
1  y  6
+ Với y  x  1 thay vào (2) ta được 3 5  x  3 5x  4  2 x  7 (3)
4
Điều kiện  x  5 ta có :
5
(3)  7  x  3 5  x  3( x  5 x  4)  0
7  x
2
 9 5  x  3  x2  5x  4
  0
7 x3 5 x x  5x  4
 
  x2  5x  4 
1 3
 0
 7  x  3 5  x x  5x  4 

NGUYỄN VĂN LỰC  0933.168.309 SP Toán K35 - ĐH Cần Thơ


PT – BPT – HPT FB: http://www.facebook.com/VanLuc168

 2 x  1
 x  5x  4  0  
 x  4
 1 3
   0(VN )
 7  x  3 5  x x  5x  4
Vậy hệ phương trình đã cho có nghiệm là x ; y   1; 2  , x ; y    4;5 .

 y 3  y  4  3 x  ( x  2) x  2
Câu 28. Giải hệ phương trình: 
( x  y  5) x  y  2 y  4  0

a  x  y
Đặt  . (ĐK: b  0).
b  x  y

Thay vào phương trình (2) ta được:


(a  5)b  a  b 2  4  0  (b  1)(a  b  4)  0
 a b4
 x  y  4  x  y (3)

Ta có: (1)  y3  y  ( x  2  1)3  ( x  2  1)


Xét hàm số: f (t )  t 3  t đồng biến trên .

Do đó ta có: y  x  2  1 (4)
Từ (3) và (4) ta được:
 x  y  4  x  y  x  2  y  2  x  2  2  y
 
 y  1  x  2  x  2  y  1

( y  1) 2  y  2  ( y  1) 2  2  y

 x  2  y  1
 y 2  y  1  y 2  3 y  3

 x  2  y  1
x  3

y  2

Kết hợp với điều kiện (*), ta được: x  3, y  2 là nghiệm của hệ phương trình đã cho.

 x 3  y 3  5 x 2  2 y 2  10 x  3 y  6  0
Câu 29. Giải hệ phương trình: 
 x  2  4  y  x 3  y 2  4 x  2 y

Điều kiện x  -2; y  4


NGUYỄN VĂN LỰC  0933.168.309 SP Toán K35 - ĐH Cần Thơ
PT – BPT – HPT FB: http://www.facebook.com/VanLuc168

(1)  x 3  5 x 2  10 x  6  y 3  2 y 2  3 y
 x  1  2x  1  3( x  1)  y 3  2 y 2  3 y
3 2

Xét hàm số f (t )  t 3  2t 2  3t , f ' (t )  3t 2  4t  3  0 t  R


Suy ra f(x+1) = f(y) => y= x+1 thay và phương trình (2) ta đuợc
Phương trình : x  2  3  x  x 3  x 2  4 x  1
2  x  2 3  x   2 
  x  2  3  x   3  x3  x 2  4 x  4    x  1x 2  4 
x  2  3 x 3
2 x  2 3  x   4
   x  2 ( x 2  x  2)
 
x  2  3  x  3  x  2 3  x   2 
2( x  x  2)
  x  2 x 2  x  2   0
2

 
x  2  3  x  3  x  2 3  x   2 
 
 
 x2  x  2  x  2 
2
0
  x  2  3 x 3 
x  23  x   2  
 0 (vi x  2 )
x  2
 x2  x  2  0  
 x  1
Vậy hệ phương trình có nghiệm x ; y    2;3 , x ; y    1;0 .


 x  x  xy  2 x  3x  4 xy
2 2

Câu 30. Giải hệ phương trình:  2  x, y  R 



 x y 1  y 2
 1  x 2
 x 2
y  x

 x  x 2  xy  2 x 2  3x  4 xy 1

 2  x, y  R 
 x y 1  y 2  1  x 2  x 2 y  x  2 

pt  2   x 2 y  
1  y2 1  1  x2  x  y  
1 y2 1 
1  x2  x
x2
Suy ra đk y > 0 , kết hợp phương trình (1) suy ra đk x > 0 (x = 0 ko là nghiệm phương
trình (2)
y  1  y2 1   1  x2  x
x2
y  1
 1 
1  y 2  1   1  2  1 ,
x x 
2t 2  1  1  t 2 1
Xét hàm f  t   t 1  t 2  t , t  0 , f '  t    0 …. Suy ra pt  2   y 
1 t2 x
Thế vào pt(1) :
x  x 2  1  2 x 2  3x  4  2 x 2
 x   x  1  x 2  x  3  x  1
u  x 2  x
Đặt  giải được u = - v ( vô nghiệm ) , u = 3v
v  x  1
1
u = 3v giải được nghiệm x  5  34, y  .
5  34

NGUYỄN VĂN LỰC  0933.168.309 SP Toán K35 - ĐH Cần Thơ


PT – BPT – HPT FB: http://www.facebook.com/VanLuc168

4 x 2  y  x  9  3x  1  x 2  5 x  y  8

Câu 31. Giải hệ phương trình: 
 x 12  y  y 12  x   12
2

 1
x   3

Điều kiện:  y  12  *
 y 12  x 2   0

 x 2  5 x  y  8  0

Ta có
 x 12  y  12
 2  y 12  x 2   12  x 12  y  
12 x  24 x 12  y  12 12  y 
2

 x 12  y  12  y  12  x 2
 
  1
 
2
 x  12  y  0   x  2 3; 0  y  12
 3

Thay vào phương trình 1 ta được: 3x 2  x  3  3x  1  5 x  4

  
 3  x 2  x   x  1  3x  1  x  2  5 x  4  0 
 
  x2  x   3 
1 1
 0
 x  1  3x  1 x  2  5 x  4 

 x2  x  0  x  0 hoặc x  1 . Khi đó ta được nghiệm x ; y    0;12 , x ; y   1;11 .

 x x 2  y  y  x 4  x3  x

Câu 32. Giải hệ phương trình:  9
 x  y  x  1  y (x  1) 
 2

Đk: x  1; y  0

 x( x 2  y  x 2  x )  y  x  0
x( y  x)
  yx0
Phương trình (1) x2  y  x2  x
y  x

 x
 1  0(VN(VT  0 x  1))
 x 2  y  x 2  x

9
+ Với y = x thề vào (2) ta được: x  x  x  1  x( x  1)  (*)
2
NGUYỄN VĂN LỰC  0933.168.309 SP Toán K35 - ĐH Cần Thơ
PT – BPT – HPT FB: http://www.facebook.com/VanLuc168

Đặt ẩn phụ:
t  x  x  1, (t  0)
 t 2  2 x  2 x( x  1)  1
t2 1
 x  x( x  1) 
2
t 2 1 9 t  2
Từ (*) ta có:  t   t 2  2t  8  0  
2 2 t  4(loai)
5 25 25
+ Với t  2  x  x  1  2  x( x  1)   x  x  ,y
2 16 16
25 25
Vậy hệ phương trình có nghiệm: x  , y .
16 16

Câu 33. Giải hệ phương trình:



( xy  3) y  2  x  x  ( y  3x) y  2
5

 ( x, y  )
 9 x  16  2 2 y  8  4 2  x
 2

0  x  2
Đk:  (*) .Với đk(*) ta có
 y  2

x  1
(1)  ( x  1) ( y  3) y  2  ( x  1) x   0  
( y  3) y  2  ( x  1) x (3)

31
Với x = 1 thay vào (2) ta được: 2 2 y  8  1  y   (loai ) . Ta có:
8

 
3
(3)  y  2  y  2  ( x )3  x (4). Xét hàm số f (t )  t 3  t  f '(t )  3t 2  1  0; t  Hàm số

f(t) là hs đồng biến, do đó:

(4)  f ( y  2)  f ( x )  y  2  x  y  x  2 thay vào pt(2) ta được:


4 2  x  2 2 x  4  9 x 2  16  32  8 x  16 2(4  x 2 )  9 x 2  8(4  x 2 )  16 2(4  x 2 )  ( x 2  8 x)  0

 x
t  2
Đặt: t  2(4  x 2 ) (t  0) ; phương trình trở thành: 4t  16t  ( x  8 x)  0  
2 2

t   x  4  0(loai )
 2

NGUYỄN VĂN LỰC  0933.168.309 SP Toán K35 - ĐH Cần Thơ


PT – BPT – HPT FB: http://www.facebook.com/VanLuc168

0  x  2
x  4 2 4 2 6
Hay 2(4  x )    2 32  x 
2
y
2  x  3 3
 9

4 2 4 2 6
Vậy hệ phương trình có nghiệm x  , y .
3 3

(4 y  1) x 2  1  2 x 2  2 y  1
Câu 34. Giải hệ phương trình:  4 2 .
2
 x  x y  y  1

Xét phương trình: (4y-1) x 2  1  2 x 2  2 y  1


Đặt: t = x 2  1  1 , ta được pt: 2t2 – (4y-1)t + 2y – 1 = 0
 1
Giải ra được: t  2  1(loai)

t  2 y  1
y  1
 2 thay vào phương trình (2) ta được: 16y2(y -1)2+4y2(y- 1) +y2–1= 0
 x  4 y 2
 4 y
 y = 1(do y  1 )  x=0
Vậy nghiệm của phương trình là x  0, y  1.

 x3  2 y 2  x 2 y  2 xy 1
Câu 35. Giải hệ phương trình: 
2 x  2 y  1  3 y  14  x  2  2 
2 3

ĐK x 2  2 y  1  0
Từ (1) ta có x=y hoặc x2 = 2y (Loại)
x = y, thay vào phương trình ta có: 2 x 2  2 x  1  3 x3  14  x  2
 2 x 2  2 x  1  3 x 3  14   x  2   0
 
3 x2  2x  1
 2 x  2 x  1 1 
2 0
 2 
 x  14   3 x 3  14  x  2    x  2  
3 2
3
 
x  1 2
 x2  2x  1  0   .
 x  1  2


Vậy hệ phương trình đã cho có nghiệm x ; y   1  2;1  2 , x ; y   1  2;1  2 .   

NGUYỄN VĂN LỰC  0933.168.309 SP Toán K35 - ĐH Cần Thơ


PT – BPT – HPT FB: http://www.facebook.com/VanLuc168

 2 x
 x  x  1   y  2   x  1 y  1
Câu 36. Giải hê ̣ phương trình:   x, y  
3x 2  8 x  3  4  x  1 y  1

 x  1
Điều kiện: 
 y  1
x3  x 2  x x3  x  x  1
1    y  2  x  1 y  1    y  2 y  1
x 1  x  1 x 1
3
 x  x
 
3
    y 1  y 1 .
 x 1  x 1
Xét hàm số f  t   t 3  t trên có f   t   3t 2  1  0t  suy ra f(t) đồ ng biế n trên . Nên
 x 
f  f
 x 1 
 y 1   x
x 1
 y  1 . Thay vào (2) ta đươ ̣c 3 x 2  8 x  3  4 x x  1 .

  x 1
  2  x  3 2 3
 2 x 1  x 1  x  6x  3  0

  2 x  1  x  2 x  1  
2
   
2

x
1 5  2 13
 2 x  1  1  3 x   x
 3  9
 9 x 2  10 x  3  0

x2 43 3 5  2 13 41  7 13
Ta có y   1 . Với x  3  2 3  y  . Với x   y .
x 1 2 9 72
Các nghiê ̣m này đề u thỏa mãn điề u kiê ̣n.
KL: Hê ̣ phương trình có hai nghiê ̣m.
 43 3   5  2 13 41  7 13 
x ; y    3  2 3;  ,  x ; y    ;  .
 2   9 72 

 x 2  xy  2 y  2 y 2  2 x (1)
Câu 37. Giải hệ phương trình:  .
 y x  y  1  x  2. (2)

ĐK: x  y  1  0.
x  y (3)
(1)  x 2  y 2  xy  y 2  2 y  2 x  0  ( x  y )( x  2 y  2)  0  
 x  2  2 y (4)
 Từ (3) & (2) ta có x=y=1.
 y  0; x  2
x  2  2 y

 Từ (4) & (2) ta có  

 y 3  3 y  2 y y   1; x  8.
 3 3

NGUYỄN VĂN LỰC  0933.168.309 SP Toán K35 - ĐH Cần Thơ


PT – BPT – HPT FB: http://www.facebook.com/VanLuc168

Vậy hệ phương trình đã cho có 3 nghiệm  x ; y   1;1 , x ; y    2; 0  , x ; y    ;   .


8 1
3 3

 2 2 xy
 x  y  x  y  1 (1)
2

Câu 38. Giải hệ phương trình: 


 x  y  x2  y
 (2)

ĐK: x  y  0.
2 xy x  y 1
Ta có (1)  x 2  2 xy  y 2   2 xy  1  ( x  y )2  1  2 xy. 0
x y x y
x  1 y (3)
 2 xy   2
 ( x  y  1)  x  y  1    0   x  y  x  y  0 (4)
2

 x y
 x y
Vì x  y  0 nên phương trình (4) vô nghiệm.
 y  0; x  1
Từ (3) và (2) ta có y 2  3 y  0   .
 y  3; x  2
Vậy hệ phương trình đã cho có 2 nghiệm x ; y   1;0 , x ; y    2;3 .

 1
 3 x (1  )2 (1)
 x y
Câu 39. Giải hệ phương trình: 
 7 y (1  1 )  4 2 (2)
 x y

ĐK x  0; y  0.
Dễ thấy x = 0 hoặc y = 0 không thõa mãn hệ.
Với x >0, y >0 ta có
 1 2  1 2 2
1 
 x y  1  
 3x  3x 7y 1 1 8
      ( nhân vế với vế)
1  1 4 2  1 1 2 2 x  y 3 x 7 y

 x  y 7y  x  y  3x  7 y

 21xy  (7 y  24 x)( x  y)  24 x 2  38xy  7 y 2  0  y  6 x (vì x, y dương).
1 2 1 1  1 2 
Thay vào phương trình (1) ta được  . 1  0   7  .
7x 3 x x  3 21 
Từ đó suy ra x và y .

NGUYỄN VĂN LỰC  0933.168.309 SP Toán K35 - ĐH Cần Thơ


PT – BPT – HPT FB: http://www.facebook.com/VanLuc168

2 x 3  x  2 x 2 y  y
Câu 40. Giải hệ phương trình :  2  x, y  .
 x  12 x  12 y  3  3 y  2 x  1

ĐK: x  0; y  0 .
Phương trình (1)  x  2 x2  1  y  2 x2  1   x  y   2 x 2  1  0  x  y
(Vì 2 x 2  1  0, x  ).
Thế vào phương trình (2) ta có

   
2
x 2  12 x  12 x  3  3x  2 x  1  x 2  3 2 x  1  3 x  2 x  1

Đặt a  2 x  1, a  1 , ta có phương trình x 2  3a 2  3 x  a  x2  3a 2  9 x2  6ax  a 2


a  x
 x2  3a 2  9 x2  6ax  a 2  8 x 2  6ax  2a 2  0  
 a  4 x  L 
 x  1 2
Khi a  x , ta có x  2 x  1  x  2 x  1  0    x  3 2 2
 x  1  2  L 
 y  3  2 2 . Thử lại thấy thỏa mãn.
Vậy hệ phương trình có nghiệm x ; y   3  2 2;3  2 2 .  

x  x y  x  x  y 1
3 2 2

Câu 41. Giải hệ phương trình:  3 .



 x  9 y 2
 6  x  3 y   15  3 3
6 x 2
 2

1  x3  x2 y  x2  x  y  1  x2  x  y    x  y   x2  1   x  y   x2  1  x2  1
 x  y  1  0 (vì x 2  1  0, x )
Thế vào phương trình (2) ta có x3  9 x 2  6 x  6  3 3 6 x 2  2
 x  1  3  x  1   6 x 2  2   3 3 6 x 2  2  3
3

Xét hàm số f  t   t 3  3t  f '  t   3t 2  3  0  t    f t  đồng biến trên .


Phương trình (3)  f  x  1  f  3

6x2  2  x  1  3 6x2  2 .

 x3  9 x 2  3x  3  0   x  1  2  x  1 .
3 3

3
2 1 2
 x  1  2  x  1  x 
3
y .
3
2 1 3
2 1
 3 2 1 2 
Vậy hệ phương trình có nghiệm  x ; y    3 ;3 .
 2 1 2 1 

NGUYỄN VĂN LỰC  0933.168.309 SP Toán K35 - ĐH Cần Thơ


PT – BPT – HPT FB: http://www.facebook.com/VanLuc168

 2 y
 x   x  y  
Câu 42. Giải hệ phương trình:  3
x y
2 x 2  y 2  3 2 x  1  11
  

 x 2   x  y  3 x  y  y 1

Hệ đã cho tương đương với 
2  x  y   3 2 x  1  11 2 
2 2

Từ (1) suy ra y  0 , vì nếu y<0 thì x-y>0, do đó VT(1) > VP( 1)


1  x2   x  y   3
x  y 1    
x2   x  y   y  0

x  y 1 x2  x  y  y 2
 x2   x  y   0
 x  y  x  y 1 x2  x  y  y
2
3 3

 x2   x  y  
x y
  x  y  1   2   0  x  y 1  0
 3  x  y  3 x  y 1
2
x  x y  y
 
Thế y  x  1 vào phương trình (2) ta được:
4 x3  4 x  2  3 2 x  1  11   2 x  1  3 2 x  1  10  0
2

Đặt t  2 x  1, t  0 , ta có t 4  3t  10  0   t  2  t 3  2t 2  4t  5  0  t  2
5 3 5 3
Khi đó 2x 1  2  x   y  . Vậy hệ phương trình có nghiệm x ; y    ;  .
2 2 2 2

 x 2  y 2  xy  1  4 y (1)
Câu 43. Giải hệ phương trình: 
 y ( x  y )  2 x  7 y  2 (2)
2 2

Nhận thấy y=0 không thỏa mãn hệ.


 x2  1
 y x y 4

Với y khác không, chia cả hai vế của (1) và (2) cho y ta được: 
( x  y ) 2  2 x  1  7
2

 y
a  x  y
 a  b  4 b  4  a b  4  a  a  5, b  9
Đặt  x 2  1 ta có  2  2  2  .
b  y a  2b  7 a  2(4  a)  7 a  2a -15  0  a  3, b  1

Từ đây ta tìm được x ; y   1; 2 , x ; y    2;5 .

NGUYỄN VĂN LỰC  0933.168.309 SP Toán K35 - ĐH Cần Thơ


PT – BPT – HPT FB: http://www.facebook.com/VanLuc168

 2 5
 x  y  x y  xy  xy   4
3 2

Câu 44. Giải hệ phương trình: 


 x 4  y 2  xy (1  2 x)   5
 4

 2 5
 x  y  xy ( x  y )  xy   4
2

Hệ đã cho tương đương với 


( x 2  y ) 2  xy   5
 4
 5  5
 x2  y  a a  ab  b   4 b   4  a
2

Đặt  , ta được hệ mới  


 xy  b a 2  b   5 a  5 a  a 3  5  a 2   5
 4  4 4 4
 3 a  5
 a  a  4  0  a  0; b   4
2

 
b   5  a 2 a   1 ; b   3
 4  2 2
 5 25   3
Từ đó ta tìm được  x ; y    3 ;  3  ,  x ; y   1;   .
 4 16   2

Câu 45. Giải hệ phương trình:



 
 xy 2 x 2  1  1  3 y 2  9  3 y
 x, y  .
 3x  1 x y  xy  5  4 x  3x y  7 x  0
 2 3 3

ĐK: x 2 y  xy  5
Xét phương trình (1) 3 y 2  9  3 y  3 y  3 y  0, y ; y 2  
x 2  1  x  0, x; y  x  0

Mà x2 y  xy  5  y  x 2  x   5  y  0 .
2
3 3 3
Khi đó ta có: x x  1  x  1    
2
1a 
y  y y
t2
Xét hàm số f  t   t t 2  1  t , t   0;    f '  t   t 2  1   1  0, t   0;  
t2 1
 Hàm số f  t  đồng biến trên  0;   .
3 3 3
Do đó phương trình 1a   f  x   f    x   y  .
 y y x
3
Thay y  vào phương trình (2) ta có
x

NGUYỄN VĂN LỰC  0933.168.309 SP Toán K35 - ĐH Cần Thơ


PT – BPT – HPT FB: http://www.facebook.com/VanLuc168

 3x  1 3x  2  4 x3  9 x 2  7 x  0   3x  1  
3x  2  x  4 x 3  12 x 2  8 x

 x 2  3x  2  3x  1 
  3x  1  4 x3  12 x 2  8 x   x 2  3x  2   x  0
3x  2  x  3x  2  x 
x  1 3x  1 2
 x 2  3x  2  0   ( Vì x   0, x  )
x  2 3x  2  x 3

Vậy hệ phương trình có nghiệm x ; y   1;3 , x ; y    2;  .


3
 2

Câu 46. Giải hệ phương trình:


 3x  5  x 2  1  y  x 2  3x  y  6 
 , x, y  . (x; y  R).
 4
 y 2
 2 y  1  y  3 x  4

Điều kiện :  y 2  2 y  1  0
 3x  5 ( x2  1)  y ( x2  1)   3x  5  y 
 y  3x  5
  3x  5  y  ( x 2  1  y )  0  
 y  x 1
2

Với y = 3x - 5 thay vào (2) ta được 4  y 2  2 y  1  1  0 vô nghiệm


Với y  x2  1 thay vào (2) ta được 4
2  x 4  x 2  3x  3 (*)
Điệu kiện  4 2  x  4 2 .
5  x4
Áp dụng bất đẳng thức Cauchy ta có 1.1.1. 4 2  x 4 
4
5  x4
Từ (3) ta có: x 2  3x  3   x 4  4 x 2  12 x  7  0
4
  x  1  x  2 x  7   0  x  1
2 2

Thử lại x = 1 thỏa mãn (*). Vậy hệ đã cho có nghiệm là x  1, y  0.

 x x2  y 2  x2  2 x  y 2 3
     .
Câu 47. Giải hệ phương trình: 
76 x 2  20 y 2  2  3 4 x  8 x  1

Điều kiện: x  y 2
Phương trình  x  x  y 2   x 3  2 x  y  2 3
0


 x  x 2   x  y 2    2  x  y 2  x  x  y 2  0 

NGUYỄN VĂN LỰC  0933.168.309 SP Toán K35 - ĐH Cần Thơ


PT – BPT – HPT FB: http://www.facebook.com/VanLuc168

   
 x  x  y 2  x x  x  y 2  2  x  y 2   0
 
 x  x  y 2  y 2  x  x2
Khi đó phương trình (2) trở thành: 96 x 2  20 x  2  3 4 x 8 x  1
8x  1
8x  1
3
 8 x  1  2  3 4 x  8 x  1
2

2 3
1
Sử du ̣ng BĐT Cô si cho 3 số ta tìm đươ ̣c nghiệm duy nhấ t của phương trình x 
8
1 7 1  7 
Vậy hệ phương trình có nghiệm  x ; y    ;  , x ; y    ; .
 8 8   8 8 

 x 5x  y    x  y  x  y
 2 2 2

Câu 48. Giải hệ phương trình:  .



 y 3
 9 x 4
 6 x 2
 1  0

1  5 x3  xy   x 2  y  x 2  y  0  4x3  x  x 2  y   2   0
3
x2  y

x  0
 2x  x2  y   ( Vì x =y =0 không là nghiệm của hệ )
 y  3 x 2

Thế vào phương trình (2) ta có y3  y 2  2 y  1  0 (*)


Ta giải phương trình (*) trên tập .
  
Thật vậy: xét y   2;2  , Đặt y  2sin t , t    ; ,
 2 2 

Pt(*) trở thành: 8sin 3 t  4sin 2 t  2sin t  1  0


 4sin t 1  2sin 2 t   1  4sin 2 t  4sin t.cos 2t  4cos 2 t  3
 sin 4t  cos3t ( Do cos t  0 không là nghiệm của pt)
  k 2
 t 
 
 sin 4t  sin   3t   
14 7
k  
2  
t    k 2
 2
   5 3  5 3
Vì t    ;   t   ; ;    y  2sin ;2sin ; 2sin 
 2 2  14 14 14   14 14 14 
Mà phương trình bậc 3 có tối đa 3 nghiệm nên pt(*) có 3 nghiệm như trên

NGUYỄN VĂN LỰC  0933.168.309 SP Toán K35 - ĐH Cần Thơ


PT – BPT – HPT FB: http://www.facebook.com/VanLuc168

 
  2sin
 y  2sin  x  14
Kết hợp với điều kiện y  0 ta có 
14 3
 3
 2sin
 y  2sin 5  x  14
 14 3
Từ đó tìm được nghiệm của hệ phương trình.

    3 
 2 sin   2 sin 
x ; y    14 ; 2 sin   ,  x ; y    14 ; 2 sin 5  .
 3 14   3 14 
   
   

 3x  4  3 y  4  x3  x 2 y  0
Câu 49. Giải hệ phương trình:   x, y  .
 x  3 y  1  8  3x
3 2

4 8 4
ĐK   x  ; y .
3 3 3

Nhận xét:  x; y     ;   không là nghiệm của hệ. Do đó x   hoặc y  


4 4 3 3
 3 3 4 4
3 x  y   3 
1   x2  x  y   0   x  y  
 3x  4  3 y  4
 x2   0

3x  4  3 y  4  
Thay x  y vào phương trình (2) ta có.
x3  3x  1  8  3x 2   x3  2 x  1   2  x   8  3x 2  0
 
  x 2  x  1  x  1 
4 3
  0 , Vì 1  x 2  0,
  2  x   8  3x 2  3x  4  3 y  4
 

Ta có
4  x 2  x  6   x  1 8  3x 2
x 1 
2  x  8  3x 2 2  x  8  3x 2

 x  1  8  3x   3  0 , với
2

 x  1  2  x  1 8  3x 2  8  3x 2   3
2 2
4 8
   x

2 2  x  8  3x 2  2  2  x  8  3x  2 3 3

NGUYỄN VĂN LỰC  0933.168.309 SP Toán K35 - ĐH Cần Thơ


PT – BPT – HPT FB: http://www.facebook.com/VanLuc168

 1 5
x 
2
Do đó ta có  x 2  x  1  0  
 1 5
x 
 2
1 5 1 5  1 5 1 5 
Vậy hệ phương trình có nghiệm  x ; y    ;  , x ; y    ; .
 2 2   2 2 

 x 3  3 x 2  2  y3  3 y 2
Câu 50. Giải hệ phương trình:   x, y   .
 14 x  2 y  48  5  x  x  3

ĐK x  3; y  0;  14 x  2 y  48  0 .
Ta có: x  x  3   x  3  x  3  2   x  3 1  
x  3  2  0  x  3 1 x  4
Mà  14 x  2 y  48  0  2 y  14 x  48  8  y  4
1   x  1  3  x  1   
3
y 3 3 y 3
3

Xét hàm số f  t   t 3  3t , t  1;    f '  t   3t 2  3  0 t  1;  


 f  t  đồng biến trên 1; 

Khi đó phương trình 1  f  x  1  f  


y  3  x 1  y  3
Thế vào phương trình (2) ta có
2 x 2  18 x  44  5  x  x  3  2  x  5   2  x  3   x  5   x  3
2

  x  5   x  3   0  x  5  x  3  x  7
2

Vậy hệ phương trình có nghiệm x  7, y  33.

Câu 51. Giải hệ phương trình: 




 xy 1  1  x 2

4 y  y 8
 x, y 
 
3 x y  2 x y  26 x  2 x  14
4 2 3 3


 xy 1  1  x 2


 4  y  y  8 1
3 x 4 y  2 x 2 y  26 x  2 3 x 3  14  2 

ĐK: y  0
Ta có 4  y  y  y  y  0 do đó từ phương trình (1) suy ra x>0; y>0
1  xy 1  1 x2  4 y  y  4 y  y 8   4 y  y 

NGUYỄN VĂN LỰC  0933.168.309 SP Toán K35 - ĐH Cần Thơ


PT – BPT – HPT FB: http://www.facebook.com/VanLuc168


 xy 1  1  x 2  2   
4  y  y  x  x 1 x2 
2
y

2
y
4
y
1

2
 2   2   2 
 x  x 1 x     2
 1 
 y 
 y  y
(3)
     
t2
Xét hàm số f  t   t  t 1  t 2 trên  0;   . Có f '  t   1  1  t 2   0t   0;  
1  t2
Suy ra hàm số f(t) đồng biến trên  0;   .
 2  2 4
Mà phương trình (3) có dạng f  x   f    x  y 2
 y y x

4 12 x 2  26 x  8  2 3 x 3  14  6 x 2  13 x  4  3 x 3  14
Thay y 2 vào phương trình (2) ta có
x   x  2    x  2    x 3  14   3 x 3  14  4 
3

Xét hàm số g  u   u3  u trên R


Có g '  u   3u2  1  0u  R
Suy ra hàm số g(u) đồng biến trên R mà phương trình (4) có dạng:
 x  1  2  nhaän 
g  x  2  g  
x 3  14  x  2  3 x 3  14  6 x 2  12 x  6  0  
3

 x  1  2  loaïi 
=> y  12  8 2
Vậy hệ có nghiệm duy nhất x  1  2, y  12  8 2.

 x2  2 x  2   y2  4 y  2
Câu 52. Giải hệ phương trình: 
6 x  y  11  10  4 x  2 x  0
 2


y  4y  2  0
2

Điều kiện:  2 phương trình


 2 x  4 x  10  0

Áp dụng bất đẳng thức AM-GM ta có:
4(10  4 x  2 x 2 ) 14  4 x  2 x 2
y  6 x  11  10  4 x  x 2  
2 4
Rút gọn ta được: 4( y  6 x  11)  14  4 x  2 x  x  10 x  2 y  15  0 (3)
2 2

 y2  4 y  2
Tương tự phương trình (1) x 2  2 x  2   y 2  4 y  2   2 x2  4 x  y2  4 y  3  0
2
(4)
Cộng vế với vế của (3) và (4) ta được:
x  1
3x 2  6 x  y 2  6 y  12  0  3( x  1) 2  ( y  3) 2  0  
 y  3
Kết hợp với điều kiện đề bài, suy ra nghiệm hệ phương trình là x  1, y  3.

Câu 53. Giải hệ phương trình:


NGUYỄN VĂN LỰC  0933.168.309 SP Toán K35 - ĐH Cần Thơ
PT – BPT – HPT FB: http://www.facebook.com/VanLuc168

4x 2 3xy 7y 2 4 x2 5xy 6y 2 3x 2 2xy y2


 x, y   .
3x 2 10xy 34y 2 47

3x 2 2xy y2 0
ĐK: 2 2
4x 3xy 7y 0

Chuyển vế nhân liên hợp ở phương trình 1 , ta được:


1 x y
x2 5xy 6y 2 4 0
4x 2 3xy 7y 2 3x 2 2xy y2 x 6y

x 1 y 1
Với x y thay vào 2 , ta được: x 2 1
x 1 y 1

47 47
y x 6
Với x 6y thay vào 2 , ta được: 82y 2 47 82 82 ;
47 47
y x 6
82 82
47 47 47 47
KL: S 1;1 , 1; 1 , ; 6 ; ;6 .
82 82 82 82

 2 y 3  2 x 1  x  3 1  x  y
Câu 54. Giải hệ phương trình: 
 y  1  2 x  2 xy 1  x
2

Đk: 1  x  1

Hệ phương trình (I) 




2 y 3  y  2 1  x 3  1  x

 y  1  2 x 2  2 xy 1  x

 y  1  x 1 , y  0 (Do hàm f  t   2t 3  t luôn đồng biến)



 y  1  2 x  2 xy 1  x
2
 2
Ta có (2)  1  x  1  2 x 2  2 x 1  x 2
 2x2  2x 1  x2  1  x 1  0
Đặt x  cos t với t   0;  
t t
Ta có x  cos t  1  2s in 2  1  x  2 sin
2 2
t
Nên phương trình (2) trở thành 2cos 2t  2 cos t sin t  2 sin  1  0
2
  t
 2 sin  2t    2 sin
 4 2

NGUYỄN VĂN LỰC  0933.168.309 SP Toán K35 - ĐH Cần Thơ


PT – BPT – HPT FB: http://www.facebook.com/VanLuc168

  k 4
t   3  3
 k  
t    k 4
 5 5
 
  x  cos
t     
0;  
 5
5 là nghiệm của hệ phương trình.


t    l   y  2 sin 

 10

ìï x 3 +12 y 2 + x + 2 = 8y 3 +8y
Câu 55. Giải hệ phương trình: í 2
ïî x +8y 3 + 2 y = 5x

ìï x 3 +12 y 2 + x + 2 = 8y 3 +8y(1)
í
îï x +8y + 2 y = 5x(2)
2 3

Ta có (1) Û x3 + x = (2y -1)3 +(2y -1)(*)


Xét hàm số f (t) = t 3 +t,"t Î » , f '(t) = 3t 2 +1> 0,"t Î » . Vậy hàm số f(t) đồng biến trên R. Từ
(*) ta có: f (x) = f (2y -1) Û x = 2y -1
Thế x=2y-1 vào (2) giải ra được y=1 hoặc y=6 thoả mãn
Vậy hệ phương trình có nghiệm x ; y   1;1 , x ; y   11;6  .


2(2 x  1)  2 x  1  (2 y  3) y  2
3

Câu 56. Giải hệ phương trình: 



 4x  2  2 y  4  6

1
Điều kiện xác định: x   , y  2
2
Xét hàm số: f (t )  2t 3  t t   0;  
Suy ra f '(t )  6t 2  1  0 nên đây là hàm số đồng biến
Từ phương trình thứ nhất của hệ ta có f (2 x  1)  f ( y  2)  2 x  1  y  2
Thay vào phương trình thứ hai ta được: 4 4 y  8  2 y  4  6 (*)
Xét hàm số g ( y )  4 4 y  8  2 y  4  6, y   2;  
1 1
g '( y )    0 y   2;   nên g(y) đồng biến
4 4y 8 2y  4
Hơn nữa g(6) = 0 nên (*) có duy nhất 1 nghiệm là y = 6
1
Với y = 6 ta có x 
2

NGUYỄN VĂN LỰC  0933.168.309 SP Toán K35 - ĐH Cần Thơ


PT – BPT – HPT FB: http://www.facebook.com/VanLuc168

 x y 7
   1
Câu 57. Giải hệ phương trình:  y x xy

 x xy  y xy  78

ĐK: x, y> 0.
 x  y  7  xy
(I)   .
 xy  x  y   78
Đặt t  xy . (ĐK: t>0)
x  y  7  t t  13  l 
  t 2  7t  78  0 .  
t  x  y   78 t  6 n
 x  y  13 x  4 x  9
t=6    v 
 xy  36 y  9 y  4
Vậy hệ phương trình có 2 nghiệm là: x ; y    4;9 , x ; y    9; 4  .

(1  y )( x  3 y  3)  x 2  ( y  1)3 . x

Câu 58. Giải hệ phương trình:  ( x, y  R ) .
 x 2  y  2 3 x3  4  2( y  2)

 x 2  y  0  x 2  y
ĐKXĐ:  
 x  0, y  1  x  1, y  1
Nhận xét x  1, y  1 không là nghiệm của hệ. Xét y  1 thì phương trình (1) của hệ (I)
x 2  x( y  1)  3( y  1) 2  ( y  1) x( y  1)  0
2
 x  x x
   3 0
 y 1  y 1 y 1
x
t , t  0 . Khi đó, phương trình (1) trở thành
y 1
t 4  t 2  t  3  0   t  1  t 3  t 2  2t  3  0  t  1.
x
Với t = 1, thì  1  y  x 1 , thế vào pt(2), ta được
y 1
x 2  x  1  2 3 x3  4  2  x  1

 x 2  x  1  2  3 x 3  4   x  1   0
 

NGUYỄN VĂN LỰC  0933.168.309 SP Toán K35 - ĐH Cần Thơ


PT – BPT – HPT FB: http://www.facebook.com/VanLuc168

 
 x2  x  1 
 x  x 1  6 
2
0
  x  4     x  1 x  4   x  1 
3 2 3 3 2
3

 
 6 x2  x  1 
 x  x  1 1 
2
0
2 
x  4     x  1 3 x 3  4   x  1 
2
 3 3
 
1 5
 x2  x 1  0  x   x  1 .
2
1 5 3 5
Với x y .
2 2
1 5 3 5
Đối chiếu ĐK, hệ phương có nghiệm x  , y .
2 2

NGUYỄN VĂN LỰC  0933.168.309 SP Toán K35 - ĐH Cần Thơ


PP tọa độ trong mặt phẳng FB: http://www.facebook.com/VanLuc168

I. KIẾN THỨC CƠ BẢN


Chuyên đề: Phương pháp tọa độ trong mặt phẳng

A. TỌA ĐỘ ĐIỂM – VÉCTƠ

I. Hệ trục toạ độ ĐỀ-CÁC trong mặt phẳng : y

 x'Ox : trục hoành j


 y'Oy : trục tung x'
i x
O
 O : gốc toạ độ
 i, j : véc tơ đơn vị ( i  j  1 vaø i  j ) y'

Quy ước : Mặt phẳng mà trên đó có chọn hệ trục toạ độ Đề-Các vuông góc Oxy
được gọi là mặt phẳng Oxy và ký hiệu là : mp(Oxy)
II. Toạ độ của một điểm và của một véc tơ:
1. Định nghĩa 1: Cho M  mp(Oxy ) . Khi đó véc tơ OM được biểu diển một cách duy
nhất theo

Q
y
M
i, j bởi hệ thức có dạng : OM  xi  y j vôùi x,y  .
j
i x
Cặp số (x;y) trong hệ thức trên được gọi là toạ độ của điểm M.
x'
O P
Ký hiệu: M(x;y) ( x: hoành độ của điểm M; y: tung độ của điểm M )
y'

ñ/n
M ( x; y)  OM  xi  y j

 Ý nghĩa hình học: y


Q M
y
x' x
O x
P

y' x  OP vaø y=OQ

2. Định nghĩa 2: Cho a  mp(Oxy) . Khi đó véc tơ a được biểu diển một cách duy nhất
theo i, j bởi hệ thức có dạng : a  a1i  a2 j vôùi a1,a2  .

Cặp số (a1;a2) trong hệ thức trên được gọi là toạ độ của véc tơ a . y a

e2
NGUYỄN VĂN LỰC  0933.168.309 SP Toán K35 - ĐH Cần

e
Thơ
x'
1 x
PP tọa độ trong mặt phẳng FB: http://www.facebook.com/VanLuc168

Ký hiệu: a  (a1; a2 )
ñ/n
a=(a1;a2 )  a  a1 i  a2 j

 Ý nghĩa hình học: y


B2 K B

A2 A H
x
x'
O A1 B1 a1  A1B1 vaø a2 =A 2 B2

y'

III. Các công thức và định lý về toạ độ điểm và toạ độ véc tơ :


 Định lý 1: Nếu A( x A ; y A ) vaø B(x B; yB ) thì
AB  ( xB  x A ; yB  y A ) B( x B ; y B )

A( x A ; y A )

 Định lý 2: Nếu a  (a1; a2 ) vaø b  (b1; b2 ) thì 


a
a  b
* ab   1 1 
a2  b2 b

* a  b  (a1  b1; a2  b2 )
* a  b  (a1  b1; a2  b2 )
* k.a  (ka1; ka2 ) (k  )

IV. Sự cùng phương của hai véc tơ:


Nhắc lại
 Hai véc tơ cùng phương là hai véc tơ nằm trên cùng một đường thẳng hoặc
nằm trên hai đường thẳng song song .
 Định lý về sự cùng phương của hai véc tơ:
 Định lý 3 : Cho hai véc tơ a vaø b vôùi b  0


a
a cuøng phöông b  !k  sao cho a  k.b

b

Nếu a  0 thì số k trong trường hợp này được xác định như sau:
b
 k > 0 khi a cùng hướng b  
a a b
k < 0 khi a ngược hướng b
2 5 C
a  b , b- a
NGUYỄN VĂN LỰC  0933.168.309 SP
5 Toán K35
2 - ĐH
B Cần Thơ
PP tọa độ trong mặt phẳng FB: http://www.facebook.com/VanLuc168

a
k 
b

 Định lý 4 : A, B, C thaúng haøng  AB cuøng phöông AC

(Điều kiện 3 điểm thẳng hàng )


 Định lý 5: Cho hai véc tơ a  (a1; a2 ) vaø b  (b1; b2 ) ta có :
a cuøng phöông b  a1.b2  a2 .b1  0

(Điều kiện cùng phương của 2 véc tơ)


V. Tích vô hướng của hai véc tơ:
Nhắc lại: y

 
 B a.b  a . b .cos(a, b) b
b b
2 2
O  a a x' 
x
 A O a
 a
a
ab  a.b  0
y'
 Định lý 6: Cho hai véc tơ a  (a1; a2 ) vaø b  (b1; b2 ) ta có :
a.b  a1b1  a2 b2 (Công thức tính tích vô hướng theo tọa độ)
 Định lý 7: Cho hai véc tơ a  (a1; a2 ) ta có :

a  a12  a22 (Công thức tính độ dài véc tơ )

 Định lý 8: Nếu A( x A ; y A ) vaø B(x B; yB ) thì

AB  ( xB  x A )2  ( yB  y A )2 (Công thức tính khoảng cách 2 điểm)

 Định lý 9: Cho hai véc tơ a  (a1; a2 ) vaø b  (b1; b2 ) ta có


ab  a1b1  a2 b2  0 (Điều kiện vuông góc của 2 véc tơ)
 Định lý 10: Cho hai véc tơ a  (a1; a2 ) vaø b  (b1; b2 ) ta có

a.b a1b1  a2 b2
cos(a, b)   (Công thức tính góc của 2 véc tơ)
a.b a12  a22 . b12  b22

VI. Điểm chia đoạn thẳng theo tỷ số k:


Định nghĩa: Điểm M được gọi là chia đoạn AB theo tỷ số k ( k  1 ) nếu
như : MA  k.MB A M B

  

NGUYỄN VĂN LỰC  0933.168.309 SP Toán K35 - ĐH Cần Thơ


PP tọa độ trong mặt phẳng FB: http://www.facebook.com/VanLuc168

 Định lý 11 : Nếu A( x A ; y A ) , B(x B; yB ) và MA  k.MB ( k  1 ) thì


 x A  k .x B
 x M  1  k

 y  y A  k .yB
 M 1 k

 x A  xB
 x M  2
Đặc biệt : M là trung điểm của AB  
 y  y A  yB
 M 2

VII. Một số điều kiện xác định điểm trong tam giác :
A
 x A  x B  xC
 x G  G
3
1. G laø troïng taâm tam giaùc ABC  GA  GB  GC  0   C
 y  y A  y B  yC
B

 G 3 A

 AH  BC  AH .BC  0
2. H laø tröïc taâm tam giaùc ABC  
H
 C
 BH  AC  BH . AC  0
A B

 AA'  BC
'
3. A laø chaân ñöôøng cao keû töø A   B A'
C

 BA' cuøng phöông BC


A

IA=IB
4. I laø taâm ñöôøng troøn ngoaïi tieáp tam giaùc ABC  
IA=IC I
B C

AB
5. D laø chaân ñöôøng phaân giaùc trong cuûa goùc A cuûa ABC  DB   .DC
AC
A

' AB ' '


6. D laø chaân ñöôøng phaân giaùc ngoaøi cuûa goùc A cuûa ABC  D B  .D C
AC
A
AB C

7. J laø taâm ñöôøng troøn noäi tieáp ABC  JA   .JD B D

BD J

C
B D

NGUYỄN VĂN LỰC  0933.168.309 SP Toán K35 - ĐH Cần Thơ


PP tọa độ trong mặt phẳng FB: http://www.facebook.com/VanLuc168

B. ĐƯỜNG THẲNG

I. Các định nghĩa về VTCP và VTPT (PVT) của đường thẳng:


ñn  a  0

a là VTCP của đường thẳng (  )  
a coù giaù song song hoaëc truøng vôùi ( )

ñn  n  0

n là VTPT của đường thẳng (  )  
 n coù giaù vuoâng goùc vôùi ()

a
 () 
a n

* Chú ý: ( )

 Nếu đường thẳng (  ) có VTCP a  (a1; a2 ) thì có VTPT là n  (a2 ; a1 )


 Nếu đường thẳng (  ) có VTPT n  ( A; B) thì có VTCP là a  (B; A)
II. Phương trình đường thẳng :
1. Phương trình tham số và phương trình chính tắc của đường thẳng :
a. Định lý : Trong mặt phẳng (Oxy). Đường thẳng (  ) qua M0(x0;y0) và nhận
a  (a1; a2 ) làm VTCP sẽ có :
y  x  x0  t.a1
 Phương trình tham số là: () :  (t  )

a  y  y0  t.a2

M ( x; y )

x
O
M 0 ( x0 ; y0 )

x  x 0 y  y0
 Phương trình chính tắc là : () :   a1 , a2  0 
a1 a2

2. Phương trình tổng quát của đường thẳng :


a. Phương trình đường thẳng đi qua một điểm M0(x0;y0) và có VTPT n  ( A; B) là:
y 
n

M ( x; y )
x
O

M 0 ( x0 ; y0 )
() : A( x  x0 )  B( y  y0 )  0 ( A2  B 2  0 )

NGUYỄN VĂN LỰC  0933.168.309 SP Toán K35 - ĐH Cần Thơ


PP tọa độ trong mặt phẳng FB: http://www.facebook.com/VanLuc168

b. Phương trình tổng quát của đường thẳng :


Định lý: Trong mặt phẳng (Oxy). Phương trình đường thẳng (  ) có dạng :


y n  ( A; B )

M 0 ( x0 ; y0 )
x Ax + By + C = 0 với A 2  B 2  0
O

a  ( B; A)

a  ( B; A)

Chú ý:
Từ phương trình (  ):Ax + By + C = 0 ta luôn suy ra được :
1. VTPT của (  ) là n  ( A; B)
2. VTCP của (  ) là a  (B; A) hay a  (B;  A)
3. M0 ( x0 ; y0 )  ()  Ax0  By0  C  0
Mệnh đề (3) được hiểu là : Điều kiện cần và đủ để một điểm nằm trên đường
thẳng là tọa độ điểm đó nghiệm đúng phương trình của đường thẳng .
3. Các dạng khác của phương trình đường thẳng :
a. Phương trình đường thẳng đi qua hai điểm A(xA;yA) và B(xB;yB) :

x  xA y  yA
( AB) :  ( AB) : x  x A ( AB) : y  y A
x B  x A yB  y A

y y
A( x A ; y A ) y B( x B ; y B )
M ( x; y ) B( x B ; y B ) yA A( x A ; y A )
xA xB yA yB
x x x
O
A( x A ; y A )
yB B( x B ; y B )

b. Phương trình đường thẳng theo đoạn chắn:


Định lý: Trong mp(Oxy) phương trình đường thẳng (  ) cắt trục hoành tại điểm
x y
A(a;0) và trục tung tại điểm B(0;b) với a, b  0 có dạng:  1
a b

c. Phương trình đường thẳng đi qua một điểm M0(x0;y0) và có hệ số góc k:


Định nghĩa: Trong mp(Oxy) cho đường thẳng  . Gọi   (Ox ,  ) thì k  tg được
gọi là hệ số góc của đường thẳng 

NGUYỄN VĂN LỰC  0933.168.309 SP Toán K35 - ĐH Cần Thơ


PP tọa độ trong mặt phẳng FB: http://www.facebook.com/VanLuc168

Định lý 1: Phương trình đường thẳng  qua M0 ( x0 ; y0 ) có hệ số góc k là :


y y

y0 M ( x; y )
x
O x0 y - y 0 = k(x - x 0 ) (1)
O  x

Chú ý 1: Phương trình (1) không có chứa phương trình của đường thẳng đi qua M 0
và vuông góc Ox nên khi sử dụng ta cần để ý xét thêm đường thẳng đi qua M0 và
vuông góc Ox là x = x0
Chú ý 2: Nếu đường thẳng  có phương trình y  ax  b thì hệ số góc của đường
thẳng là k  a
Định lý 2: Gọi k1, k2 lần lượt là hệ số góc của hai đường thẳng 1 ,  2 ta có :
 1 //  2  k1  k 2

 1   2  k1.k2  1

c. Phương trình đt đi qua một điểm và song song hoặc vuông góc với một đt cho
trước:
i. Phöông trinh ñöôøng thaúng (1 ) //(): Ax+By+C=0 coù daïng: Ax+By+m1 =0

ii. Phöông trinh ñöôøng thaúng (1 )  (): Ax+By+C=0 coù daïng: Bx-Ay+m 2 =0
Chú ý: m1; m2 được xác định bởi một điểm có tọa độ đã biết nằm trên 1;  2
y  1 : Ax  By  m1  0 y
 1 : Bx  Ay  m 2  0
 : Ax  By  C1  0
x
O x0 x
M1 x0
M1 O

 : Ax  By  C1  0

NGUYỄN VĂN LỰC  0933.168.309 SP Toán K35 - ĐH Cần Thơ


PP tọa độ trong mặt phẳng FB: http://www.facebook.com/VanLuc168

III. Vị trí tương đối của hai đường thẳng :


y y y 1
2 1

x x x
O O O
1
2 2

 1 //  2 1 caét  2 1   2

(1 ) : A1x  B1y  C1  0


Trong mp(Oxy) cho hai đường thẳng :
(2 ) : A2 x  B2 y  C2  0

Vị trí tương đối của (1 ) vaø ( 2 ) phụ thuộc vào số nghiệm của hệ phương trình :
 A1x  B1y  C1  0  A1 x  B1y  C1
 hay  (1)
 A2 x  B2 y  C2  0 A
 2 x  B2 y  C2

Chú ý: Nghiệm duy nhất (x;y) của hệ (1) chính là tọa độ giao điểm M của
(1 ) vaø ( 2 )

Định lý 1:
i. Heä (1) voâ nghieäm  (1 ) //( 2 )
ii. Heä (1) coù nghieäm duy nhaát  (1 ) caét ( 2 )
iii. Heä (1) coù voâ soá nghieäm  (1 )  ( 2 )

Định lý 2: Nếu A2 ; B2 ; C2 khác 0 thì


A1 B1
i. (1 ) caét ( 2 )  
A 2 B2
A1 B1 C1
ii. (1 ) // ( 2 )   
A 2 B2 C2
A1 B1 C1
iii. (1 )  ( 2 )   
A 2 B2 C2

IV. Góc giữa hai đường thẳng


1.Định nghĩa: Hai đường thẳng a, b cắt nhau tạo thành 4 góc. Số đo nhỏ nhất
trong các số đo của bốn góc đó được gọi là góc giữa hai đường thẳng a và b (hay góc
hợp bởi hai đường thẳng a và b). Góc giữa hai đường thẳng a và b đước kí hiệu là
 a, b  . Khi a và b song song hoặc trùng nhau, ta nói rằng góc của chúng bằng 00

NGUYỄN VĂN LỰC  0933.168.309 SP Toán K35 - ĐH Cần Thơ


PP tọa độ trong mặt phẳng FB: http://www.facebook.com/VanLuc168

2. Công thức tính góc giữa hai đường thẳng theo VTCP và VTPT
a) Nếu hai đường thẳng có VTCP lần lượt là u v v thì
u.v
 
cos  a, b   cos u, v 
u.v

b) Nếu hai đường thẳng có VTPT lần lượt là n v n ' thì


n.n '

cos  a, b   cos n, n '   n . n'

(1 ) : A1x  B1y  C1  0


Định lý : Trong mp(Oxy) cho hai đường thẳng :
(2 ) : A2 x  B2 y  C2  0

Gọi  ( 00    900 ) là góc giữa (1 ) vaø ( 2 ) ta có :


y
1 
A1 A2  B1B2
cos 
A12  B12 . A22  B22 x
O

2

Hệ quả:
(1 )  ( 2 )  A1 A2  B1B2  0

V. Khoảng cách từ một điểm đến một đường thẳng :


Định lý 1: Trong mp(Oxy) cho hai đường thẳng () : Ax  By  C  0 và điểm M0 ( x0 ; y0 )
Khoảng cách từ M0 đến đường thẳng ( ) được tính bởi công thức:
M0
y

Ax0  By0  C x
d ( M0 ; )  O
A2  B 2
()

NGUYỄN VĂN LỰC  0933.168.309 SP Toán K35 - ĐH Cần Thơ


PP tọa độ trong mặt phẳng FB: http://www.facebook.com/VanLuc168

C. ĐƯỜNG TRÒN

I. Phương trình đường tròn:


1. Phương trình chính tắc:
Định lý : Trong mp(Oxy). Phương trình của đường tròn (C) tâm I(a;b), bán
kính R là :
y
I (a; b)
b
R M ( x; y )
(C ) : ( x  a)2  ( y  b)2  R2 (1)
x
O a

Phương trình (1) được gọi là phương trình chính tắc của đường tròn
Đặc biệt: Khi I  O thì (C) : x 2  y2  R2

2. Phương trình tổng quát:


Định lý : Trong mp(Oxy). Phương trình: x 2  y2  2ax  2by  c  0 với
a2  b2  c  0 là phương trình của đường tròn (C) có tâm I(a;b), bán kính
R  a2  b2  c

II. Phương trình tiếp tuyến của đường tròn:


Định lý : Trong mp(Oxy). Phương trình tiếp tuyến với đường tròn
(C) : x 2  y2  2ax  2by  c  0 tại điểm M ( x0 ; y0 )  (C ) là :

M 0 ( x0 ; y 0 )
() : x0 x  y0 y  a( x  x0 )  b( y  y0 )  c  0
(C)

( ) I(a;b)

VI. Các vấn đề có liên quan:


1. Vị trí tương đối của đường thẳng và đường tròn:
(C )
(C ) (C )
I
I I
R
R R H
H M
M H M
Định lý:
( ) (C )    d(I;) > R
() tieáp xuùc (C)  d(I;) = R
() caét (C)  d(I;) < R

Lưu ý: Cho đường tròn (C) : x 2  y2  2ax  2by  c  0 và đường thẳng


   : Ax  By  C  0 . Tọa độ giao điềm (nếu có) của (C) và (  ) là nghiệm của hệ
 x 2  y 2  2ax  2by  c  0 (1)
phương trình:  (*)
 Ax  By  C  0 (2)
Cách giải (*): Sử dụng phép thế
+ Rút x hoặc y từ (2) thay vào (1) để được phương trình 1 ẩn.
NGUYỄN VĂN LỰC  0933.168.309 SP Toán K35 - ĐH Cần Thơ
PP tọa độ trong mặt phẳng FB: http://www.facebook.com/VanLuc168

2. Vị trí tương đối của hai đường tròn :


C1 C1 C1 C1
C2 C2
C2
R1 R2 R2
I1 I2 I1 R1 I1 R1 R2 I1 I
I2 I2 2

C2

(C1 ) vaø (C2 ) khoâng caét nhau  I1I2 > R1  R2


(C1 ) vaø (C2 ) caét nhau  R1  R2 < I1I2 < R1  R2
(C1 ) vaø (C2 ) tieáp xuùc ngoaøi nhau  I1I 2 = R1  R2
(C1 ) vaø (C2 ) tieáp xuùc trong nhau  I1I2 = R1  R2

Lưu ý: Cho đường tròn (C) : x 2  y2  2ax  2by  c  0


và đường tròn  C ' : x 2  y 2  2a ' x  2b ' y  c '  0 .
Tọa độ giao điểm (nếu có) của (C) và (C’) là nghiệm của hệ phương trình:
 x 2  y 2  2ax  2by  c  0
 (1)
 2 (*)
 x  y  2a ' x  2b ' y  c '  0

2
(2)
Cách giải (*): Sử dụng phép cộng và phép thế.
+ Trừ vế với vế hai phương trình (1) và (2) để được phương trình 1 ẩn. Từ phương
trình 1 ẩn tìm được rút x hoặc y và thay vào (1) hoặc (2) để tiếp tục được phương
trình 1 ẩn. Giải phương trình nầy ta sẽ được kết quả cần tìm.

NGUYỄN VĂN LỰC  0933.168.309 SP Toán K35 - ĐH Cần Thơ


PP tọa độ trong mặt phẳng FB: http://www.facebook.com/VanLuc168

II. TỌA ĐỘ ĐIỂM – VÉCTƠ


Chuyên đề: Phương pháp tọa độ trong mặt phẳng

Dạng 1: Tìm tọa độ điểm thỏa mãn điều kiện cho trước.

Bài toán tổng quát: Tìm điểm M : ax by c 0 thỏa điều kiện cho trước.

Phương pháp 1

B1. Đặt tọa độ cho điểm M .


am c bm c
M m; ,b 0 hoặc M ;m ,a 0
b a

B2. Khai thác tính chất hình học của điểm M .


+ Tính đối xứng
+ Khoảng cách
+ Góc
+ Quan hệ song song, vuông góc
+ Tính chất của điểm và đường đặc biệt trong tam giác.
+ Tam giác đồng dạng
+ Ba điểm thẳng hàng, hai vectơ cùng phương
Chuyển tính chất hình học sang phương trình với ẩn m . Giải phương trình tìm
m M .

Phương pháp 2

B1. Xem điểm M là giao điểm của hai đường (đường thẳng, đường tròn).

B2. Lập phương trình các đường. Giải hệ tìm M .

NGUYỄN VĂN LỰC  0933.168.309 SP Toán K35 - ĐH Cần Thơ


PP tọa độ trong mặt phẳng FB: http://www.facebook.com/VanLuc168

Ví dụ 1. Cho điểm A  1;3 và đường thẳng  có phương trình x  2y  2  0 . Dựng


hình vuông ABCD sao cho hai đỉnh B, C nằm trên  và các tọa độ đỉnh C đều
dương. Tìm tọa độ các đỉnh B, C, D.

Bài giải
 Đường thẳng (d) đi qua A và vuông góc với  có phương trình: 2x  y  m  0
A  1;3    2  3  m  0  m  1

Suy ra:  d  : 2x  y  1  0

 Tọa độ B là nghiệm của hệ phương trình: 2x  y 1 y 1


x  2y  2  x  0  B 0;1
  
Suy ra: BC  AB  1  4  5


 Đặt C  x 0 ; y0  với x 0 , y0  0 , ta có: C  
BC  5
x  2y0  2  0
  02 2
x  2y0  2
  02
x 0   y0  1  5 x 0   y0  1  5
2

 
Giải hệ này ta được: xy 0  22 hoặc xy 0  02 (loại). Suy ra: C  2; 2 
0 0

 
 Do ABCD là hình vuông nên: CD  BA  xy D  22  311 0  xy D  14  D 1; 4 
D D

 Vậy B  0;1 , C  2; 2  , D 1; 4  

Ví dụ 2. Trong mặt phẳng Oxy, cho tam giác ABC vuông tại A. Biết
và đường thẳng BC đi qua điểm I  2;  . Tìm tọa độ đỉnh C.
1
A  1; 4  , B 1; 4 
 2
Bài giải
 Phương trình đường thẳng BC: 9x  2y  17  0
 AB   2; 8 
9c  17  
 Do C  BC nên ta có thể đặt C  c;  , ta có  AC   c  1; 9c  25 
 2    
  2 
 Theo giả thiết tam giác ABC vuông tại A
9c  25
nên: AB.AC  0  c  1  4. 0c3
2
 Vậy C  3;5  .

Ví dụ 3. Trong mặt phẳng Oxy, cho hình chữ nhật ABCD có diện tích bằng 12,
9 3
I  ;  và tâm của hình chữ nhật là M  3;0  là trung điểm của cạnh AD. Tìm tọa độ
2 2
các đỉnh của hình chữ nhật.
Bài giải
9 9
 Do MI là đường trung bình của tam giác ABD nên AB  2MI  2  3 2
4 4

NGUYỄN VĂN LỰC  0933.168.309 SP Toán K35 - ĐH Cần Thơ


PP tọa độ trong mặt phẳng FB: http://www.facebook.com/VanLuc168
12
 Vì SABCD  AB.AD  12 nên AD   2 2  MA  MD  2
AB

 Đường thẳng AD qua M  3;0  và nhận IM   ;  làm VTPT có phương trình


3 3
2 2
3 3
là:  x  3   y  0   0  x  y  3  0
2 2

 Phương trình đường tròn tâm M bán kính R  2 là:  x  3  y 2  2


2

 Tọa độ A và D là nghiệm của hệ phương trình:


x  y  3  0
 
y  3  x
   
x2 x4

 x  3 2  y2  2   x  3 2  3  x 2  2  y  1  y  1
 
Suy ra: ta chọn A  2;1 , D  4; 1


 Vì I là trung điểm của AC nên: xyC  2y
C
2x I  x A  9  2  7
I  yA  3  1  2
 C  7; 2 

Vì I là trung điểm của BD nên: xy B  2x I  x D  5


 2yI  yD  4
 B  5; 4 
B

 Vậy tọa độ các đỉnh của hình chữ nhật là A  2;1 , B  5; 4  , C  7; 2  , D  4; 1 .

Ví dụ 4. Trong mặt phẳng Oxy, cho tam giác ABC với A  2; 4  , B  0; 2  và trọng tâm
G thuộc đường thẳng 3x  y  1  0 . Hãy tìm tọa độ của C biết rằng tam giác ABC có
diện tích bằng 3.
Bài giải
 Do G là trọng tâm của tam giác ABC nên:
1 1
SGAB  SABC  .3  1
3 3
x2 y4
 Phương trình đường thẳng AB là:   x y2 0
2 2

 Đặt G  a; b  , do G   d  : 3x  y  1  0 nên 3a  b  1  0 , ta có:


1 1
SGAB  1  .AB.d  G, AB   1  .2 2.d  G, AB   1
2 2
1
 d  G, AB  
2
ab2 1
 
2 2
 a  b  2  1

 1
  
a
 Tọa độ G là nghiệm của hệ: 3a  b  1  3a  b  1   2  a  1
a  b  1 a  b  3  1 b  2
b  
 2

NGUYỄN VĂN LỰC  0933.168.309 SP Toán K35 - ĐH Cần Thơ


PP tọa độ trong mặt phẳng FB: http://www.facebook.com/VanLuc168

Suy ra: G   ;   hoặc G  1; 2 


1 1
 2 2

 7
 1 1  x C  3x G   x A  x B    2  7 9
 Với G   ;   thì  9
 C  ; 
 2 2  yC  3y G   y A  y B    2 2
 2

 Với G  1; 2  thì x C  3x G   x A  x B   5  C  5;0 


 yC  3yG   yA  yB   0

 Vậy có hai điểm C thỏa đề bài là : C  5;0  và C   ;  .


7 9
 2 2

Ví dụ 5. Trong mặt phẳng Oxy, cho đường thẳng  d  : x  y  1  0 và đường tròn

 C  : x 2  y2  2x  4y  0 . Tìm tọa độ điểm M thuộc đường thẳng (d) mà qua đó có thể


kẻ được hai tiếp tuyến MA và MB với (C) (A,B là hai tiếp điểm) sao cho AMB  600 .
Bài giải
 (C) có tâm I  1; 2  và bán kính R  5
1
 Theo giả thiết: AMB  600  AMI  AMB  300
2
AI
 Tam giác AMI vuông tại A nên: s in300   IM  2AI  2R  2 5
IM

 Đặt M  t; t  1  (d) , ta có:

IM 2  20   t  1   t  1  20  t 2  9  t  3
2 2

 Vậy có hai điểm cần tìm là M1  3; 2  và M 2  3; 4  .

Ví dụ 6. Trong mặt phẳng Oxy, cho điểm A  0; 2  và đường thẳng  d  : x  2y  2  0 .


Tìm trên đường thẳng (d) hai điểm B, C sao cho tam giác ABC vuông ở B và
AB  2BC .

Bài giải
 Từ yêu cầu của bài toán ta suy ra B là hình chiếu vuông góc của A trên (d)
 Phương trình đường thẳng    qua A và vuông góc với (d) là: 2x  y  m  0
A  0; 2       2  m  0  m  2

Suy ra:    : 2x  y  2  0

 2

  x
 Tọa độ B là nghiệm của hệ phương trình: 2x  y  2 
 5  B 2 ; 6 
x  2y  2 6  
y  5 5
 5
NGUYỄN VĂN LỰC  0933.168.309 SP Toán K35 - ĐH Cần Thơ
PP tọa độ trong mặt phẳng FB: http://www.facebook.com/VanLuc168

 Đặt C  2t  2; t   (d) , theo giả thiết ta có:


AB  2BC  AB2  4BC 2
2
2
 6 
2
 12   6  
2 2

   0     2   4  2t     t   
5  5   5   5  
 2t  12t  7  0
2

t  1
 7
 t  5

 Với t  1  C  0;1

Với t   C  ; 
7 5 7
5 4 5

 Vậy các điểm cần tìm là: B  ;  , C  0;1 hoặc B  ;  , C  ;  .


2 6 2 6 4 7
5 5 5 5 5 5

Ví dụ 7. Trong mặt phẳng với hệ tọa độ Oxy , cho hình chữ nhật ABCD có điểm C
thuộc đường thẳng d : 2 x y 5 0 và A 4;8 . Gọi M là điểm đối xứng của B qua
C , N là hình chiếu vuông góc của B trên đường thẳng MD . Tìm tọa độ điểm B và
C , biết rằng N 5; 4 .

Bài giải
Do C d nên C t; 2t 5 . Gọi I là tâm hình chữ nhật ABCD , suy ra I là trung
điểm của AC .
t 4 2t 3
Do đó: I ;
2 2

Tam giác BDN vuông tại N nên IN IB . Suy ra: IN IA

Do đó ta có phương trình:
2 2 2 2
t 4 2t 3 t 4 2t 3
5 4 4 8 t 1
2 2 2 2

Suy ra: C 1; 7
NGUYỄN VĂN LỰC  0933.168.309 SP Toán K35 - ĐH Cần Thơ
PP tọa độ trong mặt phẳng FB: http://www.facebook.com/VanLuc168

Do M đối xứng với B qua C nên CM CB . Mà CB AD và CM || AD nên tứ giác


ACMD là hình bình
hành. Suy ra AC || DM . Theo giả thiết, BN DM , suy ra BN AC và CB CN . Vậy
B là điểm đối xứng
của N qua AC
Đường thẳng AC có phương trình: 3x y 4 0 .
Đường thẳng BN qua N và vuông góc với AC nên có phương trình: x 3 y 17 0
Do đó: B 3a 17; a
Trung điểm của BN thuộc AC nên:
3a 17 5 a 4
3 4 0 a 7
2 2

Vậy B 4; 7 .

Ví dụ 8. Trong mặt phẳng với hệ tọa độ Oxy , cho hình thang cân ABCD có hai
đường chéo vuông góc với nhau và AD 3BC . Đường thẳng BD có phương trình
x 2 y 6 0 và tam giác ABD có trực tâm là H 3; 2 . Tìm tọa độ các đỉnh C và D .

Bài giải

Gọi I là giao điểm của AC và BD IB IC . Mà IB IC nên IBC vuông cân tại


I ICB 450

BH AD BH BC HBC vuông cân tại B I là trung điểm của đoạn thẳng


HC

Do CH BD và trung điểm I của CH thuộc BD nên tọa độ điểm C thỏa mãn hệ


2 x 3 y 2 0
x 3 y 2
2 6 0
2 2

NGUYỄN VĂN LỰC  0933.168.309 SP Toán K35 - ĐH Cần Thơ


PP tọa độ trong mặt phẳng FB: http://www.facebook.com/VanLuc168

Do đó C 1; 6

IC IB BC 1 CH 10
Ta có ID 3IC CD IC 2 ID 2 IC 10 5 2
ID ID AD 3 2

2 2 t 1
Do D 6 2t; t và CD 5 2 suy ra: 7 2t t 6 50
t 7

Vậy D 4;1 hoặc D 8;7 .

Ví dụ 9. Trong mặt phẳng với hệ tọa độ Oxy , cho tam giác ABC có chân đường cao
17 1
hạ từ đỉnh A là H ; , chân đường phân giác trong của góc A là D 5;3 và trung
5 5
điểm của cạnh AB là M 0;1 . Tìm tọa độ đỉnh C .

Bài giải
Ta có H AH và AH HD nên AH có phương trình: x 2 y 3 0 . Do đó
A 3 2 a; a

Do M là trung điểm của AB nên MA MH


a 3
2 2
Suy ra 3 2a a 1 13 1
a
5

Do A khác H nên A 3;3


Phương trình đường thẳng AD là y 3 0 . Gọi N là điểm đối xứng của M qua
AD . Suy ra N AC và tọa
độ điểm N thỏa mãn hệ
1 y
3 0
2 N 0;5
1.x 0. y 1 0

Đường thẳng AC có phương trình 2 x 3 y 15 0


Đường thẳng BC có phương trình 2 x y 7 0
2x y 7 0
Suy ra tọa độ điểm C thỏa mãn hệ
2 x 3 y 15 0
NGUYỄN VĂN LỰC  0933.168.309 SP Toán K35 - ĐH Cần Thơ
PP tọa độ trong mặt phẳng FB: http://www.facebook.com/VanLuc168

Do đó C 9;11 . 

9 3
Ví dụ 10. Trong mặt phẳng với hệ tọa độ Oxy , cho tam giác ABC có điểm M ;
2 2
là trung điểm của cạnh AB , điểm H 2; 4 và điểm I 1;1 lần lượt là chân đường cao
kẻ từ B và tâm đường tròn ngoại tiếp tam giác ABC . Tìm tọa độ điểm C .

Bài giải
7 1
IM ; . Ta có M AB và AB IM nên đường thẳng AB có phương trình
2 2
7x y 33 0
A AB A a;7a 33 . Do M là trung điểm của AB nên B a 9; 7a 30
a 4
Ta có AH HB AH .HB 0 a2 9a 20 0
a 5
Với a 4 A 4;5 , B 5; 2 . Ta có BH AC nên đường thẳng AC có phương
trình x 2 y 6 0
c 1
Do đó C 6 2c; c . Từ IC
2 2
IA 7 2c c 1 25
c 5
Do C khác A , suy ra C 4;1
Với a 5 A 5; 2 , B 4;5 . Ta có BH AC nên đường thẳng AC có phương
trình 2 x y 8 0
t 1
Do đó C t; 2 t 8 . Từ IC
2 2
IA t 1 2t 7 25
c 5
Do C khác A , suy ra C 1; 6 .

Ví dụ 11. Trong mặt phẳng với hệ tọa độ Oxy , cho đường tròn
4 và đường thẳng : y 3 0 . Tam giác MNP có trực tâm trùng
2 2
C : x 1 y 1
với tâm của C , các đỉnh N và P thuộc , đỉnh M và trung điểm cạnh MN thuộc
C . Tìm tọa độ điểm P .

NGUYỄN VĂN LỰC  0933.168.309 SP Toán K35 - ĐH Cần Thơ


PP tọa độ trong mặt phẳng FB: http://www.facebook.com/VanLuc168

Bài giải
Ta có tâm của C là I 1;1 . Đường thẳng IM vuông góc với nên có phương trình
x 1 . Do đó M 1; a
hoặc a 3 . Mà M nên ta được
2
Do M C nên a 1 4. Suy ra a 1
M 1; 1
2
b 1 b 5
. Trung điểm MN thuộc C
2
N N b;3 1 1 1 4
2 b 3
Do đó N 5;3 hoặc N 3;3
P P c;3
+ Khi N 5;3 , từ MP IN suy ra c 1. Do đó P 1;3
+ Khi N 3;3 , từ MP IN suy ra c 3 . Do đó P 3;3 .

Ví dụ 12. Trong mặt phẳng với hệ tọa độ Oxy , cho hình vuông ABCD . Gọi M là
trung điểm của cạnh BC , N là điểm trên cạnh CD sao cho CN 2 ND . Giả sử
11 1
M ; và đường thẳng AN có phương trình 2 x y 3 0. Tìm tọa độ điểm A .
2 2

Bài giải
Gọi H là giao điểm của AN và BD . Kẻ đường thẳng qua H và song song với AB ,
cắt AD và BC lần lượt
tại P và Q . Đặt HP x. Suy ra PD x, AP 3x và HQ 3x . Ta có QC x, nên
MQ x.

Do đó AHP HMQ , suy ra AH HM

3 10
Hơn nữa, ta cũng có AH HM . Do đó AM 2MH 2d M , ( AN )
2

A AN , suy ra A t; 2t 3 . Khi đó:


2 2
3 10 11 7 45 t 1
MA t 2t t2 5t 4 0
2 2 2 2 t 4

Vậy A 1; 1 hoặc A 4;5 .


NGUYỄN VĂN LỰC  0933.168.309 SP Toán K35 - ĐH Cần Thơ
PP tọa độ trong mặt phẳng FB: http://www.facebook.com/VanLuc168

Ví dụ 13. Trong mặt phẳng với hệ tọa độ Oxy , cho hình chữ nhật ABCD . Các
đường thẳng AC và AD lần lượt có phương trình là x 3 y 0 và x y 4 0 ; đường
1
thẳng BD đi qua điểm M ;1 . Tìm tọa độ các đỉnh hình chữ nhật ABCD .
3

Bài giải
x 3y 0
Tọa độ điểm A thỏa mãn hệ A 3;1
x y 4 0
Gọi N là điểm thuộc AC sao cho MN || AD .
4
Suy ra MN có phương trình là x y 0.
3
4
x y 0 1
Vì N thuộc AC , nên tọa độ điểm N thỏa mãn hệ 3 N 1;
3
x 3y 0
Đường trung trực của MN đi qua trung điểm của MN và vuông góc với AD , nên
có phương trình là
x y 0
Gọi I và K lần lượt là giao điểm của với AC và AD .
x y 0
Suy ra tọa độ của điểm I thỏa mãn hệ I 0;0
x 3y 0
x y 0
và tọa độ điểm K thỏa mãn hệ K 2; 2
x y 4 0
AC 2 AI C 3; 1 ; AD 2 AK D 1;3
BC AD B 1; 3 .

Ví dụ 14. Trong mặt phẳng với hệ tọa độ Oxy , cho đường thẳng : x y 2 0 và
đường tròn C : x 2 y 2 4 x 2 y 0 . Gọi I là tâm của C , M là điểm thuộc . Qua
M kẻ các tiếp tuyến MA và MB đến C ( A và B là các tiếp điểm). Tìm tọa độ điểm
M , biết tứ giác MAIB có diện tích bằng 10 .

Bài giải
Đường tròn C có tâm I 2;1 , bán kính IA 5

Tứ giác MAIB có MAI MBI 900 và MA MB


SMAIB IA.MA MA 2 5 IM IA2 MA2 5

M , có tọa độ dạng M t; t 2
2 2 t 2
MA 5 t 2 t 3 25 2t 2 2t 12 0
t 3

Vậy M 2; 4 hoặc M 3;1 .

NGUYỄN VĂN LỰC  0933.168.309 SP Toán K35 - ĐH Cần Thơ


PP tọa độ trong mặt phẳng FB: http://www.facebook.com/VanLuc168

Ví dụ 15: Trong mặt phẳng với hệ tọa độ Oxy, cho tam giác ABC có M (2; 1) là trung
điểm cạnh AC, điểm H (0;  3) là chân đường cao kẻ từ A, điểm E (23;  2) thuộc đường
thẳng chứa trung tuyến kẻ từ C. Tìm tọa độ điểm B biết điểm A thuộc đường thẳng
d : 2x  3y  5  0 và điểm C có hoành độ dương.

Bài giải
 x  1  3t
A  d : 2x  3 y  5  0    A(3a  1, 2a  1).
 y  1  2t
Vì M (2; 1) là trung điểm AC nên suy ra C (3  3a; 1  2a)

 HA  (3a  1; 2a  4)

 HC  (3  3a; 4  2a).

a  1
Vì AHC  90 nên 0
HA.HC  0  
 a   19 .
 13
+ Với a  1  A(2; 3), C (6;  1) thỏa mãn.
19  18 51 
+ Với a    C ;  không thỏa mãn.
13  13 13 
Với A( 2; 3), C (6;  1) ta có phương trình CE : x  17 y  11  0, phương trình
BC : x  3 y  9  0
3b  7 b  3
Suy ra trung điểm AB là N 
B (3b  9; b)  BC  ;

.
 2 2 
Mà N  CE  b  4  B(3;  4). 

Ví dụ 16: Trong mặt phẳng với hệ tọa độ Oxy, cho tam giác ABC có đỉnh A(3; 3), tâm
đường tròn ngoại tiếp I (2; 1), phương trình đường phân giác trong góc BAC là x  y  0.
8 5
Tìm tọa độ các đỉnh B, C biết rằng BC  và góc BAC nhọn.
5

Bài giải
Vì AD là phân giác trong góc A nên AD cắt đường tròn (ABC) tại E là điểm chính
giữa cung BC  IE  BC.

Vì E thuộc đường thẳng x  y  0 và IE  IA  R  E (0; 0).

Chọn nBC  EI  (2; 1)  pt BC có dạng 2 x  y  m  0.


4 5 3
Từ giả thiết  HC   IH  IC 2  HC 2 
5 5

3 | m5| 3  m  2  BC : 2 x  y  2  0
 d ( I , BC )      
5 5 5  m  8  BC : 2 x  y  8  0.

NGUYỄN VĂN LỰC  0933.168.309 SP Toán K35 - ĐH Cần Thơ


PP tọa độ trong mặt phẳng FB: http://www.facebook.com/VanLuc168

Vì BAC nhọn nên A và I phải cùng phía đối với BC, kiểm tra thấy BC : 2 x  y  2  0 thỏa
mãn.
2 x  y  2  0
 8 6
hoặc B  ;   , C (0; 2) .
8 6
Từ hệ   B(0; 2), C  ;  
( x  2)  ( y  1)  5
 5 5 5 5
2 2

Ví dụ 17: Trong mặt phẳng với hệ tọa độ Oxy, cho tam giác ABC có phương trình
đường thẳng chứa đường cao kẻ từ B là x  3 y  18  0, phương trình đường thẳng trung
trực của đoạn thẳng BC là 3x  19 y  279  0, đỉnh C thuộc đường thẳng d : 2 x  y  5  0.

Tìm tọa độ đỉnh A biết rằng BAC  1350.

Bài giải
B  BH : x  3 y  18  B(3b  18; b),

C  d : y  2 x  5  C (c; 2c  5).

Từ giả thiết suy ra B đối xứng C qua đường trung trực


u .BC  0
 : 3 x  19 y  279  0   
trung ñieåm BC là M  
60b  13c  357 b  4  B(6; 4)
  
10b  41c  409 c  9 C (9; 23).

AC  BH  chọn n AC  u BH  (3; 1)  pt AC : 3x  y  4  0  A(a; 3a  4)


 AB  (6  a; 8  3a), AC  (9  a; 27  3a).

1 (6  a)(9  a)  (8  3a)(27  3a) 1


Ta có A  1350  cos( AB, AC )    
2 (6  a)  (8  3a) . (9  a)  (27  3a)
2 2 2 2 2

(9  a)(3  a) 1 3  a  9

    a  4. Suy ra A(4; 8). 
2(3  a)  a  6a  10

2 2
| 9  a | a 2  6a  10 2

Ví dụ 18: Trong mặt phẳng với hệ tọa độ Oxy, cho hình bình hành ABCD có
phương trình đường chéo AC : x  y  1  0, điểm G (1; 4) là trọng tâm của tam giác ABC,
điểm E (0;  3) thuộc đường cao kẻ từ D của tam giác ACD. Tìm tọa độ các đỉnh của
hình bình hành đã cho biết rằng diện tích của tứ giác AGCD bằng 32 và đỉnh A có
tung độ dương.

Bài giải
Vì DE  AC nên DE : x  y  3  0  D t;  t  3.

NGUYỄN VĂN LỰC  0933.168.309 SP Toán K35 - ĐH Cần Thơ


PP tọa độ trong mặt phẳng FB: http://www.facebook.com/VanLuc168
1 1
Ta có d  G, AC   d  B, AC   d  D, AC 
3 3

1 2t  4 t  1  D 1;  4 
 2 .   .
3 2 t  5  D  5; 2 

Vì D và G nằm khác phía đối với AC nên D 1;  4.


1  1  2. xB  1
Ta có GD  2GB    B 1; 8  BD : x  1.
4  4  2  yB  4 

Vì A  AC : x  y  1  0  A a; a  1.

Ta có S AGCD  S AGC  S ACD    1 S ABC  S ABC  S ABD .


1 4 4
3  3 3

1 a  5  A  5; 6   tm 
Suy ra S ABD  24  .d  A, BD  .BD  24  a  1 .12  48   
2  a  3  A  3;  2   ktm 

Từ AD  BC  C  3;  2.

Vậy A 5; 6 , B 1; 8 , C  3;  2 , D 1;  4 . 

Ví dụ 19: Trong mặt phẳng với hệ tọa độ Oxy, cho hình thang ABCD có AD // BC,
AD  2 BC , đỉnh B (4; 0), phương trình đường chéo AC là 2 x  y  3  0, trung điểm E của
AD thuộc đường thẳng  : x  2 y  10  0. Tìm tọa độ các đỉnh còn lại của hình thang đã
cho biết rằng cot ADC  2.

Bài giải
Gọi I  AC  BE. Vì I  AC  I  t; 2t  3. Ta thấy I là trung điểm của BE nên
E  2t  4; 4t  6 .

Theo giả thiết E    t  3  I 3; 3 , E  2; 6.

Vì AD / / BC, AD  2BC nên BCDE là hình bình hành. Suy ra ADC  IBC.
2
Từ cot IBC  cot ADC  2  cos IBC  .
5

Vì C  AC  C  c; 2c  3  BI  1; 3 , BC  c  4; 2c  3. Ta có
c  5
2 5c  5 2 c  1
cos IBC     2  .
5 10. 5c  20c  25
2 5 3c  22c  35  0 c  7
 3

Suy ra C  5; 7  hoặc C  ; .`


7 5
 3 3

NGUYỄN VĂN LỰC  0933.168.309 SP Toán K35 - ĐH Cần Thơ


PP tọa độ trong mặt phẳng FB: http://www.facebook.com/VanLuc168

Với C  5; 7  , ta thấy I là trung điểm của AC nên A 1;  1 , vì E là trung điểm của AD

nên D  3; 13.

Với C  ;  , tương tự ta có  11 13   1 23 
7 5
A ;  , D  ; . 
3 7  3 3  3 3 

4 
Ví dụ 20: Trong mặt phẳng tọa độ Oxy , cho tam giác ABC có trọng tâm G  ; 1, trung
3 
điểm BC là M (1; 1), phương trình đường thẳng chứa đường cao kẻ từ B là x  y  7  0.
Tìm tọa độ A, B, C.

Bài giải
Từ tính chất trọng tâm ta có MA  3MG  A(2; 1).
B  BH : y   x  7  B (b,  b  7).

Vì M (1; 1) là trung điểm BC nên C ( 2  b; b  5). Suy ra AC  (b; b  6).


BH  AC nên uBH .AC  0  b  (b  6)  0  b  3.
Suy ra B (3; 4), C ( 1;  2). 

Ví dụ 21: Trong mặt phẳng tọa độ Oxy , cho tam giác ABC . Đường cao kẻ từ A, trung
tuyến kẻ từ B , trung tuyến kẻ từ C lần lượt nằm trên các đường thẳng có phương
trình x  y  6  0, x  2 y  1  0, x  1  0 . Tìm tọa độ A, B, C.
Bài giải
x  2 y  1  0
Từ hệ  suy ra trọng tâm G (1; 1).
x 1  0

A AH , B BM , C CN A(a; 6 a ), B (2b 1; b), C (1; c).

a  (2b  1)  1  3 a  2b  3
Do G (1; 1) là trọng tâm nên   (1)
(6  a)  b  c  3  a  b  c  3

Ta có u AH  (1;  1), BC  (2  2b; c  b). Vì AH  BC nên u AH .BC  0

 2  2b  c  b  0  b  c  2 (2)
Từ (1) và (2) suy ra a  5, b  1, c  3. Suy ra A(5; 1), B (3;  1), C (1; 3). 

Ví dụ 22: Trong mặt phẳng tọa độ Oxy, cho tam giác ABC vuông cân tại A, phương
trình BC : 2 x  y  7  0, đường thẳng AC đi qua điểm M (1; 1), điểm A nằm trên
đường thẳng  : x  4 y  6  0. Tìm tọa độ các đỉnh của tam giác ABC biết rằng đỉnh
A có hoành độ dương.
NGUYỄN VĂN LỰC  0933.168.309 SP Toán K35 - ĐH Cần Thơ
PP tọa độ trong mặt phẳng FB: http://www.facebook.com/VanLuc168

Bài giải
Vì A   : x  4 y  6  0  A(4a  6; a)  MA(4a  5; a  1).
Vì tam giác ABC vuông cân tại A nên ACB  450.

1 (4a  5)  2(a  1) 1
Do đó cos( MA, u BC )   
2 (4a  5) 2  (a  1) 2 . 5 2

a  2  A(2; 2)
 13a  42a  32  0  
2
   14 16 
a  16  A  ;  (ktm )
 13   13 13 

Vậy A(2; 2). Suy ra AC : x  3 y  4  0, AB : 3x  y  8  0. Từ đó ta có B(3;  1), C (5; 3). 

Ví dụ 23: Trong mặt phẳng tọa độ Oxy, cho tam giác ABC cân tại A nội tiếp đường
tròn (C): x 2  y 2  2 x  4 y  1  0. Tìm tọa độ các đỉnh A, B, C biết điểm M (0;1) là
trung điểm cạnh AB và điểm A có hoành độ dương.

Bài giải
Đường tròn (C) có tâm I (1; 2), bán kính IA  2.

Ta có IM (1; 1), IM AB suy ra phương trình đường thẳng AB : x  y  1  0.

A  AB  A(a; a  1). Khi đó


IA  2  (a  1) 2  (a  1) 2  4  a 2  1  a  1 (do a  0) . Suy ra A(1; 2); B(1; 0).
Ta có IA (2; 0), IA BC suy ra phương trình BC : x  1  0, phương trình AI : y  2  0.
Gọi N là giao điểm của AI và BC. Suy ra N (1; 2) và N là trung điểm BC. Suy ra
C (1; 4). 

Ví dụ 24: Trong mặt phẳng tọa độ Oxy , cho tam giác ABC ; phương trình các đường
thẳng chứa đường cao và đường trung tuyến kẻ từ đỉnh A lần lượt là
x  2 y  13  0 và 13x  6 y  9  0. Tìm tọa độ các đỉnh B và C biết tâm đường tròn
ngoại tiếp tam giác ABC là I (5 ; 1).

Bài giải
Ta có A(3;  8). Gọi M là trung điểm BC  IM // AH .
Ta suy ra pt IM : x  2 y  7  0. Suy ra tọa độ M thỏa mãn
x  2 y  7  0
  M (3; 5).
13x  6 y  9  0

Pt đường thẳng BC : 2( x  3)  y  5  0  2 x  y  11  0. B  BC  B(a; 11  2a).


NGUYỄN VĂN LỰC  0933.168.309 SP Toán K35 - ĐH Cần Thơ
PP tọa độ trong mặt phẳng FB: http://www.facebook.com/VanLuc168

a  4
Khi đó IA  IB  a 2  6a  8  0   .
a  2

Từ đó suy ra B(4; 3), C (2; 7) hoặc B(2; 7), C (4; 3). 

Ví dụ 25: Trong mặt phẳng tọa độ Oxy , cho tam giác ABC có trọng tâm G (1; 1);
đường cao từ đỉnh A có phương trình 2 x  y  1  0 và các đỉnh B, C thuộc đường thẳng
 : x  2 y  1  0. Tìm tọa độ các đỉnh A, B, C biết diện tích tam giác ABC bằng 6.
Bài giải
1 3
Tọa độ chân đường cao H ( ; ).
5 5

1 7
Đường thẳng d đi qua G và song song BC có pt d : x  2 y  3  0. d  AH  I  I ( ; ).
5 5

Ta có
HA  3HI  A(1; 3).

6 2S ABC
d ( A, BC )  . Suy ra BC   2 5.
5 d ( A, BC )

Gọi M là trung điểm BC. Khi đó MA  3MG  M (1; 0).

 x1  1 x  3
Gọi B ( x1 ; ). Khi đó MB  5  ( x1  1) 2  4   1
2  x1  1.

+ Với x1  3  B (3;  1)  C (1; 1).

+ Với x1  1  B (1;1)  C (3;  1).

Suy ra A(1; 3), B(3;  1), C (1; 1) hoặc A(1; 3), B(1; 1), C (3;  1). 

Ví dụ 26: Trong mă ̣t phẳ ng Oxy cho hin


̀ h thang ABCD có đáy lớn CD = 3AB,
C(–3; –3), trung điể m của AD là M(3; 1). Tìm to ̣a đô ̣ đỉnh B biế t SBCD = 18, AB= 10
và đỉnh D có hoành đô ̣ nguyên dương.

Bài giải
Go ̣i n = (A; B) là vectơ pháp tuyế n của CD (A2 + B2 > 0)
Ta có CD: A(x + 3) + B(y + 3) = 0  Ax + By + 3A + 3B = 0.
Ta có: SBCD = SACD = 18
2SACD 36 6 10 3 10
 d(A; CD) =    d(M; CD) =
CD 3 10 5 5
3A  B  3A  3B 3 10
   5 6A  4B  3 10 A2  B2
2
A B 2 5
 25(36A2 + 48AB + 16B2) = 90(A2 + B2)
NGUYỄN VĂN LỰC  0933.168.309 SP Toán K35 - ĐH Cần Thơ
PP tọa độ trong mặt phẳng FB: http://www.facebook.com/VanLuc168
B 31B
 810A2 + 1200AB + 310B2 = 0  A   hay A   .
3 27
B
* A   : Cho ̣n B = –3  A = 1  (CD): x – 3y – 6 = 0  D(3d + 6; d)
3
Ta có: CD2 = 90  (3d + 9)2 + (d + 3)2 = 90  (d + 3)2 = 9  d = 0 hay d=–6
 D(6; 0) (nhâ ̣n) hay D(–12; –6) (loa ̣i). Vâ ̣y D(6; 0)  A(0; 2)
1
Ta có AB  DC  (3; 1)  B(–3; 1).
3
31B
* A : Cho ̣n B = –27  A = 31  CD: 31x – 27y + 12 = 0
27
2
31d  12   31d  93  729
 D  d;   CD  (d  3)   27   90  (d  3)  169 (loa ̣i)
2 2 2

 27   
Vâ ̣y B(–3; 1).

Ví dụ 27: Trong mặt phẳng với hệ tọa độ Oxy cho hình chữ nhật ABCD có diện tích
bằng 22 , đường thẳng AB có phương trình 3x  4 y  1  0 , đường thẳng BD có phương
trình 2x  y  3  0 . Tìm toạ độ các đỉnh A, B, C , D.

Bài giải
Điểm B là giao giữa AB và BD  B 1; 1

S ABCD  AB. AD  22 (1) . Đường thẳng AB có vtpt n1   3; 4  , AC có vtpt n2   2; 1

n1 . n2 2 11 AD
cos ABD  cos  n1 ; n2     tan ABD   (2)
n1 n2 5 5 2 AB

từ (1),(2)  AD  11 , AB  2 (3)
11a  11
D  BB  D  a;2a  3 , AD  d  D;  AB    (4) . Từ (3) & (4) suy ra
5

11a  11  55  a  6 , a  4

 3 1 7 
a  6  D  6;9  . Do AD  AB  AD : 4 x  3 y  3  0  A   ;  , I  ;4  trung điểm của BD . C
 5 5  2 

 38 39 
đối xứng A qua I  C ; 
 5 5 

 13 11   28 49 
a  4  D(4; 11) tương tự trên ta tính được A  ;   & C   ;   .
5 5  5 5 

Ví dụ 28: Trong mặt phẳng với hệ trục tọa độ Oxy , hãy viết phương trình các cạnh
của tam giác ABC biết trực tâm H 1;0  , chân đường cao hạ từ đỉnh B là K  0;2  , trung
điểm cạnh AB là M  3;1 .

NGUYỄN VĂN LỰC  0933.168.309 SP Toán K35 - ĐH Cần Thơ


PP tọa độ trong mặt phẳng FB: http://www.facebook.com/VanLuc168

Bài giải
Đường thẳng AC vuông góc với HK nên nhận HK   1;2  làm véc tơ pháp tuyến và
AC qua K  0;2 

nên  AC  : 1 x  0   2  y  2   0   AC  : x  2 y  4  0   BK  : 2 x  y  2  0
Gọi A  2a  4; a   AC , B  b;2  2b   BK mặt khác M  3;1 là trung điểm AB nên ta có hệ

 2a  4  b  6 2a  b  10 a  4  A  4;4 
   
a  2  2b  2 a  2b  0 b  2  B  2; 2 

 AB  qua A  4; 4  và có AB   2; 6    AB  : 3x  y  8  0

 BC  qua B  2; 2  và vuông góc với AH nên nhận HA  3;4 làm véc tơ pháp tuyến
  BC  : 3  x  2   4  y  2   0   BC  : 3 x  4 y  2  0 .

NGUYỄN VĂN LỰC  0933.168.309 SP Toán K35 - ĐH Cần Thơ


PP tọa độ trong mặt phẳng FB: http://www.facebook.com/VanLuc168

III. PHƯƠNG TRÌNH ĐƯỜNG THẲNG


Chuyên đề: Phương pháp tọa độ trong mặt phẳng

Yêu cầu
1) Nắm vững chắc tất cả các dạng phương trình đường thẳng trong mặt phẳng tọa
độ
2) Đặc biệt lưu ý dạng thường sử dụng sau:
Phương trình đường thẳng đi qua một điểm M0(x0;y0) và có VTPT n  ( A; B) là:
y 
n

M ( x; y )

x
O

M 0 ( x0 ; y0 )
() : A( x  x0 )  B( y  y0 )  0 ( A2  B 2  0 )

Ví dụ 1. Trên mặt phẳng tọa độ Oxy, hãy viết phương trình các đường thẳng chứa
các cạnh của tam giác ABC biết A 1;6  và hai đường trung tuyến nằm trên hai đường

thẳng có phương trình là x  2y  1  0,3x  y  2  0 .


Bài giải
 Do tọa độ điểm A không nghiệm đúng các phương trình đã cho nên ta có thể
giả sử rằng:
Phương trình trung tuyến BM là: x  2y  1  0
Phương trình trung tuyến CN là: 3x  y  2  0
b6
 Đặt B  2b  1; b  , do N là trung điểm AB nên : N  b; 

 2 

 b6 b6
N  b;   CN  3b  2  0  b  2
 2  2

Suy ra: B  3; 2 
c  1 3c  4 
 Đặt C  c;3c  2  , do M là trung điểm AC nên : M  ; 
 2 2 

 c  1 3c  4  c 1 3c  4
M ;   BM   2.  1  0  c  1
 2 2  2 2

NGUYỄN VĂN LỰC  0933.168.309 SP Toán K35 - ĐH Cần Thơ


PP tọa độ trong mặt phẳng FB: http://www.facebook.com/VanLuc168

Suy ra: C  1; 5 


AB : 11x  2y  1  0
 Vậy phương trình AB, BC, AC là: BC : 7x  4y  13  0 .
AC : 2x  y  8  0

Ví dụ 2. Trong mặt phẳng Oxy, cho điểm M  6; 2  và đường tròn (C) có phương trình
 x  1   y  2 Lập phương trình đường thẳng (d) đi qua M và cắt đường tròn
2 2
 5.
(C) tại hai điểm A, B sao cho AB  10 .

Bài giải

 Đường tròn (C) có tâm I 1; 2  và bán kính R  5

 Gọi H là hình chiếu vuông góc của I trên AB, ta có:


AB2 10 5 10
IH 2  IA 2  AH 2  R 2   5    IH 
4 4 2 2

 Đường thẳng (d) đi qua M và có VTPT n   a;b  có dạng:


a  x  6   b  y  2   0  ax  by  6a  2b  0

 Đường thẳng (d) thỏa đề bài khi:


a  2b  6a  2b 10
d  I;(d)   IH    9a 2  b 2  b  3a
a b
2 2 2

+ Với b  3a ta được  d  : x  3y  0

+ Với b  3a ta được  d  : x  3y  12  0 .

Ví dụ 3: Trong mặt phẳng Oxy, cho tam giác ABC có đường phân giác trong
 AD  : x  y  0 , đường cao  CH  : 2x  y  3  0 , cạnh AC qua M  0; 1 , AB  2AM .

Viết phương trình ba cạnh của tam giác ABC.


Bài giải
 Gọi N là điểm đối xứng của M qua AD. Suy ra: N  tia AB
Mặt khác ta có: AN  AM  AB  2AN  N là trung điểm của AB
 Do MN  AD nên phương trình MN là: x  y  m1  0
M  0; 1  MN  1  m1  0  m1  1

Suy ra:  MN  : x  y  1  0

NGUYỄN VĂN LỰC  0933.168.309 SP Toán K35 - ĐH Cần Thơ


PP tọa độ trong mặt phẳng FB: http://www.facebook.com/VanLuc168

 Gọi K  MN AD , tọa độ K là nghiệm của hệ pt:


 1

x
x  y  1   2  K 1 ; 1 
xy0  1  
y    2 2
 2

Vì K là trung điểm của MN nên: xy N  2y


N

2x K  x M  1
K  yM  0
 N  1;0 

 Do AB  CH nên phương trình AB là: x  2y  m 2  0


N  1;0   AB  1  m 2  0  m 2  1

Suy ra:  AB  : x  2y  1  0

 Vì A  AB AD nên tọa độ A là nghiệm của hệ pt: xx  2y


y0 y 1 
 1  x  1  A 1;1
  
Suy ra:  AC  : 2x  y  1  0

 Vì C  AC CH nên tọa độ C là nghiệm của hệ pt:


 1
2x  y  1   x    C   1 ; 2 
2x  y  3  2 
 2


 y  2

 Do N là trung điểm của AB  xy B  2y


B

2x N  x A  3
N  y A  1
 B  3; 1

 Phương trình cạnh BC:  BC  : 2x  5y  11  0 .

Ví dụ 4: Trong mặt phẳng Oxy, cho tam giác ABC có các đỉnh A  1; 2  . Trung tuyến
CM : 5x  7y  20  0 và đường cao BH : 5x  2y  4  0 . Viết phương trình các cạnh
AC và BC.

Bài giải
 Do AC  BH nên phương trình AC là: 2x  5y  m  0
A  1; 2   AC  2  10  m  0  m  8

Suy ra:  AC  : 2x  5y  8  0

 Do C  AC CM nên tọa độ C là nghiệm của hệ pt:

5x2x  7y5y  820  xy  04  C  4;0


 Đặt B  a; b  , do B  BH nên: 5a  2b  4  0

NGUYỄN VĂN LỰC  0933.168.309 SP Toán K35 - ĐH Cần Thơ


PP tọa độ trong mặt phẳng FB: http://www.facebook.com/VanLuc168

1  a 2  b
 Vì M là trung điểm của AB nên tọa độ M là : M  ;


 2 2 

1  a 2  b  1  a 2b
 Do M  ;   CM  5.  7.  20  0  5a  7b  31  0
 2 2  2 2

 Tọa độ M là nghiệm của hệ: 5a 


 2b  4  a  2  B 2;3
5a  7b  31 b3   
 Phương trình cạnh BC là:  BC  : 3x  2y  12  0 .

Ví dụ 5: Trong mặt phẳng tọa độ Oxy , cho đường tròn (C ) : x 2  y 2  4 x  2 y  15  0. Gọi


I là tâm đường tròn (C ). Đường thẳng  đi qua M (1;  3) cắt (C ) tại hai điểm A
và B. Viết phương trình đường thẳng  biết tam giác IAB có diện tích bằng 8 và
cạnh AB là cạnh lớn nhất.

Bài giải
Đường tròn (C) có tâm I (2;  1), bán kính R  2 5. Gọi H là trung điểm AB. Đặt
AH  x (0  x  2 5 ).

Khi đó ta có
1 x  4
IH . AB  8  x 20  x 2  8   nên AH  4  IH  2.
2  x  2 (ktm vì AB  IA)

Pt đường thẳng qua M: a( x  1)  b( y  3)  0 (a 2  b 2  0)


 ax  by  3b  a  0.

| a  2b | 4
Ta có d ( I , AB)  IH  2   2  a(3a  4b)  0  a  0  a  b.
a2  b2 3

* Với a  0 ta có pt  : y  3  0.
4
* Với a  b. Chọn b  3 ta có a  4 . Suy ra pt  : 4 x  3 y  5  0.
3

Vậy có hai đường thẳng  thỏa mãn là y  3  0 và 4 x  3 y  5  0. 

Ví dụ 6: Trong mặt phẳng tọa độ Oxy cho tam giác cân ABC có đáy BC nằm trên
đường thẳng d :2 x  5 y  1  0 , cạnh AB nằm trên đường thẳng d  :12 x  y  23  0 . Viết
phương trình đường thẳng AC biết nó đi qua điểm M  3;1 .

Bài giải
VTPT của BC : nBC   2; 5  , VTPT của AB : nAB  12; 1 ,

VTPT của AC : nAC   a; b  ,  a 2  b 2  0  . Ta có ABC  ACB  900

NGUYỄN VĂN LỰC  0933.168.309 SP Toán K35 - ĐH Cần Thơ


PP tọa độ trong mặt phẳng FB: http://www.facebook.com/VanLuc168

 cos ABC  cos ACB  cos  nAB , nBC   cos  nBC , nCA 

nAB .nBC nCA .nBC 145 2a  5b


     9a 2  100ab  96b2  0
nAB . nBC nCA . nBC 5 a b
2 2

 a  12b  0  9a  8b  0

+ Với a  12b  0 Cho ̣n a  12, b  1 thì nCA  12; 1  AB AC ( loa ̣i)
+ Với 9a  8b  0 Cho ̣n a  8, b  9 nên AC : 8  x  3  9  y  1  0
Vậy AC : 8 x  9 y  33  0 .

Ví dụ 7: Trong mặt phẳng tọa độ Oxy cho đường tròn T  : x 2  y 2  x  9 y  18  0 và


hai điểm A  4;1 , B  3; 1 . Gọi C , D là hai điểm thuộc T  sao cho ABCD là một hình
bình hành. Viết phương trình đường thẳng CD .

Bài giải
2 2
1 9
Ta có T  :  x     x    nên T  có tâm
1 9 10 10
I  ;  bán kính R 
 2  2 4 2 2 2

AB   1; 2  , AB  5 , và  AB  : 2 x  y  7  0 .
Đường thẳ ng CD AB   CD  : 2 x  y  m  0 ( điề u kiê ̣n m  7 )

2m  7 5  2m  7 
2

Khoảng cách từ I đế n CD là h  và CD  2 R  h  2 


2 2

2 5 2 20

5  2m  7  m  6
2

CD  AB  2   5   2m  7   25  
2
Ta có thỏa mañ
2 20 m  1

+ m  6 pt  CD  : 2 x  y  6  0
+ m  1 pt  CD  : 2 x  y  1  0
Có hai đường thẳ ng thỏa mãn : 2 x  y  6  0; 2 x  y  1  0 .

NGUYỄN VĂN LỰC  0933.168.309 SP Toán K35 - ĐH Cần Thơ


PP tọa độ trong mặt phẳng FB: http://www.facebook.com/VanLuc168

BÀI TẬP TỰ LUYỆN

Câu 1. Trong mặt phẳng với hệ toạ độ Oxy cho tam giác ABC có A 1; 4  , tiếp tuyến
tại A của đường tròn ngoại tiếp tam giác ABC cắt BC tại D , đường phân giác trong
của ADB có phương trình x  y  2  0 , điểm M  4;1 thuộc cạnh AC . Viết phương
trình đường thẳng AB .

E
M'
K
M
B I C D

Gọi AI là phân giác trong của BAC


Ta có : AID  ABC  BAI
IAD  CAD  CAI
Mà BAI  CAI , ABC  CAD nên AID  IAD
 DAI cân tại D  DE  AI
PT đường thẳng AI là : x  y  5  0
Goị M’ là điểm đối xứng của M qua AI  PT đường thẳng MM’ : x  y  5  0
Gọi K  AI  MM '  K(0;5)  M’(4;9)
VTCP của đường thẳng AB là AM '  3;5  VTPT của đường thẳng AB là n   5; 3
Vậy PT đường thẳng AB là: 5  x  1  3  y  4   0  5 x  3 y  7  0

Câu 2. Trong mặt phẳng với hệ tọa độ Oxy, cho tam giác ABC cân, cạnh đáy BC có
phương trình: x  y  1  0 , phương trình đường cao kẻ từ B là: x  2 y  2  0 . Điểm
M(2;1) thuộc đường cao kẻ từ C. Viết phương trình các cạnh bên của tam giác ABC.

1
Gọi H là trực tâm  ABC. Tìm được B(0;-1), cos HBC   cos HCB
10
Pt đthẳng HC có dạng: a(x-2)+b(y-1)=0( n  (a; b) là VTPT và a 2  b 2  0 )
ab
2
1 a a
cos HCB    4a 2  10ab  4b 2  0  2    5    2  0
2(a 2  b 2 ) 10 b b

NGUYỄN VĂN LỰC  0933.168.309 SP Toán K35 - ĐH Cần Thơ


PP tọa độ trong mặt phẳng FB: http://www.facebook.com/VanLuc168

a
 b  2  a  2, b  1
  , phương trình CH: -2x + y + 3 = 0
a   1  a  1, b  2(l )
 b 2
AB  CH. Tìm được pt AB:x+2y+2=0
2 5
Tìm được : C ( ;  ) ,pt AC:6x+3y+1=0
3 3

Câu 3. Trong mặt phẳng với hệ tọa độ Oxy,cho hình vuông ABCD có M(1;2) là
trung điểm AB, N(-2;1) là điểm thuộc đoạn AC sao cho AN=3NC. Viết phương trình
của đường thẳng CD

3a 2
Ta có MN= 10 , AN=3AC/4=
4
2 2 2 0 5a 2
MN =AM +AN -2AM.AN.cos45 =
8
 a=4
Gọi I(x;y) là trung điểm của CD.Ta có
 IM  4  x  1, y  2
 
 BD  17 6
 IN  4  2  x  5 , y   5

+Đường thẳng CD đi qua I(1;-2) có pt : y+2=0


+ Đường thẳng CD đi qua I(17/5;-6/5) có pt : 3x-4y-15=0

Câu 4. Trong mặt phẳng với hệ tọa độ Oxy, cho C  5;4  , đường thẳng d : x  2 y  11  0
đi qua A và song song với BC, đường phân giác trong AD có phương trình
3 x  y  9  0 . Viết phương trình các cạnh của tam giác ABC.

Tìm được A 1;6  , AC : x  2 y  13  0 , BC : x  2 y  3  0 .


Từ C kẻ đường thẳng vuông góc AD , cắt AD tại I , cắt AB tại J . Khi đó tam giác
ACJ cân tại A .
Phương trình đường thẳng CI : x  3 y  7  0  I  2;3 , J  1;2  , phương trình đường
thẳng AB : 2 x  y  4  0 .

NGUYỄN VĂN LỰC  0933.168.309 SP Toán K35 - ĐH Cần Thơ


PP tọa độ trong mặt phẳng FB: http://www.facebook.com/VanLuc168

Câu 5. Trong mặt phẳng với hệ tọa độ Oxy, cho tam giác ABC có phương trình cạnh
AB: x - y - 2 = 0, phương trình cạnh AC: x + 2y - 5 = 0. Biết trọng tâm của tam giác
G(3; 2). Viết phương trình cạnh BC.

x - y - 2  0
Tọa độ điểm A là nghiệm của HPT:   A(3; 1)
x  2 y - 5  0
Gọi B(b; b- 2)  AB, C(5- 2c; c)  AC
3  b  5  2c  9 b  5
Do G là trọng tâm của tam giác ABC nên   .
1  b  2  c  6 c  2
Hay B(5;3), C(1;2)
Một vectơ chỉ phương của cạnh BC là u  BC  (4; 1) .
Phương trình cạnh BC là: x - 4y + 7 = 0.

Câu 6. Trong mặt phẳng với hệ tọa độ Oxy, Cho hình thang cân ABCD với hai đáy
AD, BC. Biết B(2; 3) và AB  BC , đường thẳng AC có phương trình x  y  1  0 , điểm
M  2; 1 nằm trên đường thẳng AD. Viết phương trình đường thẳng CD.

B C
Vì ABCD là hình thang cân nên nội tiếp trong một
H
đường tròn. Mà BC  CD nên AC là đường phân giác
của góc BAD .
D
Gọi B ' là điểm đối xứng của B qua AC.
A M
B'

Khi đó B '  AD .
Gọi H là hình chiếu của B trên AC. Tọa độ điểm H là
nghiệm của hệ phương trình:
x  y 1  0 x  3
  . Suy ra H  3; 2  .
x  y  5  0 y  2
Vì B’ đối xứng với B qua AC nên H là trung điểm của BB’. Do đó B '  4;1 .
Đường thẳng AD đi qua M và nhận MB ' làm vectơ chỉ phương nên có phương
trình
x  3 y  1  0 . Vì A  AC  AD nên tọa độ điểm A là nghiệm của hệ phương trình:
x  y 1  0 x  1
  . Do đó, A 1; 0  .
x  3 y 1  0  y  0
Ta có ABCB’ là hình bình hành nên AB  B ' C . Do đó, C  5; 4  .
Gọi d là đường trung trực của BC, suy ra d : 3x  y  14  0 .
Gọi I  d  AD , suy ra I là trung điểm của AD. Tọa độ điểm I là nghiệm của hệ:
3x  y  14  0
. Suy ra, I  ;  . Do đó, D  ;  .
43 11 38 11

x  3 y 1  0  10 10   5 5
Vậy, đường thẳng CD đi qua C và nhận CD làm vectơ chỉ phương nên có phương
trình 9 x  13 y  97  0 . (Học sinh có thể giải theo cách khác)

NGUYỄN VĂN LỰC  0933.168.309 SP Toán K35 - ĐH Cần Thơ


PP tọa độ trong mặt phẳng FB: http://www.facebook.com/VanLuc168

Câu 7. Trong mă ̣t phẳ ng tọa độ Oxy, cho 2 đường tròn (C1) và (C2) lần lượt có
phương trình là ( x  1)2  ( y  4)2  10, x 2  y 2  6 x  6 y  13  0 . Viết phương trình đường
25
thẳng  qua M(2;5) cắt hai đường tròn (C), (C’) lần lượt tại A, B sao cho S I MA  S I MB
1
12 2
biết rằng phương trình đường thẳng  có hệ số nguyên (I1,I2 lần lượt là tâm của (C1)
và (C2)).

 (C1) có tâm I1(-1;4), bán kính R1 = 10


 (C1) có tâm I1(3;3), bán kính R2 = 5
 Dễ kiểm tra được: M là một giao điểm của (C 1),(C2)
  qua M nên  : a (x  2)  b(y  5)  0, (a, b  Z, a 2  b 2  0)
 Gọi H, K lần lượt là hình chiếu của I1,I2 lên 
3a  b a  2b
Ta có: IH  d I ;  ; IK  d I2 ;  
a b
2 2
a 2  b2
1

25 1 25
 Ta có: S I MA  S I 2 MB  I1 H .MA  I 2 K .MB  12 I1 H .2MH  25I 2 K .2MK
1
12 2 24
 12.I1H . I1M 2  I1H 2  25I 2 K. I 2 M 2  I 2 K 2
 12.I1H . 10  I1H 2  25I 2 K. 5  I 2 K 2
 144 I1H 2 10  I1H 2   625I 2 K 2  5  I 2 K 2 

 | 3a  b |    | 3a  b |    | a  2b |    | a  2b |  
2 2 2 2

 144   10   2    625  2  5   2  


 a  b    a  b    a  b    a  b  
2 2 2 2 2

 144  3a  b   a  3b   625  a  2b   2a  b 
2 2 2 2

12  3a  b  a  3b   25  a  2b  2a  b 

12  3a  b  a  3b   25  a  2b  2a  b 
  a 2 a a a 1
2    3  2  0   2   ( n )
14a  21ab  14b  0
2 2
b

b b b 2
 
  a 171  2975
86a  171ab  86b  0
2 2 2
86  a   171 a  86  0  b  (loai do a,b  Z)
  b  b 172
a
+ Với  2 , chọn a = 2, b= -1   : 2 x  y  1  0
b
a 1
+ Với  , chọn a = 1, b= 2   : x  2 y  12  0
b 2
Có hai có hai đường thẳng thỏa điều kiện bài toán là 2x–y+1=0, x+2y–12 = 0

Câu 8. Trong mặt phẳng với hệ tọa độ Oxy cho đường thẳng  : y  2  0 và các điểm
A(0;6), B (4; 4) . Viết phương trình tổng quát của đường thẳng AB. Tìm tọa độ điểm C
trên đường thẳng  sao cho tam giác ABC vuông tại B.
x0 y6 x y6
Phương trình đường thẳng AB là:   
40 46 2 1
 x  2 y  12  x  2 y  12  0.
C   C (t;2)  BA(4;2), BC (t  4; 2)
Tam giác ABC vuông tại B nên BA.BC  0  4t  16  4  0  t  3  C(3;2).
NGUYỄN VĂN LỰC  0933.168.309 SP Toán K35 - ĐH Cần Thơ
PP tọa độ trong mặt phẳng FB: http://www.facebook.com/VanLuc168

Câu 9. Cho hình thang cân ABCD có AB // CD, CD = 2AB. Gọi I là giao điểm của
hai đường chéo AC và BD. Gọi M là điểm đối xứng của I qua A với M  ;  . Biết
2 17
3 3 
phương trình đường thẳng DC : x + y – 1= 0 và diện tích hình thang ABCD bằng 12.
Viết phương trình đường thẳng BC biết điểm C có hoành độ dương.

M
A B
H

D C

Ta có : tam giác MDC vuông tại D


 (MD) : x – y + 5 = 0
 D(-2; 3)
8 2 3
MD =  HD =MD = 2 2
3 4
3a.2 2
Gọi AB = a  SABCD = = 12  a = 2 2
2
 DC = 4 2
Gọi C(c; 1 –c )  DC2 = 2(c + 2 )2  c = 2 hay c = -6 (loại)  C(2; -1)
 B(3; 2)
 (BC): 3x – y – 7 = 0

Câu 10. Trong mặt phẳng với hệ tọa độ Oxy cho đường tròn (C1 ) : x2  y 2  13 và
đường tròn (C2 ) : ( x  6)2  y 2  25 cắt nhau tại A(2; 3). Viết phương trình đường thẳng
đi qua A và lần lượt cắt (C1 ), (C2 ) theo hai dây cung phân biệt có độ dài bằng nhau.
Gọi giao điểm thứ hai của đường thẳng cần tìm với (C 1) và (C2) lần lượt là M và N.

Gọi M ( x; y )  (C1 )  x2  y 2  13 (1)


Vì A là trung điểm của MN nên N (4  x; 6  y )
Do N  (C2 )  (2  x)2  (6  y)2  25 (2)
 x  2

 y  3
 x 2  y 2  13  17 6
Từ (1) và (2) ta có hệ     x   17  M( ; )
(2  x)  (6  y )  25 
 
2 2
5 5 5
 6
 y 
 5

Đường thẳng cần tìm đi qua A và M có phương trình: x  3 y  7  0

NGUYỄN VĂN LỰC  0933.168.309 SP Toán K35 - ĐH Cần Thơ


PP tọa độ trong mặt phẳng FB: http://www.facebook.com/VanLuc168

Câu 11. Trong mặt phẳng tọa độ Oxy, cho hình thang ABCD với AB//CD có diện
1 1 1
tích bằng 14, H ( ;0) là trung điểm của cạnh BC và I ( ; ) là trung điểm của AH.
2 4 2
Viết phương trình đường thẳng AB biết đỉnh D có hoành độ dương và D thuộc đường
thẳng d: 5 x  y  1  0 .

A B
I

D M
C

13
Vì I là trung điểm của AH nên A(1;1); Ta có: AH  .
2

Phương trình AH là: 2 x  3 y  1  0 .Gọi M  AH  CD thì H là trung điểm của AM

Suy ra: M(-2; -1). Giả sử D(a; 5a+1) (a>0). Ta có:

28
ABH  MCH  S ABCD  S ADM  AH .d ( D, AH )  14  d ( D, AH ) 
13

Hay 13a  2  28  a  2(vì a  0)  D(2;11)

1
Vì AB đi qua A(1;1) và có 1VTCP là MD  (1;3)  AB có 1VTPT là n(3; 1) nên AB
4
có pt là: 3x  y  2  0 .

NGUYỄN VĂN LỰC  0933.168.309 SP Toán K35 - ĐH Cần Thơ


PP tọa độ trong mặt phẳng FB: http://www.facebook.com/VanLuc168

IV. PHƯƠNG TRÌNH ĐƯỜNG TRÒN


Chuyên đề: Phương pháp tọa độ trong mặt phẳng

Ví dụ 1: Trong mặt phẳng với hệ tọa độ Oxy, cho hai điểm A(1; 2), B(4; 1) và đường
thẳng  : 3x  4 y  5  0. Viết phương trình đường tròn đi qua A, B và cắt  tại C, D sao
cho CD  6.

Bài giải
Giả sử (C) có tâm I (a; b), bán kính R  0.

Vì (C) đi qua A, B nên IA  IB  R


 (a  1) 2  (b  2) 2  (a  4) 2  (b  1)2  R

b  3a  6
  I (a; 3a  6)

 
 R  10a  50a  65
  R  10a  50a  65

2 2
(1)

9a  29
Kẻ IH  CD tại H. Khi đó CH  3, IH  d ( I , ) 
5

(9a  29)2
 R  IC  CH 2  IH 2  9  (2)
25

(9a  29)2
Từ (1) và (2) suy ra 10a 2  50a  65  9   169a 2  728a  559  0
25

a  1  I (1;  3), R  5

    43 51 
 a  43 I  ; , R 
5 61
 13   13 13  13
2 2

hoặc (C ) :  x 
43   51  1525
Suy ra (C ) : ( x  1) 2  ( y  3) 2  25   y    .
 13   13  169

Ví dụ 2: Trong mặt phẳng tọa độ Oxy, cho đường thẳng  : 5 x  2 y  19  0 và đường


tròn (C ) : x 2  y 2  4 x  2 y  0. Từ một điểm M nằm trên đường thẳng  kẻ hai tiếp tuyến
MA, MB đến đường tròn (C ) (A và B là hai tiếp điểm). Viết phương trình đường tròn
ngoại tiếp tam giác AMB biết rằng AB  10.

Bài giải
Đường tròn (C) có tâm I (2; 1), bán kính R  5. Gọi H  MI  AB. Ta có
1 10
AH  AB  .
2 2

NGUYỄN VĂN LỰC  0933.168.309 SP Toán K35 - ĐH Cần Thơ


PP tọa độ trong mặt phẳng FB: http://www.facebook.com/VanLuc168

Trong tam giác vuông MAI (tại A) với đường cao AH ta có


1 1 1 1 4 1
2
 2
 2
 2
   AM  5  MI  10 .
AH AI AM AM 10 5

x 5 y 3
Ta có  : 5 x  2 y  19  0   :   M (5  2m; 3  5m)
2 5
3
Khi đó MI  10  (3  2m) 2  (2  5m) 2  10  29m 2  32m  3  0  m  1 hoặc m .
29

Chú ý rằng, đường tròn ngoại tiếp tam giác AMB là đường tròn đường kính MI.
+ Với m  1 ta có M (3;  2). Khi đó pt đường tròn ngoại tiếp AMB là
2 2
 5  1 5
x    y    .
 2  2 2

3  139 72 
+ Với m ta có M ; . Khi đó pt đt ngoại tiếp AMB là
29  29 29 
2 2
 197   101  5
x   y   .
 58   58  2

Ví dụ 3:
Trong mặt phẳng với hệ tọa độ Oxy , cho hai điểm A 1; 2  ; B  3; 4  và đường thẳng
d : y  3  0. ,Viết phương trình đường tròn  C  đi qua hai điểm A, B và cắt đường thẳng

d tại hai điểm phân biệt M , N sao cho MAN  600 .

Bài giải
Gọi  C  : x 2  y 2  2ax  2by  c  0 (đk a 2  b 2  c  0)
 A 1; 2    C  5  2a  4b  c  0 b  5  a
  
 B  3; 4    C   25  6a  8b  c  0 c  15  2a

Vậy  C  có tâm I  a; a  5  , bán kính R  a2   5  a   15  2a   2  a 2  4a  5


2

 C  cắt đường thẳng d tại hai điểm phân biệt M , N sao cho MAN  600 . Suy ra
1
MIN  1200  I MN  I NM  300 hạ IH   d   IH  d  I , d   R
2

2  a 2  4a  5   a 2  4a  3  0  a  1  a  3
1
 2a 
2

+ Khi a  1 ta có đường tròn  C  : x 2  y 2  2 x  8 y  13  0 ( loại do I , A khác phía đường


thẳng d )
+ Khi a  3   C  : x 2  y 2  6 x  4 y  9  0   C  :  x  3   y  2   4 (t/ mãn). 
2 2

NGUYỄN VĂN LỰC  0933.168.309 SP Toán K35 - ĐH Cần Thơ


PP tọa độ trong mặt phẳng FB: http://www.facebook.com/VanLuc168

Ví dụ 4:
Trong mặt phẳng với hệ trục tọa độ Oxy , cho hai đường tròn  C1  : x 2  y 2  10 x  0 ,

 C2  : x 2  y 2  4 x  2 y  20  0 . Viết phương trình đường tròn  C  đi qua các giao điểm của
 C1  ,  C2  và có tâm nằm trên đường thẳng  d  : x  6 y  6  0 .

Bài giải
Toạ độ giao điểm của  C1  và  C2  là nghiệm của hệ phương trình
 x 2  y 2  10 x  0
 2 
 x 2  y 2  10 x  0
  
 2

50 x  3 x  2  0
 x  y  4 x  2 y  20  0 7 x  y  10  0
2
 y  7 x  10

 x  1 , x  2  x  1  y  3  A1 1 ; 3
  vậy 2 giao điểm của  C1  và  C2  là 
 y  7 x  10 x  2  y  4  A2  2;4 

Trung điểm A của A1 A2 có toạ độ A  3 ; 1  , ta có A1 A2  1;7  đường thẳng qua A vuông


2 2

góc với A1 A2 có phương trình    : 1. x  3   7. y  1   0     : x  7 y  5  0


 2  2

 x  7 y  5  0  x  12
Toạ độ tâm I của hai đường tròn cần tìm là nghiệm của hệ  
 x  6 y  6  0  y  1

 I 12; 1 . Đường tròn cần tìm có bán kính R  IA2   2  12 2   4  12 5 5

Vậy đường tròn cần tìm có phương trình  C  :  x  122   y  12  125 .

BÀI TẬP TỰ LUYỆN

Câu 1. Trong mặt phẳng với hệ tọa độ Oxy,cho 2 đường thẳng d: 3 x+y=0 và
d’: 3 x-y=0. Gọi (C) là đường tròn tiếp xúc với d tại A,cắt d’ tại 2 điểm B,C sao cho
tam giác ABC vuông tại B. Viết phương trình của (C) biết diện tích tam giác ABC
3
bằng và A có hành độ dương.
2

Ta thấy đường tròn (C ) là đường tròn ngoại tiếp tam giác vông ABC,có đường kính
AC
Điểm A thuộc d nên A(a;-a 3 ) (a>0).
+Đường thẳng AB đi qua A và vuông góc với d’ có pt: x+ 3 y+2a=0

NGUYỄN VĂN LỰC  0933.168.309 SP Toán K35 - ĐH Cần Thơ


PP tọa độ trong mặt phẳng FB: http://www.facebook.com/VanLuc168

 a a 3
Do đó B là giao điểm của AB với d’ .khi đó B   ;  
 2 2 
+ Đường thẳng AC đi qua A và vuông góc với d có pt: x- 3 y-4a=0
Do đó C là giao điểm của AC với d’ .khi đó C  2a; 2a 3 
1 3 1
Ta lại có S ABC = AB.BC  =>a=
2 2 3

Vậy A  ; 1 , C   ; 2 
1 2
 3   3 

Do đó đường tròn (C ) có tâm I    1 3


;   là trung điểm của AC và bán kính
 2 3 2
R=IA=1
2 2

Vậy pt của( C):  x 


1   3
  y  2  1
 2 3  

Câu 2. Trong mặt phẳng với hệ trục tọa độ Oxy, cho hai đường thẳng d1: x  y  5  0 ,
d2: x  2 y – 7  0 và tam giác ABC có A(2; 3), trọng tâm là điểm G(2; 0), điểm B thuộc
d1 và điểm C thuộc d2 . Viết phương trình đường tròn ngoại tiếp tam giác ABC.

Do B  d1 nên B(m; – m – 5), C  d2 nên C(7 – 2n; n)


2  m  7  2n  3.2 m  1
Do G là trọng tâm ABC nên    B(–1; –4), C(5; 1)
3  m  5  n  3.0 n  1
 PT đường tròn ngoại tiếp ABC: x 2  y 2  83 x  17 y  338  0
27 9 27

Câu 3. Trong mặt phẳng tọa độ Oxy cho tứ giác ABCD nội tiếp đường tròn (S), có A
và C đối xứng qua BD. Phương trình AB: y – 2 = 0; phương trình BD: 3x  y  2  0 .

Viết phương trình đường tròn (S) biết diện tích tứ giác ABCD bằng 4 3 và xA > 0,
y A  yD .

+B là giao điểm của AB và BD, tìm được B(0; 2).


+Tính góc giữa hai đường thẳng AB và BD bằng 600.
+Ta có BD là đường trung trực của dây cung AC nên BD
là đường kính.
+Tam giác ABD vuông tại A có ABD  600  AD  AB 3
1
+Ta có S ABCD  2SABD  SABD  2 3  AB. AD  2 3
2

NGUYỄN VĂN LỰC  0933.168.309 SP Toán K35 - ĐH Cần Thơ


PP tọa độ trong mặt phẳng FB: http://www.facebook.com/VanLuc168
1

AB 2 . 3  2 3  AB  2.
2
+Ta có A  AB  A  a;2 , a  0, AB   a;0 

AB  2   a   02  2  a  2 (a  0) suy ra A  2; 2  .
2

+Ta có D  BD  D  d ; 3d  2  , AD   d  2; 3d  .
 d  1
Nên AD  AB 3   d  2 2   3d   2 3  4d 2  4d  8  0  
2

d  2

Suy ra 

 D 1;  3  2  . Vì y < yD nên chọn D  2; 2 3  2  .
 
A
 D 2; 2 3  2

+ Đường tròn (S) có tâm I 1; 3  2  , bán kính IA  2 nên có phương trình:
 x 1  
2
 y 32  4.
2

Câu 4. Trong mặt phẳng toạ độ Oxy cho 3 đường thẳng d1 : x  2 y  6  0 ; d 2 : x  2 y  0


và d3 : 3x  y  2  0 . Viết phương trình đường tròn (C) có tâm I thuộc d 3, cắt d1 tại A và
B, cắt d2 tại C và D sao cho tứ giác ABCD là hình vuông.

Gọi I(a; 3a – 2)
Vì ABCD là hình vuông  d(I, AB) = d(I, CD) = d

7a - 10 7a - 4
 =
5 5
3
 a = 1  I(1;1)  d =
5
3 2
Bán kính: R = d 2 =
5
 pt(C):  x - 12 +  y - 12 = 18
5

NGUYỄN VĂN LỰC  0933.168.309 SP Toán K35 - ĐH Cần Thơ


PP tọa độ trong mặt phẳng FB: http://www.facebook.com/VanLuc168

Câu 5. Trong mặt phẳng với hệ toạ độ Oxy, cho đường thẳng d : x  y  1  0 và hai
đường tròn: (C1 ) : x2  y 2  6 x  8 y  23  0 ; (C2 ) : x2  y 2  12 x 10 y  53  0 . Viết phương
trình đường tròn (C) có tâm thuộc đường thẳng d, tiếp xúc trong với đường tròn
(C1 ) tiếp xúc ngoài với đường tròn (C2 ).

+) (C1 ) có tâm I1 (3; 4) , bán kính R1  2 ; (C2 ) có tâm I1 (3; 4) ,bán kính R2  2 2 .

+) Gọi I là tâm, R là bán kính của đường tròn (C). I  d  I (a; a  1) .

+) (C) tiếp xúc trong với (C1 )  II1  R  R1 (1) .

+) (C) tiếp xúc ngoài với (C2 )  II 2  R  R2  R  II 2  R2 (2) .

+) TH1: R  R1 , (1)  R  II1  R1 , từ (1) và (2) ta có: II1  R1  II 2  R2


 (a  3)2  (a  3) 2  2  (a  6) 2  (a  6) 2  2 2  a  0
 I (0; 1); R  4 2  PT đường tròn (C): x 2  ( y  1) 2  32.
+) TH2: R  R1 , (1)  R  R1  II1 , từ (1) và (2) ta có: R1  II1  II 2  R2
 2  (a  3) 2  (a  3) 2  (a  6) 2  (a  6) 2  2 2  a 2  9  a 2  36  3 (vô ng)
+) KL: …

Câu 6. Trong mặt phẳng với hệ toạ độ Oxy , cho tam giác ABC có A(4; 6), phương
trình đường cao và trung tuyến kẻ từ đỉnh C lần lượt là 2 x  y  13  0 và
6 x  13 y  29  0 . Viết phương trình đường tròn ngoại tiếp tam giác ABC.

- Gọi đường cao và trung tuyến kẻ từ C là CH và CM.


Khi đó

CH có phương trình 2 x  y  13  0 ,
CM có phương trình 6 x  13 y  29  0.
2 x  y  13  0
- Từ hệ   C (7;  1).
6 x  13 y  29  0

NGUYỄN VĂN LỰC  0933.168.309 SP Toán K35 - ĐH Cần Thơ


PP tọa độ trong mặt phẳng FB: http://www.facebook.com/VanLuc168

- AB  CH  n  u AB CH
 (1, 2)
 pt AB : x  2 y  16  0 .

 x  2 y  16  0
- Từ hệ   M (6; 5)  B (8; 4).
6 x  13 y  29  0
- Giả sử phương trình đường tròn ngoại tiếp ABC : x 2  y 2  mx  ny  p  0.
52  4m  6n  p  0 m  4
 
- Vì A, B, C thuộc đường tròn nên 80  8m  4n  p  0  n  6 .
50  7 m  n  p  0  p  72
 
Suy ra pt đường tròn: x 2  y 2  4 x  6 y  72  0 hay ( x  2) 2  ( y  3) 2  85.

Câu 7. Trong mặt phẳng với hệ tọa độ Oxy, cho hai đường thẳng d 1: x – 2y + 3 = 0,
d2 : 4x + 3y – 5 = 0. Lập phương trình đường tròn (C) có tâm I trên d 1, tiếp xúc d2 và
có bán kính R = 2.

 x  3  2t
d1:  , I  d1  I (3  t ; t )
y  t
27 7
d(I , d2) = 2  11t  17  10  t  , t
11 11
2 2
27  21 27   21   27 
 t=  I1  ;  (C1 ) :  x     y    4
11  11 11   11   11 
  19 7 
2 2
7  19   7
 t=  I2 ;  (C 2 ) :  x     y    4
11  11 11   11   11 

Câu 8. Trong mặt phẳng với hệ tọa độ Oxy, cho điểm E(3; 4), đường thẳng
d : x  y  1  0 và đường tròn (C ) : x 2  y 2  4 x  2 y  4  0 . Gọi M là điểm thuộc đường

thẳng d và nằm ngoài đường tròn (C). Từ M kẻ các tiếp tuyến MA, MB đến đường
tròn (C) (A, B là các tiếp điểm). Gọi (E) là đường tròn tâm E và tiếp xúc với đường
thẳng AB. Tìm tọa độ điểm M sao cho đường tròn (E) có chu vi lớn nhất.

Đường tròn (C) có tâm I (2;1) , bán kính R  3 . Do M  d nên M (a;1  a ) .


Do M nằm ngoài (C) nên IM  R  IM 2  9  (a  2) 2  (a) 2  9
 2a 2  4a  5  0 (*)
Ta có MA 2  MB 2  IM 2  IA 2  (a  2) 2  (a) 2  9  2a 2  4a  5
Do đó tọa độ của A, B thỏa mãn phương trình: ( x  a) 2  ( y  a  1) 2  2a 2  4a  5
 x 2  y 2  2ax  2(a  1) y  6a  6  0 (1)
Do A, B thuộc (C) nên tọa độ của A, B thỏa mãn phương trình
x 2  y 2  4 x  2 y  4  0 (2).
NGUYỄN VĂN LỰC  0933.168.309 SP Toán K35 - ĐH Cần Thơ
PP tọa độ trong mặt phẳng FB: http://www.facebook.com/VanLuc168

Trừ theo vế của (1) cho (2) ta được (a  2) x  ay  3a  5  0 (3)


Do tọa độ của A, B thỏa mãn (3) nên (3) chính là phương trình của đường thẳng  đi
qua A, B.
+) Do (E) tiếp xúc với  nên (E) có bán kính R1  d ( E, )
Chu vi của (E) lớn nhất  R1 lớn nhất  d ( E ,  ) lớn nhất
Nhận thấy đường thẳng  luôn đi qua điểm K  ; 
5 11
2 2 
10
Gọi H là hình chiếu vuông góc của E lên   d ( E, )  EH  EK 
2
Dấu “=” xảy ra khi H  K    EK .
Ta có EK    ;  ,  có vectơ chỉ phương u  (a; a  2)
1 3
 2 2
1 3
Do đó   EK  EK.u  0   a  (a  2)  0  a  3 (thỏa mãn (*))
2 2
Vậy M  3;4 là điểm cần tìm.

Câu 9. Trong mặt phẳng 0xy cho đường tròn (C): . Viết pt
đường tròn (C’) tâm M(5;1) biết (C’) cắt (C) tại A, B sao cho AB= và bán kính
của nó lớn hơn 4.

Từ pt đường tròn (C) Tâm I(1;-2) và R= . Đường tròn (C’) tâm M cắt đường
tròn tại A, B nên AB tại trung điểm H của AB.

Nhận xét : Tồn tại 2 vị trí của AB (hình vẽ) là AB, A’B’ chúng có cùng độ dài là
Các trung điểm H, H’ đối xứng nhau qua tâm I và cùng nằm trên đường thẳng IM.
Ta có : IH’=IH=
Mà nên MH=MI-HI= ; MH’=MI+IH’=
loại)

Vậy (C’) : =43.


NGUYỄN VĂN LỰC  0933.168.309 SP Toán K35 - ĐH Cần Thơ
PP tọa độ trong mặt phẳng FB: http://www.facebook.com/VanLuc168

Câu 10. Trong mặt phẳng tọa độ với hệ trục tọa độ Oxy, cho hai đường thẳng có
phương trình lần lượt là d1 : x  2 y  2  0, d 2 : 3x  3 y  6  0 và tam giác ABC đều có
diện tích bằng 3 và trực tâm I thuộc d1 . Đường thẳng d 2 tiếp xúc với đường tròn nội
tiếp tam giác ABC. Tìm tọa độ giao điểm d1 và đường tròn ngoại tiếp tam giác ABC
biết điểm I có hoành độ dương.

Gọi M  AI  BC . Giả sử AB  x( x  0), R, r lần lượt là bán kính đường tròn ngoại tiếp,
nội tiếp tam giác ABC
x2 3 x2 3
-Do tam giác ABC đều nên S ABC   3 x2
4 4
-Do tam giác ABC đều nên trực tâm I là tâm đường tròn ngoại tiếp , nội tiếp tam giác
1 1 3
ABC  r  IM  AM  3 .
3 3 3
Giả sử I (2a  2; a)  d1 (a  1)
Do d 2 tiếp xúc với đường tròn nội tiếp tam giác ABC nên
3(2a  2)  3a  6  62 6
3  a  1(l )
d (I ; d2 )  r    3a  6  6  6  3
99 3 
 a  2
Suy ra I (2; 2) .
2 2 3
Đường tròn ngoại tiếp tam giác ABC có tâm I và bán kính R  AM 
3 3
 phương trình đường tròn (C) ngoại tiếp tam giác ABC
4
là : ( x  2) 2  ( y  2) 2 
3
Giao điểm của đường thẳng (d1 ) và (C ) là nghiệm của hệ phương trình:
x  2 y  2  0

 4
( x  2)  ( y  2)  3
2 2

2 4 2 4
Vậy giao điểm của (d1 ) và ( d 2 ) là E (2  ;2  ), F (2  ;2  ).
15 15 15 15

NGUYỄN VĂN LỰC  0933.168.309 SP Toán K35 - ĐH Cần Thơ


GTLN – GTNN FB: http://www.facebook.com/VanLuc168

CHUYÊN ĐỀ: ỨNG DỤNG ĐẠO HÀM ĐỂ TÌM


GTLN VÀ GTNN CỦA HÀM SỐ NHIỀU BIẾN

A. PHƯƠNG PHÁP CHUNG

Để giải bài toán tìm GTLN, GTNN của hàm số nhiều biến bằng phương pháp hàm số,
thông thường ta thực hiện theo các bước sau :
 Biến đổi các số hạng chứa trong biểu thức về cùng một đại lượng giống nhau.
 Đưa vào một biến mới t, bằng cách đặt t bằng đại lượng đã được biến đổi như
trên.
 Xét hàm số f (t ) theo biến t . Khi đó ta hình thành được bài toán tương đương sau
: Tìm giá trị lớn nhất, giá trị nhỏ nhất của hàm số f (t ) với t  D .
 Lúc này ta sử dụng đạo hàm để tìm giá trị lớn nhất, giá trị nhỏ nhất của hàm số
f (t ) với t  D .
 Chú ý : trong trường hợp không thể xây dựng trực tiếp được hàm số f (t ) với
t  D , ta có thể đi tìm
 f (t ) với t  D thỏa P  f (t ) đối với bài toán tìm giá trị nhỏ nhất
 f (t ) với t  D thỏa P  f (t ) đối với bài toán tìm giá trị lớn nhất.

B. MỘT SỐ BÀI TOÁN MINH HỌA

I. XÂY DỰNG TRỰC TIẾP HÀM SỐ f (t ) BẰNG CÁC BIẾN ĐỔI ĐẠI SỐ:
Phương pháp chung:
 Dự đoán khả năng dấu bằng xảy ra hoặc giá trị đặc biệt trong điều kiện để đặt
được biến phụ t thích hợp.
 Có thể biến đổi được về hàm f(t) không cần sử dụng tính chất bất đẳng thức.
 Hàm f(t) tương đối khảo sát được.
 Chú ý phần tìm điều kiện của t (phải thật chính xác)
 Thích hợp cho các đề thi khối B và D.

Thí dụ 1. Cho x, y là các số thực dương thỏa mãn x + y = 1.


 1  2 1 
Tìm GTNN của biểu thức P   x 2   y  2 
 y2   x 

Lời giải.
1
 Ta biến đổi P   xy  
2
2
( xy ) 2

NGUYỄN VĂN LỰC  0933.168.309 SP Toán K35 - ĐH Cần Thơ


GTLN – GTNN FB: http://www.facebook.com/VanLuc168

 x, y  0 1
 Do  nên 1  x  y  2 xy  0  xy  .
x  y  1 4

 Đặt t  xy 2 , điều kiện của t là 0  t 


1
16
 Khi đó biểu thức P  f t   2  t 
1
t
t 2 1
f ' t   ta thấy f ' t   0 với mọi t   0;  , suy ra hàm số f(t) nghịch biến trên
1
 ;
t2  16 
nửa khoảng  0; 
1
 16 
 1  289
 Vậy giá trị nhỏ nhất của biểu thức P là min P  min1 f t   f    .
t( 0; ]  16  16
16

Thí dụ 2. (Khối A 2006) Cho các số thực x  0, y  0 thỏa ( x  y ) xy  x 2  y 2  xy .


1 1
Tìm GTLN của biểu thức A  3
 3 .
x y

Lời giải.
S2
 Đặt x  y  S và xy  P với P  0 , từ giả thiết ta có P   S  3
S 3
4S 2 4 S 1
 x, y tồn tại khi S 2  4 P  S 2   1  0  S  3  S  1
S 3 S 3 S 3
2
x 3  y 3 ( x  y )( x 2  y 2  xy) ( x  y ) 2 xy  x  y   S  3
2

 Ta biến đổi A 3 3        
 S 
3 3 3 3
x y x y x y  xy 
t 3 3
 Xét hàm số f (t )  với t  3  t  1 , ta có f / (t )   2  0
t t
 BBT

t -∞ -3 1 +∞
_ _
f /(t)
1 4
f(t)
0 1

 Suy ra A  f 2 (t )  16
1
 Vậy GTLN P  16 khi x  y  .
2

Thí dụ 3. Cho các số thực dương thay đổi x, y thỏa điều kiện x  y  1 .
1 1
Tìm GTNN của biểu thức P   .
x y
3 3
xy

Lời giải.
NGUYỄN VĂN LỰC  0933.168.309 SP Toán K35 - ĐH Cần Thơ
GTLN – GTNN FB: http://www.facebook.com/VanLuc168

1 1 1 1 1 1
 P     
x y
3 3
xy ( x  y )  3xy( x  y ) xy 1  3xy xy
3

x y
2
1
 Đặt 0  t  xy    
 2  4
1 1 1
 Xét hàm số f (t )   với 0  t 
1  3t t 4
3 1 3 3
f / (t )   2  f / (t )  0  t 
(1  3t ) 2
t 6
 BBT
3- 3 1
t 0
6 4
f /(t) _ +
0
+∞ 8
f(t)
3 3 
 Suy ra P  f    42 3

4+2 3
 6 

1 2 3  3  1 2 3  3 
 Vậy GTLN P  4  2 3 khi x  1  ; y  1  .
2 3  2 3 
   

Thí dụ 4. (khối D 2009) Cho các số thực không âm x, y thỏa điều kiện x  y  1 .
Tìm GTLN và GTNN của biểu thức S  (4 x 2  3 y )(4 y 2  3x)  25 xy

Lời giải.
 Do x  y  1 nên S  (4 x 2  3 y )(4 y 2  3 x)  25 xy
 16 x 2 y 2  12( x 3  y 3 )  9 xy  25 xy
 
 16 x 2 y 2  12 ( x  y ) 3  3 xy( x  y )  34 xy
 16 x y  2 xy  12
2 2

x y
2

 Đặt 0  t  xy  
1
 
 2  4
1
 Xét hàm số f (t )  16t 2  2t  12 với 0  t 
4
1
f / (t )  32t  2  f / (t )  0  t 
16

1 1
t 0 16 4
f /(t) _ 0 +
12 25
f(t) 191 2
16

NGUYỄN VĂN LỰC  0933.168.309 SP Toán K35 - ĐH Cần Thơ


GTLN – GTNN FB: http://www.facebook.com/VanLuc168

25 1
 Vậy GTLN S  khi x  y 
2 2
191 2 3 2 3 2 3 2 3
GTNN S khi x  ,y  hoặc x  ,y  .
16 4 4 4 4

Thí dụ 5. Cho các số thực thay đổi x, y thỏa điều kiện y  0 và x 2  x  y  12 .


Tìm GTLN, GTNN của biểu thức P  xy  x  2 y  17 .

Lời giải.
 Ta có x 2  x  12  y  0  4  x  3
 P  x( x 2  x  12)  x  2( x 2  x  12)  17  x 3  3x 2  9 x  7
 Xét hàm số f ( x)  x 3  3x 2  9 x  7 với  4  x  3
f / ( x)  3x 2  6 x  9  f / ( x)  0  x  3; x  1

x -4 -3 1 3
f /(x) + 0 - 0 +
20 20
f(x)
-13 -12

 Vậy GTLN P  20 khi x  3, y  6 hoặc x  3, y  0


GTNN P  12 khi x  1, y  10 

Thí dụ 6. Cho các số thực x  0 và y  0 thỏa x  y  2 .


x 2  xy  y 2  x  3
Tìm giá trị nhỏ nhất của biểu thức P  .
3 x  xy  1

Lời giải.
 x0

  y0 0 x2
x  y  2

x 2  x(2  x)  (2  x) 2  x  3 x 2  x  1
 P  2
3x  x(2  x)  1 x  x 1

2x 2  2
P/ 
( x 2  x  1) 2

x 0 1 2
P/ - 0 +

P 1
3

NGUYỄN VĂN LỰC  0933.168.309 SP Toán K35 - ĐH Cần Thơ


GTLN – GTNN FB: http://www.facebook.com/VanLuc168

1
 Vậy GTNN P  khi x  1; y  1 .
3

Thí dụ 7. Cho các số thực thay đổi x, y thỏa điều kiện x  y  1 , x 2  y 2  xy  x  y  1.


xy
Tìm GTLN, GTNN của biểu thức P  .
x  y 1

Lời giải.
 Từ giả thiết x 2  y 2  xy  x  y  1  xy  ( x  y) 2  ( xy)  1
2 t 2  t 1
 Đặt t  x  y , ta có ( x  y ) 2  4 xy  3t 2  4t  4  0    t  2 . Khi đó P 
3 t 1
t 2  t 1 2
 Xét hàm số f (t )  với   t  2
t 1 3
t  2t
2
t  2
f / (t )   f / ( x)  0  
(t  2)  t 0
2

-2
t 0 2
3
f /(t) _ +
0
1 1
f(t) 3 3
-1

1 1
 Vậy GTLN P  khi x  y   hoặc x  y  1
3 3
GTNN P  1 khi x  1, y  1 hoặc x  1, y  1 .

Thí dụ 8. Cho các số thực thay đổi x, y thỏa điều kiện x, y  0 ,


1 1
xy ( x  y )  x 2  y 2  x  y  2 . Tìm GTLN của biểu thức P   .
x y

Lời giải.
 Từ giả thiết suy ra xy( x  y )  ( x  y ) 2  2 xy  ( x  y )  2
t2 t  2
 Đặt t  x  y suy ra xy 
t2
t 3  2t 2  4t  8
 Ta có ( x  y ) 2  4 xy   0  t  2  2  t
t2
x y t 2  2t
 Khi đó P   2
xy t t 2
t 2  2t
 Xét hàm số f (t )  t  2  2  t với
t2 t  2

NGUYỄN VĂN LỰC  0933.168.309 SP Toán K35 - ĐH Cần Thơ


GTLN – GTNN FB: http://www.facebook.com/VanLuc168

 3t 2  4t  4 2
f / (t )   f / ( x)  0  t  ; t2
(t  t  2)
2 2
3

t -∞ -2 2 +∞
f /(t) _ _
1 2
f(t) -2
7 1

 Vậy GTLN P  2 khi x  y  1 .

Thí dụ 9. Cho các số thực thay đổi x, y thỏa điều kiện 1  y 2  x( x  y ) .


x6  y 6  1
Tìm GTLN, GTNN của biểu thức P 3 .
x y  xy 3

Lời giải.
 Ta có 1  x 2  y 2  xy  xy  xy  1

1
1  x 2  y 2  xy  ( x  y ) 2  3 xy  xy  
3
x6  y 6  1 ( x  y ) ( x  y )  3x y 
2 2 2 2 2 2 2
1
 Ta có P 3  
x y  xy 3 xy ( x  y )
2 2

xy x  y 2
2

 Đặt t  xy  x 2  y 2  1  t
 2t 2  3
 P
t 1
 2t 2  3 1
 Xét hàm số f (t )  với   t  1
t 1 3
 2t  4t  3
2
f / (t )  0
(t  1) 2

-1
t 1
3
_
f /(t)
25
f(t) 1
6
2

1
 Vậy GTNN P  f (1)  khi x  y  1
2
1 25 1
GTLN P  f ( )  khi x   y   .
3 6 3

NGUYỄN VĂN LỰC  0933.168.309 SP Toán K35 - ĐH Cần Thơ


GTLN – GTNN FB: http://www.facebook.com/VanLuc168

Thí dụ 10. (Khối B 2011)Cho a, b các số thực dương thỏa


 a 3 b3   a 2 b 2 
2(a 2  b 2 )  ab  (a  b)(ab  2) . Tìm GTNN của biểu thức P  4  3
 3   9 2  2  .
b a  b a 

Lời giải.
 Từ giả thiết ta có
a b 1 1 a b 2 2  a b
2    1    (ab  2)  2    1  a   b   2 2   

 b a   a b   b a  b a  b a 
a b 5
 Đặt t    2t  1  2 2 t  2  4t 2  4t  15  0  t 
b a 2
a 3
b  a
3 2
b 
2
 Ta có P  4 3  3   9 2  2   4(t 3  3t )  9(t 2  2)  4t 3  9t 2  12t  18
b a  b a 
5
 Xét hàm số f (t )  4t 3  9t 2  12t  18 với  t
2
1
f / (t )  12t 2  18t  12  f / ( x)  0  t   ; t  2
2

5
t +∞
2
f /(t) +
+∞
f(t) -23
4

 Suy ra P  f    
5 23
2 4
23
 Vậy GTNN P khi a  1, b  2 hay a  2, b  1 .
4

Thí dụ 11. Cho các số thực thay đổi x, y thỏa điều kiện 2( x 2  y 2 )  xy  1 .
x4  y 4
Tìm GTLN, GTNN của biểu thức P .
2 xy  1

Lời giải.

 Đặt t  xy . Ta có: xy  1  2  x  y 2  2 xy  4 xy  xy    1
5

 
 và xy  1  2  x  y 2  2 xy  4 xy  xy  . ĐK:   t  .
1
3
1
5
1
3
x 
2
2
 y2  2x2 y 2 7t 2  2t  1
 Suy ra : P   .
2 xy  1 4  2t  1

 Do đó: P ' 

7 t 2  t ,
2  2t  1
2

 1 1 2 1
P '  0  t  0, t  1( L) P   P   và P  0  
 5  3  15 4
NGUYỄN VĂN LỰC  0933.168.309 SP Toán K35 - ĐH Cần Thơ
GTLN – GTNN FB: http://www.facebook.com/VanLuc168

1 1
t - 0
5 3
P/ 0 + 0 _
1
P 2 4 2
15 15

1 2
 Vậy GTLN là và GTNN là .
4 15

Thí dụ 12. Cho các số thực a, b, c thỏa abc  2 2 .


a 6  b6 b6  c 6 c6  a6
Tìm giá trị nhỏ nhất của biểu P   
a 4  b 4  a 2b 2 b 4  c 4  b 2 c 2 c 4  a 4  c 2 a 2

Lời giải.
(a 2  b 2 )(a 4  b 4  a 2 b 2 ) (b 2  c 2 )(b 4  c 4  b 2 c 2 ) (c 2  a 2 )(c 4  a 4  c 2 a 2 )
 Ta có P   
a 4  b 4  a 2b 2 b 4  c 4  b 2c 2 c4  a4  c2a2
 Nhận xét: Do abc  2 2 nên a 2 , b 2 , c 2 là các số thực dương
x 2  y 2  xy
 Xét A = A  với x,y > 0
x 2  y 2  xy
x t2  t 1
 Chia tử và mẫu cho y 2 và đặt t  ta được A  với t>0
y t2  t 1

t2  t 1 2x 2  2
 Xét hàm số f (t )  với 0  t  f /
(t ) 
t2  t 1 ( x 2  x  1) 2

t 0 1 +∞

f /(t) _ 0 +

f(t) 1
3

1 1
 Suy ra P  (a 2  b 2 )  (b 2  c 2 )  (c 2  b 2 ) 
3 3
1
3 3

2 2

a  b 2  c 2  23 a 2 b 2 c 2  4

 Vậy GTNN P  4 khi a  b  c  2 .

Thí dụ 13. Cho hai số thực x, y thỏa mãn x  1, y  1 và 3( x  y )  4 xy.


 1 1 
Tìm giá trị lớn nhất và giá trị nhỏ nhất của biểu thức P  x3  y 3  3  2
 2 .
x y 

NGUYỄN VĂN LỰC  0933.168.309 SP Toán K35 - ĐH Cần Thơ


GTLN – GTNN FB: http://www.facebook.com/VanLuc168

Lời giải.
3a
 Đặt x  y  a . Khi đó xy  , a  0.
4
3a
 Suy ra x, y là nghiệm của phương trình t 2  at  0 (1)
4
 Phương trình (1) có nghiệm    a 2  3a  0  a  3.
3a
 Vì x, y  1 nên ( x  1)( y  1)  0. Hay là xy  ( x  y )  1  0   a  1  0  a  4.
4
 Vậy ta có 3  a  4 .
1 1 4
 Mặt khác, từ giả thiết ta lại có   .
x y 3
2
1 1 6 9 8 16
 Suy ra P  ( x  y)  3xy( x  y )  3   
3
 a3  a2   .
x y xy 4 a 3
9 8 16
 Xét hàm số f (a)  a 3  a 2   , 3  a  4.
4 a 3
9 8 3 8
 Ta có f ' (a)  3a 2  a  2  3a(a  )  2  0, a  [3; 4].
2 a 2 a

a 3
4
f ' (a) +
94
3
P  f (a )
113
12

113 3
 Dựa vào BBT ta suy ra min P  , đạt khi a  3  x  y  ;
12 2
94  x  1, y  3
max P  , đạt khi a  4   .
3  x  3, y  1.

Thí dụ 14. Cho các số thực không âm x, y, z thoả mãn x 2  y 2  z 2  3 .


5
Tìm giá trị lớn nhất của biểu thức A  xy  yz  zx  .
x yz

Lời giải.
t2 3
 Đặt t  x  y  z  t 2  3  2( xy  yz  zx)  xy  yz  zx  .
2
 Ta có 0  xy  yz  zx  x 2  y 2  z 2  3 nên 3  t 2  9  3  t  3 vì t  0.
t2  3 5
 Khi đó A   .
2 t
t2 5 3
 Xét hàm số f (t )    , 3  t  3.
2 t 2
NGUYỄN VĂN LỰC  0933.168.309 SP Toán K35 - ĐH Cần Thơ
GTLN – GTNN FB: http://www.facebook.com/VanLuc168

5 t3  5
 Ta có f ' (t )  t   2 0 vì t  3.
t2 t
14
 Suy ra f (t ) đồng biến trên [ 3, 3] . Do đó f (t )  f (3)  .
3
 Dấu đẳng thức xảy ra khi t  3  x  y  z  1.
14
 Vậy GTLN của A là , đạt được khi x  y  z  1. 
3

Thí dụ 15. Cho hai số thực x thỏa mãn 0  x  1, 0  y  1 và x  y  4 xy.


Hãy tìm giá trị lớn nhất và giá trị nhỏ nhất của biểu thức M  x 2  y 2  7 xy.

Lời giải.
 Đặt xy  t  x  y  4t. Theo định lí Viet đảo x, y là nghiệm của phương trình
h( X )  X 2  4tX  t  0.
 Vì 0  x1 , x 2  1 nên phương trình h( X )  0 có nghiệm X 1 , X 2 thoả mãn
'  4t 2  t  0

1.h(0)  t  0 1 1
0  X 1  X 2  1  1.h(1)  1  3t  0  t .
 4 3
s
0   2t  1
 2
Khi đó M  x  y 2  9 xy  16t 2  9t , với  t  .
1 1

4 3
Ta có M ' (t )  32t  9  0  t    ;  . Suy ra Bảng biến thiên
9 1 1

32  4 3 

1 9 1
t 4 32 3

M'(t) - 0 +
5 11
M 4 
9

81

64

11 1 1 1
 Suy ra: Mmax   , đạt khi xy   x  1, y  hoặc x  , y  1.
9 3 3 3
81 9 3 3
Mmin   , đạt khi xy   x  2 y  hoặc y  2 x  . 
64 32 4 4

Thí dụ 16. Cho x, y là hai số thực thỏa mãn x 2  y 2  xy  3.


Hãy tìm giá trị lớn nhất và giá trị nhỏ nhất của biểu thức A  x 4  y 4  4 xy  x3 y 3

Lời giải.
NGUYỄN VĂN LỰC  0933.168.309 SP Toán K35 - ĐH Cần Thơ
GTLN – GTNN FB: http://www.facebook.com/VanLuc168

 Điều kiện: x  1; y  3 .


 Đặt u  x  1  0; v   y  3  0 . Khi đó hệ đã cho trở thành
u  v  a
u  v  a 
 2  a 2  2a
u  v  2a 
2
 uv
 2
a 2  2a
 u, v là nghiệm của phương trình f t   t 2  at   0.
2
 Hệ đã cho có nghiệm  phương trình f t   0 có nghiệm t1 , t 2 thoả mãn t1  0  t 2
a 2  2a
1. f 0   0   0  0  a  2.
2
 Đặt t  xy . Từ giả thiết x 2  y 2  xy  3 ta có:
+) 3  x  y 2  xy   xy  xy  3 .
+) 3  x  y  xy  3xy
2 2
 xy  1. Vậy  3  t  1 .
+) x 4  y 4  x 2  y 2   2 x 2 y 2  3  xy2  2 x 2 y 2  9  6 xy  x 2 y 2 .
2

Suy ra A  t 3  t 2  2t  9,  3  t  1 .
 Xét hàm số f t   t 3  t 2  2t  9,  3  t  1 .
f ' t   3t 2  2t  2  0, t . Vậy hàm số nghịch biến trên , nên:
min f t   f 1  5; max f t   f  3  33
3t 1 3t 1

 Để ý rằng t  1  x  y  1 và t  3  x   y   3
 Vậy min A  5 , đạt khi x  y  1
max A  33 , đạt khi x   y   3 .

Thí dụ 17. (khối B 2012) Cho các số thực x, y, z thỏa mãn các điều kiện x  y  z  0
và x 2  y 2  z 2  1 . Tìm giá trị lớn nhất của biểu thức P  x5  y 5  z 5 .

Lời giải.
Cách 1:
 1
 xy  ( x  y )  2
2

x  y  z  0
   
x  y  z  1
2 2 2
 2  x  y  2
 3 3
 P = x5 + y5 + z5 = x5 + y5 – (x + y)5 = -5xy(x3 + y3) – 10x2y2(x + y)
=  ( x  y)3  ( x  y)    t 3  t ; t = x + y
5 1 5 5
2 2  2 4
5 5
 f(t) =  t 3  t
2 4
15 2 5
f’(t) =  t 
2 4
1
f’(t) = 0  t = 
6

NGUYỄN VĂN LỰC  0933.168.309 SP Toán K35 - ĐH Cần Thơ


GTLN – GTNN FB: http://www.facebook.com/VanLuc168

t  2  1 1 2
3 6 6 3
f’(t) – 0 + 0 –
f(t) 5 6 5 6
36 36
5 6
36

 Suy ra P  5 6 36 . Vậy max P = 5 6 36 xảy ra khi t = 1


6
x  y  1 x  y   2
 6  3
 
  xy   1 3 (có nghiệm) hay  xy  1 6 (có nghiệm) 
 
 z  ( x  y )  z  ( x  y )
 
Cách 2:
 Với x + y + z = 0 và x 2  y 2  z 2  1 , ta có:
0   x  y  z   x 2  y 2  z 2  2 x  y  z   2 yz  1  2 x 2  2 yz ,
2 1
nên yz  x 2  .
2
y 2  z 2 1  x2 1 1  x2
 Mặt khác yz   , suy ra x2   , do đó 
6
x
6
(*)
2 2 2 2 3 3
 Khi đó: P  x5  ( y 2  z 2 )( y 3  z 3 )  y 2 z 2 ( y  z )
2
 1
 x  (1  x ) ( y 2  z 2 )( y  z )  yz ( y  z )    x 2   x
5 2

 2
 1  
2
 1 5
 x5  (1  x 2 )   x(1  x 2 )  x  x 2      x 2   x  (2 x 3  x).
  2   2 4
 
 Xét hàm f ( x)  2 x3  x trên   6 ; 6  , suy ra f ( x)  6 x2  1 ; f ( x)  0  x  
6
 3 3  6
       
 Ta có f   6   f  6    6 , f  6   f   6   6  Do đó f ( x) 
6

 3   6  9  3   6  9 9
5 6
Suy ra P 
36
6 6
 Khi x
3
, yz
6
thì dấu bằng xảy ra.
5 6
 Vậy giá trị lớn nhất của P là 
36

Thí dụ 18. Cho 2 số thực x, y thỏa mãn : x  y  2 x  2  y  1  1 .


2(1  xy x  y )
Tìm GTLN, GTNN của F = x ( x  y)  y ( y  x)  .
2 2 x y

Lời giải.

NGUYỄN VĂN LỰC  0933.168.309 SP Toán K35 - ĐH Cần Thơ


GTLN – GTNN FB: http://www.facebook.com/VanLuc168

 Từ giả thiết  x  2; y  1 .
 Vì  2. x  2  1. y  1   22  12   x  2  y  1  2 x  2  y  1  5( x  y  1) .
2

Nên từ x  y  2 x  2  y  1  1
 x  y  5( x  y  1)  1 . Đặt t = x + y , ta có: t  1  5(t  1)  1  t  6
1 2 1 2
 Khi đó: F = ( x  y)2   t2  .
2 x y 2 t
1 2 1
 Xét f (t )  t 2  , với t  1; 6 , có f ' (t )  t   0; t  1;6
2 t t t
5 2
 Min f (t )  f (1)  ; Max f (t )  f (6)  18 
t1;6 2 t1;6  6
5 x  2
 GTNN của F là: đạt được tại: t  1  
2  y  1

2 x  6
 Vậy GTLN của F là 18  đạt được tại :t= 6   
6  y  0

Thí dụ 19. Cho x và y là các số thực thỏa mãn: 1  y 2  x( x  y ) .


x6  y 6  1
Tìm giá trị lớn nhất và nhỏ nhất của biểu thức: P 3
x y  xy 3

Lời giải.
 Từ giả thiết ta có:
1  x 2  y 2  xy  2 xy  xy  xy  1.
1
1  x 2  y 2  xy  ( x  y ) 2  3xy  3 xy  xy  .
3
 Ta có x 2  y 2  1  xy nên x 6  y 6  ( x 2  y 2 ) ( x 2  y 2 )2  3x 2 y 2 

 1  (1  t ) (1  t )2  3t 3   1
 Đặt t  xy với t   ;1 \ 0 . Khi đó ta được P 
 3  t (1  t )
2t  3
2
 Hay P  = f (t )
t 1
Hàm số f (t ) trên   ;1 \ 0
1

 3 
2t 2  4t  3  1 
 Ta có f '(t )   0 t    ;1 \ 0
(t  1) 2
 3 
1
 Vậy MinP  P (1)   t  1  x  y  1
2
1 25 1 1
MaxP  P( )   t    x  y   
3 6 3 3

Thí dụ 20. Cho x, y, z thuộc đoạn  0;2 và x  y  z  3 .


Tìm giá trị lớn nhất của A  x 2  y 2  z 2

Lời giải.
NGUYỄN VĂN LỰC  0933.168.309 SP Toán K35 - ĐH Cần Thơ
GTLN – GTNN FB: http://www.facebook.com/VanLuc168

 Cho x, y, z thuộc  0;2 và x  y  z  3 . Tìm giá trị lớn nhất của A  x 2  y 2  z 2


 Giả sử: x  y  z  3  x  y  z  3z  z  1  z  1;2
 Lại có:
x 2  y 2  ( x  y ) 2 ,(*)
 A  3  z   z 2  2z 2  6z  9
2

3
 Xét f ( z )  2 z 2  6 z  9, z  1;2  f '( z )  4 z  6, f '( z )  0  z 
2
3 9
f (1)  5; f (2)  5; f   
2 2
 Kết hợp (*) ta có
 Vậy max A  5 khi x  0; y  1; z  2 

NGUYỄN VĂN LỰC  0933.168.309 SP Toán K35 - ĐH Cần Thơ


GTLN – GTNN FB: http://www.facebook.com/VanLuc168

II. XÂY DỰNG GIÁN TIẾP HÀM SỐ f (t ) BẰNG SỬ DỤNG TÍNH CHẤT BẤT
ĐẲNG THỨC:
Phương pháp chung:
 Dự đoán khả năng dấu bằng xảy ra hoặc giá trị đặc biệt trong điều kiện để đặt
được biến phụ t thích hợp.
 Khả năng biến đổi được về hàm f(t)là khó buộc phải sử dụng bất đẳng thức.
 Lưu ý khi sử dụng bất đẳng thức điều kiện dấu bằng xảy ra phải đúng
 Cần thuộc một số bất đẳng thức phụ để có thể đưa về theo một đại lượng thích
hợp nào đó theo ý mong muốn.
 Hàm f(t) tương đối khảo sát được.
 Chú ý phần tìm điều kiện của t (phải thật chính xác)
 Thích hợp cho các đề thi khối A và B.

Thí dụ 1. (Khối B 2009) Cho các số thực thay đổi thỏa ( x  y )3  4 xy  2 .


Tìm GTNN của biểu thức P  3( x 4  y 4  x 2 y 2 )  2( x 2  y 2 )  1 .

Lời giải.
2
 x2  y2 
 Ta có ( xy)  
2

 2 
  x  y2  
2 2

 P  3 ( x  y )  
2 2 2
 
   2( x 2  y 2 )  1
  2  
( x  y) 2 1
 Đặt t  x  y 
2 2
 (theo giả thiết ( x  y ) 3  ( x  y ) 2  ( x  y ) 3  4 xy  2 )
2 2
2
9t 1
 Xét hàm số f (t )   2t  1 với t 
4 2
9t
f / (t )   2
2

1
x
2
f /(t) +

f(t) 9
16

1 9
 Suy ra P  f (t )  f ( ) 
2 16
9 1
 Vậy GTNN P  khi x  y  z  .
16 2

NGUYỄN VĂN LỰC  0933.168.309 SP Toán K35 - ĐH Cần Thơ


GTLN – GTNN FB: http://www.facebook.com/VanLuc168

Thí dụ 2. (Khối B 2010) Cho các số thực không âm a, b, c thỏa a  b  c  1 .


Tìm GTNN của biểu thức P  3(a2b2  b2c2  c2a2 )  3(ab  bc  ca)  2 a 2  b2  c2

Lời giải.
 Ta biến đổi P  (ab  bc  ca)2  3(ab  bc  ca)  2 1  2(ab  bc  ca)
(a  b  c) 2 1
 Đặt t  ab  bc  ca , điều kiện 0  t  ab  bc  ca  
3 3
 Xét hàm số f (t )  t 2  3t  2 1  2t , t  0;  , ta có
1
 3
2
f '(t )  2t  3 
1  2t
2
f / / (t )  2  0
(1  2t )3

Do vậy f / (t ) là hàm nghịch biến: f / (t )  f /     2 3  0 .


1 11
3 3
Suy ra f (t ) là hàm số đồng biến
 BBT
1
t 0
3
f / t  -
10  6 3
9
f (t )

2
 Suy ra P  f (t )  f (0)  2
 ab  bc  ca

 Vậy GTNN P  2 khi ab  bc  ca  0 khi (1; 0; 0) và các hoán vị. 
 a b  c 1

Thí dụ 3. Cho a, b, c là độ dài ba cạnh của tam giác có chu vi bằng 3.


Tìm GTLN của biểu thức P  3(a 2 b 2  c 2 )  4abc .

Lời giải.
3
 Giả sử 0  a  b  c  1  c 
2
 Ta có P  3(a  b)  6ab  3c  4abc  3(3  c) 2  3c 2  2(3  c)ab
2 2

ab
2

 3(3  c) 2  3c 2  2(3  c) 


 2 
3c
2

 3(3  c) 2  3c 2  2(3  c) 


 2 

NGUYỄN VĂN LỰC  0933.168.309 SP Toán K35 - ĐH Cần Thơ


GTLN – GTNN FB: http://www.facebook.com/VanLuc168

3 27
 c3  c2 
2 2
3 27 3
 Xét hàm số f (t )  t 3  t 2  với 1 t 
2 2 2
f / (t )  3c 2  3c

BBT:
3
t 0 1
2
f /(t) _ 0 +

f(t)
13

 Suy ra P  f (1)  13
 Vậy GTNN P  13 khi a  b  c  1 .

Thí dụ 4. Cho các số dương x, y, z thỏa x  y  z  1 .


1 1 1
Tìm GTNN của biểu thức P  x  y  z    .
x y z

Lời giải.
 Theo bất đẳng thức Côsi ta có
1  x  y  z  33 xyz
1 1 1 3
  
x y z 3 xyz
3
 Suy ra P  33 xyz 
xyz3

3 1
 Xét hàm số f (t )  3t  với 0  t 
t 3
3 3  3t 2
f / (t )  3  2  0
t t2

1
x 0
3
_
f /(t)

f(t)
10

1
 Suy ra P  f (t )  f ( )  10
3

NGUYỄN VĂN LỰC  0933.168.309 SP Toán K35 - ĐH Cần Thơ


GTLN – GTNN FB: http://www.facebook.com/VanLuc168

1
 Vậy GTNN P  10 khi x  y  z 
3

Thí dụ 5. (Khối A 2003) Cho các số đương x, y, z thỏa x  y  z  1 .


1 1 1
Tìm GTNN của biểu thức P  x 2  2
 y2  2  z2  2 .
x y z

Lời giải.
2
 1 
2
1 1 1
 Ta có P  ( x  y  z )       3 (33 xyz ) 2   33
2


 x y z   xyz 
x yz
2
9 1 1
 Xét hàm số f (t )  9t  với 0  t  0t   
t 9  3  9
9 9  9t 2
f / (t )  9  2  0 x 0
1
t t2 9
_
f /(t)

f(t)
82
1
 Suy ra P  f (t )  f ( )  82
9
1
 Vậy GTNN P  82 khi x  y  z  .
3

Thí dụ 6. Cho các số thực không âm a, b, c thỏa a  b  c  3 .


Tìm GTLN của biểu thức P  (a 2  ab  b 2 )(b 2  bc  c 2 )(c 2  ca  a 2 ) .

Lời giải.
 Giả sử 0  a  b  c  3
a(a  b)  0 a 2  ab  b 2  b 2
 Suy ra   2
a ( a  c)  0  a  ac  c  c
2 2

 Do đó P  b 2 c 2 (b 2  bc  c 2 )  b 2 c 2 (b  c) 2  3bc
 abc 3
 Từ  ta có b  c  a  b  c  b  c  3  2 bc  b  c  3
0  a  b  c  3
9
 Suy ra 0  bc 
4
 Từ đó ta có P  b 2 c 2 (9  3bc)
9
 Xét hàm số f (t )  3t 3  9t 2 với 0  t 
4
f / (t )  9t 2  18t
9
t 0 2
4
f /(x) 0 + 0 _

12
 Suy ra P  f (2)  12 f(x)
 Vậy GTLN P  12 khi a  0; b  1; c  2 và các hoán vị. 

NGUYỄN VĂN LỰC  0933.168.309 SP Toán K35 - ĐH Cần Thơ


GTLN – GTNN FB: http://www.facebook.com/VanLuc168

Thí dụ 7. Cho các số thực a, b, c đôi một khác nhau thuộc  0; 2 .


1 1 1
Tìm GTNN của biểu thức P    .
(a  b) (b  c) (c  a) 2
2 2

Lời giải.
 Giả sử 0  a  b  c  2
 1 1
 
 0ca  2  (c  a ) 2
4
 Từ  
0  c  b  2  b 
1

1
 (b  c) 2
( 2  b) 2
1 1 1
 Suy ra P  2  
b ( 2  b) 2
4
1 1 1
 Xét hàm số f (b)  2   với 0  b  2
b ( 2  b) 2
4
2 2
f / (b)   3 
b ( 2  b) 3 b 0 1 2
_
f /(b) 0 0 +

f(b) 9
4
9
 Suy ra P  f (1) 
4
9
 Vậy GTNN P  khi a  0; b  1; c  2 và các hoán vị. 
4

Thí dụ 8. Cho các số đương x, y thỏa x  y  1 .


x y
Tìm GTNN của biểu thức P   .
1 x 1 y

Lời giải.
a b
 Áp dụng BĐT   a b
b a
x 1 x
 P   x  1 x
1 x x
 Xét hàm số f ( x)  x  1  x với 0  x  1
1 1 1
f / ( x)   . f /  x  0  x 
2 x 2 1 x 2

1
x 0 1
2
f /(x) 0 + 0 _
2
f(x)

NGUYỄN VĂN LỰC  0933.168.309 SP Toán K35 - ĐH Cần Thơ


GTLN – GTNN FB: http://www.facebook.com/VanLuc168

1
 Suy ra P  f ( )  2
2
1
 Vậy GTNN P  2 khi x  y  .
2

Thí dụ 9. (Khối B 2006) Cho các số thực thay đổi x, y .


Tìm GTNN của biểu thức P  ( x  1)2  y 2  ( x  1)2  y 2  y  2

Lời giải.
 Ta có BĐT a 2  b 2  c 2  d 2  (a  c) 2  (b  d ) 2
P  (1  x  x  1) 2  ( y  y) 2  y  2  2 1  y 2  y  2
 Xét hàm số f ( y)  2 1  y 2  y  2
 Trường hợp y  2  0  y  2
f ( y)  2 1  y 2  y
2y
f / ( y)  1
1 y2
1
 f / ( y)  0  y 
3
 1 
Suy ra f ( y)  f    2  3
 3
 Trường hợp y  2  0  y  2
y -∞ 2
_
f /(y)
+∞
f(y)
2+ 3

f ( y)  2 1  y 2  2 1  2 2  2  3
1
 Vậy GTNN P  2  3 khi x  0, y  .
3

Thí dụ 10. Cho các số đương x, y, z thỏa x  y  z  3 .


1 2
Tìm GTLN của biểu thức P   .
x  y  z 1
2 2 2 ( x  1)( y  1)( z  1)

Lời giải.
 Áp dụng BĐT côsi, ta có
1 1 1
x2 y2 z2 1 (x y )2 ( z 1)2 (x y z 1)2
2 2 4
 x  y  z  3
3

( x  1)( y  1)( z  1)   
 3 

NGUYỄN VĂN LỰC  0933.168.309 SP Toán K35 - ĐH Cần Thơ


GTLN – GTNN FB: http://www.facebook.com/VanLuc168

2 54
 Suy ra P  
x  y  z  1 ( x  y  z  3) 3
 Đặt t  x  y  z  1  1
2 54
P 
t (t  2) 3
2 54
 Xét hàm số f (t )   với 1  t
t (t  2) 3
2 162
f / (t )   2   f / (t )  0  t  1; t  4
t (t  2) 4

t 0 4 +∞
f /(t) + 0 _

1
f(t)
4

1
 Suy ra P  f (4) 
4
1
 Vậy GTLN P  khi x  y  z  1 .
4

Thí dụ 11. Cho các số dương x, y, z . Tìm GTLN của biểu thức
x y z
P  
x2  y 2 y2  z2 z 2  x2

Lời giải.
y z x
 Đặt a  , b  , c   abc  1
x y z
1 1 1 1  1 1 
 Suy ra P      2  
1 b 1 c2 
2
1 a 2
1 b 2
1 c 2
1 a 2

1 2 2 1
2 1
1 a2 1 bc 1 a 1 a
1 1
 Đặt t với 0  t 
1 a 2
 Xét hàm số f (t )  t 2  2 1  t
2  2t  1
f / (t )  0
t 1 1
t 0 2
f /(t) +
3
f(t) 2
-∞

NGUYỄN VĂN LỰC  0933.168.309 SP Toán K35 - ĐH Cần Thơ


GTLN – GTNN FB: http://www.facebook.com/VanLuc168

1 3
 Suy ra P  f ( ) 
2 2
1
 Vậy GTLN P  khi x  y  z  1 .
4

Thí dụ 12. Cho các số dương x, y, z thỏa x  y  z  3 .


xy  yz  zx
Tìm GTNN của biểu thức P  x 2  y 2  z 2  .
x y  y2 z  z2 x
2

Lời giải.
 Ta có
3(a 2  b 2  c 2 )  (a  b  c)(a 2  b 2  c 2 )  a 3  b 3  c 3  a 2 b  b 2 c  c 2 a  ab 2  bc 2  ca 2
a 3  ab 2  2a 2 b

 Mà  b 3  bc 2  2b 2 c  3(a 2  b 2  c 2 )  3(a 2 b  b 2 c  c 2 a)  0
 c 3  ca 2  2c 2 a

 Đặt t  x 2  y 2  z 2
9  (x2  y 2  z 2 ) 9t
 P  x2  y2  z 2  t
2( x  y  z )
2 2 2
2t
1 9
 Xét hàm số f (t )  t   với 3  t
2 2t
9
f / (t )  1  2
2t

t 0 +∞
f /(t) +
+∞
f(t)
4

1
 Suy ra P  f (4) 
4
1
 Vậy GTLN P  khi x  y  z  1 .
4

Thí dụ 13. Cho các số không âm x, y, z thỏa x  y  z  0 .


x 3  y 3  16 z 3
Tìm GTNN của biểu thức P 
( x  y  z )3

Lời giải.
( x  y) 3
 Ta có x 3  y 3  dựa vào phép chứng minh tương đương
4
 Đặt x  y  z  a , khi đó
x 3  y 3  16 z 3 ( x  y ) 3  64 z 3 (a  z ) 3  64 z 3
4P   
( x  y  z) 3 a3 a3

NGUYỄN VĂN LỰC  0933.168.309 SP Toán K35 - ĐH Cần Thơ


GTLN – GTNN FB: http://www.facebook.com/VanLuc168

z
 Đặt t 
a
 Xét hàm số f (t )  (1  t ) 3  64t 3 với 0  t  1
 
f / (t )  3 64t 2  (1  t ) 2  f / (t )  0  t 
1
9

1
t 0 1
9
f /(t) _ 0 +

f(t) 64
81

 Suy ra P  , f   
1 1 16
4  9  81
16
 Vậy GTNN P  khi x  y  4 z .
81

Thí dụ 14. (Khối B 2007) Cho các số thực dương x, y, z .


x 1  y 1  z 1 
Tìm GTNN của biểu thức P  x    y   z  .
2 yz   2 zx   2 xy 

Lời giải.
x2  y2  z2 x2  y2  z2
 Ta có P 
2 xyz
 Do x 2  y 2  z 2  xy  yz  zx
 x2 1   y2 1   z2 1 
P            
 2 x  2 y  2 z
t2 1 1
 Xét hàm số f (t )   với t 
2 t 2
1
f / (t )  t  2
t

9
 Vậy GTNN P  khi x  y  z  1 .
2

NGUYỄN VĂN LỰC  0933.168.309 SP Toán K35 - ĐH Cần Thơ


GTLN – GTNN FB: http://www.facebook.com/VanLuc168

Thí dụ 15. (Khối A 2011)Cho x, y, z là ba số thực thuộc đoạn [1;4] và x  y, x  z .


x y z
Tìm giá trị nhỏ nhất của biểu thức P   .
2x  3y z  y z  x

Lời giải.
1 1 2
 Ta có   với a  0, b  0 và ab  1 (chứng minh tương đương)
1  a 1  b 1  ab
x 1 1 1 2
 Khi đó P     
2x  3y 1  z 1  x 2  3y x
1
y z x y
x
 Đặt t  với 1  t  2
y
t2 2
 Suy ra P  2 
2t  3 1  t
t2 2
 Xét hàm số f (t )  2  với 1  t  2
2t  3 1  t

f / (t ) 
 
 2 t 3 (4t  3)  3t (2t  1)  9
0
(2t 2  3) 2 (1  t ) 2

t 1 2
_
f /(t)

f(t) 34
33

 Suy ra P  f 2  
34
33

34
 Vậy GTNN P  khi x  4; y  1; z  2 .
33

Thí dụ 16. (khối D-2012) Cho các số thực x, y thỏa mãn  x  4    y  4   2 xy  32 .


2 2

Tìm giá trị nhỏ nhất của biểu thức A  x3  y 3  3  xy  1 x  y  2  .

Lời giải.
 ( x  4)2  ( y  4)2  2 xy  32  ( x  y ) 2  8( x  y )  0  0  x  y  8
3
 4 xy  ( x  y )2   6 xy   ( x  y ) 2
2
 A = x  y  3( xy  1)( x  y  2) = ( x  y )3  6 xy  3( x  y )  6
3 3

3
 A  ( x  y )3  ( x  y ) 2  3( x  y )  6
2
 3
Đặt t = x + y ( 0  t  8 ), xét f(t) = t 3  t 2  3t  6  f’(t) = 3t 2  3t  3
2
1 5 1 5 17  5 5
f’(t) = 0 khi t = ; f(0) = 6, f(8) = 398, f( )=
2 2 4
NGUYỄN VĂN LỰC  0933.168.309 SP Toán K35 - ĐH Cần Thơ
GTLN – GTNN FB: http://www.facebook.com/VanLuc168

17  5 5 1 5
 Vậy giá trị nhỏ nhất của f(t) là xảy ra khi t =
4 2
17  5 5 1 5
 A  f(t)  . Dấu bằng xảy ra khi x = y và x + y = hay x = y =
4 2
1 5

4

BÀI TẬP

Bài 1: Cho x, y, z là ba số thực thỏa x 2  y 2  z 2  2 Tìm giá trị nhỏ nhất của biểu thức
P  x 3  y 3  z 3  3xyz

Hướng dẫn : đặt t  x  y  z


Bài 2: Cho các số dương x, y, z thỏa x  y  z  3 . Tìm GTNN của biểu thức
P  9 xy  10 xz  22 yz

Hướng dẫn :
P  9 xy  10( x  y) z  12 yz  9 xy  10( x  y)3  ( x  y)  12 y3  ( x  y)

Xét hàm số f (t )  t 2  3t với 0  t  3


P  10 f ( x  y )  12 f ( y )  22 xy  max f (t )

Bài 3: Cho các số dương x, y, z thỏa x 2  y 2  z 2  1 . Tìm GTLN của biểu thức
P  6( y  z  x)  27 xyz

Hướng dẫn :

 
P  6 2( y 2  z 2 )  x  27 x
y2  z2
2
 
 6 2(1  x 2 )  x  27
x(1  x 2 )
2

Bài 4: Cho các số dương x, y, z thỏa 21xy  2 yz  8 zx  12 . Tìm GTNN của biểu thức
1 2 3
P  
x y z

Hướng dẫn :
1 2 3 2a  4b
Đặt a  ; b  ; c  , bài toán đưa về tìm GTNN P  a  b  c với c 
x y z 2ab  7

14 14 14 14
2a  4b   2a  2a 
P  ab a a  ab 2  a  a  11  2ab  7  a
 7 a 2ab  7 2a 2a 2ab  7
a 2b  
 a

NGUYỄN VĂN LỰC  0933.168.309 SP Toán K35 - ĐH Cần Thơ


GTLN – GTNN FB: http://www.facebook.com/VanLuc168

11 7
Xét hàm số f (t )  t   2 1 2
2t t

Bài 5: Cho các số thực x, y, z không đồng thời bằng 0 thỏa x 2  y 2  z 2  2( xy  yz  zx) .
Tìm GTLN, GTNN của biểu thức
x3  y3  z 3
P
( x  y  z )( x 2  y 2  z 2 )

Hướng dẫn :
4x 4y 4z
Đặt a  , b , c . Khi đó a  b  c  4 và ab  bc  ca  4
x yz x yz x yz

8
Áp dụng BĐT (b  c) 2  4bc suy ra 0  a 
3
1 3 1
Khi đó P  (a  b 3  c 3 )  (3a 3  12a 2  12a  16)
32 32
1
Xét hàm số f (t )  (3t 3  12t 2  12t  16)
32

Bài 6: Cho các số dương x, y, z thỏa ( x  y  z ) 3  32 xyz . Tìm GTLN của biểu thức
x4  y4  z4
P
( x  y  z) 4

Hướng dẫn :
Do tử và mẫu cùng bậc nên giả sử x  y  z  4
Ta có x 4  y 4  z 4  ( x 2  y 2  z 2 ) 2  2( x 2 y 2  y 2 z 2  z 2 x 2 )

 
2

 ( x  y  z ) 2  2( xy  yz  zx)  2 ( xy  yz  zx) 2  2 xyz( x  y  z ) 
Đặt t  xy  yz  zx
Xét hàm số f (t )  (16  2t ) 2  2(t 2  16)
xy  yz  zx 1
Bài 7: Cho các số dương x, y, z thỏa  . Tìm GTLN, GTNN của biểu thức
x2  y2  z2 7

x4  y4  z4
P
( x  y  z) 4

Hướng dẫn :
Do tử và mẫu cùng bậc nên giả sử x  y  z  1
xy  yz  zx 1 xy  yz  zx 1 1 2
Từ giả thiết     xy  yz  zx   xy   (1  z ) z
x y z
2 2 2
7 1  2( xy  yz  zx) 7 9 9

Ta có x 4  y 4  z 4  ( x 2  y 2  z 2 ) 2  2( x 2 y 2  y 2 z 2  z 2 x 2 )

NGUYỄN VĂN LỰC  0933.168.309 SP Toán K35 - ĐH Cần Thơ


GTLN – GTNN FB: http://www.facebook.com/VanLuc168

 2

 ( x  y  z ) 2  2( xy  yz  zx)  2 ( xy  yz  zx) 2  2 xyz( x  y  z ) 
Xét hàm số theo biến z và z  min x, y, z  0  z 
1
3

Bài 8: Cho các số dương x, y, z . Tìm GTNN của biểu thức


2( x  y  z ) 3  9 xyz
P
( x  y  z )( xy  yz  zx)

Hướng dẫn :
Do tử và mẫu cùng bậc nên giả sử x  y  z  1 và z  min x, y, z  0  z 
1
3

NGUYỄN VĂN LỰC  0933.168.309 SP Toán K35 - ĐH Cần Thơ


GTLN – GTNN FB: http://www.facebook.com/VanLuc168

SỬ DỤNG PHƯƠNG PHÁP ĐỒ THỊ - BẢNG BIẾN THIÊN


GIẢI CÁC BÀI TOÁN PT – BPT – HPT LIÊN QUAN ĐẾN THAM SỐ

I. CƠ SỞ CỦA PHƯƠNG PHÁP GIẢI


Cơ sở của phương pháp này là ý nghĩa hình học của việc giải phương trình, bất phương
trình được thể hiện trong các tính chất sau.
Xét các hệ thức f  x   g  x  (1) ; f  x  g  x (2) ; f  x  g  x (3)
Gọi G f , Gg lần lượt là đồ thị hàm số y  f  x  , y  g  x  . Trên cùng một mặt phẳng tọa độ
 Oxy  vẽ G f và Gg . Ký hiệu D  D f  Dg là tập xác định của hệ thức, ta có:
1. Nghiệm của phương trình (1) là hoành độ điểm chung của G f và Gg
2. Nghiệm của bất phương trình (2) là khoảng các giá trị của x mà trong đó G f nằm ở
phía trên Gg
3. Nghiệm của bất phương trình (3) là khoảng các giá trị của x mà trong đó G f nằm ở
phía dưới Gg
Nhận xét 1
1. Phương trình (1) có nghiệm  G f và Gg có điểm chung
2. Phương trình (1) vô nghiệm  G f và Gg không có điểm chung
3. Phương trình (1) có k nghiệm  G f và Gg có k điểm chung
4. Phương trình (1) có k nghiệm phân biệt  G f và Gg có k điểm chung khác nhau.
Nhận xét 2
1. Bất phương trình (2) có nghiệm  có điểm thuộc G f nằm ở phía trên Gg
2. Bất phương trình (2) vô nghiệm  không có điểm nào thuộc G f nằm ở phía trên
Gg
3. Bất phương trình (2) luôn đúng với mọi x  D  toàn bộ G f nằm ở phía trên Gg
Nhận xét 3
1. Bất phương trình (3) có nghiệm  có điểm thuộc G f nằm ở phía dưới Gg
2. Bất phương trình (3) vô nghiệm  không có điểm nào thuộc G f nằm ở phía dưới
Gg
3. Bất phương trình (3) luôn đúng với mọi x  D  toàn bộ G f nằm ở phía dưới Gg
Chú ý 1
Đối với hệ thức dạng f  x   0 (1) ; f  x  0 (2) ; f  x  0 (3)
thì Gg có phương trình y  0 nên Gg là trục hoành.
Chú ý 2
Đối với hệ thức dạng f  x   m (1) ; f  x  m (2) ; f  x  m (3)
thì Gg có phương trình y  m nên Gg là đường thẳng vuông góc với trục tung tại điểm
có tọa độ  0; m 
 Trong trường hợp này ta có thể thay việc vẽ Gg trên D bằng việc lập BBT của
hàm số y  f  x  trên D . Các hệ thức trên còn được gọi là có dạng “tách ẩn”
hoặc dạng “cô lập”.
II. ÁP DỤNG
NGUYỄN VĂN LỰC  0933.168.309 SP Toán K35 - ĐH Cần Thơ
GTLN – GTNN FB: http://www.facebook.com/VanLuc168

Thí dụ 1. Tìm m để phương trình sau có nghiệm


x2  x  1  x2  x  1  m (1)

Lời giải.
 Tập xác định của phương trình : D 
 Xét hàm số y  f  x   x2  x  1  x2  x  1 trên .
Phương trình 1 có nghiệm  đường thẳng y  m có điểm chung với phần đồ
thị hàm số y  f  x  vẽ trên .
2x 1 2x 1
 Lập BBT của hàm số y  f  x  trên D . Ta có: f '  x   
2 x  x 1
2
2 x2  x  1
f '  x   0   2 x  1 x 2  x  1   2 x  1 x 2  x  1 (2)
Bình phương hai vế (2), ta được phương trình hệ quả
 4x 2
    
 4x  1 x2  x  1  4x2  4x  1 x2  x  1  x  0
Thử lại, ta thấy x  0 không thỏa (2). Vậy f '  x   0 vô nghiệm
Do f '  x   0 vô nghiệm  f '  x  không đổi dấu trên , mà f '  0   1  0
 f '  x   0, x   f  x  đồng biến trên .
2x
Giới hạn: xlim f  x   lim  1 và lim f  x   1
 x 
x2  x  1  x2  x  1 x 

Bảng biến thiên


x - +
f ' x +
f  x 1

-1

 Dựa vào BBT ta suy ra: Phương trình (1) có nghiệm  1  m  1 . 

MINH HỌA ĐỒ THỊ

Thí dụ 2. Tìm m để phương trình sau có hai nghiệm phân biệt

NGUYỄN VĂN LỰC  0933.168.309 SP Toán K35 - ĐH Cần Thơ


GTLN – GTNN FB: http://www.facebook.com/VanLuc168

m x2  2x  2  x  2 (1)

Lời giải.
 Tập xác định của phương trình : D 
x2
Khi đó: 1  m  (2)
x2  2x  2
x2
 Xét hàm số y  f  x   trên .
x2  2x  2
Phương trình (2) có hai nghiệm phân biệt x   đường thẳng y  m có hai điểm
chung khác nhau với đồ thị hàm số y  f  x  vẽ trên .
4  3x
 Lập BBT của hàm số trên trên D . Ta có: f '  x  
x 2
 2x  2  x2  2x  2
4
f ' x  0  x 
3
x2 x2
Giới hạn: xlim f ( x)  lim  1 và lim f ( x)  lim 1
 x 
x2  2x  2 x  x 
x2  2 x  2
Bảng biến thiên

x 4
 
3
f ' x + 0
10
f  x
1 1

 Dựa vào BBT ta suy ra:


Phương trình (2) có hai nghiệm phân biệt x   1  m  10 .

MINH HỌA ĐỒ THỊ

Thí dụ 3. Tìm m để phương trình sau có hai nghiệm phân biệt

NGUYỄN VĂN LỰC  0933.168.309 SP Toán K35 - ĐH Cần Thơ


GTLN – GTNN FB: http://www.facebook.com/VanLuc168

x 2  mx  2  2 x  1 (1)

Lời giải.
 Do x  0 không phải là nghiệm của phương trình (1) nên
 3x 2  4 x  1
 1 3x 2  4 x  1  mx   m (2)
x    
 
1  2   1  
x
 x 2  mx  2  4 x 2  4 x  1  x   x   1
  2 
 2
3x  4 x  1
trên D    ;   .
2
1
 Xét hàm số y  f  x  
x  2 
Phương trình (1) có hai nghiệm phân biệt  Phương trình (2) có hai nghiệm
phân biệt x   ;    đường thẳng y  m có hai điểm chung khác nhau với
1
đồ
 2 
thị hàm số y  f  x  vẽ trên  ;   .
1
 2 
3x 2  1
Lập BBT của hàm số trên trên D . Ta có: f '  x   2 , x    ;   \ 0
1

x  2 
3x  4 x  1
2
Giới hạn: xlim

f ( x)  lim
x 
 
x
Bảng biến thiên

x 1
 0 
2
f ' x + +
 
f  x
9

2
 Dựa vào BBT ta suy ra:
9
Phương trình (1) có hai nghiệm phân biệt  m  .
2
MINH HỌA ĐỒ THỊ

Thí dụ 4. Tìm m để phương trình sau có đúng hai nghiệm phân biệt
4
2x  2x  2 4 6  x  2 6  x  m (1)
NGUYỄN VĂN LỰC  0933.168.309 SP Toán K35 - ĐH Cần Thơ
GTLN – GTNN FB: http://www.facebook.com/VanLuc168

Lời giải.
 Tập xác định của phương trình : D   0; 6
 Xét hàm số y  f  x   4 2x  2x  2 4 6  x  2 6  x trên  0; 6  .
Phương trình 1 có nghiệm trên  0; 6   đường thẳng y  m có điểm chung với
phần đồ thị hàm số y  f  x  vẽ trên  0; 6  .
 Lập BBT của hàm số y  f  x  trên D . Ta có:
1 1 1 1
f ' x     
2 4 2x
3
2x 2 4 6  x 3
6 x

 

1 1

1   1  1  1  , x   0;6 
2  4  2 x 3 4  6  x 3  2  2 x 
6 x 
 
1 1 1 1
Đặt u  x    , v  x   . Ta thấy u  2   v  2   0 nên
4
 2x 4 6  x
3 3
2x 6 x

f '  2  0
Mặt khác u  x  , v  x  cùng dương trên  0; 2  , cùng âm trên  2; 6  nên ta có
Bảng biến thiên

x 0 2 6
f’(x) + 0
f(x) 63 2

2 6  24 6 4
12  2 3
 Dựa vào BBT ta suy ra:
Phương trình 1 có nghiệm trên  0; 6   2 6  2 4 6  m  3 2  6 .
MINH HỌA ĐỒ THỊ

Thí dụ 5. Tìm m để phương trình sau có nghiệm


6 x2  4  x  2 x  2   m  4  4  x  2x  2  (1)

NGUYỄN VĂN LỰC  0933.168.309 SP Toán K35 - ĐH Cần Thơ


GTLN – GTNN FB: http://www.facebook.com/VanLuc168

Lời giải.
 Tập xác định của phương trình : D  1; 4
 Đặt ẩn phụ t  4  x  2 x  2 với x  1; 4 . Tìm tập giá trị của ẩn phụ t khi x  1; 4
1 1  2x  2  2 4  x
Ta có: t'   , x  1; 4 
2 4 x 2x  2 2 4  x. 2x  2
t '  0  2 4  x  2x  2  4  4  x   2x  2  x  3
Bảng biến thiên

x 1 3 4
t' + 0
t 3

3 6

Từ bảng biến thiên ta suy ra tập giá trị của t là : D '   3;3
 Với ẩn phụ trên thì phương trình (1) trở thành: t 2  4t  4  m (2)
Phương trình (1) có nghiệm x  1; 4  Phương trình (2) có nghiệm t   3;3
 Xét hàm số y  f  t   t 2  4t  4 với t   3;3 .
Phương trình  2  có nghiệm t   3;3  đường thẳng y  m có điểm chung với
phần đồ thị hàm số y  f  t  vẽ trên  3;3 .
 Lập BBT của hàm số y  f  t  trên D ' . Ta có: f '  t   2t  4 ; f '  t   0  t  2
Bảng biến thiên

t 3 2 3
f 't  0 +
f t  74 3 1

 Dựa vào BBT ta suy ra:


Phương trình (1) có nghiệm x  1; 4  0  m  1 .

Chú ý: Khi đặt ẩn phụ ta phải tìm tập giá trị của ẩn phụ và chuyển phương trình sang
phương trình theo ẩn phụ với tập xác định là tập giá trị của ẩn phụ tìm được. Cụ thể
 Khi đặt t  u  x  , x  D , ta tìm được t  D ' và phương trình f  x; m   0 (1) trở thành
g  t ; m   0 (2) . Khi đó (1) có nghiệm x  D  (2) có nghiệm t  D '
 Để tìm miền giá trị của t ta nên lập BBT của hàm số t  u  x  trên D (có thể sử
dụng bất đẳng thức để đánh giá hoặc tính chất của hàm số)

NGUYỄN VĂN LỰC  0933.168.309 SP Toán K35 - ĐH Cần Thơ


GTLN – GTNN FB: http://www.facebook.com/VanLuc168

 Nếu bài toán yêu cầu xác định số nghiệm thì ta phải tìm sự tương ứng giữa x và
t . Tức là mỗi giá trị t  D ' thì phương trình u  x   t có bao nhiêu nghiệm x  D ?
(có thể xem là một bài toán nhỏ về xét sự tương giao)

Thí dụ 6. Tìm m để phương trình sau có nghiệm


4 6  x  x 2  3x  m  x  2  2 3 x  (1)

Lời giải.
 Tập xác định của phương trình : D   2;3
 Đặt t  x  2  2 3  x với x   2;3 . Tìm tập giá trị của ẩn phụ t khi x   2;3
1 2 3 x  x  2
Ta có: t'  
2 x  2 2 3  x 2 x  2. 3  x
t '  0  3  x  2 x  2  3  x  4  x  2   x  1
Bảng biến thiên

x -2 -1 3
t' + 0
t 5

2 5 5

Từ bảng biến thiên ta suy ra tập giá trị của t là : D '   5;5
14
 Với ẩn phụ trên thì phương trình (1) trở thành: t 2  14t  mt  t  m (2)
t
Phương trình (1) có nghiệm x   2;3  Phương trình (2) có nghiệm t   5;5
14
 Xét hàm số y  f  t   t  với t   5;5 .
t
Phương trình  2  có nghiệm t   5;5  đường thẳng y  m có điểm chung với
phần đồ thị hàm số y  f  t  vẽ trên  5;5 .
14
 Lập BBT của hàm số trên y  f  t  trên D ' . Ta có: f '  t   1   0 , t   5;5
 
t2
Bảng biến thiên

t 5 5
f 't  +
f t  11
5
9 5

5

 Dựa vào BBT ta suy ra:

NGUYỄN VĂN LỰC  0933.168.309 SP Toán K35 - ĐH Cần Thơ


GTLN – GTNN FB: http://www.facebook.com/VanLuc168

9 5 11
Phương trình (1) có nghiệm x   2;3   m . 
5 5
Thí dụ 7. Tìm m để phương trình sau có nghiệm
m  
1  x2  1  x2  2  2 1  x4  1  x2  1  x2 (1)

Lời giải.
 Tập xác định của phương trình : D   1;1
 Đặt t  1  x 2  1  x 2 x   1;1 . Tìm tập giá trị của ẩn phụ t khi x   1;1
x x  1 1 
Ta có: t'   x  , t'0 x 0
1  x2 1 x 2
 1 x
2
1  x2 
Bảng biến thiên

x -1 0
1
t' 0 +
t 2
2

Từ bảng biến thiên ta suy ra tập giá trị của t là : D '  0; 2 
t 2  t  2
 Với ẩn phụ trên thì phương trình (1) trở thành: m  t  2   t 2  t  2  m
t2
(2)
Phương trình (1) có nghiệm x   1;1  Phương trình (2) có nghiệm t  0; 2 
t 2  t  2
 Xét hàm số y  f  t   với t  0; 2  .
t2
Phương trình  2  có nghiệm t  0; 2   đường thẳng y  m có điểm chung với
phần đồ thị hàm số y  f  t  vẽ trên 0; 2  .
t 2  4t
 Lập BBT của hàm số y  f  t  trên D ' . Ta có: f '  t    0 , t  0; 2 
t  2
2

Bảng biến thiên

t 0 2
f 't 
f t  1

2 1
 Dựa vào BBT ta suy ra:
Phương trình (1) có nghiệm x   1;1  2 1  m  1 . 

NGUYỄN VĂN LỰC  0933.168.309 SP Toán K35 - ĐH Cần Thơ


GTLN – GTNN FB: http://www.facebook.com/VanLuc168

Thí dụ 8. Tìm m để phương trình sau có nghiệm


 

x  x 1  m x 

1
x 1

 4 x  x  1   1

(1)

Lời giải.
 Tập xác định của phương trình : D   0;  
 Khi đó: 1   x  x  1   m x  
1
 4 x  x  1   1
 x 1 
1
m x  4 x  x  1  x  x  1
x 1
1
 x 1   4 x  x  1  1  m  x
x 1
x x 1
 4  1 m (2)
x 1 x
x 1 x 1
 Đặt t  4 , do x  1 nên 0   1  0  t  1 . Tập giá trị của t là: D '   0;1
x x
1 1
 Với ẩn phụ trên thì phương trình (1) trở thành: 2  t  1  m  m   2  t  1
t t
(2)
Phương trình (1) có nghiệm x  1;    Phương trình (2) có nghiệm t   0;1
1
 Xét hàm số y  f  t     t  1 với t   0;1 .
t2
Phương trình  2  có nghiệm t   0; 2   đường thẳng y  m có điểm chung với
phần đồ thị hàm số y  f  t  vẽ trên  0; 2  .
2
 Lập BBT của hàm số y  f  t  trên D ' . Ta có: f '  t    1  0, t   0;1
t2
Bảng biến thiên

t 0 1
f 't  +
f t  1



 Dựa vào BBT ta suy ra:


Phương trình (1) có nghiệm x  1;    m  1 . 

Thí dụ 9. Tìm m để phương trình sau có nghiệm


3 x 1  m x  1  4 4 x2 1 (1)

NGUYỄN VĂN LỰC  0933.168.309 SP Toán K35 - ĐH Cần Thơ


GTLN – GTNN FB: http://www.facebook.com/VanLuc168

Lời giải.
 Tập xác định của phương trình : D  1;  
x 1 x2 1 x 1 x 1
 Khi đó: 1  3  m  24 3  m  24
x 1  x  1 x 1 x 1
2

x 1 x 1
 Đặt t  4 , do x  1 nên 0   1  0  t  1 . Tập giá trị của t là: D '   0;1
x 1 x 1
 Với ẩn phụ trên thì phương trình (1) trở thành: 3t 2  2t  m (2)
Phương trình (1) có nghiệm x  1;    Phương trình (2) có nghiệm t   0;1
 Xét hàm số y  f  t   3t 2  2t với t   0;1 .
Phương trình  2  có nghiệm t   0;1  đường thẳng y  m có điểm chung với
phần đồ thị hàm số y  f  t  vẽ trên t   0;1 .
1
 Lập BBT của hàm số y  f  t  trên D ' . Ta có: f '  t   6t  2 , f '  t   0  t 
3
Bảng biến thiên

t 1
0 1
3
f 't  + 0
f t  1
3

0 1

 Dựa vào BBT ta suy ra:


1
Phương trình (1) có nghiệm x  1;    1  m  . 
3

Thí dụ 10. Tìm m để phương trình sau nghiệm x  1;3 3 


log32 x  log32 x  1  2m  1  0 (1)

Lời giải.
 Tập xác định của phương trình : D  1;3 3 
 Đặt t  log32 x  1 với x  1;3 3  . Tìm tập giá trị của ẩn phụ t khi x  1;3 3 
x  1;3 3   1  x  3
 
3
 1  log32 x  1  4  1  log32 x  1  2  1  t  2  t  1; 2

Tập giá trị của ẩn phụ t khi x  1;3 3  là D '  1; 2


 Với ẩn phụ trên thì bất phương trình (1) trở thành: t 2  t  2  2m (2)
Phương trình (1) có nghiệm x  1;3   phương trình (2) có nghiệm t  1; 2
3

 Xét hàm số y  f  t   t 2  t  2 với t  1; 2 .

NGUYỄN VĂN LỰC  0933.168.309 SP Toán K35 - ĐH Cần Thơ


GTLN – GTNN FB: http://www.facebook.com/VanLuc168

Phương trình (2) có nghiệm t  1; 2  đường thẳng y  2m có điểm chung với
phần đồ thị hàm số y  f  t  vẽ trên 1; 2  .
Lập BBT của hàm số y  f  t  trên D ' . Ta có: f '  t   2t  1  0 , t  1; 2
Bảng biến thiên

t 1 2
f 't  +
f t  4

 Dựa vào BBT ta suy ra:


Phương trình (1) có nghiệm x  1;3 3   0  m  2 .

Thí dụ 11. Tìm m để bất phương trình sau có nghiệm


4 x  x5  m (1)

Lời giải.
 Tập xác định của bất phương trình : D   5; 4
 Xét hàm số y  f  x   4  x  x  5 trên  5; 4 .
Bất phương trình (1) có nghiệm x   5; 4  có điểm thuộc đường thẳng y  m
nằm phía dưới đồ thị hàm số y  f  x  vẽ trên  5; 4 .
1 1 4 x  x5
 Lập BBT của hàm số trên trên D . Ta có: f '  x    
2 4  x 2 x  5 2  4  x  x  5 
1
f ' x  0  4  x  x  5  x  
2

x 1
-5  4
2
t' + 0
t 3 2

3 3

 Dựa vào BBT ta suy ra:


Bất phương trình (1) có nghiệm x   5; 4  m  3 2 .

Thí dụ 12. Tìm m để bất phương trình sau có nghiệm


mx  x  3  m  1 (1)

NGUYỄN VĂN LỰC  0933.168.309 SP Toán K35 - ĐH Cần Thơ


GTLN – GTNN FB: http://www.facebook.com/VanLuc168

Lời giải.
 Tập xác định của phương trình : D  3;  
x  3 1
Khi đó: 1  m  (2)
x 1
x  3 1
 Xét hàm số y  f  x   trên 3;   .
x 1
Bất phương trình (2) có nghiệm x  3;    có điểm thuộc đường thẳng y  m
nằm phía dưới đồ thị hàm số y  f  x  vẽ trên 3;   .
5 x  x 3
 Lập BBT của hàm số trên D . Ta có: f '  x  
2 x  3  x  1
2

f ' x  0  x  3  5  x  x  4
x  3 1
Giới hạn xlim f ( x)  lim 0
 x  x 1

Bảng biến thiên

x 3 4 
f ' x + 0
2
f  x 3

1
0
2

 Dựa vào BBT ta suy ra:


2
Bất phương trình (1) có nghiệm 3;    m  .
3

Thí dụ 13. Tìm m để bất phương trình sau nghiệm đúng với mọi x   2; 4
4  4  x  2  x   x2  2x  m  18 (1)

Lời giải.
 Tập xác định của phương trình : D   2; 4
 Đặt t   x 2  2 x  8 với x   2; 4 . Tìm tập giá trị của ẩn phụ t khi x   2; 4
x 1
Ta có: t' , t '  0  x 1
 x2  2 x  8

Bảng biến thiên

x -2 1 4
t' + 0
NGUYỄN VĂN LỰC  0933.168.309 SP Toán K35 - ĐH Cần Thơ
GTLN – GTNN FB: http://www.facebook.com/VanLuc168

t 3

0 0

Từ bảng biến thiên ta suy ra tập giá trị của t là : D '   0;3
 Với ẩn phụ trên thì bất phương trình (1) trở thành: m  t 2  4t  10 (2)
Bất phương trình (1) nghiệm đúng với mọi x   2; 4  Bất phương trình (2)
nghiệm đúng với mọi t   0;3
 Xét hàm số y  f  t   t 2  4t  10 với t   0;3 .
Bất phương trình (2) nghiệm đúng với mọi t   0;3  đường thẳng y  m nằm
hoàn toàn ở phía trên phần đồ thị hàm số y  f  t  vẽ trên  0;3 .
 Lập BBT của hàm số y  f  t  trên D ' . Ta có: f '  t   2t  4 , f '  t   0  t  2

Bảng biến thiên

t 0 2 3
f 't  0 +
f t  10 7

 Dựa vào BBT ta suy ra:


Bất phương trình (1) nghiệm đúng với mọi x   2; 4  m  10 

Thí dụ 14. Tìm m để bất phương trình sau nghiệm đúng với mọi x 
m.4 x   m  1 2 x  2  m  1  0 (1)

Lời giải.
 Tập xác định của phương trình : D 
 Đặt t  2 x với x  . Tập giá trị của ẩn phụ t khi x  là D '   0;  
 Với ẩn phụ trên thì bất phương trình (1) trở thành:
4t  1
mt 2  4  m  1 t  m  1  m  (2)
t  4t  1
2

Bất phương trình (1) nghiệm đúng với mọi x   2; 4  Bất phương trình (2)
nghiệm đúng với mọi t   0;  
4t  1
 Xét hàm số y  f  t   với t   0;   .
t  4t  1
2

Bất phương trình (2) nghiệm đúng với mọi t   0;    đường thẳng y  m nằm
hoàn toàn ở phía trên phần đồ thị hàm số y  f  t  vẽ trên  0;   .
4t 2  2t
 Lập BBT của hàm số y  f  t  trên D ' . Ta có: f '  t    0 , t   0;   ,
 
2
t 2  4t  1

NGUYỄN VĂN LỰC  0933.168.309 SP Toán K35 - ĐH Cần Thơ


GTLN – GTNN FB: http://www.facebook.com/VanLuc168

Giới hạn: lim f  t   0


t 

Bảng biến thiên

t 0 
f 't 
f t  1

 Dựa vào BBT ta suy ra:


Bất phương trình (1) nghiệm đúng với mọi x   m  1 

Thí dụ 15. Tìm m để hệ phương trình sau có nghiệm


 2 x 3   y  2  x 2  xy  m
 2 (1)
 x  x  y  1  2m

Lời giải.
 
 x2  x  2x  y   m
 Ta có : 1   2
 
 x  x   2 x  y   1  2m
u  x 2  x 1
 Đặt  . Điều kiện của u là u  
v  2 x  y 4
uv  m u   2m  1 u  m  0
  2
2

 Hệ phương trình trở thành:  


u  v  1  2m v  1  2 m  u

1
Hệ phương trình (1) có nghiệm  (2) có nghiệm thỏa mãn u  
4
1 u 2  u
 Với u   , ta có:  2   m  2u  1  u 2  u  m 
4 2u  1
u 2  u
với u   ;   .
1
 Xét hàm số f  u  
2u  1  4 
Phương trình  2  có nghiệm u   ;    đường thẳng y  m có điểm chung
1
 4 
với phần đồ thị hàm số f  u  vẽ trên  ;   .
1
 4 

 Lập BBT của hàm số trên D . Ta có:


2u 2  2u  1 1  3
f ' u    ; f ' u   0  u 
 2u  1
2
2
Bảng biến thiên

NGUYỄN VĂN LỰC  0933.168.309 SP Toán K35 - ĐH Cần Thơ


GTLN – GTNN FB: http://www.facebook.com/VanLuc168

u 1 1  3
 +
4 2
f 'u  + 0
f u  2 3
2

5
 
8

 Dựa vào BBT ta suy ra:


2 3
Hệ phương trình (1) có nghiệm  m  
2

BÀI TẬP RÈN LUYỆN

Bài tập rèn luyện 1


Tìm m để các phương trình sau có nghiệm
6
1) x  3x 2  1  m ĐS: m 
3
2) 4 x 2  1  x  m ĐS: 0  m  1
3
3) 4 x 4  13x  m  x  1  0 ĐS: m  
2
4) x x  x  12  m  5  x  4  x  ĐS: 2 3  
5  2  m  12

Bài tập rèn luyện 2


Tìm m để các phương trình sau có nghiệm
1) x2   m  2 x  4   m  1 x3  4 x ĐS: m  7
1
2) 3 x  1  m x  1  2 4 x 2  1 ĐS: 1  m 
3
3)  

x  x 1  m x 

1
x 1

 16 4 x  x  1   1

ĐS: m  12
37
4) x  9  x   x 2  9 x  m ĐS:  m3
4
6 2 9
5) 3  x  6  x  3  x  6  x   m ĐS: m3
2
6) m  
x  2  2 4 x2  4  x  2  2 4 x2  4 ĐS: m  1

Bài tập rèn luyện 3


1) Tìm m để bất phương trình sau có nghiệm
 x  2  m x 1  m  4 ĐS: m  2
2) Tìm m để bất phương trình sau nghiệm đúng với mọi x   4;6
 x  4 6  x   x2  2 x  m ĐS: m  6

NGUYỄN VĂN LỰC  0933.168.309 SP Toán K35 - ĐH Cần Thơ


GTLN – GTNN FB: http://www.facebook.com/VanLuc168

3) Tìm m để bất phương trình sau có nghiệm


5
m x2  1  x  2  m ĐS: m 
4
4) Tìm m để bất phương trình sau có nghiệm x  0;1  3 
m  
x2  2 x  2  1  x  2  x   0 ĐS: m 
2
3
5) Tìm m để bất phương trình sau nghiệm đúng với mọi x 
3
9 x  2  m  1 3x  2m  3  0 ĐS: m  
2

NGUYỄN VĂN LỰC  0933.168.309 SP Toán K35 - ĐH Cần Thơ


GTLN – GTNN FB: http://www.facebook.com/VanLuc168

ỨNG DỤNG GTLN VÀ GTNN CỦA HÀM SỐ TRONG PT VÀ BPT

Giả sử f  x  là hàm số liên tục trên miền D và đạt GTLN, GTNN trên miền ấy. Ký
M  Max f  x 
hiệu: m  min
xD
f x
 xD

Khi đó ta có các kết luận sau:

1) Phương trình f  x   a có nghiệm x  D  m  a  M

Ví dụ 1: Tìm a để phương trình sau có nghiệm 2 x  4x  a


Ví dụ 2: Tìm m để phương trình sau có nghiệm x  m  4  x2  0
Ví dụ 3: Tìm m để phương trình sau có nghiệm  x  3 x 1  4x  x 2  2m  1  0
Ví dụ 4: Tìm m để phương trình sau có nghiệm x   3;0

x  2x    m  1  x 2  2x   m  1  0
2 2

2) Bất phương trình f  x   a có nghiệm x  D  a  M


Bất phương trình f  x   a có nghiệm x  D  a  m

Ví dụ : Tìm a để bất phương trình sau có nghiệm x  1  4  x  a

3) Bất phương trình f  x   a nghiệm đúng với mọi x  D  a  m


Bất phương trình f  x   a nghiệm đúng với mọi x  D  a  M

Ví dụ : Tìm m để bất phương trình sau nghiệm đúng với mọi x   2; 2
x  m  4  x2  0
B. Bài tập

Bài 1: Cho phương trình 2  x  2  x   2  x  2  x   m (1)


Tìm m để phương trình (1) có nghiệm

Bài 2: Cho phương trình 2  2  x  6  x    2  x  6  x   3m  1  0 (1)


Tìm m để phương trình (1) có nghiệm

Bài 3: Cho phương trình  x 2  1  2x 2  x 2  3m  2  0 (1)


2

Tìm m để phương trình (1) có nghiệm

Bài 4: Cho phương trình x  2  x 2  x 2  x 2  5m 1  0 (1)


Tìm m để phương trình (1) có nghiệm

 
Bài 5: Cho phương trình m 1  x 2  1  x 2  2  2 1  x 4  1  x 2  1  x 2 (1)
NGUYỄN VĂN LỰC  0933.168.309 SP Toán K35 - ĐH Cần Thơ
GTLN – GTNN FB: http://www.facebook.com/VanLuc168

Tìm m để phương trình (1) có nghiệm

Bài 6: Cho phương trình 2  sin 4 x  cos 4 x   cos 4x  2s in2x  m  0 (1)



Tìm m để phương trình (1) có nghiệm x  0; 
 2

Bài 7: Cho bất phương trình  x  4 6  x   x 2  2x  m (1)


Tìm m để bất phương trình (1) nghiệm nghiệm đúng với mọi 4  x  6

NGUYỄN VĂN LỰC  0933.168.309 SP Toán K35 - ĐH Cần Thơ

You might also like